Sie sind auf Seite 1von 522

INTERNAL

COMBUSTION ENGINES
(IncludingAir Compressors and Gas T\rbines
and Jct Propulsion)

By

R.K. RAIPUT
M.E. (Heat Power Engg.).lons.-Gold Medallist ;Gtad' (Mech' Eryg' & Elect' Erqg')
M.l.E. (Indb) ; M'S.E.S.I. ; M.I.S.T.E. ; C.E. (Ib)
Princlpal (Formcrlfl,
Puaiob College of Infonnotion Teehtplql
PATIAIA (Puajob)

rN(Ml PUBLICATIONS (P) tTD


BANGALORE. CHENNAI o GOCHIN o GUWAHATI o HYDERABAD
JALANDHAR O KOLKATA o LUCKNOW o MUMBAI o RANCHI
NEW DELHI

..
PREFACE TO THE SECOND EDITION
I am pleased to presen the Second edition ofthis book. The warm reception,
which the
previous edition ofttre book has enjoyed all over India, has been
-att"r orgt"t ;atisfaction to
me. "
The book has been thoroug_ hly rwised, besides adding a new chapter (No.
22) on..short
l* Answer Quections'to enable the itudents to prepare more effectively forpro ctical Viua-ue
E xamhtatia ns and I nter v iew s.
It
Any suggestions for improvement of this bbok will be thankfully acknowledged
and

t
incorporated in the next edion.
r.qlo{r-ii',#f,8rrtb er rro
1lB, Golden House, Daryasani.
New Delhi_il000i -
phonc :011_4A { -Author
bg 25 00
.Far : 011-4t 5g 2E 28
www.Ian ipublicatioe.com
. info@lnipublicaions.com

Compiled y; SnL RAMESE nA.rpUT

@ Nl rights rewrud, utth tlu pubtishrs.


No part of this publilxrtfun
m.ay be rcprodued,, sbed in a.retrizu!
fotm or by aryr means, "lrteii, t*i*^rr.d
io oo,
"t""t*".,
or otherwise without the
;;h";;;;*;;;,*, rca.it*
pnor urrtten penni,ssion of the publishen

Price:fu. S98.fi) Ozly.


s eayt Editia n, r*r, liiJ^Ylrtt,,Zffi
OFI.TCES
@ Bangalore 080-26 61 15 61 @ Chenal 044-24347 26
@ Cochi 0484-239 70 04 @ Guwahati 0361-254 36 69,251 38 81
@ Hyderabad 040-24 65 23 3A @ ctalandhar otSL-222 t2 72
@ Kolkata 033-22 27 43 84 @ Lucknow 0522-220 95 78
@ Mumbat 022-24 9t 54 75,24 92 78 69 @ Ranchi 0657-227 47 U
EIC.O65O.395.INTERNAI COMBUSTION
ENGINES
Ilpeeet at : Goswami Associatee, Delhi. c_t74ilosto4
Printed at: ljitfuintars, Delhi.

{ i..
PREFACE TO THE FIRST EDITION
CONTENTS
on olnternal Combustion Engineco (Induditg
gos turbnes) contains
This treahise
lucid and direct language' It eirvelops
.$
comprehensive treatment of-e suject matter in a_simple, T,
Pages
typical worked examples from
a large number of solved p*ff"-J n""nerly graited including Chapter
I
Ir
examination point of view.
I
I. BASIC CONCEPTS OF TEERMODYNAMICS 1-32
Thebookcomprises2lchapters.Allchaptersaesaturatedwithmuchneededtext,
y ,i.pf"."rr *fi"*pi"".tow-figurs-.At the end of eactr chapter-Highlights'
Eximples have been i; 1.1. Deffnition of ThermodYuamics 1
"upport"J
r:.ti"" ivp.'qo."u"i", it .i.ti {uestiors and unsolved
Objective Tlrye Questions 1.2. Thermoclynamic SYstems 2

(with Answer"
"queJion
added ; besides tfris a Bank' containing "Adtional
Sofii""-Comments)",'Theoretical Questions withAnswers" and -t 1.2.1. System, bobndary and surroundings
1.2.2. Closed sYstem
2
2
"oa
..Addiional Typical p*"ipl"" (Includ,ing lniuersities and Competitiue Exomination 1.2.3. OPen sYstem 2
rnake the bok a comprehensive and a complete unit in all
L""o io"to"Tto a
A;;J;;;; t
"r,"
L.2.4. Isolated sYetem
o
respects. L.2.5. Adiabatic sYstem
The book will prove to be a boon to the students
preparing for engineering undergradu- L.2.6. Homogeneous sYstem

e..i.O., post graduate, U'P'S'C' and other competitive examinations' L.2.7. Hetemgeneous sYstem
ut", 3
Theaut,hor,sthanksaredueohiswifeRameshRajputforextendingallcotiperation 1.3. Pure Subgtance
3
during preparation ofthe manuscript and proofreading' 1.4. Thermodynanic Equilibrium
3
his gratude o Shri R.K. Gupta, Chairman, 1.5. Properties of SYstems
In the end t;.e author wishes o expresspuucauons 1.6. 4
sh. saurabh Gupta, Managi"g Di;.**, l,axmi
hrt. Ltd., New Delhi for taking a State
4
out"th; book with very good presentation in a short span oftime' 1.7. Process
lot of pains in bringrng 1.8. Cycle 4
Althougheverycarehasbeentakentomakehebookfreeoferrorsbothintextasrvellas 1.9. Point Function D
enors present are brought to ltis
ir solved examples, v.t trr" u"irro, shall feel obliged if any 1.10. Path Function 5
,roti.". Corr"trrr.tive criticism of the book will be warmly received'
1.11. TemPerature D
6
!.12. 7'erclh Law of Thermodynamics
b
1.13, PresEure
b
1.13.1. Definition of Pressure
-Author 1.13.2. Unit of Pressure
7
1.13.3. \rpes of pressure meaaurement devices
8
1.14. Rversible and Irreversible Processs
1.15. EnergY, Work and Heat I
1.15.1. EnergY I
1.15.2. Work and heat 9
10
1.16. First Law of Thermodnamics
11
1.17. The Perfect Gas
1.17.1. The characterietic equation of state 1l
1.17.2. SPecific heats t2
1.17.3. Joule'e law 13
1.17,4. Rlationship between two specific heats 13
1.17.5. EnthalPY i t4
1.1?.6. Ratio of sPecific heats 15
(vni)
(ir)
Chapter
Pages Chapter
Pages
1.18. Steady Flow Eaergr Equation (S.F.E.E.)
1.lg.l.
Enerry relatios for flow process ... 16 3.5. Constant Pressure or Dieeel Cycle
1.19. Limitations of First f., of fn"._-Jyotio ^. L7 3.6. Dual Combustion Cycle 110

1.20. performance of Heat Eagine and ... 3.7. Cornparison of Otto, Diesel and Dual Combustion t20
n"r1,""" lleat 18 Grcles
1.21. Statement of.Seconil f-* of fn"._Jilaurics Engine ... 18 A.1.L. Efficiency versus compression ratio 136
1.21.1. Clausius statement ... 19 3.7.2. For the sane coopression ratio anil e same heat input 136
13?
1.21.2. Kelvin-planck statement 19 3.7,5, For congtant maximum pressue anil heat supplied
L.22. Entropy ... 20 S.8. Atkinson Cycle L37

1.22.1. Introduction ... 20 3,9. Ericsson Cycl 138


L.22.2. Tempetature.euhopy diagram ... 20 3.10. Brayton Cycle 741
1.22.8. Characteristics of entrop| ... 3.11. Stirling Cycle t42
20
1.28. The Third Law ofThermodynariics 3.12. 153
2l Mille" Cycle ) :..
1.24. Available and Unavailabt" irr""gy- ... 2L 3.13. Lenoir Cycle 155

Hightights ... 22 Highlights i


156
Objectiue fupe euestions ... 23 Obj ectiu e Type Q ues tions
ID
Thoretical euestions ... 25 Theoreticol Questinns 158
... 32 Unsolued Enmples
159
2. IMRODUCTION I1O TNIEnNAL CoMBUsfioN ENGINES
160

2.I. Heat Engines


33_&t /4. FT,'EI-AIR AND ACTUAL CYCI,ES
162_200
?.2. Developmeat of I.C. Engines ... 33 4.L. Fuel-air Cycles
...
2.3. Claseification of I.C. Engines ... 95 4.I.7. Introduction
...
762
2.4. Appcation of I.C. Eagil-es ... 35 4.1.2. Factorg considered for fuel-air cycle calculations
...
762
162
?.5. Engine Cycte-Eou"gy B"l*"" 38 4.1.3. Aesumptions oade for fuel-air cycle analysis
... 163
2.6. Basic ldea of t.C. Engineg ... 39 4.7.4. Importance of fuel_air cycle
...
2.7. Different parts of I.C. Engines ... 40 4.1.5. Variable specific heats
...
163

Z.B. Terms Connected with I.C: Oi6ne" ... 41 4.1.6. Effect ofvariation of epesific heats
...
163
170
2.9. Working Cycles ... 66 4.1.7. Dissociation
... L7t
2.10. Indicator Diagra- 68 4.1.8. Thermal efficiencr and fiel consumption
... 172
2-.Il Four Stroke Cycle Engines 69 4.1.9. Efect of @rmon engine variables
173
?
12 TVo Srroke Cycle Engines ... 69 4.1.10. Charactristics of costant volume fuel-air cvcle
... 178
2.I3. Intake for Compression Igaition Engines ... 73 4.1.11. Combustion charts
... 178
2.L4. Comparison of Four Strokl ."a f*iit-t" ... 77 4.1.12. Gas tables
... 178
2.15. Comparison of Spark lgntion Cycle Engines 4.2. Actual Cycles
fs.i.l anJoirop."".ioo Ignition (C.I.) 78
4.2.L. Introduction ... 181

2.16. Comparison between a petml Engine ... 4.2.2. Causes of deriation of actual cycles hom fuel-air cycles ... 181
79 ...
and a Diesel Engine 181
2.L7. Hott to Tell a T$o Shoke Cy"l" ;;;'-f; ... 81 4.2.3. Real fuel-air engine cycles ... 181
Cycle Engine !
a Four Stroke 4.2,4. Difference between real cycle and fuel-air cycle ...
Highl,ighta ... 81 4.2.5. Comparison of operations and working media for ,air cycle,, 182
,actual
O bje ct iv e Ilpe
e ue stians '.. 82 _-. - -. _fuel-air cycle' and cycle' of S.I.-engrnes ..
Theoretical euestions ... 83 Highli.ghts
...
ro

3. AIR STANDARD CYCI,ES


... u Objectiue Type euestians
Theoretical Questions
...
r97
197
... 198
3.1. Denition of a Cycle 8L161 Unsolved Exarnples
... 199
3.2. Air Standard Efficiencv ... 85 l 6. COMBUSTION IN S.T. ENGINES
3.3. The Carnot Cycle 86 20t-226
3.4. Constant Volume or Otto Cycle ... 5.1. Introduction
...
86
5.1.1. Definition of combustion
... 201
94
5.7.2. Ignition limits
... 201
... 201

{,.. i
()
(i )
Chapter
Pages Pages
Chopter
5.2. Combustion phenomenon
5.2.L. Normal combustion 202 AIR CAPACIT OF IIOT'R SIROI{B ENGINES 24&-268
5,2.2. Abnormal conbustion 202
3 Effect of Engine Variables on Ignition Lag 204 7.t. Introduction 248
I5.4. m4 7.2. Ideal Air Capacity 249
Spark Advance and Factors Afiecting lgni-tion Timing
5.5. Pre-ignition 205 7.3. Volumetric EfEciengy 249
5.6. Detonation m3 7,4. Effect of Various Factors o Volu.metric EfEciency 250
5.6.1. Introduction 208 7.5. Inlet Valve Mach Index 253
Highlights 264
5.6.2. Process of detonation or knockiug 208
Objectiue Type Questions 265
5.6.3. Theories of detonation 208
2r0 Theoretial Questions 265
5.6.4. Efects of detonation
6,6.5. Factors affecting detonation/kocks 210 unsolued Emmples ) 266
J.I. Performance Nuober (pN) 2r0
D.. Higheat Usefi:l Compression Ratio (HUCR) zLl 8. TWO SIROIiE:EF{GINES 297-28r
5.9. ztl 8.1.
99Tbuslion Ch-ber Desigrr-'S.I. Engines 2Ll
General Aspects 267
5.9.1. Induction swirl 8.1.1. Construstion and working 267
5.9.2. Squish and tumble 212
213
8.f.2. Comparison between two-stmke cycle and four-stroke
5.9.3. Quench area cycle engine 269
5.9,4. Turbulence 213 8.1.3. Disadvantagee oftwo-stroke S.I. engine comtared to
5.9.5. Flarne propagation 214 twoshlke C.I. engine 269
5.9.6. Swirl ratio 215 8.1.4. Rason8 for use oftwo-eroke C.L engines for marine
5.9.7. Surface-to-volume ratio 275 propulsion 270
6.9.8. Stroke-to-bore ratio 215 8.1.5. Reasons for the use of two-stroke S.I. engines for low horse
5.9.9. Compression ratio (C.R.) 216 power two wheelers 270
5.10. Some Tlpes of Combustion 276 8.2. Intake for Two stroke Cycle Engines 270
Chambee
5.10.1. Divided combustion chambers 218 8.3. Scavenging hoces 271
Highlights 220 8.4. Scavenging Parmeteru 274
Objective Type euestians 223 6.D. Scavenging Systems 275
Theoretical euestbns 224 8.6. Crankcase Scavenging 277
225 8.7. Scavenging Pumps and Blowers 278
6. COMBUSTION IN C.I. ENGINES HishliAhts 280
226-247 Objective Type Qrestbns 280
6.1. Introduction
Theoretical Questiorc 281
6.2. Combustion phenomenon in C.I. Engines 226
6.3. Fundamentale of the Conbustion pr-ocegs
in Diesel Engines
227
9' CI{N}trCAL TIIERMODYNAMICS AND FTJELS
Delay period (or Ignition Lag) in C.I. Engines 230
9^_
6.5. Diesel Knock 234
(CONIENTIONAL AND ALTERNATIVE) 282-356
6.6. C.I. Engine Combustion Chambers 236 9.1. Chemical Thermodynmics 282
6.6.f . Pimay considemtions in the desiga 237 9.1.1. General aspects 282
of combustion
chambers for C.I. engines 9.1.2. Baic chemitry 282
6.6.2. Basic methods of generating air swirl in C.I. engines 237 9.1.3. Fuels 283
combu.stion chambers 9.1.4. Combuetion equations 284
6.6.3. Types of combustion chambers 238 9.1.5. Theoretical air and excess air 286
6.7. Cold Starting of C.I. Engines 210 9.1.6. Stoichiometric air-fuel (A/F) ratio 286
Highlights 241 l 9.1.7. Air-fuel ratio from analysis of products 287
Objective Type euestons 245 9.1.8. Analysis of exhaust and flue gas 287
Theoretical euestons 246 9.1.9. Internal energy and enthalpy of reaction 289
246 9.1.10. Enthalpy of formation (AlI.) 293
9.1.11. Heating values of fuels 294

{. r.i
I (xii)
Chapter ( ii)
Pqg" Chapter
9.1.12. Adiabatic flame tenpeatur
Pog""
9.1.13. Chemical equilibriui 296 11.3. Factors Influencing Cabrretion
9.1.14. Actual combuetion 2W
9.2. _""i" 11.4. Mixture Requirenents 368
Conventional Iuels (For lClffi""-- 298 11.5. Distribution 369
9.2.L Introduction 238 11.6. Transient Mixtue Requirenents 370
9.2.2. Desirable-propertiee of good I.C.cngioes 29E f1.7. A Sinple or Elementary Caburettor 37L
9.2.9. Gaseous fuels fuels
298
9.2.4. Liquid fuels 11.8. Complete Carburettor 372
2gfl 11.9. Carburettors 373
9.2.8. Structure ofpetrolen 300
9.2.6, petrolelo aod coinpoeition of 11.9.1. Eseential featues o,fgood commercial 381
crude oil 300 carburettor for
9.2.7. Fuels for automotive engines
303 11.9.2. $pes of carburettors 381
9.2.9. Knor "p""f_ig"iUoo ""gi""l-
s2 s,,Jil:ffi"jj*;H;",T:*ne
9.2.10. tue,s
305
310 -_ _
11.10.
11.9.3. Description of some important maLes
^ Petrol Injection
of carburettors
382
383
Dieeet fuel 314
9.A. 11.10.1. Drawbacks of modern carbuettors 391
Alternative Fuels for I.C, Engines 314
11.10.2. Introduction to fuel iqiection 391
9.4.1. General aspecto 316 11. 10.3. Direct injection 391
and dieadvanrases of using 316
l.i:i. ifl:i:i.*"" alrerative fuele
3r7
11. 10.4.
I l. 10.S.
Indirect injection
Injection considerations
391
391
9.9.4. Alcohol-gasoline fuel bleds 318 11'10'6' comparison ofpetror idection 393
9.8.5. Hydrogen 323 and carburetted fuel
9.8.6. Natural gas (Eethae) supply systeme
325 11. 10.2.
Electrouic fuel injecti,on 393
9.9.2. LpG and LNG 325 11.11. Theory of Simple Carburetor 394
9.8.8. Biogas 327 Highlights 395
Hisht@hb 327 Objectue Type etnstiow 410
Objective Type euestions 350 Theoretical euestions 411
Theoretical euestions 351 Unsolued, Eramples 412
Unsolued Examples 354 413
ro. F.UEr./ArR MrxTuRE REQUIREMEIYTS
355 12. FUEL INJECIION SYSTEMS FOR C.I. ENGINES
416-440
10.1. Introduction 357-366 72.r. Introduction
L2.2. Functional Requiremeats of an 415
10.2. FueUAir Mixture $ection System
l0'S Optimum FueUAir RaUo" -
' --'
Bequirements for Steady
vwqqr IRunning 357 12.3. Funtions of a Fuel Injection Srstem 415
357 12.4. Fuel Injection Systens tro
10.4. Idling and Low Load 360
10.5. Normal power Range or 12.4.1. Air injection ato
Cruise Ranse 361 12.4.2. Solid or ailess injection
10.6. Maximum power RLge -- -'v '.*Es 362
416
L2.5. Pumn and Fuel Injector (Atomiser) 417
10.7. Transient Mixture Requirements 362 l^ue_l
12.5.1. Fuel pump 420
10.?.1. Starting and warming 35:t
up hitu.e requirements 12.5.2. Fuel atomiser or injector 420
10.2.2. Mixture
10.8. Effects of operarine
"eqrrire-ent fol ;;;n"" 363 12.5.3. Faults, causes and remediee
of injectors 423
variables ;; j;;;"-t"quiremenk
.. 363 It a T}pes of Nozzles d Fuel Spray pattems 425
10.9. Mixture Requirements f""
Di"""l-;;;;;;
--- -'6ee 364 12.6.-1. Main reqrris6sts of an 426
Highlights 364 injector nozzle
12.6-2. Classification and description 426
Objectiue Type euestions 366 10 a of nozzles
Engine Starting Systems 426
Theoretical euestinns 366 t2.8. Fuel,Injection Computation in C.I. 429
366 Engines
rT. CARBT.IRETION AID CANBURTTORS Highlights 430
O bje c t iu e Type
eue stians 438
11,1. Introduction 367_414 Theoretical Questions 439
11.2. Induction System 367 Unsolued Etamples 439
439
367

.l i-.. i
. (p)
Chapter
Pqgss Chapter
Pages
13. IGMTTON SYSTEMS (S.r. ENGINES) 441-460 15.5. Effects of Operang Vriables on Engiue Heat Tlansfer
490
13.1. Introduction ... tul 15.6. Cooling Air and Water Requirements
49r
13.2. Requirements of an Ignition System ... ,l,ll 15.7. Cooling Systeqs 493
13.3. Basic Igaition Systems ... 412
15.7.1. Ai-cooling system
493
13.4. Battery (or Coil) Ignition System ... 112
15.?.2. Wateriquid cooling system
496
13.5. Magneto Ignition System ... 4{9
15.8. Components of Iater Cooling System
503
13.6. Firing Order ... 452 Hishlights
508
13.7. Ignition Timing ... 452
Objectiue Type Questions
508
13.8. Spark Plugs ... 4153
Theoretical Questions
509
13.9. Limitations of Cbnvetional Ignition ... 455
13.10. Electronic Ignition Systems 16. SUPERCEARGING OF I,C. ENGINES )
... 456 511-536
Hightights ...
' 468 16.1. Purpose of Superchaiging
Objectiue Type Questions ... 511
459 16.2. Supercharging of S.I. Engines
Theoretical euestbns ... ,t60 513
16.2.1. Natually aspirated cycle of operation
513
14. ENGINE 16.2.2. Supercharged cycle of operation
FB,ICTTON ND LI]BRICATION 461--{81 16.2.3. Comparison of actual natwally aspirated and supercharged
513
14.1. Introduction
14.2. Total Engine Friction
... re r
16.2.4.
engine pressure_volume diagrams
Boost pressure and pressure ratio
5L4

14.3. Effect of Engine Parameters on Engine Friction


... 461
16.2.5. The effect of presaure ratio on air charge rmperature
514
... 46:| 515
14.4. Determination of Engine Friction ... 4&
16.2.6. Thermodynanic cycle and superchargiag power
516
14.5. Lubrication 16.2.7. Supercharging limits of S.I. enginee
... 1U 16.3. Supercharging of C.I. Engines
518
14.5.1.Definition and objects ... ,164 al
14.5.2.Behaviour of a journal in its bearing 16.3.1. Supercharging limits of C.I. engines
... 465 16.4. Modification of an Engine for Supercharging
, 519
14.5.3.Properties of lubricantg ... 466 520
14.5.4.Types of lubricants r6.5. Supercbargers
... 469 520
14.6. Lubcation Systems ... 177
16.6. Supercharging Arrangements
520
14.6.1. Introduction 16.7. Turbochargers
... 171
16.7.1. Introduction
52r
14.6.2. Wet sump lubrication srstem ... 472 521
14.6.3. Dry sump lubrication systen 16.7.2. Altitude compensation Kta
174
14.6.4. Mist lubrication system 16.?.3. Turbocharging-Buchi system
... 175
16.7.4. Methods of turbcharging
524
14.6.5. Lubrication of different engine parts ... 475
14.6.6. Lubrication of ball and roller bearings 16.7.5. Limitations of turbocharging
... 477 526
. L4.6.7. Oil filters ... 477
Highlights <ae
14.7. Crankcase Ventilation ... 478
Obj ec tiu e Typ e Quc stions
534
Hishlights Theoretical Questinns
... 478
Unsolued Etamples
<ca
Objectiue Type Questions ... 535
478
?heoretical Questians ... 480 L7. TESTING AND PER'ORIITANCE OF I.C. ENGINES
537-611
15. ENGINE COOLING 482-510 17.1. Introducion
537
15.1. Necessity of Engine Cooling ... 482
77.2. Performance Parameters
537
1a e
15.2. Areas of Heat Flow in Engines ... Bagic Measurements
540
484
15.3. Gas Temperature Variation ... 484
L7.4. Engine Performance Curoes
549
15.4. Heat Transfer,'Temperature Distribution and Temperature profiles ... 485
L7.5. Comparison of Petrol and Diesel Engines_Fuel ConsumDtron
15.4.1. Heat transfer Inad Outputs and Exhauet Composilion
... 485 r/.b- Governing of I.C. Engine
551
15.4.2. Temperature distribution ... 553
487 Noise Abatement
15.4.3. Temperature profiles ,.. 488 554
Highlights
605
(s') (nt )
Chapter
Pog* Chapter Pages
Objective Type euestins
606 19.3.3. Difrerences between camot and stirling engines 646
Theoretical euestions
608 19.3.4. Engine geomstry and driving mechanism 646
Unsolved, Emmples
608 19.4. The Wankel Rotary Combution (RC) Engine 650
18. AIR, POLLUIION TB'O LC. ENGINES 19.4.1. Introduction 650
AYD ITS CONTR,OL 612-437 19.4.2. Construction and working 650
18.1. Introduction 19.4.3. Features 651
18.2. Pollutants 612 19.4.4. Constructional and other details of Wankel engine 651
18.2.1. pollution deriyed from combustion pmducts 6t2 19.4.5. Perfomance of Wankel engine 654
18.2.2. Mixtue and oombustion product characteristics
612 19.4.6. Advantages and applications of rotary combustion engincs boa
-rlegth
18.3. Spark Ignition (S.I.) Bngine Emissiom
614 19.4.7. Why Wankel rotary engine could not become successful ? baJ
l8.B.l. Crantcaee euision 615 19.5. Variable Compression Ratio (VCR) Engines 656
18.8.2. Evaporative eoission 6r6 19.5.1. Introduction bcb
18.3.3. Exhaus! siion 617 f9.5.2. Methods to obtain variable compression ratio 656
18.4. S.I. Engine Emission C,onhol 617 19.5.3. Analysis of VCR engine 658
18.4.1. Modificatioa in the engine deeign and 619 19.5.4. Performance of VCR engine 658
operating parameers 619 19.6. Free-Piston Engine Plant
18,4.2. Exhaust gas cidation 659
18.4.3. Exhawt ghicion control by fuel variation 620 Highlights 661
18.4.4. BIow-.by conkrd 623 Objective Type Questions oor
18,4.5. Evaporation iaion control device (EECD) 621 Theoretical Questians 662
18.4.6. .Control of oddcs of nit,ogen (NO ) 621
18.4.?. Total siaim mtol paclageg . 621 20. AIR COMPRESSORS 664--803
18.5. Diesel Engine Enieciru 626
20.1. General Aspec'ts 664
18.6. Diesel Smoke and Conkol 627 .

18,6.1. Exhaust smole 629 20.2. Classification of Air Compressors bba


629 20.3. Reciprocating Compressors 666
18.6.2, Causes ofsnoe
629 20.3.1, Construstion and working of a reciprocating compressor
18.6.3. Measuremmt of smke . (eingle-stage) 666
18.6.4. Control of smo[e 6:t0 ' 20.3.2. Single-stage oompressor : equation for work
18.6.5. Diesel odow ad contol 631
(neglecting clearance) 667
18.7. Comparison of Gasoline and Diesel Emissions G|l
20.3.3. Equation for work (with clearance volume) 670
18.8. Zero Emission 632
20.3.4. Volumetric efciency 67r
18.9. Air Pollution from Gs Turbines and its Control 632 / 20.3.5. Actual pV (indicatr) diagram for single-stage compressor 673
f8.10. Effects of Engine Emissions oo ffo-uo U""ltl 633
20.3.6. Multi-stage compression 674
Highlights 634
20.3.7. Effisiency of, compressor 683
@jectiue Ilpe euestims 635 I 20.3.8. How to increase isothermal ef,Eciency 'l 684
Thoreticol euestions 635
20.3.9. Clearance in conpressors 684
636
20.3.10. Etrect of cleaance volume 685
19. MTSCELII{N]EOUS EIVGINES 20.3.1l. Free air delivered (F.A.D.) and displacement 686
638-663
19.1. Duel-fuel and Multi_fid Engines
20,a. f
l Compressor perfomance 647
19.1. 1. . Duel-fuel engines 638 . 20,3.f. Etrect of atmospheric conditions on the output of a compressoi oot
19. 1.2. Multi-fuel engines 638 20.y,f4. Conttol of compressors 687
19.2. Sratified Charge Engiu 642 2O/3.Lf. Anangeq.rgnt of 'reciprocating conpressors 687
19.2.1. Intrcduction 643 20.3.f6. Intercoolef 688
'
19.2.2. ClassificatioD 643 20.3.17. Compreesed-hir motors 689
19.2.8, Advantages ad disadvantages of stratified 643 20.3.18. Reciprocating air notor 689
19.3. Stirling charge engines 645 20.3.19. Rotary trpe a nctor 690
Engine
19.3.1. Stirling cycle 645 20. . Rptary Compressors
19.3.2. Working princide of stirling engine 645 20.4. 1. - Classification 747
645
Chapter
Pages
20.4.2. Displacement compreesors
20.4.3. Steady-flow compressorg 742
20.5. Comparison betreen Reciprocating and Centrifugal
Compresors
20.6.
20j.
Comparison betreen Rcciprocahn! and notr"y
C_omparisou between C""t"in ga-a"a eri"i
Hishlishts
. C-o"_i1""""r"
f"i^C""loi""**
786
787
787
1
Objective Tlpe (fustions 795
Theoretical Quaions 796 Bosia Coneepts of Thermadynannics
Unsolved Era.mfu"s 798
800
2t. GAS TURBINES AND iIET PROPUISION 1.1. Denition of thermodynamics. 1.2. Tternodynamic aystems-System, boundary
804-881 and surroundings+Closed system--Open oystem-Isolated eystem-Adiabatic system-
21.1. Gas ?urbines-Cieaeral Aspects Homogeneous systen-Hetarogenoous Bystem" 1.3. Pure substane. l.4.lheruiodylamic
21.2. Classification of Ga Tubines ... 804 equilibrium. 1.5. hopertieo of syBtems. 1.6. Stt. 1.7. Process. 1.8. Cyde. 1.9. Point
21.3. Merits of Gas Ttnbiaes ... 804 function. 1.10. Path fungtion. 1.11. Temperature. 1.12. Zeroth law ofthermodynanics.
21.4. Const&rt Pressuro Combustion Turbines ... 805 1.13. hessure-Definition ofpressure-Unit forpressure-$pes ofpressure meaerement
21.4.1. Open cfde Sas turbinee ... 806 devices. 1. 14. Rversible aad irreversible procesrs. 1. 15. Energr, work and heat -Enerry-
21.4.2. Methods for impmvement of thermal efficiency ... 806 Work and heat. 1.16. Firgtlaw ofthermodynamies. 1.17. lbe perfect gae-Ibe characteristic
of open cycle equation of state-Specific heats-Ioule'e lsw-Rlationship between two specifi c heat*
gas urtine plalt
of variables
... 808 Enthalpy-Ratio of specifrc heats. 1.18. Steady flow energy equation (S.F.E.E.)-Energy
:1 1q lgect ryerating on thermal efficiency ... elations for flow procees. 1.19. Limitations of first law of thermodynamics.
?1 1 1 9]*4 sde
gas rurbine (con-sran;";;';;je 812
21.4.5. Merits d demerfts of closed cycle) ... 815 1.20. Perfornance ofheatengine and reverged heat engie. 1.21. Statemente ofsecond law
cycle gre turbine "y"i"';"_;;b;J-ou.. op"r, ofthermodynamics-Clausius statement Kelvin-Planck statement. 1.22. Entropy-
2t.5. Constqnt Vsluhe eo[ustioa Tubines ... 820 Introduction-Temperatre etropydiagram-Characterietics ofenhopy. 1.23. The third
2t.6. Uses ofGas I\rbines ... 820 law of thermodynamics. 1.'1. Available antl unavailable energy-Highghte-Objective
2L.7. Gas Turbine Fuels ... 821 Type Questions-Theoretical Questions.
27.8. Jet Propulsion ... 821
21.8.1. Turbo-jet ... bt
... 856 1.1. DEFINMON OF ISERIUODTNAIfiCS
21.8.2. Turbo-prop
21.8.3. Ram-jet ... 872 Thermodynamics may be defined ae follows :
21.8.4. Pulse-jet eagine ... 873
Therrrndynatnics s o axiomotic science which d.eals with the relatons arnong hcat, work
21.8.5. Rocket engines ... 874
... 875 and. properties of system whih are in equilibrium. It d.escribes stote ond. changes in state of
Highlights
Objectiue Ilpe Qustions
... 877 phyecal systems.
T he o ret ical Q ue.atione ... 877 Or
Unsolued Eramplcs ... 879
... 879 Thermodynamics is [e science of the r4ularities governing processes of energy conver-
22. SHORTANSWER QUESTIONS sion.
882-904
QUESTIONS BAK-With Answers Or
(Including Universities and Competitive l-98
Examinations, euestions) Thermodynarnics is l scince tnat deats with the interacton between energy and mate-
PART.T. ADDITIONAL O&rEcTfvE TrpE rial systems,
QrrEsTroNs
A. Choose the Correct Answer Thermodynamics, basically entails four laws or axiomg known as Zeroth, First, Second and
B. Match List I nith List II Third law of thermodynanics.
C. Competitive Exalrinations euesions (With
Soluions_Comments) the Firs I throws ght on concept of internal energy.
D. Fill in the Blanks -
tbe Zeroth Joar deals with thrmol equilibrium and establishes a concept of temperd,ture.
PART-rr TEEORETTCAL QUESTToNS TVTTIT ANSWERS -
50 tbe Second I@ indicaes the limit of converting heot into work and introduces the
PART.III ADDITIONAL TTPICAL WORreD EXAMPT.ES - principle of increase of entropX.
APPENDD(
INDEX
t-22 - thid law defines the absolute zero of entropy,
These laws are based on experimental observations and have o mathematical proof. Like
1---3 all physical laws, these laws ae based on logcoJ reasoning.
I
fNTERNAL COMBUSTTON ENCINES BASIC CONCEPTS OF THERMODYNAMICS 1

r.2. THERMODYNAMIC SYSTBMS 1.2.4. Isolated Systen


1.2.L. System, Boundarr and Sunoundings An isolated system is that system which erchanges neither energ r.or matter with any
system. A system is a finite quantity of matter or a prescribed. regbn other s)tstem or with enuironment,
o/space (Refer Fig. 1.1)
Boundary. T\e actual or hypothetical enuerope enclosing the system 13.6. Adiabatic Syetem
the system The boundary ma{ be qrcd or it may ,nou",
is the boundary of
and when a system containing a gas is An adiabatic systen is one which is thermcilly nsulated, from its surround,ings.
compressed or expanded. rhe boundary maybe rear or imaginary.". It can,
real boundary but an exampre
It is not difticult to envisage a however, erchonge worh wh its sutoundngs. Ifit does not, it becomes an isolated system.
llinacinag bo'ntrary *oold b" ne drawn . .o.r-
sisting of i'i'o trosh mixture about to enter L cynder f a I.c. ""ooo trr""yst"-
engine togethei*tr,
Phase. A phase is a quantity of matter which is homogeneous throughout in chemical
of the last cylinder charge after the exhaust p"o"""" ""-^.ruot" composition and physical structure.
G"f". fig. f]Zl.
Convenient
1J.6. Homogeneous Syetem
Surroundings
imaginary A Bystem which consists of a single phase is termed as homogenaus sysfen. Examples :
/,--...ttBoundary Mixture of air and water vapour, water plus nitric acid and octane plus heptane.
/\ system 1.2.7.' Heterogeneous Systen
\ /
\l A system which consists of two or more phases is called a heterogena us syslznz. Examples :

'b""h# System Water plus stearn, ice plus water and water plus oil.

1.3. P[]N SI'BSTANCE


Fig. 1.1. The systen. Fig. 1.2. The real and imaginary bounilaries. A pure substance is one that has a homogeneous and invariable chemical composition even
1.2.2. Closed System though there is a change of phase. In other words, it is a system which is (c) homogeneous in
composition, () homogeneous in chemical aggregation. Examples : quid, water, mixture of liquid
l'3' Ifthe boundary ofthe system is impervious to the flow ofmatter, it is
Refr Fig'
a clos.ed sqste-'.
called water and steam, mixture of ice and water. the mixture of liquid air and gaseous air is not a pre
An example of this system is mass oi g^" o, vapour contained in
cylinder, the boundary ofwhich is drawn by the cylinder walrs, an engine substance.
the cylinder head and piston
crown. Hee the bound,ary s continuous on no tnd! enter or leaue, 1.4. TIIERMODYNAMIC EQI,'ITIBRITM
^ft",
Mass remains conslant A system is in tlumd.lnamic equilibrum if the temperature and pressure at all points
regardless vadation of are same ; there should be no velocity gradient ; the chemical equilibrium is also necessary.
boundar'es Systems under temperature and pressure equilibrium but not under chemical equilibrium are
sometimes said to be in metastable equilibrium contions. It is only undcr thermodynamb equi-
librium cond,itons that tlrc properti.es of o system can be fi,rcd.
firus for attaining a stsrta of thermdlrwmic equiJibrir the following three types of equ!
libium states must be aieved :
1. Thermal eQuibrirm. The temperature of the systern does not change with time and
has same value at all points of tho system,
2. Mechsnical equilibrirrn. llere are no unbalanced forces within the system or between
the suroudings. The pressure in the system is same at all points and does not change with
respect to time,
3. Chemical eqiribriun. No chenical reaction takes place in the system and the chemi-
System
cal'composition rvhich is sarne throughout the system does not vary with time.

1.5. PROPERTIES OF SYSTEMS


Fig. 1.3. Closed system. Fig. 1.4. Open system.
1.2.S. Open A property ofa system is a characteristic ofthe system which.depends upon its state, but
Systen
not upon how the state is reached. There are two sork ofprbperty :
Refer Fig. 1.4. An open system is one in which matter
of the engineering systems are open.
flous into or out of the qysern, Most 1. Intensive properties. These properties d.o not d.epend on the mass of the system.
Examples : Temperature and pressure.
rI\ I E^!AL LUMI'USI'ION ENGINES

2. Extensive properties. These properties depend on the nass


ofthe qstem. Exampre :
volume' Extensive propertis ae often divided by mass associated
with thern to obtain the inten-
sive properties. For exanprg if the volume of a system of mass.r
is v, then the specifi.y6lu6 6
matter within the system i" = which is an intensive properry.
# "
1.6. STATE
state is the contin of the system ot an instant of time as
describeit or measured fur its
properties. Or each unique ond,itan of a system
is called a state.
rt follows from the definition of state hat each property has a singre
value at each state.
stated differently, all properties ate state or point
funciions. r\ercfore, all-pmperties ae identical
for identical states.
on the basis of the above scussion, we can <reterrnine if a given
variable is properfy or not
by applying the following ests :

- A uarbble is a propert,, if and onty if, it has a single ualue at euh equilibriwn state.
- A uarfuble is a ptoperty, if and, only if, the change in ts v (Volume)
value between dny tuo pre-
scribed, quilibrium states is sngle_ualued Fig. 1.5. Cycle ofoperations.
Therefore, aty variablz whose change s fired, bl the end, states is a property,
1.9. POIIVT FI,JNCTION
r.7. PROCESS When two properties locate apoin on the graph (co-ordinate ates) then those properties are
A process occurs when the system called as pont functian.
undergos a change in a state or an energy transfer
steady state. A process may b rnn-flow in which a fixed at a Examples. hessure, temperature, volume etc"
mass within the defin uoudary is
undergoing a change ofstate. Exampre : a substance which 12
is being heated irra.i*"a cynder =v2 - % (an exact dffirental)'
undergoes a non'flow process (Fig' r'B)- crosed, systems
und.ergo non-flow processes.A process
l1v
may be a flow process in which mass is entering and reaving
through the boundary ofan open 1.10. PATH FIJNCTION
system. In a steady flow process (Fig. 1.4) mass is crossing
ilu lorriar"y orn srilou.rnc' ,t
entry, and an equal mass is crossing the boundary at the exit
so that the total rnass of the system There are certain quantities which cannot be located on a graph by a pont but are given by
remains In an open system it is necessaiy to take account ofthe work f}re area or so, on that graph. In that case, the aea on the graph, pertaining to the particular
-constant'
surroundings to the system at entry to cause detivered frour the proeess, is a function of the path of the process. such quantities are called path functions.
the mass to enter, and also ofthe work delivered from
the system at surroundings to cause the mass to leave, as Exarnples. Heat, work etc.
well as any heat or work crossing the
boundary of the system. Heat anil work are inexoct dffirenfiols. Their change cannot be written as difference be-
tween their end states.
Quasi-static ProcesE. Quasi means'almost'. A quaslstatic process is also called
uersible process. This process is a succession of equlibrium a re-
states and infinite slowness is its Thus l. 6Q '. Qq
JI - Q, and is shown as tQz or Qtz
characteristic feature,
t2
1.8. CYCLE
_Jr| 6W +Wz- lYr, and is shown as ,Wror Wr-,
Similarly
Not, The operator 6 is used to denote inexact ffeentials and operator d is used to denote exact
Any process or series of processes whose end, states o.re fferentials.
id.entcar is termed a cycre. The
processes through which the system has passed
can be shown on a state diagram, but a comprete
section of the path requires in addition a statement r.11. TEMPERATT'RE
of the heat and work crossing the boundary of
the system. Fig. 1.5 shows such a cy, cle in rvhich a system o The temperature is a thermal state of a bod.y which distinguishes a hot,body from a
commencing at condition .i, changes in
pressure and volrne through a path 128 and returns cotd body. The temperature of a body is proportional to the stred molecular energy i'e.
to its initial condition ,1'.
the averge molecular kinetic energy ofthe molecules in a system. (A particular molecule
does not have a temperatue, it has energy. The gas as a system has temperature).

,t. r
J,it

INTERNAL COMBUSTION ENC I N\ES


BAsIc coNcEPTs oF THERMoDYNAMCS .I
o fnstruments a" temperatures ae known as,thermometers,,
fl.::l_t"S{lnary
those for neasuring high temperatres are known _pyrometers,, and between two prssures. Thus, it is the dffirence between thz pressure eerted by a
ftuid. of nter-
as
r It has been found.that a gas will not occupy any volume at a certain temperature. est and the ambient atrnospheric pressure. Such devices indicate the pressure either above or
temperature is kown as absorute zero This below that ofthe atmosphere. when it is oouc the atmospheric pressure, it is termed gczge
tu|iperature. The temperatures measured
absolute zero as basis are calred
;;;fu;;'t:;;"r"rures. with pressur and is posifiue, when it is below atmospheric, it is ncgaue and is kown aa
Absolute temperature is stated udcuum.
or Vacuum readings are given in millimetres of mercury or millimetes of waer below the atmos-
;:,*1f;H:jiff*;Tt'"pi"t "'oi,'tf,^p"""tu"e is found to occur at 273.15"c phere.
Then : Absolute temperature Tbermometer
= reading in .C + 2?3.15. It is necessary to establish an absolute pressure ofthe changes
scale which is independent
Absolute temperature in degree centigrade in atmospheric pressure. A pressure of absolute zero can exist only iu
is kno; as degrees kelvin, denoted complete vacuum. ny
by K (SI unit).
pressure measured. aboue the absolute zero of pressure is trmed an,absolute pressure'.
r.12. ZENOTH LAW OF THERMODYNAMICS
A schematic diagram showing tbe gauge pressure, udcuum pnessr and the absolute pres-
o 'zeroth of thermodynamics' sure is given in Fig. 1.7.
'awto a third, they are
temperature that if two s,,stems are each equal in
states
Mathematically
"q"d;;;;;;"rature to euh other. :

fi,Tlll;l1ff:ir *:J*i l^try*,:t Ji. *",,.ore"s encrosed in a rigid vesser


(i) Absolute pressure Atmospheric prssure + Gauge pressrre
=
:"'i::J1lf ,';,:::i,f lf,:,'lli"::'t.'i:*1i'"iJ":#ffi:f ffi 1i""X'1,ilT$,i:'"Ti pb, =patu *pc"ug",
il:;1i#il1'iT-:;i',l::i'":"::::l*ri'1";ffi;";,1';ffiTJ:'.T::ll?
:T#ffi ;,;,:ffi'":"S'.:#:
(i) Vacuum pressure Atmospherit pressure Absolute pressure.
= -
l#,,i;:i"'trT:,#li*:::::i1:t;{:|iffifi
:i;::l"J:fl,;1;:"i._l:yllt*:,.:::1T_tf .-*j#ffiH,l"fff ii""':',",:ffii;""ifl:
Vucuum is defined as the obsene of pressure, A perfect uacrn is obtained when absolute
pressure is zero, at this instant rnolecular mamentum is zero.
;ffi '""?"ffi
*act with;;i:'""-'::-:::*:'.-:;;il#":;ffi'r:i:il;:f :ffi:ilT:*'"7
each other ;J;;iiJT' #fi:5;HH:":::;Tj"T'"TTT:
Atnospheric pressure is measured with the help of barometer.
:::i,iH*
"*"'ff
equilibrium.
Positive
gauge pfessufb
1 Ariospheric
I
Negative gaug6 pressufe
e
orvaqum
o
q
p
o-

OO
Zro absolut pressute

Fig. 1.7. Schematic diagram showing gauge, vacuum and abeolute pnessures.

1.13.2. Unit for Pressure


The fundamental SI unit of pressure is N/m2 (sometimes called pascot, pa) or bar. 1 bar
= 106 N/m2 = 105 Pa. Standard atmsphdd pressure = 1.01825 bar - 9.76 - ir.
Low pressures are often expressed in terms of mm ofwater or mm of rnercury. This is an
Fig. 1.6. Z,eroth law ofthermodynamics. abbreviated way of saying that the pressure is such that which will support a liquid column of
stated height.
r This law was enunciated by R.H._Fowler
in the year 1981. However, since the rst
second laws already.exist ut
trt"t ii-",'if.." designated as zerothlor and . 1.13.3. $pes of Pressure Measuremen Devices
preced,es the first and second
l"*" toii)ii o toguot sequcnce. so that it The pressure may be measured by neans of indicating gauges or recorders. These instru-
rnents may be mechanical, slecf6msqhnical, electrical o" in operation.
r.T3. PRESSURE "t""t-.
1. Mechgnical instnment& These instruments may be classified into following two groups :
1.13.1. Definition of pressure Tbe first group includes those instruments in which thb pressure measurement is made
Presture is defined
- by baloncing an unhnown force wth a known force'.-
as, a per unit aiia. pressures ae exerted
liquids' The instruments that.forcd by gases, vapours and TheseI,otld.groupincludesthoseemployinggzantitatiaedeformationofanelast.cnember
we ;;;;';;; n=iurr"., record pressure s the difference -
for pressure neg,surement,
ffi 8 TNTBRNAL

2, Electro-mcchanicel instrumens. These inshunents usually employ a mchanical


coMBusrloN ENctNEs BASIC CONCEPTS OF THERMODYNAMICS 9

meons 1.15. ENERGY, rvonr AND IIEAT


for d'etectittg thc pressure and. electrical neans for v/rirzclting or record.ing tlt d,tected
i pressure, l.16.l. Energa
3. Electronic ingtrunents. Electronic pessuro measuriiig instruents normally de- Bnrgy is a geoeral term embracing energl in transitbn and, stored energr. The stored
pend on some phpical change that can be detected ad.idicated or recorded electronicallv. energr of a substance may be in the forms of mecion ical energr ard intennl nr8C, (other forms
ofetored energy rnay be chemicd energr and electrical energy). Part ofthe stored energy nay take
1.14. BEVERSIBI,E AND IRNSVERSIBLE PBOCETIES
the form of either potential energr (which is the gravitational energy due to height above a chosen
Revereible pncesr. A reaersible process (also sclzldrincs datun line) or kinetic energy due to velocity. The balance part of the energy is knolrn as nterrlol
hwwn a,s quasi-stat process) is one whh can be $opfi d atqr ervr&t.lla tnn-flow process usually there is no change ofpotential or kinetic enerry and hence
stage and, reversed so that the qtstem and turround,irqt are a,- change ofmeclranical energy will not enter the calculations. ln a flow process, however, there may
actly restored, to thir nial staEs. be clanges in both potential and lcinetic energy and these must be taken into account while
This process has the following charateristias considering the changes of etored energy. Heat utd uorh an the forms of energt in transition.
1. It must pass rough the eane states on th reversed path Thes ae the only forms io which energr can cross the boudaries of a system. Nether heat tnr
aa were initially sited on the forward path. worh an ezist as stored energy,
2. This process when undone will leave no history ofeveots in
the surroundings. 1.152, Work and Eeat
3. It must pass through a continuous eeries of equibri-n lYork
staes. Fig. 1,8. Reversible process. Work is said to be done when a force moues through a distance. If a part of the boundary of
No real proes is truely reversble but ame prwsct naXt approach reversibity, to close a srstem undergoes a displacement under the action of a pressure, the work done I{ is the product
approrimation. ofthe force (pressure x area), and the distance it moves in the diection ofthe force. Fig. 1.10 (o)
Exomplea. fu etamples of rcarly reversible pnGarrn ae : illustat,s tis with the conventional pieton and cylinder arrangement, the heavy line defining the
(j) Frictionless relative motion. boundary of the eystem. Fig. 1.10 () illustrates another way in which work might be applied to a
() Expaneion and curnpreasiou of spring. system. A force is exerted by the paddle as it changes the momenturn ofthe fluid, and since this
(iii) Frictionless adiabatic expansioo or compression off,uid. force moves dwing rotation of the paddle room work is done.
ll
ri
(iu) Polytropic erpansion or comprression of fluid,
(u) Isothermal expansion or compression.
(ui) Electrolysis.
Boundary
Irreversible proceaa. An irreversible p.ocess is oc in which heat is transferred, through Boundary
-_ .
a finite temperature.
Examplee:
(i) Relative motion with friction (a) Combustion (a) (b)
(i) Diffusion (iu) Free erpansion
(u) Throttling (ui) Electricity flow through a resistance Fig. 1.10
(uii) Heat transfer (ui) Plastic deformation. 'Worh' is a tansent quontit! which only apped.rs at thc bound'ary whIe a change of state
An irreuersible is usually representeil by a dathl (or
process is tohing place within a slsteim. "Worh is'something' which appears at the boundary when a
discontinuous) line fuining the end, stctes to ittdiate that tlv intr- qrstem changes its state d.ue to the mouement of a part of the boundary und.er the acton of a
mediate stotes are indterminate (FiA. 1,9). 1
I force.
Irreversibilities are of two grpes: \ _..zNonEquilibrium
1. External ineversibilitie& These are associated withds-
'{ / srares Sign convertion :
spating effects outside the worhing flui.d. '.f Ifthe work is done Qy the system on the surroundings, e.g. wheu a fluid expands pushing
a piston outwards, the work is said to be posiiue.
Examplc. Meclunbal friction ucurring d,uring a praess dve -"-----2
to gone extental source. i.e., Worh output of the sXstem = + W
2. Internal ireversibllities. lhese are associated with dis- v Ifthe work is done o the system y the surroundings, e,g, when a foe is applied to a
sipatng effects within tlc worhing flud,, Fig. 1.9. Irreversible proces. rotating handle, or to a piston to compress a fluid, the work is said tnbe rcgatiue.
E .mple. Uwestricted erpansbn of gas, vscosity td inertia of the gas. i,e., Worh input to system = - W

Sr
lo TNTERNAL coMBusrtoN ENGINES
,:: BAsIc CoNCBPTS OF THERMODYNAMCS 1I
Heat
Heat (denoted by tbe symbol e), may be, defined in 6 lsgoua to the system. The First Law of thermodynanics can, therefore, be stated as follows :
is'nnething'whbh appeers at the boundary wrvn a
way to work as follows : . {hen a eystem underaoee a tlernodynarnic cycle then the net heat eupplied to
u,so,m changes its std'e d,ue to the syetem from the eurroundlngs le equal to nct work done by tbe system on lte
a d,ffirerue in tempemture between the system ond its cur"rrlwdlittgs,,
eurroundings
Heat, like wor\ is a transient quantity whichonry appears
. at the boundary while a change
is taking place within the system. or $e=6dw
t't
It
is apparent that neittre 6ll or 6e are exact difrerenots
ad therefore any itegration of
where rpresents the sum for a complete cycle.
the elemental quantitiee ofwork or heat which appear
during a chge from state tio z most
f
be rvritten as "tat" The frst law of Thermodyomics canrct be proued, analytically, but erperimental eui.dsrce
( 6W = 17,-, or has repeatey confirmed, its valt!, and gince no phenomenon has been shown to contradict it,
the firet law is accepted as a ldw of nature It may be remarked that no restrictiou was inpooed
' I ,W, (or llr), and
which limited the application ofst law to reversible energr transformation. Hence th? firgt law
2 applies to reversible ae well as irrcverible transformations : For non+yclic prooess, a-rnoe gen-
* = Q;2or rQ2
(or Q
eral formulaon of rst law of tfermodyn-cs is required, A new concept which involves a term
calbd internal energy fulfi'lla this need.
srgn convenro" ,'
rftle the First Law of thermodynamics may also be stated as follows :
heat flows in o a system from the surrounngr, the quardty
is said to be postiue -
and, converselv, if heat frows Ileat and work
are mutually convertible but since energ:f esn neither bo cre-
?om the system to tbe surroundings ii is sa tn lre ,neotr".
In other words : ated nor destroyed, the total onergr associated with an energy converlon emains
Het receiied by the systetn + e
coDstant'.
= Or
trIeat rejected or given up by the tystern _
= e. fio can produce energy without correrpondlng expenditure of en-
Comparison of \tork and lleat - ergy,machine
i.e., it is lnpoeeible to construct a perpetud noion macbine of first
Similarities : kind".
(i) Both are path and. inerat
functions dffirentizts. 1.17. TEE PER'ECT GAS
() Both are boundary phenomenon
i.e., both are recognized at the boundaries ofthe system
as they cross them. 1.17.1. The Characteristic Equion of State
(tti) Both are assosiated with a process,
not a state. unke properties, work or heat has no At temperatures that are considerably in excess ofcritical temperature ofa fluid, and
meaning at a state.
- also at very low presBure, the vapour of fluid tends to obey the equation
(iu) gys"-. possess energy, but not work
or heat.
Dissimilorities : ff=constant=E
(i) In heat transfer temperature
In practice, no gas oben this law rigidly, but many gases tend towards it.
difference is required, An imaginary ideal gas which obeys this law is called a prfect go,s, and the equation
() In a stable system there cannot be work
hansier, however, there is no restriction fo the
transfer ofheat. n, is catled the chdrateristic equatian of o state of a perfect gas. The constant I is called
(iii) The sole effect external to the system could ff =
be reduced to rise ofa weight but in the case the ga.s consaut. Each perfect gas has a diferent gas constant.
of a heattransfe other effects are also observed.
Units of ,R are Nmftg K or hllkg K.
r.T6. FIRST III1V OF TIIERMODYNAJVIICS Usually, the characteristic equation is written as
It is observed that rvhen a sJstem is made to undergo a comprete pu=RT ...(1.1)
on or by the systern, consider a cycle in which cycle then net work is done
n"t i.'ao." uy the system. since energy cannor or for n kg, occupying V mg
be created, this mechanical en:rry ",o"t
har" ."n ,rpplU fr
system has been retuned to its-initiar.r.nu:t Now the pV= mRT ...(r.2)
"._;;;;;;;;;;
state: rteee,'it" ior"* energy is unchanged, and The characteristic equation in another form, can be derived by using kilogram-mole as
hence the mechnicar enerry has Dot been p"";idJ;;;"
involved in the cvcle is theLat which was
system itself. The only other energy - a unit.
in various processes. Hence, by
the law ofconsenation ofenergy, tbe net work "rppriJ",etectd
ii"" y ii" ryrt.. is equar to the net heat - :r..,:r.c 'I\6 kitogmm-mole is defined as a quantity of a gas equivalent to M kg of the gas, where M
is the molecula weight of the gas (.1. since the rnolecular weight of orygen is 32, then 1 kg mole
of oxygen is equivalent to 32 kg of oxygen).

r.i
INTERNAL COMBUSTON ENCINBt
BASIC CONCEPTS OF THERMODYNAMICS l3
As per denition of the kilogram-mole, for m Lg of a gas, we have
We have
.m=nM ...(1.3) dQ = m cp dT For a eversibte noa-Oow poce8s at constant Pressurc "'(1'6)
where n = Number of moles.
and dQ = m , itT For a reversible non-flow process at constant aolume "'(1'7)
Note. Sine the gtndard of mas is the kg, hilogram-mole will be written simply ae nole. The values of co and c,, for a perfect gas, are constant for any one gas at all pressures and
. Substituting for m from Eqn. (1.8) iu Eqn. (1.2) Cives tomperatures. Hencc,-integrting Eqns. (1.6) and (1.?), we hve
pV = ntrIRT Flow of heat in a reversrole constant pressure pmcese
or *r= # = mcp (Tz- T)
Flow ofheat in a reversible constant volume process
,..(1.8)

, Acgoldinq tn Auogadro's hypottusis the vorume of 1 more of any gas is the sam as the = mcu (T"- T1) ...( 1.9)
volunre of I mole bf ny other gas, when the gses aE at the ame t;;"J";; In case of recl gases, c, aid uory with tunprdtwe, but a suitable auerage value may be
;d pressue. cu

Therefore, used for most practical Purposes. \


is the same for all gases at the same value ofp and ?. rhat is the quantitr
f ff i" 1.17.3. Joule's Law
conatd,nt for all gases. This constant is called uiueroot gaa cor.stant, ^
and is given the symbol, Bo, Joule's law states as folows :
L., MR=Ro= "The intenwl oi o perfect gas is a furctbn of tlw a'bsolute temperature only"'
or
# i.e., "n"rg u=flT)
pV= nRoT ...(1.4) To evaluate this function let 1 kg of a Peect gas be heated at constant volume'
Since 8 = fto then
According to non-flow energ equation,

It has been found erperimentally that


n= #' ...(r.5) dQ=d,u+dW
d.W = O, eince volune remains constant
^-^ .
the volume of I nore of any prefect gas at 1 bar and
0'C is approximately 22.71 ns, :. dQ= du
Therefore from Eqn. (1.4), At constnt volume for a perfect gas, from Eqn. (1.7)' for 1 kg
p _ pV _Lxlo5x22.7l dQ = codT
-.- nT - _r;?sls dQ=d,u=cpT
= 8314.3 Nm/mote K and integrating u = co T + If, f
being conetant.
Using Eqn. (r.5)' the gas constant for any gas can be found when the
molecula weight is Accorng to Joule's law = ?), rhich mesns that internal energy varies nealy with
known. absolute temperature. Intemal energy can be made zero at any arbitrary reference temperature.
Example. For oxygen which has a molecular weight of 82, the gas constant For a perfect gas it can be assumed that r = O when ? = 0, hence constant K is zero.
i.e. Internal eners, t = c,,T for a perfect gas ...(1.10)
*=#= -v
8|14
= 259.8 Nm,/kg K. or For mass m, of a prefect gas
1.17.2. Specific lfeak Internalenergy, U=tncoT ...(1.11)
The specifrc heat of a solid or liquid is usuay dened as the heat requred. For a perfect gas, in any process between states I aod 2, we have from Eqn. (1.11)
- lzzass through one d.egree tenperoture rise,
to raise unit
Gain in internal energl,
Fo small quantities, we have 12- ur= mc, (T"- Trl ...(1.12)
-
dQ = mcdT Eqn. (1.12) gives the gains ofinteroal energ/ for a perfect gas between two states pr any
where n = Mass process, reuerslble or irreuersble.
c = Specic heat
d? = Temperature rise. L17.4. R.elationship Beween Two Specific Heat
For a gas there are an infrnite uumber ofways in which heat may be Consider a perfect gas being heated at constant pe$ue from ?r to ?r.
added between any two
temperatures, nd hence a gas courd, have an infite nutnber of specificeos. According to non-flow equation,
However, onry two
specific heats for gaseq are dened. Q=(Uz-Ur\+W
Specifrc heat at constant voiutne, cu Also for a perfect gas,
and Specifrc heat ot constclnt pretsure, co. U2- U,= mcu (T,- T,\
Q = mcu (Tr- Tr) + W
t4
coMBusrtoN
BASC CONCEPTS OF THERMODYNAMICS
In a constant preasure process, te wod< done l5
o" *" o*nt**Al
w = p(Vz_yt) 1.U.6. Ratlo of Specific Eeats
lhe ratio of specific heat at coDs+ant preseure to the specic
f': ptl.r=vftfi, I heat at constant volume is
=mXTz-T) I *W=,nr; I
given by the symbol y (gamma).

i.e.,
On eubstituting I = p i
Pr Pz= +J'is cse]
. "' =9p-
c" .'.(1.15)
A =_y: Tr) + mR (Tr- ft' = m(co -+ t|t
Bu for a coDst^nt pregguro \rr-
p(rcsss, - "'vo R) \t(Tz_
z- T)
t t) since c, = cu + R, it is cler that c, must be trran for any pe;fect gar.
co rt follows,
'reaer
_
By equating
e = mc, (T2_ T1) therefore, that the ratio, a = 7 s ahrnls grater tlan unitXr.
the two expressions, r" 1"""
m(cu + R)(Tz- Tr) = nur(T"_ Tr) In general, the apprfomaUi valuer of Tare is follows :
cr+ R= c" Ft monoatomrc garee such O org,or, lulium 1.6,
cr- n Fq d.itomi gases such * abon ttnwid, =niir"n", nitrogen
cv=
...(r.13) For triatomic gases such
*a oryg"n ) Ll.
Dividing both sides b c,, we get wbodiifu arrd, iulpiur df""*ti = t,
For some hydro.carbons the value of
3s
cDA y is quie low.
;-r=d Ie.g., for ethane I = I.ZZ, and for isobutane 1 = 1.111
n Table 1.1. Summary of pocess for perfect Gas (Unit
me$)
...tl.ls (oI
<vin"n "=
"",
Similarty, dividing both sides by cr, we
Bet
Proes Idx
v
n6taddd
r:* p,v,T
relations
Sprrifuheatc

Coutant
-_iE_
-p- f-l ...tr.r3 ()l precaue n=0 cr(Tr-Tr) Pfu"- vr) b,=s
Tr vl cp

Constnt
Tr=A
[T $ff;;"::;:; r ;= ",5
". = volme c,(T"-Tr) 0
?z p2 c

1.17.6. Enhalpy "qol Congtat


temperare P$log"9 Pr\log lz
one t'he fundamental quanties which
- sum of
=1.
occu invariably in tennodynnics rt t'l Ptvi= Pzv2
of intenal euergy () and preseure vol"-" pJo"t is te
(pu). This gum is called
Enthatpy (). pLL{ = pdrl
i.e., h=u+pv Reversibto

-
The enthalpv of a fluid is jh qronertr
of the fluid, sic,e it conists th" ,#;tf4l
adiabatic n=l 0
ht - P2u2
T-1
&-l* )'-' 0
property and the product of-the tw prperties. \ \t'z)
sinJ "f
ual energr, preasure, epecific.volume;J;;;;;*, entharpv is a property like inter- 1-1
it cn [s itroduced into any
__
problem whether the process is
a flow or oro".*. =l It
Ttre total enthalpy of mase, m, of a fluid "
can be
"*lno* lrr /
H=U+pV,whercH=mh. Polytropic n=n c"(h-\) pL\n = p2a24
For a perfect gas, "' =",,Ir-"'
-\1-zl
Beferring equation ( 1. l4), =f fr-o) hur- Wz r"
=coT+nT t'.' RT'!
li;j
x(72-T1)
-1 \ =(L\"-t
luz)
E)T
Pu =
= (cu +
l-n
=l;^wort
.

,,e.,
=
"rT
h=cpT
t'.' co = cu + R)
done(m.-flm)
=lo)#
\rr /
and
Note. Equatioas must n*d l""pi"g di-"o"i;ffi;;
(Note rhar, since it has 0""" J;;lftt = 0 ar ?= 0, theu = 0 at ? = 0).
be

{.,
r.o,
IIFPRNAL.
1.18. STEADY F,OW EI\TERGY TEUATION (S.r.N.E.)
COMBUFTTEN,.Br{CiAgES
ffil Bnsrc coNc'prs oF r'aRn{ciDy*^Mcs
T7

The steady flow equation can be expressed as follow


In many practical problems, tbe ratc at rhich te ftuiil flows :
thmugh a rnachioe or piece of
appalatus is constant. This type of flow is called *eX
fuw. ut * * * to+Prur+ =
A,csumptione : 4', "r, + pur+ w
Q uz+ ,..(1.16)

The following assumptbru aie nade i te s,stcn anbnie I


(ur+ prvr) *
(i) The mass flow through the rysten r@ains
oostanlt
!

# * z,F * e = @r+ prv") * 4 * Z"s + w


(tD Fluid is uniform in compoeitioa. I
I
hr*+ +Zg+e=hz+ $ *zrg+w t... h=u+
(dij) The ouly interactioa betwesn
tho syge. ad surroradFgg ae wqk and. beat.
I
I If Z, Pvl
(iu) The state of fluid at any point
remin octa*t wit time.
'| -^- and, Z"are neElcted, we g
(u) trn the analrsis oqly potcctFl, I
ftinatl] d &ow eneqgis ae coneidb,ed.
+e=hz*$ *w
Fig. 1 ,1 1 shows schematic flow proccss r
s oFco ayrtm. .dn open srstem is one il iyhere
hr*
4 ,..[1.16 (c)l

llich lolh mass and ener,F Day croes the bo?qr*is. *i" i.tr.""d ; I e = Heat supplied (r emteriag the bordary) per kg of fluitl ;
pl'ace wifhin aa pen systen' It the sy8th *"y t*tu t
be a rnrtootic eugioe rith the inlet"rr""s,
DaniftIil at th
W = Yltork done by (or wcrk coming out of the boundary)
1 kg of fluid,
first state poiqt aF4 eebat$t nine as tbg secqsd ga1 lbee wqrd C = Velocity of flud;
be ea iutercharye of chemicat
enetgr in the fuel' kinetie euerlgr of notriag patidcr, iate.rnd
and shaft work within the system' rbra Fr!. r,11it
energr of gas art*ar*t r"a Z =Height aborre {afuiri;.
is obvious that if there is no variatios of flow p = Pressue ofthe fluid;
of mass or energr with time aerosg the bourdaries
dtbe system tb stedy llow will prevail. The z = Iutemal enerry per kg
contions may-pass through te cydie r,r non-cyelb I+angas
sirhi tbe systeo. As a ressh tbe orf fluid ;
,nass entering the syaeo equals tbg mass leaviag, pu = Energr required for I kg of fluid
aho enerqy entering the ystsm equals energr
leaVing. Tliis-equation io applrcabre to anymedium in any eeady
. frow. It ie appcable no'ly to
rotary rnachines such as centrifugal fans, punrps and cumpreeao*
but arso to reciprocatipg ma-
Boandary chiner euch as sterm engines.
Io a steedy flow the rat" ol masg flow of ftuid a any secion
is the same aa at a'y othr
section' Consider any secion of_c'oc-sectioal erea r{,
where thc fluid velocity is c, tbe rat of
volune flow past tbe section is cr{. Also, since mass flJ
is volum flow divided by specic volune,

Mass 0ow t".be,;=+ ...(1.17)


(wherc u = epecifrc uolwe at tte section)
Thir equation is known as the conntity of maer equa6on.
With reference to Fig. 1.U.

=crA, =44 ...[1.17 (c)l


u1 02
1.18,1. Energy Relaflone for Flow proeec
The energr equation (m fu offluid) for a steady flow
systern is given ae follows :

^ (o.4. *-4 ., =
^F
*$ * r* * o*) * o
t.?., e = -,,) + (z* - 4E) +(+ - +). r*r- a,rr] .w
Fig. r.ll ^ b
l.e. r e=
^lr-^)+se,-;a+(q#)*@*- w)l *w

,i. ,
l8 INTERNAL COMBUSTTON ENGTNES BASIC CONCEPTS OF THERMODYNAMTCS
t9
= LU + LPE + LKE +A (pu) + Itr ' Ar
LU=m(ur-u) Co-efficient of performanoe, (C.O.p.)nrp,,,re
= w ,..(L.22)
L PE = mg (22- Zr) where Q, = IIea transfer to ,t rcenoir
(c,'-c'\ W = Net work transfer to the heat pump.
LKE = m
l_-J
Lpu = m(p"vr-p1u1)'
Q-tU=l^PE+ KE + WI ...(1.18)
^(pu)+
2
For non-flow process, Q=AU+W=LU+ Lpdv

i.e., e- w= [inav ...(1.19)

1.19. LIMITATIONS OF FIRST II\W OF TUERJT,TODYNAMICS


It has been observed thal eturgr can flow from a system in tbe form of heat or worh- TJne
first law of thermodynamics sets no mit to e amount of the total energr of a syetem which can
be caused to flow out as work. A limit is impos"d, however, as a result of the principle
euunciated
in the second law of thermodynamics which states that heat will flow uaturally from oue energr
resevoir to anothe at a iower tenperature, but not in opposite diection without assistance.
This
is very important because a heat engine operates between rwo eDerg'y reservoirs (a) (b)
at different tem-
peratures. (c) Heat engine () Heat p.-F or refrigerator.
Further the frst law of thermodyn amiq estoblishes equvalence betw,een the quatttty of Fig. r.l2
heat used and' the mechanial worhbut dbes wt specify the conditions under whbh
conversian of In all the above three cases application of the
heat into work is possible, neitltr tlp dirttion in which heat transfer can taheplace.
This gap
first law gives the relation
e, e2= W, and _
this can be used to rewrire the exprtsions ror tr,"r-ai"m"i;;;J";;;;ritJp""ror."r,.u
has been bri.d.ged by the second law of thermrodynamics.
solely in terms of the heat trasfers.
1.20. PENtr'ORMANCE OF EEAT ENGINE AND RBYERSED HEAT ENGINE
Refer Fig. 1.12 (a). A heat etqine is used to produce the maximum work ransfer
qo =q:g ...(1.23)
.
given positive heat transfer. The measure ofsuccess is called ttre thermal efficiena
fron a
and is defined by the ratio :
ofthe engine (c.o.P.)rcf =#6
,..(L.24)
W (c'o'P')'wtP
Thermal efciency, ,Lu =
e, ...(1.20) P=d3" .,(tz6)
where f{ = Net work transfer from the engine, and t be seen that 'n is alwals thn o'd (c'o'P.) ;
less is
Qr = Heat transfer to engine. uoir, .lt ^ 'itv pup always greater than

For a reversed. hzat engine^[fig. l.l2 () acing as a refrigerator when the purpose
is to 11. STATEMENTI OF SECOND I.AW OF IHERMODYNAT}ICS
achieve the maximum heat transfer from the cold resJnoir, tle easue of r,r."r"
is called the
co-efficient of performance (C.O.P.). It is dened by the ratio : The second law of ther:arodyramics
has been euunciated meticulously by Clausius, Kelvin
and Planck in slightly different words although both statenents
are basically identical. Each
Co-eflicient of performance , (C,O.P,)-f. = 92 ...(1.21) etatment is based on an i*euersiblz process.l\e
first considers tf"irrrtli-if heat between
two thermal reseruoirs while the second, considrs the transfonnation
where Q, = Heat.trasfer ftorn cold eseruoir of heat intL work.
1.21.1. Claueiue Statcnent
W = The net work transfer to the refrigerator.
For a revereed heat engine [Fig. 1.12 ()l acting as a hcat pump,the
'It is impossibre for.a serf acting machine worhing in a cycric process unaided.
by any
measure ofsuccess external agency, to conuey hear from a body at a lower tenperature
is again called the co-efficicnt of perfqrnance. It is defined ty the ratio : perature". to a boily ot a hgher tem_
rn other words, heat of, itsel{, cannot flow from a colder to a
hotter body.

.i
INTBTNAL covBusrul BcHBs &AS,TCCONCETTS OF TmnMqDyNAHTCS
r.21.2. tervrn-planck statement 2l
I fluid receives a emalr smo',rt of Led de in an elementar-y portion
o of an ope,ration B when
"ftn "::;,tf'::;:i #:ff;l?i*ifT;"!"!fo::!*.*"":ti,ts in a cycb prcdrut to other temperatur is r, and if de is rwreeecm t tb; ;;-;:ffffi";
d";ir" ff;
;
rj ordiaare; rhe
I width of the figure mrrrt be# . lais is cslled 'bwrenzent of entropy, and is denoted by ds,
"0"*.,t1',n,"Jili:*,:*ff "ffi,H#i{:z#:i,:#i"#ffiIL{,1ft
vt e'wr $atunrcnt implies
violatian ittw' of
the
r.22. ElvrR,opy i"J:13:#ffffi"ffifr be siven bv the area uder the cu*"e B and (s, - so) wr
1.22.1. fnrodueion From absve we coclde tbat :

In heat en''ine theory, ,h:


which by other rnethods can l":- :ntropy plays a vitel role aud leads to importan reeuls Entropy chaugt, g= -- SeatChange(Q)
.Aboeflute temperature
be obtained _u.h Joi,.iuuo.ioorr". (Z) '
It mav be notecr that.alr "Entopy mey oleo fu defincd as
l^":1* l" lrdly **"le for.converting into wor* Ireat tbt is
th,e thermar property of a subeta.nce which remains
constant wrun suhswe t e$t or compressed, oint""uy' n
;::i:l:ii'"",f:ffil:I'Ifilf"T:trJfj'1Tt"" po.,i'*y orconvereion inr,owc*
than Notet
.atcd
d stDdrfoi secic eotrcpy wfreees .S,meaas total entroID, (., ,g
"-li""b. )
= n$).
,*u"fi,T"l,#0."#::H"";x:H'ly#r:::"showethepossibitttof uu*nbnof 1.22.8, Chorctt$Ice of Eofi:ppy
a3d is.sreater when i8 addpd' at a hish t"^p*"ht; \\e cllrlrorlteri&ics of ttopy in nnmarised form
ae given below :.

ffi
tat add,ition
" "r"ii "'2:f:-!*: rt ioceases FL.o hed b cupplied irespective of the fact
'f:;':,::;:';;W;iy::!::,,:;#i;;;,ffi";:n#fr ::#ffi o" oot,1' whether temperature changes
2' It deceases cen heat i8 reuo'ed whether temperature
changeo or not.
L It remais unchagd in all adiabatic frictionlesE pr,ocesses.
",,"r*"i:n::{"it#l'-'f*e}Hn;f
p-U"tilitv.
thermodynami" ;i_T::ystT.relchesarabreequibrium
aximum dieoder and i
of --;-uui one rt irasei f tmperature ofheat is lswered wito*
work being done s in a thottling
oro""J'
1.22.2. Tempereture-Entropy
Diegrein
Ifentropy is proted horizonta,y Table 1.2. Sunnery of Fornule
rrrd .dote ternpe:ature. vertica'y,
is ealled temperatire entropy (T+),"g."_. the diagram
rtI'iil*a_ is showriu
so obtained
ne. i.rJ. rf rorhing Chonge oferopf @er hg)

G) culos"fr **r*,f (intcmofTandu)

(ii) c,lo4 *",tog,f, nrrmsoflanau)


ff
Gi) cotos,
7 - *r*,f 0n terms of ranaet
Contstvolube
",",c,+
C@setFersorB -72
cp1o8,
E
IsotheDDI *r*,t
Adibeti: Zero
Polytneic
".(#),,"7
I3. IITD THIRD I./TW OF TEaNMODYNAMICS
Fig. t. 13. Temperature_euhopy
diagram.
The third law of therrnodynamics is stated
as follows :
"The entropx of atl perfect crystalrne sorids is zero
at absorute zero temperature,,.

+ i..
Z' INTBRNAL COMBUSTION ENGINBS BASIC CONCEPTS OF THERMODYNAMTCS 23
The third law of thermodynarnics, often referred to as Nerrrat law, provides
the basis for
the calculations of absolute entropies of substances. HIGIIILIGHTfI
According to this law, if the entropy is zero at T 0, the absolute
= entropy 9o,. of a substance
at any temperature ? and pressur p is expressed by the expression L Tlernd'ynamis is a! axiomrtic science whidl desls rrith the relation amogbea! wort and properties
ofqrstems whidr are in equilibriunr" It basically mtails four laws or arioms hqD ashrcth, First, Second
='o =', and fIrird law ofthemodynamics.

where
""0.
=
[4
T" = T, = T"f = Ta
"o, +. +. f ", # * E f,"* # ...(1.26) Z A syctem is a finite quantity ofmatter or a preecribed region ofspace.
A syeten may be aclovd,, open or isolced system.
...for fusion,
& Apce is a qumtity of matter which is homogeneous thmughout in chemiol compoeition antl physiel
Tf, = T, = Tfc = T* ...for vaporisaon, 8tructure"

cr", cr, cr, = Constant pessure specific heats for solids, liquicls 4 A,lonryenazssysem is onewhichcosists of a singleptuse,
and gas, and I Alctcrogenaus sysfzrn is one whic cosi.sts of ftro or more phavzt,
h"f , hf, = I^t otheats of fusion and vaporisatiou.
(L ltpttrc subctme is one that has a homogeou md invriable chernical compcitim evln though there
Thus by putting s = 0 at f= Q one rray integrate zeo kelvin is a change ofphase.
ad stanlad state of29g.15 K
and I atm., and find the entropy.difrerence" 7. Asysterrisinthznd'yrumicquilibriumtempratureanilpreesumatallpointsaresame;thereshould
Further, it can be shown that the entropy of a crystalline substance be w velor;if gndiznt.
at ? = 0 is not a
& Aprcperllofasystmisacharactristicofresystemwhichdependsuponitsdate,brutnotuponhowe
function of pressure, ,", f+.1
l.dp.ir=o
=0.
gtatis readred.
Intznsive prcprter do not depend on the mass of the system.
However, at temperature above absolute zero, the entrdpy is a function Extzrciw prcpertics depeud on the mass of the eystern
ofpressure arso, g. is the condition of the system at an ingtant of time as descibed or measured by ite properties. Or each
'Sae
1.24.. AVArI,ABLE AND T,'NAVAIIIIBI,E ENERGY mique condition of a system is called a etate.
1O. Aprooess oceurs when the system tndergoes a ange in state or an energy banrfertales placs t a steady
There are many forms in whi aD energr ccn exist. But even stta.
under ideal conditions all
I these forms cannot be converted conpletely intowork. fiiis 11. Any pmce.ss or series ofproceesee whose end states are identical is termed ac)cr.
indicates that energy has two parts :
- Available part. 12 Theprussn of a system is the force exerted by the syrtem on rit area ofbouilrie. Vactrum is defitred

Unavailable part. as the absence ofpresilne.


- 13. A rcwrsiblz process is one which can be etopped at a-ny stage ad revesd so tlat the system atrd
'AuaIable energl i8 the maximum portion of energy whih
could. be conierted. into usefut surroundings are exactly restored to tleir iitial state.
work by id'eal processes whih red,uce the syem to d d,ead s!e (a
state in equibrium with the Aomve''ible ptwss is one in whidr het is transferred tlrough a finite temperature.
earth and its atmosphere), Because there can be only one value for
system alone could do while tleacending to its dead state, it
maxiurum work which the 14' Zcroth law oftermodynamics stat8 that iftwo systems are each equal in taqerature to a third, they are
follows immediately that ,Auaildble equal in temperature to ead other.
energy'is a propert!.
lS. rnfinite slmees is the draracteristic feature of a quasi-static pmess. A quari<tatic pmcess is uccsion
A system which has a pressure difference from that of surroundings, of equbrium states. It is also called a rwersible prous.
work can be obtained
from an expansion process, and if the system has diffu""ot temperaure, l& InteruI erergy is the heat energy stored in a gu. The internal energy of a perfect
heat can be transferred gas is a fmction of
to a cycle and work can be obtained. But when the "temprature tmpcmtureoaly.
and pressure becomes equal to that
of the earth, transfer of energy ceases, and although the system 17. Firet law ofthermodyaamics states :
contains internal energy, this
energy is unauailable. Heat and work are mutually convertible but eince energy can neither
summarily available energr denote, the ratent capability of enerry
- total energy associated with an energy conversion remaitrs cogtant.
be created no destroyed, the
to do wor! and in this
sense it can be applied to energy in the system or in the surroundings. Or
The theoreticar narimum amount of work which can be obtaned, No madrine cm produce energr without corresponding erpenditure of energr, ie. it is impossible to
p t and, T, when operating with a reseruoir
from a srsten at an! -
Y': at the constant pressure and. temperature po and construct a perpetual motiou mchiae offint kind.
To i.s called 'avitability. ,
Fint law ca be erpressed as follows :
e=AD+V{
Q = AU + I{ ... ifelectric, mgnetic, chemical energies arc absent and changes in
potential aad kinetic energies are neglected.

+i
BASIC CONCEFTS OF THBRMODYNAMICS 25
1& I'here ca be no maine whi wuld continuoucly
'il eupply nedradcal wort without soe fom
disappearing simultaneouly. su a ctuou ncrrine of enagy
sntenr performs
is cana a perpctuI mtior mschi'e of the first 'When a a revenible qrcle, then
kind, or in brief, PMMI. A pMMf itue impossible.
d 19. Ttre eDergr of an ieolated ryateu ie elwar constaa s r!9)
20 Incaseof ffi\r t =o,
(i) Reverible conetalt votroc proceae (u
= sonstaD
but who the cycle *, *TT*)
"
(ii)Reversibrec.o"*"r*ji";[[:"1;l.*f L lT ) .0."
"Qz-r) Cr.lc '
(i)Bcversibr,e*-*'"tJ':1';:li:#'"-K=8j13"r,r, 24 Etropt' i! a frraction of e quaotity ofheat whidr shws tle poeeibility of mveraioa of tat hat into
wort ltireaeineobopyiremallwhenheatiaddedetahightaopcabeaodiagreaterwheuheat
a a=r atldition is mds at lower tmperature. llg fe qlrimrr cntropy, tiie i! r nniru availabity for
where = s)rpansr"" ..fl]5;efttoa
", pmr(pu?=
convenin ito work and for miium entropy there is nn 'r avilabili$for coaveion into work.
(iu) Beverlble adiaboti,c 2 Thethtdlwof termodynanlcistatedasfollorg:
couetat) ')
The enEopy
t&= r &4__4) ,=(*)+ of all perfect crystalline ode iE zero at absolrdc rero temperture'.
w= ,*=0,+=(#l
(u) Polytroplo reversible proccee (pu OBJESITVE TTPE QTJESf,ONS
= contat)
Chooe tbc corcct anrer :

*e ffi':,;'':;;..
L A dofite are or 4lace wbere some therrnodynamic pmcesc tatee placa is Lnown as
(a) thcmodynaoic ryatem () themodynamic sycle
(c) ttcrodyo:rricproess (d) themo4ynaniclaw.
An open syrtacrir one i whidr
2L Steady llow equation can be e4reeced as followe : (o) het ad work croaa tbe boundary ofthe syrtem, but the mrrs ofe wutiag substancs does not
() narr of wo'*ing ubgtace croe the boundary of tba ryrtcrn but te beat ad rork ilo not
ur* zg + (c) both the heat asd wort a well a mass of the workiag eubctaacc croee tbe boundary of the rystem
# + pror+e =u"* 4 +zrg + pr,+ V
(d) neitb e het and *ork uor the mass of the worting sub3ts 6osr e bouudary of the system.
Al iaolatcdeyutem
nr*$ re=n"*$ +l7,neglelngZrald!2" ...(tt) (c) ir e rpcdfed region whee tranfer of eaergr aaor marr taLe place
where Q = Hea eupplied pBr Lg offtu ; If = Wck I tgd0uid;
ilone by
() lr r rrgio of cortt .re rtrd otdy euergr ie allowed to ooes t boundrig
C = Velocity of fluid;
(c) c!d trrnrfrr citber energr or maas to or from the currmndingr
Z = Height aboveiltuo; (d) ir me iwhic,buaas withintlre rystenir not neceerarilycstrnt
p = Preagure ofthe f,rid ; z = Jatml eoe4t per lg of f,uid ; () none ofthe above,
pu = Energr required per kg offluid.
In an edeosivc pronrty ofa themodynarnic sysem
This equatioor is applicableto anynedium in anysteady (c) erteocivo heat il trasfered () ertensive wori( is done
f,ow.
Clauius tetement: (c) extroeive eneqg i utilied @) allofeabove
Tt is impossible for a serf-acting maLine working in cyclic proeer, rmaidetl by () one ofte above.
convey heat from a body at a lower tomperatur"
a e ent'lal agency, ro
to . t*iy a ltr, tcopemtua.; Wbich ofthe followingir aointearive pmpertyof a themodmamicryrtam ?
Kelvi-Planck tatement : "t (o) Vohme () Tenperature
tt is impoesible to coastruct au eogine, which wlile operating in a cycle producec no (c) lf (d) Energt
other e&ct exept to
extract heet from a siagle resenroir and do equivalen:t
amorit of woL. A Whicl ofthe foondng is the elteneive property of a thermodynamic ystn ?
Although above statements ofecond-Iaw ofthermodynanics (c) hcs!r () Vohme
aprear to be difieeut, tley are rea'y
equivalent in the sense tat violation of eitler stater*jt (c) Tempcehrre (d) Dosity.
impliee violation of orer.
Clausius inequality is givea by, Whootsobodies arein themral equilibr{uwith athirdbodytherare alaointhemal equibrium with
eac.hotea. lbis atatemeti called
Y r!e') ()Zsrutbwoftheruodyaamic ()Firatlawoftlermodyaamics
k\r)<0
vFtc (c)Seccndlroftharnodynanics (d)Kelvin-ptcHtw.

t..
^a'

k*-*r-

26
INTERNAL COMBUSTTON ENGINES aa
BASIC CONCEPTS OF THERMODYNAMCS
The tempenture t whic the volume of a gm becomes zero ie eed
(c) absoldclcaleoftenperatue () absolutezemtempeatrre 21. Itre main cause ofthe irreversibity ie
(c) absolulctoperature (d) none ofthe above. (a) mechmiol aod fl uid friction () urestricted erPani@
L lhe value ofmo ba ( SI units) is equal to (c) heat trafer wi a finite temperatue ilifrerence
() lO0llrf () 1000N/m (c)lx10rN/m2 '(d)alloftheaborre (c) noeofeabove.
(d) I x 1grqot () I x 106N/mr. 2a Acmrdingtokineceory ofheat
10. T?re absolulG rero presure will be (d) tempentuo should rise drring boiling () tempenture should fa during freezing
() whe mdecular monetum ofthe
system becomes zero (c) at low temperatue all bodies are i sod state
() atselenst
(d) at beolute *rc there is absolutely no vibration ofmoleol
undervaomon'itios []1ff"Hffi1il:";fr*
(d)
() none oftlre bove.
lt. Absolute rceotempenture taken is ag
(d) -273'C
2& Asystm @mprising asinglephrue is called a
(b) 273C (c) closedslBtm () opensystem (c) isoltdEstzm
(c) 237"C (d) -373.C. (d) homogeneous system (e) heterogeneoue sntern
l2 Which of thcfollordngis correct ?
(c) Absoha preasure 2L If all thevariables of a stream are independent of time it is seid to be i
= 9r96 pressure + atmospheric pressue (a) steadYflow () unsteadylow (c) rniform flow
(b) Gaugr Fsue absolute pressure
= + atmcpheric preesue (d) dcedflow () eonstatflow.
(c) Atno4:ric peseun absolut prcssue
= + gauge pessue 25. Aconholvolumsrefrsto
(d) Abaolo peasurs
= gpugs pessure _ atmospheric pessure. () afiedngioninspace () aspecifiednass
1& Theunitofcaergrinsluitsis
(d) a reversible pocess only
(c) mioltedsystem
() Joule (
() Joule metre (Jn) (e) adedsyrten.
(c) Tttatt(W)
(d) Jouldmete (Jt). 26, Internal energr ofa perfect gaa depeuds on
14 Onewattircqual () temperature, specic heats and enthalpy
(a) teryerature, specic heats and pressure
(c) lN/ () f N/Din (c) l0N/s (c) tenpeatue, spcifcheats.inalntroPy (d) temperature onlY.
(d) 100Nn/b () lffiNrr/n Zr. Inrwersiblspolytopicprocese
l Oe joule (is equl to (a) trueheat hasfer ocsurs () the entropy remairo constilt
(a) lNm () kNm (c) t,he enthalpy remains comtant (d) the intemal enersf remains coDstant
(c) r0Nn/s (d) r0kNm/s. (e) thetmporaturereminr costant,
te The amouat of heat rquired t raise the temperatun
of l kg of water trr,mgh r.c is carlod 2& Anisenbopicprocecsiealwayr
(o) specic beat at cooitant volure (a) iresersiblo nd adiabac (b) rwesible and isothemal
() epecifc heat at corstat prssure
(c) kilo caluic (c) frictioless andineversible .
(d) revenible and adiabac
17. fire heaingmd expandingof agaa is called () none ofthe above.
(c) thermodpamic tlrgtn 29. The net wort doae per kg ofgas in a polytropic process ie equal to
() therrrodraanic cycte
(c) thenno{aamic proceee
(d) thermodynamiclaw.
l& (a)Prurb&
fr
() pt (u1-u) t"lorl*-al
lT::If:.rn,whichtakeplaceinacertainord*"orot
(o) re*rsiblccycle ""t"rO_"rO_*knowaas \ v2)
() ineversiblecycle

19. fire
(c) thennoQnamic crcle
conditionforthe wersibty of a cycle is
(d) none ofthe above. @rry @)u#
(c) t'he prcerure 'd
temperature of the working substance Dust 30. Steady flow ocore when
sumundiogr at aay stagein tlre process
lot differ, apprciably, fiom those of the
(o) conditius do s chnge with time at anypoint
() alt e poceases, taking place in the ' () cmalitimg are e sane at adjaceot poins at any instDt
cycle of operation, must be extremely slow
(c) tlre wotbgparts of the engine nust (c) onilition chage steadily with the time
be friction free
(d) there shoold ba no lms ofenergy during
the cycle ofoperation fau\
(e) allofthebove (d) isconstaL
(fl uoneoftheabove' I:J
a). Inairrwsibleproceas,thereisa 81. A reversible process requires tlat
(a) loss ofhert
() no loss ofheat (c) there be o heat transfer () newton's law ofviscosity be satisfied
(c) gain ofheat
(d) no gain ofheat (c) tcnperature ofarsten and surmundings be equal
(d) therc be no viscous or coloumb fiction in the system
(e) heat transfer ocos from surrourdings to system only.
NTERNAL COMBUSTTON ENGINES BASTC CONCEPTS OF THERMODYNAMICS
3Z the first law of thermorlynamica for eteadyforr
(o) accouts for all cnergrentering:andlcavingacontrol d5. thsgtetgsfaeubetalcewhose wapontioofromitsquidetateiscompleta,islcnown as
()
volune (o) valnur () perfectgas
is an eoergy balance for a :pecied ,.- iu
(c) is an orpreaeion ofthe conevtio. of linea momeotum
(c) h (d) stan-
(d) is primaily concemed
vith heat bafer. tt6. I SI Eait, ths value oftle miwrsal Sas cotrstnt is
() i.restricted iD its applicatio (c) OtlSl4JbolelK () E,314J/tole/K
to pede grs.s.
u rne characteristic equatio of gasespV zrffl
ho$s good for
(c) 83.1,1Jnole/K (d) 8illJ/noldl(
=
(o) monoatomicgases () (e) 8314J/nollK
rthtolricgs (c) nalgaaes
(d) idealgaes ni:tunofgsa
(e) Wheo te gas i,o heated at constantpreraure,tbs heat nrrped
tl4 Agas which obeys
kinetictlreoryp""f*tIyf"f"* h) increases the iaterual eneqy dtbe gac () iiraeaes the temperature ofthe gas
(c) monoatomicgas -
fal aatlcg -"" (c) realgas
(c) ds sne *tmal work drring erpauim (d) both () and (c)
(d) puegas (e) perfecg; (e) otr ofthe above-
85' Wok done in a free eryansiouproceer 48, The garconetot(.R)iseqdtoet )
le
(a) zero () -il*- (c) sum of two specifi c he () feence oftwo specifcheats
(d) positive
(c) _"*m_o (e) poduct of two spedfic list
(e) regrve. (d) ntiroof tro specic hears.
36. Whidr ofthefollowingis not apoprty
oftGsrrtem d9 the hoat absorbed or rejected riag a polytropic procees ie
(alremperatue () precurr
?

(d) Heat fv -'\


C) |.i:1J x work done ()
82. In-che polytropic procese equa;Ul"*.*r, -''-" *:X::" [t+)'xworkdone
()coeh*pre (c)cm8t&rrodperahlr ("-o\w
fiiffin1!""*" lijj
x wortdone (d) (t:!r-J 'worldooe.
BB' Inthe polykopi" n"oor.
*o"oJl*ffia ifn ir fufiaitely hrge, the process i8 ttr'd
",
(c) corutautvotrme as 6lL Secoud lrr of thermodynamice rh6nes
d) otst ; (c) cortanttemperatura () hat () worL (c) enthalpy
(d) aabatic () iotcol
89; ?herocesses or aystens that do ot
(d) entropy (c) intrnaleer$.
itvohc het re called
(a)isotlernalpocesses 61. For reversible adiabatie prrceea, the ge h empy ir
()qrdfoi;-; (c)rermalproceeses
(d) steadyprocesses (a) z*o () niinum (q) ntmtn
(a) adiabaticgooesses.
4{L In a reverible adiabatic proceee tLe ntio ("rllr) (d) ifinito (e) wity.
eq".f t" 6A For aly reversible process, the ag in entropy ofthe system aad surroundingp is
frl t
Y-l
(")
()
\.-
/,.. 1
(c) ero (b) rlaity (c) negtive
\p2 ) 11! |
\rz )
(d) neitive (e) ifinite.
6E For any inevmible prrcss the net ltmpy change i8
v-I
(c) () zero (b) pcitivs (c) negative
lula)fi. 1a [d'.
\ut.i
(d) inita (e) uity.
41. Inisothermalprocess the rocesses of a Cauot cycle re
() tempeature inaeares gradually (a) two adiabatic ad two mmtat volune
() volume emains coDstent
(c) presoure remaine costant () cre costent volume and one coDstast prrssswe and two iseotropics
(d) enthalpy ctralg is
(e) dange ia otenal enery marimum
is zero. (c) two aabatics ad two isothemab (d) two costart volumes md two isothemals
During throttling process () two isotermals ad two isentrooics.
(c) ioternal energr doee not
(c) entropy-doee ot change
chaage () preesure doeo mt
chaag? 6& Isentropicf,owis
(d) eothalpy does not (c) iraersible adiabatic flow (D) iileal f,uid flow (c) perfect gas flow
() volume chage ie change
negtigible. (d) frictioless reversible llow (e) reversible adiabaticflow,
when agas isto be stored, the type'ofcorpresaion
isotheimal
() () adiabac
- idear i
tlat would be 6d In a Carnot engine, when the working eubetane give heat to tle eik
(c) polytqic (c) tlre temperature of the sit inceasa (b) the temperature of the sink remains the same
(d) constnt voiums (e) noe of thc bove.
(c) the temperature of the or:ee decneacs
be.rroped at any stag aad rwersed
Ift*::".S+
restord to teir inu"r-"Lt"i, iii, too*o
so rhat t ly8t and surrouadirya
are eractly (d) the temperaturea of both tbe giDt and the source decree
(o) adiabaticproess
irl ""
i**,"rtn"tp"o"""" (c)
(e) chaagns depend on the operating coaditions.
(d) frictionless process idealprocess
() energyless pocess.

't. .
30
ENGINES
BASIC CONCEPTS OF THERMODYNAMICS
67, Ifthe temperatur of the source is j:::"
iroeased the eficiency 66. T.l:e entropynay be expresed s a fuction of
(a) decreaees
() inqeaca " """'"usnoN (c) pressure and temperature () temperature and volune
(c) does not angu
(d) rill be equal to the eficiency ofa practical (c) heat and work (d) all ofthe above
(e) depends on otler factore. engine
() noe ofthe above.
5& The efficiency of a ideal
Carnot engine dependa on 47. The mge of entropy, wheu heat is absorbed by the gas is
(c) working substace
,.r_on e
() ..- ^__
temprature of the source onry (o) positive () negative (c) positive or negative,
(c) on the lemperature
of the sint only 6& Which of the foowing statemeats i a comct ?
(d) on the tenperahrres () Tle icraase in entropy is obtained from a given quanty ofheat at a low tempeiature
of bo e sorru and tle eint
() o the constructio () The ange i etropy may be regarded as a measure of e rate of re availabilitr
of engine. of heat for
69. The effrciency ofa Carnot transformation into wort
engine using an ideal gu aa the working
eubetance is

b)
rrlr, Tr
(c) The enkopr represents the maximm amout of work obtainable per degree op
(d) All ofthe above.
in temperature
()
'1 Tt-Tz
"rffi 60' The conon for the revenibtr of a cycle is

60.
@w
I. a reveraible cycle, the entropy
*,ffi. () the pressure and tenperaturo ofworkin! substance mu6t not tlifrer, appreciably fmm
surroundingr at any atage in the process
() alltheproceset"lo.gplaceithecycleofoperaon,roustbeextremelyelow
tlose ofthe

(c) increases
thj.;;
"f (c) the wortig parts of the engine must be friction fre
() leceae
(c) does not change (d) there should be no loss denergr during the cycle ofoperatioa
(d) first inaeares and ther decreueg
(e) depends on the pmrertiea (e) all ofthe above,
. ofworking eubstance
^_
61. A-frictionless heat engine can Ue fOO* 70. I an irrwecible process ere ia a
emae* mly ifits erhaust temperature is
(c) equal to its input temporaturc ; (a) loes ofhet () no loss ofwork
() leas lhn its input tempraho
(c) 0'C (c) gain ofheat (d) no gai ofheat.
(d) 0'K ()
d lGlvin-plac,elawtlealswitt -100"C. 71. The mai cauge for the irr,cversibilityir
(c) conservation of energr () conservation (o) mechical ad fluid frfutim () unreshicted erpansion
(d) conversion ofheat into
ofheat (c) cunervation ofmas (c) heat traDcfer with a fiite temperature diffeence
work (e) convesion ofwork into heat.
o (d) allofeabove.
H*f*efollowingstatementsiscorrectaccordingtoctawiusstatementofsecod"J;;L*, 72. fire efcieosy of e Carnot rycle maybe iacreased by
(c) rt is impossible to trasferhet (a) increasige higheettgmperature () decreasiog the highest temperahre
from a body at a lmer teinperature to a body at a higher temperature
t" (c) increasing the lowest tenxrature
*iilTf:*.1rfiT** m: * "i",.** "mperature to body ar higher remperarurc,
(d) decreaeing tlre lowest tsmprtur
a a (e) keepinge lowest temrnture constanL
78. Tfhich of tbe following ieteorac statement ?
"' ffifl:t*:*m"h""*,' abodvatalowertemperature toa bodyatahigherremperarureby
(a) All tlre evereible engine have the same efficienc:/
(d) None of tle above. () AII the sersible and irrwewible eugines have re eame effcienry
6if, According to Kelvin-planclCs
satement ofsecond (c) Inwersible engines hqve n-im'n eficieary
law ofthermodynamics
t'll,:,Hff;?:tocmshuctaneogirc'*"';;;";"*';;;i."or"o"r"eitoonverrhear (d) Atl ergines are desigled as rcersitle in order to obtei. m-imrm.effisienoy.

*' an engie working on a cyc proceas, whoee


lt":rffi$"frco8buct sole purpose ia to covert the
(c) It is impossible to
constluct a device which while working
L () 2. (c) & (c) tL (e) () 6. () 't, \@t
rhan the trasfe orruat in a cyclic process prod.uces no effect other & () s (d) lo ()
run a coieiuy t"i-Irllii
u""
ln. (o) l& (o) 18. (o) 14 (o)
(d) whe two dissimilar 1&'(a) tA (c) r7. ()
metale ar-e heted at one end ad cooled
at the other, the e,m.f. ateveloped is
1& (c) lg. (e) tL () 2t (d)
proportional to the diference
ofteir tqrp;;;;the two erd"
z. (d) 13. (d) !( () %.(a) n@) tl. (al a.@)
(e) None ofthe above. n.kD Sl, () EL (d) 8 (d) 8& (c) Sd () 86. (c)
* which iDcresres or decreases ar the heat is supped
88. (d) 87. () 8& (c) 8& () {ft, (o) L (e) a^@)
3::"1iTTi*T,:"H:#bstaDce or removed i a tl3. (c) it1, (c) aB.() {A (e) 4fl, @' ae
(o) enthalpy
(D) {O ()
EO (d) 6L (n) 6& () 6& () L (a)
() internal energy 65. () 6A ()
(c) entropy 67. () 6& (d)
(d) erternal energr. S, (a) dt (c) 8r. (d) @"(d) 0s. (b)
64 (e) 6 (c) S. (s) 87. (a) 8A (d) 69. () 7o' (o)
7t. kL 7z (d) ?il. (c).
TNTERNAL COMBUSTION BhCINES

THEOR^ETICAL QI'ESTIONS

l. Define a thermodynanic syrten Difiereatiate between open srstr\ clced system nd a ieol,td sys-
tem.
z How does a homogeneou rrrtendiffer fron a heterogeneous system ?
,& What do you mean by a pure substonce ?
tL Explin the following terms :
( State, (it)Poces,and (i;i) 6!de.
Erplain briefly zegth taw ofthernodyaaaies.
6. Whatisaquasi-it[iprocei?" : -rI:.', il . :,' il:,i,.! n-i. tt ;-; l:,i1, :',: i.: ;. r t... Introduetion to Internal Cornbugton Engines
'1. Wat do you tr byteveigttciotr ? " .."
& Defnelnternalenerg/an{p.rwetlatitisp-lprqgfatSlFtFr$i.:,,"i:.ri.
9. Explain the'FirstLaw ofllbegrodyaanics aereferr{-to_elosgil rrste.ns unttergoi{i ry
,,,!:; .;.., 2.1. Heat eugines. 2.2. DEvelpiiidt,r;f tC.,erginasr 2.3, Classification of I.C.
1(I
.dit""g"
state the First Iw of fiermodynamics and prcve'tlat fora ob-flow process,.it leads to the onergr 2.4. Appnci'ii?,IiG; iinbs.'2,5. Engine iycle-Energy balmce. 2.6. Basic idea")rro"". of I.g.
equationQ =AU+W. engin6. 2.7:'Dlfiertrt'parti'bf I:C. engines.' 23. Terms connected with I.C. engis..2.9. Worting
11. Wtat cycles. 2.10. Incator diagram. 2.11. Four-troke cycle engines. 2.12. TWo stroke cycle engines.
is the me.hanical equivalernt ofheat ? Write downits valuewhen heatis epresedinkl anilwork
is expressed in N-m. 2.13. IntakE fdfompression'iga,ition engines. 2.14. Cornparison offour stoke aud two stroke
tz cycle eughe. 2.16. Cooparieol of epuk ignition (S.I.) mil conpression ign:ition (O.I.) enginea.
What do you mean by ?erpeturl motion macline of first ldad.pMM l, ?
2.16. Compuison between a petrol engiue and a dieeel elgine. 2.1?. How to tell wo stroke
13. Why only in mrotmt pressure n*.liw process, the athalpy ange is equal to heat ba*fa ?
cycle engine &om a fou atmke cycle engine ? Highlights--Objective tlpe Questious-
t4 hove that the rate of drange ofheat interclarg peruit chaage of volume when gas ie conpressed or theoretical Questione.
expandedisgiven W#"#
wri-te down thegeneralenerrgraqumforsteadyfloweystenandsinplifywh*.fitiat*"ton*iog 2.1. IIEAT ENGINES
systems:
' (i) Ceatrirgal water pump (tii) Recipmcating aircomprreeeor
Any type of engine or machne which deriues heat energt from tle combuctian offuel or
(iii) Steamnozzle any otlrcr source and. converts this energy ihto mechanal worh is termed. as a lrrltit engine.
(iu) Steamtubine
(u) Gastwbine. Het engines may be classifizd into two main classes as followe :

16. Explain clearly the dilferencc between a non-flow and a steady flowprocesa. 1. Erternal Conbuson Engines.
t7. State the limitations of rst law of temodynamics, 2. Intemal Combustion Egires,
1& TVhat is the diftereoce betweeu a heat engine and a rtt8sd h.Ht bgiDe ?
1. External combudlon nglns (E.C. enginer)
19. Enmerate the contiom whicl mut be fuIfied by a rerrerrible process. Givo soms.'qFros of ideal
rwersible procesees. In thie case, combustion of fuel takee place outsi& tlu cylndcr as in
cas of stedm engines
m. What is an irrevenible proes ? Give some emple of,ineversible pmceasas. where the heat of combugtion is enrployed to generate eteam which is used to move a pieton in a
21, Give the following statemets of se6nd law of tlerodmamiee. cylinder. Other examples of erteriial combustion engines are hot oir engines, steam turbine and
(i) Clauiwstatemat elnsed cyck gas turbine. lheee engines are generally u6ed for driving locomotives, ships, genera-
(iD lGlvin-Planck statemnl tion of electric porer tc,
a, Defne heat engine, rfrigrator and heat pump. 2. Intnol combugtion englnes (I.C. enginee)
B, What is the perpetual motion madine of the second kind ? In this case, combuston of th fuel with orygen of the ar occurs within the eylind,er of lhe
?A What do you mean by'Ttrermodynamic temrrature'? engine. the internal combustion eugines group includes enginea employing nixtures of combusti-
.
'What do you mean
2E by'Claueiue inequality' ? ble gases aDd air, known as gas ergirrs, tho* uing lighter l4uid. fueJ or apirit known as pefroJ
%. Describe tlre working ofa Caot cycle. 1
engines and' thooe using heavier liquid fuels, known as oiL cornpressDn ignitian ot disel errgines.
yt. Derive an erpressioa for the e6eienry oftle rversible heat eagine.
n What do you mean by the term'htropy' ?

l. i..
35
NTRODUCTION TO INTERNAL
COMBUSTTON ENGINES
INTERNAL COMBUSTION ENCINES
1. Starting torque is generally high' Even solid fuels can be
The detailed classifcation ofheat engines is given in Fig. 2.1. of fuel, cheaper fuels can be used.
2. Because of externaicombustion
used advantageouslY'
it is possible to have flexibility in arrangement'
3. Due to externat combustion of fuel
4.Theseunitsareself.etartingwiththeworkingfluiilwhereasincaseofinternalcombus.
is used for starting the engines.
Porcr Fodwing tion engines, *_;;;i;;"'J equipment o. d--".ri""
Ootstrityh.atngir)
I
II Air r---- RotrlgEntoB 2.2. DEVELOPMENT OF I.C'
ENGINES
I onPresr lptpumpg I'C' engines is as follows :
Brief early bistory of dweloprneot of during the
ofinternal combustion engines were built and tested
lntemel @mbGli$ Ar rnoloc ExlsrBl combGti$ a Many clifferent styles
seond half of the 19th century' apeared on the
was invented bv J'J'E' Lenoir which
ReiprocEtngtype Ryltpc Ffrip|utingtlPe Rolary t}! o The frst fairly practical enginenext ilecade, r"i"J.iu""a*d ofthese engines were built
scene about 1g60. During the
lll
I G&tB R*ip|wtng slam onglno with power tw t" mechanical efEciencv upto 6%'
"p".J;5
spa;kigniim Compsskiiffie smpl compound
r-------t-----r eTlreotto.LangenenSinewith.efficiencyimprovedtoaboutll%wasfistinroducedin
pto""" during the next decade' This was
wee
a
Uniflou
186? antl eeveral thousands ofthese
/--i-i"""""1
rt( typeofatmosple;;;st""withthepowrstrokepropelledbyatmosphericpressure
Petrol (and GaE srriE acing against a vacuum' given credit
trgire an9m3 -l St3cm on four-stroke cycle design' Otto was
k@no)
n dim Although nrany people were working
;-h"; hj; prototvpe engine was built in 1876'
, wh. Nr Ytor tl
Faifdfril In the 1880s, the
in automobiles' Also in
internal combustion engioee frrst apoeared was manufactured in
AJdal
;"""t" ptttiiol and
l*_I-
coolod @dd
| ,I
.l
Factril turbino this decade the
Iarge number.
twostroke;;;";

z-stbt<c 4-st'rctc zsrrr. 4strck


fi t"ffi ff
'l*3'.?J::i:'l#:fffi
,"18e2,haaperred"l\i:*-p':1'-'T^11t*i::"Tffi::ittrlfy"*i
ffi"i'ii#";i;r-r"n:t-t":'eropmentrcorkwhich
il.T""d"d il;Ji*rt t"r r" his earlv
experimental
:"cti:t:
o"J*""
nlvg
.,L
w
Pufo reacon Combimtlon lune
(rbst porrer ilEtion)
#ffi ;il;";;**4;t"*1y,1:o,,'!:::.::*?,IYrengines'rhe'were'
DUuevtrLP..oo
noweef, teBeru.J
.
thon sporh
l;;;r;;, generallv more effuient -"-'- -r
ettwve''
-,'' -
inition engtnext:__:,:^-^ '
"w'e were maile
iirat multicvliniler,"1-pl-"j"ioo ignition engines
o It wasn't until tbe 19zos with and trucks'
rl"ff "*U to be used automobile
at NSV' Gernany in 1957'
Beck
pfsuf a foUt"'s irst rotory engine was ested
turbine in small numbel are being produced eince 1965'
a The practical stirltry engincs
F, f. CUs"in"l6o" ofhet edins. and advanced technology for manufacture'
eBgines require costly material
Z.

- These than 307o have been obtained'


- iiti-"mci"ni"'
Advqntagee of recipi$cdifi! interiral cobutio iingines over exernal higher
emission ond' mult'fuel capability '
combus.tiod eirgines : Tbead,uaitcgeo of stirling e tgine are low exhaust
Reciprocating internal combustion egines offer th following ad,uantdges over external
-
combustion engines : 2.S. Cr,ASgrFrcATrL)-{ oF r'c' ENGTNES
1. Overall efficiencY is hign.. ag gi'en below :
Internal combus9n engines nay be classified
. 2. Greater nechanical sirnplicity. ofoperation
. 1. Aacodirg tq cycle
:
3. Weiht to power ratio is geneially low. () Two stroke rycle eugiaes
4. Generally lower initial cost.
(ii) Fo.ur stroke cYcle engines'
5. Easy starting from cold conditions.
2. Accordlng to cycle oJ gomtustion :
6. Thege units are conpa.ct and thus require less space, at constant volume)
(D Ot'to pycle gngtne (c9mbustion
Adventagee of th qternal combustion engines over internal corbustion () Diesel cyclerengine (combustioo at constant pressure'
englnee:
The external combustion engines claim the following ad.uantog* over internal combustion
engines :
INTRODUCTION TO INTERNAL COMBUSTION ENqINTJS
INTERNAL COMBUSTTON ENGINES
(iii) Dual-combustion or semi-Dieser cycle engine (combustion (du) OPPoeed qYlinder
partry at conetant vorume "gio" (a v-engine with
and partly at constant pressure). o Two banls of rylin<lers oiposite each other on a single crankshaft
3. According to rirangement of cylinder : Refer Fig. 180" v).
2.2.
o These are common on small aircraft and some automobiles
witb even number of cylin-
dere from two to eight or ore'
(u) W'engtne
oSameasV+ngineerceptwiththreebanksofcylindersonthesamecrankshaft'
Not common, but some have been developed for racing automobiles'
(ui) OPPoed Picon engine
.tnthigtypeofenginetherearetwopistonsineachcylinilerwithtbecombustioncham.
ber in tbe ctre between the pistons'
@
(i) Single cylinder () lis
rAsingleconbustioupooaSacausestwopowerstrokes,atthesamotifne,witheach
-
fuffib.,t"s p*l i*"v
to a separate crank-
from the centre anilrdevering rcwer
ghaft at each-ed of thia cYlinder'
q. th'ai&,
(uii) Redlal engine
oltisaenginewithpistonspositionedinacircularplanearoundthecentralcrank.
rod which, in turn,
master
shaft. The connecHnirods ofihe pistons are connected to a
is coDnectd to the crankshaft'
of cynders ranging
o In a radial engine the bank of cylinders always has an odd number
from 3 to 13 or more.
.opratingonafour-strokecycle,everyothercyliaderfiree.and'hasapowerstrokeae
the cranhshaft rotates, giving a smooth operation'
.Manyrnediumanillargesizepropeller-drivenaircraftuseradialengines'Forlarge
the other on
one behind
(du) Opposed cylinder (u) IVangine aircraft wo or moe banks of cylinalers are mountsd together,
a single crak;haft, rraking one powerful smooth engine'
o Very large ship engines exiet with upto 54 cylinders' six banks
of9 cylinder each'

4. According to thelr uss :


() StationaY engine (i) Portable engine
(iii) Marine engie (du) Automobile engine
(u) Aero engine etc'
5. Accordlngto the sreed ofthe engine:
(i) Low speed engine (ii) Medium speed engine
(40 High speed engine.
(ui) oppoeed piston (uii) Radial eogiDe 6. Accodhg to method of ignition :
(i) Spark-ignition engine (ii) Compression-ig-niion engine'

shaft.
(i) singre cyrinder l-';:tr"":1T::':Sfl.T::ffI.ffiT,,** ro the crank_ 7, Accordtng to nethod of cooling the cyllnder :
(i) Air-cooled engine (ii) Water+ooled engine.
(ti) In'line orstraight enginee. Cylinders
,,
other along the lngth ofthe crankehaft.
are positioned in a straight line one behind the
8. According to method of goveining:
(D Hit and miss governed engine I () Quality governed engine
(did) V-engine
o An engine with two cylinderlanks (d.e., two-inline (ii) QuantitY governed engine.
engines) inclined at an angle to
each othr and with one cankshaft. 9. Accordlng to valve srangement :
o Most ofthe bigger automobiles use the g-cylinder
v-engine (4-cylinder in-rine on each
(i) Over head valve engine () Z-head tYPe engine
side of V). (iD ?-head type engine (iu) F-head tYPe engine.

i, ..
INTERNAL COMBUSTION ENGTNES
INTRoDUCTION To INTERNAL coMBUsToN ENGINES 39
10. According to number of cylindere :
(i) Singleeylider engiae
(ij) Multi_cylinder engine. o These engines also find applications in very small electric generating sets, pumping
setg etc.
11. Accordilg to eir intake pocess :
(i) Natumlly espirdd.
No intake air pressure boost system. 2. Smsll four-stroke petrol enginee:
IntLe air pressure increased with o These engines are primarily used in automobiles,
..rro"f']rf't-arged' the compnessor driven off the engine
o These are also used in pumping sets and mobile electric generating sets.
Inake air pressure increased wirh These days diesel engines are taking them over, in the above mentioned applications.
the rurbine-compressor driven by rhe
""rr"J':']#ilur#:n
(iu) crankw 3. Four stroke diesel enginee:
. --p3ar: stroke-cycrg
ar compressor' Limited dev.elopment work engine which uses the crakcase as
he intake o The four-stroke diesel engine (a versale prime mover) ie manufactured in diameter
has alsoten done on the design and
four-stroke cycle engines utb L"rr.."""-;"il#;J construction of rangiug from 50 mm to 600 mm with speeds ranging from 100 to 4400 r.p.m., the
power delivered per cylinder varying from 1 to 10OO kW.
12. According to fuel employed :
egine
0) Oil Dieeel engine is employed for the following :
() petrol engine
(iii) Gas cngrne - Pumping sets
(du) Kerosene engine
(u) LPG engiue
(ui) Alcohol_ethyl, methyl
engine - Construction machinery
- Air compressors and drilling jigs
(uii) Duel fuel eogine
(ui) ess6.1 (90% gasoline
and LY%alcohol).
18, Method or st input for S.L engines : - Tractors
( Carbuettcd, - Jeeps, cars and taxies
( Multtptint ptt fwl i4iection
one or more iqiectors at each cyliniter
intake.
- Mobile and stationary electric generating plant
(iii) l7s. bdy
ftul i4j*tior" Iqiectors - Diesel-electriclocomotive
"n"" in itake nnifold. - Boate and ships.
2.4. APPLICATION OF I.C. ENCII\ES
4. TVo stroke dieeel engines :
The LC. engines are generally used
for : o These engines having very high power are usually employed for ship propulsion and
(i) Road vicles (e, scooter, generally have bores above 60 cm, uniflow with exhaust valves or loop scavenged.
rnotorcycle, buses etc.)
(ii) Aircraft
Erample. Nordberg, 2 strohe, I2-cylinder 8O cm bore and 155 cm strohe, diesel engine
(jdi) Ioconotives
develops 20000 hW at 120 r.p.m.
(du) c*r**oo in civil engineering equipmont such as
bull-doze, scraper, power shwels 5. R"adid piston engine in emall ai-rcraft propulsion :
(u) Pumping sets Radial four stroke petrol engines having power range from 300 kW to 4000 kW have
(ui) Cinemas been used in small aircrsfts.
(ud) Hospital o I modern large aircrafts, insted of these engines, gas turbine plant ae turboprop
(udi) engine or turbojt engine and gas turbine engines ae used.
$everal industrial applications.
T\e applintbns of varbus englws separately 2.5, ENGINE CYCIJ.EIYERGY BAIAI\TCE
are listed below :
L. SmdI ro-gtroke petrol engines
: Refer Fig. 2.3. It shows the enerry flow through the reciprocatjng engine. ihe analysis is
t vthre simpticity and. the based on the first law of thermo(ynamics which stateg that eDergt' can neither be created nor
f;;:mTr##T:::t* tsw cost of the prime ,,over are
degtroyed, it can be converted from one form to other.
t develops maximum braie power (8.p. o In an I.C. engine fuel is fed to the combustion
ff :3 :i ##: or t.o kw at 5000 r.p.m. and chamber where it burns in the presence
of air and its chemical energy is converted into heat. All this energy is not available for
o The 10o c'c' engine developing maximun
brake power ofabout 3 kw a b000 driving the piston since a portion of this energy is lost rough exhaust, coolant and
used'n xooters' The 150 c.c.-e"ct"; J;;;;;'m"*imo- r.p.rr. is
at 5000 r.p.m. brake power of about 6 kw radiation. The remaining energy is converted to power and.is called indicated energy or
o indicated. power (I.P.). The atio of this energy to the inpu fuel energy is called indi-
The 250 c'c' engine developing a maximum
brake power of about g kw at cated therral efficiency [r*.,r,J.
is genera\r used in motor cycles, 4500 r.p.m.
rt I EAL UUMUSI'ON ENCINES
INTRODUCTION TO INTERNAL COMBUSTION ENCINES
Fuel energy

Energy losses c),nder


(Exfiaust, coolant, radiation)
Grdgon *t
'I
Connoc{hg rod

Fly wheel
Eneryy losses
.(Filc-tion, p.rmping, unaccountd)

Outpd shaft Cankshaft Crankshafl


I.P, =Indicatdpower
B.P. = Brake power

(lr), = Indicated tnermal efiaiency ,.*i


= n/, x C.V.
-
CJ 2

(where , mass of fuel in kg/s and C.V. = cleifis yus


=
(no = 3t"" *"nnal efficiency.
Fig. 2.8. The energr flow through the recipmcating engine.

o The energy available at the piston passes through the connecting


- rod to the crakshaft.
In this ransmission of enerry/powr there are iorr"" u" t" ti"ii"", pr-pi"g,
etc. The Fig. 2.,1. Baric idca ofl.C. cogia
sum of all theoe losses, *"u.9+d to power, ie termed nt"r"o il-.'r. (F.p.). The
remaining enerSr is the useful mechanbar energr and""is termed. cs
brake power (B.pJ..Tte rotb of enersr,at snal n
ehft enrgy or 2.7. DIFFER^EI{I PARTS OF I.C. ENGINES
thermal efflciency [.,"r]. fu ";;;;";;;r"ud, brd*e
Here follows the detail ofthe various parta ofan iDtertrd combuatiou engine,
o fuj r.atio of slwft energt to the energr auailable ot the piston is called. mechanical A coeg-section of an air-cooled I.C. engine wittr pricipal parts ir showD in Fig. 2.5,
efficiency (q.".u).
A- Part common to both pebol and dierel engtne i
2.6. BASIC IDEA OF I.C. ENGINES 1. Cylinder 2. Cllinder head
3, Piston 4. Plston rinB
The basic idea of internal combustion engine is shown in Fig.2.4. 6. Gudgeon pin 6. Connecting rod
The cylinder which is
closed at one end is filled with a mixture offuel and air. 7. Cranksbaft 8. Crank
As the cranksiaft turns it pusrres cylinder.
The-piston is forced up and corrpresses the mixture in the 9. Engine bearing
top ofthe cylinder. The mixture is set 10, Crahcase
alight and, as it burns, it creats a.gas pessure on the pison,
forringit down the cylinder. This 11. Flywheel 12. Goveror
motion is shown by arrow'1'. The piston pushes on the
rod whichporh". oo the cank. The crank 13. Valves and valve operating mechaiens.
is given rotary (turning) motion as shown by the anow ,2,.
The ny wheel fitted on the end of the B. Parts for petrol enginee only :
crankshaft stores energy and keeps the crank turning steadily.
1. Spark plugr 2, Caburettor
3. Fuel pump.
.t

INTERNAL COMBUSTION ENGINES


INTRODUCTION TO INTERNAL COMBUSTION ENGINES
C. Parts for Diesel engine only :
1. Fuel pump.
2. Cylinder head
2. \iector.

Inlet valve EdEust vah Admisson


Vahn gring
J

E)aust port
Inlet port
C!nder head

Coolng fins

Pston

Wrbt fin

Cylinder

Crankcase

Fig. 2.6. Air-cooled cyliader. g. 2.7. Water+ooled cylinder.

Grank pin One end of the cylinder is closed by means-of a removlc eylindzr lwad. (FiS, 2.6) which
Crankshaft usually contaius the inlet or admission valve [Fig. 2.8 (aI for adnitting the rnixture of air and

Crank Margin Exhaust vah,

. -Fig.2.6.Air-cooledl.C.eogino.

A. Part_e cornmon to both pe&.ol and dieael e-l,-es :


I Cytir\der
The cylinder contains gas under pressure and guides the piston.
It is in direct contact wih
the products of combustion and it must be cooled. Te irleal form cousists of a plaiu cyndrical
barrel in which the piston slides. The mov-ement of tbe piston
or shoke being in most cases, longer
than the bore' This is known as tlae "stroke bore ratio'. I\e
upper end of a combustion or
clearance space in which the ignition and combustion of "oori"t.
the charge takes place. In practice, it is
necessary to depart fom the ideal hemispherical slope
in order to accomrroate t1e valves, spark-
ing plugs etc. and to control the combustion. sectiong of
an ai+oolJ
clinder are shown in Fig. 2.6 and 2.? respectively. The exlnder is tl;d*;;" water_cooled
nid.e of hard, grad,e c,,st iron
and, is usually, cast in one piece.

(a) Inlet valve (b) Exhaust valve

Fig.2.8
INTRODUCTION TO INTERNAL COMBUSTION BNGINES 45
fyel a{ exhaust valve [Fig.2.8 ()l for discharging the product ofcrnbuetion. Two valves are kept
closed, by means ofcams (Fig. 2.9) geared to he engine shft. The passage in the cylinder
hed split at one point so that they can be expanded and slipped orer the end ofthe piston and into ring
leading to and from the valves are called-pors. The pipes whic.h connecf the inlet ports of the grooves which have been cut in the piston. When the piston is installed in the cylinder the rings
varioue cylinders to a commo intake pipe for the engine is called l}e inlet manifold. If the exhaust
ports are sirnilarly connected to a common exhaust system, this syetem of piping is called
exhiust
manifold,. Compresson
rlngs

Ol control rng

Fig. 2.9. Canr and rocks arm. Slotted skrt


Fig. 2.10. Section tluough a splitshirtpistm.
. fll man purpose of the cylnder hea.d is to seat the worhq ends of ttu giirers and. not
to permit entry and, exit of gases on couer had. voJue ngircs. ttJngie cavity f head ie are compressed into ring grooves which have been cut in tbe piston- Wheu the piston is installed in
called
th.e combuson chamber, into which the mixtue is compressed for ring. Its-slwpe
controls the
tbe cylinder, the rings are cornpressed into the riug grooves sb that the split ends come almost
d.irection a.nd rate of combustion. Heads are drilled and tapped with coirect tlrea to take together. The rings frt tightly against tbe cylinder wall and against the sides ofthe ring grooves in
the
ignition spark plug. AII the combustion chanbers in an engine must be ofsane shape and size. the piston. Thus, ey form a good. seal between the piston atd thc cXlindzr walL TTle ringe can
Ttre shape rnay be in part controlled by the pison shape. expand or contact as they heat aod cool and still rrake a good deal. Thus they are free to slide up
The cylinder hcad. is usually mad.e of cost iron or aluminium. and down the cylinder wall.
3. Piston
A piston is frtted to each cylinder as a face to receive gas prfes{ure and tranemit the thrust
to the connecting rod.
The piston must () give gas tight seal to the cylinder through bore, (ii) slicle freely, (iii) be
light and (iu) be strong. The thrust on the piston on te power stroke tries'to tilt the piston as the
connecting rod swings, side ways. The piston wal\ called the skirt nust be strong enough to
etand
upto this side thrust. Pistons are made of cast iron or dlumium altcx
for leitness. Light alloy
pistons expand more than cast iron oe therefore they need large clearances blhe bore,
wen cold,
or.ne9i_al provision for expansion. Pistons may be solid gkirt or spt skit. A section
through a
split skirt piston is slre "n in Fig. 2.10.
4. Pieton ringa
The piston must be a fairly loose fit in the cylinder. If it were a tight t, it would expand as
-t+t
t-
Ir-
it _got bot and might stick tight in the cylinder. If a piston stick* it coul ruiu te engine.-On the I
other hand, if there is oo much clearance between the piston and cylinder walls, much of the
P-ress]re
from the buming gasolinc vapour will leak past the piston. This means, tht te push on
the piston will be much less effective. It is the push on the pis-ton that delivers te power om
the Pislon groove
engines.

. To provide a good sealing fit between the piston and cylinder, pistons are equipped with Fig. 2.11. Workingof a piston ring.
piston ings, as shown in Fig..2.10. The riags are usually
mad of casi iron of fine grain and high
elasticity which is not affected by the working heat. Sorne rings are ofalloy spring iteel. They ie FtE. 2.77 shows how the piston ring works to hold in the cornpreseion and combustion
pressure. The anows show the pressure above the piston passing through clearance between the
f .. :"

46 NTERNAL coMBusrroN BNcrNBs tNTRoDUcrtoN To NTERNAL coMBusrtoN ENcrNEs 41


piston and the cylinder wall. It presses down against the top and
against the back of the piston The connectiug rods are made of ickle, chrome and chrone vanum steels, For small
rings as shown by the arows. lhis pushes the piston ring firmly agiust the bottom of the
ring groove' As a resul there are good seals at both ofhese points.-The higher the pressure liston
in the
engihes the material may be aluminium.
combustion chamber, the better the seal. ?, Crank
Small wo stroke cycle engines,have two rings on the piston. Both ae compressioli rings The pieton moves uP and down in tbe cylinder. This up ad dowa notion is called recdpro-
(Fig. 2'12)a Two ringe are used to divide up the job of holdini the compression
and combustion catry notbn- The piston movee in a straight line. lte straight line motioa must bo changed to
pressure. This produces better sealing with less ring pressure against te cynder
wall. rotaty, or turning motion, in mot machines, before it can do any good. Tha is rot8ry motiou is
required to make wheels turn, a cutting blade apin or a pulley rotte. To change the reciprocating

:4V
motion to rotarr motion a crak and connecting rod ae used. Oigs. 2.18 and 2,16). The connectin!
rod connects the piston to the crak

Fig. 2.12. Compressioa ring. Fig. 2.13. Oit ring.

. Folr stroke cycle engines haue on ertrd, ring, cailed. the o contror ng (Fig. 2.18). Four
stroke cycle engines ae so consructed that they pt much more oil i the wall than do
two strole cycle engines. This additional oil must be scraped offto prevent it"yn"r
from getting up into
the combustion cho-ber, where it would burn and cauee trouble.
- Refer Figs. 2.12 aod 2.13, the compression rings have a rectiliiear crose-section and oil
fnss ar9 provided with a groove in the middle and wiih through holea spaced at certain interval Pislon
from each other. The oil collectedfrom r,he cylinder walls flows lhrough these holes into the piston
groove whence through the holes in the body oftbe piston and down its Piston pin
inner walls into the engine
crankcase,
5, Gudgeon pin (or wrb pin or piston pin) Connecling
Thede ae ha'rdend steel parallel spindles fitted through the piston bosses and the small
end bushes or eyes to allow the counecting iods to swivel. Gudgon pins are a prees fit in the piston
bosses of light alloy pistons when cold. For removal or ficing] tL'pisto" Gonnecting rod
dipped in hot
be
\Pater or hot oil, this erpaads the bosses and the pins can be removed "n"d
or fftted freiy without
damage. Crank pin Rod c@ bolt
It is made hollow for lightness since it is a reciprocating part, Cank Crank
6. Connecting rod Rod cap
Refer Ftg. 2.L4. The connecting rod transmits the pieton load to the crank, causing the Crankshaft
-latter to turn, thue converting
the reciprocating motion ofihe piston into a rotary motion ofhe Grank cheek
crankshaft. The lower or.obig end" of the conneiting rod turns on "crdnk pins", Crnkshalt

Rod- bearing cp

Fig.2.15 Fis.2.16
Ne lbe crak end oftle cosnecthg od ie caled rod tig nd'. Ttre piston-ead ofttrre connecting rod is
called the rod'mcd nd',

itffli;i;il* of the crakshan. The crankshaft of an inrernal combueriou engine


receives viaits sranks the efrorts srrpped by the pietons o the connecting rods. All the engines
auxiary necbanigms with mecha[ia tmsmigsion are geared in one way or the another to he
cranlsbaft. It i8.us,nlu a steel forying, but some makers use special grpes of cast iron cuch es
spltcroidal graphiti or nichel allaX castings which are cheaper to produce and have good senrice

Fig. 2. 14. Connecting rod:


life' Refer Fig. 2,L7. The crankshaft c3nyerts
the reciprocating motion to rotary motion.
shaft mounts in bearings which, encircre trreo""na'J The crank
it can rotate freely.

Inne race

NN
Connecting
rod

Bearing in rod f,,*ffiffiil"


iCrank in
Rotary motion
of crankshaft .Sleeve sleane
Split Batl beadng Roller bearing
barng beginO
C

, Fig.2.l9.Bearings.
Main journal
The sliding type of bearings ae sometitres called usings or cl,eeue bedrings because they
Fig. 2.1?. Crank ehaft and oberparts. are in !!e ahape of a sleeve that fits amund the rotating joumal or shaft. The sleore.type connect-
The shap of the crenkshaft i.e. the mutual ig rod tig end bearings usually called simply rod bearings and the crankshaft supporting bear-
ar:rangemeut of the cranks depend on ieE cdled the main bearings are ofthe split sleeve type. They nust be split in oider to permit
nunber and arrangement the
thst"rJc order of the engine. Fig. 2.18 ehows heir aeeembly i1!o the eo-gine. In the rod bearing, te pper halfofthe beariug is installed in the
typical crarkshaft layout f9i1fa{indep-.Td
four cyii{gr a
rod' the lower half is instaod in the rod beai'ing cap. When the rod cap is fastened the rod shown
""*i";:--' in Fig. 2.16 a complete sleeve bearing is formed. f,-ite*ise, ttle upper halves of the main bearings
are assembled in the engine and then the main bearing caps, with the lower beriring halves are
o attached to the engine to complete the sleeve bearings supporting the c.nkhaft.
/q
I
typical bearing half is nade of sCeel .or bmnze
I
-bearingThe
rraterial is applied. Refe Fig.
back to whih a lining of relatively soft
I
// |
2.20. This rlatively soft bearing material, which ie made of

:-J I
'.o- sweral naterials such ae copper, lead, tin and other metals, has thJability to conform to slight

\l // I
6
irregularities ofthe ehaft rotating agaiust it. Ifwear does take place, it is the beariug that wears
ad the bearing can be replaced instead ofmuch more erransive crankshaft or other engine part.

-l / Inner layer of
E o Steel back bearing alloy
2- -
g
Thin overlay of a sotl
FiC. bearing alloy
2. 18. tpical cranbhaft layou
9. Engine bearing
the crankshaft is supported by bearing. The
connecting rod big end is attached
pin on he crank ofthe crankshaft by to the crank
a be."r"g. e pi"t"" pin at the rod
the rod to the piston' The piston pin rides emall end.is used to atach
i" l";;;-.-E ery where there is rotary acion
engine, bearings are used ro supporr the in the
moving p"rr..-Th;:p;;;..lio"JJrTrrro
rhe parrs ro move e"s'v. rearinln-are rubricatd reduce rhe
::Tr'"T**iow *ith rhe relative
"li;;L
Bearings used in engines are of two typea
: slidhry or rcUi.ng(Fi& Z.1g),

Barrlr platlng

Fig. 2.20. Bearing hatf (details).


.t

50
INTBRNAL COMBUSTION ENGINES
INTRODUCTION TO INTBRNAL COMBUSTON ENCNES 5I
' llhe rolling'type bearing uses balls or rollers between the atationary support
and the rotat-
ing shaft' Refers Fig. 2.19. since the balls or rollers provide rolliry The weight of the flywheel dependa upon the nature of variation of the pessrre, The fly-
contact, i-" tl"UJ*ri"i- wheel for a double-acting steam engine is lighter than that ofa single-acting one. Similarl the
ance to movement is much less. In some roller bearing, the
rotlarg are eo snall tha they are flywheel for a two-stroke cycle engine is ghter tha a flywheel used for a four-stroke cycle engine.
hardly bigger than needles. These bearings are called rcedh bu,rhgs.
Also eome rollers bearingr Lighter flywheels are used, fur ml-eXlhder engines.
have the rollers set at an angle to the races, the rollers roll
in ae tapered. These bearinge are
called tapered' roller bearings. Some ball and roller bearingr 12. Governor
are sealed with their lubricat al-
ready in place. Such bearings require no other lubrication. A governor may be defined as a devbe for regulating outomatirally output of a mazhittc by
Othe do require lubrication from the
oil in the gasoline (two stroke cycle engines) or &om the en$ne lbrition reguloting the supply of working fluid,. Ylhen the apeed decreases due to increase in load the
system (four stroke
cycle engines). supply valve is opened by mechanism operated by the gov.ernor and the engine therefore speeds up
again to its original speed, Ifttre speed increases due to a decrease ofload the governor mgslnis
. The type of bearing selected by the designers of the engioe depends on the design of the
engine and the use to which the engine will be p7t. eeicratty, closes the supply valve sufficieutly to elow te engine to ita original speed. ?hus the functon of a
,t"n uiring", being ress apensiue gouernor s to control the lultuotiaw of engine sped duz to chonges of load..
and satsfactory for most engine applcetions,-ane used. In
foct sbeue beiinge ire used atmost
unuersally in outomobile engines. But you wiil Comperison btween q Flpiheel and a Governor
find. sone engcs with futt -u";;;;;;;;
bearings."d
support the cmnhslnft and for tlw connecting rod and pistin_pin
10. Crankcase Flywluel Govemor

The main body of the engine to which the cylinders ae attached and
crankshaft and crankshaft beariug ie called c.ulnhcase. This member
which contains the 1. Itis provided on engine and fabricatingmachines It is provided on prime movers sud as engines
also holds other parts in uz., mlling mills, puning nac.binee ; shear ma- andtubines.
alignment and resists the erplosion and inertia foces. It also pmtects ines, presseo etc.
the parts ftom dlrt etc. and
serves as a part of lubricating system.
2. Its function iB to store the availqbh mochnical Its function is to rgulate the aupply ofdriviag
11. Flywheel energywhen it is in eceas ofthe loadrequirement fluid producing energy, according to the load
andto partwith the eomewhenthe available energr requinenent so that at different loatl almmt a
- Refer,Figs,
performs
2.4 and 2.21. A flywheel (steel or cast iron rlisc) eecued on the
the ftllowing functinia :
crank ehaft i less than that requiredbythe load- conatant speed is maintained.

(o) Brings the mechritsm out of dead cedtrs. 3. It woks cootbuously ftom qrcle to cycle. It works btermittently ie. only whea ere is
erhange inload.
() stores enerry required to rotate the shaft
during preparatory strckes. 4. In engines it takes ce off,uctuetions ofspeed It takes care offluctuaHons ofspeed due to vari-
(c) Makee crankshaft rotaion more uniform.
during thermodynanic cycle. ation of load over long range of working engines
(d) Facilitates the starting.of the engine aidtubines.
and bvercoming ofshoft tibe over loads as, for
example, when the machine is started from rest. 5. In fabrication madrines it ia very economical to use But fm governor, there would have beeD rmec-
it in that it reduces capital investrrent ol prime eerarilynore consumption of drivingfluid- Thu
movers ad the rrnniqg e1!e.e8. it economie.e its consumpon"

Connctlno rod $pes of govrnor:


Governors are claseified as follows :

1. Centrlfugal governor
(i) Grouity controlled, in which te centfugal force due to the revolving masses is largely
balanced by gravity.
(ii) Spring controll,ed,, in which the centrifugal force is largely balanced by springs.
.2 Inertia and flywheel governors -
(i) Centrifugal type, in which cenbifugal forc* play the major part in the regulating action.
(ii) Inertia govenror, in which the inetia effect predominates.
Tlte inertia type governors are fitted to the crnlrn[sft s flywleel ofan engine arrd so differ
radically in appeaance from the centrifugal govemors. lbe balls are so arranged that the inertia
forse caused by an argular acceleration or retardation ofthe shaft tends o alter their positions.
Fig. 2.21. Flywheel secured on mankshaft_ The amount ofdisplacement ofgovernqr balls is controlled by suitable springs and through the
govemor mechanism, alter the fuel supply to tho engine. The inertia governor is more sensitive
than centrifugal but it becones very dificult to balance the revolving parts. For this reason cn-
trifugal gouernors are rrlore frequently used, We shall discuss centrifugal governors only.
INTERNAL COMBUSTiON BNCTNES
li Important centrifugal governors are :
1. Watt governor
2. Porter governor
3. Proell governor 4. Hatnell governor.' Upper arms
1. Watt governor
It is the primitive govern_or_as
used by watt on some of his early stearn
for a very slow speed engine and this is why it engin$. It is used
* ."0-e obsolete.
Refer Fig. 2.22. T\o,arms are^hinged at the "o*
rop of the spindre and two revolvingbans
fitted on the other ends of the ."mr. on"io or""i-tr-"ri.rts arc
the other ends are hinged with the ste"ve, whi"n are hinged with the arms, whe
crankshaft is transmitted to the spindre- til"ough -"^"riau o"er the spindre. The speed of tbe
arrangement' so the rotation ofthe spindle ofth "-pi. "r bevel gears by rreans of a suitable
governor causes the weights to move Lowgr arms
the centre due to the centrifugal foqce. rtir away from
movement of the sleve is tra-nsmitted bv the -rte"iiuil"e*e to mou" io the upward direction. rhis
tever to the thotile
closes or
opens rhe steam pipe and reduces or inieases
-- th" ;;;ry ;
--- *-r "rrr"-;;h;;jay
:;,;;; 'ao the engirie
i speed may be adusted to a normal limit. ";;;;;h"

Fig. 2.2i1. Porter governor.


3. Proell governor
Refer Fig. 2.24,.ft is a modification ofporter governor. The governor balls are carried on an
e*tensionofthllower arms. For given value ofweight ofthe ball, weight ofthe sleeve and height

Fig. 2.22. Watt govemor.

2. Porter governor
Fig' 2'28 shows diagrammatically a porter governor
governor balls rotate about the axis of te governoJJait where two or more masses caned the
which is driven ti"""gh ;it.ure gearing
from the engine crankshaft. The go"urnoi "it.-..l"itt."n"a
attached io the sreeue which acts as a central"iia.r to the arms. Tie lower anns are
the speed of the rotation of th balls
increases owing to a decrease, of lo"d o" tt"
moves upwards thus closine,the.ll"l
;;;;",?t u'gou""rroi'"1i, 'r,, **""a]";"d the sreeve
titl tr ensi'ne sp"ed .oo,"s i""r.l"ltJ".isr"d speed.
Ifthe engine speed decreas owing li"*t" to an increase ofl"oad, the governor bas
sleeve moves downwards th_s one.nrng th" fly inwards and the
d"l p; morg for oil till the engine speed comes
sp-eed. The engine L said to be runn'ing
ll*,? trlqed force is just
lt-" centrifugar
rnertra or
at its designed speed when the outward
"^."' 2.24. Proell governor
balanced by the inward contro[irfi i;;;-* F ig.

{.; 'i
f
ai

54 INTERNAL coMBusrroN ENcrNEs TNTRODUCTION TO INTBRNAL COMBUSTION ENCINES


of the governo, a Proell go:"To:."o-tt. at,a rauc a porter governo. In ordcr to gue
the same equilibrium speed. a ball of smaller
tha open inwards, the pressure in the cylinder helps to keep them closed. The valves are lifted from
b used in pril g;irrrr.
^o'rrr.,l their geats and the ports opened iither by cans having projecting portion designed to give the
4. I{artriell governor period ofopening required or by eccentrics operating through link-work. Ofthese t'wo methods the
The Hartnell govemor ie a spring loaded goveroor in which the cam gear is more commonly used, but in either+ase it is necessary that the valve gear shaft of an
controlling force, to a great
extent, is provided by the spring thrust. engine should rotate but once frorn beginoing to end of a complete cycle, however many sttokes
Fig' 2'25 shows one of the types of Hartnell gneenor. It consists of casing xed may be involved in the completion of tha cycle. This is necessary to secure a coninuous regulation
to the
spindle. A compressed spring is.placed inside the casinlwich presses
agri*t th" t";;th;;;; of the valve gear as required. For this purpose the cams or eccentrics of four-stroke engines are
and on adustable collars. The sleeve can move up j6y on the vertical spindle <tepending mounted on shafts driven by geariug at half the speed of the crankshaft. The curves used for the
"rd upon
the speed of the governor. Governor balls are caried on bell crank le.,er which are pivoted on the acting faces ofthe came depend on the speed ofthe engine and rapity ofvalve opening desired.
lower end ofthe casing' The balls will fly outwards o inrardg Ftg. 2.26 shows a valve gear for LC. engine. It consists of poppet valve, the steam bushing
as the speed ofthe lovernor shaft
increases or decreases respectively. or grride, valve spring, spring retainer, fter or push rod, camshaft and half speed gear for a four-

Valve face

Valve bushing

Valve spring

Bell crank
Sprng retainer

Valve stem

Valve lifter

Lifter gude

Bwd gears
Cam
Camshaft
geqf
,Camshaft
driv6 gear
Fig.2.ZE. Harhellgb<nenor.
,
5. Valvee and valve gears
With few exceptions the inlet and exhaust ofinternal combustion
enlipes are controlled by
poppet valves. These valves are held to their seatiug
by strong springs, *i\trr" usually
"*r.res\ Fig, 2.26. Valve gear for I.C. engine.

\\
il
1l
INTERNAL COMBUSTTON, ENGINBS
, INTRODUCTION TO INTERNAL COMBUSTON ENCINES 57
I stroke engine' The poppet valve, in spite of its shortcomings
of noise and diculties of cooling is
commonly used due to ils simpciy and capacity for effective
seting under all operating conditions. shaft rotation before T.D,C. on the exhaust stroke and stays open until 50' of crankshaft rotation
The valve is subjected.to very treavy duty. tt hoUs in combustion
Ih"rU"" an is erpaea to higl after B.D.C. on the compression stroke. The exhaust valve opens 50e before B.D.C. on the power
temperatures of burning gases. Eaust vqlve .itself may
atain temperature while external stroke and stays open 15" after T.D.C. on the inlet shoke. This iives the two valves an overlap of
cooling is not available. Special. heat resisting alloys
th"""for" used in the conshurtion of the 30' at the ind ofexhaust stroke and beginning ofthe contpressian stroke.
"""
exhaust valve and it may sometines have a hollowtnstrucion
for heat dissipaion. The salt becone liquid when
lled with mineal salts to provide
is woring and transfer hea from the
\,'l
head to the stem from which it ie caniedlhrough the "al"e
stem guide to the cylincler bloc.
T"^:11:f :L:TI^1:T1:.1h"tu openin^g ard closinglvfth espect to the rravel of
the pioron !e1-.5/
H,J":,-';:*:T:H{:jg:rj::1Yt"F;;cil;:rffi ;tr"';il:ffi ;H:il:; Intak valv -\. i---
3:l*"-"-:l'-::*y:i,r*:.'.rca$gqinrgeai""irJ"cJJ"id.'i;;;ffi:
:i"*"H-T^di:^1","*:::11,1r"-Tlist"bidr;**;d;i#;"[:::'ff
therefore, to avoid use ofmurtiple gears ottog
ihainJi" trr" camshaft drive, 'lihlff
Valve timlag
lheoreticay.th9 valveS open and close a top dead
centrc (T.D.C.)
(B'D'c') but practically they do so some une or t botton dead cenhe
uerorJ or aner the piston reaches the urrer or lower
limit of travel' There ie e reason for this. Iok
a the ilet valve, for example. It nonnay opens
several degrees of crekhaft-rotaion before T.D.c.
on the eaust atroke. That is the intake
valve begins to open before th erauat stroke ie Gished,
Ihis gives the valve enogh time to
reach the fully opeu position before the itake stroke begins. Iteq,
when the intake sboke starts,
he.intake valve is arready wide open and air tuel
ately' Likewise the itake valve-emain open for 'ril;-;r,,;;'"
" u"rila*, inndi-
quite a few degrees of crankshaft ,.aation after
the piston has pasged B.D.c. at the entl of the intake
shoke. This allowa additional time for ai fuel
mixture to continue to flow into the cylinder. The fact
that the pietou has .r.Jy p"*"a r.o.c.
and is moving up or the compression stroke while
the intake valve is *ill openis not effect the
movernent of air fuel mixtre into the cylinder. Actually
air fuel mixure is still flowing in a.s the
intake valve starts to cloee.
This is due to the fact that air-fuel mixure has inertia. I
That is, i attempts
-ri" to keep on t_
flowing after it once starts through the carburetor
." i"to tbe engine ir"r.
ofthe rnixture then keeps it flowing into the cyldereven -o"r"rrro*
so"- i
_\+L==_- -5v
thoug the piston has started up on the
compression stroke' This pachs more ai-fuel mixture I
into the cyliader and results ia a stronger
power stroke. In other words, this impmves
volumetri efficiency.
Fig. 2,27. Tlpical valve timing diagrarl
For a some what simila reason, the exhaust valve
opens well before the piston reaches
B'D'c' on the power stroke: A" piston nears B.DJ., most of the pueh on the piston B. Parts oornmon to petrol engine only :
.g"
and nothing is lost by opening the has ended
eaust valve towaids tbe eud of the power sroke. rbis gives Spark-ptug
the exhaust gases additional tim to etart leaving tie
eylinder so that .ort is *"u uy The main fuuction ofa spark-plug is to conduct the high potential from the igrrition system
the time the piston passes B.D.c. and starts upln "t"rt"athen into the combustion chamber. It prodes the pmper gap across which spark is produced by apply-
the exhaust stroke. fire exhaust valve
starts pening for some degrees of crankshaft rotatiou ing high voltage, to ignite tbe combustion chamber.
after the piston il ;;; T.D.c. and
intake stroke has started. This makes good uee ofmomentum
6ferhs! gases. They are moving A spark-plug entails the following requirements :
rapidly towards the exhaust port, and leaving the
exhaust varve open for a few degreer after the (i) It must withstand peak pressures up to atleast 55 bar.
intake sroke starts giving the exhaust g"r ,o-"
additional time to leave the cytinder. This () It must provide suitable insulation between two electrodes to prevent short circuiting.
allows more air-fuel mixtue to enter on the intake
stroke so that the stronger po*er strote (iii) It must be capable ofwithstanding high temperatures to the tune of 2000"C to 2500'C
results. That is, it improves volumetric efficiency.
over long periods of operation.
The actual timing ofthe varves varies with different
four stroke cycle engines, but the (iu) It must offer maximum resistance to erosion burning away of the spark points irrespec-
typical example for an engine is shown in Fig. 2.27.
Note that the inlet ,rulJ" op"rr.- 15. of crank- tive of the nature of fuel used.
ftf
ill
IJ

liss (u) ft
INTERNAL COMBUSTION ENGINES INTRODUCTION TO INTERNAL COMBUSTON ENGINES 59

|.lt
d,l must possess a high heat resistnsg so that ths Iectrodes do nt become
sufficieogy r Opetatlng Heat Renge :
hot to cause the preigaitiou of tbe chage wit\irr the engine cylinder
il A spark-plug heat range i8 a measure of the plug's ability to transfer heat from the
(ui) Ttre insulating material rrust witstand
satishctorily the chemical reaction effects of - central electrode ad insulator nose to the cylinder-head and cooling system.
ll, the fuel and hot products of combusioq.
(u;i) Gas ightjoints between the insulaor and When the heat abeorbd by the plug'a central electrode ad insulator nose exceeds
lr ii
metal parts are essential under alr operating - the capability of the plug to dissipate this heat i the same time, theu he plug will
conolf,rons.
ouerhedt and the certral electrode temperature will rise above its safe operating
limit of about 900 to 960'c. Abooe tlw plug't upper worhng temperature-limit, tIle
il Tsrminal
central electrod will glow atLd igtt e the air-fwl miure before the md. epcrg
actuolly occurs. this cotion is hnoun as auto-.lgnlttn as i\ dutomaticall!
starts the combustion process ndependzntb of t e clntrolhd igniian sparl. The

danger of this occing is in the fact that it rnay take place relatively early in the
compression siok6. Consoquently, the pressuregeuerated in the particular cy-lin-
ill der suffering from auto.ignition will oppose the uprard movement of the piron.
Excessive mechanical stressed will be irroduced in the reciprocating and mtating
lil
components and an abnormal rise in the cylinder temperature would, if allowed to
continue, danage the engine.
If the plug's ability to transfer heat away from the central electrode and insulator
I - tip meed,s that of the input heat from combustiori, over the sarne tine span, hen
the plug's central electrode and insulaor noge would operate at such a low tem-
li peftiture as to permit thc formatian of carbon deposits around the central tase of
ril the plug,\\is crtical lower temperaiure region is usually betueen 550"C and,
400'C lrod, at temperatures below this, carbon or oil-tleposits will fout the ncula-
ioz, creating conductiug shuirts to the inside of the metal casing of the plug.
Consequeritly, if deposits are permitted. to form, a proportion of the ignition spark
f
Shell . energy will bypase the plug gap so that there wiil bp insufficient energy left to
Gasket
anize the electrode wth thc result that misfiring will result. Establishing a heat.
balance between the plug's input and output heat flow, so that the p&4/s tempera-
ture remains just in etcess of 400"C, proai.dcs.o self cleaning action on both the
Insulator
surfaces of the electrod.es and insulator.
Centre electrode
A good spark-plug design tries to natch he heat flowing from the plug to the heat
SidE electrode
- flowing into it, caused by combustion under all working conditions, so that the plug
Fig.2.28. Spark-plug. operates below the upper temperture limit at full load, but never drops below the
lower limit when idling or runniag under light-load conditions.
Refer Fig' 2.28. The spark-plug consists ofa metal shell having
two electrodes which are
insulated from each other with an air gap. High tension currentjumping o Firing Voltage :
from the supply erectrode
produces the aecessary spark. Plugs are sometines A certain minimum ooltage ie necessa4r to make the spark jump he electrode air gap,
identifieit by tiie ieat range or the relative
temperature obtained during-operation. The correct type ofplug with the actual magnitude of the voltage required will depend upon the following fccfore :
correct widt:h ofgap between
the electrodes are imporant factors. The spark-plug gp (i) Compression pressure () Mixtue strength
cao beeasily checked by means ofa feeler
gauge and set as per manufacturers specifications.
It is most inportant that while adjusting the (ii Electrode gap (iu) Electrode tip temperature.
spark plug it is the outer eathed electrode i.e., tip which is moved
in or out gradqally for proper r Tighnese of Spark plug :
setting ofthe gap. No bending force should be applied on the centre-electrode
fr adjusting the gap
as this can cause crack and fracture ofinsuration and Tte seatjoint tightness is essential for good heat diasipation.
the plug may become absoluery useress. -
Porcelain is commonly used as insulating material in spark-plugs, Spark-plugs should not { over tightened otherwise the plug uretal casing may
manufacture. Mica can also be used as insulating material fo.
as it is cheap and easy to - become distorted, causingihe centrat electrode insulator to break its seal and be-
splrk-ugs. Mica, however, can-
not withstand high temperatures successfullv. come looge. comb,ustbn galt ma! then eecape through the plvg with the result that
it overheats.
60
INTERNAL COMBUSTON
-'*-*-'F-
I ENGINES
INTRODUCTTON TO INTERNAL COMBUSTTON
- ff;:dlfjchtned
plug may work itsetf loos and cause
combustio
ENGNES
l
wit resutt-'prus
rr ie
* **tor;;:,::rH'
and cvlinder-head oluq.ho.le
tu*'* i"'J*"rplltf.?*T:11:
no,.
The arrangement is such that when the oil reaches particular
inlet passage and thus cute offthe fuel oil supply. on
a level the float valve M bldcks the
the fall ofoil level, the float descends down,
":;;J"ff:4-j'u
Simpte carburettor.
consequently intake passage opens and again the
chamber is filled with oil. Then the float and the
float valve maintains a constant fuel oil lwel in the
float chamber. JVis the jet fron which the fuel
* ,, ,I!"7!rT"T:o:["i;:"*:ttor ie to atomise and metre.l! th:ilny_t mis it with the air
is sprayed into he ai stream as it enters the carbuetto
throa or venturi I' The fuel level. is sghtly
at the inlt ,g and passes tJrrough the
r""t-""'l",i#,t;:;:":;;:;l:Jf;l#T #;#.,"*t",,a "d;,'';;,:;;""
"nd
of operation below he outlet of the jet when the carbuettor is
All modern -t*U,s inoperative,
carburettors are based rpoo
heorem, As the piston
-"*, o"*" t rffiffi;;., sucrion is produced iu he
cylinder as well
asin the induction manifold e as a result ofwhich
air flows through tbe CarburettOr. Thevelocity
il*fi *';?.ff1":ik?"""*ff etre,secad,rigrhe ofair increases as it passes through the constuction at
the venturi g and pressure decreases due
fiTheI{iH'"rY"}ff
equation of mass rate of "' - ffiry;ffi to conversion ofa portion o{pressure hesd into
kinetic enerr. Due to dcreased proasure at the
clr*i;,
flow is venturi and hence by rtue ofdifference in pressure (between
the float chmber ad,the venturi)
the jet issues fuel oil into air strem. since
where p is the densitv
of the fluid ^=PIJE
t" ,rJ-"1""-sectioal aea
the jet has a very fine bore, the oil issuing from the jet
is in the form of fne spray ; it vapourises quickly
rnFig' 2'29 is shown simpre ""0, of fluid strean. ancl mixes with the air. This air fuel mixture
caturetto..i ir"'nort enters the engine crlinder ; its quantity being
tuel supplied under gravity
action or , n *artr q the srorage of tuer. The
"io"_-o-"#j=o the floa hmher throug the filter n.
valve ?.
controlle uy
"""yl"g
in"-;il" of the tbrottle
I
Linitationa :
(i) Although theoretically the air
fuel ratio supplied by a simple (single jet) carbuettor
should remain constant as the throtle goes
on opening, actually it provides increasingly
I richer mixture as the throttle is opened. This
.air tends to decrease is because ofth" i""roo that the density of
as the rate offlow increases.
() During idling, however, the
nearly closed throttle causes a reduction in the mass
flowing through the venturi. At such low rates of air
of air flow, the pressure difference be-
tween the float chmber and the fuel disarge
nozzle becomes.very rmall. It is suffi-
cient to cause fuel to flow through the jet,
(i) carburettor does not have
arrangement for providing rich mixture during
warm up. atarting and
In order to corect for faults
(i) number of compensating devices
are used for (ij) an idling jet is used whicl helps in
running the engileduriag idng. For (i) oke arrangement is used.
FueI pump (for carburettor-petrol engine).
Refer Fig' 2'30' This type of pnmp is used in petrol
engine for supply of fuel to the caburet-
tor' Due to rotation ofthe crankshaft th"."m poshe,
the lever in the upward direction. one end of
the lwer is hinged while the other end putts
th" iprrr"g rd wjth the dizphragm. so the
.diaphragm comes in the downward direction against the compression of the spring and
vacuum is produced in the punp chamber. thus a
This causes the fuel to ente into . p,r-p chamber
from the glasc oul through the strainer
arxl the inletlake, the impurities of the fuel
any' deposit at the bottom ofthe grass bowl. ; if there is
on the return etroke the spring pushes the diaphragm
Fig. 2.29. Simple carurettor. direction rorcing the tuel rrom trre p.rmp
chamber into lhe carbure*or through rhe
n:I::X::
62
INTERNAL COMBUSTION ENGNES TNTRODUCTION TO INTERNAL COMBUSTION ENGTNES

Stralner
Passag (P)

sFing (S)

Barl (B)

(Y) Supply port splll port (SP)


Plungsr (L)
Bad( (B)
Fig. 2.30. nel pump for cartuattc,
Prts for Dleeel eaglne ouly :
T"UDL PI'IUP
Refer Fig' 2.31- L ia the plunger which is iLiven by a carn and taryet
mechanism at ths
bottor (not sbbrra b ttre gure) B is the.barrel i whicbthe piuager;;;;r. rbere ig the
rectangular vrticl groove in the ilunger which extend om top
to auoter helical groove. V ie
the devery valve which lift otrits,ieat Dder the quid fuel pressure
and against the spring force
(s). The fuel pn;p ia connected to fuel atorriser
thi;ugh the pasrage p, sp yare tue epill an
supply ports rspectively' When the plungr is at its botton
stroke the ports sp a" y a"e
ered (as ghown in the Fig. 2.31) and oil ftou low pressure prirnp (not "J-ie
shown) after being filtered
forced into the ba.rel. wheu the plunger moverup due to
cnm an<t tappet mectranism, a stage
reacheg when both the ports SP and Y ere clogsd
8nd with th nttler upwara norement of the
plunger the fuel gets cohpis8ed. The higb pressure thus
lifts e devery valve of its Fie!Z31.Fu.lpry.
sea 8nd fuel flows to atoiser tfrough the passage
P. With furtber riae,gf the plugbr, t a certaib ::. ' 'i
momet, the port sP is conegtqd to the fuer in the upper part Fuel.atomiser or i4iector
of ttapiuuger-th"o;rgu te ctan_
Refer Fig, 2.32. It consistE ofa nozzle valve YV) fited in the nozzle body (NB). The nozzle
f,.".."-.Y:3-ry*:
by^T: he[&i gruive as a resutr of which a suid.o .p "
ano tb 6lellvt? valv fatrll beck atial orlcples lt t adtiidt
pi".*. o.",r"" valve is held on its seat by'a spring'S'wLich exerts pressrr tbrough the spindle E. 'A,S'is the
th liting force. the pluager is
rotated by te fek -8 which is rhwed in or out by.thB gorrimr. adjusting screw by which the nozzle valve l[ian be a{iusted. Usually the nozzle valve is set to lift
of tlu luliroJ ginave (bt rctn,tirg the plunger) af tl' plinger
& "l-rre,;rrs thc ergurar fusitil,n at 135 to 1?0 bar pressure. .FP is the feelirig pis whi indicates whether valve is working properly
rclative to tlu lwb pi,-r"-irrsri or not. The oil under pressure from the fuelurnp enters tbe injector through the passages B and
of stmhe dttrthg which the oil ta d,etivered can be oir. ard
therebx quo*ity- i iuet &livlred to C and lifts the nozzle valve. The fuel trayels down the nozzle N and injected into the engine
the hgirv i alea verbd ueorngly.
cylinder in the form of fine sprays. When the pressure of the oil falls, the nozzle valve occupies its
seat under the spring force and fuel supply is cut off. Any leakage of fuel accumulated above the
valve is led to the fuel tank through the passage A. The leakage occus when the nozzle valve is
uiortL out.
INTERNAL COMBUSTION
ENGINES
INTRODUCTION TO INTERNAL COMBUSTION ENCINES
65

Feeling pin (Fp) List oj engine parte, materials, method of manufacture and frrctions
:

Cylindzr Hard grade cast iron Contains gas -ndar res- Casting
Adjusring scrw (AS) sue mdguidestepirtm.
Cjlinder head Cast ircn or aluminium Mai fi ctioa ig to el the Casting, forging
working ad of the etiliDdcr
andnot toprmityed
exit of gales on overhed
Sprlng cap nut (SCN)
valve engiues.
Pston Cast imn or aluminium alloy
I
I It acts as a face to receive Casting, forgirg
Passage Spring (S) gas pressure and transnits
(4, the thrust to the mnect-
.
ingrod.
Piston rings Castiron Their main functio is to Casting
I prwide a good waling fitbe
i tween the piston and rylin-
ll --, der.
il Gudgeon pin
i1
rif
lN
Ful lrom
Hardenedsteel Itsupports and allows the
connectig od to swivel
Forging

l1
fuel pump Connecting rcd Albystel; for small engines
ll
the material may be alu- .It transmits the piston load Forging
li,, to the crant causingtjre iat-
I
Passage (C) splndte (E) minium ter to turn, thus onverting
the reciprocating noon of
the pistoninto rotary rnotion
ofthe ccarkshaft,
In general the crankshaft is It converts the reciprocating
made from a high tensile motion ofthe piston into the
fmging, but special cst iroDs rotarymotion.
are sometimes used to pro.
duce a light weight crank
shaft that does not require
Cap n (CN) a lot of machining.
Mainbearings The typical bearing half is The finction of bearing is to
made ofsteel or bonze back reduce the friction and allw
to which a Iining of elatively the parts to nove easily.
soft bearing material is ap-
plied.
Nozzte valve (NV)
Flywhcel Stel or cct iron. In engiaes it takes care of
'Nozzt body (NB) fl uctuations of speed during
Nozzte (N) t,I.ermodynamic cycle.
Inbt ualue Silicon chome steel with Admits the air o mixture
about 37 carbon. of ai and fuel into engine
Fig. 2.32. F\el atorriser
or iqjector. cylinder.
Exhaust ualue Austenitic steel Discharges the product of
combustion.
67
66 INTERNAL COMBUSTION ENGTNES INTRODUCTION TO INTERNAL COMBUSTION ENGINES

2.8. TERMS COIECTED WrIU I.C. ENGINES An engine with L = D is often called a aquare e4gine;
-
Refer Fig. 2.88. - IfL > D the engine is under tquere ;
lfL < D the engine is over aquere.
- lorge engines are olways und,er squore, with sro lengths up to four times bore
Intake manifold Eas{ ud.meter.
manifold Swept volum e. The volume swept through by the piston n -mouing betus-een top d'ead
ond.iotton dead, centre, is, cdlled "uept volu.me or piston displocenrenl"' Thus, when
Top dead centre Clearace centre
tr.D.c.) volume piston is at bottom dead centre, total volume = swept volume + clearance volume.
Pston at . Typical values for engine displacement range from 0.1 cm3 for small model airplanes to
upper lml aout S litres for large automobiles to much large number for large ship engines. The
of linear tawl displacement of a modern average automobile engine is about two to three litres.
a For a given displacement volume, a longer stoke allows for a smaller bore (under
.qrr"""-, resulting in less surface area in the combustion chamber and correspondingly
less heat loss. Chis increases thermal efficiency within the combustion chamber.
However, the longer stroke results in higher piston speed and higher-friction losses
that reduce the output power which can be obtained off the crankshaft. If the stroke is
shortened, the botu -o"1 be ino"ased and the engine will be over square. This decreases
Piston at friction losses but increases heat transfer losses. Mos modern automoble engines are
Boltom dead
centre (B.D.C.)
lower lnt near square, with soine st8htty over square and' sorne slightlY under squa're'
of linear trawl
compression raio. rt is ratio of total qilinder uolwne to clearance uolume.
Refer Fig. 2.33. Compression ratio (r) is given by

'=V'+V"
u
where V" - Swept volume, % = Clearace volume'
ThecompressionratiovariesfromS:1to11:1(averagevalueT:1to9:1)ins.I.engines
and from !2 : ! to24 : 1 (average value 15 : 1 to 18 : 1) in C.L engines'
r Modern spark ignition (s.I.) engines have compression ratios of 8 to 11, while compres-
sion ignition (C.I.) engines havJ compression ratios in the range 12 to 24.-Engines with
,ip"rZhorgu" or turochargers usuUy hor" lower cornpression rotios thon naturally
aspirated engines.
r Various attempts have been made to develop engines with auorioble compressian ratio-
Fig. 2.33. Terrns relatirg One such system uses a split piston that expand's due to changing hydroul-ic pressure
LC sagines. built which
caused. by ingine speed, and toad. Some two-stroke cycle engines have been
Bore. The inside diameter of the cylind.er is called. .bore". harre a that changes the slot opening on he exhaust port. The piston
reciprocates inside the engine cylinder, "i"u.rJ-typ"valve
wheretheexhaustportisfuIlyclosedcanbeadjustedbyseveraldegreesofengine
:l^T:!": +:
,^:..^_ posrtrons the,ei*on i has gotlimiring upper and
rower beyond which it cennot nove and reersal ofmotion rotation. This changes the effective cotnpression ratin of the engine'
positions. takes place at these limiting
Piston speed. The average speed of the pston is called' "piston speed"'
The linar d'istance along tlu qndr a.sis between two liming positions,
s catled. "stroke'. Pistonspeed=2iN
Top Dead Centre jT.P.c.). The top tnost position of the p*toi to-o
cover end, side of where Z = Length of the stroke, and
the cylind'er is called, "top dnad centre'. ln ase of orizonal e, rgines, this is known as inner dead lf = Speed of the engine in r.P.m'
centre.
Bottom Dead Centre (B.D.C.). The lowest positian of the piston towar's the cranh
end
. Average engine speed for all engines will normally be in the range of 5 to 15 m'/sonwiththe
sd'e of the cylinder is called 'bottom dad, centre" .
ln case of horizontal engines it i . .:alld outer l".ge i"sei"rrgines on the low end and high performance automobile engines
d,ead. centr. higi end. Ther are following ttto reasons why engines operate in this range :
clearance volume' The uorume contained in the cxtt .',rer aboue the top First, this is about th sale limit which can be tolerated by materiol strength of
the piston is at top dead centre, is cailed, tii iii"oron""
of the piston, - the engine comPonent*
'shen _ .tme-.
Bore sizes of engin';s range from 0.5 m down to 0.
^rn. The ratio of bore of stroke D/2, The second reason why maximum average piston speed is limited is because of the
' for small engines is usually from 0_g to 1.2.
- gas into and out if cylind.ers. Piston speed determines the instantaneous flow
flow
TNTERNAL coMBUsrIoN EN.INE.
into the clindt lNTRoDUcrtoN To NTBRNAL coMBUsfloN ENGINES 69
iiiTjf'::t
;mi"m;lii[iR"*,," j1,."*,:]:ni**H,fff immediately. The fuel i{ection is continued until the poiut of cut of is rcached. T}re burning of
fuel at fit takee place at conetant volume and continues to burn at constant pelsur tluring the
I Som Other Tems
*?'",:? first part oferpansion or working stroke. TLe field ofapplication of'his sJcle i" no"y oil engines.
:
. Direc I{iection (DJ.).
F 7"", into 2.10. INDICATOR DIAGRAM
the main combustio"_:p*r
3r:ln:*f*":gitli::!Kri or an ensine.
A indicator diagram is a graph between presse and volume ; tbe former being taken on
jr,1)::":.i';#igl;i*n*,H;r###ttH##X::: vertical "ris ad the latter on the horizonal axis. This is obtained by an instrtnent known as
ind.icator.Theincator diagrams are of two types : (c) Theoretical or hrohatical, () Actual. The
theoretical or hpothetical indicator diagram is always longer in size to the asual
,xTf:: :;#;#,i;y,!!*;::ry:;::;,;::t#."Ltr:,1,"i;33*r."tarac,eriscs "r --p"td
one, since in the former losses are neglected. The ratio ofth area bfth"."tori indicator diagram
Engine Mrnasie'entsvsr'en(E'M.s.).""-o"l*olJi"oj;';ffiJr'ff*.;, to the theoretical one ia called dqram factor,
. engines. i
Wide Open Thotfle (WO.TJ. 2.11. FOUR SIROXE CYCLE ENGII\rES
nurn power Engine operated with
and, I or sPeed throttte vatve
rgnition D'o*'o"
; ;',r;;:;,ne
is desired' fulgr open whn ma.r- Here follows the description of the four stroke oto and diesel-cycre euginee.
It is the time interual between otto engines. The otto four stroke+ycle referr to its use in petrol engines, gas engines,
"r"n .,
Alr-FueI""rrlli{n!r'^D')'
gnitiDn initiation and,
the actual light oil engines in which the mixture of air and fuel are drawn in the engine cynder. Since
,;:;;'if",T
Rtio (AT.). It is the ratio
of ignition in these engines is due to a spark, therefore tbey are also called spoih tgnition engines.
Fuer.Air Rtio (F/{.;;; il K:;:{"X:r,::"W* The various strokes of a four stroke (otto) cycle engine are itetailed belm.
Refer Fig. 2,34.
2.9. WORKING CYCLDS
An internal c Air fuel s.P.
can work
urK on anv one orthe mixture
tol con'tunt '13'Yionc'ngine rollowing cvcres :
ro co.,,t..,t JllHi"otT:L.J,t""
(c) Dual combustion
cycle.
These rnay be either
(o) Conetant voluno
four strohe grcle v'
or t strohe qrcle enges.
^- ^.._-_'",' _Luo

#ffi$nlnif"*iH'-;l:if ,t*#Tjilf,i*trf{ft*,:,*niffi rff j


frnilln**t**:;l::;f*'
,l'Tl#:i:::iTi,ff :T**;;'*::,ffi ilniHJil:'ffil"mi*lr""'.p".".
() constent il::":
-"e *ure ilorrys towards
eng'ine
rH:
^- .,-
;"""--*t3;iiif::l j::::,'i"J1?,:'"'".1i$.q,:on,vairisdr
metered ouantity e6.,lrire-incease v
a.'i'si*;"":' lf,lill;YtT
" "oiril""""'l'p^"iye

tr#d$iiil{$,llTi.#{,ffi+#f,fr ,'il'ji,.#
*.1u.: r,""r i. "lij""i1i""f!"'),";I)i,,1#[;.n"]::.:.1te sem-deset cycte. rtis so naned
strok
iililiiixx"iff.:*:l*-_-ttfi :".ii'ii:':i:"*:,i'"-in*"i"i#iiJ LV. =lls y\re, E.V. = ffrr$ yve, E.C. = Engine cylinder, C.B. eo661ig s6
ii"Ti":xi;fi C = Crank, S.P = Sparkplug. =
"T:irir"mii::#[rl[rd"#fi[;j:til]##'*:_ll Fig. 2.34. Four stroke Otto cycle engine.

_+--_
::--+- --:

70
r ENGINES 7I
TNTERNAL coMBusrroN ENcrNEs INTRODUCTION TO INTERNAL COMBUSTION

1' Suction strohe. During this stroke (also known as inductio stmke) the piston morres The loop which has area 4-5-1 is called negotiue loop ; it gives the pumping loss due to
is the total or gross
from top dead centre (T.D.C.) to trottom dead centre (B.D.C.); the inlet valve opens and proportion- ailmission of fel air mixtur and removal of exhaust gase6. The area l'2'3-4
from the area
ate fuel air mixture is sucked in the engine cylinder. This operation is represeoted by the liire
5-1 woit( otaine from tle piston and netrYork can be obtained by subtracting area 451
(Fig. 2.32). The exhaust valve renains closed throughout the t-2-3-4.
stroke.
2. Conpression stroke. In this stroke, the piston moves (1_2) towards (T.D.C.) and
compresses the enclosed fuel air mixtue drawn in the engine
ryliader rluriag suction. The pres-
sure of the mixture rises in the cylinder to a value of about 8 ba. Just befoe the end of thig stroke
the operating-plug initiates a spark which ignites the mixture aud combustion takes place at
constant volume (line 2--3) (Fig. 2.35). Both the inlet and exhaust valv* smnin doaed during the
i
stroke.
I
I
6

t
I
I
e
f
Volume--+
a
G Fig. 2.36, Actualp-Vdia8am ofa fow stroke Otto cycle engiae'
E
Diesel engines (four stroke cycle). As is he case of Otto four stroke ; this cycle too is
completed in four strokes as follows' (Refer Fig' 2'37)'
FI

Volume_--__r
Fig. 2.35. Iheoreticalp-Vdiagram ofa four stroke Otto cycle engine.
3. Exranslon or working stroke. when ttre mirture is ignitd by the epark plug the ho
geses are produced which drive or throw the piston from T.D.C. to B.D.b.
and-thug the work is
obtained in this stroke' It is during this stroke when we get work from the engine; the other thee
strokes name$ suction, coqnessio and exhaust being idle. The
flywheel mounted, on the enginc
shaft s.tores energy d.uring this stroh and, supptics it diring the idti strokes. lhe expansion ofthe
gases is shown by 3-4. (Fig. 2.35). Both the valves remain ciosed during
the start ofis stroke but
when the pistonjust reaches the B.D.C. the exhaust valve opens.
4. Exhaust troke. This is the last stroke ofthe cycle. Here the gases from which the work
has been collected become useless after the completion fthe expansi shoke and ae made
to
escape through exhaust valve to the atrnosphere. T'his removal ofgas is acconplished . -.
during this I
stroke. The piston moves from B.D.C. to T.D.C. and the exhaust gases are driven out of the engine
dinder ; this is also called scauengng, This operation is represnhd by the ne (1-5) (Fig. 2.85).
Fig. 2.36 shows the actual.indicator diagram of four stroke Otto cycle engine. It may be
noted that line 5-1 is below the atmospheri. pr"s"rr" line. This is due to the fact at owing to Stoke Stroke Stroke
restricted area of the inlet passages the entering fuel air mixture cannot cope with the speed ofthe F.l. = Fuel injacto, l.V. = lnlt valvs, E.V = Exhaust valv
piston' The exhaust line 4-5 is slightly above-the atmospheric pressure iine. This is due to
re- Fig. 2.3?. Four stroke Diesel cycle engine.
stricted exhaus passeges which do not allow the exhaust gses to leave the engine-cylinder quickly.

TNTERNAL COMBUSTTON t
ENGINES
INTRODUCTION TO INTERNAL COMBUSTION ENCINES
withthe movement or the piston
,i""k",1f""i:tli;"J,'J""Ti fron T.D.c. to B.D.c. during rhis
air. at atmospheri" I
cvlinder ; rhe exhaust
'qYe nowever :ht
remains closed. Thie oper.ation
o"otr""lJiirrl'rl"1t.
the engine I
Oig. z.g). ""r;:1t^.:.1-d i, *or".uo*ii, the line 5-1 ,

r,""-r';3."JilTil0T;.'*"T"t";ffi r,il:11.""*Tureduringrhesuctionstroke
the piston .o"". r-- ilT#Jff
n.o.c. to T.D.c. ft;s;|}";,H,":"1--'-"s.bT.*9 600"c respectively) as ^I
inlet and exhaust valves do bv 1-2 (Fig. 2.38). Both the
not open during any.part I
3' Expansion or
quantitv of tuer is injecred -:"y"g
"ffpff
a"-ti" pi.* ,r"* moviag E
o
inro rheror"t""r,..
co;;r;;J""tti to" ro*r" or;: from T.D.c. a matered 6
di,"r";;and it (tuer) E
f;:ffi ill::i:5 ff ff::tff:i**"re showabv the ';;-g. Al the point 3 tuer suppry is cur off. rhe
buf*u,h; ;;;o;;;;""ilTTi:friiilfi d*", :rH
4, thus doing work on the piston.
rfr" *p*Joni ffi ff tri:f:*i,,5i",,""ffi
o" r* (Fig.
"f,o"*o 2.s8).

Volume -------t
Fig. 2.39. Actualp-V diagram of foustroke Diesel cycle.

Valve Timing Diagrams (Ot'to and Diesel engines)


1. Oto engine. Fig. 2.40 shows a theoretical valve timing diagram fot four stroke "Otto

I I

g
a
6
E ;

c
f$
T, (t
(!
Fig.2.3g. Theoreticalp-Vdiagram
of a fouratroke esel cycle, il
4. Exhaust stroke. The piston
tr,e rttorprre""--l{,'oush rhe
moves from tht
tll
o li
exhau^st1"valve closes and
::_l_"p" exhaust ,,,r1u".'nf;;?;c;* *1.^D.:9.:d,the.exhausr
be piston
gases o
the i" Lvupeeq' r'hrs stroke
cycte -
---v vJve reaches the r.D.c. the
(Fig. 2.Bg). --pl"i"i'ri.wren is represented by the line 1-5
Fig.2.39 shows the ectual idicator
noted that line 5_l is below diagam for a

fh" "t*o'p".-",ii,'.-i;-"|ii,.,Tli3HrTff:lx;,}r11ff
resrricred area orrhe i,,r"t
r'
:*_11" which
passages
+sl.',ler,tr;Xiln'J:il::ffiffii":Ti::9:
rn8 rs f'h
,n',pJ
"ii" pisron. rhe
because of the resticted E,V..O.
do not allw the exhaust
rh e r oop or aea 4 _5- r
c"*;U; "au"i
r"a ous;riu"i;;o",l; "rrfi
arr and removal of exhaustgases. _is ""r ;t"T. J,,lffi r:T::io
prsron and net work The;""" i-;-'Ji.il"
ca be obtained u" r.JtoJtrg=ji"r r"l"_orgross-work ot"io"a
"*.rio,,
from the
or E37
;
n-u-, oo- aea 1-2-3-4.
Fig. 2.40. Theoretical valve timing diagram (four stroke Otto cycle engine).
1)

INTRODUCTION TO INTERNAL COMBUSTION ENGTNES


!"-!."-' lnsi*,
rnstantaneousrv '1hich is self-explanarory.rn actual practice, it is ai"H ;" :a
; so as to ce'better
"oJ" ""-
Fig' 2'41 is shown an acruilvlve rfo;;'o'rii? ".rgro" the varve timingr are modified. In
firir"li"..ltiiltr"r.,r"re
orthe T'D'c. posirion ro enabre.the is opened 10. ro 80. in advance
at the same time, to escaoe to.the
fre.;;;;; i'Lr r""yrioo Ii t"laitr" burnr gases
B'D'c' posirion'"t*orprt.""]i
irr.io' ortr'. mixhre *ntioo". op to s0,_40.
m" ll,l"i
ffi;*rl:i#* "ir" "ilr'". "nathe conpression of the enrrapped

.9
fn
(t :l
t
IJJ
o
c o
.3
q
Jr
ol il
ol an
.El
xl
u
;l c
o
o o I
o
IB.D.cJ

i
Fig. 2.42. Actual valve timing diagram (fow stroke Disel cycle engines).

2.12. TWO STROXE CYCLE ENGINES


In 18?8, Dugald-clerk, a British engineer introduced a cycle which could be completed in
two slroles of piston rather than four strokes as is the case with the four stroke cycle engines. The

i engines using this cycle were called two stroke cycle engines. In this engiae suction and exhaust
strokes arseliminated. Here instead of valaes, ports are used,. The exhaust gdses ore driuen out
Sg. 2.41. Actual valve timing agram (four Shoke
Otto rcle sncings). from engine qtlind.er by the fresh charge of fuel entering the cylinder nearly at the end, of the
The sparking plug produces a spark working strohe.
more time to burn' The oressure hf"T t!: T.D.C. position ; thus fuel gets
30' t 40"
exhaust valve opens so'
becomee -J-rrL o.*ly r0, pa't he T.D.c. position. The Fig. 2.43 shows a two sroke petrol engine (used in scooters, motor cycles etc.). Ttre clinder
q-q9- n"r",* d;;D;;rion and the gases are ive out of the L is connected to a closed crank chamber C.C. During the upward stroke of the piston M, the gases
its upward -ou"-"o-I{" lxh.uet in rr arecompressed and at the same tirne fresh air and fuel (petrol) mixture enters the crank
frq||: ll.#:[j#:: varve Joees wheu pisron is nearly
chamber through the valve V. When the piston rnoves d.ownwards, V closes and the mixture in
2' Diesel engrnes. Frg. 2'12 shows the
the varve timing di agram of a crank chamber is compressed. Refer Fig. 2.43 (i), the piston is moving upwards and is compressing
engine (theoretical valve timing diagram, fuur stroke "Di*er cyfu,,
to 25' in advance of r.D.c. polionLa
f"i";""*ii" srlre as Fig.2.1o).Iulet valve opens 10.
an explosive change which has previously been supplied to L. Ignition takes place at the end
ofthe
opens 30'o 50" in advance "r"r;;;;;; h after the B.D.c. posirion. Exhaust valve I
stroke. The piston then travels downwads due to expansion ofthe gases (Fig. 2.ag (ii)) and near
of B.D,c. p*iti"i"-ri ro" b 160 after the T.D,C. position. The I the end of this stroke the piston uncovers the exhaust port (E.p.) and the burnt exhaust gases
place 5" to 10'i"*-lic. "il:", ecape through this port (Fig. 2.43 (jrt)). The transfer port (T.p.) then is uncoverecl
ffi1]fr:|Ajakes i'Jr?oo and conrinues up ro 15. to 2b. near inmediately,
i and the compressed eharge from the crank chamber flows into the clinder and is deflected up-
I wards by the hump provided on the head ofthe piston. It may be noted that the incoming air petrol

I l
rfi
IIVTERNAL COMBUSTION
ENCINES INTRODUCTION TO INTERNAL COMBUSTION ENCINES 77
ru'
ffi:Tj:fi.T;::ilff:"f
Ii:"
beeer uae engtue-cvn1:l_,rlt^"1.r"
tl" cylinder, t" r""J :i:_ll"-,j*i,51::o*.il-*,in^case these exhaust
exhausrgases
gases do not shows the p-v diagram for a two stroke cycle engine. It is onry for the main
piston then agJ:";;H3::TB
againstarts **t,r?l-"n.;j;T.#. -Fig.2-.44
t"'::1L:l*
(exhausrportiancrrp."*.",""*
iexhaust port) and T.p. are
l:j"f # l!,1ecv.
j'ily,^1";Jl"i.irf;;fr
or the engine wlr decrease.
ffl;n".or'n" ";;;il;.rease rhr rhe
charge gets compressed
clinder-or the top side of the piston. ?ig. 2.45 shows self-eiplanatoiry port timini diagram for a
covered bi the nsr.n. . _'',,- .,^r_0"
f JT j:?:?i::""; when E.p two stroke cycle engine.

plug

fll
)a
o
I 9

a
i1
o
rl J
I

Fig. 2.43. TVo shoke cycle


engine

Fig. 2.115. Port timing agrm.


In
a two strohe Diesel cycle engine oJl the operatinns are the sarne ds in the sparh igniton
(otto cycle) engine with the d.ifferences
; firstly in ths case, only air is ad,mitted into qlind.er
instead' of air fuel mirture and, secondry
E
fuer injector is fitted. to supply the fuet instead of a
sparhing plug.

2.I.S. NTAKE FOR COMPRESSION IGMTION DNGINES


o The compresson igntion (C.1.) ngnt are operated, unthrottled. with engine
speed
and' powercontrolled by the amount offuel injected durng eoc. cycb. T1is allows for
high volumetric efrciency at all speeds, with the intake system d.esigned,
for uery rittre
flow restriction of the incoming air. Further raising the volumetric efliciency is the
fiact that no fuel is_ added until late in compression stroke, after air intake
is fully
Volume ____________r completed. In addition many C.I. engines are turbocharged,, which enhances
air intahe
Fig. 2.,44. p-V diagram foia euen more.
tw-o stroke cycle engine
INTERNAL COMBUSTION ENGINES INTRODUCTTON TO.INTERNAL COMBUSTION ENCINES

The addition of fuel is made late in the compression stroke, starting somewhere around S..lVo, Aspects Four Strche Clele Enginec Tuo Stmhe Cycb&ngtm
20' before T.D.C. Injectors mounted in the cylinder head inject directly into the com-
t FIywheel required Because of tle above tuming-move- More ifom tuming movemt md
bustion chomber, where self ignition occus due to the high tenperature of the air I
-heaaier or lighter ment is not so uiform and hence hence lighter flywheel is neded.
caused by compression heating. I
he av ic r 7w\eel is needed, I
It is important that fuel with the correct cetane number be used in an engine so that
self-ignition initiates the start of combustion at the proper cycle position. a Power prcd'uced for Again becawe of one power stroke Because of ooe power stroke for one I
same size of engine for two revolutions, power produed revolution, power produced for sme I
For C.I. engines, the iqiection pressure must be much higher than that required for for sme size of engine issmclJ or for size of engine is more (theoretically I
S.I. engines. The cylinder pressure into which the fuel is first injected is very high near the same power e engine is heavY twice, actually about 1.3 times) m for I
the end oftbe compression stroke, due to high compression ratio ofC.I. engines. By the andbulky. the ssme power he engine is ght I
time the fial fuel is injected, peak pressure dung combustion is being experienced. and compact.
Pressure must be high ewugh so th.at fuel spray will penetrate across tlw entire com- Because of one power stroke in two Beeuse of one power strcke in one I
4. Cooling and. lubrica'
bustion chamber. Iqjection pressures of200 bar to 2d)O bar are comrnon with auerage tion requirerents revolutions Jesser cooling and lubri- revolutiongroarer cooling and lubri- |

fuel d.roplet size generally d.ecreasing with increasing pressure, Orifice hole size of cation requiremenk. Lesser rate of cation requirement. Great rat ofl
injectors is typically in the rauge of 0.2 to 1.0 mn diameter. wear and tea. wearandtear. I

The mass flow rate offuel (nr) through an injector, during injection, is given by the o. Value and,ualve mha- The fou stroke engine coains valve Two stmke engines have u valvs
relation : nism and valve medlanism. but only ports (some two stroke en-
gines are fitted with conventional
m, = CoAo (2.1) exhautvalves).
6
The total mass offuel (m) injected into one cylinder during one cycle is given as : 6. Initial cost Because ofthe heavy weight and com- Because ofght weight andsimplic-
plication of valve mechanigm, ghr ity due to absence ofvalve necha-
mr= CrAn r@r4p taeneo Nl ...(2.2) is the initial cost. nism, cf,rcper in initial cct.

where, C= Discbarge co-efcient ofinjector, Volumetric effrciency Volmetric efEcienry mrs due to more Volumetricefficiencyess duetolesss
tine ofinduction. time forinduction.
An = Flow aea of nozzle orifice(s),
Thcrnol part'lmd. 'fhermal effi.ciency higher, part load llhemal efiiciency lower, part load
Pr = DensiY offuel, 8. and'
I efciency better than two stroke cy- I efficiency lesser than fou stnke cy-
Ap = Pressure difrerential across injector,
I
cle engine. lde engine.
A0 = Crank angle through which injection takes place (in degrees), and I

.iV = Engine speed. 9. I Used where eicieocy is importan' ; lIn two stroke petrol engine some fuel
I in mrs, bvses, trrchs, trutors, ind.w- lis exhausted during scavenging.
Again, ...(2.3) power gen- lUsed where (a) low ust, and,(b) com'
P,q. = 4P Itrial engires, aeroplare,
and, P*i. -M ...(2.4) I erators etc, light weig ht mportant.
(air cooled) petrol engines
(To ensure that the crank angle of mtation through
wed in very small sizes onlY, bwn
which injection takes place is almost constant for all speeds) moves, swters motnr cxcbs (ltbri'
Large engines must haue uery high injection pressuri and, high spray velocity. oil mixed with petrol).
- For optinum fuel viscosity and, spary penetraton, it s importont to haue fuel at esel engine used inrery
- the correct tetnperature, Iarge si*s more than 60 un bore, for
ship propulsion because of low
(Often engines are equipped with temperature Bensors and means of heating or weight and compactness:
cooling the incoming fuel. Mony Lage truch engines are eqipped with leated. fwl
filters. This allows the use of cheaper fuel that has less viscosity control.
In small engins more costly, lower uiscosity fuel is requred. ' 2.16. COMPARISON OF SPARK IGIITION (S.I.) A}'ID COMPRESSION IGNITION (C.I.)
- ENGINES
2.14. COMPARISON OF FOUR STROI(E AI\D TIVO STROI(E CYCLE ENGINES
S.]Vo. Aswts S,I. enginee C.I. mginet
S.N. AsretB F our Stroke Cyc lz Eng lre s T\oo Srv*e Cyclc Engines
t. Thernodynamic cyclc Otto cycle Diesel cyde...... Forslow sPeed
engines
l. Completion of cycle The cycle is conpleted in four The cycle is completed in uo Dual cycle...... Forhighspeetl engines
stmkes ofthe piston or intuo reuo- strokes ofthe pistonorin.ore reuo.
lutim* of tfu cmnlsaf. Ttru one Iution ofthz cronkshaft. Ihus one eel.
power stroke is obtained in every power stroke is obtained in each
Fwl used Petrol
two revolutions of the crmkshaft. evolution of the makehaft,
80
INTERNAL COMBUSTION ENGINES
INTRODUCTION TO INTERNAL COMBUSTION ENCINES 8l
S.No. Aspuh S.I. engnet
C,I. engi.nec 2.16, CoMPARTSON BnTtvEEN A PEI|ROL ENGINE AND A DTESEL CWn
3. Air-frel ratio 10:1to20:1 18 : I to 100: l. ,9'lVo. Petrolngbu Dieselengirc
Compression ratio upto ll; 12to24i
Averagevalue 7 to 9; Average value lE to 18 ; I. Air petrol mixture is sucked in the engine cylin- Only air is sucked during suction stroke.
Upper lirrit of ompression ratio Upper limit of compressim ratio is der duing euction stroke.
fued by a n-hnuh e wt ity of frk I. mited by f/urmcl and mechanbal 2. Spark plug is used. Employs an iqjector.
stBss.
Combustion Power is produce by spark ignition. Power is produced by omprmion igo.ition.
Sparkignition Cunpression ignition.
Thermal effi ciency rp to 25Vo. Themal effrcienry up to 40%.
o. Fuel supply By carburettor ...... dreap method By injection ...... e:pemive metod- D. Ocopies less space. Occupies more space.
\
7. Operating pressure o- More mning cost. I*sr running cost.
(i) Compresion 7 barto 15 bar
30 bar to 50 bar Lightinweight. Heavy ia weight.
pressue 45 bar to 60 bar 60 ba to 120 ba. Fuel (Petrol) costlier.
(ii) Maximum pressure 8. Fuel (Disel) cheaper.

Operating speed High sped : 2000 to 6000 r.p.m.


9. Petrol beingvolatile is dangerous. Diesel is non-dangerous s it is non-volatile.
Iow speed:400 r.p.m.
Medium speed : 400 to l2m r.D.m. 10. Preiglition posible. Pre-ignition not possible.
High speed : 1200 to 3500 r.p.rn Works on Otto cycle.

"r*
11. Works on Diesel cycle.
9. Control of power Quantity gorerdng...... by throttle Less dependable.
I Quality govemiry -._. by rac&. 12. More dependable.
10. Calorific ualue
12Mr1r{. 13. Used incrs andrutor cycles. Used in heavy duty vehides lits rs, ses
I
ll. Cost of runnine I adheat! mtchrgry.
low.
12. I l
I Maintnanceeost Minormaintenacerequired Major overall required but less fie- |
I
qently. 2.I7. HOW TO TELL A TWO STROIID CYCLE ENGINE FROM A FOIJR. STROTE
I
CYCI,E ENGINE ?
lJ. Supercharging Limitllby dcllmtbnUra ontv io I r.i'tritd by bbwer pounr and. me. I
ircraftengires, clnnircl and tlennal s besxs. W*tcly S.Na, Distngz,ishfu feotureg Foar otrche Two etrohe engine
carclc engtne c2rcle
I
ued I

14. T\ao stroke orerotian 2ss szircJe, fueI lms in scavene- I I


ia scavensing. More l. Oil sump and oil.filter plug It hu an oil sunp aDd oil-filtr It does not have oil sump and oil-
ng. But small two stroke enginJs *Jj."*
I I plug. filterplug.
re used in mopeds, scooters and I
notocyclesrdue to theirsinf/rc;rf I Oil druins etc. It requires oil drains md refills In this type of engine, the oil is
periocally, just an automobile

;l
I added to the gasoline so tlat a
High powers
Yes. do. mixture of gasoline md oil passes
16. Dstribution of fuel llratiois opti-r-in through the carburettor md en-
not
ylinderengines. -rlti I &ellent dishibution of fuel in nrlti- ten {he crankcase with the air.
rylindr engines.
I
t7. Startng 'asy,lowcralkingeffort. 3. Inationof muffler It is istalled atthe head enal of It is imi ..:d towards the middle
Difficult, high cranking effort. (uhaust silener)
I the cylinder at the exhaut valve of the cyr crier, at the exhaut port
18. Exhaust gas tempera- loetion. locsl,ion.
Low, due tohigh themal efficienry.
ture
4. Nmeplate If the name plate mentiom the If the name plate tells to mix oil
19. Weight per unt power type ofoil and the srankcase c- two stroke
|Iigh (3.3 to 13.5 kg&W). with the gasoline, it is a
20. Initial capitdl cost pacity, or similar data, it is a fou cycle engine.
Iigh due to heavy weight and study stroke cycle engine.
:onstruction, costly constructiotr,
l.2Sl.5times.
21. Noise onduibration Il
I ore idle noise problem.
22. I U""" M lues, trucks locomotives, tractors,
pl arth moving machirery and sLation-
ED
ry generating plants.
82
INTERNAL COMBUSTION ENCINES
INTRODUCTION TO INTERNAL COMBUSTTON ENGINBS 83

(ui ) Measurement of air consumption by air bor method :


l' Any type of eugine or machine whi <Ierives heat
_
mergr from te mmbution of fuel or any other souce Ih
_ and converts this energy into mechanical work is
terur as a freat engine. Volume of air paesing through the orifice, { = 8a0AC, J f
2 The function of a carbuettor is to atomise and
meter the liquid fuel anct mix it with air 6 it enters ttre
injection svstem ofthe engine maintaining unaer and mass ofair passing through the orifice'
aii c-aitior" orop"""tion fuel airproportion approxi-
mate to those conditions, m. = 0.066 C x d .fi,lp1 tg*i"
& 'Ihe two basic ignition systems in onent use an :
(i) Battery orcoil ignition sysrem where, A = Area of orifice, mz
(ii) Magxeto ignition system. d = Diameter oforifice, cm
h = Head ofwater in 'cnf causing the flow
4 Following are the methods ofgoverning I.C. engines
: o = Density ofair in kgi/m! under atmospheric conditions.
(j) Hit and miss method (ii) quat"tot"*tt
(iii) Quantitygoveming.
6' Pre'igrrition is the premature combmtio which starts
before the appcation ofsprk. overheatd spark
OBJECTTVE TYPE QI,]ESTIONS
plugs and exhaust valv* which are.tlt.
-"i" o*;;f;;;-ignition should be carefully avoided in engines.
A very sudden rise to pressure dwing mmbustior Choose the comect answer :
detonation' The region in whicl detnation oeurs ly metallic hamme ,'ke eound is called are
"."o-pi"a
i"J"lt".t from the sparking plug, and is In a four stroke cycle engine, the four orerations namely suction, compression, expmsion and exhaust
named the'detonation zone'and even with.*"t" ""-ored completed in the number ofrevolutions ofcank shaft equal to
a"t"n"iioo this zone is rarely more than that one
quarter the clearance volume. (o) four () tlree

octane in the nture (c) two (d oue.


" Tl",::3i:_...Tf;*..T.1Tr"":ntaseof
heptane (low rating), byvoluel which
urte" nu."-"
[of isomtane (high raring) and normal
Tli:i In a two stroke cycle engine, the operations namely suctim, compression, expansion and exhaust are
knocks
Delay period or ignition lag is the time imrneately -iiiffiff
on"wi"g iq""tio" of fuel during which the ignition completed in the number ofrevolutions ofcrank shalt equal to
prmess is beinginitiated and theprersure (b) ee
does notris'e buyou-re"atoe itwouldhave due to compression (o) four
(c) wo (d) oe.
Higher the cet"ne rating ofthe fuel leseer is the pmpensity
for diesel knock In general a high octane value In a four stroke cyde S.I. engine the cam shaft rug
implies a low cetane value. () at halfte sPeed of crank shaft
(o) at the same speed as rnk shaft
lo. The, purpose of supercharging-is to rai.ae
the volumetric efEcimry above that value that which can (c) at twice the speed ofcrank shaft (d) at any speed inespective ofcrank shaft speed.
obtained bv nomal aspiration. superc'arging be
of p"t-t "."." orit, po- "i"."""riv,
very popular and r used only when a to"ge
r*uot ""ei";; 4 Ttre following is an S.I. engine
comperuate altitude los.
orpo*i" J "ua.a
o" wh"nio." po*""lil"" t" "" (o) Diesel engine () Pehol engine
ll, Dissociation refers to disintegratio-nof bumt gms (c) Gas engine (d) none ofthe above'
at high temperatures. It is a revemible process ad
rnceases with temperature. Dissociation, in geieral,
ar"r'"a . fo"'u of po*u" *a um"ie."y.
- The following is C.L engine
Performance ofLC. engines. Some important (a) esel engine () Petrol engine
relations :
(c) Gas engine (d) none of the above.
(i) Indicated power (I.p.) np^rr'ANk xro
=
6
kw & In a four stroke cycle petrol engine, during suction strcke
(c) only airis sucked in () only petol is suc}ed in
(ii) Brs p.*.' q:6o d)lv mixture ofPetol and air is sucked in (d) none ofthe above.
{n. p.) =
r{E#' o* ". [=ffi**) 7,
(c)
In a four stroke cycle diesel engine, during suction stroke
\ () oly fuel is sucked in
(o) only air is sucked in
(iii) _J,,,,d=
Mechanical efnciency,r-*
I.p. fj (c) mixtue of fuel ud air is sucked in (d) none on the above.
(iu) Ihermal eficiency (indicated),rr,n I.P. & The two stroke cycle engine has
=
rh, xC () one suction valve ud one exhaust valve operated by one cam
() one suction valve and one exhaust valve operated by two cmr
and thermal elficiency (brake),r*.", B.P. (c) only ports covered md mcovered by pision to effect arging and exhausting
=

where f = mass of fuel used in kg/sec. ^a 9. lbr


(d) none of the above'
same output, same speed md same compression atio the thermal efficiency of a two stroke cycle
petrol
lthemal engine as compared to that for four stroke cycle petrol engine is
(u) n ()less
= tlair-stedad () more
(c) same aslong as coirpressionratiois same (d) same as long ro outPutis same'
10. The ratio ofbrake power to indicated power ofar I.C. engine is called
(c) mechanical effi ciency () themal efficiencY
(c) volumetric eflrciencY (d) relative efficiencY

ii
1l
84
r
INTERNAL COMBUSTON ENGINES

| ^{mrrnr I
r. (c) z (d) & () 4 () (a) 6. (c) I
& (c) e. () lO (c). 7. (a)

_
:nrronnucAr er.r'sTr;G-l
l. Name the two gen"r" r".r""
| Air Standard CYcles
z Discuss thererative advataro "ffieylasicauy
d aris"r*t"g&-"ii"i"JJ.b.ouoo
sp-
difrain principle ?
& baeic ty{s u_t"- *Ju*["]?#'z w,.t mginss. 3.1. Definition of a cycle.3.2. Air standard efrcienry. S'3' The Carnot rycle'3'4",Comtant
ill;H"Jlii:? "od"rt"-"Ioorrhuti-on
*," tuadamental cycle'
"". difiereacss volume or Otto cycle. 3.5. Constant Preasure or Diesel cycle' 3'6' Dual combustio
L What is the function .i. bo*pu.o" Jf oto, Diesel antt Dual combustion rycles_Efiiciency versus courpression
portegovernor.ofa Eoven
- nor?Enumeratethetypeeofgovemorsanddiscusswianeatsketchte
ratio--Fr the same compression ratio md the same heat input-For coDstant maximm
5. Dilfermtiate between a flywheel and a govemor. pressure and heat supplieil. 3.8. Atkinson cycle' 3.9. Ericsson cycle' 3'l0 Brayton-cycle'
6, (al the fuction of a carburettor i.fr-Sti"tlrg 3.12. Miller cycle. 3.13' lcnoir cvcle-Highlights-Objective lYpe Quegtiorp
_Jrrt in a petml engine. "y4". Examples.
() Describe a simple Theoretical QuestioeUnsolved
carburettor
with u o""t
Explain with neat sketches the construction "t"tr-LJio state its limitaons.
and worling ofthe following :
(d) Fuel p'mp
& Exptain tie roitowing rerms as apptied 8.1. DEFINITION OF A CYCLE
r" I.c.
Bore, stroke, T.D,C., B.D.C., clearane ,h,*t"t #':: A cycle is defined as a repeated, series of operations occuring in a certain ord.er, lt may be
volume, mept volurne, cor perfect
9. Dxplain withsuit"i".r"o tr" *""H"c;;;;:;;"*k" repeated 6y repeating the procJsses in the sam oriler. The cycle may be of imaginary
ffiff:":-*tioandpietonspeed_ In ideal
to. Discuss the difference between i uo" o. tout errgii". lbe former is called ideal cycle and the latter actual cycle'
11. i.:Tff t"::i$fr h:[*:*,,m;Tlroii:"*:,1;m;* .yJtu att teat losses are prevented and the working substance is assurned to behave like
,,ffiTi ::fi#n:: cyc,e spark a perfect"."i"ntal
working substance.
tz Disruss t,'e difference between trreoreticar
l& ad actual valve timing diagrams
of a diesel engine ?
the developmmt of two
; 3.2. AIR STANDAND EFFICIENCY
ffiJ!"-""" "rote "rg;;;;'; are rhe two main trpes of two sroke
it is ofparamount importancethat the effect ofthe
t4 To compare the effects ofdifferent cycles,
calorific value of the fuel is altogether eiiminated uttd thit can be achieved by considering air
15. i"T?:#1T:',r',i*:[:t]ffi::ffi:Tcvcresparkignition(sr)ensine.Howsindicaroragran
compare the relative advutages and disadvantages (which is assumed to behave as a f,erfect gas) as the working substance.in the engine cylinder. ?he
offour stroke and two stroke cycle
engines. efficiency of engine usng ar as ti" workng medium is kniwn os on "Air standard
efiiciency"'
This efliciency is oftenly called ideal efficiency.
of that cvcle
The actual efciency of a cycle is always less than the ai-standard efficiency
under ideal conditions. This is trken into account by introducing a ne$' term "Relative
effi'
ciency" which is defined as :
Actual thermal efficiencY ...(3.1)
lrehtivc =
Air standard effrciency
The analysis ofall air standard cycles is based upon the following assutnptions:
Assumptions :
1. The gas in the engine cylinder is a perfect gas i.e., it obeys the gas laws and
has con-
stant specific heats.
2. The physical constants ofthe gas in the cylinder are the same as those ofair at
moder'
ate temperatures i.e., the molecular weight of cylinder gas is 29'
cp = 1.005 kJ/kg-K, cp = 0'718 kJ&g-K'
place without
3. The compression and Lxpansion pro."".", are adiabatic and they take
internal friction, i.e., these pocesses are isentropic.
4. No chemical reaction takes place in the cylinder. Heat is supplied or rejected-by
bring-
or a body in contact with cylinder at appropriate points during the
ing a hot body cold
Drocess.

85
86 INTERNAL COMBUSTION ENCNES 87.
AIR STANDARD CYCLES

5. The cyclc is considert:d r:lrsed with the same'air' always remaining in the cylinder
to Stage (1). Line 1-2 fFig, 3.1 (c)l reprerente the ieotbermal expansion whic,b take place at
repeat the cycle.
temperatuie ?, when source ofheat lf is applied to the end ofcylinder. Heat eupplied in this caee
It.S. THE CARNOT CYCLE is given by R?, log" r and where r ie tbe ratio of oxpansion.
Stage (2). Lino 2-3 represents the epplication of non-conducting cover to the end of the
This cyclc has the lrrgcsf ressible fficienq and consists of four simple operations namely, cylinder. This is followed by the adiabatic apansion and the temperature falls from Trta T2,
(a) Isothemal expansion Stage (3). Line 3-4 represents the isotermal compression which takes place when sump
() Adiabatic expansion ,S' is applied to the end ofcylinder. Heat is rejected during this operation whose vslue is given by
(c) Isothermal compression RTrlog" r where r is the ratio ofcompression.
(d) Adiabatic compression. Stage (4). Line 4-1 represents repeated appcation of non-conducting cover and adiabatic
The condition of the canot cycre may be imagined to occur in the foilowing way : compression due to which temperature incceass fron ?, to ?t.
One kg of a air is enclosed in the cyliuder which (except at the end) is made of perfect non- It may be noted that ratio of xpaDEion during isotherrnal 1-2 and r*tio of conpression
conducting material' A source of heat 'II' is supposed to provide unlimited quantity of during isothermal 3-4 must be equal to get a closed cycle.
hea, non- Fig. 3.1 (b) represents the Carnot ccle on ?'s coordinates.
conducting cover 'C, and a sump 19 which is of infrnite capacity so that its ierrpeiature
remains Now according to law of conservatim of energy,
unchanged irrespective ofthe fact how much heat is supplied to it. The temperature
of source I/ is
?t and the same is of the working substance. Ttre working substance while rejecting hea to sump Heat supplied = Work done + Heat rejected
',S'has the temperature Tri,e.,tLLe same as that of sump S. Work done = Ileat supplied - Heat rejected
Following are the fozr s4ges ofthe Canot cycle. Refer Fig. 8.1 (o). =R?r.log"r-RTrlog"r
Work done Rlog.r (T1-72)
Efciency of cycle = ll*t""ppj*d=- n1-t*t
Tt-T,
--TL ...(B.z)

From this equation, it is quite obvious that iftenperature ?, decreasos, efficiency increases
and it if ?, becomes absolute zem which, ofcourae is impossible to attain. Further
becomes 100%
more i is not possible o prod.uce an enginc that should work on Cornot's qrcle o.8 it would.
necessitate the pston to truel uery sloui$r furing first portion of the forword. stmlrc (isothermal
erpansian) ond. to travel more quchly duing the renainder of the strohe @d'iabatt ezpansiPn)
which however is not practzable.
Example 8,1. A carnot engine worhing between 400'c dnd, 40"c produces 130 hJ of work.
Determne :
() The engine thermol efficiency.
(i\ The heat ad.d.ed',
(ii) The entropy changes during lual rejectinn procss'
Solution, Temperature, T1= Tr= 100 + 273 = 673 K
Temperature, Tr= Tr= 40 + 273 = 313 K
Work produced, lV = 130 hI.
(i) Engine thermel efciency' q," :

n,^. =
gHg = 0.6:|6 or 68.7o.
(Ans)
(ii) Hea added :
Workdone
rl. =
Het added
(c) Four stages of camot cycle ()T-s agram
0.5s5 =
Fig.3.1 Healk;
.'. Heat added = # = 243 kr. r,lns)
INTERNAL COMBUSTTON ENGNES
AIR STANDARD CYCLES
(iii) Entropy change durlng the hee rejection proccss,
(S, - S.) : (ii) The volume a the end of isothermel expansion V,
Heat rejected = Heat added - Work done
:
Heat transferred during isothermal expansion
= 243 - 130 = 118 kJ

313 K

T-S diagram
p-V dagram

FiC.3.2 Fig. 3.3. Camot cycle.


Heat rejected
= ?r (Sr - S.) = 113
= p1v, ln(r) - mlTrt' |.9] = 4o x ld ..,..(Given)
(ss - s{) = =;|} = o.B6r rar/K. (Ans.)
\ vr,/
Example 3,2. 0.s
# or 0.5 x 282 x 58b.4 ln fg) la
rs of air (ideat gas) e,,zcutes a carnot powera thermarcycre having = 40 x
of 50 per cent. Tr ruot transfer'ili \0.12l
"[ni:n"v the sothrmar exponsion is 40 hJ. At
the beginning of the isothermar expanson the"t"-"ra
pressure is 7 bar and, the uorume is 0,12 4ox1o3
Determine mB, or nl]\ = os'zst;s8r4 = o 47e
\0.12./
(i) The mazmum and minimum temperd,tures or
for the cycle n K ; Vz = O.lZ x (e)o.476
= 0.193 m3. (Ans.)
(i) The uolume at the end. of isothermal (ril) The heat
e:;panson in ms ; transfer for each of the four processes :
(iii) 7" heat transfer for each of the
four processes in hJ. Process Classfication Heat transfer
For air cu = 0,721 kJ / hC K, and co = I.0OB kJ I kg K. (U:p.S.C. 1998) 7-2 Isothermal expansion 40 kJ
Solution. Refer Fig. 8.S.,Giuen : n = 0.5-kg i
mal expansion = 40 kJ ; p1= 7 \6 = geoo; Heat transfered
during isother_ 2-3 Adiabatic reversible expansion zero
Vt = O.t'o,r ;-", ='b.ZZf kJtS K-, ;r-= i.'Oi iin
_bar, sf at Isotherrnal cornpression
() The maximum and minin,n temreratures, \, T, : ' - 40 kJ
4-r Adiabatic reversible compression zeto. (Ans.)
P1V, =
'17,
Z x 10 x 0.12 O.S x
$Example 3,3, In a Carnot cycle, the matitnum pressure and, temperature are limited. to
= 287 x T,
18 bar and 410"C The ratio of isentropc compression s 6 and isothermal expansion is 1.5.
... Maximum remperature, r, = t;:T;#, = 585.4 K. Ans.
Assuming the volurne of the air at the beginning of isothermal expansion as 0.18'm3, d,etermine :
(i) The ternperature and pressures at main points in the cycle.
(i) Change in entropy during isothermal expanson.
no^"=L# + o.s= q#a (iii) Mean thermal efficiency of the cycle.
.'. Minimum temperature, Tz= EgS.4 _ 0.5 x 885.4 = 22,2 K. (Ans.)
(iu) Mean effectiue pressure of the cycle.
(u) The theoretical poruer if there are 210 worhing cycles per minute.
9l
AIR STANDARD CYCLES
90 INTERNAL COMBUSTION ENCINBS
/ \1= t, I'v,q=ql
Solution. Refer Fig. 8.4. Ps=Pzx
t,J " lb),
\ro)
fL %l
Maximun pressure, pr = 18 bar
Maximurn temperature, ?t= (Tz) = 410 + 2ZB 693 /r \L4
= t' " [;j = o'e7 bar' (AnsJ
Ratio ofisentropic (or adiabatic) conpression, 3V1 =6
Hence pr = 18 bar \=?--685K
I
Ratio of isothermal exransion,
pz=t2bat I 0\ns.)
? = t.U.
Pe = 0.9i1 bar Ts = Tn = 333'2 K
I
Volume of the air at the begiming of isothermal expansion,
% = 0.1g ms. p = 1.46 bar )
(d) Temperatures and pressuree at the nin pointe in the cycle : (ii) Change in entroPY :
For the isenropit process 4-l Change in entropy during isothermal expanson,
1.. pV --mRT

= (G)r'1-r = (G)04 - 2.05 s2- sr = mJ,tos"(+\=+'"u


\v1,/ r
l9l
tVt/ lor ,nR= PV
I

e =
[+)" L T
r,=#=#=ae3.2K=?s -
18 x 10o x 0'18
loe- (1.5) = 0.192 6 $ns.)
10o x 683

(iii) Mean thermal efficiency of the cycle :


lv-\
Heat supplied, e" = n1Y1t.*
lu, ,J

= 11 (s2 - sl)
= 683 x 0'192 = 131.1 kI
/ \
Heat rejected, e,=pava." l.]
= Ta (Ss - Sa) becauee increase
in entropy during heat adilition
il eqiral ti d"".uu"" in entropy during heat rejection'
Q" = 333.2 x 0.192 = 63.97 kJ

.'. EfficiencY,
a"-a, .
,,=-A- Q,

63 97 ot 51.27o. (AnsJ
FiC.3.1 - 1- - 131.1
= 0.512 i

Mean effective pnessrre of the cycle' p- :


(iu) :i
Also,
..
Pt
pL = (fl =(6)t=12.2e The mean effective pressure ofthe cycle is given by
Work done per rycle
P-= --3t of"ffie-
P= =!!==
t229 # = 1.46 bar
v"
Fot the isothennal process L-2
Vl =6x1'5=9
P1V1= P2Y2 8 x 0'18 = 1'44 mg
% = V, - Vt= 9Vt- Vt= 8Vt=
o'=$= 18
l5 = 12 bar .- (Q"-Q,)*. _ =
(Ar_-QJf] (... J=1)
v2
For isentropic process 2-3, we have
P^=
-- v" -i"
68.97) x 103
p2Y21 = psYrr =(131.1-t,+;A-
_ - 0.466 bar. (AnsJ
=

rI
I

I
92
NTERNAL COMBUSTON ENGINES
r AtR sTANDARD cYcLEs 93
(u) Power of the engine, p :
Power of the engine working on tbis Example 3.G. An id'eal engine operotes on the Carnot cacle usng a pet'ects gos as the
cycle is given by
workng flui.d. The ratio of the greatest to the leost uolume is fired atd s r : 7, the lower tempera-
Examp, e s.4. A,_"::!,,;;;;;W:#i:i,,-*T# ,1!; ,#::,0
ture of the cycle is also fi.xed, but the uolurne compression ratio'r'of tln reuersible d.iabatic
compression is uariable. Thc ratio of the specific heats is ^.
X:I:;:T:;",:,'!t:";:o
is reduced' av zo"c,;;;-"tu';,; is doubrcd. Find, the teiperature
when the
'
of the Show thot if the worh dnne in the cycle s a manimum tlen,
Solution. Let ?, = Temperature ofthe source (K),
,,
and (Y- 1) Ioe L *J--t=0.
rl-t
= Temperaure of the sink (Il).
Firct case , Solution. Refer Fig. 3.1.
vs v1
\-Tz _-6! =xi vL=r
Tr vr
t.., 6Tr- During isotherms, since compression ratio = e4ansion ratio
GTr= T,
or 5?, = 67" ot Tr = 1.272
vs
Second case : ...G) -Yz
Vr-vt
Tt-Tr_(70+27a)l | vt vl lx
TL _
-3 Also -vs
va-vt x;; =&x fr
Work done per kg ofthe gas
T1_T, +343 I
Tr -3 = Heat supplied Heat rejected = F?rlo;, t
3Tt- gTz +
- : -RTrlog,
LO2g = Tr

,, ,,#],=xT; _ilJlf (.''


= R(rt - r,) tog"
| n, (ft - )
= rt" i
Tt = 7'2 Tz)
or
2'47-, = Sfi - rc2g r, '= (r)'-l
,,
O.6T, = 1929

T"= = l?16 K or
But

.'. Work done per kg of the gas,


T2
=
[fl
and # 1442"C. (Ane.)
I
. . Exaup,e o^,,,"^lo:--",!;l,i-:;;::";"r;ffi"i;,,if}e
".u.min' rhe temperature o hroi-ilu,"" nw o" o n"o,
* = RTz 1r-r -1 og.
!!'Jiu:k'li!";frhr,t is reel K and thai of sinh is 8s0 K. rs
Differentiating l{'w.r.t. 'r' and equating to zero
Solution. Temperature of heat
Temperature of sink
source, ?r = lgg0 f
# = rr,f -' - t,{;,.,- ",-')} * r"s" 111 - r>r -'? t] = o
"
Heaisupplied, ' ?z = 850 K
fo*,"" auu"top"d engine, (/-r-1)(-1) +(r-1) x rr-2 ros"L =o
by the
The most effcient engine is one
: r:?iry^*
that " Carrnt qcle
*ork" oo
1
n T, -T, r99o - 850 - -z*!*-2 (y-r)log" =o
-
4 = --1ee0 = o'573 or 67'3vo
Arso, thermar efficiency
"rrJl;;;", ,r-r[-t* ..,'1r+(y-1)
' "l = o
toge -J

0.4
[
.'',h. =- _\91k done _ 0.4 x 60 I
H;rG;;,lied = CttTrr) -t*;7- +(y-1)roa | =o
whih is not feasibte as no e1i
ca." ;'::::;:::7,han that workins on carnot cycte. (t L
Hence claims of the inventor
is not true. (Ans.) - 1) lo& -l=0. Proved.
94
INTERNAL COMBUSTION ENCTNES
AIR STANDARD CYCLES
9.4. CONSTA'T VOLUME OR OTNO CYCL,E
ul
This cycle is so namcd. as_it was conceived by 'otto'. on this
cycre, petrol, gas and maoy 'Let compression ratio, r r-.r-
types of oil engines work. r b th sbndord of comiarison U2
for internl engines.
"iuurtoi
Fig' 3'5 (o) and () shows the theoreticalp-V diagram and ?-s diagrans
of tis cycle respec- g
tively. expansion ratio, r" (= r) =
t)g
The point 1 represents that cylinder is full of ai with volume (These po rotios ore sorne in this cycle)
temperature ?r.
Vr, pressure p, and absolute
| \'f-f
Line 1-2 represents the adiabatic compression of air due to whichpr,
V, and T, respectively, v, and ?, change to pr, As '2 - l,r I
T, \,2)
Line 2-3 shon's the supply ofheat to the air at constant volume
p3 and ?, (V, beingthe san as Vr).
so thatp, and T, change to Then, T"=Tr' ftf -'
Line 3-4 represents the adiabatic expansion of the air. During expansion pr,
change_to a finalwalue of p,vror y, and f. respectivety. V, and ?, Ts 1,,)t-t
;-=l-l
t4 \us,/
Ine 4-1 shows the rejecion ofheat by air at constant vorume t'l original
state (point 1) -r
reaches. T'= Tn ' Q)r
Consider 7 hg of air (rryorkiug subsance) : Inserting the values of T" and' ?r in equation (i), we get
Heat supplied at contant vol.-e c,(Tg_ T2), T, _7, .L- Ta-Tt
= flo
Heat rejected at constqt volu-" - =i__ =
tin _ . rr-\\-T,)
But'workdone ",
:;";:?ji"j;i;:'-?,";"' = 1_ :-:;-
I

(r)' '
Efficiency = c, (Tt -Tl) - cu (T -Tt)
=!e4!99- -
flear supptrecl This erpression is known as th'e o;ir stor.dard. efficiency of the Otto cycle'
cu (T3 _ T2)
It is clear from the above expression that effrciency increases with the increase in the value
Tt-Tt of , which means we can have maximum effrciency by increasing r to a considerable extent, but
-'- T"-T"
-i
due to practicoJ d.fficulties its ualue is limited, to about 9.
The n worh dnne per.kg in the otto cycle can also be expressed in terms ofp, u. Ifp is
expressed in bar i.. 105 N/m2, then work done

y=(o"ur-poro_prur-llx102kJ ...(s.4)
\ 1-I T-I )

Arso *=r= X
.. q=U=r_
P2PLP
where r, stands for pressure ratio.

and Ur= frZ = U rUg [.. o=.='l


= Luzu3l
- J
Swept volurne

Total volume--J
*=*[,,,(ffi-')-",.(f ')]

*1,,.[#-')-,,,,t* ')]
(rr-1-1)-pr i.r-l- r)]
(b) #[o,
Fis.3.5

T-
I I
I
96
r
NTERNAL COMBUSTION ENGINES AIR STANDARD CYCLES

= Solution. Bore of the engine, D=250mm=0.25m


tt[,"'-'-t)(p-pt)f Stroke of the engine, L = 375 mm = 0.3?5 m
Clearance volume,
-rxro -r)] % = 0.002ffi ms
1" Initial pressure, Pr=1ba
,*, ,ttlir
'...13.a
Mean effecrive pre.sure
Initial temperature, ?r=50+273=323K
=
". [*i -f. - *'i- ,^)-r,, -,rl * ...(3.5)

?{L1"1-r _t)(r, _1)


AIso
P-=
@t -rz)
ffi("t-'-r)(" -tr
,r-?
-&!1"r-t-r)(t -r)l
--------
,,lrl)
'\ r /
i.e., .- Prrfir7-t-f)(1 -f)
o_-
17_1lr_t) ...(3.6)
t.7. The efficiency of an Otto qrcle is
60Vo and T = 7.5. What is the
,oo ?E*r^ple compressinn
Solution. Eiciency of Otto
Ratio of specific heats,
cycle, \ = 60Vo Mamum pressure, ,, =::;:t
Compression ratio,
1= 1.5 Swept volume, V,= t14 D2L = trl4 x 0.252 x 0.3?5 = 0.O184 mg
r =?
, = L#=
Efficiency of Otto cycle is given by compression ratio, o'otiltf;T'ut
= r,
llouo=l-:+-
(r)1- I
(i) Air standard efciency :
The air standard efficiency of Oto cycle is given by

0.6 = 1_ (r)+:r rotto=1 ,h =t- r+- =r- *-.


or1 = 1 - 0.435 = 0.665 or 56,67o. (AnsJ
GIt =0.4 or ()oo= 2.5 or r-6.25 (ii) Mean effectivo presaure, p :
# =
Hence, compresson ratin = 6.25. (Ans.)
For adiabatic (or isentropic) process 1-2
Example B.g. An engne g[- zl0 nn tor" oriTZS. mm strohe works on P1VI = PrVrr
clearance uorum is 0.0026s-mr.
The initiar p-r"es"ll"4 Otn cycle..The
maximum pressure is timited to zs tunperd,ture are I bar and 50"c. If
bal, i'rl"|"h"ir, ,
() The air stand,ard efficiency
the
,r= o'(h)' = 1 x (r)r'4 = 1 x (g)r'r = 18.88 bar
of the cycle'.
(ii) T" *"on effective pressure
Assume the id,eat conditons.
for the cycle. .'. Pressure ratio.

The mean effective pressure is given by "=f:=ff6='au


prr[(rr-1 -lXrp -1)] _ 1x 8t{(8)14-1 -1}(1.g6-1)
,_ = (r-lXr-1) ...tEqn. (3.6)l
(1.4-l)(8-1)
98 INTERNAL COMBUSTION ENGINES AIR STANDARD CYCLES

8 (2.297 - lX0.36) or 0.72 (\- 689.1) = 1500


= -------:u.4x / = 1.334 bar 1500
- or '3 - +689.1 = 2772'4Yv (An'l
Hence mean effective pressure = 1.334 bar. (Ans.) "^ o.7z
Example 3.9. The ninimunt pressure and. ternperature in an Otto qrcle dre 700 kPo ond p2 ps - - pzTs - 18379x2772'4 bar. (Ans.)
,r= ,t - Pt= 7, =
= 73.94
27'C. The amount of heat a.d.ded, to the air per cycle is 150O kJ I hg. Also, 689'r
(i) Deternne the pressures and ternperatures at all points of tle air stonddrd. Otto cycle. 34 :
Ad.iabatic ExPansion Process
(ii) Also calculate the specific work and thermal fficienq of thc qcle for a compression z rY-l
ratioofS:1. 4-=[s{] =(r)r-r = 2.297
Take for air : cu = 0.72 kJ I hg K, and 1 = 1.4. (GATE,1998) T4 l.rsl
Solution. Refer Fig. 3.7. Giuen: pr = 100 kPa = 105 N/m2 or 1 bar ;
T Ts -2772'4 1206.9 K. (Ane.)
- 4=
Tt = 27 + 273 = 300 K ; Heat addecl = 1500 kJ/kg ; 2.279 2.297 =
r = 8:1i cu=0.72kJkS;t=t.+. /.. \'Y /t\La
I a pt="'|.?,J =4o23 ber' (Ans')
Consider kg of air. Also, p,u3r = P4v41 =za.e4' l;J
() Specic work and thermal efciengy :

specnc work =
"".,.u11]-,{:";,",":l _ r,, = c,r(rs_ ry _ g::-r!)
(1206.9 - a00)l = 847 kJ/kg. (Ans.)
= tlz\<zttlz.t_689.1) -
Thermal efficiency, n* = 1-
#:T
=1- = 0.5647 or 56.479o' (Ans)
-+-;
(8)"'-'
ExampleS.lo.Anairstand.ardottocyclehasauolumetriccornpressonratoof6,the
Iowest iiia ond. operotes betuteen temperature lmits of 27"c and 1569"C'
qrcle presswe of 0.1
0) Cotculote the temperoture and pressure after
the isentropc expansion (ratio of specific
ss = L4).
cycle operating cond-
() Since it is obserued that vohns in (i) are well aboue the lowest
process w@s ollowed' to continue d'own to a pressure of 0'1 MPa' Which
tions, the expansion
-ir"J"
Fig.3.7 i".equred' ti
compt'ete the clcle ? Nome the cycle so obtaned"
(i) Determitu by whai percenage the cycle eff't'cicncy has been
improued" (GATE' 1994)
(i) Preseures and temperatures at all points :
Adiabatic Compression process 7-2 : MPa= l bar;?' =27 +273
Solution-ReferFig' u2 vg =r=6;pl =0'1
3'8'Given'.}=3
'i /' \'f-I
; I =l?l
\%/
=1"'-' =(8)La-L =2.2e7 K
= 300 K ; ?g = 1569 + 273 = !842 ; T=
1'4.
(i) Temperature and prcsrure after the lsentropic elpansion' T4r P4 :
.l \ Tz = 3OO x 2.297 = 689.1 K. Gns.) air:
Consider 7 hg of
AIso pp{ = pzuzl Ft the comPression Process 7-2:

* =,,.(tl
a
lz=lrtl'=(8)La pru1l = pu] + = 1x(6f =123 bar
=18.8?e
Pt \uz)
:. pz = 1 x 18.379 = 18.379 bar. (Ans.)
Constant uolume process 2-3 : t, =lyr')t-t =(o)r.r-r = z.oa
Heat added durin the process,
Also Tt lrr)
cu(l- ?r) = 1500 Tr= 300 x 2.048 = 614.4 K
r
INTERNAL COMBUSTION ENGINES AIR STANDARD CYCLES
For the constant uolume process 2-J :

+=+
4 B
+ pr=hri =12.sr1842
ri2 614 = 36.9 ba

Fig. 3.9. Alkinsm cycle.


j,
- -t-1 *
L4-t

Fig.8.8
Now,
A=tfr)" or ?u= 184,
" (#) =65?K
For the expanson process 3-4:
li
r. rr. 1='1(9=57;3P']
amson -r - (1842-614.4) = 0.5929 or 89.299o.

.'. Improvement ln efficiency = 59.29 - 51.16 = 8.137o. (Ans.)


+:(f)"=(6)La-l= 2.048
Example 3.11. A certain quantity of air at a ptr;ssur:e of 1 bar and temperature of 70'C is
compressed, od,iobaticalQ until the pressure is 7 bar n Otn qcle engine. 465 kJ of heat per kg of
1842
^7"
' zo48
=<-
900 IL (Ans.) air is now odded. at cot sta.nt uolumc. Determine :
2"048 = (i) Cornpression ratio of the engine,
(ii) Temperature at the end. of compression.
Atso psqt = plvor * o. = r, , (iii) Temperature at the end. of hat ddd,ion.
[f.) Take for air c, = 1.0 kJtkg K, c, = Q./ffi kJthg K.
Show eah operation on p-V and T-s diagrans,
p =86.e, f+l'
\o./
=8 bar. (AneJ
Solution. Refer Fig 3.10.
(ii) Process required to complete
the cycle : Initial pressure, Pr=1bar
Process required to complete the cycle
is lie constant pressure scavenging. Initial temperature, Tt= 70 + 273 = 343K
The cycle js called Atklnson cycle (Refer
Fig. 3.9). Pressure after adiabatic compreseion, pz= 7 bar
(ili) Percentage improvemenUincrease
irrlffi"i"rr"y, Heat addition at constant volume, Q = a6S kJ/kg ofair
n -r I - 1 Specific heat at constant pressure, cp = 1.0 kl&g K
'rouo=r- (rt:f =t_ GF:T =0.5116 or 5l.t6qo. (Ans.) Specific heat at constant volume, c,, = 0.706 kJkg K

Heat supllied - Heat rejected c^ 1.0


lArki,"o, = -.YI\4#= -
ueat supplied ---.- r=;=o-6=1.41
Heat supplied
(i) Compression retio of engine, r :
- ",(rz-\).-"{4-T) =r_"r!.,r.: -Ir!=- t_
^ ?(4 -2i) According to ad.iabatc cotnpression L-2
q(Ts -T) cu(T3 -Tz) <E _f"
P1V1r = PrVt'l
INTERNAL COMBUSTION ENGINES AIR STANDARD CYCLES

(i) Compression ratio.


(i)Thermal efficency of the cycle.
(iii) Y{orh done.
Tohe'1 for air = 1.4.
Soluion. Refer Fig. 3.11.

p (ba4 T (K)

p-V diagram V(m.) T-S diagram s(kJ/kgK)

Fig.3.r0

Ivrl P2
t;it = -Pl
\vz J
-s - !.2 (v,)
Dt
l' h-') s(kj/kgK)

or --l.)i
r?'\* =#-: t tt
=
Hene compression ratio lr=r^l;:1,;l?rr.
Fig.3.ll
(ii) Temperature at the end
of comprcssion, T, : Initial temperature, ?r = 38 + 273 = SLLK
In case of ad,iabatc compression I-2,
Maximum temperature, ?s = 1950 + 273 = 2223If..
T2 lv, )t-t (i) Conpression ratio, r :
= l.J = (3'e7)r'4r -' = 1'76 For adiabati compression 7-2,
"'Tz 1.76 Tt PlVl = PrVrr
:: = = 1.76 x B4S = 609.? K or ggO.?.C
nence temperdture at the end of cornpressioz ggg.7g. (Ans.). (y'\' o"
\v") -
(lll) Tspslure at the end =
of heat addition, T, : nr
According to constant uolume heating
operation 2-S Po
oi = tu
Q=c,(?3-T)=4ob But "'(Given)
-\,
0.706 (?3 _ 603.?) 465
=
or \-608.7= # "'
(r)r = 15
(r)l'a = 15
[ "=#]
or ^ = 465 + 603.7 = 1262.3 K or 989.3.C 7
o,'e r= (15)il = (15)0.?11 - 6.9
Hence ternperature at the" end, of heat a.dditan 9E9j.C. (Ans.)
Example 3'rz' In a constant uorume 'otto cycre',.=
Hence compression ratio = 6.9. (Ans.)
the pressure at the end of compression s (ii) Themal efficiency :
at the start, t.he tempe-rature o oir"iil'n,
!_l:y:: th.at
tnaxttnum tenaperature attained. n the cycre
beginning of conpression is S8"C and 11
i 1s50". Detemine : Thermal effrciency, 4r = 1- (rf:T = 1- (e9F4-1 = 0.538 or 53.87o. (Ans.)
I
I

r,
INTERNAL COMBUSTTON ENGNES
AIR STANDARD CYCLES
(ill) IVqk {qs .

Again, for ad.iabat compression I_2, p (bar)

?[ _(vrl"-,
=
14., =
r.r_l-
(rr, =(6.e)1.1_1=(6.e)0.1=2.16

= 311 x 2'16 = 671'? K or 398'7"c


For ad.htic epansian
or!:r:1i," ''16
T^ /rr \7-l
d lJ= = (r)r-t =(d.e)oa=2.rd

-tn= T" = 2223 = 1029 K or 756'c


TR 2t6
Heat supplied per hg ofair
= co(Tt- Tr) = O.717(2228 _ 67I.7)

= f112.3 kJlkg ofair


I n 0.287
lc.=-=-.
1
v (*t)
l" T-l t.4-'
I

Heat rejected, per hg of ar


| =o.zrznrgr j

= c,(Tr-- Tr) = 0.7t7(LO29 _ g1t)


= 514.8:kl&g of air
.'. Worh done
= Heat supplied - Heat rejected
x =(+)'= (r)' or
'= [*) = (+)* = (11)0'714 - 5.5
= 1112.3 - 514.8
= 59?.6 l.I or 592600 N-m, (Anc.) '= (,)1-r
:"::::*jj":!::y:f
*.,rkins on otto qrcte has a votume of 0.45 nr, pressure I bar
Also
? =(+l = 1.977= 1.98

i#;:x::::'::;i{S T,liri:;r::i":i::ii:l:;;':;;;.x';ff"T;7"#,if,-Y,i,!:,
nlr
the pressure is 11 bor. zn Tz= Tt x 1.98 = 303 x 1.98 = 6fl) K. (Ans,)
ni,fx"',;;;';";;;;;iT;,xkf,r,i",il!,i?Wressi Applying gas laws to points 1 and 2, we have
(i) Pressures, temperdtures and. uolumes
at salent points in the qcle, ptVt
(ii) Percentage cleara,nce. _ pzYz
(ii Efficicncy. TL-Tz

+,X, u, = flffi
t, (iu) Net work per c1rcle.
(u) Mean effectiue pressure. ,r= = o.oel s. (Ans.)
1r
(ui) Ideal power dcveloped' by The heat supplied during the process 2-3 is given by
engine if the number or-working qrcles per minute
is 270.
:
.the
Assume the.qrcle is reuersible.
i
I
Q" = tn cu (Ts T2)-
I Solution. Refer Fig. B.12 I
ptVt 1x106x0.45
l Volume,{=0.45m9 1 where
^=ffi= ,8?-goa =o'517ke
Initial pressure, p1 = 1 6u"
210 = 0.517 x 0.71 (?, - 600)
Initial temperatr"",?, = B0 + 2?B = 303 K
Pressure at the end ofcompression stroke, p, 210
= 11 bar = o-iilT.z--f + 600 = u72 IL (Ans.)
Heat added at constant volme 210 kJ " "
= For the cozsnt volume process 2-3,
Number of working cycles/min.
=21O.
(i) Pressures, temperaures
and volues at salient points le=22
For adiabatic compression 1-2,
: /B t2
PlVl = prVrt Ta p, :
IL72
or= x 11 = 21.48 bar. (Ane.)
; n ' =
Vs = Vz = o.o81 m3. (Ans.)
i

-- =J+.*
TT_

AIR STANDARD CYCLES t07


INTBRNAL COMBUSTTON ENGINES

For the adi.abatb (or isentropb) prress 34, (b) Determine the ar-standard. effciency of the cycle when the cycle dzvelops n'Laxtmurrl
worh with the tetnperature limits of 310 K and 1220 K and, working fluid is oir. What wll be the
PsVsr = PrYrr percentage change in efficiency if helium is used. as working fluid. instead of oir ? The cycle
operates betueen the same temperature lirnits for maxnutn work dcuelopment.
P.=Psx (f)' ="' (i)' Consid.er that all conditions are id.eal.
Solution. Refer Fig. 3.13.
/r\L{
= Zr.a8 ' li\o.D,,f| = 1.97 bar. (Ans.)

Arso r _ lyr)'-t _=1.;i I r \"-' =o'505


f1)'-' _=l*,1
4=l%J
t' Tt = 0'505 fs = 0.505 x 1172 = 591.8 K. (Ane.)
Vr= Vr= O.45 m3. (Ans.)
(ii) Percentage clearance :
Percentage clearance
i'09!-
= t';= ' 16e =
r-*E -- ' 1oo

= 21.95%. (AnsJ
(lii) Effieiency :
The heat rejected per cycle is given by
Qr= mco(Tt- Tt)
= 0.517 x 0.71 (591.8 - 303) = 106 kJ
Fig.3.1.3
The ai-standard efficiency of the cycle is given by
(o) The woik done per kg of fluid in the cycle is given by
210 - 106
=W
,l*, -Q.-zt1
- = 0^495 or 49.6%. (Ans.) W= Q- Q,= cu(Tr-7")-cu(Tl-T)
Altematively : I '=
Iotto = 1-
11
= 1- ('sf----.:r =
0.495 or 49.64o. {lnsJ
But
?= [fi (r)1-l

Gt-r I Tr= Tr. (r)1 -7


(iu) Mean effective pes{ure, pr 3 Similarly, Tr=To. Q)Y-r
The mean effecive pressure is given by

P^=
Tf (work done)
- Q"-Q,
w=",1r,-!.(r)1 '-#.n]
V"(swept volume) (Vr -Vz) This expression is a function of r when ?, and ?, are xed. The value of Wwill be maximum

..
\ =,!',T-,t99lltot=
(0.,15 - 0.081) x 10" = 2.818bar. (Ans.) dw
=0.
d'r
(u) Power developed, P l
Power developed, P = Worh done per second +dr =-7,.(^t-1) (r)Y-2 -?s(1 -T) (r)-1 =0
= Wort done per cycle x Nurnber of cycles per second ?, {r-r =Tr(7)t-2
OI
= (210 - 106) x (210/60) = 364 kW. (Ans.)
' Example 3.14. (o) Shout that tlv compression rato for the marimum work to be d.one TB ,-\2(y-1)
per hg of ar in an Otto qtcle between upper and lower lmits of absolute ternperatures T, and. T, ry = \')
is giuen by / m \U2(^r -r')
I l. I

r= Pr,ved'
--(bl'"'-"
'- l.4l l4l
I
, l0g
INTERNAL COMBUSTON ENGINES
lr' AIR STANDARD CYCLES
() Change ln efciency I
For ar .l L.4
= Tz=Tt= Jryr. Proved.
I /^\v2(L4-1, /--- \t/o.E () Power developed, P :
.=l1.:l
'=lrr) _ [L22o\'
=|.sloj =tM 4=310K I
l The air-standad efciency is given by 4 = ra50Kl ....(Given)
m =oe kgJ
nouo = 1 #- = t - **r= = o.498 or 40.67o. (Ans.) Work done W = c, (?s - Tz) - (Tt- Tr)l
I
lf helium is used, then the values of
Tr= Tn = lEn = 1610 l45o = 6?0.1 K
cp = S22WikCiK and c, = g.1g ktkC K "
li W = o.vL K1450 - 670.4) - (670.4 - 310I
... c" 5.22
Y= =t'sz = 0.?1 (?79.6 - 360.4) =297.6kJtkr,
ii
ii
*=g,' Work done per second = 297,6 x (0.38/60) = 1.88 kJ/s
The compression ratio for 1o!-u6 work for the temperature limits ?, and ?, is given by Hence power developed, P = 1.88 kW. 6ns.)
- -', Eqmple 3.16. For the same compression mtin, show that tlw fficiency of Otto cycle s
,= (+)"' " ry1u',u,
1.4/ =
[sroJ = 2.77 greater than that of Diesel qcle.
Soluion. Refer Fig. 3.14.
The air-standard efficiency is given by

Ioto=l- =0.49d or
,h=t- r9.59o.

Hence change in elficiency is nil. (Ans.) "#-


Exanple 8.16. (a) A engire.workir* on otto cycle, in whih the
sotieni points are 1, 2, S
and. 4, has upper ond, Iower temperature tiiits To and.i
be done, show that the intermed.iate temperatui is giuLn r. If the mo*mum-*""i"i'ns of air s to
by
Tz=\= JTtTs.
(b) If an erqire
works on otto cycle between temperature rnits 1450 K
ad gI0 K, find. the
maximum porter deueloped by the engine assuming tie crcur.atinn
of air per ;n;" * 0.Jg hg.
Solution. () Refer Fig. B.tB.(Example 3.14).
Using the equation (iii) of example 8.14.

w=
"1"
",1r"-n.,"n-, (rr'- --h.r,l |

and differenbiating Ww.r.t. r and equating to zero Diesel cycle

/a
I Ir
\u2(t-r) I
F8.3.r4
r= lrl
l?r / We know that

= r/(r)"r -L rlotto = 1_r+_


substituting rhe varue *, ,lio=l'ot I:*i"3
-' -
r"*r=r- 1 1fol-rl
,, = r,V+)'n-
"]' = n(?,)"' = lM G)Fxrt-j
As the compression ratio is same,

Similarly, ,-=
vr vr'
v"=W='
oft_,ry"=#=Jrn v"' rr, then cut offratio, p= v"' r
ltrl J 14 J O
t= =;
INTERNAL COMBUSTION ENGINES
11t
Putting the value of p in 4u,*,, we get

l"" = I -

From above equation, we observe


lrt
11

Let r, - r- 6, where 6 is small quantiy.

66263
Then
i=*==('-i)-'='. -+-+-
,1213

Fig.3.15
volume increases from V, to V, and temperature Tztn ?3, conesponding to point 3. This point (3)
is called tbe pont of cut off. The air then expands adiabatically to the conditions pn, Vn and Tn
fr.6 . r(r+1) 62 respectively corresponding to point 4. Finally, the air rejects the heat to the cold body at constant
... rr""c,=l--'il#
.

volume till the point 1 where it returns to its original state.


Considr 7 hg of oin
|
Lrr' -+-;+...... Heat supplied at constant pressure = cp(73- Tz)
Ileat rejected at constant volume = cu(Tn- T)
f9*r+r 62.
='-#='l#l
L;+P+"""
I

J
Work done

n..
= Heat supplied - Heat rejected
= cp(Ts- Tr) - c"(To- Tt)
.= Work done
'drese Heat supplied
The ratio inside the bracket is greater than 1 eince the co-efficients ofterms /r2 is greater
than 1 in the numerator. It means that.something more is subtracteil in case ofdiesel cycle than _ ci(Ta-Tz)-c,(Tt-T)
in Otto cycle. co(Ts -?12)
Hence, for sctme cornpresson ratio 4o* > n.
-, (T4 -Tr) l.t :e=t
= '- rt4lD "'(;) lr'
3.5. CONSTANT PRESSI'RE OR DMSEL CYCLE L-ul
' This cycle was introduced by Dr. R. Diesel in 189?. It ffers frtm Otto cycle, in, that heat Volume at cut - off
Let compression ratio, r = 5 and cut offratio. o = !3- i.".
is supplied at constant pressure instead ofat constant volume. Fig. 3.15 (o and ) shows the p-V u2 u2 Cleaance volume
and ?-s diaglarns of this cycle respecvely. Now, during a.d.iabatic compression L-2,
This cycle comprises of the following opeltions:
Q) 7-2..... Ad.iobatic compression. ?7\= lo)t-'=(r)r-r or rr=rr.(")r-'
(i) 2-3.....Addition of heat at const@nt pressure. 1,, )
(iii) 3- 4..... Ad.iabatc erpansion. During consont pressure process 2-3,
' ^:
=p.rr=p. f,.
(iu) 4-l.....Rejection of heat at consto,nt volune. lc U.
or ?, (r)1-r
Point 1 represents that the rylinder is full ofair. Lct pr,V, and ?, be the corresponding t = t=p
pressure, volume and absolute temperature. The piston then compresses the air adiabatically (i.e. Dunng adinbatb expansion 3-4,
pVY = constant) till the values become pr, V, and, T, respectively (at the end of the stroke) at point / \1-1
2. Heat is then added from a hot body at a constan pressure. During this addition of heat let
ts lu |

T4 l.rrj
INTERNAL. GQMBUSTJON f NSINES 113
/ \ltj1
lr I. (... n=o=st,.4='rl
\P,/, \ us ug u2 ug p)

...(3.9)

= pquo(p - a PgPuz-.pt'ttz P*z- Ptntz


T_l , -
1
T_1

Fis'
''rG
-
Compression *tio,
l \
v-
' = rs
l= fJ
lfor air = 1.4
=(:)'=r" '1 Air standard efficiency ofdiesel cycle is given hy

l ' I fot-r-l
ser=r-thr:Tlr-J ..(,)

|L *=lr.')'
P, \uz) ", Pz=Pt'rrandS=r or uz-utr_,I
-
where p =
",rt-of.atio = f
] j-
prurrl -1 h(p - l) - rt -r (pr - 1) f^-v^=
zloo V" (V" = stroke volume,
------- I
(1 - 1) ,(3.8)
I

I
f
?'
,ri( _
AIR STANDARD CYCLES ll5
114 INTERNAL COMBUSTION ENGINES

rai*a=1
= 0.06 (Yr - Vr) = 0.06 (15
[,
Vz- V2)
*h[H]='
...
= 0.84 Vz or V, = 1.${

o=Y,=s=LYvz =r.r^
'Vzv2 When the fuel is cut-off at 87o.
= 1 - 0.248 x 1.563 = O.6L2
we have
"*n-'lq#]
or 61.2%

Putting the value in eqn. (i), we get P-l 8 P-l R


or ffi=*o
1
= t* =o'oe
-
\"*t=L-
l{r.an)tn-rl' ,'' = P=1+1'04=2.04
l-4GIr=fl 1s4L l lpt-rl . l
I
_i
ldi"""r=r itz.oto-ll
=I- 0.2417
' r.e = 0.612 or
81.2%. (Ane.)
J=t-;"t=-L m4L I
Example 3.La. The strohe and cylinder d.iameter of a. cornpression ignition engine are 250
mm and 150 mm respectively. If the clearance volume is 0.0O04 m3 and fuel injectian takes place = 1 - 0.248 x 1.647 = O.69f oi 69.17o. (Ans.)
at constant pressure for 5 per cent of th.e strohe determine the effuienca of thc engine. Assume the Hence percentage loss in effrciency due to delay in fuel cut-ofr
engne working on the d.iesel cycle.
= 61.2 - 59.1 = 2.1%. (AneJ
Soluion. Refer Fig. 3.16. Eranple 5.2O. The meon effective pressure of a Dicsel cycle is 7.5 bar and compression
Length of stroke, L = 25O mm = 0.25 m ratio is 12.5. Ftd the percentage cut-offof the eyle if its initial pressure s I bar.
Diameter of cylinder, D = 150 nm = 0.15 m Soluion. Mea effective pressrre, p- = 7.5 ba
Clearance volume, Vz = 0.0X m3 Compression ratio, r = 12.5
Swept volume, V" = ttJ4 L = /4 x 0.152 x 0.25 = 0.004418 mg Initial pressure, Pr=lbar
Total cylinder volume Refer Fig. 3.15.
= Swept volume + Cleaance volume
The mean effective pressure is given by
= 0.004418 + 0.0004 = 0.004818 mg
P^=
prrl[T(p-1)-rr-r(pr -1) ,,.tEqn. (8.9)l
Volume at point of cut-off, Vt = Vz + V, (7_r)(r_1)
ft
- L1 (pt1
1 x (125)11 [1.4 (p - 1) - (125] - 1)
* ,0.004418=0.fi)0621 me
=0.0004* 7.5 =
(1.4-1xr2.5-1)

.'. Cut-off ratio,


% 0.000621
p=il.=-=r.Do 7.5 =
3433[1.4 p - 14 - 0.364p1 + 0.364]
4.6
7.5 = 7.46 (1.4 p - - 0.364 pl1)
1.036
Vt 0.0041418 + 0.0004
--
'-
Compression ratio,
v2=V"+Vz
v2
=
0.0004 = L2.O4 1'005 = 1.4 P - 1'036 - 0.364 Ptr
2.O4= 1.4 p - 0.364 pr'a 0.346 pr'{ or - 1.4 p + 2.04 = 0
r [o1-rl - r -rl
ltrslln SoMng by trial and error method, we get
Hence, uut =I - y(.)rle-t l='- *;*F =l rsr-r I p = 2.24

= t- x 1.54 = 0.593 or 69.87o. (Ans.)


O.264 Vo cut-off= r t* = # x irXl = 1O.784o. (Ans.)
n\nple 3.19. Calculate the percentage loss n the idzat fficiency of o di.esel engne with "-LJ
compression ratio 14 if the fuel cut-off is dlayed. from Vo to 8Vo.
exlvcnple 8.21. An engne with 200 mm eflinder d,iameter and. 300 mm stroke works on
tlleoreti,al Di.esel eycle. The intial pressure and temperature of air used, are 1 bar and 27"C. The
Solution, Let the clearance volume (Vr) be unity. cut-off is 8/o of tle stroke. Determne : t
Then, compression ratio, r = t4 (0 Pressur* ond. temperotures at all salient points.
(ii) Theoretical air standard. fficiene1.
Now. when the fuel is cut-offat S4o.wehave
(ii) Mean effective pressurc.
p-l 5 (iv) Pouer of the engine if the worhing gtcles per minute are 380.
r-1 = 1oo o
fr| =0.05 or p-l=13x0.05=0.65 Assume that compresson ratio is 15 and. working fluid. is air.
P = 1.65
. Consid.er all conclitions to be ial.

I
il INTERNAL'CC'[SiIdi IDNOTNE$ 1t7

ii i For the ad..abatic (or sentropic) process I-2, : r.{i ,.,.rri,ir.r..i r:

,..t t.j::. , ' ,j,.. /t \r


.i,j : i!:1,: j
. ,, 1{,t1,n{zj e' ,f ={f,,J =t"t'
l]g l-- ri

i: ii = i,1 .,.

ll_ 0.00e42
Hf(rAns.)
,' ;I.; ft; tnJ:,
",!t$i:g;!Jffi
_
= b366tlB
1
;i;' t'i'}
,i.1.,. ) t ;;t;,..
^
-p-=!b-j
p;-=p-.=-44;81 bg..v i*r., :'

E ctfif iaiitJ
:t': i' ,:,.r:i'1 r 'ft-.,{';:, :rii:' .'
:',r,:r,
?t.,i 'iS: j.i. .. i\l-,-i ]i;l'a:!, {r|\nii. .: ,ii1 } .:
8 :'r)r :''- :':
': Pl+-!
I- ir " : l
' 100;',15=1' i, ,

t:e. ,'t
.l]..''!i.'f,\'i;i.,:p:-5ry.6gi\r1=i.t
fltlilitit!l\i!
-'r.a
\',ij ,:tl i,,:ii1,.. r%:
'.iitiril.\':
:'z:rz* b.Oooetzs:'o.0oi4% ms.
11r:.,ir: i,!. q1i.11,1-1,r\: ...ri.1,. .
(Ans.)

{.4) il;i,a ;r: .'t;,11-.:. :,'i., fvs


can als r'calcuta$ SfffrhVe:-. . .: l
,- ",,'
Fis.S.
:
/, : r.: ii :, lyi ='.efrr ..4, = 0J8 x Oi0094? + 0i00O6?38 = 0.00 1ae6 m3 ]
For the '", :, 1'.| ,
:
,
Cylidder diameter, D=ZOOmmor0.2n ,'i ' t r\"iiianttli,&iiEpt'bls'z.}, r:
Stroke length, ' Yi;i"t ':':' :i " "
Z=300mmor0.Bm
Initial pressure, P = l'0 bar
Ts Tz
Initial temperature, Tl= 27 + 273 = 30O K Ts=rr, v. 0.001426
= 1878'8rL (Ane.)
=886'2x
Cutolf 8 For the isentropic process 3-4 "
= i' Y' = o'08 Y' psVsr = paVaI
(i) Pressures and temperatures d salient polnts :
Now, strok volume, V,,='r44 D2L = rl4 x o,22 xb.3 = 9.66942 - -..(v,'l' 1 f.
v4
=v4 xv2 =Lrb
v3v2vsv2vsl I
-t e{
lJ
x
=,Ds x
(?tT = PB

Vr=V,+V"=V,+ jj\/f v.=l--l


t" " r-L) =ffi=r*'bar. (An*) t
I
=i,' u=v=fi=tot)
=v ft*-L);- rn _ (vr\'-'
":rJ=;'%
16 .,,, 15 .:.
Also,
4 [%] =l?,?J
= =ll_)'0,-' =o'457
L..
Y, _
,,
i
.

'
iEJ*% =fr x 0.00942 = 0.0101 m8. (Ans.) Tr = ? t 0.457 = 1878.3 x O.457 = 858,38 IL (Ans.)
Mass ofthe air in the cylinder ca{r be calctlated
by usrng the gas qquation,
V=%=0.0101 m3. (Ans.)
(ii)'Ttreoretical air standaid efficiency :
PTV.=
^37,
-= Vt=lxlou"o.010t T"ne, =' *hi#]=' o#-
= 1 - 0.2418-x-1.663 = 0.598 or 89,87o. (Ans.)
l#l

I
I
lt8 INTERNAL COMBUSTION ENCINES AIR STANDARD CYCLES
(iii) Mean effective pressure, p : Me effective peaaure' pn :
Mean effective pressure ofDiesel cycle is given by Work done
_ _
P- =
by the cycle.
--3*6fioid-
Pl(r)l[fo-1)-rr-r(Pr -1)
^ - ----
'- (i_lx_lj-- Work done = Heat added - Heat rejected
x (15)u[r.4(2.12 (1s)1- L4 (2.1214 Heat added = mcp(Tt- ?), and
_ 1 - t) - - 1)]
(1.4-1X15-1) Heat rejected = mcu (Tt- T)
Now assume air as a perfect gas and mass of oil in the air-fuel mixture is negligible and is
_ aa.3rt1.56t:_0.9,,,,,,,,,,,,,,,.38 x t.8631 not taken into account,
= 7.424bar. (Ans.)
Prress 1-2 is an ad.iabati.c compression process, thus
(iu) Power of the engine, P :
- ,, r1-1
workdonepercycle
Work done per second
=p^v"===*ft*t =6.99kJ/cycle rt \Y2 J|
+=l+ o" rz= rt,
? = 300 x (15.3P'a = 893.3 K
Iar (since 1 = 1,4)

= Work done per cycle x No. of cycles per second


= 6.99 x 380/60 = 44.27 hlls = 44.27 kW
Hence power of the engine = M27 klf,. (Ans.)
Also, p1v{ = p2v2r pz= pt"
- f+lt = 1 x (16.s)1'a = 46.5G bar
Example 3.22. The uolume ratios of compression and, apanson for o d.iesel engine as
\vz)
tneasured. from an indicator diagram are 15.3 and 7.5 respectively, The pressure and, ternpera- Preess 2-3 is a constant pressure process, hence
ture at the beginning of the compresson are 1 bar and 27.C,
Assurning an ideal engine, determine the mean efftzctive preasure, the ratio of marimum '12 'ti + r^=u-{'
2=3 " Yz
=r.onx8es.3= LB2z.sK
pressure to meo,n effectiue pressure and, cycle fficizncy, Assume that the volume at point 2 (Vr) is 1 m3. Thus tbe mass of air involved in the precess,
Also find the fuel comsurnption per hWh if the indiroted, thrmol fficiency is 0.5 of ideal
effiiiency, mechanical efficiency is 0.8 and. the calorific value of oil 42000 hJ lkg. l'. Y
l =vt =Lrb
Assutne for air : cp = 7.005 kJlhg K; cu = 0.718 hJthg K "f 1.4.
= (u.P.s.c. 1996) -=*=tf;;i#'
Rrz =,777w l"_ L 'n"'_l;,:.^.1
=rr=z.otl
Solution. ReferFig. 3.18. Giuen, !v2= fS.e ' 3vs =r.U 1". =.
Pless 34 is an ad.iabac epanson process, thus
Pr=lbar;Tr=27+273=300K;r*,r,=0.5xr*-"*,ui4necr,.=0.8;C=42000kJ&g.
The cycle is shown in the Fig. 3.18, the subscripts denote the respective points in the cycle. t' l]h')t-t =ll)"-' = 0.4466
Ts= [Vrl U5/
T = L822.3 x 0'4466 = 813.8 K
"' work crone ==T?rfirolu'r;#i:t1l;rfi-
0.?18 (Bls.B - s'o)r = 100a6 kJ
Workdone 10035 10035 10035
P- = ffiptiJi = (vr-vrr= erst/r4rr= 148
= 701.7 kN/m2 = 7.017 bar. (As.)
('.' Vz = 1m" aseumed)
Ratio of maximum presrure to mean effective peasure

=6.4e. (Ans.)
=
X=m
Cycle efficiencyr lla"a 3

Work done
l"y" =
Heat supplied

Fig. 3.18. Diesel cycle.


l
NTERNAL uot{BosrloNrENctNEs
AIR:-STI(ND*ID;,G1EIH i r/ :j:]'j ri.ir
1009_ _ 10095 : 6e cvileslls
rrl.
= ;;64 =
@-;r;lebb
tearra?e "l'J-.48%.
rBsM.
(Ans.) Considr I kg of air. .i.:g *vr .riit ;;0i"+:;;ls lri ,ll t-, .,uilr sr-:l glliliut
FueI consunption per tWf,
i,glo,, lqruE- Total heat supplied = Ileat supped duringlhe operation 2-B
rtr,oi = 0.5 rr4" = 0.5 ,Iuto,&yrtry.Hoz4sfth"
bg:[y, i;!G ;i.!rj;/ - during the operation B-4
It<ar = 0.3bg{ lsnf"r$i?4ir = bsbbstsel{
'r*,Eqfrsupped V
- nT)
ai bas4l8gitgan , s'..amrtrn'
ei grurxic-I I'r . .- -
-Al "T - -^ 3oo0 t'sr"sici rnsH
H;F&=ffif;Sil#iq::ff
Heatrejectedduringoperarion;?!:;l:r.?"$'-4'
work done noi"Jiu ir;.fl**a
*,nl;,,
riionib tnj 3i s-!' ?\)olq ",
=

or r-Irl,,,o.rnr'-1tl@foiazsrqmor
(S.I = y o:rrriz) | L". l ,. . - 14 xa2000'l :t\
or ial^,.='"1[* l.ij-,-i "T'

ir18ffi se2-trIqEco0re0*3*514 kskwh. (Ars.) j'.,r ,;r,r+Jffi


iQtzii!1,I*"" (r4-r3)
;i tetu =--+++-
cr(76 -T1) I c-\
...G) lr Y=. 1.
,j',,.',(Tt+721,+'(Ta+\)
,,.;
"= "

, -,i ,,.Jnq.1:
.,, .)..!. , - 12., ,i -,. ,; ..,

i ri _a. p,
(u) -L-Rejection of heat at constqnt, rolu^". , . ' "
,, = ;, = where pis knowl aa pnessune or erplocion rao,
.,: ,o-!-!t'r,_
'2- a, ...(iii)
Dunng adiabatic epagsion p.roc*s (. ,
T1 lru'\t-t
;=l;l
1l \u4 /
, .tll
_ trl ...(iu)
'\p/
,f'' x lz lt lc l,
' \. 3'$t- +
Vt= +
tk u4= u2x u4= p''p being ratiol
--- -- ---'-/
-- cut-off
-----' the
During consant prcEsure.heating proceis S-2,
u"='n ::
Ta Tt
.I
i,=7.\=oT^.
Fig.3.19 Putting the value of fn in tl. uq*tiolttirl, *; ,*
z rY- I
f=6)'-',' or t,= r r. [+.,l
INTERNAL COMBUSTON ENCINES
AIR STANDARD CYCLES 123
Putting the value of ?, in equation (ji), we get
Ts
anple 3.23. The swept volume of a dizset engitu worhing on dual eycle is 0.0053 m3 and.
clearance uolume is 0,00035 n3. Th ma.timum pressure s 65 bar. Fuel injection ends at 5 per
F -rr-r cent of th stroke. The tmperature and pressure ot tlu start ofthe comptession are 80'C and' 0'9
Tr bar. Determin the air stondard effteicncy of the qcle. Tahz l for air = 1.4,

= 4t
Solution. Refer Fig. 3.20.
p-'tr-t
Now inserting the values of "Z:1, T2, ?. and ?, in equation (i), we get
| (" 1)
-
rlu = I - _,
---I
Gt:rle'
tl
e.,

T(e - r)
l\1- p.,l+
L., tlo*r=1-",! ...(3.10)
Work done is given by,

1r=pr(ua-ur)* ry
=psu3(p -r, *
@ova - pnz):!p2aa - pLrus)

_N
tl
p',rte -rxr -u. p,,, t^I
lr_ V. = 0.0053 m
e.,,, - -
_ [e f")- (1
#,') Vs = V. = V. - 0.00035 m-
7-l Fis.3.20
Also zE=[,.')'=fef and p, =("\\' =., Swept volume, V = 0.0053 m3
P4 \u,/ \r/ h lrz)
also, Clearance volume, V"=V"= Vz = 0.0O035 ms
Ps= P$ ut = us, u5 = ul Maximum Pressure, Ps = P1 = 65 ba
b[p5 (p. 1) (y - 1) + ps (p - prrr-1) - p2 (1 - rr-r)
w= Initial temperature, ?t = 80 + 273 = 353 K
(y-D Initial pressure, Pt= 0.9 bar
pPe[9(p -1)(T -1) F(p - pll-]) - (t -r1-r)
+ flot - ?
- (Y_1) The efciency of a dual combustion cycle is given by
_ a(rIqtfPr(p -r)+ (F - r) -rt-l,(Bpv- r) _-1- F,'Pt=-,t ,,.|
_ .t 'f,.
.,rduar ...au
Y-1 (r[_1 ltF_rl* Ffp_l)J
(
- Aq/-rtFT(p-1)+(p-1)-rr-\pp1-1) ...(3.11) rario, ,=
o'o0x1if;0J03u
= re.rn
Mean efiective preasue (p-) is g1ven by,
"t -1 compression
+"="#=
t,' Vz =V" = Clearance volumel
w w arur[rr-1gr(p-r)+(p-r)-rr-r (Fpr -r)
P^= l=--1"4=-
",1;/ (t-t)*[+) Cut-offratio, p=
vs vs vc
V2=Vs =Vc)

pr(r)Y[9(p-r)+(9-r)-rr-r (ppr
o
E- (r_tXr_l)
-t)] ...(3.12)
0.05x0.0053+0.00035
= 1.757 say L.76

t
r"24
INTERNAL icoritBftlslf4rt*Gtr{Es
AIR] STANPARD OTO.D8 J,/r If f !TI4 i
leonrprusp_Jffiqpeacr{gna bA:h.i,,r, :ririiro,:
l:.::"r:r.Y.i9**sit\g
r6r'c:!) 5ri' ''oI::si-ni l':u;v_rred[.Vrai r....!,;:, si\
.tX,* sqrcexit
Initial pressure, ii a.iqFJ{b.! x 0 ', 80.::.il x ,T = ui
I.i.ri', tr'r'.)'(lf\ 'r"o .r.,izz."(sri.--i 91ri.3,ri rn.,,,n,rr,,,, r(-i, r;1. .,..rt(!-).\)..1
:,n,::,,,\:i..rjr:.
or -'t = sl\!{,:lrie lil} ln:.rrr,,:rli i,.,;,,r)i,,,r.r..,lilt:,i,T ..i,.:t, y:\, r.., ,.
*=t
,t}trb:R1'linr:,',t";;i1;"i=
u. 1"*i"*1 '," .,,
Initial temperature, f, = 30 + 273 = 303 K -?-1i ?.?r:')"q :rnsutr;
Maximumpressure,
Cut-ofr .
pg=p.=5"t'
voluDe .s-l=
tsrt'..1r) er.") '!i)';l

or = 4% of shoke 3,{ sq
Number of working cycledsec'g 3.
'i :r .tSi' = r.,. T. -_ i
B= !c= 95= = f.Z
Pressureorexplosionratio, 1{ S0S = =;;
(bar) I r-r.l: '.t
puningthevalue | .l-q
ofr, p anit p h:il"f":'dr, ;;r
r,
*-.+-'i.l
i

c.fj util l- 1

- t" g
t.I = q ru $U,U = ';-
IE
;
.'itt1q lliJi;lYIq :i 11 ) ! a i!r'-r

ratio 14 and, the explosion ratio

soluion' Refer Fip.,,sn]-p,l*"*{l


----- ; [;\'." rJ i,rl$ l..
ratio, 1' v)
Comr,r'ession |
it-' - -. I
- . :- . I .a \ ,oiler roierr*qxs ctir1-
r?.:\r!i'i(: )!i$(\:)iLD aO t
{s.l-r. l
P = r.78

j..,.ii,'i.,; ; .l-1

. r.',':::i')1 ;,::: t',


,,,-p*"-p,e3.25rhe,"*;;Hm;,y;;::;::-:#",du,a,cyc,e
is e. The maxinum pressure,in the ': )i0.', rt
cyti"tu i;il";"t1b!.to;, rh";;;r;;;;;";ftumper4ture
the air at the besiniing of tle ct!1a11 t"i i#rt.b. of
od.d."d. during connt pressure
e =r
-r %,-.,
f:r, ,!:
tl:q,-ti"":;o;,;; ;;'fii
';ff::;3:;,':;' :":;';:l:!;;"t##i! ,uo
*
o%tf.;
.',., :,..(i\.{!re : air pn4d,ar@ effigiency of the qcle. ^*,^ ^ = g oplgs,i",, .,,, ,,, *r..', ..,.,r,
r;
(ii) The pou:er aer"i.n"1.-iytny_numbei Vt= V"+ Y"= 0.0L47 + 0.0018' j 9.666:ie ;'
of worki4g cycles are B per second,. For the ad.inbatic Q1 iyn!rop!g),.899esg 1,Q,
, '.,- air cu = Q./7 hJlkg:K a@ : cp r.0 O.liW*.., ;

Tg!".f!,
Sohtion. P=
' ..-.:' l
Refer Fig. 8.21.
Cylinder diameter, D = 250 mm = Q.!g
--:,'i'1;'.
Compression ratio; r -.. ;;g I , I r'r'' :' :.
Stroke length, . = 300 mm = 0.3 i,r '

I
t

I
121
AIR STANDARD CYCLES .
126 INTBRNAL COMBUSTION ENGINES
cExample3.26.InanengneworkingonDuoleycle,thetemperatureand,pressureatthe
Tz= Ttr
2.408 = 303 x 2.408 = 729.6K compression r'at9-::',The maximum
beeinning of the clcle are 90'C ood I bo' respectiuely'.Theper ig of air is 1750 kJ' Determine :
For the cozsont uolume ptzr,es 2.9, i"""ri7 i iid. to ea a*-""1otit n"Lt
"uppu"a
Ts (t) Pressure and temperotures at oll salient points
=T" (ii) Air standard efficiency
hh
60 (iii) Mean effective Pressure'
To=T". l
pz =?29.6, F =20zox Solution. Refer Fig' 3'22'
p-l 1
Also,
J=lgO or 0'04
p-1
9-l =0.(X c e=1.32
For the consfnt pressure prws 34,

+=+ * t=* ='


Tt= fs x P =2fl?oxL'32=2666'4K
'' v- V, t
Also exPansion ratio' =V5 xVt
vr =Vr,.
v2 vl = p ['.' V =Vr andV2 =V3]
r v2
For ad.inbatic process 4-5,

a=fyrI-'=ls)'-'
Tt lvsl \"/
r=rtx(:l'=2666.4i (T)'' ' =rz37r
Also PlVl = P"Vtr
FE.3.Z2
/v. Y I'Y lm2\'n
ps=pt.lJ =*'l.oj =60'[.9J =4.08bar itial pressure, pr=1bar
Heat supplied, In\al lemperature' ?, = 90 + 273 = 363 K
Q = co(? - Tr) + cr(To- Tt)
Compression ratio, r=9
= O.It (Wn - 729.6) + 1.0 (2666.4 - 2O2O) = 1562.58 kJ/t<s
Maximum Pressure' Ps = Pa = 68 bar
Heat rejected, Q,= co(T- T) = 1750 kJ/kg
Total heat suPPlied
= O.7r (1287 - 303) = 663.14 kJ&g
(i) Pressures and emperaures at salien points :

For ttre isenroPic Process L-2,


\ l-ttla<t = 0.6766 o 67.56%. (Ans.) prVl=Prv]
() Power developed by the engine, P : . .,y
rt
(Ans.)
1x (9)14 = 21'67 bar'
Mass of ai in he cycle is given by " [ir) =lx(ry =
p=pr
rx195 x9..0-roo
^ = +- = 0.0189 kg '
Arso, T2=f]i)'-t =(r)1-r =(s)La-1 =2'408
.'. Work done per cycle = m(Q,- Q)
i'fU K' (Ans')
.. T2 = Trx 2'408 = 363 x 2'4Og = 814'l
= 0.0189 (1562.58 - 663.14) = 16.999 kJ bar' (Ans')
Ps = P = 68
Power deueloped = Work done per cycle x No, ofcycles per second
Fo the consont uolurne process 2-3,
= 16.999 x 3 = 60.99 say 61 kW. (Ans.)
128
INTERNAL coMBUsTIoN
ENGINES
AR STANDARD CYCLES
PL 129
=b-
724
b :' p- = h @v" -v !!sr-&J lz - *'"=,
Heat added at constant voluoe
T=?x =874'7.
# =2742pr( (Anc.) -il
r=9,p=1.16,y=1.4
i
+
-?*.f
- T2) = o'71 (2742'9 - 874'1) = 1326.8 kJrkg Pr = I bar, pz = 21.67 bar, p, = pn = 6g bar, p6 = B.g1 bar
.'. Heat added at consranr,xs Substituting the above values in the above equation, we get
= Total heat added _ Heat added a
constant volume 68"1i*_sr.B1*e
,.
"r("4 "B)
I,O(T|_
- :
l75o - t326'8 = 423'2 kJ/*c r_ = o!loacrrs-tr* tr?l
.\
= f tro.z + 109.?? - 91.6?) = 11,04 tm
2742.9) = 423.2

;^- T = 8166 K. (Ans.) Henen, mean effective pressure


ror consta,nt pressure process B-4, = 11.04 bar. (An*)
Example 8.27. An I.C. engne operating on the dual cyck
Aimited pressurc cycle) the
temperdture of tlw working flu'd (air) at the beginning of comjressian
l
,=+=+=ffi=rra maritnum and' minimum pressures of the qrcte * zO aid compressian
is zz;c. The ratin of the
ratio is 15. The amounts of
For adiabatic (or isentropic) process --'-'- heat added. at constant volume and, a.t constant pressure are
LS, equar, compute the air standard,
t!.e1mal efficiency of the cycle. State three maiti rea,sons why
the actucll therrnal efficiency is

AIso
+=t-+=#,"+=; d,ifferent from the thoretical ualue.
Tahc y for ar = 1.4.
PqvaT = povur
(u.P.s.c. 1997)

" P5='.,[F,J =",(:)'=ee;ffi'n=s.s = =


PUI
Solution. Refer Fig. 3.28. Giuen : T, 27 + 2ZB 3(X) K; '.-
u2 u3 = fs
?l =70, -=-'

Again, !L l&y-'-lp)t-l
11= (vu) -t;/ =l./r.15\11-r
,j =o'4ge
t T, = T. x 0.4tg = 8166 x O.nr,
(ii) Air standard efciency
:
= ,rrr., K. (Ans.)
Heat rejected during constant
volume process 5-1,
= ,, = 0.71(l38e.8 _ s6B)
.'. "f:ru,_ -Tr) = 72s kJtkg
nair.radod ==gl$ga = a" -e"
a,
T#,"-.?X"',
(rii) Mean effective
= -l?5t- = 0'68i14 or 68'$47o. (Ans.)
o..o.
M".' .;;",i";;;;"1l,ffi;"'
p- = %IF d-on" P"t cy.l"
stoke volume

psvs Fig. 3.23. Dual cycle.


o^ = PvL -
ffo, <v, - vr + 1ir standerd eficiency, Iah-tdard i
Consider I kg ofair.
1:[;;;;'u=va=u",v =pv",
f' "=\:=r-;]"l
Adiabatic compression process 7-2 :
l

1." %=(r-r)%
=(t)ta-1 = 2.gs4
130 INTERNAL COMBUSTON ENCINES
AIR STANDARD CYCLES I31
. T2 = 3fi) x 2.954 = 886.2 K
3. Effect ofvariable specific heat, heat loss through clinder walls, inlet and exhaust vejoci-
et ties of airlgas etc. have no been taken into account.
pr -[qf -,rrn =+ p2= 44.3 pr
\u2 ,, o<rExample 3.28. A Diesel engine worhing on a dual combustion cycle has a stroke uolume
Constant pressure process 2-3 : of 0.0O85 m3 and. a cotnpression ratio 15 : 7. The fitel has a calorific ualue of 43890 hJlhg. At the
end of suction, the air is at 1 bar ond, 100"C. Th ma*num pressure in the qcle is 65 bar
lz=b and ai.ir fet ratio i.s 21 : 1. Find for ideal cycle th thermal efficiency. Assume cp = 1.0 hJ / hg K and
T2 Ts
c" = 0.71 kJ lkg K.
Solution. Refer Fig. 3.24.
Ts= Tzx '4a = aeo z
pz ' ' +!
44.3p1 = 14oo K
p (bar)
Also, Heat added at :onstant volume = Heat added at constant pressure ...(Given)
or cu(Tt- T) = cp(T+- Ts)
or Ts- T2= 1(T. - ?r)
ro=r^* Y
"T = 14oo +
tlrtfrg = u6? rc
Constant uolume process 34 :

:+
fr=t t=+=ffi =,,u
Also, = 1'26 or u = o'084 ur
f = dt
Also, us = ul
Ad,obatic etponsion process 4-5 :
^ / \r-l1.4-l I -f- 1(1oo"c)
Ts lur / =l-+-l
+=lYLl ( 0.08au1 =2.6e
./
I

To
" = 2.69=1767
T. 656.9 K V" = 0.0085 m3
L69 =
Work done Heat supplied - Heat rejected Fig.3.24
'rair'shndard - Heat supplied Heat supplied ?1= 100 +273=373}{
=
. Heat rejected
Pr=1bar
'- Heat s"ppliud

1- cu(Ts -Tt) Stroke volume, V" = 0'0085 mg


=
%(\ -T) + co(Ta -Ts) Air-fuel ratio =21l.l
(Ts-\) Compression ratio, r=l'c:I
,
(\ -T) + 1(Ta -Ts) Calorific value of fuel' C = 43890 kJ/tg
cp = 1.0 kJ/ke K cu = 0.7I kJilrg K
(656.9 - 3oo)
(1400
= 0.653 or 65.37o. (Ans.) Thermal efficiency :
- 886.2) + 1.4(1767 - 1400) % = Vt - Vz = 0'0085 m3
Reasons for actual thermol effi.ciency being dffirent from the theoretical ualue : v,
1. In theoretical cycle working substance is taken air whereas in actual cycle air with fuel and as r= 7f
V2 = 15, then V, = 15V,
acts as uorhing substance.
75V2- V2 = 0.0085
2. The fuel combustion phenomenon and associated problems like dissociation of gases,
dilution of charge during suction stroke, etc. haye not been taken into account. or 14V, = 0.0085
0'0085
or v^=
z v^=V
c- a = o.ooo6 m3
INTERNAL COMBUSTION ENCINES
AR STANDARD CYCLES
For adiabatb compression process l-2,
P1V{ = prv{ or ,r=
h=# =L447.5Kor 1124.5.C

tfi =rxos)L' lt=;=#=t.4


pz=pt Heat rejected during constant volume process 5-1,
=mcu(Tu-Tr)
= 45.5 bar = (0.00854 + 0.0004) x O.7l (1447 ,5 - 373) = 6.713 kJ
Work dooe = Heat supplied - Heat rejected
= (2.898 + 14.66) - 6.?13 = 10.845 ltt
Also, (ry-l=(101.41-r :.
?=(l'= = B.o4 Thermal effzcency,
,. Tz= ?r x 9.04 = BZ3 x 3.04 = 1184 K or 861"C .. =- Work done 10 R14
=O,6126o161.167o. (AnsJ
Fot constant volume process 2-3, Heat supplied (2.898 + r+.oo.r
eExample 3.29. The cotnpression ratio ond erpansian ruto of an oil engine worhing on
Pz _Ps
tfu dual qtcle are I ancl 5 respectiuely. The intial pressure ond, Emperature of the air are 1 bar
Tz Tt and 30'C' TIE hfut lberated dt ccinstant pressre is twce the hat liberqted at constant volume.
The expansion dnd. compression folLow the law pVL25 - consta,nt, Determine :
Ts= rz' fr =1184x fr =rezox (i)Prssrs and temperotures at all salient points.
According to characteristic equaion ofgas, (ii) Mean effective pressure of the cycle.
PlVr= (iii) Efficene1 of the cycle.
^7, (iu) Pouer of the engine if working cycles per second. ae g.
m _ .p _ 1xld xo.q)g
RTt 297xg7g = 0.0084 ks (atu) Assume : Cylind.er bore = 250 nm and strohie length
= 400 mm,
Heat addd. during constant volum process 2-3, Solution. Refer Fig. 3.25.
-mtcu(\-Tz)
= 0.0084 x 0.?l (1G20 - 1194)
= 2.898 kI
Amount of fuel added during the constant volume process 2-3,
2.898
= 4g890 = 0.000066 kg
Also as air-fuel ratio is 21 : 1,

.'. Total amount of tuel added = = 0.0004 kc


W
Quantity offuel added during the process B-4,
= 0.0004 - 0.000066 = 0.O0OB34 kc
:.Heot added d.uring the constant pressure operation B-4
= 0.000334 x 43890 = 14.66 kJ
But (0,0084 + 0.0004) c, (?, - ?.) = 14.66
0.0088 x 1.O (74 - 1620) = 14.66

t, = + 1620 = 3286 Kor


Again for operation 3-4,
# 3018"C

b=v, or Vo= Ys4 _ 0.0006x9286 = 0.001217 m3


Ts 1620
For adiabatic expansion op"r"tio? -# Fig.3.25
-r Initial temperature, T, = 30 + 273 = 303 K
A =f%./
= | o.oog
l&)'-' -=lororn?J '1tar
Initial pressure,
4 =2-27 Pl=1bar
Compression and expansion law,
pllr.zs = constant
I35
134 INTERNAL COMBUSTION ENGINES STANDARD CYCLES

Compression ratio, r.=9 ly.)"_- t =g:-=4.?zbar.


- a (Ans.)
Expansion ratio, =5 P5= Pax = P1' (5Fr5 -
Number of cycles/sec. = 8
l%J
Cylinder diarneter, D = 2ffi mm = 0.25 m
Arso +=f}I '=+-="*- =o',,,
Stroke length, tr=400mm=0.4m T1 [Vsl Q,Y-' (5r- ^

x 0'668 = 1445 K or 1172'c' 6ns')


Heat liberated at constant pressure
il uu"o effective '0'66s:2163'4
= 2 x heat liberated at constant volume
(i) Pressure and temreretures a all salient pointe : """"lii"]';:
Mean effective Pressure is given bY
For compression process 7-2,
r-= {frrtv. -vi*#
vrVi = nzY{
p=p1 x
lv,'\"
=1x(e)1.26=15.59ber. (Ane.)
=riof*(p-l)+W
-rrL(rc "#l
tJ Now,t"=p,p=1-8't=t'25'pr=1bar'p'=16'59bar'p3=35'7ba'
-'=(e)1.26-, na = 35.7 bar, po 4.77 bar
3=fE) " =
A1so,
Tr lvz)
= r.z'2
,- =
" tr e - r + ellffifre - "11h-'i']
.'. T, = Tt x 1.732 = 303 x 7.732
= 624.8 K or 251.8'C. (Ans.) 1
tza.se + 85.32 - 26.361 = 10.e4 bar
= i
Also, cD(Tl- Tr) = 2 * cu(Ts- T2) (nven)
For constant pressure process 3-4, Hene mean effective pressure = 10'94 ber' (Ang')
v... u u EfficiencY of the cYcle :
(ii)
Tt _Vt ^ _ Compression ratio (l;) uo"'= ur" Work done per cycle is given by W = p^V,
4 V3' Expansionratio() u1
=iX-
here

=9= tt v3P = 19!g#!g9q kJ/cycle = 21.44 kltcycre


D V'lr- "
Tt=t.8Tz =-x-=i
Vz P P /eat supplied per cYcle = mQ,
r^ r^
p==; - i" th" mass of air per cycle which ie given uy
, v"
v, =V"+v"=
v4 ^= # where
f1
Substituting the values of ?, and Tn in the equation (i), we get
1.0(1.8?s -Tl=2 x 0.71(?s -524.8) '"=L{='*fr * u"=# I
0.8Tr= 1.42(Tr- 524.8)
.. vt=v,.#=*[t.
O.8Tr=l.42Tr-745.2
0'62Tt=745'2
#)=*'.1
q
"' T = 1201.9 K or 928.9'C. (Ans.) 0.02205 m3
\ ni= '0.0196 =
Arso, +=t ..,,..for process 3-2
;. m= 1# = o.o2bs5 kg/cycle
Ts 1201 9
Ps=P2 x T, = tr.ro, - 85.7 bsr. (Ans.) Heat supplied per cycle
-524.g
"' me"=0.02585tc,r?3- Tr) + co(Tn- Tr)l
=
P =P, = 36.7 bar. (Ans.)
0.0i535[0.71(1201.9 524.8) + 1.0(2163'4- - 1201'9)]
=
T = 1.8?s = 1.8 x 1201.9 = 2183.4 X or 1890.4'C. 6ns.)
For etpansion process 4-5, = 36.56 kJ/cycle
Work done Per cYcle 21.4
PaVl^ = PsVun
Efficenqr =
E*t *ppit"d p"t tytt" = 3656
= 0.5864 or 58.647o. (AnsJ
INTERNAL COMBUSTTON
ENGNES AIR STANDARD CYCLES
(iu) Power of the engine, p :
Power of the engine, P = Work done per second S.7.2. For te Same Compreeelon Ratio and the Same Eeat Input
A comparison of the rycles (Oto, Diesel and Dual) on thep-u.and ?-s diagrams for the s/e
= Work done per cycle x no. ofcycles/sec
omprusinn ratia and, luat eupplied, is shown in the Fig. 3.27.
= 27.44 \ 8 = 17t.62 k\y. (tns.)
3.7. COMPARISON oF OIto, DIESEL AND DUAL
COMBUSTION CycLEs Otto = 1,2,3',4'
are the important variable factors 'Diesel
which are used as a basis for = 1,2, 3, 4
.""t", lo"o*tng comparison ofthe Dual = 1',2', t, 4'
.Compression ratio.
o Maximum pressure
. Heat supplied
o Heat rejected
r Net work.
Some of the above mr
,"t .o-u"tio;;:'; ;fl:iT;XHfo'"'are fixed when the performance of otro, Diesel and
3.7.1. Efficiency Versus Compression
Ratio
Fig. 3.26_shows the comparis
combustion
iron cycles
cycres at orthe
of-the otto,
ouo, Dieset
.""rr':lj:;"t1""i::"::11T9."qciencies
ar various vvrvso'rur
""ti"*;;;;;il";il';"4i"ff,"{1"i"".i""
ranos and *lll,4y"" cut Diesel and Duat
Dual combusiion
combustion cycles. tiil" jllgy"" cut.off-ratio
It is evident from rha Fia r-.1.L_r11,1n.a"1a,"1
a o
off_ratio for the Diesel aJ
gl.!1" rncrease "y.t"".
wEn rne i191eae i" t"
" ,h" the air standard erficiencies
.m"iJi,.r". io"""usi
increase
efficient
eff "";;::::1"1J:Tjy:*; j"fi:jjt:1]
""-p""Jil;:1il';;.'r;";f,:nu
centuthtetheDirr"t"!;i;;";;';;;f;n;i:"i.fi
uthile the Diesel qtue rs the ted.st cffraiont h t compr*sion ratio otto cyce is the most
:: Fig. 3.27. (a)p-u diagam, (b) ?-s diagam.

We know that. =l- - Heat reiected


- ,..(3.13)
Heat supplied
Since all the cycles reject their heat at the same specific volume, process line from state 4 to
1, the quantity of heat rejected, from each cXcle is represented by the opproprate area under the
line 4 to 7 on the T-s diogram. As is evident from the eqn. (3.13) the cycle which has the least heat
rejected will have the highest efliciency. Thus, Otto cycle is the most efFcient and Diesel cycle is
FAA
the least eflicient ofthe three cycles.
*"" i.e., Sott ) l'ldual > Idiqa .

For Constan Meximun Pressur and Heat Supplied


3.7.8.
Fig. 3.28 shows the Otto and Diesel cycles on p-u and ?-s diagranrs for constant maximum
pressure and heat input respectively.

operatino c-.-.I engtne opefating


S.l. engine
,mnracc^h rari^
compression -^_-_^
ratio range corpre"iiln i?n"ou
Compression ratio, r-_-____+
Fig. 3.26. Comparison ofelficiency
at various compression ratio.

.,rt. .,il""11;ffi,il:i,:i:,:iliff'"ilT"*'r"rg petrorcnsine is rimited by detonation. rn their


ll: respective

Fig. 3.28. (o)p-u diagam, (b) ?-s diagram


138 AIR STANDARD CYCLES
TNTERNAL coMBusrroN ENcrNEs
For the maximum pressure the points 3 and (i) l-2-Eeot rejeetion at constdnt pressure
- B'must lie on a constant pressure line.
on ?-s diagram the he rejected from the Diesel () 2-?-Adib atic c omp re s sio n
- the line 4 to I and this aa is less than the
cycle is represented byibe area under
hence the Dieset ycle is more
otto cycle ;d", t"-curve ( to 1 ; (iii) s-4-Addition of heat at constant uolume
"6"i"it-ino, the otto qcb for tre cond.ition of maxi-
murl pressure and heat eupplied. (iv) A-l-Mizbat ic ex pan s io n.
Bqmple a'w. (a) with.tlz h'etp of p-u ond r-s d.iagran
otto,.d'iesel and d.uar combustian compne tlp cold air standard
"wt;" f";
;";;;""imu-m presstie or tenperd.-
^ii^u^
(AMrE Sunmer, 1998)
solution. Refer Fig. 3.29. (a, b).
The air-standard otto, Dual and Dieser cycles
for the same maximum erTlu: and ;;;;;;;;".ture, are drawn- on commotrp-u and ?-s diagrams
for tbe purpose of comparison.
Otto 1-2-8-,1-1, Dual 1-218,-8_4-1, Diesel t-2._S_4L(Fig
8.29 (c)).
volume lines on ?-s diagram is higher than
t,""r. ,Flls.llconstant tha of constant pressure

Fis.3.30

Consider 1 kg of air
Compression ratio =2 =o

Expansion ratio =1t=.


U1

Heat supplied at constant volug = s,(?. - ?3)


Fig. S.29
Here the otto cycle must be limited to a low
Heat rejected = cu(Tt- Tz)
compression ratio (r) to fulfill te
point B (same maxirnurn pressure and
temperl;;;.; b" . .o-.ooo t"ii rl"'"ilti"condition
tt
that
cycles.
Work done = Heat supplied - Heat rejected
The constructio" of^rJ:r"" o'. ?-s diagram proves ""e = cu(Tt- Tt) - c,(Tt- Tt)
rejected is same for all the three cycles (""""-;;d;"';;essthat for te given conditione the heat
line 4-1). Since, by definition.
' = Workdone -_c,(T+-Ti-cr(Tt-Tz)
const. Heat supplied cu (Ta - T3)
n=t - E3ljggctgg,a, -.| _
(Tr-T2)
the cvcre, with srearer heat addition *rff:ti::il*,.1r. *"*?;" ?.s diagram, _ ,.
=1_r (\_\) ... (.,
A",or = Area under 2,13
Dunlrg adabatic compression 2-3,
Qtua = Aea under 2-318
Q<q$o, = Area under 2-3. ^ / r1-l
5=l2l
_ It can be\eem tnat, tdn*i"r,, e<u"u, e"(or") T2 l.,t/ =r.')r-1
and thus, tld"""r ) Tldo"l ) lo.. -t
Tt = T, 1sY ...ui)
3.8. ATKINSON CYCLE During constant pressure operation 1-2
This cycle consists of two ad,iabatics, a constant UUq
uorume and a consta,nt pressure process.
p-v diagram of this cycle is shown in Fig. TL r,
3.30. It consists of the folrowing
/our operations :

I
140
INTERNAL COMBUSTION ENGINES ATR STANDARD CYCLE,S

L=rt =y p (bar)
Trull ...(iii)
(o=oxuB=%x%=g)
Dwng adiabatic expansian 4-1, \ur U9 Ut lt ur r)
/ \1-l
A=l3ll -\"
Tr lr.J -r-rr-r
r,= *-
(rr' - ...(iu)
Putting the value of ?, in equation (iii),
{i we get
T,= Tt-r' a
I G)f ,
ll
tl
=,
Substituting the value of ?, ir, (di), we get
"q,riloo v(mt)

"s=411sr-r=lS)'.
t Fig.3.31
Finally putting rhe varues r,, rfl"a * ,, Te=Tz* 1.284=800 x1.284= gB5.2K or!12.2.C
"r "l,llu"" ,r'r, *, ,o For constant uolume process J-4,
Pq _Pa
T4 T3

16x3852
- PtTz -
, t=;= =1540.8Kor1267.8'C
4
Hene.e, air stand,ard, effi.ciency For adiabatic expansion process
=1 - t. l+a+') - 4_1,
)rn , ,y-l L22_L
s.st. A pe,fect ,", ,^**""\l'r"7" r6).F
,"u", r*]#!"ri Tt \p'J '
2=ta)
of the fottowngr-:::: =f = r.ena
"ucosisrs
(a) Heat rejection at
\r/
constant pressure.
(b) Ad.iabatc compresson
I
_ To = 1540.8
Tt=T.648 934.9 K or 661.9.C.
from bar and 27.C to 4 brlr. 1"648 =
(c) Heat addition at t (i) Work done per kg of gas, W :
to a finat pressure of 16 bar.
(d) Ad.iabatic
:::ume Heat supplied = cu (To- T")
Calculate : () ""r";;::::'
Work donelkg of gas.
(i) Efficency of the qrcle. = 0.75 (1540.8 - 385.2) = 866,? kJ/tte
Heat rejected c, (Ty-
= ?2) = 0.92(934.9 -
300) = 584.1 kJrC
?!" ' "o Refer kJlke K, c, = o.7s kJl ks K.
Solution.
= 0.92 Worh done / kg of gas, W = Heat supplied - I{eat rejected
Fig. 8.81.
Pressure,
=Pt = [
-
= 866.? 584.1 = 282.6 kJ/hg = 282600 N-m,4<g. (Ans.)
Temperature,
P2 1 (ll) E.'.n"" of the cycle :
Tz=27+2?3=300X
Pressure after adiabatic compression, pz= 4bar Effcency, n' = J:*!"1:= =r=3! (Ans.)
Heat srrpplied 866.2 =0.326o1 82.67o.
-
Final pressure after heat addition,
For adabatic compression 2-3,
ii = fe U".
3.9. ERICSSON CYCLE
It is so named as it was invented byEricsson. Fig. B.32 shows p-vdiagram of this cycle.
INTERNAL COMBUSTION ENGINES ATR STANDARD CYCLES I43
It comprises ofthe following operations : The variouE operations are as follows : ,

(i) l-%-Rejection of heat at consto,nt pnessure Operation 1-2.Ihe air is conpressed isentropically from the lower pressure p1 to the
(ii) 2-3-I s ot her mal c o mp r e s sion upper pressure p2, the temperature rising from ?t to Tr. No heat flow occurs.
(ii) !-4-Additon of heat at constant pressure Oreration 2.9. Heat flows into the aysten increasing the volume from Vzn Vs and tem-
(iu) 4 - L-Isot her mal exp an sion. per'ature fuom, Trto ?, whilst the pressure renains constant at pr. Heat received = mcr(T"- Tr).
Operaion 8.4. the air is expanded isentropically from p, to p' the temperature falling
from 4 to ?.. No heat flow occurs.
Operetion 4-1. Heat is rejected from the system a8 the volume decreases from V{to Vl and
the temperature from To to ?, whiht the pressure rernaisconstantatpr.Heatrejectcd=mco
(T1-T).
do,o"
r.trdr = ,
=.*ltkrecelveo
rleeE
_ Heat eceivecycle - Heat ejecteiVcycle
Heat receivecycle
mco (Ts -Tl) - nrc, (Tt -T) _, _ Tt -Tt
_ =,_84

Fis.3.32
Consider 1 kg ofair.

Volumeratio, ,=h=\
ug u4

Heat supplied to air from at external source tii

= Heat supplied during the isothermal expansion 4"1


= RTrlog.r C=Compessor T=Tuine
Heat rejected by air to an external source (a)
= RTz . log" r
Work done = Heat spplied - Heat rejected lll
= fiIr - RT". log"
. lo& r r = I log. r (TL- T2) l,;

_'' Work done Rlog. r (T1 - T2)


Heat supplied RT1.log" r
T'-T' ...(3.15)
= Tr
which is he sime as Carnot cycle.

BRAITON CYCLE
t
S.10.
Brayon cycle is a constant pressure cycle for a perfect gas. It is also called Joule cycle,
The heat transfers are achieved in reversible constant pressure heat exchangers. An ideal gas
Fig. 3.33. Brayto c,ycle : (o) Basic omponeats of a gas turbine power plmt
turbine plant would perform the processes that make up a Brayton cycle. The cycle is shown in the
Fig. 3.33 (a) aod it is represented on p-u and ?-s diagrams as shown in Fig. 3.33 (, c). ()p-V diagan (c) ?-S diagram, t:i:
jir
.ril
!tl
nli
ifl
ii;
INTERNAL COMBUSTTON
ENGINES
Now, from isentropic expansion AIR STNdARD CYCLES 145

TI h p*lofj given turbine the minimum temperature ?, and be maximum temperature ?,


a,=(a\ f p.rTcribed'
ar.e. being the temperatue of the ahosphere irnd ?, the rraximum temperature
rr la/ -Tt
which the metals ofturbine would withtand. Consider the specific hat at constant pressure co to
be conetant' Then'
II ,-l
Tr= T1 b) r , where ro = h.essue ratio. n
Since, t=rrr' ='
Similarly a=l)f ot
rl )
rr laj Ts=Tbbr'l
Using the constat 2'=
+,
.'. 1- T" -T' we have, work outpuUcycle
na*tadad =
---ti------E -' - ---T:T 1

| ( ,\
rat) r -T{rolr G)T ...(3.16)
I
L T *T;)
w = K lrsl t - l-t, (rn" - L)l
J
Differentiating with respect to
1

Is ' o ror a maxirum


#=*Fr,#^-rruou-,,1=
F
e
' '
A
rPQ+i
=TF(rPl" -r\

," =+
1
rp =(T{\)ub i.e. ro = (Ts)zl-r) ...(3.17)
Thus, the pressure ratio for manimum work is o functon of the limiting tenperature
Fg. 8.84. Eflect of preszure ratio
on the efficiency of Brayton
cycle. Work Ratio
The eqn. (3.16) shows that
!!1T::Z "t,ih"
,*"t :yt, qrcle inoeoses wth the pressure worh rata is defined as tle rcdo of net worh output to tlt itorh dnw by the turbne.
i^ii-i,t"ra,ure o*he
ii,?,,ii;i;:'::i::::,"1;:;:;,r.;';iir
process atone, no
turtherieltipg
X:*::! ? 'i:
loii";";;::r;::n;;:#:;rr"r::X{S;:;":;
.'. Work ratio =
W'.;%
wT
s
'::;;':"':,::::;';,-#:*::::"t*"t"; b"r;;;;;';;,n a*)on,,ft;,*; excess work fwhere \f = Work obtained om this turbine,l

Pressure ratio for maxinun worA


land Wc = Work supped to the compressorJ
: mco(Ts-T)-mcr(Tz-T) Tz_Tt
tu*p"I::r::"shall
prove that the pressu re rotio for maximum worh is a mco(Ts-Ta) =t' _ Ts-Tt

f'.
furction of the limtng
Work output during the cycle
I
= Heat receivecycle _ Heat rejectecycle
= mco(Tr- T") _ mco e1_ T) =r- ;l_b+-rl=,_fto,,? ...(3.18)
= mco (Ts - Tr) _ nco ez _ T)
'st--f- |

=mc,r,(,_+) -(+ ,) |- ,r,TJ


Eranpfe 8Jl2. Air enters tln crlmpressor of a gas turbinc plant operating on Brayton qtcle
at 101.325 kP_a'
!7"C. The preseure ratio in the cycle is 6. Calcutte the matimim temperature in
tIc cyde and th qch ffiicncy._Asswrn Wr= 2.6 W" where Wrand W"are the urbine ond
thz eompressor uorh respectiuely. Tahe " - L4. (p"U.)
INTERNAL COMBUSTION ENGINES 147
AIR STANDARD CYCLES
Solution. Pressure ofintake ai, p 101.825 kpa
Temperature of intake air,
= Eranple 833. A gas turbne is sttpplid with gas at 5 bar and 1000 K and' etpand's it
= 27 + ?lB = B(X) K ad.iabatinllX to 7 bar The mean specific heat at constont pressure and. constant aolume are 7.O425
", rD = 6
The pressure ratio in the cycle,
hJ I kg K and. A.7662 h,I I kg K respecaely.
(i) Maximum temperature in the cycle,
T, :' (i) Draut the temperature-entrow di.ryrcm to represent the processes of the simple gos
Refer Fig 3.35. turbine systen,
(ii) Catculete the power d'eueloped. in hW per kg of gds per second' and the erhaust gas
tetnperoture. (GAT8,1996)
Solution, Giuen ; p, = bar ; Pz = 5 bar ; ?, = 1000 K i cp = 1.0425 kJ/kg R
1 ;

cu = 0.7662 kJ&g IL
l'uT
t- cD -
'=b= -
0?662 =r.so
(i) Temperaure-enropy diagtan :
Temperature-entropy diagram represenng the processes of the simple gas turbine system
is sbown in Fig. 3.36.
p2=5bar
Fig.3.35

m / tt-t 1-l r{-r


!:=l?zl '
tt 1fr,l =r".1 ' =(6)T = 1.668

?z = 1.668 = 1.668 x 300 = 500.4 K


.Ll "r
Ll-l
*=<r; 1 =16 rr
Also, Pr=1bar
= 1.uut
Ts
T'=
* 1668
But Wr=2.8 Wc (Given)
mco (7, - Ta) = 2.6 mc, (T" - Tr)
-T-lr\
",- = 2.5 (500.4-300) = sor * = sor
refu
", |.t-fuj
Ts= = 1251 K or 9?8'C. (Ans.) FIg.3.36
1- I
L668 (u) Power required :
(j) Cycle efifrcienc5l,.r1."o,. : r-l 136-1
T- 9xr
ra -= l-lr
r
Now, ?, 1" =ll)-r"- s 0.658
=
r=ffi =?5oK
lp2) \5/
\
Network _ ztc, (4 -4)- mc, (4 -fi) ?. = 1000 x 0;653 = 653 K
" r.y.t"
Heatadded -
- - rr"r7rl) Power'developedper kg ofgas per second
trzr-ibqt-rpo - gobl - = co(Ts- Ta)
(1251_500.4) = 0.4. or 4A%. .
(Ans.)
- 653) = 861.7 kW. (Ans.)
= t.0425 (1000
T
Eample .3.84 , An isentropit ar turbine is used. to suppty 0.1 hg I s of air at 0.1 MN I tn2
cne*,ncyae=1--7f,=1--+=T= o.4 ot 4vo, (Ans.) and at 285 k to a inbn. The pressvre at nlet to the turbine is 0.4 MNlmz' Determine the
I tenperature at turb.ine inlet and. the power deoeloped, by the turbine. Assume co= 1.0 kJlhg K.
|
r ,")('J (6)-14
v D/ (GATE' 1999)
r48
INTERNAL COMBUSTOJ{
ENCNES
AIR STANDARD CYCLES
Solution. Gioen: i6 - 0.f hels i pr = 0.1
YN/*
cp : f Ut
1.0 k.lng r
T, = N K ; p"= o. lrbt=i bar ;
(iu) Heat rejected in he coole,r, Q":
= (W)
Tenperature
Work output = Ileat eupplied (e) - Heat rejected (e2)
t rto tnlef T, : :. ez= Ql- W = 466.27 - 136.88 = 819.39 k l/kg. (Ans.)

g=(a\i-rnl#
Y-l

-(il Tuno
(u) Temperaure of air leaving the turbine; Tn :
Fot e*panson (isentropb) pruess 3-1, we have
11 (n/ = 1.486
IJ rr-1
:. Ts = 2&5 x 1.486
= {SSj X. (AE.) r =(8.5) 11 = 1.a \
Power developedn p : =Or,
p = rhocr(Is_T)
t = *=Ir* = 610.5 K. (Ans.)
= 0.1 x 1.0 G?3.6_2u5)
- - [Check : Heat rejected ai cooler at constant preseure during the process 4-l can also
in. the
= tS.8E lW. (Anr) Fis.3.37
_
be calculated as : Heat rejected = mx cp(T1-T)
= 1 x 1.00b x (610.b: ZSS) = 819.1 kJ/kgl
ErnPle 836. Conaid ur ai wwtt
*tt n t^*t o"n :- _-., ! , ttu_r
,^r^ ,o.-?!i"l Exanple 3.36. Air enters thc compressor of a gas turbine plant operating on Brayton cycle
1.0-y" 'q zec, nn-i*i*
and, eC. Ttn preasure ff mr,?:k:-y!bl tlu-air enhn tle ompresnr
enten the ompresnr at at 7 bar,27'C. The pressure ratb itt ttu eXcle is 6. If Wr= 2.5 Wc, rh"r" Wrand.W" are the
ttb amnressor J.6 toi'-trp",wuo
turbin
curbu ialet h 6UfC.
inJet is b*#JSHp
6oec. b"r ;;;;r:
"f ot turbine and compressor worh.rcsputively, calculote th marimuk tumperatire andhe c.vcle
A Etrzcienel of tlu qcl.e, ( Hat supplbd, to air, efficieny. (GATE, 196)
()Worh evaille at tlu.W,
(iv) Heat rqjected
(v) Tempeatutz in the wbr, qd, Soluion. Giuen:pr=1bar; Tt=27 +2?B=gg9 X; ? =6;W,=21.5yy"
of otr talg tle turbin. PI
lolair t= L4 and, cp= 1.06 hJlhg K Maximum temperature, T, :
Solutlon. Refer Fig. g,gS.
m / .1-t L{-r
Pressure' of
air enteriag the ompreaaor, pr Now, f = l.l t =(6)T=(6)
o.{

=1.668
= 1.0 ba tp|/
Temperature at the inlet of
rn K t
"r*of ;;;m:';'#J""rtt
hessuto = t. Tz = 300 x 1.668 = 500.4 K
Temperature air at tuinc idet, i iOo + Zzg = AZg X ^ /
Pzl'
.1-t o.a
(r) Etflcteacy of te cycle q*. : ", Arso, $
t4 =l\pr,, = (G)il = 1.668

1- I ( ?3 = 1.668 4
l"v" = -E
,
= 1- -+-, = o.3b or g0%.
(AnJ
l" o=ff-#=*J Now, compressor work
("r) (g5)-r- W" = mc, (Tz- Tr),
0) Eeat cupped to ir: and turbine work,
Ft compreesbn prcess I-Nwe bave W, = mco (Ts - T)
f r1-l - Ll-l Since wf = 2.5 Wc ......(Given)

t,-l'") =(;
frltfJ'=(Tl-='* :. mco(Tr- T) = 2.5 x men(Tr- T1)
Tz= Tt x lry\
ar. e1= Co(T, ;;;;;il
29 x 1.48 = 1.43
= 293 lrs - :*l
Fis.3.38
,.
i* orrK= 16s.27 t,nrs. (Ans.)
EL Eqppue( o 419 = 2.5 (500.a - 300)
\ r-bb/
, Work
0) #:l'"T$1': il;""1i.f,.:
vailble atihe ch W :
r.fu- I
" \ 1.668/)=sor
We know that,
n_,.=Hi##
.. Te = 1251 K. (Ans.)
g.39
= -I- or W = 0.3 x 456,27
= 136.88 hrrg Cycle efficiency, Tlayq :

W, -W" _ mc o(7, - T) - mc o(Tz - Tt)


nrcYcle
.=
mcr(Tg-?2) mcr(Tg-?:2)

-A
-T

INTERNAL COMBUSTION ENGINES AIR STAND{RD:CYCLES

FvnFle 8.38. In a gas turbine plant working on Brayton cXcle, the ab at inlzt is ZZ"C, 0.1
_
f*t-l*'l-('001-soo)
r L668' MP.. TItc pnessure ratp is 6.25 and. the maimum temrcrature is 9MC, Tle turbnc and, com-
(u5r_500.4) prcssor effu:izncis are each &ub. Fn compressor work, turbine worh, hcat suppted, qrcle effi-

=
tHPn o.r0 or 1o%. 6$)
ciengt an'd' turbine exhpust temperature. Mass of oir may be considered, as 7 hg, Draw T-s d.ia-
gratn.
76(16 = (AllfIE Summen 20fi))
Erample 8.37. A cro*d qcre idar gu pbnt operates betwen temperaturc timits Solution Refer Fig. 3.40.
of 9p-'c and 30'c and prd,uces a puEr lm-turbu
tw.'iru ptantrs dcsigned, euch tr,,t trere is ta

need, for d' regenrator. A fwl of calaiftc asm
n t t *g is uod. calcai" tn" i
through the plant and tzte of'f"el rxluni";- " - rto, mte of ,,i.r
tAssume cp = I kJlhg K ard = Ll.
.l
. (GAIE, 19e8)
Solution. Given : Tr= 3O + II3 = fl)3 K; ?. g00 + nA 1073I(; C
_
cp = 1 lcllks K; T = t. ; W;- = = = 45000 kZkg ;
W* = rOW.
thr rir :
since no regenerator is used we c- assrme the tur. bine
a way that he exhaust gas temprature a,o te Spands the gases upto ?. in such
tubine is equal L tn" t"-*r-t."J"rair coming
out of the compessor Le., T"= T.
_1 . ,l
Pz Ps p,
pt - pl , p=[&_)rt
l\) ""0 =[+j'-' f T-s dlagram

&-=r" =&.
'. Tr T1 T2 Giuen :
Fis.3.4O
T, = 27 + 273 = 900 K ; pr = 0.1 lttPo ; r, = 6.%, Ts= 800 + 275 = LO73 K
("' Tt=7, """assumed) Tlo-p.=lturuirr=0'8'
Tz2 = TtTs ot Tr= lffi For the cornpresson praesa I-2, we have
i-:!
t{-l
?r= SGffi =6?0.2K =(625) 1' = 1.688
Now, W*r*"=
+=e)+=r',,?
Wru*r"- rz x Cx (f Tz= 3OO x 1'688 = 506'4 K

loo= m x45(xx)'
-- ---vv ^
' ft-3:]il
L 4-"l
Arso, r.o.p.=
ffi or 0.8=
Tf,|# Iil
roo
=,itrx4**tt-ffi:il Fis.3.39
?r, = !9%; + Boo = 868 K li:

= rhs x ZlOg5.9 "' compreesor worle ==1:";;1il1'l*,


"*' = Z'e.*ekr/kg. (Ans.)
*f. = ,i
100
= 1.71 x, lstrg/a (Anc.) For exponsion process 34, we have fit
suin{ wr*br." - w*o* = 1{)0 kw
(rito + rhXTs
-T1, - tho xlx(72 _ ?i) = IOO +=l+lf
T1 tp./ =(",)f =1u.ruti = r..* fil
trt
(rit" + 0.0O474X10?g ii!
- SZO.2) - zr" (5?0.2.-.BOB) = 100 ,1

r. =
or
(rh"+ 0.00479 x 502.8 -267.2 tiz" = l0O #=# = 635.66 K lJ

602.8 t4 +2.383-267.2
or *
= 1O0
235.6 d = 97.61?
Also, rlt-bi,"= ffi or 0.8=
#-#; t{

ir" = 0.414 k;gs. (Ae.) |:


NI
tl

-l
r
ti
I

,'t 152
TNTERNAL COMBUSTTON STANqARD CYCLES
ENctNEs
or T, = l0?8 _ 0.8 (1OZg _ 6q5.66)
153

,'. T\rrblne wor! W,+_ = 1 x co x (TB_ T)


= ?23.13 K 4rl:soo
. fuel mass) +3oo= 482.2K
Ner work o, = 1 1.006 (10?S _ ?2B,lB)
= 35I.6 kr/kg. 6inB.)
";=
Norr', Heat supplied by the fuel = Heat taken by the burning gases
'r."t - 't*;'=ezsr krks
"opffil'. i:= I*FlI:^;:361'6
=I x 1.O06x(10?B_658)
O.9 x m x C = (mo + m)x co x(Ts -?z)

=6t7.E7hfAg. tns.l
cvcle
rurbine
efcienc5r, n*, = F=# = o.I?ttit or
t?ai|%. (An.) "=W)'4#=[ff.'),.u%1
T:yi:*TT*tFig. S,i0.
lhe ?;s diagram is ehown in = 72s.rs Kor 450.1sc. (Ans.) 42000(+.)* r'oo(rrt=sez)
= l*.'l = ?se.?8
f,npre g.gg. rnd thc
required oir-fire. ratb-in a gas \',? )
pressor efficencies are turbinc wru*
a co*, ,""p""a""t1. M*i^;; ;;"ii"orrr" turbe and, com-
1 = 65'77 say 56

and'
-85%
working flui.d m" u:..!:.*.:"."i
%: l;;;4 k,
27'C, The nressurS rutia is 4.
.r
is 875.c. The
= t.iii:;;;;;, il" compressor at 1 bar
+=#t-
fu f*, l.
*'*{ri?#:"ii:,:
't" - t i
**'lla,f"i^0",.
^ituu "otori" valuc of d2000Iurths'
,m;,i# 8.11. SITRIJNG CYCI,E
A/F ratio = 66 : (AnJ

800 K; =l.i [j;":irh-bi.e l*o'".n =,20% : Tr= 278 + 875 = 1trf8I! Tt 27 + 278
:
kcE-1'#(iltriH,';il1'h=ri"'ftra;
_=
"" jff,f *;i."ilLuirrl'=aldn*i
= EiC. 3,42 shows a stirling cycle (182?) on p-u and T-s
and two constant volume ptocesses.
diagrams. It coosbts of ilrlo isotherms
For isentroprh rrlmpresrlbn T_2 : Pocess 1'2 is the i&thcmal ampression with heat rqiection
temperatue fr. Q, to the tiilroundings at
Pocese 8'1 is the inthennat expansion wirh hcat addition
ture ?* e from a Bouce at tempera-

Fig. .4t

, .l-1
?
+=l?l
1 \A/ =r,,tii=r.se
\,/ -
? = 300 x 1.486 = 445.8 K
_
rfmposq = n-n (c)p-u diagram
() ?-s diagram
ffi Fig. 3.42. StirliDgcycle.
08=## Process 2.8 nd 1-l ae constant aolum heat transfer pr@esses.
For t
.

hg of idea.l gas,

Ul
Q7-2(Heat rejected) = Wt.z= - RTLI (mmpression)
u2
NTERNAL COMBUSTION ENCINES
AIR STANDARD CYCLES I55

Qzt= cu(TH-TLt; W = 0 (since u = const.) = 287 x 310 ln (5.33) = 148828 J/kg = 1a8.88 kJ&C
Procese 2-B : Qz_s= W,-s = 0 (since volume is constant)
Qs.a (Heat supplied) = Ws,t= RTE6 ? = RTx ln | (Expansion)
-b"u2 Process 3-4 : Heat supplied, Qs-t= Wt-t = R?a ln(r)
(.' pa
= u and u = r) = 28? x 930 ln (5.33) = 146634 J/kg or 446.63 ktkg
Qa-1=-cu(T"-T*) or c,(T, -T"l; W-r = 0 (since u = const.) Process 4-1 : Q+r = - c, (T- Tr) or c,(7, - Tr)
The efticiency ofthe Stirling cycle is lecs '\n that ofthe Carnot cycle due to heat tmnsfers Heat supplied during the process 2-3 = Heat rejected during the process 4-1.
at constant uolume prxesses. However, if a regeneratve arrangemet is used euch that
Qqt= i.e. area under 1-4 is equal to area under 2-3, then the cycle efficiency becomes
.'. Work done = Ne heat exchange during the isotherms
Qz-s
= 446.63 - 148.88 = 297.?1kJtkr
nqrnS-n4rn5
" v2 t)2 work done 2917F.
rl' = -.- =Tr:7, ...(g.19) ... Thermal ef;cien<
J,r,= Hffi = = 0.6676 or 86.7to.
RT1ln\ TH r*
Since Stirling cycleis completely reversible, its efficiency is also given as,
which means the regeneratiue Stirlng grcle has sarne effrcieney as the Carnot c7rcle.
Tu -T, 930-310
The following points are worth noting : n=-
2b: = t* = o'667 r 66'79o' (Ane')
As far as the impractibility of actomplishing isothermal compression and expansion
processes with a gas is concerned, the Stirling cycle suffers frorn the lirnitdtinn of the
S.12. MILLER SYCLE
Carnot cycle. But, it does n suffer from oher drawbacks of the Carno cycle, uiz, very
low m.e.p., the narrow p-u diagran ad great susceptibility to tbe internal efficiencies The Miller cycle (named after R. H. Miller) is a modern modification of the Atkinson cycle
of the conpressor and the expander. and has an erpansian ra'tio grcater than the compression rafo, which is accomplished, however,
The mean effective pressue (m.e.p.) of the Stirling cycle is much greater han that of in much a different way, whereas a complicated, mechanical linkage system of some knd, is
the Carnot cycle. required for an engine designed, to opero,te on the Atkinson clcle, d Miller qrcle engine uses
a A reversed Stirling cycle with regeneration can sinilarly attain Camot C.O.P. unque ualue tining to obtain the same desired results.
a The Stirling cycle can suitably replace Otto cycle (hang two constant volume proc-
esses) in reciprocating I.C. engines.
Example 8.40. A ar stondard Strling clcle is equipped with a 100 percent fficient
regenera,tor sytem. The isothenna.I compresbn drmtwnces ftom I bar and, 310 K and. eubse.
quent keat od.dition ot constant uolune rai.ss tlre preseure dnd, temperaturc to 16 bar 930 K The
cycle e ft,nally completed. through an isotlttrtrlrl
"-pansian and. constdnt uolume ho,t rejection.
Arnlyse e@h of the four processes for worh er.d lnot trdnsfer and d.etermine the engine fficn;ct.
Solution. Refer Fig.3.42.
Give'n: p, = l bar; T1=T"= 310 I{;p, = 16 bar iTs=Tx = 930 It The closing rycles for :

Consfiler t hg ofair. .Early intahe valv e : 67 -L-7 -2-3-4-5-7 -6


Le intake valve : 6-7 -5-7 -2-3-4-5-7 -G
Pp1= RTt

', =
#=u:*# = o'88e7 m'/rrg

similarly uz= * = = 0.1668 ms/kg


#=H#
r\ ')=
o.*rt u.rrl
|.,. compression ratio. r = 5 llr =
t u2 (usl 0.1668=
I
Process 1-2: Tr=T"= 310K= ?

Heat rejected,
T.D.C B.D,C.
Fig. 3.43. Air-standard Mer cycleformthottled a naturallyaspirateelfour-stmke rycle S.I. engine.
156
NTERNAL COMBUSTION ENGINES AIR STANDARD CYCLBS
In a Miller cvcle air iJake is zn thr-ottled.lhe quantity
of air ingested into each cylinder
bv ctosing the intdhe valve a,t i;-;;op", time, is
'i7 ;:;:;:t" tong'bero" .o.c. (poinr ? in ----t-\

with the movemen


of the piston towarde B.D.c. during -t--
'
the later part of the intake stroke,
the cylinder pressure is reduced along the p.o""""
Z--i. /',/
r,,"""u#;,:"'l:fJ',filJf;:
B'D'c' and stats back towards the r.D.c.,

Then, the resulting rycle is: G7_l-7-2-A4i+6.


The work produced during 6-? (inr'Le process)
rhe pressure again
,V
- is caneeiled by ?-6 (exhaust process).
The process Z-1 is cancelled by 1-?.
-
*t
indicared work = Aea within the toop 7-2_B_4_5-7; there being no pump
i;:;:
rt
The compression ratio = !_
u2
i
...(3.20)

The larger compressioq ratio =


!!=vt
...(3.21)
A greater net indicated work
' r"1 9raf,, of*ro* as a result of the shorter compression
sroke which absorbs work, combined with
the loog",
""p"orio"
,i- *u" produces
Fig.3.44. Airetndard approximation forhistoric Inoire.gne qycle, 1_2
i
l+%1. .

o Further' by permitting-air to.flow through the-intake


system unthrottled, a mqior ross
experienced by most S.I. Thermodynanic analysir :
engines ie elimrlo,ted..
r Due to the absence of pump work, the Miller
cycle eagine has a higher thermal
Consider the clearance volume to be esseutially nit, Cancelling the intake process 1-2 and
latter half of the stroke 2-1 thermodynamically on p-u coordinat-eg, the cycle then becomes
.ciency. efti-
2-3-4-6-2.
o Milrer cycle en*ines are^u.rrllv supercharged
or turbocharged with peak intake mani- Process 2'3. Constant uolume heat input (combustb); All valves closed :
fold pressures of 160-200 kpa.
Pz = P t = Poan', U g = z i Wz-s = 0, Qz_g = Q o = c u(Ts - T") = (u, - u2\
r Automobiles with Mille-r cycle engines were
first marketed in the latter half of the Proceeg 3-4. Isentropic power or expansan srole ; All valves closed : -
1990s. A typical varue ofthe compression
ratio i,
"""t , l, *ltr, ." "i0"".", ratio of
g Qs = O ; Tt = Tsfu{vy, ; pn = p, (v{u n)r ; .
about 10 : 1.
Ws = @p+ _ psus)l(t_ 1) = A(?. _ T"ye _,)
Pocess *5, Constant uolume heat rejection (exhaust blow d.own); Exhaust valve open
3.13. LENOIR CYCLE and intake valve closd ;
Fig 3'46shows an air standard approximation fo a historic uE= 1= us.o.c.i W-= 0, Qr- = Qout = cu(To-Tr)= ut-
2-1.
Lenoir engine cycre, 1-2-3-4-5- u
Proees 5'2- constant pressure uhaust stroke i p"^ Exhaust op"t, nd intake
;
o Thefirsthalfofthestrokeisintake,withair-fuelenteringthecyrinderatatmospheric valve closed. "rvJ
presstrro ,.,',,,,,.) _ proCesS l_2. Ps= Pz= Pt = Pon. i Wu-r= po^(vu- uz) ; Qs-z= Qout = (h5- h2)
At about halfway through the first stroke, the intake
varve is crosedand the air-fuel
= cp(Ta- Tz).
mixture is gnited.without d.ny cotnpressrioz. combustio" Thus, thermal efficiency oflenoir cycle,
in the clindero rrnost at constant-vorume ""i";, il; ;ierature and
inrherro*--o.,iig (q),.oo- [*t =Q-;Q*
;::r"* "rr'il+process q4" Qio =1-9tt
- Qi"
r r""onu half of the first stroke then becomes the power _ , _ fc,(\ -Tl) + co(Ts -T)l
l\1 or expansion process
cr(Ts -72)
o Near B.D.C., the exhaust valve opens and
blowdown occurslrroceec 4_5. _, _ (T - Ts) + "t(Ts - 4)'l
Then follows the exhaust stroke 6_1, thus
completing tbe two stroke cycle. Ts-Tz
...(3.22)

.-t
I
INTERNAL COMBUSTION ENGINBS
AIR STANDARD CYCLES
ilfi
EICHLIGHTS Z The air standard efficiency of Otto cycle ie given by

L A cycle is defined as a repeated series of orerations mrring in a certain order. -l


2.lhe elliciency of an engine ueing air u the working medium ie linown a8 an 'Air atandard efficiency'.
(4.,4= I+ ()n= l- (,rh
-
& tanve emosw.i =
Actual themal efficienry
Air ctrdrd efeienw
(c)n= 1- --
()r "
1
(d)=2-
(,)h
Ii
& The tiermal efficienc, of theoretical Otto cycle
mm
Carnot cycle efficieacy, rr"*= -A-. (a) increases with incease in ompression ratio
() incremes with increroe in isentropic hdexT iii.
() does not depend upon the pressure atio
Otto cycle efEciencr, neo=r-
rt 4.
(d) follows all tle above.
The work output oftheoretical Otto cycle
A f(rr-r -lXr, -1)l () inaeaaes witl increase in compression ratio
Mean effective pressure,
- P,*,= .
iil
-dtix_ii-- () inseases with inceaee in pressue ratio

Diesel cycle efciency, 4m,=r-


r for -rl (c) icreases with increase in adiabatic indexy
(d) follows all the above. lt
,1,t=TLeL l 5. Forsamecompressioratio rll r
[y(p - 1) - r1- 1(pr - tI
Mean eflective pressure, Pr.= Arr (T-U,_l) .
(a) themal efficiency of Otto cycle is greater than that
() themal efciency of Otto cycle is less tha that
of Diesel cycle
of Diesel cycle

7. Dualcycleefficiency, rm -- t I
=r
ro.ot -rl I
(c) thermal elcieacy of Otto cycle is eame as that
for Diesel rycle
(d) thernal efciency ofOtto cycle caaaot be predicted.
{lH
(",t:il(p-:ilE6:tj 6 In air stmdard Diesel cycle, at fixed compression ratio mcr fixed value :ilfi
of ailiabatic index (p
(o) thermal eftcien.}r inceases with increase in
Mean efective pressure, ., _ - _
Arrlp(p- r)+(p-l)-.r-1 (ppr -t)l () thermal efficiency decreases with increase in
heat atrdition cut offratio
'(,ru (r_lXr_l) heat addition mt offratio
(c) themal efciency remains sarne with
increase ia heat aalarition cut ofiratio
(r - cr) (d) none of tlle above.
& Atkinson cycle ecieocy, tlrm= 1-7.
/ _at
where o = compression ratio, r = erpaasion ratio.
fANERs I
9. Braytoncycle, larwton =r - whei rp = pessute ratio. () z (b) 3. (d) 4 (d) n(a) c (b).
-''!',
(rr\ r
THEORETICAL QUESTIONS
Ts -Tt
1(). Stirlingcycle, t*w='E-. rt
1L Millercycle engrnes areuuallysuperchargedorturbocargedwitlrpeak intake manifoltlpressm of 150 l. What is a cycle ? What is the differece between an ideal and
actual cvcle ?
-20OkPa. ?- What is a air-stadad efiicieucy ?
3. What is relative efficiency ?
It Leloircycle; (tr);* Q4 - T+JS6 - r
= L - + Derive expressiom ofefrciency ia the following cases :
fd) Canot cyde (ii) Diou.t"ru (iii) Dual combwtion cycle.

oBJECTM TYPE QL'EstflONS " *y"1?:r"*"rjlr,*analysis"whichhasbeenadoptedforl.C.enginerycles.Statetheassumprionsmade


for air stanclard rycles.
e Derive an exprssion for.Atkinson
rycle',
Chooee the cotlct arawr ! Derive an expression for the themal efficiency of Stirling
cycle.
& Exdain the following cycles briefly and deri-ve upressi of
(a) two coustant prgLe procssseE ad two co8tant volume proceaccs (i) Miilercycle "f"iuo"y.
Q9 coaelgntfreasur and two consta[t etropy pr@sses (ii) Lenoir cycle.
(c) two constant volume proceesea and two constant entropy plooegSGs
(d) noe ofthe above.
160
INTERNAL CoMBUSTION ENCINES
AIR STANDARD CYCLES

UNSOLVED UAMPI,ES *
fffffifrmt*:""::*1l.*:::X11"^?10 -y, ."g a cylinrter rlia:neter or 165 mm. nre crearance
A carnot eagine working betweea 3z?"c and B?.c producas
120 kJ of work. Detemine : ;Sffi "hHf .'""atr,"n"ire,t"""t"rli'1fiAffi?1ff
l;,T",T"ffi$,T.:Xr"ffi;;
(i) Theheataddedihl. (i) The entropy change durirgheat 16. lte followiag:::,rrh";;si";;;",,;;;T"ff
H."li:_"f data belong to a Diesel lAne.6l.
d
,i:1.3"""".H:l:*:i:.::_
jr j\
^L
^ ^_ j_ -
rejection prccess. cycle :
rAna.(i)z2s.5kr;(joo.ssshr/K (;Daz.snl *a"a
^
Find the thermal efficiency ofa carnot engine whoee hot
and 15'Creapectively.
aad.cold.bsdies have temreratures of r54.c
;ffiffi:il"f,:;ii"'=t;rY;:t
Dete:rrine:
= z56 "; Lowesr pressure in tre
rycre = I bu ; Lowest

n-r..^ an tAns.B2.55%lj
& Derive ^_expres"oo q ch"'ge-i-r-efficienry for a ange incom.pressiou ratio. Ifthe compression rautn
(i) thernal efrciency ofte
cycle. (ii) Mean efi'ctive pressure.
The efficiency of an Otto ycle i|*
ttree.r1n1_as_l rnilease in effi ciency
4, ;.':T:::"^T:i.1:111 is 60% and.lis 1.6. What is the cornpression 16, rhe rT,g:-:,,T31l-Dual
ratio ? ffi;; co_mpreseion rario
G An engine worting on otto cycle,has a volume of-0.5 mr, pressure lnited to?0 bu' rhe presu. re and temperature
crcte is t2:ld.rhe. maxi.ffi,:,1"1t"J',3:Hl"1l
1 bar and tenpuature 2z.C at the f cycre ai tl" r"gioning orompression proce's are r
comencement of compression stroke. At the ed of comireesion
-- prirre
stroke, the ----'# t0
bar and 800 K Cat".t.t , (i) Th"*"i"ft1" i"iii"""l,"r"ctiye prersure.
added duringthe coutantvolme proceds i8 200 k I. Detenoiue ^" bar.
"-' Heat Ass'.ie; ry"'bore
(i ) Percontage clearance
: = 260mm, st-ke t""sor:;00;,;"= i;;=;;18 andy= 1.4.
(ii) Air
(iii) M""o passuro ",-Ur"U "ffi
cimcy
u'ltecompYilr"1tT:H*r*":lj*:::ryg:-q*ry:*ffil"1ffi IAn*(i)67.92% i(jj) 9.84? bal
"O"*oe
;:H;:lj:
u,"rrnall-dlffiffi:k-. .;;1;ff,ln:trffif ;,#;,J
(du) Idu **"t ueveloped by the engine if the engine runs at 4gonp.m. bo thsere $e 200 complete
cyclesprminutes. tAre,(i)2g.76%iiii)4i.zq;ilinr (iu)32lkw
;,#;""f
675 fftre*1i*T,rxy_",'}.:;;i#ffi
#.ftFin
h/kg of air.
AairstadardDualcyclehasacomo""r": . tAD'.og.bzol
IAs,59.5Vo1
6 1he conFreesiou ratio in a air..sland.ar-d Olto cycl ia g.
At .lre beginning ofco,pessi6
18,
.ri"'a;;
.li';?ffi
*ji"
zo r,". ,q1T1f::::li "l16, "-r.
and compreaeion begis ai I ba, 60"C.
process, the isza,.r"rr;;ffiffi
mumrun Deaam i.
,l?ffi The
pressueis lbaratrdthe temperatureie 300Ii rte heat'h"*r*t"-tli"
Jip"""y"it"-rsirj0k Ifuofair. friffiSJffi
volume. Determine: ffi f ffi ij,',j"T;igl;fi :
(i) Themalefficiency (ii) The mean effective pressure.
(i) The cycle efrciecy. (ij) flr" _"*
*"ctive pressure of the cycle.
Take : c, = 1.005 kJ/kg-I! c, O.?18
= lykC_K
z An ensine 200 m rg' computl t'e
ora B-raon cycle operaringbe**"
bm mtr 300 mm stroke works on ono cycle. rrre
n*** and temperarure are l bar anr 60rc. ftu ","j*l.t::A-Jf ;:;#;iHi pressure of12"i" "t""a"J"maenry
bar. Ta.kel= 1.4. "
f"i,1?"Ti?;:'*:t"t Hi
Hlt -"*i-,- ";;;;;;ed ro 24 bar, tAa.50.g%l
(i) lhe ainstqndard efliciency ofthe
rycle (ii) The nea effective pressure for the rycle.
Assume ideal condtions.
& calculate rhe air standard erriciency of a four srroke ouo
o*'
Piston diarneter (bore) = IBZ mm ; Length ofstroke
cycle engin" .jf"T3rfr::i|t;'Lt"lt'
Clearance volume = 0.0002g mr,
= IBO mm ;
Express clearance as a percentage ofswept volume.
0' lAns,56.LVoll4.6Vd
In a ideal Diosel cycle, the temperatw^s at the beginning
of compression, at the eod of compression and
at the end of the heat addition are g?"C, ?89.C m f
$9:C. fin ,he th;;;
"ffi;;";"f
lo' An air-standard Diesel cycle hts a mmpression ratio lAns.69.67ol
of 18, aad the heat transfeneal to the working fluid
per cvcle is 1800 kJlg. Atthe beginning ofthe
compilsio".trot"rll*;r;.irJi"-r'r" u.a r.
temperature is 300 K Calculate : (i) Themal efficien.i, ()
tfr" ,rr"* .ffud;"-pr;;;".
t1. r)<g orair isiaken th:ough
lpiesd cv_cle. rnitiany the air is ar 15.. ,
jf;d3"t#;:tlr::'::rH
15 aad the hear added is 1850 kJ. d"olate , r"r"J"v"1. "".
pTessue. "r#,i;;; d;|Hm effecriv
12' what will be loss in the ideal erriciency of a Diesel engine
delayedftom$votog%? -- with
"-" comn*r".f;l:?;i.,'r'l,ll"Hl
vv'F^voe u""'

l& The pressures on the compression cune of a esel
".r.r*l
engine are at th stroke 1.4 bar aad at
14 bar' Estimate the compression ratio. calculate
f f th stroke
the air standad efciehcy ofthe engine ifthe ot ofi
occurs at ft of the stroke.
[Ans. 18.54;63.79]

I
--Lf-
lil
li
163
FUEL-AIR AND ACTUAL CYCLES
clinder as the temperature and
4. The variation in the number ofrnolecules present in the
preesure change.

4.1.3. Assunptions mede for Fuel-Air Cycle Analysis


are md,e for th analysis of
Beside considering above factors, the following as'Jumptions
fuel-ar qrclc :
1. Prior to combustion there is no chemical change in either fuel or air'
2. Subsequent to combustion, the change is always in chemical equilibrium'
Fuel-Air and. Aefiiol Cyeles S.Theprocessesarcd'iabatc(i..,thereisnoexcbangegfneat,Uetwlel,thegasesand
_cylin.<lerwalleinanyProcess).lnaddition,theexpansionandcomPresslonpocesses

arc frbnless.
4.1. Fuel-air cydes-lntroduction-Factors considered for fuel-air cyde caleulaou- The velociti es are rcgligib\r small in cose of reciprccatng
eWinel'
4.
Assumption maile for fuel-air cycle analysis-Imporan@ of fuel-air cycle-Variable spcic
Furthermore, in case of a consa,nt volume fuel-oir cycle the following assumptions are
heats-Effect of variation of specifie heats-Dissociation-lhermal efficiency and fuel
consumption-Effect of common engine variables--Characteristics of conetat volme fuel-air made :

a The fuel is completely vaporised and perfectly mixed with the


ai ; lll
cycl+Combustion charts-Gas tables. 4.2. Actual cycles-Introductim--4uses of deviation 1t
(at constant volume)'
of actual cycles from firel-air cycles-Real fuel-ai engine cycles-Iliference between real a The buroing takes place instantaneously at T'D'C' ,ti Ir
cycle and fuel-air cycle-Comparison of operations and working nedia for 'air cydd, 'fuel-air ll'h 1

4.1.4. Importance of Fuel-Air Cycle


i
cycle' .and 'actual cycle' of S.I. engine-Worked Examples-Hiehfights-Objective Type l1r II
cornpression ratio on
Questions-Theoretical Questions-Unsolved Exmples. Whereas the air standard cycle exhibits the general-effe-ct-of
, efficiency ofthe engine, the fuel-air cycle rray e calculated for various fuel-air ratios, lrtj
inlet temperatu.""?rrd pr","o"es (It is worth noting that fuel-air,ratio
and.compression
4.T. FUEI-AIR CYCI.ES llti
are much more important parameters in comparison to inlet
conditions)'
ratio
4.1.1. Introducion oWitbthehelpoffuel-aircycleanalysisaoerygoodestimate'.ofpowertobe.erpected'uery
to-dp-Prcrinate
In air standard cycles analysis highly simplified approxirrations are made. The air from the actual engine cai be mad' Furthermore, it,ispossible
standard theory gives an estimate of engine performance which is much greater than
'closely
peak pr*sir"s and exhaust temperatures on whi.ch desgt and, engine
structure ilti
the actual performance, This large variation is partly due to the non-instantaneous' dePend.
burnlng and valve operation incomplete combustion etc. ; the major reason being ouer'
sitnplication in using th values of the properties of the unrhing fluid for cycle onolysis.
.1.1.5. Verisble SPecic Eeats
In air cycle approximation it is assumed that air is a perfect gas having constant 4.1.5.1. General asPects
specific heats. In actuol engine the worhing fluid is rwt air but d mirture of air, fuel Th.especifultcdtofonysubstanceistheratinoftheheatreqvired,taraisethetemperature
have differ-
and, exhaust gases. Furthermore, the specific heats of tlu worhing fluid' are not con' of a unt substa,i,,e through one dzgee centi'radz. Different substances
,n*" i ttn
stant but increase with rise in tetnperture, ond at high temperdture th cotnbustion ofspecific beat. In case oflases, tbe temperature can be raiseil
in two ways' e'g' either
and c'' It is
products are subjected, to d.issoeiation. at constont pnessre sr @n;lfrrnt volitttc' Accotdingly we have wo speA!! n!f!..cr
"ot.,r.loe"
M
The theoretical cycle based on the actuol properties of tlu cylinder gcses is called the often convenient to use specific beais for the mol of a substance. A mol is kilograms, called
Fuel-air cycle approximation ; it provides a rough idea for conparison with the M is the molecular weight of the substance.
f.i"g;--"t abbrcviate,as kg mol. Here
actual performance, Thus molcr sPecific hzat
a,
-
4.1.2. Factors consideredTfor Fuel-air Cycle Calculations C=M.ckJ/molK
, The following foctors are /aken into considerations while making fuel-air cycle calcula- Sir49, Cp = M' c, kJ/mol K
tions : / and Cu = M. c, kJ/nol K
1. The actual composi.flon of cylinder gases (consisting of fuel, air, waterVapour in air and In general, the specific heats a"e not constojnt' The specific heot varies largely with tem'
residal gas). Dylng the operation ofengine the fuel-air ratio changes due to which the p"roturi ut not'u"rx, ignfnontll wth pressuie er'cePt at uery high pressure' Thus in simple
relative aryrlts of COr, water vapour etc. also change. ;;i;;do"t, the varation in specific heat with pressure is neglected'
- -- uibratonal
2. I-crem-of specific heats ofgases (except monoatomic gases) with temperature increase, in"pecific heats of gases increase with the rse in ternperature since.theheats on the
subsequently the value of 1 also changes. lecutn inirons with tetnperature. The effect of vaiable specifrc to studv
3. Since the fuel-air mixture does not completely combine chemically at high teurperatures "n"rgy-oi-tl;
;.,,-,i.'.,tH;;ce at ugrrer nperatureis considerable ancl it is, therefore, necessary
(above 1600 K), therefore, at equilibrium condition gases like CO2, H2, and O, may be these effects.
present. Itisgenerallyassumedthatthespecificheatisalinearfunctionoftemperatureandthe
following relations hold good-

162
164
NTERNAL COMBUSTION
ENGINES rR AND ACTUAL CYCLES
cP=alI{f
-
where , and K are
c,,= b + KT ...(4.1)
const. ...Q.2)
(."r-
b + Kr) - (b + r(I) - a - b
"] -"J =
."o'd,Tf;;:,";;ffltT#"*'"t:""ti "o" """i*00 K Abve 1500 r! the increase,;;Tl I ,c
I
cP=a+K1T+I{r7z I

rrrl-!. cu=t+xrr*rrz /l .-.t+.+


Y
r rne rnternal energy ofbration ...(4.5) b 2.O
is ignored the s teats orgases mav be written
n - "' t ;i';;.i.iffi* as : o
For monoao-i" g;? -5 t.5
",=+ kJ/kg K or kcal,&s "K

M"" = kJlmol K or kcaVmol .K


where
ff G
t.0
? = lbe chaacteristic gas
and t R =-Y:t::t1weieht x chaail;;;;";:
= Motecular;;;;";jjtant.exPressed
c
in kJ4<g K or kcal/ke.K; .g
6
o
= .Univesal gas constant
a)
= 8.814 k tkC mol K or 1.986 .I(
cu is expressed in kJlkg K or - -- kcalrkg mol
"yssa6
kcal,&g "K
Also cr=R+rr=n* 5B
T =
T 1500 2000 2500 3000
a"o= ua Temperature'K
| --+
20'787 kl/kgmol K or 4.e65 kca!&g Fig. 4.1, Incease ofspecic heat with temperature.
fi:'.t#:'-tilT3J";
For diatomic gacee :
mor .K
Fig. 4.2 shows the effect of humidity on properties of air. .R is the characteristic gas
constant ; coand cu are the specific heats at constant pressureand constant volume respectively,
",=|aora",=f,ltn T = ratio of'specific heats.
+10
5t
+8
"p=6R+8=in
2
= 2e.tN/kg.} x o. 6.e5 kcar&g mor .ri
+6
;,
.#:,.f:::1,:l
For po\ratomic gaees :
(Approx.) c, = BR or M.u gllp
+4
= +2
c^ = 4R or 1t1",
Thus M", (porvabric) = 41,4"o
gg.26-i<Jn<c'ill in
= x o. 7.944 kcal.rkgmol "K. E
0

It can be shown, from above


relations, that the value
ofatio A fo" th" gases is :
E
e -4
Goses
-6
Monoatomic "nl"u
5/3 E
Diatomic ;{ -8
'1 -10
-- -.^,1],F"::,,
as -----1v
shown in Fig. "l9"pt,,"""":'J"*;#:.es,
4.1. 'q'ED'
'ow
show an rncease in specific incr
heat at high temperature
0.02 0.04 0.06
k9. moisiure/kg.ar --------'
Fig. 4.2. Effect ofhumidity on properties ofair.

-&_
166
INTERNAL COMBUSTION FUBL.AIR AND ACTUAL CYCLES
of internal eners and ethalpy during a process with
variqble
"p".m1113i"change
The small change in internal energy (du) of a
temperature (d7) is given by unt ma.ss of a gas for small change in
:

du=uf,7
:. ur-ur= !l'"uar =
!r"{b+xr)ar
=
f; (T22- Tf,
b(Tz- rr) *

- 'Lf*rlL*tl
=(ro-r,)
\ f-)l=Qz-r)(b+I(t^)
where, ?- (mean temperature)
= \*
c, =b+ I(T^(c^ means mea specifc heat at constant volume)
Iftbe expansion or conpression is polytropic, then
:' u2- ut = cu (T"- Tr) pu = constant, where n < 1'.
...(4.6)
Similarly, change in enthalpy is Siven Uy : Diferentiating the above equation,
dh=co7 p . nun-r , dv + u" . dp = O
:' ht- ht = cr (?:2- T)
...(4.7)
where, cr- means specific eat at constant pressure. u. tdo =_p.n
du
4.1.5.3. Hea trensfer during a pDoess
with varieble srecic heats Inserting this value in eqn, (4.9), the heat exchange in a polytropic process is given by
For a closed system, the heat-flow is expressed dQ L ,-.,_ _ _r. ff /_ _ _\
as :
dQ = du + dW (considering 1 kg ofgas) d, = (*t)q'p-p.nt+ (o_\p-p.n)
dQ=cudT+pdu (r,-,) -.P+ r<r
= (T_D _ G_\L-n,P . ,,
n)p== lt,-l _ "-ll*rl,
I

dT
+p
L/-1-("-D^'l
\",';;
AIso,
*= [s3-**'*]',
T= (According to gas equaion)

dT= @dp + vdp)


=l(*) t=)*1,* ...(4.10)

If
1

K=0

Substituting this value in Eqn. (4.g), we get


q=
fi .aw. lr

4.1.6.4. Isenropic expanslon wlth variable specific heats


dQ c,, ( The hea transfer to a system is expressed as :
ud.n\
d, = E lo*)*o / cte=du+d,W
Now inserting the values of
cp= a + I(I, cu = +.KT and R = a_ b
I dQ =
For isentropic process, dQ =
c,,

Q
dT + pd'u (considering one kg of air)

dT +!rod.u
in the above equation, *e
'- Idu-= l.* Y
s"t *o)|
(o , '- .'. crdT+pdv =0 or cu
r =o
fo-) (''6)*o
TNTERNAL COMBUSTION
ENGINES
FUEL-AIR AND ACTUAL CYCLES
169
dT rlt,
'' "u.f *R.:a=O l. _n't
,. --=;)
Inserting this value in.equation (4.18), we get
Inserting the value of.R and c, in the \ 1 s2 - sl = (a - ) lo4 @lvr) + btos"@,/pt) +log,(vy'ur\) + K(Tr_
Tr)
above equation, we get
= a lo4urlvr_b log,vy'v, + b log,py'p, + b log"(u/ur) + K(Tz _ T)
dT
-ir
(b + ' '- Li
+\a-b).
Ifl') dD l,
; =o " l" 2 - ar = a 1og, (vrlvr) + b log" @y'pr) + K(T, _ Tr) ...(4.14)
To express the equafion in terms ofp and ?
.. (o*I(T dr + (a - b) )" o log, (u/u t) = lo4 e,ll1) - log, (py'pr)
== \
I
J
rnregrating both sides, ..
J": ( lnserting this value in eqn. (4.14), we get
b log"T + KT + (a s2-isl= alog"'(?r-Tr)-@-b)log,(py'p) K(,T"-Tr)
;
- ) log" u = constant + ...(4.1b)
OI log" (nt + log, (eF" + log, (vfa 4'1'6'6' Effects ofvarlable specific heats on air stendard efficlencies
= corstat of otto and
(7Y Diesel cycleg
GF @y4 = consta.nr 1. Oto cycle. The air gtandad ef;ciency of otto cycle is given by
Krl'c-\
T.(e)b
-. fl -l
(r)lt-'J =constant ...(4.11)
=r-;==i
(rr' -
T K^ a _a (
i ' f">' .1r;I =constant =1-(r) ",
T p p _a
[ '-'=*)
But,
R @-b) 1-q= (t) "'
p Taking log on both sides, we ger
Inserting the value of I = in the above equaticn, we get
R (1- ) - R
;; to'.r
log
KTa
pG)b .ful =constant Differentiating the above equation, we have

or PfuYtb .eMl = constant r(t\,r.


- 1-'r d=-Rtos"rl-j&,1 =n' ] tos"r
.,.(4.72)
\cu ) c
p"o"t"s wfth variable specific hears
:1.t", ,3:*::y _"o:rn":' 9"i".. " The change in the efliciency with variation in specific heat is exrressed
ffj il
"nt"opyT"f ;1', ri,? ia; *
"'il*,*t*" *::T"*il".#ff
*#;l(s:.,?i;,J"ii.JH#i:R',:Tff *,,ffi;
',;:',l'fi ::":'":f ;:: ch au ge i n
dtl l-r R
as :

d'=#=+,# n =--t1 '1'|'roc'r dc

+I =-+I '(1-l)'tog,''u
-' '..(4.16)
=^^dudr+C (
l'.'
p R\:=- The negotiue sign ind.icates the dcrease n ",
-UUT ffici.ency with increase in cr.
\ .
|

- u) The eqrration (4.16) gives the percentage uariaton in air standard. efficincy
" of Otto cycle
UT*<u*tmt
=(a_b)4! on dccount of percentage voriatnn in cr.

Integrating between the limits, we get 2' Diesel cycle. The air standard efficiency ofthe diesel cycre is expressed
as :

- sr = @ - b) log, (u y'u r) + b log, (T/T r) + K(7, _


s2
To express the equation in tems ofp
T r) ...(4.13) Il=l-
r l-lfpr_rl
_i (.)r_r
and v
ptut _ pzuz
lr(n_r)l
Tr - T2 -^, lz _ !ru, -
Tr= orr, .r-n= I fot-rl
Iog" Ty'T, = log" (py'pt) + log, (v lm_r,1
ti
rtu r)
"rl_r
I

-1
-r-
FUEL-AIR AND ACTUAL CYCLES
ENCINES
but the erpansion taking
Taking log on both sides, we get o
- The reversible ad.iabati etpanson from 3'would be 3'4",
specin" nli"L is above 314'and is represented by_3--4'.lhe point 4'
variable
. - t) log. r 1 log, (pr- 1) - log.
(1 - n) = (l log. (p _ 1)
- log-"
Differentiating the above equation with res-pect toi
1 - "ccount
willbebelowpointa{theidealexpansionprocessstartingfrompointSbeingalongS.4).
Thus, it i8 seen that lbe effect of varin'tion o!:!u'f" heats,is.,to louer^temperatures
ot point 2 an 3 d h"n"" n dIuer-less worh than the coftespond.ing
r-n'dt - pt-t -i ond. pressures
cycle with constont specific heots'
dn
d^
=(1-n)['*."-#ff.i] 4.1.7. Dissociation
r Dissociation (or chemcot equilibrium loss) refers to disintegratnn ofburnt gases
at high temPera'ture' It s a reuersible process and' ncreoses with temperature'
Multiplying the above equation by dt
n oDuringdissociationaconsiderableamountofheatisabsorbed;thieheatwillbeliber-
atedwhentheelementsrecombineasthetemperaturefalls.Thgstlesenel{efr{',of
dq l-n ,^. f,^- - pt.log.p. 1l period and lib-
of the heat during the combustion
n -= ,l ."7 [,oc.".-?ji*tj "; ;;
...(4.17) dissociation is suppression
identical with the effects
eration ofit as expansion proceeis, a cnondition which rs really
But cp- Cv= R pi""."Iuy the change t" ;Jd" heat' However, the effect of d'ssociation is much
when -E remains constant as the changes in c, and c, are considered with the same ate, imolter than that of change of specifi'c heat'
The d.issociotion, in general, lowers the temperature and',
consequently' thz pressures
:. .!-1= &
Lt'in" i"s"""g of i'he stroke, this causes o loss of power and fficizncy'
Differentiating the above eq,r.till r The dissociation is mainly of CO, into CO and O, ;
2CO + O, ZCO,+ heat
q =- 4 R 4' =-t't-t *n =-' at abovt 1000"C and at 1500'C it amounts to
' dc"=- c ' The d.issociation of CO,
cv co co
I per cent.
"o^"n""
dt=-Q-D *t' ..(4.18) There is very little dissociation ofHrO ;

Inserting the value of dy from eqn. ta.f8l'nto eqn. (4.1?), we get


2H" + O, =- 2H2O + heat
Dissociotion is fnore seuere in he clwmicay arrect mirture, If the mixture is weaker,
dr1 plue while if
1- r l- ii" t"^p"rotures tower ihan those ,eqiired. for dissociation to take suppress the
qq ..(4.19) ii i" i.ri,'a"ng "ombostioo it *n g, out io and o, both of which
The eqn. (4.19) gives the percentage varia ,n in ai stsqdard efficiency of Diesel cycle on d.issocintion of CO, -\
account ofpercentage variation in c,.
No di ssocla( n

.ttl
4.1.6. Effect of Variaion of Specific {eats
l
The specific heats ofgases increase with inease
of temperature. Since the difference betw*r, {rrd
"^
is constant, the ualue of , d,ecreases as *mfe$turt
"..
Lncredses. 2800 .\\
o Refer Fig. 4.3. During the ^
"ornpre$ioo
stroke, if the variation of specifrc heat\ is I
2600 tl
taken in account, the final terrperature a\d |
pressure would be lower than ifconstant spe\
cifrc heats are used. With variable specific
o
'..
g
2400
E rl
Wth dssociation

heat the point at the end ofcompression is E


slightty lower, 2' instead of 2. E
o / o
o At the end of combustion, the pressure and 7
_o
22oo
oo
temperature will be lower, represented by P

3'instead of3. It is because ofthe following Fig. 4.3. Efrect ofvariation ofspecificheats 2000
/
redsons
\ i

The temperature rise due to given heat release decreases asco increases, and
The temperature at 2'is lower than 2.
- Degree of richness
--l streng'
Fig. 4.4. Effect of esocition on temperature at fferent mixture
NTERNAL COT,BUSTION
ENCINES
173
FUEL-AIR AND ACTUAL CYCLES
o The dissoci.aon hs a t
o,",#ii':.1;!..:i91"*Tnc.r.ensineqre
\:1","q;;;;-;;;;;ff
crrssociation.
dissociation.
:,::#ffi
\:,-1,:q"::""" ^i,t;; ,lLj,""ilff",,1#i!"Ti;1.r"^l$^1._":
Tn c.r. ensineq
.u .uuer tne tunperdtue re the mirture supplied to an engtg is enfclgd,, This is due to the icreosing losses owing
to
and henc the ,*,o" h"o,to and ei,6bn as the engioe temperatures are raised by enrishment
Fig. 4.4 shows the effec ofdissociam towards te"p""ifii
cLemically correct, ratio. If the enrichnient coniinuee beyond the chemically correct
on bmperaure atdifrerentmixturestrength.
H ratio it results in the zupply of unwabls elcess fuel.leadingto rapid fall io the thermal efficiency'
It would, therefore; appea* that themal eciency. wouldlincrease as the mixture is weakened' loss of
However; beyouil acrtain rveakening the:combustion becomes erratic rvhich results in
efciency; Thus the maximum.ecienlEt lies'within the weak zone trear chemically correct ratio'
-shows (s'f'c')
Fig. 4.6 a plot:betweenmea,efrtive pressure (m.e.p.) us. specic fuel conirmption
at coDstant speed, and consant thmttlersetting.
4.1.9: Efftc of Gohronr Ehne
The efrec of common engine variablee temperature and pressure within the engine
cylinder can be clarly unclerstood by fuel-air , as discussed below.
1. Compreselon ratio :
o The fuel-air cycle efficienry h compreseion ratio in the same manner as
the air etandaril efEciency, this is in Fig. 4.7.

65

60
15 t6
Air-trol rao 4ss
l*
I
Fig. 4.S. Effect ofdissjatiou
on power.
Fig. 4.d shows the effect ofdissociation.on
power. 15
4.1.9. Thermal DfnllncV
and Fuel Coneumption *
whereae air standard t},"
rure strength, fuer-air anaryds, r"t-ot"ioriili ,l vari.ation of thermal po
o
however, efticiency w*h mix-
"uss^i ,hri ,ne thermar efficiency w,r d.ekrtorate as E3s
I E
E
E30
o
ts
25
E m
c
8 3 -4 5 6 7 A 9 l0 11 12
o
ComPresslonratio --------+
o
o Fig. 4.?. Efrect of compressioa ratio and mtrne stength on themal efficiency'
o
s.f.c. ItmaybeobservedfromFig.4.?bhatthecurveforg}Toofthetheoreticalfuelis
trlgher than that for 1007o theoetical fuel, and with still higher percentage of
fuel'
the thermal efficiency drops. It also shows tl.rat maeimum efficency is obtained
Air standard thoory wth leaner mistures.
o Fig. 4.g very clearly depicts the effect of mixture strength on thermal efficiency for
various compression ratios (r).
m.a.p. It shows that r*. (r, at a particular compression ratio, for the fuel-air cycle is
Max. ,.a.p. __-___-
higher for teurr t,iliot"t and falts constantly as the mixture becomes rich' till
Fig. 4,6. Mean eective preesure
chemicallycorrectfuel.airratiosarereached,andfurtherfallsmorerapidlywith
at constan t speffi ;ff f ff f ,",1"' :'#;;-ou"" thefurtherenrichingofthefuelbeyondchemicallycorrectmixtureratio.
NTERNAL COMBUSTION ENCTNES
FUEL.AIR AND ACTUAL CYCLES t75
0.6s
t In view ofthe above, the air standard cycle concept for predicting the performance ofS.I
I o* = engines is misleading, whereas the fuel-air cycle concept seems to be very reliable.

N
2. Fuel-air ratio:
g o.ss -\ (i) Efficiency
F I
o It has been experimentally evaluated

o
0.50
that the rl,. 1 is highest at lean fuel-
o
:0
E
o
o.4s
'8-
:7-
Jr

SN \ ./,n air mixtures of the order of Fo = 0.85


(As the mixture is made lean, due to t
p o.ao 212 less energy input the temperature rise I

during the combustion will be less ?


D

fo o.ss
\\ Ziz which resultsin lower specificheatand F
F12 eventually lower chemical equilib-
6
0.30 N r--- tO rium losses. This results in higber ef-
ficiency and as the fuel-air ratio is
reduced, it approaches the air-cycle
o.25
0.8 1.0 efficiency as illustrated in Fig. 4.10).
1.2 1.4 1.6 o As shown in Fig.4.10, in the range of
Relatiw liel-ar-ratio, Fn Blave fuel-air ratio, FR ----t
-------t the mixture ratios of operation, for S.I. .
'
engines, usually
srrtsrrE, F" = 0.6 to 1.4, the Fig'4'10'Etrectolyi:,"-t::l*-hofr*,,,ata
uuarJ '
FiS. 8 given compression ratio.
fuel-air cycle very closely reserrbles the
t1*. o, is increaeed with increase in compression ratio at a fixed fuel-air actual curve (experimental). The air-standard cycle concept miserablyfails, not influ-
3"
Ihe range shown is from 6 to 24 (In practice ratio. enced by the fuel-air ratio.
the value of r in S.I. engine hardly
exceeds 10), (ii) Masinum pol.e:
As shown in Fig' 4.9, when a graph ir plotted for Fig. 4.11 shows the effect of miture
F/A ratio (Fr) against ratio of fuer-air
crclelndicated thermar efficiency to ai standard strength on cycle power.
"mauor;'-o;t
revearing i, o According to air-standard theory
"".olt
maximum power is at chemically T
correct mixture whereas by fuel- |

1.0
-t-l | | air theory maximum power is ob- :
-fl<P.= t'ols tained when the mixture is about
I o.t $
|
I
0.8 _,\'f 10 per cent rich. The efficienq
drops rapidly as the mitture be-
i
i o.z Or=6 comes enriched ; this is due to
the following resons :
o
]ou I trr-8
F o.s \.0 o'= rp Losses due to higher specific
heats i
o.4
0.4 0.8 1.2
\ Chemical equilibrium lmes ; o.7 0.8 0.9 i .o 1.2 1.4 1.6
1.6 2.O Insufficient air which will Relative fuol-ar ratio, FR ----+
Relative ftrel-air rao, F" result in formation of CO
Fig'4'11'Efrectofmixturestrengthoncyclepower'
and H, in combustion, rep-
F\s.49 resenting diect fuel wast-
It is worth noting hat for a particular value
of Ir, the ratio of fuel-air cycle
age.
indicated thermar efficiency and air standad (iii) Marhnum temperature :
there is no variation.of
specific heat and no dissociation and tberefore "f.l$.y F. diferent compres-
peraure must reach. But bt-fttel_oir qrcle
*i"" 4 = 1, the maximum tem_ FiC. 4.12 shoKs the effect of on maximum cycle temperature Tt(IO at
concept the-nazimum sion ratios.
shifted. to richer ualues of relatiue ^ temperature is
fueiar The maxirnum temperature at a given compression ratio is reached when the mixture
^,ti"l - is slightly rich,
6% or so as shown in Fig. 4.12.
t76
TNTERNAL CO{BUSTION ENGTNES
AND ACTUAL cYcLEs 777

. s"i'=fi It follows more or less the same pattern


- the as the maximum temperature versus f". Bz

2800 4 Cofi p. ralio


maximum
that of
occurs ot stightty higher value of relotive fuel-r ratios as compared to
temperare. This ia owing to molecular erponsion. There is increase of the
ii
I
l1
Y
; '/t
7t)
/il \ \l (v)
mole of products after combustion.
Erhauet temperahne
h-'
st 'e600
N'i Fig, 4.14 shows effect of F" on cycle exhaust gas temperature, Tr. The ethau3( gas tenpera-
il g
o /z 7 \; ture is masimum at the chemically correct mi*ture.
x
iG
2400
/t i+_F.
-+- |
= 1

/
't.0
-\ ompfel ision
|ali

Fn____-_:_=>
1.4
I ,/i\
Fig' 4.12. Effect ofrr o! p;mum
ii (Io at different compeasion ratios.
cycl, temt'eratue
I
V
- In case ofa chemicalrv correct ratio there
because of chemical equilrbuur ere;il
is etill s.ome oxygen present at ttre point
;#;;cb mixture wilr cause more tuer to3
combine with oxygen at point S, ,"i"i.;;h;-;;;;;
,-.
1 600
0
*
oP
10

:ffi L:?;ilffi;;irbo*J*"",".-';;#;;l*"Tff""r:,TLlilr,ll,?:fl #it t5 /


(iv) Maximum pnessz,ne
n.tt showe the effect of .F., on maximum
cycle pressure
.ro., rs (bar) at fferent compres_
""Xob..
110 0.6 0.8 t.0 1.2 1.4
tl
Cotp. rao
RBlativs fuel-ak ratio, F ---------)
\t I
tu Fi9.4.14. Effect ofF, on ?"..
n
6
I
The variation in exhaust temperatures at a fixed value of f, shows that it decreases
I with increase in
compression ratios. This is due to the fact thot increased erponson
ratio causeslhe gases to do more taorh on the pi.ston leaving lesc heat rejection at the
e 60
J end, of the stroke.
6
E 6 The results are similar, for variation of compression ratio, for the air-standard cycle as
g 40 r well.
o
o (ui) Mean effective precsune :
i
:d
20 Fig. 4.15 shows the effect of F* on m.e.p.
a m.e,p. increoses with compressio ratio as effrciency increases.
o It follows closely the maximum pressure curve, but the m.e,p. occurs at slightly less
1.0 1.2 1.4 1.6 richer mixture ratios as compared to that for pressure.
Fn------------+
Fig. 4.13. Effect oftr'" on maximum cycle pessurep,
(bar) at different compression
ratios.
NTERNAL COMBUSTION ENGTNES FUEL.AIR AND ACTUAL CYCLES 179

are congtructed assuming enthalpy and internal energy to be zero at absolute zero
t r:F
cprqnrbn
_ rrdo temperature and integtatiog zero pressure specific heat equations fron 0(K) to the
iii
itr
I
I
.{i F \l
-rol given temperature ? (K)'
oF 'oI z .F,: I
-
The most important assumptian is the validity of the equation pv = RT, where u, R
and ? denot pressure, specific volume, the characteristic gas con8tant and the abso-
ci
c"t ;F 24 $ lute thermodynamic temperatue respectively. This is tre wlvn the gos lra.s o crica'l
6 ternperdture very liw as compdred to the rane of ternperature met wih in engineer-
f ef ing applications. For air at 0"c and 20 bar the deviation is only 1 per cent; ad at 0"c
g
o
o
'f t- Fn=
lhx. Ll !\9.

1.1 -
antl 1 bar, the deviation is 0.1 per cent.
The enthalpy and internal energy are function of emperature and, therefore, their
values can be computed wih sigJ voridble ProPert!, i.e., tempera'ture' -
o :F Thus enthalpy l and internal energy at any temperature ?. (K), are giveo by :
6
o T
^L n= Jo'
| c^dT
"o.6 o.8 1.0 1.2 1.6
Rshfiw fil-er rali], Fs ---+
,= lr",ar
JO
Fig. ,L15. Efiect ofF, dr m.e.p.
The entropy chonge nvolaes both variables namely pressure and tetnperature,
4.1,10. Characeristice of Contant Volume Fuel.Air Cycle
Ingastables, h,p",u,u,andQarerecordedfordifferentvaluesoftemperature?(K).
The constant volume fuel-air cycle entails the following characteristics
:
1' The variables other thaa cornpreesion ratio and fuel-air ratio have little effec on the Rolative Peaaure, Pr :
dT - ...(4.20)
efficiency. T ds = dh - a d,p = co u d.p

2. The efficiency decreases as f/.1. ratio increases, (': dh = crdT)


when tbe latter (f/A ratio) is variable.
3' When 'F! is above 1.1 approximately, expansion For isentropic process' ds = 0, we get
temperature decreases with increasrng
FlA ratio, but the combueion is incomplete because mirture is rich and o = cpdT -v dP ot v d'P = sofll
the net result
is decrease in efficiency with increasing F". Dividing by pu = R?, we get
4.l.ll. Conbutlon Cherts
o The thermodynamic charts
vdp _codT
pu RT
or +=? +
embodying characteristics of cylinder gases are employed
for computing fuel-air cyclee, avoiding laborious celcrIations.
...(4.2t)
o There are separote charts for fuer-air mirture ar
chrts
prod.ucts of combustion.
for The " (*)=* f""o-f;=nro,>
l:?dj:t".1":nlytlol
and ae used for
cz for fferent fiut_clir ratia (say.F,
= 0.8 1.2) ;
of pressure, temper@ture, volume and eneigy at uarious
where 1.o is selected as a base temperature. It is seen that ratio Z i, function oftemperature
points in the cycle after tahcn place. Po "
o The chart for unbumed niturc only, and is indepeadent ofthe value ofentropy.
for &ulatian of pressure, tempera-
ture, and energt of the cylin&r anten burning takes places. From eqn. (4.21) I, can be calculated in termi of tempeiature'
- an isentropic path, for two states 1 and 2, we have
On
4.1.12. Gas Tables
The problems involving vaiable specifc heats <{ be solved by the folrowing
method,s i
\= pdpo - Pt, ...G'2)
!. By t>ttegration of epecific hat equation. P," hlh P2
is tedious and time consuming.
2. By enthalpy-entromt clrrs. The mqior Thu, the ratio of relative pressure for two states having the same entropy is equal to the
of this method is large enough ratio ofthe absolue pressures foithe same two states i pe is chosen as unity for computed values.
charts which can yield fairly accurate resultsrae not available.
3. By gas tables. (Gas tabies 6r air are givea in Appendix) Rlative volume, u, :
t fls = d,u + pd.u ...(4.23)
The gas ables which taH'into cousideration the variaion
of specific heat wih tern- For an iseirtropic process, we have
perature, give the enthalpy, intemal energy and energr
function and have been com_ 0=du+pdv
puted and compiled for many gases and mixture ofgases,
incruding air. These tables pd'u=-ilu=-cdT
180
INTERNAL COMBUSTION ENGNES 181
FUEL.AIR AND ACTUAL CYCLES
Dividing by pu = ft\ ss ge
The enthalpy, relative pressue, and Q of air are computed by cornbining the corrsponding
Nu __crdT values ofthe three constituents ofair according to the law
PU HI h=Z'rhr ...(4.30)
/,)- l f dr E=8110 ...(4.31)
Inl- - -
-('t-i l'"% 'l;- = tn (u) ...G.24) ln (p,) = I'r, ln (P.,) "'(4'32)
u" (designaed as relative volume)
is a nction of temperaturc where.r, denotes the mol. fractibn ofgas 1 and the summation ie carried over all costituet gases.
only.
The internal euergy is calculated by using the following formula :
Alernatively, ,.= 4 h=u+pa=u+RT
...(4.25)
Thus for any hdo states on ilr" illt.opi", or u=h-RT=h-0.287 T.

5= r, 4.2. ACTUAL CYCI,ES \


ur, q ...(4.26)
Entnopy funcion, rf
4.2.1. Introduction \
: In actual engine operation the followi\ losses occur, due to which actual cycle efticiency is
Tds=cpdT-vdp much lower than the air standard efficiency :
1. Dissociation losses
e=4!-q! "na TpI
-1= 2, Losses due to vaiation of spocific heats with temperature
3. Time losses
d,s=cp.#-- 4. Losses due to incomplete corrbustion
Ifthe zero for entropy is +
aken at ?o temperaiure andpo
5. Direct heat losses
preseure, wherepo unitr, we 6, Exhaust blowdown losses
= have
7. Pumping losses.
,=
[o"o.t-nnb) specift.c heats and dissociation ore subtracted from the "air
o If losses d,ue to variable*fuel'ar
standard cycle", we get cycle analysis''
= q _.R In (p)
...G.27) . Furthermore, if other losses are further subtracted from "fuel-ar cycle arwlyss', we
where g '"-{
= JTorT can uety closely approimate the "octual cycle'.
...(4.28) 4.2.2. Carueee of Deviation of Actual Cycles frorn Fuel'Air Cyclee
Q is' therefore'
"
*T:,:T:;,,'i Important causes of &vntion of actual cycles from fuel'a'r qcles ore :
1. The progressive combustion rather than the instantaneous combustion'
...4.29 (a)l 2. The heat transfer to and from the working medium during compression and expansion.
3. Loss of work on the expansion stroke due to early opening of the exhaugt valve, and
0z-0r-Rhrr-l exhaust blow down.
=
...14,2e (b)l 4. Gas leakage, fluid friction etc.
\P,' )
The properties of air ae computed
by assuming the following properties 4.2,3. Real Fuel-Air Engine Cycles
ofair :
Ct The actual cycle w'hich an I.C. engine experiences is not a thermodynamic cycle, in a true
Molecular weight
Percentage by rclunte sense. A id,e:al ai-standad, thermod,ynzmb cycle occurs on a closed' system of coistant compo-
Oxygen
32.00 sion. This is not what actually happens in an LC. engine, and for this reason air-sandard
20.99
Nitrogen analysis gives, at best, only approximation to actual conditions and outputs.
28.016
78.t Major differences are listed below :
ArEon 39.95
0.98 7. Real engnes operate on dn open cycle with changing composition. Not only does the inlet
gas composition differ from what exits, but often the mass flow rate is not the same.
"u.o""f,xi,lo'ecularweightofairand,u,*,,.,*ffi7kJ|kgK During combustion, total rnass remains about the same but molar quantity changes.
- There is a loss of mass during the cycle due lo creuce flow and. blowby past the pis'
- ons. Most ofthe cevice flow is ternporary loss ofmass from the cylinder, but because
-lr
INTERNAL COMBUSTTON EICINES
FUEL.AIR AND ACTUAL CYCLES 183
it.is greatest at the start ofpower
stroke some output work ie lost duringc_fsn*6, l,
or'o.'. i"'liJlriilu", 1. Time losses :
by as much rd ao"i,,g i
"fi;:LffiiTffi'fithe.3mouat "i Time losses may be burning time loss and sparfr tirnings loss,
2. Air-standard analysis tree (a) Burning tim bss
il

,n crs an idat sas. to o r""l!,)!.t the entire engine as air'a,.d approti-


ffif '";;:,T"*!":X'i,,:i1#:i;'"ii:;"#l"f
fuet, either soseous
l-yj 3:: !"-ryh::f;""::Yi:X;rfffiiTif ; The burning time loss or merely tine loss is defined, as the loss of power d,ue to time
"i ^li"i-o',T;:;. required for mring the fuel with air and for complete combustinn.
f; *ffi:f*:":,*::::::^tr.1l uyj in a engine cycre were air, sonu er,a: woutd. o In theoretical cycles the burning is assumed to be instantaneous, whereas in actual
ii::::i::;!#,r*ix:;J;*i#;;##;;:"v:i::&?kKtr,#:l
lr:s-sures of inlet and er,haust,' -- w"
higher
nsher pressures
*J,T";:';::oe tleated
^- weututelv "p"clllc
hcats. At tle tow
::::'"!, ;;:r;"d;"';";l';;"
idcal goa., but at tfu
I
cycles the burning process is completed, in a finte interual oftne. T:he time required
depends upon :
du";,- i
pressure" during ",
^-ti,:I^::y:":fU.be 'rearect -an
dlvrgte froy ,a"" i""-u"ri#,rl'Ir"
-o8
I
(i) Fuel-air ratio ;
senous eror
serious error is inrrorf,m,r i!,:!:.:,_?!,.ilt
ig introdiced
, ;i j::-"::l
cific hears of a gas
cific-heats.of uu!mr specltc heats-for;t"
speciric heats for the n'lysig. Spe- (ji) Fuel chemical structure and its ignition temperature ;
hao.-|
sas h""" strong :11t-t-T' "-ffi.$"- *
on temperatrr" ord--ilit
much os eo%J0% i t"p,J)i";";;;::"tr;,y".
ttte ,.^*,rj,r,!
: in tlp
:":h the cvcre of a real
3'.During
-dependen.c)t
tr""";; il;l;;;t
(ii) The flame velocity and the stance from the
combustion chamber.
ignition point to the opposite side ofthe

analysis. ""gio" eh atenegr*tedi air-stsndaar o The time required for combustion is such that under all circumstances some increase
in volume takes place. The time interval between the passage of spark and completion
- i:1iil*"1T*fr""*::f:i::::
what is predicted. ihr;;:'-" l'tl:rplt temperarre aud pressure from
vwED ulu8r peai( temperature-and of flame travel across the charge is approximately 40' crank rotation.
work outnut *l"".,"l.fi,Ti:t
ouiput during jT:k:, therefore, strtg at r ro.".'in"outi,'."a Fig. 4.16 shows the losses between real cycle and its equivatent fuel-air cycle.
lork expansi"":; ;-";;;;:
- sure
f,::,r|;nttr
below the L:1ru:l*:lr-*::::i:drhis
ideal isetml '
..q !u ruwerB-Etle
rowers the Emperatue
temperature and pres-
pres_
or neat transfer in lawo.,::::^Tjn!*Td*hg end of the poweist ole. ffreTsurt
::lAL,:*"""i,.ro*",il;'ff
analysis.
;r#H:"d:,"J#tr.fi ffi :ilTyhTffii
Heat transfer t".-:!o.p:":".J during
- conpression, which deviates the prccess
;:fi:::#"ff""' ti i' L"'-';;'T;' h";;;J;;ffi from
fr,"" to the rower
requires a ghort but finite ine
o"" , t.s:bustion to occur, and heat addition is noti'stantane_
1
- i;lr,:"lir,"!r1
engines generallv have conbustion
efficiencies of about 95s ad 98%
6

E
ff"*flj;ffi:TT-, . ". rS"
.uE uq
a H::11_Tib.yr
occur at constant volume
ffi:
nnite cycle
13 a Dnte time, and doe not
cycre rime, nor i
"*"':H::nj:::::'::il;ilffi
r"""Ei;:':"?:::Tl*_,::Y.'i.:j"i&1'Hili:[1;lli""*ffi]
open 40'to eo; T:" '".'o", tir"
"'L;;;;
T'i plI
rl
E
L

j*r*;ir'iffiHTiJlTleeoort" ,tbl

'g!"1;ff:":g*ilY:;,,ft ffiT-.::r-"
.lT"ffi
rlq:or t\e varve, air j; j i
l"l",htif,ffi*""ff l,i,*lF*i;TJ'Jff ""#ff;,?:fi S ;lBH*:fl "i';"J,tJ#;J:,T
j *::ffi :
iilji,Hll,":m::f l*".1:"iiiCitTiIiiil",f :Tft ,Ti;
,i""",t::,::"**,"r.oc"t"i;;;i;#:ll;ff
dicted by air-etanaara cyct"s. iinl is sE
ignition
thr pre- 1234567
7. Engine valves rcquire a Compression ratio
finite time to actuate.
-)
Fi!,.4.16. The effect of time losees onp-u diagrm.
4.2.4. Difierence bgtwecn Real
Cycte nd Fuel.Alr i1
Assuming cycle to consist, of co^mp""rri"r, it The effect of nite time being iequired fo combrstion is that the maximum.pres-
tween a'ral cych aqd it. *i Irp"rrrio' onl the frerenceg be-
t1i
ii; sure is not produced when the volume is minimum, as is expected. It is produced
"q"iurr""in to tol factors :
ii$
sotnetime after T.D.C. The pressure therefor rises in the first part of the working
"_J"""i"'r.i"r"eTue
losses (Icluding combustion
.t. Jime loss)
stroke from to c, as shown in Fig. 4.16. The point 3 represents the maximum
2. Direct heat loss
3. Exhaust blowdown loss
pressure had the combustion been instantaneous.
.li The difference in area ofactual cycle and fuel-air cycle shows the loss ofpower (the
4. Pumping loss
hatched-aea).
5. Rubbing friction loss.
184
INTERNAL COMBUSTION ENCTNES
FUEL.AIR AND ACTUAL CYCLES 185
6)
Sparh timing bct
After generation of spart in the 4. Purnping lossee :
fuel' Tte effect of this is tl.i rylinder, a definite iSguirert to start the burning of
-":ir""r n""rsIriolr"..ru, fim.e
T.D.c. and it reachep late in the o The pumping loss is d.ue to pumpitg gas ftom.low inlet pressure to higher ethaust
":iffi:i stroke' ?l time which tri"w ax uorxing the arryte of atane (sparh preSsure. \
"nilr#u" r The pumping loss increoses 4t port thrttle because throttling causes reduction in
(j) If the spark is given at T.D.c., the maximum pressure suction pressure,
is lo-w
due to erpansion of gases.
() If the spark ie advaced by 40' to stat
com'bustion at r.D.c., the combustion o Pumping loss itureases with increase in speed..
takes
liilL*1?"?"X;t*lld'ro"'"-Jln'l*iao"t."Jmuf il'hu;iagainwork 5. Rubbing friction logg

cases, the work aea is less, The rubbing friction lossee are caused due to :
and, therefore, power &oeloped per qcre
and.
"ffi"t"r},t::"
Thus for^y;::.hro
getting maximum work output, a modrote sparh adaoncc (ii) Friction in various bearings ;
best. of 15" to 25- is the (i'i) Friction in auxiary equipmey't such as pumps and fans.
btaomplete combustion lllslrac The pison friction ncrea|ls rapd.ty with engine speed and to small extent by in-
The loss due to incornplete combustion - creases in m,e.p.
- homogeneous fu"I-ait -r-ur" d"
is incruded,in time loss. It is
not poosible to get
il;"";;; The bearing a-at'yotiuy friction also increase with engine speed.
are present in te engine cylinder befo"e-t'ue cvrinder as tuer, air and iesiduar gases -The engru effi.cicncX
start of ignition. There may be excess is masimun at fult tod and, red,rces with ttt dzcrease in load ft is
o:rygen in one pat and excess n
ul i" due to the fac[ that irect heat loss, pumping loss aud rubbing friction loss increase at lower loads.
"not'"".part of the cylinder. Therefore, izcorn_
':;;:::n'ii-:##'{lT:'"tni*"""r';;;;;f*t'"1;;:;;:;"bothwiu 4.2.6. Comparleon of operatlone end working Media for'Air cycle" Fuel-alr cycle'
It is obserued that ernrgr and 'Actuel Cyclp'of S.I. Engines
- rerease when comprete combustion
reredse in s.L engine is, onry about g5 per cent
of the encrgy
-'tiu1"i" nl:e w_ith nar stoichometric 1. Air cycle :
ratio. rn actur engine, energv release
i.1""io t" ss f""-*oiiriofi.Il.ry fuer-air The working rnedium is air throughout the cycle. It is assumed to be an ideal gas with
- It is arwan prefer.able to.use a lean mixtureto. e-liminate fuer waste, wh'e
inpot.
constant properties,
mixture is required to utili"e all te,oxvg"" a rich
efficiency, but too lean_mbture will
. s!eIii-r""" ,nsr,nlu,r The working medium does not leave the system, and performs cyclic processes'
burn causing tot"r fyt
buii ^rr"*"*iJ"
stowty, inc.rease burning loss or may not o There are not inlet and exhaust strokes.
rr *tt ii-rr."
amount of oxygen i n-otr"availabre . ""-" r,i"1inisrich
definitetv wste as adequate o The compression and expansion processes are isentropic.
Th; rt;;;';p"ed, mbture is bu) and, causes
burnins titte toss teading to t"i"r"i-lif,lf"ilr"y. o The heat addition and rejection are instantaneous at T,D.c. and B.D.C. respectively, at
2. Direct het loss: r constant volume,
when combustion of fuel t{1
n,""" fotowed by the expansion stroke, the 2. Fuel-air cycle :
takes place from cyrinder gaseJthroug,. flow of heat
water jacket or cooling onr.:_i-!*l
tt" *"ts and cylinder head into the a The cylinder gases contain fuel, air, water vapour and residual gases.
f,""i "vi"i""
ri"i"'ii" pirtoo head and flowe ro the a The fuel-air ratio changes during the operation ofthe engine which changes the relative
"i
:ii:i::ffi ,,SflJn,''::;;;;;i.'."."'il'i""*."ry*,uoici;J"ri*,i.r, amounts of COr, water vapour etc.
The loss of heat which The variations in the values of specific heat and 1 with temperature, the effects of
d":rl,::" during combustion has tbe maximum dissociation, and the variations in the number ofmolecules befoie and after combustion
; *p"*i t*t" i jl'i'i'r effect, while
;; ;" ":,r
:i:1 i:':3;f,":rl3;;;;'j,f "'i' " "tr".r, ;;;;';l are considered.
r During combustion and expansion, Besides taking the above factors into consideration, the following assumptions are com-
about 75% of the totar heat is lost.
however, much is lost too lte n tL out of this, monly made for the operation :
nave done any useful work.
r "y.t"
In case all heat ross is recovered, aboot
20-per centofit may appea
() No chemical change prior to cornbustion.
as useful work. (ii) Charge is always in equilibrium after combustion'
3. Exhaust blowdowrr loss:
At the end ofeaust stroke, the cyrinder pressure (iii) Compression and expansion processes are frictionless, adiabatic'
opened at B.D.c., the piston has,t is about 7 bar. Ifthe exhaust varve
o *rrlg"iri"irg cyrinder pressure is (du) Fuel completely vaporised and mixed with air.
exrraust stroke. when the exhaust val"" costing part of the (u) Burning takes place instantaneously, at constant volume, at T'D'C'
i. o;";;;;; rry entrre part of the expansioD
lost' Thus' best compomise is.tht.exhau;r;;;"ned stroke is
reducing the cvlindei pessure to tran"ayJo 40. i" io;i"r""rii.c., rhereby, The fuet air cycle gues a very good estnate of the actual engine with regords to efficency,
ai;;;# before the start of the exhaust stroke. power output, peah pressure, exhaust temperdture etc.
186
INTERNAL COMBUSTION ENGINES
FUEL.AIR AND ACTUAL CYCLES 187
3.
aThe working substance is a mixture of air
and fuel vapour, with he products of
Example 1.2. Determine the effect of percentage change in the efftciency of Otto cycle
combustion left from the previous cycle. houing a compression ratin of 8, if the specifi,c heat at constant uolume ircreases by 7.7 percent,
a The working substance undergoes change Solution. Giuen : Compression ratio, r = 8
in the chemical composition, ;
a Variation in specific heats ta'.e place.
AIso the temperature and composition changes
due to residual gases occur- Increase in specific heat at constant volume, 4n = t.tC"
a The combustion is progressive rather than co
instantaneous.
a Heat transfer to and ftom the working medium dq:
to the cylinder walls take place. Percentage change in Otto cycle efficiency,
a Exhaust blowdown rosses i.e. loss of work due q
to early opening of the exhaust valves
take place.
o Gas leakage and fluid friction are present. The Otto cycle efficiency (r) is given by :

,1=1_
WORXED ErAMPI.ES (dh
Example 4.1, Ded.uce_an. etpressbn
for tlu change ,n entront of t hg of gas in terms Now, cp. cv=n, t-t=
!= constant
!!: ^l*"t",.: T, o\!,T? Unre ang if the law of compression is pun *
:n"f'-^i**ioa
and the specific rra,ts an of tlte form,
co= d + I(l n = I - (rI'{1-r) = 1- (r)-sq
or (1-q)= (r)-Rlc"
Solurion. We know ,n^r, o\:uol Si
= co d.T + pdv
or ln(1-n)=-4lo(')
cu

Dividingboth sides bv r,# ="" # - + Differentiating both sides, we have


I n ..
ln (r' x dcu
- 1-t1 on =
*=",T * | .o, ;E.
. (l-n)Rln(r). i
C=-
- -------'-ClC-
Insertinithevalueofcu, *=U# +Xaf +
f; au ...(t cun lir
ill
Also, Pv=EI (1- rlXr --11c, x ln (r)
pdu+ud,p=ftfl|
ar., = - .
iil
Pv"=C dq _ _ (1- nxy - 1) ln (r). dca
^ ll

P . n(u)n-t . d,u + un .dp =o ".G)


nflcu I
I

nPd,u = - flp
..ui) I
From (li) and (lij), we get n-1--=0.565
' (8)'- -'
pd,u -npdu =M?
dq _ _ (1- 0.565X1.4- 1)ln(8) 1.1
,MT
=
Pctu q 0.565
x
100
r- n
- - - O.7O1% (decreased). 6ns.)
Inserting 1ir (l), we get ds=[ + KdT+(1 R
-n)'
dT
T =fa- n
| (n-l)JI
dT
T + Kd,T
E:anple
ratio of 7 to 1 f
13.. Fnd th percentage change in efftcizncy of an Otto
the specific heat at constant uolume increases by 39o.
qcle for a compression

Integrating, we get f-nlr- Soluion. Ttre change in efficiency wlth variation in specific heat is given by
f -C=ltl 'ot"t *Ksz-r)
s2-sl= :

Substituting, R=a-b,
dtl l-tl (r- &
=- 1) loe-, ...(t)
nqcD
s2- s, =
f'-.-+t] ." E T2
+ K(T, - Tr). (Ans,)
1- =t - = o.s4r or
n=
orL ,+- 64.7%
I88
INTERNAL COMBUSTION ENGNES
FUEL.AIR AND ACTUAL CYCLES 189
Inserting this value in the equation (i), we get
2
dn _ (r-oslr'\ .,. .
= - 0.226 (2.89 - 1.065 + 0.714) x
100
=--0.01147 =-t.147%
n =- l 0541 )$'4-1)los"7xo'03 Negative sign means decrease.
= O.0r98 - or
-1.98%. (Ans.) Hene percentage decreose n effuienca = 1.147%. (Ans)
Negative siga indicates dcrease in efficiency.
Exanple 4.6. Tlu following data relate to a petrol engine :
Example 4.4. The foilowing particulars relate to
a Diesel qrcle : Compression ratio =7
Compressbn ratio = Ig,
:uf?tr =^ S% of stmke, mean specifiz heat cu for cycle
hJlhgK, charocterisc gas constclnt = 0.ZI Calorifi.c value of fuel used. = 44 MJlke
= 0.2g5 *Jhg X. The air-fuel rata =15:7
Knowing that th specific hzat increases witi temperature,
if the nzan sp*ific heat for the
cvcle in*eases bv 2 per cent deterini--ih" p.r""ntog" The temperature and. pressure of tlu clurge ot tlu end, of stroke = 65'C, I bar
\'oi!!Hi* change ii tti air stand.ard Ind.ex of compression = 1.33
Soluion. Compression ratio, The specfic heat at constant volume, k) = 0.71 + 20 x loa T hJlkg K, when T is in K.
r=18
Cut-off Determine the martnum tressE-i1{cylinder. Compare ths ualue with thot of con-
= 5% of stroke stant specific heot c, = 0.71 kJtkg K
Mean specific heat for cycle, \
cu = o,7l kJlkg K
Characteristic gas constant, R = 0.285 k/kcf^ Giuen:: r =
Solution. Giuen =77 :i C = 44 xld kJ/kg ; AY{ ratio
kJ/kg;AY*F' rado = 15 : L
10p ; Tt = 65 +273 = 338 K;
L;Tt=
Increase in c, . pt= lbar;n=1.34,c,=0.71 10.5TkJ&gK.
=2%. \20x
Change in air-standerd efciency : Refer Fig. 4.1?. p+ \
The percentage variation in the air standard
tion in the value of c, is given by
efficiency on account ofthe percentage varia- Maximun pessura in the cylinder, p, :
Consider compresson process 7-2:

+ = (+) rr_u
[r"s","_dyqr.il * (,)

[Aticle 4.1.b.6, Eqn. (4.19)]


pyvln = p2u2n

pz=ptl
/\
1!l =tx17tsr
\u2l
- %cutotr
p=- ,.
ruu (r_1)+1=_
Now, 5
*^ 100
(18-l)+1=1.85 = 13.3 bar
DtUr DoVe
and
T=1+!=r+99=r. Now, -.1=
c! 0.71 T
Also,
dcu 2 To=''(W)
c 100 "' - .\prq)
Efficiency ofDiesel cycle is given by

1 fol-rl = .tt " (# f) = unr., * Fig.4.1?


1=r--yl(,t=r I frr'aslta-rl "
L*t1=r- i4otFr:r Lffl Average temperature during combustion of charge = (**)
=7-0.2248 (#) = 0.6887 ot 6r.Bivo Mean specific heat of product during combustion,
Now sbstituting the values of and other quantities in the eqn. (j), we get cu mean = 0.71+ 20 x 10-6l^#l
dn (1- 0.638?) . l, (1.8s)11 tos, 1.85 1
'l Assume 1 kg of air in the cylinder
n-_ t8 - i=++!_:ie. .
.
x (1.4 2
- 1)
0.6s87-
ro& *l _ Heat added per kg ofcharge = 4# = 2750 kJ/kg ofair.
= - 0.56s2 , o.r - !1!I + 0.714] x _L Q = mass of charge x cu x (Tr- Tr)
[z.ss
zlso = r, ,o*ro-u(r'*:n )]
,r, - ur.r,
[o.rr*
T.-..-
1t
.l

190
NTERNAL COMBUSTION ENGINES l

FUEL.AIR AND ACTUAL CYCLES il


2750 = O.1L (Ts _ 642.2) + t0 x 10$ (7, + 642.2)(7, _
il
ll
642.2)
Mean specific heat during constant volume heat addition
27Eo = 0.71 ?s _ 4s6 + 1f (?i2 _ 4tz42t)
(T, ^'
iii
ii

e750 + 456) = 0.21 ?s + LO_t{z _ 4t.21 cu*on, = o.zll? + 2.t x loa


or ,* Tt" + 0.7tT, = 3247.24
IT,J
Thus heat added at constant volume/kg of charge
or +
42 7700 \- 8.247 x t0? = 0 48000
= 16 = 3000 kJ/kc ofcharge

r, -- ?1oo I rf?ldt;;;;z;67 Ad aooo = fo.zrrz


L f&+Eqll
+ 2.1x 10-a
\ 2 )l
x (?3 - 256)
-7100* t3427
consider constd,nt
= 3163'5 K .'. 3000=9.7117 r,*2!x104 7"2* 2'lxloax756 T.-0'77r7 x756
,oru^" orn:""J3-_ "2
p2 2.1 x 10-a x 755 ,- _ 2.LxLOa x rDbx
T,=b
Ts
_
2
lt-
z
/t)b

= 0.7tL7 ?, + 0.000105 - 598


: e,=+=13'3x#=65.6bar. (An)
Ts2

ror constant specifrc heat : = 0.000105 T] +o.7r!7 TB-5e8


2750 = O.7l x (Ts or 0.000105 7"2 + O.7L77 ?s - 3598 = 0
Ts = 4515.1K-
- 642.2,)
Solving for ?r, ?g = 3370 K
.', Maximun pressure n the cycle,
Ps= 13.3, 98.6 ba. (Ans.)
W= bar.
fr =rr.tr. #
Exanple p3= p2x = to2.l (Ane.)
4'6' An otto cycre engine u.ith
,"W:::l:Z:::i?f,# iry
a compres.siolratio of
r. rhe
70 petroreum fuer of
uses a
When c, remains constant at 0.7t17 kJ/kg K
"i,-r""l,i:;;';', tempera,,n o,:)n"uure or the ;

,:1"**.t:#::"T#:i:;Ti::Y,"i:xItir"ji,T?;*,ilT,#"Wr*:::::;":; 3000 = 0.7117 (?s - f56)


or Ts=
ffif 756=4s7t:K
c" = 0.Z1lZ + 2.I x IF hltkg K where T is th,e mean temperature.
ot i'ztti nnng K, and
and Pr=er* =22'9Lx # = tuo'tbar' (Ans')
,,",,"!,!J,o|ii';;,"!r:'#:::constant atso
"o'^p,","io, ind,ex is
sorution.
"".o""..;f
Calorific value
:i rT'""',0! i{'""0 F Example 4.7. Combustion n a diesel engine is assumed to begin at inner dead centre and,
to be at constant pressure. The air-fuel ratia is 27 : 1, the calorific ualue of the fuel is 4300O kJ I he,
ofpetroleum fuel
Air-fuel ratio
= 4gOO/kJkg and the specific heat of the prod,ucts of cornbuston is giuen by :
Temperature ar rhe end of sucrion, cu= 0.71 + 20 x 10-6 T ; R for thc products = 0.287 hJlkg K.
,r-lXt*'rrr=
'- Bit0 K Ifthe compression ratio s 15 : 1, and the temperature at the end of compression 870 K
.. hessure at the ed of suction,
Oi = , U* ftnd at what percentdge of the strokti combustion is cornpleted.
index of compression,
z = 1,g6.
n*'
_Mean
tft*= eY;sa
Solution. Giuen : Air-fuel ratio
Calorific value of fuel,
=27:1
c = 43000 kJ&g
:" er=etx
Specific heat ofproduct ofcombustion '. cu= [.ll + 20 x 10{ T
= 1x (10)1'36- 22.srbat .R for products = 0.287 kJlke K
lk)
Compression ratio, r=ID:I
Arso, * =(*)+ - f22e1)*# Temp'erature at the end of compression, Tz = 87O K
) _2.2e
;"=rYln.rn=l*t*
- ^- Percentage of the stroke when combustion is completed :
For 1 kg offuel the charge is 28 kg and the heating value is 43000 kJ/kg
INTERNAL COMBUSTION ENCINES FUEL.AIR AND ACTUAL CYCLES
t3p
dQ=m Solution. Giuen: ?r = 87 + 273 = 360 K;pl = 1 bar; r = 14;
lr"ro, ,
cp= cu+ R gr" = # = 850 kJlkg of air ; Qs- = 850 kJ/kg of air ;

=0.21 +20x10r$T+O.2g7
yfor compression = 1.4 ; R = 0.287 kJrkg K ; cu = 0.71 + 20 x 10-5 ?
= 0.997 + 20 x 10-6 ? Consider compression process I.2 :
)e^M cI"
)' (0.ss7 +20 x to-s T) dT prul = p2u2r
# =

lTa
= Jr,=rroo'997 dr +(20xrc-5 T dr) P2=Px =1x(14)r'4=40.23bar

r, ,,.\'
1585.? =
fo.nnr.*ro"
,o-t.+f,=r^ --rf % l.-_ rr--(1,, )
Fig.4.t8
/.. \ '/-l
or 1535.7 = 0.997 (?s
- 870) + 10 x 10 (TB2 _ B7O2) r"=r,l2l
' '\u2) = 360 "\0,,..' = ro84.6 K
or 1535.7 = 0.997 Ts
- 867 .4 + I0. Ts2 _ 75.69
or Consider constant uolume p rocefs 2-3 :
l}aTs2 + O.gg7 Ts- 2478.8
or Tr2 +9970 \-2.47g8 x 107 - 0 Qt_z= n
r:""
o, Fig.4.19

-=_
T _ -9970tV(9920)z +4x2.4788xI07
= r zoxro-' T)dr
2 Jaro.zr*
- 9970 I 14091
K | - zlrt
Now, for constant pressu"" 8s0 = + ro * to-"
rlr**;;:2060 10.21? ?.1r"
u2
T;--!3
T"
or 850 = 0.?1(? - ?r) + 10 " 10-5 (?s2 - ry2
f2 )

or 850 = 0.71 (?s - 1034.6) + tOr 142 - (1034.6)21


or ut _Tt 2060
,r= rr=El =z'sl or 850 = 6.71 \-734.6 + 1.1a Tsz - 10?.04
or ue= 2'37 u' or 1}-{ Tsz + 0.71 ?, - 1691.64 = 0
' Combustion occupies - 2'37w-uz l'3?,,
us =- l4., = u'u978
n n, or g'78%o Stroke. (Ans.) OI ?32 + 7L00 ?s * 1691.64 x 10a = 0

f' n =* -u"=u1-u2l o Ts= -


?1001J(7100)2 +,4 x 1691.64 x 104
L = L5u2-v"=t4nrl
Exmple 4.8, In an.oil eng^?-1--*hi:q^y
pressure at the beginning ofcornpression dual combustion cycle the temperature and. - ?100 t 10866.3
are g7"c and. 1-bar respectiuery. The com)ressnn ratio = 1883 K
,:,,,: supptied per is.of air ii n:oo ll "titf
:^!;.T!"
and na,l !"".,
at constanl pressure. Calculate "f
which s supptied, at constant uotume pzTs _40.23x1883
(i) The maximum pressure in
the cycle.
Further, +=+
Tz
or pr=Pi!!=
-
Tg T2 1034.6
= 73.22 bar. (Ans.)

lii) The percentage of strohe at which cut-off occurs. (ll) Percentage of stroke at which cut-off occurs :
Tahe: yfor compression 1.4; R 0.2g7 cp= cu +t = 0'71 + 20 x 10-s T + o.287
= = hJlh8 K and,
c, for products of combustion
= 0.71 + 20 x l0'l T. = 0.997 + 20 x !O-5 T
I

*tf-

194
INTERNAL COMBUSTION ENGINES FUEL.AIR AND ACTUAL CYCLES

Consider constant pressure process J-4 :


The actual mixture strength (tr'/A ratio) ex-
pressed in terms ofthe chemically correct value is
Qr-=
^ f' ",
ar = t l'r,sst r zo x to-5 T) d,T
14?.'l
ffi " r00=111.496
aso.=
ifo.soz
It
r.,. ro-, {]ln
2 t I,
I
i.e. the mixtue used is 11.47o rich in fuel. Tl.e com-
bustion is, therefore, incomplete and hnce CO will be
= 0.997 (4 - ?r) + 10{ (712 _ Tsz) formed,.
(71- 1.114 C6Ht4 + 9.5O, = aCOz + CO + cHrO
= 0.997 1883) + 7O.l (712 - 18$2)
or _ tB77.g5 + Equating atoms of the same element before and
8S0 = 6.997 Ta t}a To2 _ 554,57
after combustion, we get
or ,o-n Tn, + o.997 ?o_ 8082 0
= 1.114 x 6 - a + b ; 1.114 x L4 = 2s ;
or Taz + 9970 T4 _ gO82 x 1d Fig.4.20
=0 9.5x2=2a+b+c
1._ - 9970 f J(99?0)2 + { x gsg! x tQa or a+ = 6.684 ; c =7.798,2a+ b + c= !9
ll
2 Solving, we have : a = 4.52, b =2.16, c - 7.8
lt+yi=2476K By adding nitrogen on both sides we get the actual combustion equation as given below :
1.114 C6H14 + 9.5 O, + 9.5 x (79121) Nr 4.52 QO" + 2.L6 CO + Z.g HrO + 9.5 x (29121) Nz
iii
f:

= 1i

Further, Ul Ut T4 2476 Moles before combustion = 1.114 + 9.5 + 3b.?4 46.854 say 46.85 lI

,t=i or u4='r"='r* 1.315 u,


=
1883 = Moles after combustion = 4.52 + 2.L6 + 7.8 + 85j4 ii
= 6O,22 it

Cut-off= - - 1'315 us - u _ 0.315


uus 50'22 - .46'95
ut - us l4us _ u, l'.' MoI""rIo, "rpansian = 46.35 -'---- or
= 0.0835 -'-'""'"
S.Bsqol t
1g L I
Fmm compression process 7-2, we have :
=o.0242 or (Ans.) 1.. a=Sr=rl
2.427oofsrroke. 'i.
nmple
\uzu3) 4=fsrl'-' h
I
4'g' An engine wolkr'\tg on-otto qrcle has
a compressinn ratio g, It uses herane Tr \l,, )
"",:1".[:,^ :!:! tu y "ato"iic ;! y" ;' Mr t W. ln _i' i"i x', n" mist ure s
,,""#').,
T2
",
3rt =
(8r.36-r - 2.0? :. T2 = 7to t< i#
.r', Lenperature and pressure hJlkg K r\achad in Q2-s= cu(Ts- Tr) for 1 kg of mixture ifl
the cycle :
(i)Without considering the nolecular ' 1r
con\acrioz ; 44x103 I it.
(ii) Considering molecular contraction.
t4S = 0.716 (T3 - 710) or Tz= 48G2K i

Ignoring molecular expansion, I

3r!1= A%
Tr Ts
wT" 4s?
The stqlchiometric equation can be written
Ps=Prx*i=t"u' = 119.4 bar. (Ans.)
as,
(i) Considering molecular contraclon :
CoHrn + x0r= yCO"+ zflre
Since mass of the reactants and products is same and specific heats are assumed same, the
Equating atoms of the same element befoe and temperature of the prod'ucts it molecular expansion will remain sarne as without molecular
after combustion, we get
!=6,2=Zand=9.5 expansion ; only the pressure toill change
.'. CuHr, + 9.5O, = 6COz + THzO pu = nRT, where n is the nrmber of rnoles
P*n
Gravernetric air-fuel (/F) r"tio = 9{I!2(19942 = 15.37 50.22
72x6 + Ix 14 Pressure with molecula expansion 122,87 bar. (Ans.)
=
46.35
INTERNAL COMBUSTION ENGTNES FUEL-AIR AND ACTUAL CYCLES 19"t
f,nple 4'ro' An i*:l g," eycre oyerathg gn air
a cornpresson rqtio 7. The tem-
has
peratures at the end. of isentr-opic compression * {<z"c
Using gas tables determine th cycle irrn i"'iil,r"t."" "ln"-i;;;;;;;i"" is 1347"c.
Solution. Giuen : r = 7 ; Tz = 442 + 27J= ?1S K i ?r = t. fhe specific heat varies largely with tmpsrahs but not very signicantly with preesure except at very
1B3Z + 2?g = 1610 K 'highpressure.
Irom
.ing to T, gas tables (Refer appendix), correspond-
= 715 Iq we haye
z lhe specic heats of gmes incere with rie i tempertnrc since the vibrtional mergy of molecule
increases with tmperatu.
_. + 64.58
62.02 & C}ratge ofinternal ezary (per kg) during
""=__-1.-=*'. u2-u1=c(T2-Tr)
a process with variable specific heats,

u,
520.23lj44]! Changeofenthalpy, \"-hr=cr (T2-T)1
= = ,zL.z kilkc, 4 Heattrmsferduringaproeswithvuiable
I
. From gas tables, corresponding to
we have 4 = 1610 K
li
,,, = wf& =s.es
p(uYtb.Ft=consteal
,. _ 1298.30+1316.96
6,
"n---_-_l-- tsntropy change' during a process with variable specifc heats,

= rs07.6g kJ&c ,-,,' =" be'\Iil fe'l ft(r,-r,).


l+l -G -)b&-'(Pl./
lblume compressionratio lL=5 z 7. PercentagevariatiouinairstandardefFciencyonaccountofperentagevariationinc,,incaseof:
= u2
ur, =
:' (i) otto cycle : 4 J:-lq - Dlog" rx 4c,..
ut
=7 x un = ? x 65.8= 460.6 -nI'cu = -
From gas tables, corresponding to un 460.6,
= we have
(ii) Dieset cycte r dr I - I (, pt log" p gs..
-- - pl_r .' llI]"o
[,*-, .

By interpolation rt= - rl = - - u
l-
:

#H#
340 x (454'1 - 4oo.G) - Bss K
& Dissxiation refers to dishtegratioD of bunt gases at high temperature. It is revereible process and
increases with temperatue.
ul = 235'61 .
ffi6el x (338 - 330) = 241.3s .7, 9.
lo.
Mujmum effrciency is obtained with linear mixture.
The exhaust gas tenrerature is nmimm at the chemically conect mixture.
Also, we have Yl-=L =t ll. If losses due to variable specific hats md disseiation are subtmc-ted fon the "ai standard cycle", we ger
% ur" "fuel'air cycle malysie". If losses are furthe subhacted fron'fuel-air cycle analysis" we cm very closely
approximate the "actual cycle".
+,=f=f=o.srs
From gas tables, corresponding to u^
oBJECTM TYPE QUESTTONS
= O.g1B, we have
Fill in the Blanks or Say (Yes'or fio":
?, : 2800 K and u, = 2462.5kJA<g l. The value ofc, ofa real gas ...... with ioease in temperatue.
T]:e work done
= Heat added - Heat rejected Z The value ofc, of a real gas ...... with increase i, temperatue.
= (ur- u) - (uo - ur) & The value ofc, ...... with inoease i moisture content.
4 The value oflfo air ...... with inmeme in moisture coutent in air.
= (2462.5 - 524.D - _ 241.88)
OBO7.68 = 872.05 kJ/kg. (Ans.) 5. The vaiue of 1 for a real gro ...... with increase in tmperature.
Thermal effrciency, -'!,r = Work done g72.05
6 Thechangeofintemalenergyduringaprocesswithvariablespecicheatsisequaltoc"^(Tr-Tr).
E"a-iiIEI'= e4625-sr{D ?. Thechmgeofenthalpyduringaprocesswithvariablespecifrcheatsisequaltoc"_(?r-?r).
- 0.45 or 462o, (Ans.) & The theoretical cycle based on the actual properties ofthe cylinder gases is cslled fuel-air cycle approxima-
tion.
198
NTERNAL coMBusrroN ENcrNEs FUEL.AIR AND ACTUAL CYCLES 199
9' the " " ' of my substmce is the
lltio of the heat required to rais the tempatw of a
substmce through one degree centigrade. mit nms of 13. Explain clearly the efrect of compression atio md mixture otrength on thermal efficieny.
10. ...... refers to disintegration of bunt gases
athightemperature. 14 What is the effect of mixtue strength on thermal eciency at a given compression ratio.
11. Dissociation is ...... process and increases witl tempanan. l What is the efrect of mixture stength on cyde power ?
l2 the effect of dissociation is much ,..... than that of-ange lG Stte the effect ofF" on naximum cycle tcmpentue md pressure at different compression ratios.
of specitic heat.
l& rhe dissociation of cQ commences at about l(x)o'c l7. State ths characteristiG ofconstant volume fuel-air cycle.
d t 1500.c it amouts to 5 percent.
14 Dissociation is ...... severe in the chemicaly conettrirhrre. l& Dismbriefly"combwtionc,hats".
15. lhe dissociation has a more ponouned efiect in....... 19. What $e combution drats ? Where these are used and why ?
eogines.
16' The fuel-air cycle efficiency increasee with cmpnesio-ratio 20. Writeashortnote ongastables
in t}re same mamer aa re air standartl
efiiciency. 21. Disffi the effect ofthe following variables on prcu6 md temperature at salient points of Otto cycle on
17. Muimum efficiency is obtained wi...... mirhes. the basis offuel-air cycle.
l& The exhaust gas temperature is maximum wit e (i) Compression ratio (iD F\el-airratio.
emically correct mixture.
19. Tle mean effective pressure increases with cmpresim 22. What i the difference between air cycle md fuel-air c1rcle ? What are the assumptios in fuel-air cycle ?
20. In S.I. engines the combustion is ....... .
rao.
n What is e use of fuel.air cycle ?
21. In C.I. engines the combustion is heterogeneous. What is the difreence betwen i studard cyde md fuel-aircycles.
22" T}.e """ tine loss is defined as loss of plwer due
to tine required for mixing the fuel with
2t Make a mmparave statement of oreratioro md mrking media for air syde, fuel-air ryde md actual rycle
complete combustion. air and for ofS.I. engines.
26. E:glain why e S.I. engine fails to operat if the air-fuel atio i more tlan 20 : I while a C.I. engine can
operato on an air-fuel ratio ofeven 50 : l.
27, Explain how (i) time losses and (ii) inomplet combution losses are accounted for in the real-cycle
t. m&eases Z inoees & ieaes anal'sis.
4 decreaes 6. decreases
6. No 7. Yes I Yeg I epecificheat a "Air-fuel ratio in a S.I. enginevarie from 8to 16 approximatelywhile suchvariationin a C.I. eugine is from
lt. reversible 12. smaller 13, No l{. more
10. Dissociation
100 atno-loadto 20 at,rll load.'. Explain.
16. s.L
t6. Yes 17, leaner l& Yea l9 Yes 2fL homogeneous
2t. Yes 22. burning.
I,JNSOLVED ETAMPLES

THEORETICAL QUESIToNS l. FindthechangeinefficiencyofaaOttocycleforacornpressionratioofT,ifthespeificheatatconstmt


volume increues by 1nrcent. [Ars. - 0.6637"1 I

l. 2. 'Ihe following data relate to a petrol mgine : il


I

z Enlist the assumptions which are made for rel-air


cJ,cle aaalysis.
Compressionratio = 6
& State the importance of fuel-air cycle. Caloric value of fuel used = 44fl)0 k I/kg 1il,
4 What are molar specificheats Tle air-fuel ratio = 15 : I
? 1t

a The temperature md pressue of the drarge at the end of the stroke = 60'C, I bar ,1

Indc ofcompression = 1.32


lhe speifrc heat at orotantvolme,co = 0.?1 + 20 x 10{ ?k I&g Kwhere ?is inK.
Detemine the maximum pressure in the cylinder. Compare this value with that of constant specific heat
c, = 0.71 kJ/kg IL [Ana 56.6 bar ; 80.6 bu]
S, The combugtioD in a aliesel engine is assumed to begin at iler dead centr atrd to be at constat pressrre.
lhe air-fuel ratio is 28: l, the caloric value ofthetuel is 42 trrll/hg, and the specific heat ofthe products of
For Otto qcle : n I l-n, 1aa combwtion is given by:
c,= 0.71+ 20 x 1016 ?;R for the products = 0.287kJkCK
(
T=L-;"tt-Dxloc. 1j=r- Ifthe mmpression ratio is 14 : 1, and the temperature at the end qfcomprersion b 80O K find at what
dn r-t(r_ ptloe. p percentage ofthe stoke combustion is completed. lAna.10.96r'oshokel
For Diesel cycle
"L'-""'--pt-l *lld.,
tl =_ rl " uf,on-._ -tl; 4. In an oil engine working on dual combution cycle the temperature and pressure at the beginning of
cornpession ae 90"C and I br rspectively. The ompression ratio is 13 : 1. The heat supplied per kg ofair
9. What is dissociation ? How does it affect power
developed by the engine ? i 1675 J, half of which is supplied at constnt volume and half at constat pressm, Calolate : il
10. Describe briefly the effect ofdissociation on temperatrire (i) Tlre maximu pressu in the cyde i
at-difierent mtur strena+.
11. Explain the phenomenon of dissociation. (ii) The percentage of stroke at whidr rutoffocrus.
t2. Are dissociation effects equallypronouneil in
S.I. Take : yfor compression = 1.4 ;,B = 0.A7 kJlkgKand,c, for products of combustion = 0.71 + 20 x lf ?.
and C-L engines ? Explain.
lAns,66.2 ba I 2.64% ofstrokel

rlr
t_

200
INTERNAL COMBUSTION ENGINES
q An engine working on the OttoJyclg having compression
ratio Z, uses hexane (C.H,.) ro the fuel. Ihe
calorificvalue of rhe tuel is aa M.Ifu. rt".i"-r"rJioi-tir-"'iiureis
temperature md pressure reached in te cycle : t3.6z: 1. Deriiineemuimurn
(i) without consideriagthe molecularcotactio
;
(ii) considering nolecular contrastioD-
p
Assume o = 0.718 kJ/kg I( compression follow's-e
mture at the beginning of compression are
lawpul.a = C. The temperatue and pressure of
SZ.i;ilfu"
"*p"ci""fy.
the
o
fo-raiiaearQtt3ens:iaeoperatia'.o,*'-ffi"3;1ffi::,f"::::,ii1filll1ffitrl;lTr'#i
and at the end of expansion is 134?;c. rhe cornpna&
efficienry. rau" ii. u.i"gg*ffiffiil;;il'Jorr an Combustion tn s.r. ingines
tAns979.4kJtkg;a7.6%l
5.1. Introduction-Definition of combustion-Ignition limits. 5.2. Conbustion pheomenon-
Normal combution-Algornalunbuqlion. 5.3. Effect of engine variables on ignition lag.
5.4. Spark advancg.alif factors atrecingtqtion timing. 5.5. Pre-ignition. 5.6. Detonation-
Introduction-Pfocess of detoation or k\cking-Theories of deonatio-Effects of
detonation-Factors afnectiag detotration/knock)5.7. Performanc number (PN)..6.8. Higheet
useful compression ratio (HUCR). 5.9. Combustion chamber design--S.I. engines-Induction
swirl-Squish and tumbl*Quench area-Tubulence-Flame propagation--Swirl ratia-
Surface-to-volume ratio--Stroke-to-bore ratieOompression ratio (C.R.). 5.10. Some types of
combustion chambers-Divided combustion chmbers-TVorked Exanples-Highlights-
Objective Tle Quesione-Theoretical Questions.

5.1. INIRODUCTION
5,1.1. Denition of Combustion
Combustion may be defined. as a relatiuely rap.d. chemical combination of hydrogen and
carbon n the fuel with thc orTgen in the ar, resulting in liberation of energy n the fonn of heat.
Following conditions are necessal for combustion to take place :
1. A combustible mixture.
2. Some means to initiate combustion.
3. Stabilization and propagation offlame in the combustion chamber.
In spark igrrition (S.I.) engines, a carburettor generally supplies a combustible mixture and
the electric spark from a spark plug initiates the combustion.
5.1.2. Ignition Limits
It has been observed through experiments that igaition ofcharge is only possible within
certain limits of fuel-air atio.
The 'ignition limits'correspond approximately to those mixture ratios, at lean and rich
ends of the scale, where the heat released by the spark is no longer suffrcien to initiate
combustion in the neighbouring unburnt mixture.
Fig. 5.1 shows the ignition limits for hydrocarbons.
JI Practical limitfor L
carburetled enoine I

-t .Too rich k- lgnton limits lor hydrocarbons ---t Too lean k-

9 14.5 21
Air-fuel ratio
Fig. 5.1. Igaition limits for hyocarbons.

201
-

INTERNAL COMBUSTION ENCNES


COMBUSTION IN S.I. ENGINES ll
The igrrition limits are wider at increased temperatures :i
- reaction and' higher thermr fusivty cefficieits o heher rotes of
because of '-'
.LiYQM assurnes compresson curue hav'rng no ignition.
tne iitiri.-- ll
- The lower and upper limits ofignition of t}e mixture
depend upon thetemperature and o First stage ofcombustion, the ignition lcg, starts from this point and no pressure rise
mitture ratio. is noticeable,
l

In case ofhydrocarbon fuel the stoichiometric fuel-air ratio is o Q is the point where the pressure rrc e can be d.etected. From this point it deviates frorn
- air ratio lies between about 1 : B0 and 1 : ?.
about 1 : 15 and the fuel-
the simple compression (motoring) curve.
5.2. COMBUSTION PHENOMENON
The time lag between fust igniting of fuel and the commencement of the main phase of
combustion is called tbe perid of incubation or is also known as ignition leg. The
time is normally about 0.0015 seconds. The maximum pressure is reached at about 12'
5.2.1. Normal Conbustion
after top dead centre point. Although the point of maximum pressure marks the
In a S'I' engine a single-iutensely high temperature spark passes
across the electrodes, compietion of flame travel, it does not mean that at this point the whole of the heat of
leaving behind a th ttrea i nr.". r-Trir
irritl"trn""a, combustion spreads to the envelope fuel has been liberated, for eveu after the passage of the flame, some further cbemical
of mixture immediatelv surrounding it
ends primarily upon the temperature of adjustments due to reassociation, etc,, will continue to a greater or less degree tbroughout
";;;;i"l;p
the flame front itself end. to a, seconh,ary"tdqr*,
the temperature ond, the densitr of the the expansion stroke. This is known as after bvrninp.
surroun'ding enuelape. In the actual e,igi" "N;;oth ihe rnixture-is not at est but is in highly
turburent condition. The turburence ;,";h"1i; "yino, Effect of englne veriebles on flame propagation !
preselttiy9 a far greater area of surface frt4;;;;7-;-fr;;;;i" lrie"" front, thts
l, Fuel.air raio. When the mirture is mad.e leaner or is enriched, and silJ more, the
speeded up enormously.
fum whih lreat * t"iig rad.iated ; n"ilZ-it, a.d,uance is
uelocity of flame d.imnishes.
. A":ordittg 2. Compreesion rato. llte speed of cqmbustion increases with increase of compression
l
to Ricardo, the combustion process can be imagined as if developing
,--,
tng two stages
in the follow-
: ratio. The increase in compression ratio results in increase in temperature which increases the l
(i) The growth and development of a serf-pro-pagating tendency of the engine to detonate.
nucreus of flarne (ignitionlog). This I
is a chemical procss and depends opon the?oiloing 3. Intake temperature and preerane, Increase in intake temperature and pressure in-
:
I
The nature offuel ; creases the flame speed.
The temperature and pressure ; 4. Engine lod As the load on thc engne increases, the cycle pressures increase and I
I
The proportion of the exhaust gas hence the flame speed increases.
;
The temperature co-efficient.of the fuel !.e., the relationship 5. Turbulenca The flame speed is very low in non-turbulent mixture. A turbulen motion
beween temperature
and rate ofacceleration ofoxidation or burning. of the mixture intensifies the processes of heat transfer and mixing of the bumed and unburned
t'
(ii) The spread of the flame throughout portions in tle, flame front. These two factors cause the velocity of turbulent flame to ncrease t,;
the corbustion chamber.
Fig. 5.2 showe the p-0 diagran of a petrol engine . practically ii proporton to the tubalent velocity. rf; i
6. Engine epeed lllle fldme speed, itreases ahrast lineor$r with engne speed..l\e cranh rft{
p (bar) a.ngle requlred for flame propagatiott" whh is thc main phnse of combustion, will remain alm.ost
constant ot all speeds. rfifi

For besl performance 7. Engine aize. The number ofcrank degrees required for flame travel will be about the
at 10'to 12" same irrespective6f engine size, provided the engines are similar. iri
r
Max. pr.
5.2.1.1. Factors affecting nomal combugions in S.I. engines. ll
40
The factors which affect normal combustion in S.I. engines are briefly discussed below : j[[
!. Induction pnessure. As the pressure falls delay period increases and the ignition
Exparsion must be edrlier at low pressures. A uaauun control may be incorporated.
2. Engine cpeed- As speed increases the constant time delay period needs moe crank lfil
angle and ignition must be earlier. A centrfugal control may be employeil'
3. Ignition timing. If ignition is too early the peak pressure will occur too early and il
work transfer flls. If ignition is oo late the peak pressure will be low and work trans-
fer falls. Combustion may not be complete by the time the exhaust valve opens and the ti
S valve may burn. tN

4. Mixture strength Although the stoichiometric ratio should give the best results, the
BDc ts0"12o.so. 60.ls0. TDC oo.foo" soj l2o.rso"eoC
effect of dissociation shown in Fig. 5.3 is to make a slighly rich mixture necessary for tr
maximum work transfer, f:
5. Cornpreesion rotio, An increase in compression ratio,ipcreases the maximum pres- I lir
Fig. 5.2. Pressure-crmk angle diagrm ofa petrol engine sure and the work transfer. f,il

ffi
,I
INTERNAL COMBT',T!O} ENGINES coMBusrroN rN s.r. ENcrNEs 207

between the hot-spot and spark plug is their


respective nst.,nt of ignit'-. Thus, the o The early ignition created by pre-igrrition extends the total time and the burnt gases
sparking pl'gs provides a timed and-cont"diJ remain in the cylinder and therefore ncreases the heat transfer on the chanber walls,
surface forming the hot-spql builds upto th"
;;;;of ignition whcceas the heated
;;;;;;"" ternperature during each as a result, the self-ignition temperature will occur earlier and. earlier on each successive
compression stroke and therefoe the aciual
iitr"ii- ti e"irian s utrprdiable. compresson sroe. Consequently, the peak cylinder pressure (which normally occurs
at its optimum position of 10"-15' after T.D.C.) will progressively aduance its position
spar* oaarrs spak occqs to T.D.C. where the cylinder pressure and temperature will be maximised.
lgnlted by bot dryifl
The accumulated effects of an extended combustion time and rising peak cylinder
pressure and temperature cause the self-ignition tempeature to creep further and
further ahead of T.D.C., and witb it, peak cylinde pressure, which will now take
place before T.D.C. so that negative work will be done in compressing the com-
bustion products (Fig. 5.5).

..Combustion befirs ....


Rgular Jftqt Drt_
o
o
;40
q
o
a
b30
-=
o20
..Con$nues.... lgnltes reo|drlrEld-

trSll 60 40
B.T.D.C.
20 o204060
T.D.C. A.T.D.C.

ilffitt B.T.D.C. = Belore top dead centre;


A.T.D.C. = After top dead centre.

ffi
....Detomon...,. ..Fame frsb co&- Fig. 5.5. Cylinder pressure variatioD when pre-ignition occurs.

Effects of pre-lgnition :
1. It increases the in the engines.
tendency ofdetonation
2. Pre-ignition is a serious type ofabnormal combustion. It increases the heat transfer to
the cylinder walls because high temperature gases remain in contact with the cyJinder for a longer
period. The load on the crankshaft during compressibn is abnormally high. This may cause cranh

A
illffitf B
failure.
3. Pre-ignition in a single-cylinder engine will result in a steady red.uci,tion in speed and
power output.
4. The real undesrable effects-of pre-ignition are when t occurs only in one or more
(A) Normal corirs!on, (B) Detonaton. cylinders n a multi-cylinder engine. Under these conditions, when the engine is driven hard, the
unaffected cylinders will continue to develop their full power and speed, and so will drag the other
Fig.6.4 piston or pistons, which are experiencing pre-ignition and are producing negative work, to and fro
until euentually the increosed. heat generated. makes the pre-igniting cylinders' pistons and, rings
sieze.

C
INTERNAL COMBUSTION ENGINES

I combuslion
E
sion ratio).
c
o

tr Corbustion with
P dissoeiation 5.4. SPARK ADVANCE FACTORS AFT'ECTING IGNTTION TIMING

ture must deliver its maximurn pressure at a time when the piston is about to commence its
Stoichomstric outward stroke and is nearst to T.D.C. Since there is a time lag beween the occurrence of spark
Air_tuel ratio ___+ and the burning of the mixture, the spark must take place before the piston reaches T.D.C. on its
compresson stroke, e., re sparh timing is a.dvanced. Usually the spark should occu at about
15" before T.D.C.
G. Combusio"
The correct instant for the introduction of spark is mainly d.etermined by the "gnition
uuoid n"nnP-"
flame path to ;1:.TT_".. "r*rlil:chamber shoutd be designed to siue a short
Z. Fuel choice. ""i'i-lio p*^ot" optimurn turbul.ence. Iag". the factors affecting the ignition timngs ae discussed below :
.1. Engtne epeed. Suppose an engine has an ignition advance of 0 deglees and operating
. The induction period of the fuel
will affect the delay period. 0
. value and rhe enthalpy orvapo.irutiorr.JU speed in z r.p.s. Then tiue available for initiation of c:ombustion ir S6il' seconds. Now if the
H:#i::tc "rf"cr
the temperatures engine speed is increased to 2 r.p.s. then in order to have the same time available for combustion,
5.2.2, Abnormsl Combusion an ignition advance for 20 degrees is required. Thus as the engine speed is increased, it will be
necessary to advance the ignition progressively.
weak mixrures combstion mav
n"**"?lil""iliessiverv be slow or mav be mis-timed.
lhese are 2. Mixturc strength. In general rich ms.tures burn faster. Hence, if the engine is operat-
There ae two combustion abarmalities, ing with rich mixtures the optimum spark timings must be retarded, ie., the number of crank
which are less obvious : angle before T.D.C. at the time of ignition is deereased. and the spark occurs closer toT.D.C.
o The rs of the
partictesi"th"":h;:,:::fi ,J*,iifl 3. Par-load operstion. Partload operation of a spark-iglition engine is affected by throt-
o The second abnormality is. generalry
:rill:#:".H:il:#,#";::;;:ffi::*" tling the incoming charge, Due to throttling a small amount ofcharge enters the cylinder, and the
knowo-as.o" and is a comprex dilution due to residual gases is also gteater. In order to ovecome the problem of exhaust gas
manv facets A simpre condition with
"'ptati* ir,o*-. iiat kocr ";;;-";;;;Th.inburnt porrion
of the gas in the coribust"
dilution and the low charge density, at partJoad operation lhe sparh advaue must be increased.
combustio' 4. fyp of fuel. Igrition delay will depend upon the type of fuel used in the engine. For
ten'perature becones greoter "h;;;;;;";d bv uod so that irs
_than tnr'tir 4i1i,r7 ,"^iir"r"il"u'"ii^"
combustion is not coipreted, b"f;;r;'h, ".dutiooprogressiue rnaximum power and econonly a slow burning fuel needs a higher spark ad,vance than a fast
explosion of the unburnt.g.u,. rtirr ,n" ruction periodthen a simurtaneous burning fuel.
(pressure) *uu" ;;";":"rrh1f exlosion is accompanied by
;
:T:h *r.r,*ii
".."p""iir rii""" from rhe .vui"r a detonation
iii, 5.5. PRn-IGNTIION : .-',
high frequency resoaance serting up a t
siu"" ;;rudible noise. Th debnatiorr wave causes Refer Fig. 5.4. '
ff::il:"'.ff:1,il"0. "*" destrov; tt'" tr'"'- bo,,,,;,r-i;;"i',i'""ri.,a", -.I" Pre.igniion is the igntion of the homogeneus mixture in lhe cylindzr, before the
[Not. Refer articles 5.5 and 5.6 fo deails timed. gnition sparh occurs, caused. by the local ouerheating of the combustible mixture.
ofpre_ignltion ad detonation respectively.]
For premature ignition ofany local hot-spot to occur in advance ofthe timed spark on
5.3. EFFECT OF ENGINE the combustion stroke it must attain a minimum temperatr.rre of something like 700-
VARIABLES ON IGNNIONI,AG
Ignition lag (the time lag between.first 800'c.
-
phase of combustion) is igniting of fuer and the commencement Pre-ignition is initiated, by some overhea.ted. projecting part such as the sparhing plug
not a period of inactirjl"-llr", j" of the main
terms of crank angres is ro" to a^ch-emical process. The ignition lag electrodes, ethaust ualue head, netal corners in the combuston chamber, carbon
zo' an
The duation ofignition lag depends
;. ;flill, o.ooru second or so.
in
dtposits or protrud.ing cylind.er head, gasket rim etc.
l. FueI. Ignition lag depends on
*
tfr" foii"rvirrg factors : However, pre-ignton is also caused, by persistent detonating pressure shockwa,ues
nat, The higher the self gnition scoring away the stagnant gases which normally protect the combustion chamber
tem.perature <s.t.i. cf "hemical
fu, lonser the ,u;ir:;"r:;'"el' walls. The resulting increased heat flow through the walls, raises the surface
2' Mixture ratio. Ignition_lag is-the temperature of any protr'uding poorly cooled part of the chamber, and this there-
;;;;;;;, the mixture ratio which giues the
tenperature. rr" -i"tr."'"iii" isfor
somewhat richer than stoichiometric fore provides a focal point for pre-ignition
:#::'^ o The initiation ofignition and the propagation ofthe flame front from the heated hot-
spot is similar to that produced by the spark-plug when it fires, the only differencc
208
INTERNAL COMBUSTION ENGTNES
Ihus, the danger of the majority COMBUSTION IN S.r. ENGINES ,fK)
of operating efcienily
more cvrinders-are subjectedt eice-ssjve
,y_lr:d"r., rbile one or
pre_-igni;b"
aware of a loss in speed and powe",n-tte"eol il;";tu"" rilr only be
to"compensate for tni., ru"[ _;i bwil;Le "Ti.e bader
oJv ,lninsficc the piaiua, iiiro^ utttrsiaure
the following poitg ae unrthnotitrg
:

o*, o1"lf-iTi:'j:#"-:'-*""sibte-ror;norn*L^Tr_lg*o:rpoe""u"e,but tere ca be a


creeping r""*""d;;; f.B18."ffi,f"f:tJTffffi ilInf#';;l il o*o p,"*,,"1
2' rf pre-ignition
g: same time * *:.,3:o sparkiag plug res,
***:tirignu"n
appear a8 nonnal. Therefore.
is Jtci;-#, q".". combustion will
*o,i
,ffi:iffi::ill'"X"" "ol"
ti-".,J,p*p.":,i'ff L
""iti",iJo
operate at the
m:tn:'*i"i"1"'it*r ser-ienitio;
B'
ii
which ure ttv main causes of pn.ignition
1r "u,,rot;"o!;1,f"X,'ff**T*"nr*;;uob", Fig. 6.6. Combwtion with lomking.
ll T.eets for pr.e-igultion combustion. During the preflame reaction period the flame front
could move ftom BB.
u
. for.pr:lsnion is to thut off th.e isnition.
tn cc', and the knock occurs due to auto-igrrition of the charge ahead
of cc,. Here we
il mrtlmrf# If ttu ensi,t iil fires, it
have combustion unaccompanied by flame, producing a very
high rate ofpressure rise.
''il showsthair;'"!fi,*?i'ji,ffi ##"r#;;.:::y,:iiX',^".";;;
-
. P",p"g:r:re-time diagram of detonating combustion in S.I. engines is drawn and
ll
o
of mechanical ntuitonns
Tlte proofo,
best
; r*; i,i; itri!,jj':!-3y,h euidence no labelled below:
)i

"p""aioi"r.i,lrTf **i,r_",*,#t#!fj;:f#:nwhahish Extremely rapid pressure rise


(Less than 50 micro seconds)
'I 6.8. DETONATION

6.6.1. Introduclo
At present the amouat ofower that Rapid Relative Slow
ca ue aevelSl{ in the cylinder compresston
K*{:"1:!!lK;* dLi;;:;;frt["J *1.
of a petr,ol engine is
name has comprered iL ol vapour-air
constant
pressure
rising
pressure
rernaining uburnt charge mxture
without external firea throughout ".o,""n.oosty
it" or"J"pont "oo""
"rri"t"oil
The result is a tremendouf Gas vibration
ad local ,oT::r" in-pressure
waves that hir the cvnderra'8 wih_8;;h;;"ii'rii,
-ranid which sets up pessure
the ping that manifeets detonatio.n. the walrs emir a sound iil
rn* a .pr.ns,,. It is
companied by metallic hamwr ,i"".o pre"sure d.uring ambustian
"'iizffen
*
Une sou call?",oo"tioo. a_
region in whih debtntbn occurs
_The
named the'ietonation '*'on is f"ii^,thc sparhing ptug, and' is
;;-;;;;ili-*y*!
rwtion this zone is turely iwre ltm
quafter the clearance "r""
;;^", than one
5.6P. procese ofDetontion
or f,nocking
' ft-il:"i,'liTr-n:'*:ti:""T::::r*,f*Su mav be exprained bv referrine --1 I o.oo,
sec.
I Time ----+
to
advanng f.;;" to3bustion chamblr with flame
spark plug location o. *o^t^tl-l Fig.5.7
end charge BBt rarth:si r""li
ii",oi*;l",;, fiil'i:,:,lT:,ttHHltr":";fj o The'intensity of detonaton'will depend mainry upon the amount of
energy contained
n * ,'*"i"im. ,"'n=* advancing
;"#i$::H:lH",""ltJl;i"*:";ffi:tffj *ay'"take in the 'end'mixture' and the rdte of chemicol i"o"iioo which rereasis * i-tn"
place in end charge leading heat and a high intensity pressure-waue. Thus, the earlier in tt u form of
further increasJln its ternperature. to process
Ifthe end charge BB'D- reaches its the detonation commences, the more unburnt end-mixture will be "o-rrrtioo
available to intensify
auto-igaition temperature and the detonation' As little as b per cent of the total mixture charge
time to complete the preflame.""Ji;;;;".'h;ge remains for some when spontaneously
will autoigni teleadtngtn hnockng ignited will be sulfcient to produce a very violent knock.
-.r.
COMBUSTON TN S.I. ENGINES
2to and' prornotes
Aduanced ignition tming incteases peak pressures
ENGINES
4. Ignition timing.
5.6.3. Theories of Iletontion : knock.
There are two gneral theories ofknocking/detonation : 5. Mixture strength. Optimum mixture strength gives high pressures and protnotes
(i) The auto-igntion thcory knock.
(ii) The detonation theory. 6.Compressionratio.Highcompressionratiosincreasethecylinderpr,essuresandpro.
(j) Auto-igrrition heory. Auto-ignition refers to initiation of combustion without the ne- rnofes knock. I

cessity of a flame. The auto-ignition theory of knock assumes that the flame velocitr is twrmal T.Combustionchambrdesigrr.Poordesigngiveslongflarnepaths,poorturbulence
before the onset-of auto-igrition ad tha gas vibrations ae created by a number of end-gas ele-
ments auto-igniting almost simultaneously. and insufrcieut cooling all of which promote knock'
(ii) Detonaion theory. Ia the auto-igoition theory, it is assumed that the flame velocity 8.Cyllndercooling.Plrrlrcoolingraisesthemixturetemperatureandprono'sknock. I
ll
is normal before the onset orf auto-ignition whereas in detonation theory a true d.etorntng waie
formed by preflame reactioos has ieen proposed as the mechanism foi explosive auto-ignition. 5.7. PEN'ORMANCE NUIGER (PN) .'

Such a shock wave would travel through the chomber at about twice the sonic velocitr and would Performancenumberisausefulmecil|reofdetonotiontend.enc!.Ithasbeendeveloped
pressure (hlimep)' when inlet pres-
compress the gases to preesures and tenperatures where the reaction should be practically instan-
from the conception ofknock limited indicated mean effective
taneous.
sure is used as the dependent variable'
In fact knocking or detouaion is a corrplex phenomenon and no single explanaion may
be sufficient to explain i firlly. performance number GN = ffiff:#
6.6.4. Effects of Detonaion of cond'itions
1. Noise and roughness Ttre performance number is obtained. on'specified engirw, undzr specifieil set
2. Mechanical damage by uarying the nlet Preisure'
iijj
3. Carbon deposits (HUCR) ,
6,8. HIGHEST USE.T,'L COMPN^ESSION NATIO
1ilr
4. Increase in heat transfer
rijl
5. Decrease in power output and efficiency Ilrehighestusefirlcompressionratioisthehighestompressionratioetnployed.at
specifie set of oPeratng- condtons, at which
6. Pre-igtrition. whch o fuel can be used io o sp""d engine und.er i lili
igniton o,r strength a.djusteil to giue the
dtonation first becomes audiile wth both the ^ittur"
Control of detonation : ,[d]
hghest efficienc'
The deton'ation ca be conholled or eveu stopped by the followin g nethods :
1. Increasing eng:ne r.p.m. ENGINES
6.9. COMBUSTION CHAMBER DESIGN-S'I'
2. Retarding spark.
Enginetorque,poweroutputond'fuelconsumptif,nareprofounillyinfluencedbythefollow-
3. Reducing pressre in the inlet manifold by throttling.
4. Making the ratio too lean or too rich, preferably latter. ing: nil
(i) Engine comPression ratio ;
5. Water iniectio* Water injection increases the delay period as well as reduces the
(i) Combustion chamber and piston crown shape ; .
flame temperature.
6. Use of high octane fuel can eliminate detonation. High octane fuels are obtained by (i) The number and size ofhe inlet and exhaust valvee ;
adding additives known as dopes (such as tetra-ethyl of lead, benzol, :qdene etc.), to (u) The position ofthe sparking plug' .

petrol. design :
The following are the objects of good combustion chamber
Fig. 6.4 shows norm.al combustion, detonation and pre-ignition.
l. To optimze th fitliry and' emptying of th qlindr wth ftesh unbvrnt clarge respec-
tiuely ouer the engnz's operating speed' range ;
5.6.6, Factors Affecting Detondtion/Xnocks : and'
The'Iihetitlood. of httoch is hrcreased' by ony red,uction in the ind.ucton period of combus- 2,TocreatethecotditionintheeIind'erfortheoirandfueltobethoroughlymixedand
,.
tion and. any reduction in the progressiue etplosion flame velocigr. Particular factors ae listed burning cif the chorge wll be con'pleted in
thn excited into a hiNy turbulent stte so that the
below: the shortest Possible tmc'
1. Fuel choice. A low self-ignition temperature promotes knock. as far as possible in order to achieve
3. To prevent the possibility of detonation at all times'
2. lnduction pr8sure. Increase ofpressure decreases the self-ignition temperature and
-
the induction period. Knock will tend to occur at fult thrcttle.
thesefudarrentalrequirenentsitisimperativetobeawareoftt,Lefo,cto':sthatcontributeto.,
ward'sinducingthecftargetoenter'thecylinder,tomi:-intna.tI!,toburnbothrapid'lyand
3. Engine speed. Iow engine speeds will give low turbulence and low flame velocities smoothly ond' to expel the burnt gases'
(combustion period is constant in angle) and knock may occur at low speed.

--'-_=-_ -- ilfl
212
INTERNAL COMBUSTION
ENCINES 213
6.9.1. Induction , COMBUSTION IN S.I. ENGTNES
Swirl
Refer Fig. 5.8.
o 8oi s the rototiorutflow of chorge within
eis.
dbout its the rylindzr
. y:::_r:":y by comtrucins rhe intake
gve a tangential commnent sysrem to
toth"i"td." niw as-:ilo*",
tlre cylinder. Itis is ione Ay
intahe manifold ualue ports "napirg
d;;-*!rl,ri* rn,
t Swirt swtly enharcts r?
on
"rin ,t";:;;";L,
o, oi-nnt'i-ci""
a homogeneous mirtrre n Tb"e the very"fehort time available
for rhis j
modern high speed
mechanism for spreailing or "";;.-i;;;" J.*
combustion process.
tn" rore t""ii""",
The induction ports are clasr ""
sified as follows: ReferFig'
L. 5'9'
Direct straight port.
2. Deflector wail port. (ii) Maskod vatue port Cw) Hefcal port
3. Masked, valoe port. Fig. 5.9. Induction ports.
4' Helbal prt. The intensity of swirl is ifluenced
Fig' 5'8'Iductinswirl.
a of volut|etric efficien4 in the upper
by the Itese ports, bowever, suffer from loss

;:fr:"ff".or "e
porr hetix and the _;;;;;;of the spiral flow path abour rhe sped range of tfu order 5 to 709o.
Helical ports usually.prode
o In chanber wall deflected induction swirl, the downwad and circular movemeat
higher flow discharges for equivaluent l.euels of swirl of the -i*-r dn erpanding, and then e contruting specidl stoirl obout the
compard, with rected po"ts
bi"aui" ril;l;;p"rpffi;ffJ Jl"rrre
cylind,er axis during both the induction and cornpression strohes, respectively'
"nerates
;ffitr'f,i"j',"",:-li':l; lld'."' '*"lit, ;'i;L vorumerric efficieucies can
be Methods of Intensifying the Rate of Burning

ffiir fr t"Tn"j,J=.r"""",::l;:"ff:;,,,",*;;;:::"",^".
liinlv on.the port geonetry above
r"l"e roi
6.0.2. Squirh and Tumble
As the piston approaches T.D.C. at the end of compression
how enters tt" *riifldl
ooer. Generally, the magnitude the not
etroke, the volume around the outlet edges of the
valve ft. of r*i
"", *iii,r."i".*o combustion chamber is suddenly reduced to a very small
value. Maay modern combustion chamber designs have
most of the clearance volume near the centrelioe of the
cylinder, As the piston approaches T.D.C. the gas mixture
occupying the volume ot the outer rad,us of the cylind.er
is forced rad.ially inward as this outer uolume is reduced
to near zero. This radial inward moton of the gas
mixture is called.'sqush'. It adds to other mass motiong
within the cylinder to mix the air and fuel and to quickly
spread the flame front. Fig. 5.10 shows a typical
compression squish.
As the piston nears T.D.C. squish motion generates a
secondary rotational flow colled 'tumble'. ?Iis Fig.5.10, Compression
rotation occurs about a circutnferential aris near the squish.
outer edge of the piston bowL
(l) Direc{ed straight port, 5.9.3. Quench Area
The quench area is defrned by lbe paraltel porton of the piston and cylind.er head
- whch almost touch eoch other as the piston approaches ?.D'C. These opposing flat
INTERNAL COMBUSTTON ENGTNES 215
/ COMBUSTION IN S.I. ENGINES
surface' a.thin_raminilf
chargc between t,,e-_, have a rarge surfoce
relative to"dvihing
tlu sma'aorume trapped between ti"-. con."qo"otry 2. The increase offlame speed due to turbulence reduces the combusion time and hence
amout of heat tranfeq1a rrm this thin there willL a targe minimises the tendency to detonate.
walls. The reult is a rapid- cooling
l"^i".'"rooi."c"]i"""!r ii"l-"trr 3. Turbulence increase the heat flow to the cylinder wall and in the limit excessive turbu-
q;;;;;e"t, ty these parallet surfaces. lence may ertinguish the flame.
aea is defined ""
p"r""nlog"
- ^ of oiporns flat ara relatiue to the piston 4. Excessive turbulence results in the more rapid pressure rise (though rnaximum pres-
ffi;Ht
sure may be lowered) ancl the high pressure rise causes the crankshaft to spring and
5.9.4. Turbulence rest of the engine to vibrate with high periodicity, resulting in rough and noisy running
o "Turbulence'cossfs of randomly d.ispersed.
uortices of different sies which become
of the engine.
superimposed into the air, air an petrol li",uno, stream (Fig.5.11).
-or
vortices' which ae carried arong These 5.9.5. Flane Propagation
with.th flow st"Jri, ."p"""*ot smarl inegular Typical flame propagatioa velocities range from something like 15 to ?0 m/s. This
ways that taLe on a concentric spiral motion breaks-
ffig. JjZl. would relate to the combuston flame velocity increasiag from about 15 m/s at an idle
speed of about 1000 r.p.m. to roughly 70 m/s at a maximum speed of 6000 r.p.m.
When igntion occurs thc nucleus of the flame spread.s with the whirling or rotating
uortices in the form of ragged, burning crust from the initial sparh plug ignition site.
The speed. of the flame propagaton is roughly proportional to the velocity at the
periphery of the uortices.
5.9.6. Swirl Ratio
Induction swirl can be generated by thngentially directing the ai movement into the
cylinder eitherby creating a preswirl in the induction port or by combining he tangential-
directed flows with a preswirl helical port. 'Cylind,er oir swirl" is defined o s the angular
rotational speed. about the cllind,er os.
suirl rotio is defrned as the rotio of air rotational speed to crankshoft rotational
speed,.
Swil hrge Helical ports can achieve swirl ratio of 3 to 5 at T'D.C. with a flat piston crown.
msrrnenl
However, if a bowl in the piston charnber is used, the swirl ratio can be increased
Fig. 6.U, Intate turbulet nixture flow. to about 15 at T.D.C.
Fig. 5.12. Iltustratiorofdulenoe superrinpoeed
on mixhn wiL
6.9.7, Surface-to-Volume Ratio
As the vortices whirl they will contact a-dj acent vortices causing o In order to minimise tiie heat losses and formation of hydrocarbons within the combustion
viscous shear interaction.
This rapidly speeds up the rate of heaitran"r""
r*t -iriog.
The amount ofvorter activity, that is the "rrJ
formation ofnew vortices and the disintegratioir ^I
of others, ',,,t'",ses th tuibutent
fi"; ;;;';;;;;'"rlin" E
"o*^
role in combusrior,-ph".,9ouooo
& 400
}*:'"Xlf:i
engines. The ftame l:1lTp""t"i in s.r. (as we c.r.)
i.-"r-t"riril"iliff"*"1;;;;:;r;;X"";;
sneed is v-err lo,
a

ilj^!!:::::rfr:.t*
unburned portions
,6 u.
proceases of heat transf2/and
tln JLsm
in rrv (diffusion).
mirins of the burned and zoo
fronf (.dtnusion).
flane Jronr
Trese two factors cause the velocity of I
turbulent flame to incease practicalry i" p.;g/.ti;;1o
turbulence u ue urxure
of the nixtue rg.due
ie due to admissior/of
admi""i.r/^ fuel-air
lhe urbulent verocity. The
*,*,._^ jL_^--_L
t, ,.1_.r- mixtue _-_
through compaa_
ro
of the inrake pip", jo tU"
*"]v^13l_":_*j.s srroke. The rurbu_
ren can be increased
bustion chamber which"tinvolves
tne
"natrr" g"orn"td
;;f"J# il;:;;":ff:TL,J""[ltJ:
orii'" ""V/" "i." "o*ioo
oi.yii"i"" rr* and pison crown. ' Surface/volume ratio --+
The degree ofturbulence increases airectlj,
- tf,iththe piston speed. Fig.5.13
Tlte effects of turbulcrce can be summed up
folloro. , chamber, the chamber volune should be maximised relative to its surface area, that is,
1' Turbulence accererates chemical action "" '
by intimate mixing of fuer and oxygen, Thus Lhe chamber's surface area should be as small as possible relaue to the uolutne occupied.
weak mixtues can be burnt. by the combustion chomber (Fig. 5.13). The surface-to-volume ratio is t}l.e rotio of the
combustion surface area to tho.t of its uolume.
The surface-to-volume ratio increases linearly with rising eotnpression ratio.

\
I

INTERNAL COMBUSTION ENGINES COMBUSTION N S.. BNGINES


217
5,9.8. Stroke-to-Bore Ratio
100
o Fo various engines the stroke-to-bore(L: D) ratio can range from 0.6 : 1 to.1.4 : 1.
4 .=10
When Z = D, the L: D ratio is said to be qrnre
;
I
I
I
lno a-
When Z < D, the L : D ralclo is said to fu ouerquare; (! 70 c.R. = 180
When.L > D, the engine is said to be und,ersquare. o !r F70
c '.'ouersquare" engines are E
60 9oo
,tnore sutdble for saloon co,r petrol engnes, whereas 3
q i
"und,ersquare" engines are better utilised, for large d,icset eignes. o $uo
E
5.9.9. Conpression Ratio (C.R)
o
4 \'= 10
:. 30
when compression ratio increases from 5 : 1 to 10 : 1 the cylinder,s compression pressure
30
increases from 8.0 bar to 19.0-ba respectively (Fig. 5.14). orrespo"i"gV,
tfr" = .0 ::::::::
eylinder pressue increasee from 32 bar to g2 bar and b.m.e.p. generated also-Li-"_ Eto --

from 9.4 bar to 11.8 bar over the same compression range respectively,
increases 20 oo
The effect of higher cylinder pressure is to cause a corresponding rise in cylinder
T.D.C. slo.C 60 t0 20 0 20 40 60
temperature from 360"c to 520'c over the sme compression ratiirise. Raising
CYlinder swept volume ---r Crank-angle movement (deg)
tne ---->
cylinder temperature reduces the ignition delay period for one set eogi."
(Fig' 5.15). Thus, for an engine rurylq in its mid-ipeed range, Fig. 5.16. Efrect ofeompression ratio on
"'p""a Fig. 5.l?. Effect of @mprssion ratio on te
the ignion ti;;ng the chracteristic preesuevolume diagram
would be reduced from BT.5' to t2.5" before T.d.c. if it-s cornprLsior, i" aractaistic pressurc.eank :angle diagram for a
increaeed from 6 : 1 to 10 : 1. ""iio for a petrol engine. petrol engine.
o T\e merits and, limitations o_f raising the compreseion ratio with regards to thermal
g' efficiency and mechanical efficiency Le shown in rig. g.rg, *rrer""Jrig. s.rs slows
-.
g/ 22 90
coo50 --- ()
t},e benefits of increased po*". fuel consumption with rising
ol20 80 g- compression ratio. "od ""drrr"d "p."ifrc
ol -3FO

bl18 70 9E os f 90 20
E 16 6 _-- EoA
60 9E 30 5(Xl 80
f
EV
Pg 14 s0
,} o
!i!c
E
I t
E-6 l
I -50
zs
c tol Itu 280

6po. 40t .9

I *l
12 20 I
o
'6 EE 4OO 3
312 260
Eto 30
5 E"tu
E
c
E
o
Ec 20
'EFto o c uoi .s o
o i 300
o b8
3
24O 2.
s678910 5678910 E ool
lt
o
IL
a
Compressionratio --------) Compression rato 84
Fig. 5.14. Effect ofconpression ratio on the b.m.e.p. Fig. 5.15. Efrect ofcompresion
- ratio on the 'oI
compression and maximm cylinder presaues. 20t
air temperatue and ignition lag.
o The effects of compression ratio on the characteristic pressure-volume diagram and
the
0 15 7 I I 10 .t
1

characteristic pressure-crank angle diagram for a petrol engine are sho* in Fig. 5.14 Compression ratio ____-__-_____)
Compression ratio
and 5.15 respectively.
c The main reason for raising the engine compression ratio is due to the increased,
Fig. 5.18. Effect ofcompression ratio oa an --->
Fig.5.19. Effect ofcompression on engine power
engine's themal and mechanical efciency.
density of the air-fuel *\ty at the point f ignition, so that when the energy is md specifrc fuel comumption.
released' it is better utilized.. It therefore, raises bth the engine thermal For S.L engnes' conbustion design practice, summarily, the
foltowing ore required. :
the deueloped power. "ffi""""y "" 1' The smallest ratio of chamber surface-area to
chamber vorume as possible ... to mini-
i '
Out of the major unwanted sid'e effects of raising the compression ratio is that there mise heat losss o the cooling system.
wjll be a corresponding in2.:!t! in cylnd.er pre""ur" .ii"h, in tum, icrei-ilr" 2' The shortest flame-front tavel stance as po"ribl"
,.. to ninimise the combustion period,.
p-iston-ring to cylinder-wall friction and, compression and expansin 3' The prosion for ouenchin_g the mixture farthesr from
heat losses. the .-..-o'ornu"n,
c-o,nsequently, the higher compresiion roto priduces o red,uctiin in the the-end-sas ouerhens. .';*;";;, ;;;;;;;;';."""sive "o^;kl";;l;;
mchaiiial -a as this would prevenr the
efficiencl.-subsequently, increasing the compression ratio produces an increase in end-.s.as3g burnin-g an, therefore,'f ;;;ld-;;;;
rrigh l.rrel of hydrocarbons ro be
thernal efficiency but at the expense of a faillng mechanical efficenc1. expelled to the exhaust)
I
.- /
_ I
INTERNAL COMBUSTION ENGINES
COMBUSTION IN S.I. ENGINES
4. The most central sparking plug position possible ..- to minimise tlw flame sprcad path
(or, alternately, twin plugs can be used to achieve the same objective)'
2. L-head combu;tion chamber. Refer Fig. 5.21.
5. The location ofthe sparking plug should be as ilose as to the erhaust valve as possible ...
to maximise the tenperoture of the m.rture surrouring the sparhng plug electrodes'
6. The incoming mixture must have adequate squirrel (but not too much as this could lead
to excessive heat losses) ... to mk tle air dnd fuel rapid'l1 on'd intimotely '
just
?. The prwieion for squish zones ... to excite the mi.rture into a state of turbulence
before thc combustion occurs.
8. The provision for cooling of the exhaust valve ... to preuent ouerheotng, d.istortirry, and'
burning occurring.
9. The provision for incoming fresh charge to sweep past and cool the sparking plug elec- (b) Ricardob ltbulent hoad
trodes ... to ouoid pre'igniton under wid throttle opening'
Fig. 5.21.Lhead combwtion anber.
10. The utisation of the highest possible compression ratio ... o marmise thz engine's
thermal efftciencl without Promoting detonation. It is a modification of the ?-head type of combustion chamber. It provides the two
11. The inlet and exhaust valve sizes and numbers should be adequate ... to espel the er- on the same sde of the cylindcr, and the valves are operated hrough tappet by
values
haust-gases and to frtt the cylind'er with the ma*imum mass of ftesh charge n the a single cdnshaft.
upper sped mnge. Fig. 5.21 (o) and () shows two types ofthis side-va]ve engine. In theee types it s eosy
12. The degree of turbulence created should be controlled ... to preuent ercessiuely hgh to lubricate the oalve mechanism with the detachable head, it may be noticed that the
rates of burning and, correspondingty, Imt very high rates of pressure rse which cylinder head can be remoued. for cleaning or decorbursing without disturbing ualve'
would nuse rough and noisy running. gear etc.
5.10. SOME TPES OF COMBUSTION CIIAMBENS
In Fig. 5.21 (o), the ai flow has to take two right-angled turns to.enter the cylin-
der. This causes a loss ofvelocity head, and a loss in turbulence level,resulting in
A few representative types of combustion chambers of which there are many more varia- slow combustion Process'
tions are enumerated and discussed below
Fig. 5.21 () is the Ricardo's turbulent head degn' The main body of the
:

1. ?-head combustion chamber' combustion chamber ig concentrated over the valves leaving a slightly restricted
2. Ihead combustion chamber. passage communicating with the cylinder, thereby creating addtonal turbulence
3. f-head (or overhead valve) combustion chamber. d,uring the compression sroe. This design reduces the hnocking tend,ency by
4. F-head combustion chanber. shortening the effective flame travel length by bringing that portion of the head
It may be noted that these chambers are designed to obtair the objectives namely: which lay over the further side of the piston into as close a contact as possible with
o A high combustion rote at the start' the piston crown, forming a quench spoce. The thin layer of mixture (eritrapped
o A hi.gh surface-to'uolume rato near the end, of burning, between the relatively cool piston and also cooled heail) loses its heat rapiilly, thereby
o A rather centrolly locoted sporh plug. avoiding knock. By placing the spark plug in the centre ofthe effective combustion
space but with slight bias towards the exhaust valve, the flame travel length is
1. T-head combustion chamber. Refer Fig. 5.20. reduced.
Spark
This type of combuetion chamber (earliest type) y' plug
Adaantage*
ws used by Ford-motor corporation in 1908 in its fa-
mous model '?-. (i) Valve mechanism simple and easy to lubricte.
Tbe ?-head design has the following d.isa.d.van- (ii) Detachable head easy to remove for cleaning and decaburising.
.\
tdges i Exhaust
(li) Valves of larger sizes can be provided.
valve
(i) Requires two cam ehafLs (for actuating the
let valve and exhaust valve separately) by Disodoantages :
cams mouted on the two cam shafte. (i) More surface-to-volume ratio and therefore ,nore heat I'oss.

(ii) Very prone to detonation. There was violent (id) lnger length of flame travel.
detonation wen at a cornpression ratio of4 (with Fig. 6.20. f-heatl combwtion chamben (iii) Valve size restricted'
a fuel ofocane number of50). (iu) Thermal failure in cylinder block also. Inl-head engne the thermal failure is confined to
cylinder head only.

\
220
INTERNAL COMBUSTON ENGNES COMBUSTION IN S.r. ENCNES
B.I.head (or overhead vlve) conbugtion 221 :,

chanbe Refer Fig. 5.22.


- this type of combueion chamber t"" "m-t. i"let valve and the
exhaust valve located in the cylinaer
heaa. iliJi i. *perior to
side valve engine at hig compression
ratioe. ""sr"
Ad.vantages:
(j) Reduced pumping losaes.
Secondary
,", (since the tarer vatvee
ffi:tn'Sffi;:fffffl and larger cfiambr
(Rlcfi mixtue)
(ii) LeBs prone o detonation
(since the path offlame
(iu) travel is educed).
14"" force on rhe head tfr*r"*-or-r'iUiry of leak-
. age ofcompression gasesTL!:"g
orJacket water.
(u) Lower surface-volume Fig.6.22.I-head
raio and, tl"*f"*, less hat combustioa chamber.
air pollution. loss ad lege Man combustion
(ui) Eagier to cqt. chamber
(Lean milure)
4. F.head conbutlon chonber
o rn such a combuetion chanber o-ne
valve ie in head and other in the
o
Piston
is a compronise betweu -Uea."
.r One of the most-perfec.F_head-eogio", i-ii combusion chambers. block. This design Fig. 6.23. Divided combustioa d,ohr.
(.ge ype) is the one used
company for several years. Its by the Rover which enhancee he conbustion here, creating an orifice that can
(i) High volumetric efciency "d*hrs"; *" i . mqjor
do a this ie a
design challenge.
; .-
() Maximun compreseion
ratio for rel of given octane rating o A divided chanber-engine, oftenly, will also be a stratified arge engine.
(i) High thermal efrciency ; slsten i8 d'esigned to,supply a rich mixture in the secondary-chariber Th intahe
and, a lean
(iu) It mi'xture in the main chamber, The ich mixtre with very
can operate on.leaner air-fuel ratioe higtisreiri in tue eecondary
without rnisfiring. ghapb_er wilr ignite readily and combust very quickly. ih" -n;;; gasea expanding
ttrii design id tne a^pta ilt
onism for operatian of uolues and back through the orifce will then igaite th lan mixture in
"r" ,;"7;ro:ffifnoof expen- mixture often so lean that it would beiflicult to ignite with a
the-ain chamber, a
spark plug alone. The net
design of this type of chamber result is an engine thot has good ignition and. colmbustion, y"
,TJ*;uccessfirt is that used in $/iIys jeep. iir-ol" *ostry rean to
";r'aii:xi;ffiii6H*:#ffi
";. 3:nxm:i:#;riw,*^x:,
giue good fuel econom!. placement and tirring
of intake
proper air and fuel to all parts of this engiire are extremely
Note' A variatio oft'his tJpe ofcombustion chanber on some
"u"t
irli".;is to rupply the
irnportant.
C.I. engines is one with a totally passive
secondary chamber, with allvalves ard injectorrtl-ii1l-"
of rheif rrarge conbuetion c]ambers -"i" when combustion occrs in te
fff:T-1iirrffiTffiffiilT,*ffi*t;ffi,#'Byus ancr main dramber, high pressure forces gae through the u..y
*ifi." and raisee the preseure in the cecondary
"rramber.
*ilti:;9"'ffi fl Hffi ",fi ff:y;ri**mffi ;.ffi fi :#;s''"T#tJ'"'l:Til: chmber also. wn tle oressure intl
in tlu nnndzrvchotbr low fu*intotln ^""""^tt, ili"ia
rnl
"-"ll au;og trn p*er strcra ttu highpnessure gd*g
n"utrn"preesure intleminchambrtaahigcr
6.10.1. Divided Combustion ChemberE ^oin"ho^ir.
bvel ftr a short tin and aves sm**,."tWill-l*""i
it#"j u* part^ d.uring thepowershoft. firis kd of
eecondary chamber usua.y consists of aboul s-io-per""riioiit
o some engines have divided combustion
charnbers,, usualry
.urr"rce volme.
creaance volume in ttt" with
o rhe about g0 percent
v_olume as
-"1"-"i"i-r"",trr" pisto_n and about 2,'percen of
a eecondary the WORIIED EXAMPL,ES
combustioi i l')"'"a in "h";;;;;; a eTat! ofici (tc' s'z'
!!:yt!
the smalt se'""*
throughthcoritue,where.tr;;;;;;;";;tr;"i:f:.r{#X:X#::::#7;Xi Eranple 6.l. A s.I- engine operating at I2o0 r.p.m, has a r0.2 cm sparh
bore with plug
in rhe main .0"_o:L^lill"B:;Fcril:lJ offset bv 6 mm from the cent.re.-The Bparn p;"g
x ii"'at 2o.c before T.D.c. It tokes 6.5. of engine
grearer volumetric effrciencv' r".ior"r" sysrem can be designed for Totaon for cornbustion to develop oia g"i tito
chamber' and the otin"e
tt is esibl
uetne"i;il;;r"".r r,r-"" *th; ;fi iJlrr'" ,""ooa".y
l" speed, is 15.8 mls, Cahulote :
o^u p"opagation mod,e, where tru ouerage flanne
is shaped to suppry this often, the
second'ary chmber is-ca'ed
t ""riri rh;;r;;. e, gr_" gases in the secondary @ Tim of one combuston process (i.e. time for flame front to rah the farthest qrind,er
chamber wu) Ln 9ec. :
ffi :Hf*p"::mbustion, tr," p""e",r"e-Js"" *a n"iJrg g"" rhrough (ii) Cranh angle position at the end, of
*,hi;;.k;;;:ff :i:;ii,":,,iJ",i;il::"1i1#1?li,J[:**i{lr,#T
"ro"""ii".n combustian. (Madras Universlty)
222 COMBUSTION IN S.I, ENGINES 223
NTERNAL COMBUSTION ENCINES
I
() Half throttle half speed will result in an increase of l4/o in delay time over thet
Solution. Maximum distance of flame travel *t full
throttle halfspeed i.e. by
=ju""+snarpue-otrset 14
I ,* xll=1.54'
=xLO.2+ $ = u'.,,, .'. Delay angle = 11 + 1.54 = 12.64"
Combustion period remains same as 1?"
6'7 xlo-2
Time of flame travel
t = --iS^S - =3.6076 x 10-3s .', Total period = 12.54 + L7 = 29.54" ; end ie 18" cT.D.C.
12oo
.'. fime of spark = 29.54 - 13 = 16-C. TJ.C. Ge.)
12oo r.p.m. = 16- x 360 = 7200 des/s This is accornplished by o vouum dcuice conmtted, to the inlet manifuld.
.'. Crank angle for flnre travel
= 3.6076 x 10-3 x 7200 = 25.975 deg.
Time for combustion to develop = 6.5 crank degees
6.5
Conbueiou naybe defined as a relativelyrapiddremi:al cmbinationofhSrdmgen anilcarton i the fuel
=
7r or 0'9028 x l0'3 s with the o>rygen in the air, resulting in beration oferg5 in th fom ofhcet.
Ignltion lag is the time lag between the fixt igniting d fuel and mmenement of the main phase of
(i) Tine for one combustion process
ombution"
= Time to develop + Time for propagation a Pr-igtritlonis t}le ignition of the homogeneous,pixture in the cylinder, before the timed igation spark
i
= 0.9028 x 10-3 s 3.6076 x 10-3 occurs, caused by the local overheating of the combustible miture, The stsilard test for preignition i6 to
= 4.6104 x lO. (Ans.) shut offthe ignition. Ifthe engine sll firee, it is assurned that pre-iguition was taking place when the
(i) Total crnl rotetion ignitionwas on.
A very sudden rise ofpressure during combustion acconunied by metallic hanmer like sound is called
= 6.5 + 26975 = 31.48 degees ofcrank rotation.
Since spark is fired at 20" before T.D.C, the crank position will be (38,48 - Z0) detouaon.
or Performmce umber (PM is a usefrl meure of detmstion tendency,
13.48 degrees after T.D.C. (Ans.)
Example 6.2. In a trial on S.I. engne at full speed futl power (.e., fuy open throttle) the .,. _ klimep of teet fuel
spork occuned, 26'bT.D.c (before bp dad centre) ar d.etay endd 1'bT.D.c. Assuming ttnt the klimep ofiso-odane'
conbustion per:wd. slnuld finish 13" aT,D.C, (ofter top d.ead. centre) fur madmum power and. that G The higrnst u&ful compression mtio (HUCR) ia tfre highest compression ratio employerl at which a fuel
the effect of half closirg tlu throttle ot constdnt sp*d, is to increase the detoy period by 74Vo of the can be wed in a specified euiine undera specifed set ofopra"g coDditins, at which detonation rst
uolue dt full throttle, estnate tle opmum sparh mng for mo*mum power under following beome audibler"itrr both the ignition aad mirhrerbeag a{iustedtogivethehighctefficiency.
cond.itions : Swirl is rotational flow of drarge within e cylinder&ut its rs. It is genratetl by constructing the
(i) Under full throttle anditions when the engilw is opera.ted, at half thc maximum speed, ; intake eystem togive a tangential componsntto te intaLe flow ae i enters the cylinder.
(ii) When the enghw is operated dt conditiotts of half the marimum speed ond, the throttle & squish is the radi"I iwud motio ofthe gar ni:ture. AsepistonnearT.D.c. sguishmotioagenemtes
half open a smndarSrrotationalflowcalledtmlC,Ihimtamocon abouta cimmfemtial axisnearteoter
edge ofthe pistou bowl.
State how these alterations in optirnum spark timing may be rchieaed. in practice.
9. Quencb aree is defircd by the parallet portioa ofthe pistoa md cylindahead which almost touch each
(Bonbay University) other as te pieton approactresT.D.C.Itis defircdaspccentage ofoppcingllat arearelave to.the piston
Solution. gom area.
.'e The d'elay period, qt constdnt throttle, is constant in time an:d, thus increases in angle 10. T\rbulence consists ofraadomly dispersed vortices ofdifierent sizes yhich beoregrytrimposed into
with the speed. tlre air, or air and petrol mixture flow sbeam-
o The corlrzsfon pernd s coata,nt in angle. ll. the speed ofthe f{ane propagation is mughly proportional to the velocity at the periphery ofthe
The delay period = From 26'T.D.C. to 4' vorticss.
.D.C. i.e,,22"
The combustion period = From 4' T.D.C. aT.D.Q. i.e., L7".
t?- cylilder air rwirl is delined as tlrc ratio ofangular rotational speed about the cylinder axis.
1& swlrl ratioi.s dened as the ratio ofairrotatioaalspeed.tocrakghaft rotaonal speed.
(j) Full throttle halfspeed will result in delay lL The gur{ace to volune raio
is the ratio ofthe combustion urface area to tha ofits volume. It increases
being reduced =11'for the same
linerly wittr rising compression ratio.
time thus igrition timing should be arranged so that the "?
11 + 17 = 28', ends 13' oT.D.C. l5. ovensquare (l< D) engies are morc suitablefo galoon car petrol engin*, whereas under square
.'. Time of qrark = 28 - 13 = f5" T.D.C. (Ans,) (Z > D) engines are bettr utilieed for lage diesel engines,
A centrifugal deuee is used to accomplish this task.

\
224
INTERNAL COMBUSTION ENCINES COMBUSTION IN S.I. ENGTNES 225

OBJECTIVE TTPE QUESIIONS


THEORETTCAL QIJESTIONS
Fill in the Blanks or Say Ter, or t{o, :
"' my be defined as a relatively rapid chemical combiutim
l. Defme 'combustion'. State the general conCitiom oecessary for combustion.
ofhydrogen and wbon in the fuel w.iti the
orygen in the air resulting in liberation of energr in e form of heat. -2 Discuss the ignition liniits ofhytlrocirbon fuels.
?- The lower and upper limits of ignition of tbe mifurederadupon the temperate
3. Explain brielly combustion phenomenon in S.I. engines.
&
an mixture ratio.
Itre time lagbetween first ignitingoffuel and comncoemeut of the mainphase of combustion
4 tr/hat do you meaa by pre-ignition ? IIow can it be detected ?
is called o. E:rplain the difference between (j) pre-ignition, (ii) auto-igniiion, and (iti) detonation. \
the period of ...
6' E:Plain the phenomenon of auto-ignition. Explainhow auto.ipition is responsible for knockingin S.I.
4 The ignition lagis a chmical proces. engines.
An icrease ii' compreseion ratio decreases the mimue pressure aad ttre work transfer. Explain the phenomenon of kmking in S.I. enfines. What are the difierent factom which influence the
6, The higher the selfignition temprature of fuel, .,. tbe ignition lag. knocking ? Describe the nethotts used to supp. -;a it.
7. Ignition lag is th smarlest for the mixtue ratio whiih gives the marimum temperature. & Exrlain the main factors that i
& Igrrition Iag is ... ifthe initial temperaure and pressue ae inserged. 9. What is performance numbe ?
9. Igrrition lag is rruch affected by turbulence. 10. What are the factors that limit the compression ratr\o.qhat can be used in petrol engines ?
10, Usually the spark should occur at about 15. fore T.D.C. ll. "Abgormal gmbustion knmk produced by surface igni\n in S.I- engines is more harmfil than nomal
cmbustion \nockl. Justify the statement.
ll. The conect imtnt for tle itroduction of sparhie ninly determined by the ...
. t2.
12. In general riclr nixtures bu fastr,
l&
1& For maximum nwer atd eeonorny a slow buming ftd needs a higher epark a<lvance
tha a fast burning l/L Spae of the clearance volme controls the detoation in caee of S.I. engine. Comment.
fuel.
15. What is ignition lag ? Disrus the effect of engine vriable on ignition lag.
14...istheignitionofthehomogeneousmirtureintherylinder,beforethetimedignitionsparkoccurs,
16, Discuss the effects ofthe following variables on engine heat transfe :
caused by e lcal overheating of the combutible nixture.
(i) Spark advance ; (ii) Engine output;
15. Pre-ignition inceasee te tendency ofdetonation in the engines. (iii) Pre-ignitioo md knocking.
16' A very sudden riae ofprcsure during comburon accompanietl by metallic hammer le sound is callecl 17. "Thehighest compression ratio that catrbe wedin aS.I. engine is linitedby the itetonation ctraractristie
of the available fuel". Juti$ the statement.
17, .., numbet is a ugeful measure ofdetonation tedenc. 1& -rhe retarding ofspark timing in a s.I. engine
will reduce detonation". JusS the statement.
I& ... is the rotational flow of charge within tbe cylinder about its
axis. 19. What action can be taken with regard to the following
fariables, in order to reduce the possibility of
19' fire ... area is defined by the parallel portion of the piston mil ryliniler head which almoet touch each detonation in a S.I. engine ? Justifyyour answers by reasLps.
otherasthepistonapproachesT.D.C.,' (i) Compressionratio; (ii) MmJo{chargeinduced; .
2D....consistsofrmdonlydispersedvorticeofrlifremtsizeewhichbecomessuperi4posedintotheair,or (ii) Mixtueinletternperature; (iu) Enginefu;
air and petrol nixture flow steam. (u) Distmce offlame travel.
2t, 'cylinder air swirl'is defined as the mgrrlar rotational sred about the clinder axis. 20. Discuss the effect ofthe following engine variables on flame propagation:
n ... ratio is defined as the ratio ofairrotational speed to cnnkshaftrotaonal speed. (i) Compession ratio ; (ii) Fuel-airratio;
2& The degree ofturbulence inseases .rectly with tJre piston speed.
(iid) Turbulence. (iu) Engineload;
(u) Engine speed
2t Whentr < D, thetr : D atio is said to be ... .
2L. "Auto-iglition is the cause of detonation". Jwtifythe statnenl
26. Divded combustion chambers offer Ngh voturnetric efficiency, good fuel ecoiromy, a1 cycle operation 22, "Compressednatural gas (CNG) is preferable in S.I. engine than C.I. engine" ?JustiS the stetement.
fl*ibitv.
a, Why is spark advance required ? Discuss te facto that afrect ignition timing.
?4, On what basis are S.I. engines fuels compared when they are better thm iso-octane in anti-lmock chrac-
teristics ?
zft. Discuss the three basic requiremats of a good S.I. engine combrxtion chamber.
t. Cornbustion 2 Yes & icubetion 4 Yes 5. No 26. Discuss the general principles ofS.I. engine combustion chamber design.
6 Longer 7. Yes & redued 9. No 10. Yes 27. What are the advantages of overhead valve combwtion chmber over sid.e valve combustion chamber ?
11, igrdtionlag 1 Yes 13. Yes 14 Peignition l Yes
16. detonation 17. Performance. 1& Swirl 19. quench AL Turbulece
21. Yes 22" Swirl 23, Ys 24. ovenquare 25. Yes.
COMBUSTTON N C,I. ENGINES 227

Also, because of heterogeneous mixture, leaa mixture is used,


-
These factors make the engine heavier.
Tlre incomplete combuetion of heterogeneous mirture, ard d.roplet combustion
result in the emohe and, ofur.
o c.I. engines ae maufactured i the following range of speeda, speeds and power out-
puts :

Partiulqs Range
Combustion in C.I. Engines 1. Piston diameters 50 mm to 900 m-
2. Speeds 100 r.p.m. o 4400 r.p.m
6.1. Introduction. 6,2. Combustion phenomenoa in C.I. enginee. 6.3. Fundnentals ofthe 3. Power output 2 B.P. to 400m B,P.
combustion pms in diesel engines. 6,4. Delay period (or ignion tag) in C.I. Engines.
6.6. Diesel knocr. 6.6. C.I. engine comhrstim hrhe*Primuy qilertions in the design 6. COMBUSTION PHENOMENON IN CJ. ENGINES
of conbustion clnbers for C.I. enginee--Qasic methods of generatlg,aiir nitl in C.I. tngines
combustion hnhen-'Fypes of combui cbambers. 6.7. Colil sufing of C.I ergines- o The process of combustion in the compression igLition (C.I.) engine is fundamentally
Highlights-Objective Ilpes Queeone-Thiortical Questions. different from that in a spark-iga.ition engine. tn c.t. enghe combustion occurs by the
high temperature prduced by the compression of the air, i.. itia an auto-igtior" For
this a minimur. compression raio of 12 is required. lbe eficiency of the cyJe increases
6.I. INTRODUENON with higher valuea of compression ratiout [6 6-inutr pregsure reac]red in the cyl-
inder also increases. Thie requires heavier construc{,ion. Tte upp". rinit 6f 6p[s-
The compression ignition (C.L) engine was developed by Dr. Budol Diesel, he got a patent of sionratio in d c.I. engins is r. to rrechanical hctor and is a compmmise between-hi
his enginein 1892. between-high
\ efficiency and low weight and cost. The uomal
uormal compreesion raios
raioiare
are in the ranse
range of
of 14
It is a very inportant prime rrover\these days and is nding wide applications in : to,17' but may be upto 23. The ai-fuel ratiog used in the c.I. engine e betweenlg aud
buses trucks, tractors ; \ 25 as against about 14 in the S.r. engine, and lence c.I. ert.n.inei ore bigger ar huvicr
- for tlu same power than S.I. enginee.
locomotives,
- In the c.I. engine, the intake is air alone and the fuel ie injected at high pressure in the
pumping sets ;
- form of fine droplets near the end of compression. fiie leds to delaipeod in the C.I.
stationarr industrial applications ; engine, is greater than that io the s.I. engine. The etact plerwnerwn of cr,mbustian in
- the C.I. engne is explaind below.
small and nedium electric power generation ;
- Each minute droplet offuel as i enters the highly beated air ofengine cynder is
narine propulsion.
- quickly surrounded by an eavelope of its wn vapour and tbic, in turn ail at an
The following points are worth noting about C.I. engines : appreciable interval is inflamed at the surface ofthe envelope. To evaporate the
I lts thermal efficiency is higher thdn S.I. engines. liquid, latent heat is abstracted from the ruqdig air wbicl reduces the tem-
* C.tr. engine fuels (diesel oils) ae less expensive han engine fuels (perol or gaso- perature ofthe thin layer ofair surrounding the drople! and eone time must elapse
llne). Furthermore, since C.I. enginee fuels have a higher gravity than petrol, before this temperature ca be raised agaiu by abekacting heot from the rnain
and since fuel is sold on the volume basis (tres) and not on (kg), more kg of bulk of air in this viciuity. A.s soon ae this vapour and the air iu actual contact with
fuel per lileae obtained in purchasing C.I. engine fuel.
it reach a certain temperature, ignition will take place. once igniion has been
started and a flane established th heat required for further waporation will be
Due to the abw.e nentioned factors the running cost of C.I. engines is less than S.I. supplied from that released by combustion. The vapour would be buming as fast as
engines and as a consequence these engines find wide application i i transport and it can find fresh oxygen, Lr-, it will depend upon the rate at which t is moving
other appcations. . through the air or the air is moving past it,
. A qI- Qh rs not muh favoured. in passenger cars due to the 11 the C.I. engine, the fuel is not fed in at oce but is spread over a definite period.
(t)Hee#e&eieht; The first arrivals meet air whose temperature is only a little above their self-
ignition temperaure ancl tbe delay is more or less prolongefl. The later arrivals
(ii) Noise aad vibration ; frnd air already heated to a far higher temperature by the burning of their
(iid) Smoke ; predecessors and therefore gbt up much more quickly,lmost as they issue from
(rtp){}kr. the injector nozzle, but their subsequent progress is handicqped for there is less
'
tn view ofthe utilisation ofheavier compression ratios (I2 L to 22 : 1 compared to oxygen to find.
- 6 : 1 to 11 : 1 ofS.I. engnes) the heavy forces act on the parts ofthe engine and If the air within te cylinder were motionless, only a sqq* proportion of the fuel
would finil sufficient oxygen, for it is impossible to distribute the droplets uniformly
therefore heavier parts are required.
throug-hout the combustion space. Therefore some air movemeot is absolutely
226 essential, as in the s.L. engine. But there is a fundamental difference between the

I
INTERNAL COMBUSTION ENGINES .ceMBusrroN lN c.r. ENGTNES z2g
air movements in the two ypes ofengines._In
ttre s.I. engine we carit turburence
and mean a contu-sion ot*li.l"
*J.Joes with no c";;;i;;Jien o The delayperiod exerts agreatinfluence in the C.I. engine combustionphenourenon. It
break up rh-e surface.of tm-ri'-"-n"it,.3-d flow, (to is clea that the pressure reached during the second s[age will depenil upon the dura-
hroughout aa,externally preparedt.'UtUUf" tg ilril;; iiT"ri""o, of flame
-of
_ixture). tion of the delay pelod ; the longer theetay, the more *pa higr the pressure
. it air swirt ard -uu" a
au ora'.;;-;;::i"::t:
orderly movementof.the;t1" 5:{. rn
In the Lr.r.
C.J. ensine we calt
r

t-"y J,iiie air, with or


nse, since more fuel will be present in the cylinder before the"^i rate f burning comes
*ithoot.o_ild.Tgan
ecoag or turbulence, so aa to ."g under control. Thig causes rough running and may cause desef &noc, TheJfore we
to each burof and sweep away the produits
. .o"L,n"lii" Iopplv orru.l ,i. inust aim to keep the delay period a.s short as posdble, both
.r,r;llfr::,", orcil]r.ii"r-#ii" for the sake of smooth run-
tend to
"therwise
' ning and n ord'er to mdintan control ouer the pressure clnnges. But som.e elay period is
Three phases of C.I. englne.combustion ' recessdry otherwise_the_ d.roplet would. not be dispersed, in th oir
f* complete com-
: bustian, However, tho delay period imposed upon is greater than what is nieded and
mavbe considered in thee distincr the designer's efforts are to shorten it s muc as poJsible.
i:rffi,,:::il;ilHlu'tion srages as shown in Fig.
6.t. 2' Period of rapid or.uncontrolled combustion. The second stag of combusiion in
2. Period ofrapid or uncontrolled combustion. C'I' engines, after the delay period, is the perid. of rapid. or uncontrolld coibustion. This period
3. Period of controlled combustion. is.counted. from the end, of the d.elay period to tlte pint of rnaximum pressure
on the ndicator
d'iagram. In this second stage ofcombustion, the ris ofpressure is rapid because during the delay
(or burnig on the expansion
^* o}/"T'!l#if {#::i";b^#;rru::nins stroke), which period the droplets offuel have had time to spreatl themselves out over a wide
fresh air all around them. About one-third oiheat is evqlved during this process.
area and they hav!
1. Ignition delay period
The rate of pressure rise d.eperd,s on the dmount of
r The delay period is counted om fho a+o* r:-:--!- - fuel present at the end of dclay pernd.,
bustion curve departs
busrion uo..* fr ' - --- -ry ' v {'Euoua,lf,me
nJil*T^3:j$_gp:crion.ro rhe Potnt
poinr where the

rhep_0 com_
degree of turbulerrce, fineness of atomzatian and spray'pixrin.

. The rtelaw ,^":-T,*_lTTfln \e' sv


jor ini.tion ur
uo 6!!uu oirotoring)
moEonng/ cur.lr'e'
curvJ 3. Period of controlled combusion. At the end of second stage of combustion, the
delay na_ia,r
period can
^^- be rot
*toy temperature and pressure, are so high that the fuel dioplets injected in th. thi"d stage burn
tay. rhe period
'"i "i"^i"ot *.
orrryrij*jjijjlgf-9intg 4hrci,ea!
':r;x:::,ii:#:ir::;:x-*l::#:ilxri.Tfti;vi"iTrrii!*!; almost as they enter and any further pressure rise can be controlled by purely mechanical means,
i##?::,T#::.;r"n"^it"Tii"i"?#.:#::;"!#i:"1!,:#;:!#,
the fuel is atomized, vap6r:'--:--
' *"
svwt'o't8'- Ln
ad raised,tne
pysical delay period, i'e. by the injecion rate. The period ofcontrolled combustion is assumed to end at maximurn cycle
chemcal
:hemical detay
:hemicar ,"_^.r|1jo:.!I4_with air, then
nerio,r ,"ai
delay period .ad rarsed
1ii::1
i"_t"_p"."lu.
rn temperature. iilinu
temp.erature. In the temperature.The heat evolved by the end ofcontrolled combustion is about ?O to 80 Der cent.
or ieniti,n tahes nrno "";ri11o:,ii*^T,p-11
-3i irjll.i??! ::d
crccererates untit inflammation
::.:"#ifr ;fi
r,_
T:#:t jjjn:i:{$*,ff i"!J,,iTi!,{!"xm,:,
"J.jfi the chemical delay inile . engine). ls
4. After burning
o The combusion coninues even aft", the fu"I injection is over, because ofpoor distribution
of fuel particles. Thie buming may continue in theexpansiory stroke upto 20" to g0" of
crank travel from T.D.c. This continued burning, cillgtl-the after birnng, may be
considered as the fourth stage of the combustion. The totat heat evolved by the end of
entire combustion process is 95 to gTvo ; B to 57o of heat goes as unburned fuel in
exhaust.
i160 o In thep-Vdiagram, the stages of combustion are not seen because of little movement of
I piston with crank angle at the end and reversal of stroke. So for studying the combus-
tion stages, therefore, a pressure-crank angle or tirne, p-e ot p-t diagram is invariably
gso used. In the actual diagram, the vaious stages of cornbustion look merged, yet the
o individual stage is distinguishabie.
Faatorc aftcng combustion in C.L engine
The faetors affecting combustion in C.I. engine are as follows :
(1) Igition quality of fuel (cetane number)
(2) Injection pressurie ofdroplet size
(3) Injection advance angle
(4) Compression ratio
(5) Intake temperature
(6) Jacket water temperature
40 60 100 120 (7) Intake pressure, supercharging
Time, degrees of crankshalt otaton ___)
(0) (8) Engine speed
Fig. 6.1. Combwtion phenomenon (9) Load and Air to fuel ratio
of C.I. engine.
(10) Engine size
(1 1) Tlpe of combustion chambe.
23o TNTERNAL coMBusTror{ ENcrNEs COMBUSTION IN C.I. ENGINES 231

6.8. TUNDAMENTALS OF TIIE COMBUSTION PROCESS IN DTESEL ENCNSES Outer laysr


Outr Droplet
of separated upstfean
Eff'ec of comprteeion Ratio ad Englne speed on cylinder pressrts nd ren- drops Inner layer raygr diskbution
d9
perature (core)

o The power output ofa diesel engne is controlled by uarying the ambunt offucI sproy
i\iected into a ql_indcr filled with compr*sedtnd. heated aii whereas tbe peid engirg
is controllqd by throttlitq the pre-mbed clwrge enterihg the cylinder.
o llte pressure and. ternperature reached. at the end of the compresson stroke will &pend,
primarily upon the compression rotio, intake tenperdture aid speed of the engine.
It has been observed that injection usually commences 15o to 20. before T.D.c. t.
when boh cylinderpressures and temperatures are much lower. Asa-eztnple,a
15 : 1 ompreeeion ratio engine would havi something like 600'C maxinum tem- I
/\.
perature at T.D.C. but at 15" before T.D.C. this would only amount to 590.C. Downslrsam
sdge
Further it can be seen that the pressure and temperature rise in the cylinder with
increased speed is laryely due to the redrced. time auailable for comprcd, ar to (a) Wide-spray small{roplel pnetratiqr (c) Fuel spray disttibution with ar swirl
escape past the piston rings and. heat to be lost through the cylind.er walls atd head-
Diesel Engfne Eeterogeneoue Charge Mixing
The ai-fuel mixture formation, in the esel engine, is of a lreterogeneous natures, that is, it + Lenglh of spray
is locally concentrated at various sitee and is thcrefore unevenly distriiuted throughout tle eylin- W" = Wdth of sPraY
der and. combustian chamber. 0s = lnchrded angle of spray.

Injected fuel spray penetrates the highly compressed and heated air nas whee it is
- pulverised into nfleqr small droplets in a localised formation. 1'5s nig of the
localjsed spray of fuel drop)ets i the hot air charge causes stoichiometeric (1,1i? : 1 by
weigbt) air-fuel ratio combugtion zones to be establihed which are compleely surmunde
!V nure air only. Thus the overall (averaged out) air-fuel mixture ratlo range may vary
from a rich, full load,20: 1, to aweakno-load, 100: 1, air-fuel ratio.
Distribulion
Mosf engines operate with at least ZMo e'.ceee ai due to dfficuttx of htrd,ucing
- sufficient exposed oqtgen to the fuet u4pour in the given um'auoiiobie s that the
of fuel
(b) Natrow-spray larg&droplet ponetraon
combuston process can be completed, before the exhaust ualve opens. If the oxygen
aupply is portidlly prevented. f'rom gettng to the fuel vapour early enough durirq tftc Fig. 6.2. Inje.ted fueI sPray chamcteristi.
power strohe tlen incomplete combustnn, polluted. exhaust gos and d,ark smoh,e will
result. o The liquid core, now surrounded by layers of heated vapour, oxidiaes burns as fast as it
can ; that is it finds fresh oxygen to teep the chemical reaction going on.
Dleeel Engine ltrJectd Spray dornbustion process o When tble physicol deloy tn convert the fuel spray into tiny droplets and the cemi'
o Towards the end of the compression stroke when injection of the fuel into the combus- cal reoction d.elog to establish iglition fron the initial oxidation process are over, e
tion chamber commnces, the quantity offuel discharged is spread out over a rredeer- rate of burning is depend.ent on tlv speed at which the d'roplets ore mouing through
mined period. the ar or the air is mouing past the d.roplets.
Tte fuet spray enters the hot combusion chamber bt does not inmediatebr igite,
instead it breaks up into very small droplets (FiS. 6.2) airdjonce these liquid droples Compression Ratio (r) :
are formed, ther outer surfaces wi.lt immed.iatel! sturt b eudporate so the will-be a Increase in compression roo exercises the following efecs :
liquiQ core aurroundcd, wth a laler of uapour. At this point itlhould be explained that
he brning ofa hydrocarbon fuel in air i: purely an oxidation process. lbus, initially,
o The cylinder compression pressure and temperature inciease; the ignition tiine lag
heat liberated from the oxidation ofthe fuel vapour is less than ihe rate at whih heat is between the point ofinjection to the instant when ignition first commences reduces.
extrated fur conuection and. conduction, but eventually a ciitical tem,peraturc is reached, a The density and turbulence of the charge izcreose, and this i ncreases the rate of burnng
when the rate of heat generated. b1 or,id.ation ezceeds the heat being lisspated b1 anuec- and, accordingly the rate of pressure rise ond' the rnagnitude ofthe peek eylindcr pres'
tion and rad,iation. As a result, the temperature rises which, in turn, speedi up the sure reached. The characeristics ofthe pFessure rise relative to.the piston stroke or
oxidation process thus futher increasing the heat released until a flame si& or sites crank-angle movement is illustrated in Fig. 6.3 and Fig' 6.4.
are established,, this being known as the ignition and the temperature at which it
Thermal efficiency and the specific fuel consumption are improued (Fig. 6.5)
occurs is called the eelf.ignition temperature of the fuel under these condition. ?
heat required' for further euaporation of the fuel droplets wll thus be provi&d. from r Raising cglpression raQg.esults in red,uction in the mechancal efficiency as shown
heat released by the oxidation process, which is referred. to as combultion in Fig. 6.6{since the higher cylinder pressuies increaee the pumping losses, friction
NTERNAL COMBUSTTON
ENCINES COMBUSTION IN CI ENGINES
losses and compression
and erqansion losses as
gether the rapped more work is done in squeeziqg q-
"ir ""1.

t

II
I
1
,l
s t*9 t
I 60
\\ {
I
I

6
50
\
fr\--
o


"449ry9 I
o
e
o
401 F
E

E
o
o
301
I
I
tttta\

\i\ E
o
kqqg=,ou* ^E o
E
o
o
e
o
t0
M
r0 12 14, 16 18 m
0 ComPression rao -------)
T.O.C.
B'D'c'
cyinderswptwrume Fig. 6.5. Efrect ofcompression ratio on the thennal
Fig. Effect ofmmpreeeion ratio
-----' eficieacy and specicfuel consumption,
on the chaacterigtic
pressure-volune
-6.3.
diagrarre for a die.s"i;si";: - ^

+ ko"l'g""","n*
I
I
t
t I 4
l
q E
I
E
g 50 6 E
o o
E

E
*l o
a

;;fi-**.M E
E
c
o
b 3ol o
o
,ol J

to 12 14 16 18. 20
Comprssson rtio
----i
to Fig. 6.6. Effect ofcompression ratio on the punping, friction, compression
Bobro r.J.c. o#
oo
and expansion losses md the resultmt nechanical efficiency.
Crank_angle rno\mnf (deg) _-___,

_-^Il. 9.4. Etr:ct ofcompression ratio on the characteristic


pressure -crmk-angle movement "cr"-" ;;d;;;;i"T;".
COMBUSTION IN C.I. ENCINES
INTERNAL COMBUSTON ENGNES
Effects ofVeloue Factor on Delay
Perlod:
Irtriection Spray Droplet size intoke temperoture' comression
ratio'
Effects of various factors such as firel eryry't.?t' advance are discussed below :
o The rate _of burnhq dependa on th relatiue nauenunt of the burning d,rcphts b trtc engine speed, type of co-ut'ilo-Jth"tb";'
toi iqi""tiott
surround,ing air charge,
o The time taken to establish and ignite a film of vapour surrounding a quid drqlet ir 1. Fuel ProPerties: which
(S'I'T') is the most important property of the fuel
practcal$r ind.ependnnt of the size of thz oplet. However, therate olbunirg and orre- t Tlte self ignition temperatwe
sponngly the pressure rise followiag ignition , willbe dependznt ipon the pod, an- affects the delaY Period'
face area of tle vaporising liquid, dmpWs. _AlowerS.I.T.meonsawid.emarginbetweentandthetenperatureofcompressed
-i,
o A compromise must bo nade ta maintoin sufficent d.roplet size (and, therefore, mo. perod"
ooAhenelouer dlay ., ,)-^.^---^+;^-
;;;;' ""*r""*"* lower d'eta! Peroit and unaot^ni,Y"ili#J;ic
*";;;l;;:,;;el
- ;;;;
r
mentum so that a fresh zupply of air comes continuously into contac with the hrik-
Xni":ffilffi ;.r# -a yyre ar affi
p] :-7,wnb n n
ing size of the unbunt portion ofthe liquid droplets) aad, to have auailabl euf.tcient
;an;; ip:,.!
;':,!"'o" er'
numbers of small d,roplets which prouidc an adequate surface vapour area
fuf npid ffff":ffifl :: E;;;r;A; ii7!' .' ietane n

combustion, properties which affect delay period are:


The oiher fuel
o It is possible, to some extent, to contrcl the droplet size by the injection neettle spring (i) VolatilitY ;
closing load' Generally the greater the iqiector spring loadjtbe smller and finerw be
the drople size, whereas a ght spring needle lad tends io produce coare liquid rop (ii) Latent heat ;

lets. (ii) ViscositY ;


(iu) Suface tensiou' uapour'
tohen to form an envelope of
6.4. DEI.AY PERIOD (OR ICNTTION I,/\G) IN C.I. ENGIIYSS Votatitity dnd totent berrtaffect the time
ension influenc e t'tre fineness of atomsotion'
o In-C.L (compression iga.ition) engine, the fiel which ie in atotbised form is considerably l.he uiscosiy ond
'uo""
colder than the hot compressd air in tbe cylinder. Although the actual ignition ie al-
2. Intake tmperature:
moet instaatan eoue, an apprecbble time elapses before tlie combustion k in
fuil pqres.
This time occupied is called e delay perlod or ignition lag. It is the iim imndi- -oInreoseinintahetemperoturewoulilresultinincreaseincompressedairtemperature
ately following injection of tle fucl durhg whiah the ignitinn process s being initiabd *-li.t.o"r redvce the &lay perind'
and' the pressure does ot rise bexord th ualue it wutd. haie d.ue n comlression of 3. ConPrecsion rstio:
air. period as it rases both ternperature and'
r The delay period extends for \bout 13o, rnovement ofthe crank. The time for which it
c Increase n compression ratio red'uces dclaX
occu dcreoses with ircrease\in enginc speed. 3"'#;*-a"aseincompresaio.nratio'PTry::::*rfr:"trffi1$tJli,"tffi
o In C.I. ngine, the length of thL aaay perioa plays a vital mle, ?is p*bit *'cs a deceas'
:'#:
the minirrum auto'ignition temperature the time
useful purpose in that it allaws tlu fucl jet to penetrate wIl nto the combwtion cpce. If morecules which thereby reduces
oressed air reslti"i'""Jr"#iiJof
tleye were na d4lag the fuel wotdd, bun at the injector and,.there would, be an (Eygen ireaction when fuel is injected'
deficienex around. the injectot rzsulti'4 n rcomplete combustnn. If th delq it fu oir and' minimum auto'igni-
long the amoni of fuq!-auailable for sitnaltdneous explosion ls too great'and tltc ls the dfference between compresaedperiod' 'temperatvre
- ;;r;;;#oz'." in"'"o"'s' the d'elov d'eteases'
resultittg pressure rise is too rapid,
The delay period affeets the rate ofpressule rise and hence knocking. It 4. Englne sPeed:
terns of obsolute tme (i milliseconds)
also al[ects or crank
startability. o Delay period can be grven either in
some delay eriod is necessary otherwirge the droplets would not be dispersed in angle rotdtion.
to the delay angle'
air for complete combustion. At constan speed, delay p *iod's proportianal terms of millsecond's
Factore on which the deley period depeda
- lt variabl, oiJo*n'-aeb1 ne;od may dztease in
: - "p""
but increase in terms of cranh qngl'es'
\\e d.elay period dzpend,s upon tlu foltowirg :
(i) Tempereiure and pressure in the cyliader at tlie time of injection. 5. TYPo of combustion chamber-:. ' combustion
compored' to on open type of
(ii) Nature of the fuel mixture streugth. { o A pre'combustion chamber giues slnter delay
(jjj) Relative velocity between the fuel injection and air turbulence. chamber'
(iu) Presence ofresidual gases. { 6. Iniection dvance :
reason for in-
in-njec.ton adutance angle' Tlne
Delay period, increase-s with increase
(u) Rate offuel injection. o angl is that pressures and
creaserin "tt.v p"ioc;it r"i"t ri
(ui) To small extent the finess of the fuel spray. i":cuo"
"Ju*t"
The clelay period increases with load but is not much affected by injection pressure. ;;-p;;tt;;;"t lo*"t when injecion begins'
o The delay period should be as short as possible since a long delay periad, giues c nxtre
rapid, rise in pressure and thus causes hnoching, il
I

\
236
INTERNAL COMBUSTTON
ENGINES COMBUSTION IN c.I. ENGINES 237
'n;iliil"$a;:ffi1:H5l?ffi :T#:,,*l?_T1od.""g:"lqandoperationof engine o A very long ignitian
ge amount of fuel burns lag after injection crtuses 4 large proportian of tlu fiuI d.ischarge
Abnonnal eombustion ln c.r. d"tiogutpt r"lon. to enter the qlind.er and. to atomise before gnitian and the propqation of burning
enginee
h,.C.l. engines, abnormal combustion
: octuolly occurs. Accord,ingfu, when combustion does cotntnence.a relotive amount of
,
^._ is not a a heat energy will be released. almost immediately, this correspondngly pioduces the
rhi";;;;"';;;;;tr;:;;"::#;i"'l,i;l;,T"i,T,"Tj abnormally high rate of preseure rise, which s mainly responsible for rough and.
:i:Tl"i";"':::'::,!;rr,t" the c'tinder for ttw sim,uttai"ou" lill noisy combustinn process under these cond,ition (FrC. 6.7).
B::j::* d;;;;;;"'o{*t
i. g; t ",it*a*i'iiti".
ll rt ,^t" or o It has been observeil generally, that provided the rate of pressure increase does not
Iy i""mav ""'" Ji ,fl ffi il1"""T*?i1,ffj1,, :h: ^n "
"ll;S, u d n" " ".
o u
o u
stresses in mechanict exceed 3 bat per &gree of cmnh-anglc movemEnt, combustion will be rela{ively smooth,
damage the n*n j|iiri";;;"
engne. 'nqeqse and thermat wheeas betuteen a 3 and,4 bar pressure rise there is a tend.ency to knuh il; above this
bv choosins a uet with . fuet chosen and may be auoid.ed.
-:raterist, ty d" ngt
:^F::"::f,!h"
. "i;;;;rt";;:";'f &ue aoo bng a d.elay periad.. ra,te of pressure rise, d.iesel knoch wII be promneht.
6.5. DIESEI. XNOCK Differences in the lcnocking phenomenon of the S.I. and C.I. Englnes :

' ?::;iri:ili;"'k:::#,!":y::!,b!,!" u:o rp!',\ate:r eressure rise


second phdse of combustia" d.uring the
The following ete lhe dffirences in the knocking phenomena of tlu S.I. and C.I. engines :
1. In the S,I. engine, the detonation oceurs eor the end, of combusfio whereas in the C..1.
."."iuuly hh;r;;;,#;11'ea r" p"i.rry;;;;;"" engine detonation occurs nar the begnnng of combustnn.
av perioJ can b; ; ;iil:
##:i av p ria d ?Ftg. I ii.'#"",i," "r-
der dc t
d i;? e
2. The detonation in the S.I. engine is ofa homogeneous charge causingvery high rate of
t' "i,:{
*t''p"'-itii"g onlv pressure rise and very high maximum pressur. In the C.I. engine, the fuel aud air ae
t"';'J"*";n;H"Tfit:" a marginar serf-isnition rem- imperfecly mixed and hence the ate of pressure se is narmolly lower thnn that in the
() A low combustio dztornng port of thc charge in the S.I. engne.
0i,)poo"r""rrs"ft ;:;ffri"ilfl:1:::1il,H:1"Jff 3. In the C.I. engine the fuel is injected into the cylinder only at the end of the compression
(iu) A poorlv atomized ::ttisearingvarves; stroke, there is no question of pre-ignitbn os in S,L engiw.
T, t;;;;***-i"al" ignirion to be esrablished . 4. In tlte S.I. engine, it is reLdtiuely easy to distnguish bptween hnockit4 and tnn-knock-
[ifl i]1t#:1?ffi":3t."iri#&ucingcoarsedropretronnariin;
(ui) A very row air inrake temperature
in cord wintry ,*inl iil llT,lltJ,ill$il;
ing operaton as the human ear easily find.s the distilrctian.
5. Fators that tend. to reduce dtonation in the S.I. enline increase hnoching in the C.I.
engine. \
Methods of controlling diesel knock (Reducing delayleriod) :
\\e d,iesel knoch can be controlled by reducing delay period. The delay is reduced by the
following :

t6( (D High charge temperature.


I (id) High fuel temperature.
Fsc (iii)Good turbulence.
E (iu) A fuel with a short inducon period.
fia
E 6.6. CJ. ENGINE COMBUSTION CIIAMBERS
o
b30
E
c 6.6.1. Primary Considerations in the Desigrr of Combustion Chambers for C.I.
a
()20 Engines
In C.I. engines fuel is injected into the combustion chamber at about 15'C before T.D.C.
during the compression stroke. For the best effrciency the combustion must complete within 15' to
20" ofcrank roation after T.D.C. in the working stroke. Thus it is clear that injection and combus-
tion both must complete in the short time. For best combustion mixing should be completed in the
Belore
40 60 short time.
Crank-angle movEmsnt (deg)
After
o In S.I. engine mixi-ng takes place in carburetor, however in C.I. engines this has to be
___+ in the combustion chamber. To achieve this requirement in
done a shot period is an
Fig. 6.?. Effect ofshort ad extremely difficultjob particularly in high speed C.I. engines.
long delayperiod on the
aracteristicp_, l*;;. I From combustion phenomenon of C.I. engines it is evideqt that fuel-air contact must be

limited during the delay period in order to iirrlft the rate of pressure rise in the
*,

*l
INTERNAL COMBUSTTON ENCINES
second stage of_combusion. thi:s resurt COMBUSTION IN C.. ENGINES
can be obtained by shortening the detay time.
To achieve high efficiency tn, combuston must be completed. when the
"" iii Ad.vantages of indttctbn swiri :
if,'1,foTff!:imr?;f":il;;.;n"u-ii;n""i;;i;;ndrduns r..Easier starting (due to low intensity ofswirl).
o Tbe design of comb"etion clamber for
c.I. engines 2. High excess air flow teraperaure), low turbulence (less heat loss), therefore indicated
injection system and nozzles " tnust also tahc consideration of fuel
b b" thermal efficiency is high.
The consi&ra.tions ccln bc curruwrizcd as ";"d.-
3. Production ofswirl requires no additional work.
fuUowt :
1. High thermal efEciency. 4. Used with low speeds, therefore low quality offuel can be used.
2. Ability to use lees expensivo fuel (multi-fuel).
Disad.aantages :
3. Ease of starting.
1. Shrouded valves, smaller valves, low volumetric efficiency,
4. Ability to handle variations in speed.
5. Smoothness ofoperation i.e avoidnce
2. Weak swirl, lowair uilisation (609o), lowe m.e.p. and large size (costly) engine.
ofdiesel knock and noise. 3. Weak swirl, multi-orifice nozzle, high induction pressure, clogging of holes, high main-
6. Low exhaust emission,
tenance.
7. Nozzle design.
4. Swirl not proportional to speed ; effrciency not maintained at variable speed engine.
8. High volumetric efficienqr.
9: High brake mea efective pesEue. 5. Influence niimuur quantigr of fuel. Complication at high loads and idling.

u't'' Baeic Methods of generating


Conpresssion ewirl :
o."" Air swirl in c.r. Engines combuation chq-
o The eecond method of generating swirl is by
compression swirl in what is known as srirl
of generctins swirt in a C.L engine combustian chamber. A swirl chamber is a d.iuid.ed. cham-
.o'r"n ft"fi"ilr"" !""Olr:yrmethods chamber,
er. A divided combustion chamber is dened
l' By d'irectw the flow of d.urhg its entry --
,eir
Ttris method is used-in open combustio" ' to tltc qtndcr known as lndtrctiot twirl. as one in which combustion space is divided
"o*t"1. into two or more distinct compartments, be-
2..8y forcing the air through qtansential
pdssage into a sepa,rote tween which there are restrictions or throats
clnnber d\r small enough so that considerable pressure
pression strohe, known as"wi"t
cozrp""o" ,)-- "o'-
method
fti.
--- Inlt differences occur beween them during com-
is used,in swirl chanibers. 1 valv6 bustion process
3. By use ofinial pressure rise dh to par_
uat conbuson to create swirl turbuleniq horyn This swirl is maximrim at about 15' before
as combuction induced oirl. This T.D.C. i,. close to the time of injection. The
u-ged in pre-cotnbustion -l,tL i, fuel is injected into the swirl chamber and ig-
chambers.
chambers ,i ii-""U nition and bulk of combustion takes place
therein. A considerable amount of heat is lost !
Induction swirl : when products of combustion pass back
o In a four stroh engine induction swirl through the sorne thoat and this loss of heat Fig. 6.9. Compression swirl
can be obtained either by careful forma_ is reduced by enploying a heat insulated cham-
tion of air intake passages or maskins ber. Thus, i serves as a therrral regenerator
or houding a portion oi receiving heat during combustion and expansion ad returning the heat to air during
of inlet valve. ,llhe angle of"ir"u-f".uo.i
mask i.s from compression stroke. However the loss ofheat to.surface ofcombustion chamber is greater
90. to 140. of th'e circumf,erence. than induction swirl.
a In tuo;stroke enginc, induction swirl is Fig. 6.8. Induction rwirl bymnshing
credted by suitable inlet port o ln cotnbustian swirl, a uery strong swirl whch increases with speed is generated.
forms. the inletvalve.
o The inducon.swirl senerat{b air intake passages
is very weak. Ifa masked inlet
Adtsantage of comprzasion uoirl :
valve is-used, it providds ari obstruction'in
te pasiage *hi.i ;";;;;;1""ti. 1. Large vlves, bigh volumetric efficiency.
ciency. Therefote swirl oerurgted h weoh
single orifie injectibn nnot providg the""""
,irtiitrri, method. witn a .,oeJ s*i.1, "m-. 2. Single iqjector, pintle type (selfcleanirig), less maintenance'
desireJ air fuer mixing. Therefore, with in- 3. Smooth engine operaion.
duction-swirli we haue to use
-tiiptr.i""-i""nr. 4. Greater air utilization due to strong swirl. Smaller (cheaper) engine.
5. Swirl proportional to speed, suitatle for variable speed operation.

I
24O
INTERNAL coMBUSTIoN ENGINES COMBUSTION N C.I. ENCINES 241
. Disad.uantages:
1' Cold starting trouble due t high loss
due to strong swirl, mechanical
r ;rower indicared efficiency ; E to Bio_or" foul
efficiency lower.
**;;;;""
lff:j#::i"?l ; ttecreased
3. Cylinder more expensive in construction,
' 4. Work absorbed in producing swirl,
nechanical efficiency lower.
Combustion induce srirl :
This type of suirl ls ind'uced by use of
initial pressure rise d.ue to partial combustion.
t chambers which use this t5rpe
of swirl are not
3l;"o-o"ttion much fauoured. these
6.6.3. Types of Conbustion Chambers
In C'I'
engines severar trpe.s of combustion
peculiarities, and desiable, u. chambers are used. Each of these bas
;d";i;i;;#"*s.,Any its own
-"u "" one of these combustion cham-
oers mayproduce Bood resultsri:ff*ll Urt 1"." desirable, or wen poor results in
another. No one combusrion chamber
"",rrlr."ti"",
design has yet been
F;ilil;ffii"i#"* rhe besr
il:*Ji;:iifi::::?flii: ff ffiHT'flfi{IiJJ", ,r,"o, must be that which accomp,isbes
Four specific designs which find w.ide use in c.I.
Fig. 6'11. (a) Ai motion by shrouding the inlet valve. Fig. 6.11. (D) Squish air motion insido cylinder.
engines are discussed below ;
A. The non-turbulent type By shrouding the inlet valve swirl motio/is given to the air entering the cylinder
(i) Open or direct combustion chamber. which is believed to persist during comfression stroke and the tim of injection.
This system gives better performance at/low speeds, however volumetric efficiency
B. The turbulent type
reduces on account of reduction in inlet\area ue to shroud.
(j) Turbulent chambcr
Squish is provided by pushing the air at\he end ofthe compression stroke in the
(ii) Pre-combustion chanber ' space whose diameter is smaller than the cylinder bore. Because of the small
(iii) Energy cell. clearance between the head and piston top when at T.D.c. air is pushed into com-
bustion space providing air movement known as squish. The squish helps in mix-
l' Open or direct combustion chamber : ing offuel and air.
o Fig. 6:10 illustrates the usual desiga,sfp
cotnbustton 2. Turbulent chamber. Refer Fig. 6.12
c ha mb e r, which is representative

r T!9 fgel is injected directly into the"i upper


""r_i"iU"i"t*,p".
po"tio., oltn"
,.:.a::j: . In the'turbulent chamber'(Fig. 6.12) the upward moving piston forces all the air (or 70_
cylinder, rryhich acts as the combustf.rii.t""rl""."ft 80Vo of all ai) at a greater velocitz iito a small antechamber, tius imparting a rotary motron to
type depends little on turbulence t" p;"o;; the air passing the pintle type nozzle. As the fuel is injected into the iotating air, it is partially
Consequenfly, the heat
;il;;;;"g mixed with this air, and commences to burn. The pressre built up in the antecramUei Uy ttrl
loss to the

proper penetration and dispersal of


"fr.."i
tively low, and, easer starti ng,"rr". f
"
*"il.lsi"r"-
-0."l
to'oi.r"
the fuel .ru"urr"* fo.
n expanding burning gases force the burning and unburned fuel an air mixtures back int the
main chamber, again imparting high turbulence and furthe asisting combustion.
Advantages :
mixing with the air, h"owever, high injectioil;;;;r""
and multi-orifice nozzls are required. ti. (l) The insulated or hot running combustion chamber short-
o".urrit"r".
small nozzle openings and.resulis i" _;-."q";;l;j"g- ens the delay period and limits the rate ofpressure rise,
giqg or diversion of the fuel_sprry by resulting in smoothe running.
particles, with consequent highr ;-"i*"i""
"."o*ol,u-i"'.""". (jj) The turbulence is responsible for rapid mixing and burn-
This type of chamber is ordinarily used on ""rtr.
lo* ,o"ri Fig. 6.10. open or ilirect ing of fuel during the third stage of combustion.
glzeg where injection is spread ttt.ougt, g".l#;il
a "r- cmbusuon chambe.. (lil) Suitable for high speeds as the amount of turbulence is
of time and thus ignition delay is;;;ffi;."j proportional to piston or engine speed. The burning in the
tant facto. Consequently, ff;; third stage will be completed early without resulting in
leis- costly fr"t; iihi;,&i' ignition deloy
may be used. late burning.
Many attempts wee made to improve the ai
motioi in open chambers, the important (iu) The demands on the fuel injection system are not severe
(a) b! shroud.ing the inlet ualue, as it is not to be depended upon for mixing, distribution,
Rcfet Fig. 6.11 (a) Fig. 6.12. Tbulent chanrber
(b) by prouidinE sgzjsl, Refer etc.
Fig. 6.11 ()
ZA COMBUSTION IN C.I. ENGINES 243
INTERNAL COMBUSTTON ENCINES
Th" d,isnlaantage progresses at a more rapid.rate. The pressures built up in the minor cell, therefore, force the
-., , -:_t!:t
watts durins the compressian
c.ord starting is dfficurt since air lose heat to combustion
stroh. lh.e ourtoi il-oil", , *l,y cat at burning gases out into the main combustion chamber, thereby creating added turbulence and
:?:#:; the producing beter combustion in this chamber. In the mean time, pressure is built up in the major
cell, which then prolongs the action of the je stream entering the main chamber, thus continuirrg
3. Pre-combustion chamber. Refer Fig.
6.13. to induce turbulence in the mpi. chamber.
Here the conbustion cbamber is separated
intou,o 5. M. Combustion chanber:
chambers. The smaller one of the .nU"'..
3.0 per cent of total combusnon
.""rt
spcae.Ihe "*rpv
communica- o After twenty years of research in 195,1, Dr. Meuner of
tion between two chamber is a narrow M.A.N., Gerrnany dweloped M-process engine which ran
sage or a number of small holes.
.;r;;i"tJ;;_ PreomhJs$on
The ar is force irto.u ct|mber without typical diesel combustion noise and hence it wag
pre-cornbustion chamber by piston named'ruisper engind.
u"g tiu
:r-1:,1""\. tt"pro_riUi;;;f,;-
F ,et is_ij_ected ini" "o_o*"- o Fig. 6.15 shows a combustion chamber developed for
r"." cnamber rs desigued to run hot and thie small high speed engines. It difrers from the other open
i:..,
rn shortening the delay period of tuel results
*hi"h i"tishi;;;- combustion chamber engins in the respect that fuel
sirable. The products oL tU. spral impinges tangentially on, and spread.s over, the
sqlce thrguSh "r-u."^Jrrl;;ffi;;
passag;B;ffi, surface of a spherbal spue in the pi.ston. There is al-
-".lTil.r':n e*rictea
;iff il.lJlT;ff; ;:',*n*fi::, ]i,'.ili:H:
fluel reaching the main comb,,stion
ways some impingement of spray on the combustion
chamber walls in all successful diesel engine designs,
This impingement was not considered desirable till
rp"* " ;";i;
n^o detny.period as the temperat*u
is'areJy ;h il;; M.A.N. combustion system was experimented.
comousttorl rn pre_combustion chamber
and combustion Fig. 6. 13. Preonhntion mbe.
o The M.A.N. system's theory is that enough of spray
rapid and comptete (j.;. ir-.t will ignite bdore impingement so that dclay period will
T1d",:hi.Ieris
combustion)
g"
be normal while most of the fuel spray will evoporate
due to olent air motion. "r
Ad.aantagec : from the hemispherical combustion spare in pston prior
to cotnbustion. Thus the second. stage of combustion s
(i) Due to short or practically slowed down auoiding ercessive rate of pressure rise, Fig. 6.15. M.AN. M'combustion
no.delay period for the fuel entering the main
space, tendcncy to hnock is minimit4 ombustion Shouded inlet valve is used to give air swirl in direc- chambe.
urd, a sih ,rnning is smath.
() The combustion in the third tion of arrow,
stage is mpid.
(iij) As the ning of rel Adlantages :
and- airier.horougbdue to alent projecion
from pre-chamber, the fuet iaz\i!"i"- -"''"*a of combustion products
not be criticer. 'M-chamber' claims the following ad,uanta.ges :
Disadvantagec: \ (d) l,ow peak pressure.
(l) Th-e.velocity of buming mixture (ii) lw rate of pressure rise.
is too bigh during the passage from precharnbers,
heat ]oss is very high. This causes reduction so the
ii te ierniat t;, l,ow smoke level.
offset by increasiug the compsion ratio. *'"." "
"m.rt (iu) Ability to operat on a wiile rauge of liquid fuels (multi-fuel capability).
(ll) Cold stariug will
be difficult as the ai loses hea
to chamber walls during compreesion. Disadvontages:
(i) Low volumetric efficiency.
4. Energy cell:
(i) Since fuel vaporisation depends upon the surface tempereture ofthe combustion cham-
T\" ,erurgy
,. chamber. Itcell'is
,busfion morc conplex than the p:e.com_ ber, cold starting requires certain aide.
is illustrated ii fig. e.i.-t-A" oi._ (iii) At starting and idling.coutions hydrocarbon emissions may occur.
ton moves up on the compression ,t
forced_into th( mqior ,rrd "lq ""."d "fJr,i. i, Table 6.1 $ves comparison between open combustion chambers and divded combv.stion
-ioo" .1.-"", tl"
W" jL" tu_et is irdected tn"ooer, i. piii;;"r-
::]l nart ""l"cy chambers.
of the fuel p.r""" ...osslhe main ;;;J""
-z,le'
carnber and enters the miaor cell, where it i.
the entering air. Combuerion fi*r _irJ*itf,
;;;;;;;1"'"?m
combustion chamber where tl" turap"rai,riu
Fwlused Ca consume fuels of.good ignition Can consume fuels of poor ignition
t ""ii*
the rate oflurning is stower in tlfr-l*"i"i,'i'.i.?*r-
quality, i.a ofshorter ignition delay quality lager ignition delay.
i.. ,
'or higher cetane numbe. or lower cetane number.
ncrent_mixing of the fuel and air. The
burnng in t"ti_ Type of injectian Requires multiple hote injmtion It is able to use single hole injection
nor cell is slower at the start, but due t"
llt't"i J"g, Fig. 6. 14. Energy cell.
rczle used nozzles for proper irig offuel nozzles and mbderate injection

--l
I

, I
I

I
T
244
INTERNAL COMBUSTTON ENCTNES COMBUSTION IN C.I. ENGINES

ad ai, ad also higher injection-


presgures. It can tolemte greater . open chamber direct injection engnes are easiest to cold. start because of the follow'
pnesst[es. ng reasons :
degree of nozzle foulilg.
Senstivity to fuel Smsitive. (i) They have smalles surface to volume (S/V) ratio' as a consequence heat loss is
spray characteristic Insensitive.
minimum'
Miring (r) They bave lowest intensity of swirl, ilue to which stagnant gas film remains on the
of ftnl Mixing of fuel and air is not so
and, air Ability to use higher fueVair ratios cylinder walls which reduces heat transfer.
efficient and thus high fueVair
without smoke, due to proper
raos are not feasible without
mixing and consequent high a Cold starting aids for C.I. engines :
smoke.
utilizatim facto.
. Several methods hae been used in the past to achieve easy cold starting.9ew
ofthem are
D. Cylinder construction Cylinder construction is simple., listed below :
o- Starting More erpensive cylinder @nstmctioD.
Easy cold startilg- Difcul cold strting because of !. Preheating thc bngine cylnder by wartn woter.
geater heat loss through the 2.Injectionofasmall'quantityoftubricotingoilorfuetoil'Thismethodtemporarily
throat raises the compresbion ratio, and seals the piston rings anl valves'
Thernal efficieruy Open combustion chambers
are Divided combustion chambers suffer 3. Prouision of cartridges.
thernally nore efr cient.
from irreversibilitie ke throttling 4. Modifyins ualue timngs for starting. ..
throue the throat during the com- 5. Storting as Petrol engine by providing Tc.arburettor and a spark plug' At starting
pression and expansion; thus lead_ compression ratio is reduced by providif an auxiliary chamber'
ing to pressure losees and irvailable Modern starting aids ofhigh speed e+gines 3

heat losses. Therefore, these en-


gmes ar thermally less efficient The following basic three types of startips aids are used on nodern high speecl diesel
engines :

1. Electric glow plugs (in the combustion chamber)


a parricular combustion chamber
i:T::,,l"it R:r":::j*::
perrorm a given job. No one
combustio.'c"*i".
design rnusr " .noJl
cJr;o:ffffi;ffi'rol;:::::T:rj: 2. Manfold hears (which ignite a small feed of fuel)
tasks. As most engineering work, 3. Injection of ether.
the d".tg;
full consideatio"Jortrt"l?liortlng factors ;;; ;;;i
must be based on a compromise, after
'"r
,
(i) Heat lost to gombustion rrrGHLIGIrrS
(ii) Injection pressure.
chamber walls. I I

(ii) Nozzle des'6'n. The three phases ofC.I. engine combustion ae :


(i) Ignition delay Period (di) Period ofrapid or uncontrolled combustion
(lu) Maintenance.
(ii) Period of contolled combwtion.
(u) Ease ofstarting. phase ofcombustion'
The third phase is followed by after buming, which may be called the fourth
(uj) Fuel requirement. 2, The perind of physical delay is lhe time between the begiming of injection and attainment of chemical
(uii) Utilisation of air. reaction conditions.
(uirl) Wsg..lation of engine ln t}re chemical delay period. reaction stuts slowly md then accelerates until iflammation or igniiion
to power outDut. takes place.
(:) Capacity for variable the ignition process
speed operation. 3. Thedelay period is the time imerliately following injection of the fuel during which
due to comPression ofai'
is being nitiatetl md the pressure does notrise Ueyonil the value it wouldhave
pressure
6,7. COLD STARTING OF C.I. ENGINES 4 The delay periocl should be as short u possible since a long delay period gives a more rapid rise in
and thus causes knocking'
The important requiement ofe c.I. part ofthe
engine- is its easy starting from
cord. To furfil this I Diesetrc is the soud produced by the very rapid rate ofpressure rise durng the early
requirement frequent,y compression uncontolled second phase of combustion.
ratios higher thin necessary are
even so, may become difficult under used. cold work,
the folloi,ing.ooitiorr. , 6, Four specific designs which fmd wide use in C.I. engines are :
When the cylinder liner is heavily A Thenon-turbulenttlPe:
worn ;
When the valves are leaky (i) Open combustion chamber
;
Exteme cold climate (like Himalyan B. TheturbulenttyPe:
l
- region). (i) Turbulent chamber
Therefore, sometimes, it is necessary to prorzide some erectrical
aid for cold starting. (ii) Pre-combustion chamber
I
(iii) EnergYcell.

I
246 a^1
INTERNAL COMBUSTON ENCTNES COMBUSTION IN C.I. ENGINES

oB{rEgfIvE TYPE QT ESTIONS 12. Explain briefty the phenomenon of "Diesel knock
l& State the difference in the knockig pheuomena of S'I. and C'I' engines'
Fill in he blnks or Say Tes'or t{o': 14. Enlist various methods of controllig diesel knock'
l. The compression igaition engine was developedby ...... 15, what should be the prirnary mnsiderations in the desiSn of combustion chamben for c.I. engines ?
2. The thermal efliciency ofC.I. engine is ...... thm SJ. engines. 16. Explain briefly basic meods of generating air swirl C'I. engines combustion chmbers'
3. In C,'I engines the incompleta combretion of heterogeneous mixture, 1?. Enlist the advantagc and disadvantage of induction swirl'
md dmplet combustion ruult in
smoke and odou. 18, State tle advantages and disadvantag* ofcompression swirl.
4 The.periodof ""'delayist}letimebetwenthebegimingofinjectionmdattaimentofchmielreaction 19. Explain briefly any two of the following combustion chmbers :
condition. (i) Open or diect combuetion chambr (i) Turbulent dlamber
In the "" ' delay period, reaction starts slowly anil then accelerates unl inflammation or ignition takes (iii) Pre-combustion chamber (iu) Energycell
place.
6.
20, Give the comparison between open crmbustion chanbers and divided combustion dtambers.
The second stage of combution in c'I. elgines after the
deray period, is the period of ...... combusr,ion. 21. Write short lote on cold starting of C'I. engines.
The air-fuel mixture fomation, in te esel engine, is of
a..-.. mixtue. 22 E:rplain briefly old starting aids for C.I' mgines.
8. The delay period should be m ...... as possible.
2& Explain the phenomenon of knck in C'I' engiues and compare it with S'I' engine knoc'k'
9. Increase in intake temperattm would reult in inaease
increase the delay period.
in compressed air tenperatw whicl woutd 24. HowdoesthemixtuecmpositioninombustionmberofaC.I.enginedifferfromthatofaS.I.engine?
lo. Increase in compession rao reduces delay period.
25, ,,The factors that tend to increase detonation in S.I. engine tend to reduce knodring in C.I. eogine"'
ll. Discuse the above statement with deence to the following hlluencing factors :
At constmt speed, delay period is proportional to e delay mgle.
(i) Compression ratio; (i) Inlet temperature ;
12. Delay period decreases with inmase in aitvmce angle.
(ii) InletPresure; (iul Self-ignition temperature offuel ;
13. A pre-combustion chambergives shorter delay compared
to an open trpe of mmbustion chmber. (u) Time lag ofignition offuel ; (ui) r.p.m. ;
t4. ' " ' is the sound produced by the very rapid rate ofpressue rise
during the early part of the mcontmllecl (uii) Combustion chamber wall tenPerature
second phase of combustion.
15. Factos that tend to reduce detonation in 26. Why des rate of pressure rise duing combustion s linited to a certain value ?
S.I. engiae increme knocking in the C.I. engine. Explain how the
16. Induction swil results in easierstartiDg ofthe 27. Diecuss tJrc influence ofigaition delay on combustim processes in s.I. and c.I. engines.
C.I. engine. presence of a lmock hhibitor in fueI oil helps to change the ignition delay in c.L engines.
17. 'M-process'engine, dweloped ia 1954, was mmedbhiaper
than in a
engine'.
28. ;Tlre ,eqoi*-"ot of air motion and wirl in a C.I. engine ombution ctrarnber is more strigent
S.I. engine". Justi$ the statement.
the state-
25. "The induction swril in a C.L engine helps in increasing indicated thermal eftrcienqy''. Justify
nent.
l. Dr. RudolDesel Z higher & Yes 4 physiel 5. dmicl 80. How are c.I. engine conbustion cl,assifed ? what type of swirl is used in these chambers
?
6. ucontrolled ?. heterogeneous & short 9. No lO Yes 31. "In agriculture field, it is better to use c.I. engine tha:r s.I. engine". Justify the statement.
ll. Yes 12. No 1& Ye lL Dieselknock 16. Yes 8. Hm m a disel agine be mnvqted to CNG engine ?
16. Yes lZ. ys- by the cetane draractristics
33. The maximum ubstitution of cliesel engine by cNG in a c.I. engitre is limited
of the available fuel". Justifythestatement.
IEEONDTrcAL t4 Wte a short not on aids for starting C.I. engines uder extreme cold clinate'
to D.I. drambers'
86. Dessribe the M-combustion system anil discuss is relative merits with respect
l. When was C.L eng.ine develord anrl by whom ?
2. State the applicatiom fC.I. engines.
& Enlist the easons forwhich C.I. engine is not much favoured in passenger
cars.
4 Explain brielly the combustionpheaomenon in C.I. engine.
I
Desoibe briefly various prse3 of C-I. engine conbustion,
6. State the various factos which afrect combwtion h C.I. engine.
7. Explain brieffu diesel engine injected spray combustim poess.
8. What is delay period in C.I. engine.s ?
9. What is the difference between physical delay and chemical delay ?
I
.*-f
10. State the facton on which delay period depends. I
ll, Explain the effect of the following factors on delay period : 'l
(l) Fuel properties (ii) Intake temperatwe
(iii)compression at'o (p) Engine speed
(u) Type of combustion amber (ui) Injection advance.
AIR CAPACTTY OF FOUR STROKE ENGINES 249

o Tbe air capacity of an engine can be increased bXr :


(i) Better inlet-design ;
(ii) Increasiug r.p.m ;
(iii) Supercharging (i.e. increasing inlet pressure) ;
(iu) Cooling the inlet air.

7.2. IDEALAIR CAPACITY

!9:Oaeity of Four Stroke Engines The ideal air capacity corresp ond^s to filling the displaced uolume (i.e. pstoniwep uolume)
wth fresh mkture at inlet corld.itons,
7.1. Intoduction.?.2. Idealairooaifv 7? r^r.._-r:, p Thus, for four stroke engines, the ideal air capacityper cynder is given as :
Effecrorvariousractore
fl?j,,,,,,,,,,,,,,,1^"X-."1{;;a#;::i;?tTJ:J;';r",lLTitil.liT,TXl:';n
'IJPe
Questions-Theoreticat euestions-Unso-li
ri-"io|;*Ino,*-Highligts-objwtive
(o)i""r =I ,"r, ...(7.3)

U^
7.1. INITRODUCTION = -T A'Pt ...(7.4)
'.. V"= Ao (piston area) x .L, (piston length)
' I:;,i?::";{.:-1fi:'1"',',",ftffi,'ff fr?;;:gi:" h air throagh the ensine per and tlr=%']{ or N= 2L;
o The indicated power output
of an engine -
e -- may vv v^Pr
-'-r be expressd t: where
Indicated power, I.P. ,
Esscq as rollows
follows (rno )""= Ideal flow rate of fresh mixture per unit time,
= C) n*
rvherc, ? u, ...,r. r, N = Engine revolution per unit time,
- = Mss of f;i;""unit time.
C = Calorific value of fuel,
V = Engine piston swept volume,
and
P = Inlet gas density,
Indicated thermal efliciency.
b-,- ,_ ,.r1u,or = Up = Mean Piston velocity, and
tle.,ensrne depends on the Ap = Piston area'
"-.":;;;il":::,,llll,1?^1"#"',T:1,:1?'-iffi*
the amount oroxygen i:l:ll_{.rr Note. The fresh charge in C.I. engine consists ofair ozly (ilry air + water vapour) whereas in S.I. engine
*ufi.br"
supplied to the engine. The fi:el_air lg,,:;$;ilrT:ilij,','"T$:T.iy,i"f:i:1,ffi:i,";
atio is defined'as lle
it consists ofoirplus frel. lhe qumtity ofair taken in by an engine
fuel in the air md hence it is assumed that only a is present.)
is practically unaflected by the presorrce of
= eqn. (Z.l) then becomes
^r*".The
o rhe indicat"a tn""_"i"k,1":1,1f.,r1,"1*lXginu ...(7.2) VOLUMETRIC EFFICIENCY
0) Compression rauo.
a"p".,a, o., , 7.3.
The volumetric efciency of on engine is defined os the rdtia of actuol air capocty to the
(jl) Ignition timing. ideal air capacity. This is equal to the ratio of rnass of air which enters or s forced into the qlinder
Qi fU (Fuel_air) ratro. in suctdon strohe to the mass of free ar equivalent to the pston displacement at intahe temperature
'W and pressure conditions.
fixed conpression ratio and. optimum
it.h
ignition timng, \,, depends
only on F/A _
n,='
l'lvol. =
Mass ofcharge actuallyinduced
ratio. When FlAu""s ctosety Mass of charse represented by volume at
witt i" ii"
range of 0.025 and 0.0g5, efficiency
I intake tenrperturei and pressuire conditions
de-
creases as F/A increases,
makinj the =
q,n ;
o q"ot. =@
ll:-*"].Ir/' this' range; eventually
consrant over".9 u,l "oo.o"i^'" Dideal
f.i. tuEcttal
oecomes proportional /o mo.
power developed by the engin,
Thus the g
6 =^/-:
-
'-i-v.
fo". giurn .9 p,
valy3 of
portional,{/A atio and rn,r, will be-pro_ 2 flaclual
to the amount ?i. t","*
can take in per unit time, as illustrated -
Air capacity (kg airts) -___, N% Pt
in Fg. 7.1. 4 actu^l
Fig.7.l = ,.(7.5)
UeAepi
248
250
INTERNAL @I!{BUSTION ENCINES
AIR CAPACITY OF FOUR STROKE ENGINES 251
where, zaciual = Measured valve of inlet gas or dry air mass flow
rate. and
. p = Measured value ofinlet gas or dry air density. Multipoint injectors which add fuel at the intake ualue pors will have better efficiency
Thus, indicated power (I.p.) may now be written as : because the air is displaced until after the intake manifold. Fuel evaporation does not
occur until the flow is entering the cylinder at the itake valve.
l_P. = muaot(FlA x C) rlurrl
Those engncs that inject fuel directly into the cylinders after the intake valve is closed.
M
= v" o, n, (FlA x c) r1^* ...(2.6)
wll expericnce no uolumetric efficiencyloss due to fuel euaporation. Manifolds with late
; fuel addition may be designed to further increase volumetric efficiency by having large
Dividing At diarneter runners. High velocity and turbulence to promote evaporation are not need,ed,.
V*we get indicated mean effective pressure (i.m.e.p.) given by
n They can also be operated cooler, which esults in a dense inlet air flow)
i.m.e.p. = p I,. (F/A x C) lo Fuels like alcohol wlrich have a srnaller air-fuel ratio will experience a gregter loss in
ro ...(7 .7)
uolumetric efftcienca. Fuels with high heat of vaporisation will regain some of this lost
From eqn' (7.?), we find that if the type of fiel, fuel-air ratio (F/A) and indicated
^- .
efficiency (r,n,n)remain constnt, then
thermal efliciency due to the greater evaporation cooling that will occur with these fuels. This
cooling will create a denser air-fuel flow for a given pressure, allowing for more air to
r.m.e.p. E p rlvol.
Power output of an engine is proportional to volumetric efficiency provided enter the system. Alcohol has high heat ofvaporisation, so some efficiency lost due to
-^*-, I"*'
complete. the combwtion is air-fuel is gained back again.
Gaseous fuels like hydrogen and methane displace more irrcoming air than liquid fuels,
7.4. EFFECT OF VARIOUS FACTORS ONVOLUMETRIC
EFFICIENCY
\ which are only partially evaporated at the intake system. This must be considered when
trying to modift engines made for gasoline fuel to operate on these gaseous fuels. It can
It is desirable to have maximum volumetric efliciency in the intake of any engine it vares be assumed that fuel vapour pressure. in the intake system is between 1 to 10 percent of
;
-it! "ls_rry speed. Fig. ?.2 shows a graph between volumtric efficiency antr enge speed for a total pressure when gasoline-type liquid fuel is being used. Wben gaseous fuels or alco-
typical S.L engine.
hol is being used, the fuel vapour pressure is ofr,en greoter than 10 percent of the total.
Intahe manifolds can be operated much cooler when goseous fuel is used, as no
s vapourisatnn s required.. This will gain back sorne lost volumetric efliciency.
v o The loter that fuel voporises in the intake slstem, the better is the uolunetric efficiency.
J On the other hand, the earlier that fuel uaporses, the better are the miring process and
34 cylinder -to-cylind.er d,istribution consistency.
.9
ol
1

Ef 2. IJedt transfenlllgh temperaure :


3l o All intake systeni ae hotter than the surrounding air ternperature and will consequently
o
E
J
heat the incoming air. This lowers the density of the ar, whch red,uces uolumetrc
I efficiency.
o Intake maifolils of carburetted systems or throttle body injection systems are pur.
lq)o 2000 3000 t000 5000
posely heated to enhance finl euoporatio. At lower engine speeds, he air flow rate is
Englne speed, N (r.p.m) --------f
slower anil the air remins in the intake system for a longer tine. It thus gets heated to
Fig.7.2 higher temperatures at low speeds, whichlouers the volumetric efficiency curue inFig. 7 .2
at the low-speed end.
There is a certain speed at which the volumetric efficiency is namum, decreas-
ing at both higher and lower speeds. firere are several physical and operating vari- o Some systems have been trie d which inject small amounts of water into the ntake man-
ables that shape this cuwe. fold. This is to improve the uolumetric effcency by increasing the resultittg euaporatiue
Effects of variaus fadars which affect the uotumetric efftcicnry are discussed berow : cooling that occurs.

l. Fuel: 3. Valve overlap:


The vo{umetric efficiency ofa naturally aspirated engine will always be less than 100%
At the top dead centre (T.D.C.) at the end of exhaust stroke and the beginning of the
intake stroke, both intake and exhaustvalves are open simultaneously for a briefmoment.
becaus fuel is also being added and the volume of the
fuel uapour uill d.spla.ce some When this happ ers, some er,hault gas can get pushed through the open intahe ualve bach
incoming air- The type of fuel and. how and when t is added will determirc how much the
into the intoke sy*m. Tlhe e*,haust then gets carried back into the cylind,er with the
uolumetric effici*nqt is affected.
ntake air-fuel charge, d,isplacing some of the ncoming air and. Iowering volumetrc
o systems wihhcoburettors or throttle bod.y injection add
fuel early inlheintake flow and efficency, tlois problern is greatest at low engine speeds, when real time of ualue overlap
generally have lower ouerall volumetric is greater. This effects lowers fficiency curue in Fig. 7 .2 at the low engine speed end.
fficienqr. This is because the fuel will immedi-
ately start to evaporate and fuel vapour will displace incoming air. Other factors that affect the above Droblem arc the intahe and haust valve location
and, compression ratio.
252
INTERNAL COMBUSTION ENGTNES
AIR CAPACTTY OF FOUR STROKE ENGINES
4.Fluid friction losses:
. When air moves t\9uSh any llow passage or past any flow restriction, I The position where the intake valve closes on nost engine is controlle.d by a crankshaft
pressured'rop.Forthisreason,thepressureorairenteringth".i"-"*ir*"it undergoes a ,and.cannot change wth engine speed. Thus the closing cycle position is designed for one
sunounding atmospheric air pressure, and the amount thanthe engine speed, depending on the use for which the engine is designed.
,ri' .rr "i".1"g the cyrinder is
subsequently red,uced.. The flow frictio that affects tl" ai. a, ii p""ses through 7. Exhaust residual:
' -uisco-us
the air filter, carburetor, throttle plate, intake -""ii";"d i"t";;i"" red,uces
the A1l ofthe exhaus gases, during the exhaust stroke, do not get pushed out ofthe cylin-
volumetric effciency, viscous d'rag which causes thz pressure
loss increas* with square der, by the piston, a small resdual being trapped n the clearonce uolume. The amount
of flow uerocitv. This esults in decreasing ttre emcienry
on ;h" ;;"d end of the of this residual depends on btle compression ratio, and sonewhat on the\ocation of the
curve in Fig. 7.2.
value ond ualue overlap.
A lot of development work has ben carried out to reduce pressure The exhaust gas lesidual, besides displacing some air, interacts with the air in.iwo
losses in air
intake systems: smooth wals in the intake manifol, other ways. When the uery hat gas mses with the incoming air it hea'ts the ar, lowers the
trt" of sharp cor_
ners "*ia..*"
and bends erimiaatibn of tbe carburettor, and close-fitting
fa*s alignment gas density, ond d,ecreases volunetric efficiency. This is counera'cted slightl!, however,
with no gasket protrusions all contribute to decreasing iot t p?e-"ior" by the partial udcu.utr created in the clearonce uolume when the e*haust gas is in turn
torr.
one of the gredtest flou) restriction is the " cooled by the incoming air.
flaw through the intale ualve. To red,uce
ths restricton, the intake uarae
flow area ias be"" i;*;;;;;;;irng Exhaust gas recycle (EGR) :
engines hauing t no or euen three intahe values per qilinde, " - - ''-- muhruabe 8.
, In several types ofengines and all modern engines, some uhaust gas is recycled, (EGR)
o The flow ofair-fuel into the clinders is usuolly iliverted
into a rotational flow pattern
within the cylin der-to e.nhance evaporation, *ixing into the ntake system to dIute the incoming air, This reduces combustion temperatures
ac:complished. by shaping intake runners and, "nd.
ftaie";;;;;t;"* pattern is in the engine, which results in less nitrogen odes in the ethaust. Upto abott 20 percent
contouring the surface ol the uarue" and, of exhaust gases will be diverted back into the intake manifold, depending on how the
ualue ports' This increases the inlet flow restrction oo"nrror""ia"h"i*
efficiency. engine is being operated. This erhaust gas not only d.isplace some incoming air, but it
I In case the diameter ofthe intake manifold runners is increased,
flow velociy will be also hecits the incoming air and, lowers its density. Due to both of these interactions the
decreased and pressure losses wilr be decreased. However, uolumetrc efficency of the engine is lowered.
a decrease in vetocity wilr
result in poorer mixing ofthe ai and fuel and less u...r"ut" In addition, eng\e crankcases are vented into the intake systems, displacing some of
.vli"a".-t"-.ili"der distri-
bution. This need,s proper compromises in d,esryn, the incoming air ari lowering the volumetric effrciency. Goses forced through. the crank'
o In order to get better air-fuel mixing in some low performance,
high fuer_eflicient en- case con drnount to about 1 percent of the total gas flow through the engne.
gines, the wails of the intake manifold are made
rough n In these 9. Piston speed and engine size:
engines, high volumetric efficiency is not as important. ""h";r;;"r;;l;ce.
Itcan be proved that inertia stress * Lr.2. This indicates that all geometrically similar
5. Choked flow: engines reach maximum allowable inerfia stresses at the same piston speed. Therefore,
r when choked flow occurs a some location in the intake system, it is
the extreme case of
whether due to consideration of rnaxinum air capacity or limitng inertia dresses, geo-
flow resriction. When the air flow is increased to higher metrically similar engines are generally designed to run ot the sarne speed'.
velocitie", it reaches
sonic velocibr a some point in the system. Tltis choked. "r,"rrto.lly a The power d,eueloped by an engine is not proportional to swept I dsplacement uolume.
lo* ,onaltlonl"'ine ma.ximum
flow rate that can be produced in the intahe sxstern regardless of how controllng a Large engnes deuelop less power per unit uolume.
tions are changed. This causes lowering of th efficien,ry cond-
curve th" ;;gl-*p"ed end in 10. Design of inlet and exhaust systems :
Fig.7.2. ";
The occurrence of choked flow takes prace in the inost restricted The volumetric efliciency is affected by the inlet and exhaust pipe design (length and
passage of the system, diameter) ; the effect ofinlet pipe system being greater.
usually at the intake valve or in the carbureti,or thoat
on those engines with carburettors.
By experimentation it has been found that, at certain speeds long inlet pipes give high
6. Intake valve closure after B.D.C. : volumetric efficiency.
r The amount of air-that ends up in the cyliniler is affected by the timing of
the closure of 7.5. INLETVALVE MACH INDEX
the intake valve- The idear tme for the intake uarve to closi
is *nm inz p)"rrure equari-
sation occurs betueen air inside the cylind.er and, the .air n n, *.""iila.If it croses The flow of intake charge, in a reciprocating engine, takes place through the intake valve
-the
before this point' air that was stiu enterig the cylindzr is stopped. and, a loss of
volumet- operring which varies during suction/induction operation. The maximum gas velocity (Q) through
ric efficency is etperienced'. If the uolue is ctosed,-after th" p"t"t, this area is limited by the iocal sonic velocity (U,). The following relation is used to choo3e the gas
piston will force some air bach out of the cytind.ei, again
tri"[ io pr"rr"d, by
with a"i,toi, ,ui.u^"tr" ,rn_ velocity :

ciency.Tbis valve-closing point in the engine cycle, at which


the p."..r"u inside the
cylinder is the same as the pressufe in th intaie manifold,
is rrigrirv " d on en_ u-=A'uo
t
gine speed. K Au
AIR CAPACITY OF FOUR STROKE ENGINES
INTERNAL COMBUSTION ENCNES

where Ap = Piston aea, Soluion. Giuen 20.3 cm = 0.203 m ; L = 30.5 cm =0.305 m ; N= 300r.p.m.
A, = Nominal intake valve opening area, t\'o.=78Vo'NF ratio = 4: 1
a.nc!
4 = Intake valve flow coefcient, Volume of gas used per minute :
lt= AoUo
and Stoke volume =L x 0.2032 x 0.305 = 0.0098?1 m3
u, 4u KiU"
4
Volume inhaled = Iuol. x stroke volume
( o^.\' ti
=ll x F -
=7. = 0.?8 x 0.0098?1 = 0.00?699 m3
\D" ) Ku' .. 0.007699
where Dry = cylinder diameter, Gas inhaled = ffi = 0.00154 m3
D, = Inlet valve diameter,
U, = Mean piston speed, ms/mtn. (Ans.)
Gas inhaled per minute 0.00154, = 0.281
U = Inlet sonic velocity, and =
#
Z = Inlet valve mach idex. Example 7 .2, A four-stroke, eight-cytinder engine is testd while running at 3600 r.p.m. The
inlet oir tumjerature is 15"C and. the pressure is 760 mm of Hg. The totol piston displorement uol'
. -Fiq' 7'3 shows a plot for e-" us z (unaltered by varying inlet valve dimeter, valve lift and
valve design), ume is 4066 cn|. The air-l'uel ratio of the engine s 14 : 7 andb.s.f.c. is 0.38 kelkwh. Dynometer
from which it can concluded that thre is a particula value of Mch
Index after rea.ding shows a power output of 86 hW. Find the volumetric effwicncX of the enginz'
which volumetric efficienry starts falling; this ts appioJrr ately
Z = 0.55, or 288K;
Solution. Given: N=3600r.p.m.;Inlettemp. ?= 15"C
0.9 p = 760 mm Hg = 1.013 bar
0.875 y = 4066 cm3 or 4066 x 10r m3 ; A./F ratio = 14 : 1 ;
l\t 6 b.s.f.c. = 0.38 kgftWh
B.P. = 86 kW
Volumetric efficiencY, r,ot. :
86x0.38x14
+ Air consumption, - =ffi =7.625kg1in

I o.z Also, pV = mRT


v _ mRT _7.625x287 x288 =6'22mslmin
"= o =
q
E
.9
a r-ore;lor
o 3600
0-6 Displacement or swept volume = 4066 x 1015 x
2
@
E
f = ?.319 m3/min
I n . = 6:222 = 0.85 or 857o. (AnsJ
?.319
Example 7.3. The oirflow to a four-cylind,er, four-strofu oil engine is measured. b! a 5 cm
d,ianeter oriirce hauing a coeffi.cient of disclwrge of 0.6. The engine lnuing bore 10 cm an stroke
12 cm runs at 1200 r.p.m. Pressure drop across orfice is 4.6 cm of woter and, ambient temperature
and, pressure ore t7"b and 7 bar respeitively. cal.culdte the oolumetri fficiency based on free air
0.4 0.5 0,6 0.7 0.8 0.9 1.0 1.1 cotd.ition.
Inlst vahn Mach Indx l---+ cm = 0.12 m
Solutibn, Giuen : n = 4; d, = 5 cm = 0.05 m; C, = 0.6, D = 10 crn = 0.1 m ; L = 12 ;

Fig.7.3 .t h, = 4.6 cm = 0.046 m ; ? = 1? * 2{g = 290 K ; p = 1 bar.


iY = 1200 r.p.in
t'lrol 3

WONTED A&|'MPLES
ra=
o- l==,287x290
RT
1119",= = 1.2015ke/mg
7.1. A single'exlindcr, 4-strohc c1rcre engine using cNG (compressed. natural gas)
^ - -Examnfe Head causing flow, metre of air,
as fuel has a cylinder, 20.3 un bore x 30.5 cm stroh and )uns at 300 rpm. Ifthe volurnetric efficiency
of the engine based' on conditians approaching the cylindcr s 78Vo nd -
n"=
h... o... 0.046 x 1000 88.285 m
ihe air / uel ratio is 4 : 1, -; = L2015 =
d,etermine the uolume of gas used per minute,

I
NTERNAL COMBUSNON
ENCTNES ArR cApActry oF FouR srRoKB ENcrNEs 25'l
Air velocity = JUE =1Ft8smB =27.407 ns Displacement volume/min. = Displacement volumdstroke x No. of cycles per rrin. '
%ctua = Co x areax velocity
fO{- x
450
= 0.6 xI = 7000 x ;- = 1.575 m3/min
x 0.0F x Zrl1O7 =0.0328 ns/s
Volume of charge displaced
y*"pt
= rr x x D2x.L x no. ofcycledsec. I'or = -Giiifii"-
!
' nx o.1z x 0.12 , fuoo\
n,
1.195..
=Tl '100 = zrs796. 15".
=
lfffrJ = o.o3z? ma/s
E*ample 7.5, A si,x-ryIinder four.stroke S,L enghc hung a piston d.isplacement of 730 cma
)

"' rl"t =
e=ffi or
=0'856? 85.677a (AnsJ
per cylinder deueloped' 80 kW at 3700 r.p.m. and. consuncd 28 hg ofpetrot per hour, The calorific
ualue of petrol s 44 MJlkg. Determine :
Example 7'4' A sinor. cylind'er, (i) The volumetric efflci.enry ofthe engirc ifair-fuel ratio is 13 and,the intahe air is at 0.88 bar,
4'strohz rr* *y-:yne with a-disptacement uorume
'"?:::::"i",'":'::;;l !^Y:;":::t"; ;;;;";;;trfr^ or0.i22 hsl hwh rhe consumption of 27'C;
conitions;fi.dt;;;::#'ry^'" crH,u' If 30 perce"t'*i"'"'"1"-i',if"i"t",**ruejt
intet
ot
(i) The brake thermal fficiency ;
(i) The air-fiel ratio (i Th brake torque.
; Gil The wlumctrc efficency of the engine. Solution. Giue.'Number of clinders = 6 ;
air, take Eo,, = 0.282 kJ I kg K.
lor Piston splacement per cylinder
Sotution. Guii, n=t t=r.i_-r_^r.^__^r-\ r, - G_IU.I.E., I.C.Eng:ines) = ?30 cml = ?30 x 10-6 mB
Power produced per cylinder, B.P. = 80 kW a 8100 r.p.m.

...Inlet
"l',=,Tii:r.:=?j;lHtr#k,",,r=:iijt,"=,n,o*,
conditions : p = 1.018 bar; ? g0
*;;-= ,308 K; Rn 287
=
Petrol consumed per hour =28kc
(d) The air_fuel , = JlkgK. Calorific value of petrol, C = alMJlkg
".ioequation
Chemical combustion Air-fuelratio \ =13
for giveu fuel CrIIr. is given
CzHr. by the expression, Intake air conditions : 0.88 bar, 27G
* = ,lcor' "* gII2o
12x7+lx16 --tt_O, () Voluetric efflciency, r*r.:
llx2x16
= 100
Mass
=352
ofoxygen g52
n.'=#ffiH= p"xv"xTN ffa

-r,f;;"f tu"r =
iod' = a.z
Air-fuel ratio Mass of oxysen/O.2g p_ = + = _*9gl!=
ra = r.oz2kgtm'
= *ss{=
Mass of fuel
- RT 287x(27+273\
M"ss of fu"l--
(28x 13)/60
_ Mass of o:gBen
.- I g.52 Tlvol =
31oo = 0.874 or 87.4Vo. (Atts.l
MassofE"l- " 15 =
o:5 = r5'3' (1"s.) 1.022 x (?30 x 6 x 1o{)'2
x
(ii) The volumetric ('-' Air contains oxygen2Svo by weight)
effiicir
:ncy-of the engine, q"* :
(ii) The brake hermal efficiency, r*,r, :
m/ = s.t.c. x power develooed
-,tt.lr1.B, __ B.P.
th" xC
= 0.272 x fn.Z = a.999-Oa,
mo = LS.B x 8.999 61.185
= kg/h 80 x 103
airsupplied
Actual =
(rsft600, r (44 r r05 = 0'234 or 23'4vo' (Ane')
=61.185+61.185,
Mass of charge t # = 79.54kgth (iii) The brake torque, T:
tn = Mass of actual ai" iulass
orru"l
= 79.54 + 3.999 = 89.54 kgh u.". = -?Illr- kW, where ?is in Nm, and Nis in r.p.m.
volume of charge sucked 2x x 3I0D xT
60 x 1000
==,rHr_**
I0l3 x l0 = 1.1e5 ms/min
t= 9*%# =248'4Nm' (Ans')
258
INTERNAL COMBUSTION ENGINES AIR CAPACITY OF FOUR STROKE ENCINES
Bomple 7.6. The wlumctric-effuiznc1 of a ptml engine at furl load s g0 percent, atntos- For change in inlet temPeroture:
pheri.c cond'itions being 1.013 bar and l|'C,
pheric pressure. The compression rotia ofthe ""
"t i"i on,,ust piessures ore ,q n the atmas-
engine is ?.i. Ifthe inlet temperaturi is raised, to 45.C
and exhaust pressure is raised. to_J.'I5 bar, dteimiw :
nu"=iltrtr ={3oo
ffu =1'054
(i) The uolumetric efrcidiq
;
(ii) The percentage cnge in indicated. output .'. ", pressure and ternperature'
Fo both change in inlet
of the engine.
Soluion. Giue4, i rlp, = 80% =0.g ; p. 1.019 ba p. 1.15 Tt= 25+273-298K;
b =r.otrux 1.054=t.o6?
= ; = rbr
4" = 45+ 273 = 318K ; r = ?.5. .'. in volumetrc fficbncy = 6'7%' (Ans')
The percentage increase

(j) The volumetric efficienc Iu, : (ii) The percentage change in indicated ouput ofthe engine :
OutPut*Px4,
For pressure change : 1.013x 106
rt,, p)!,1 _ ?.5_(Ll5/totg)vl
r= -!L-
.-
an 287x3oo
-,.r*
r,, =r-(p"t
r -(p"l p)ir 2.5-(Lotg/Lolgf/t{ -=-
Dt l3x 105
= 1'36 kg/mg
_ z.sjo948 'z Rn 28? x ggg
= 0.e854 Pz _Pzx\v, __1.3X , = t.284
For inlet temperature change : Pr pr x rlu, L176 '.OOZ

r,,=t/{
tr =t/2e8
I5r8
or Percentage increase in power = 23.'17". Gn&)
Example 7 '8, A petrot engine operatng 4t ftril throttte dcvelops 32 hW with
80 percent me'
= 1'033 if"r" it^otpliri, bor pressure ond' 35'C tem'
',,, chanical efficienel dt ," l"i on{ons are 7.013
is 5"c. A
Now volumetric.efficiency, considering both pressure and temperature, peroture. The engir* x mouid n a hill station wlwse altitud is 2000 m dnd temperoture
4,r = 0.8 x 0.98b4 x 1.038 = O.8143 or Bf.%. (Ans,)
;;;;;;;; ;; ;l;;;;;; l*"meter readins mav be assumzd for euh 100 m of rse in atttude'
Determinethepercentdgechdngeinvotumetriceffrcizncloft|uengincandthebrahcpowerof
(li) The percentage change in indicated
output ofthe engine : the engineif t runs at the satrv speed and, full throttle'
= 35 + 273 = 308K;
Output * (p, x r,) or
n-. -
for constant inlet pressure
Solution. Giuen: (.P.)t=32kW i4-*,.=8096 iP1 = 1'013bar;?,
Ii Tz= 6 +273 =278I{
,'. Percentage red,uctinn in output Percentage change in volumetric effrclency:
1. Therefore t,bere is no change in volumetric efficiency due
(0.80/298)-(0.814/318) The ratio ofp"/p, in both cases is
_ 0.814x298
to inlet and exhaust Pressure change.
(0.80/298) 0.80 x 818
For inlet temPerature change'
0.0465. or 4.66%. (AN.)
Example 7'7' A 4'stroke d'ieset engne has a compression rato of 14 and. works
condition of I'013 bar and 27'C. A supercharger s
in ambient Y=E=.1@=o,ru
sure to 7'3 bar and th inlet ternperature to 60'C,
ae to the engine which rases tc inlet pres- 1,, 14, Y 308
other cod,itions iemaining the same. Determiw :
(i) The percentage change in chargry
fficicnq ; ;. Percentage d.ecreasern n, = L*E x 100 = 67o' (Ans')
. (ii) The percentage change in ind,icated. output ofthe engne.
Brake power of the engine, (B.P.)r:
Solution. Given : r = 14 ; p, = 1.013 bar ; p, 1.8 bar
= i \ = 27 + 278 = gOO K .
pi or Y
Tz= 66.+ 273 = 3gB K Indicated power output (I'P.) - rt, x
(i) The percentage change in charging efiiciency
:
For change in pressure (r.p.),* +#t-q#9-
T, 308
rru, xo.ooa28e

\,, r -(p" / p){t _ 14_(1.019/ls)vrl _ 14_0.8gT Drop in pressure at hill station = pg N/m2
=
n;-;-6/ e)t = 14-(1rts/l.0fiPF = 14-1 =r'or25 10-5bar=026?bar
= (13.6 x 1ffi0) x e.81 " [#"#) "

I
INTERNAL COMBUSTTON AIR CAPACITY OF FOIJR STROKE ENGINBS
ENGINES
p2 = 1.013
- 0.262 - 9.745 5"r.
or !u. x P"'
(LP.), e. rq"P' o'95 fu, x o'?46
rh *
278 * o oo2549 4ur
(I.P.), = (I.P.),
' x p'xP"'
(LP.L o'ryn
0.002&lfn. = ru' 1111t = 102 kw. (Arc.)
aEf =oE- =o'zzs
0.81x 1173
Example 7.L0. A I'cylindzr, 4-stroke cltclc engirc
- a test th.e gas consumption was netered, o.2s mllmir at 120 mm of water and 27.c.
In
los a bore of 320 mm and. strohe 380 mm.
(r.p.)s Th ar
= (r.p.), x q.775 = (#) ,o.rru klv consumption was 3,36 fulmin at atmosplwric pressure of r.0rg bar and temperofure 27"c. The
= Bl
speed' of the engine was 280 r.p.m. Th catorfr value of gas used, was 1g600 hJ l;rs at 25oc and.
:"'ff;::";:ffir:to*u ts same at two pl", ui" fricrion power aud hence mechanicar 1.013 bar. Calculate :
",".", efr- (i) The uolumetric fficieney of the engine ;
(B.P.),
= 31 x .8 = g4g kW. (Ane.) (i) The heating value of 7 m3 of the ch.oge at Z.C and. 1.013 bar.
EI:-OI" 7,9. A petrot ensi
w'cn arnbtent conditions ,f ,f Z.! Solution. Giuen : n = t ; D = 820 mm = 0.82 m ; L = Bg0 mm = 0.3g m ; N = 280 r.p.m.
ltri #h^:r*ry::::_:-^
i:i;;::::":::"';:;;";!:tril,::#;hzr{ff:;if":::,#Hy;::^r,y##:i
t:3j!:?:",::".1.r8 b";;"d"t-;;;;X;t;:ry**rger
d.euetops Z5 hW ind.icated,
is added wnth i,rc,easeilii C.V. = 19699 ktm3 a 2b"C and 1.013 ba
;

votumetriceffi c.::.r*::;"y:';;;;trff
uolumetric efficiency
;f;r**:yf*:J!:y:;*
of thi eryuu is BIlo uitlnnt ou-*ry-T::h:'yit: id.entical contio*. I;i; (i) Volumetric efficlencyr
4;, :
(.i) The uolumetric Volume of air consuned at inlet contion,
ffiicnq;
(ii) Indicated, power of mRT 336x287x(27+273)
the iuperchatged erqine.
Solution. Given: r ?-5: lp _ ?K Lur. tF
u=-o = . ff'rl0r--=2.8558mVmin
= ; 14 = 27 + 273 =300 K; pi = 1,01 bar ; p (l2ql-1-000)
1.B8bar, r," = 4s * rrr Gas supply pressure = 1.018 +
=rrl"',';=:Jrt-:" = 1.02b ba
= (1 bdr = 10.2 m)
(j) Ttre volumetric t0.2
efciency of the super.charged Gas consumption at inlet condition
For change in inlet pressure
engine rlv :
x l'os
tt_r-(p"l ntr = 0.25 = 0.268 m3/min
1.013
I,, r-1p"1 p)!r =##+##=#=roa .'. Volume of mixture consumed in inlet condition
For change in inlet tenperature = 2.8558 + 0.253 = 3.109 m3/min
:

u=E-m:--
n; -fd =1too= = r'osaa
'l"or =
3.109
Swept volume

For both change in inlet pressure =0,727 or 72.7%. (Ane.l


and temperature, x (038) x(280/ 2)
tlu,
f,xQ.s2)2
(i)
The heating value of 1 n3 of the charge :
= 1.0s x l.03rr4 = r.06&f
The heating value of I m3 ofthe charge at 26'C and 1.018 bar
tl,, = lq x 1.0654=0.g1 x
(ii) Indicated power
1.0654=0,g629
of the eupercharged englne, (Lp.)2
:
or .2Wo. (And.\ =
ffiffi x 1860o =
ffi ,. 18600 = r61s.6 kJ/ms. (Ana.)
cExemple 7.11. A 6-cylindcr, 4-strohn petrol engine with a bore of 125 mtn and. a strohe of
Pa'==## =r'r73lg/mg 190 mtn was supplied, d,uring a test with petrol of composition C gZVo and. Ho lgVo by mass. The
= =
d.ry ex.haust compositian by uolume .^ CO,
= 11.19(, Oz = 3.6III and, N" = S.2go.
p""=fr=## =r.4e8ks/mg
(i) Determine the nass of air supped per kg of petrol, the percentage of ercess oir and the
volume of the mirture per kg of petrol at I7'C;nd 0,bg bar, whch re the coldiiions
for the mi*ture
entering the cylind.er d,urng the test.
(I.P.), _ tu, xp.,
(ii) Nso d,etermine the aolumetr efficienq of the engine based on intake cond.tions
(I.P.)t lo, xpo, mass
when
petrol used per hour during the test was sI hg and,-the engine speed, was 1600 r.p.rn. The
_of
petrol is conTpletely euaporated before entering the cltinder and thi effec of its uolurne on the volu-
metric efficiency should be includ,ed.
INTERNAL COMBUSTON ENGINES AIR CAPACITY OF FOUR STROKE ENGINES
Asume the following :
. Den sity of petrol uapour as J..s5 times
of ar at 0"C and l.0IJ2 bar occupies 0.72J4
that of air at the samz temperature and pressure,
- 1 kg Volume of1 kg ofpetrolvaPour =ry
mr. Ai, iiar,
orygin b, ;;;r:
2JVo
(229 + 17)
n = 6 ; d' = 125 mm ; 1e0 mm c
= ; = 827o i Hz = 78vo ; co, = 11.1s6 _ 1x 85.62 x
= r.3l\g'.;;..' .
x
0.98 10"
= 0.2585 m3
", (i) Theoretical mass ofair required per
kg offuel for complete combustion
lo,n (a \ Volume of 18.835 kg of air =
+
= zs lic*eH,.
l&8!5 x 28? x 290
t# = , millor = 15.996 m3

o |,l.
o.* + 8 x o.r8') = 15.268 ks.
\ t .'. Volume of mixture per kg of petrol
VoL of constituents = 0.2535 + 15.996 = 16.25 m3. (Ans.)
per nole of Mus of carbon (ii) Now, uolumetric fficiency
per hg
dry flue gases Volume of air per min at intake condition
(a)
offlue gu
Swept volume Per min

CO, = 11.19 -ffis.p5'ffi$


16.25x(31/60)

oz = 3.6L = 0.?54 o" 769o. (Ans.)


Nz = 85.20 Example 7.12. On testing a sparh ignitbn engine it was observed' that the uolumetric effi-
ciency is *oi^u^ when inlet ualve Mach Indt is 0.55 and. the ndicated' torque, and ind'iated.
mean effectue pressure occured' at mazimum volumetric fficienc1.
The engine hauing a bore of 1 10 mm st strohe 740 mm produces marimum indicoted torque
Mass of dry flue gases/kg ot*", - Mass of carbon per kg of fuel when running at 2400 r.P'm'
M""" of
"rttoo
p"" klof flGfr (i) Determne the nominal d'ameter of the inlet uolue.
0.82 (ii) If the sone engine is required. to &velop ma.ximum ndicated. power at 2800 r.p.m., haw
= 18.28kg. wilt the inlet ualve size be modified ?
0044858 =
(i.i) If the sdn engine runs at 2800 r.p.m. ut wul ony inJet valve modifuotinns, how will
Mass of unused air per kg of tuef uolwnetric efficiency get affected ?
= tra.Zf x 0.03859) = 8.062 kc.
S Pressure at intake ualve = 0.88 bar ; Temperatue at intake ualue = 340 K; Intet value flow
.'. Air suppliet<g of ftrel = tS.Zee;ehn kc. (Ans.)
= 18.88b coefficent = 0.33.
.i '
Excessarr= 3'067
x l0O = 19.464q (Ans.) Assune : Fuel-air miature as perfect gas with ^ = 1.4 and R = 287 J I kg K
(iu) what would be uolumetric efficency at tnasimun power speed of 4800 r.p.m., for un-
Now pV = nRT
- mod.ified engine.
Solution. Giuen : Z = 0.55 ; D- = 110 mm = 0.11 m, -L = 140 mm = 0.14 m N = 2400 r'p'm';
For air I _ pY _ (1.0132x 105)x0.ZZg4 i 1.4' R = 287 ilkgK.
;

mT Lx2tB p = 0.88 bar, ?= 340 K ; l(, = 0.33 ; 7


() Nominal diameter of inlet valve' D" :
At the same temperature *u n";r:::"tt" For the properties of mixture given in the data, the local sonic velocity ofmixture ofair-fuel
\' at the inlet or suction valve is given by :
ppetrcl
wpou
= 3.35 = _ *__ u"= ,llffi = JfAxrf = 36e.6 m/s
Pair fipetrol vapour

287 u, uo =z
:' Rpetrol
v4ou =
335 =
85.67 Also,
u"=(oto\'"
\Da) KU,
...tEqn.(2.9)l

At 17'C and 0.98 ba,


Ur=nistonspeedl
(*)'"%ff=" l',
L =2LN/60 I li,
litl
i.J
lii!
ii l
',
lri
264
INTERNAL COIBUSTTON ENCTNES
AIR CAPACITY OF FOUR STROKE ENGNES 265

[o.rr)' ftz oJ1x2.4oot 6


lal "L-'
., The volumetric efciency of a.n engine ia ilefiaed as the rao of actual air capacity to the ideal air
=o'* capacity. this is equal to the ratio of mass of air which enters ur is forced into the cylinder in suction
stroke to the mass of fiee ai equivalent to pistoo dieplacement at intake temperatun end ressure
2 ontiom.
,, -
-."
_ | (o.rr)2 x(Zx0J4x24(Nt6O\Y
xt2x0J4x24ut t 6o)
h .or 44.95 . 1. Power output of an engine ia proportional lo volunetric efficiency pmided the ombution is omplete.

038 x 86ffx O55-J
-----iTll-lll;-:----l-::- = 0.04495 (An.)
6. ltre volumetric efficiency of an engi.oe is affected by many variables su as compression ratio, valve
timing, induction md port design, mixture streng, latent heat of evaporam of firel heating of the
J::"-".t:-l?:lhl:'"'*":"5d*gtuFig'?'BiaBT.Eroapproximaterv. induced drarge, cylinder temperature md atmospheric contions,
"1'lf
to be mo-imum at 2800 r.p.m., Z
indicated m"'"
"tr;"t;:';;;;"ll."lfiJl'i;" ef- \
","""r[i'#i,"#,"'l'"lTffi':1fquo' = 0.85.

| (0.11)2 (2 x 01. oBJECTTVE frPE QLIESTIONS



"tu- |
/.=I-
\v.4l x^ \ ^ vr*4 x
^ 6uul t bu,
2AO0 60) I
033"g69s-oj5 ]*
| =0.0sss or. 48.66--. (Ans.)
Fill in tbe Blarks or Say {ee' or fio' I
but renai consr," ez.e6 ar 2800 r.p.m., rhe inler valve 1. At lower speeds volmetric eficienry is nearly mrotant, at high epeeds it ....... .
Tij*:"j..:f:
diameter is inieased T:rrr^":^.f:"u
to 4.5S mm. 2. The higher the altitude ,... will be the volurnetric efficiency.
3. The volumetric effrciency is defined as the ratio of ideal air mpacity to actual air capacitr of an engine'
(idi) fuain, wirhout i ' the nominar di-
4. . The ideal air capacity is ilefired m \^ ma,es trow of fresh air through e ergine per unit time.
amete or rhe inrer varve
= :ff H,::ffi:ll';"d"[:lmj.T" "" 6. Volumetric efficiency of 4.stmke cyclelG-engines ... with incrcae in inlet temnrattm
8. lte maximum volumetric effisiency speed can be increaeed for kole I.C. engines by..., the inlet valve
_ 2 x oJ.4j< 28oo diaeter.
= 18.o6? rls. 7. fiie volumetric efcienry ofrl-stroke cycle I.C, engine ... with increase in coolant temperahrre.
Thus, the new value ofZ is given by: 8. For 4-stroke cycle diesel engines exhaust gas tempraturo increaos rith loed.
9. In-a 4-stroke cycle I.C. engine, ideal air apacity vries directly ar e piston ePee
=
(D-=-\' , uo -( orl 12 - 18.06z 10. Incated mean effective pressure for a Gstorke I.C. engine vuic inversely as inlet air density.
" 11. Ihe volumetric efficiency of a ,l.stmke I.C. engine varies directly ae the diameter of the cylinder.

drops in the
,1:21":T";lH,,mr:;::hd
. correspondingproportion
same
ro
brque a,so
t2, The volumetric efficiency of a ,l-stroke I.C. engine varies ,,.... as inlet air density.
qur. as 18. Inlet valve Mach Intlex for ma*inum volmetric efficiency for four stroke engine is appnoximately ......
t4. Ihe volumetric efciency of a 4-stroke S.I. engine remains faly cdstant with increase in F/A ratio.
(iu) Again, u, at 4800 r.p.m. = Zti#e, = 22.4 tts 16, The isentropic index ofcompression (1) increases as the F/A ratio ofoctaneair mixture icreases'

t=(#-\'x--221-
\0.04495, x869.6
--r.l o.3B
Conesponding to Z = l.t,from Fig. ?.8, l. falls rapiclly 2. lower 6. decreues
I,. _ 56% app. 6. inmeasing 7. decreases lO. No
It may be noted that whereas speed has inase.
from 2400 r.p.m. to 4g00 r.p.m. !.e. 100 per- ll. No 12. inversely 16. No.
cent; the volumetric efficiency has fallen ,,
%;E = 0.86 or BMo.
THEORETICAL QUESTIONS
- ltl';llll11"X"::X"** i:T:_:y !.r,*2400 r.p.m., rhe vorumerric efficiency
'i::"\"::i*':Y:;:*,!::!!,tyy,!i;;;2;;;;;;'*';;;;":;:;:,':::;T{. I. Dene ideal air capacity ofan engine. How does it differ from ac'tual air apacity ?
';::":!:i::::::.':f'::y*i*'tiiiin""*";i;*;;;;;;;;;il"'#,
_!;"::,:2:::::::^2,y:;elt1jnesryd.7na^*i^u^io.u.Llii,,#!o""nbeuar. 2. I{ow do you define volunetric efficiency of an I.C. engine ? How is it related to the power output of the
engine ?
ied' ond flrcd, at the required o"7*t itn tlil"'*t"rlt'ri etc:. L How is the volumetric efficiency affected by speed and altitude ?
^"ifications
1, Discuss brielly the efects of the following factors on e volumetric efficiency:
(i) Frel (ii) Heat trnfet-high temPerature
(iii) Valve overlap (iu) Fluid friction losses
L Air capacity is defmed as e mass flow of fresh air tlmugh the engine (u) Choked flow (ui) Intake valve closure aftr B.D.C.
2, The"ideal air capacity conesponds to filling
per mit time.
thu ai"oru roir*e with fresh nixture at inlet cond! (uii) Exhaustresidual (uii) Exhaut gas recycle (EGR).
6. What is the effect of "Inlet Mach Number" on the volunetric efrrciency of an engine ?
266 INTERNAL ENGINES
"O'"U"'O*
6. Why the inlet valve be kept open for a few degrees of crank mgle even when the piston is on the compres-
sion sroke ?
Assme that the engine under oomideration is a high speed one.
"A 4-stroke I.c. mgiae is always economical and les pollutant than 2-stroke engine". Justi& the stste-
ment.
Two Strohe Engines
UNSOLVED ETAMPL,ES
cycle and
8.1. General aspects-Construction and woking-Comparison between two stroke
1. A six*ylinde-r-four'stroke spark ignition engine having a piston displacenent of 200 m! per ioo" .t.ot" cycie engines-Disdvantages of two stroki S l engine compared to two -stoke
cylinder
78 kw at 3200 r.p.m' and consnmed 2? kg oi peiror per hour. rhe calorific C.I. engine-Rea"ot-" fo, *" of two stroke engines for marine propulsion-Recons for the
?los:-":d
44MJkg. Detemine:
va"J ot pet or is us" of iwo stoke S.I. engines for low horse power two wheeles 8.2. Intake fo two stmke
systems'
(i) The volumetric efficiency of engine if ai-fuel ratio is 12 md the intake cycle engines. 8.3. Scavenging process. 8.4. Scavenging parameters' 8'5' Scavenging
air is at 0.9 bar, 32.c ; .g. C.ua"u"u scavenging-. a.?. Scavengirg pumps and blowers-Highghts-Objective
Tlpe
(ii) The brake themal eficiencr ;
(;ii) Thebraketorque. Questiom-Ttreoretical Questions.
tAne,()?8.r6%; (ai)2a.64%;(ii)zggNm)l
O-n testing a S.I' engine it was obseryed that the volumetric
effrciency is maximum when inlet valve 8.1. GENERAL ASPECTS
Mach Index is 0.55 and the indieted torque, and ineted mean efTective pressure
occured at maximu
volumetric efficiency. 8.1.1. Construction and Working
the engine having a bore of 120 m and stroke 150 m. British engineer introduced a cycle which could be completed
running at 2500 r.p.m,
produces muimum incatetl torque when o In 1g28, Dugald-clerk, a
(j) Detrmine the norninal diametr of the inlet valve. in two sro& of piston rather than fuur strohes as is the case with the four sroke cycle
engines. The engines using this cycle were called o strohe cycle egns. In this
(ii) If the sme engine is required to develop maximum indiated power
valve size be modied ?
at B0O0 r.p.m., how will the inlet enlne suction and exhaus[ strokes are e]iminated. Here instead, of valves, ports are
(ii) If the used.Theehdustgosesaredrivenoutfromenginecylinderbythefreshchargeof
same engine runs at 3000 r.p.m. without any inlet valve modifications, how will volumetric
efficiency get allected ? fuel entering the cylinder nearly at the end of the working stroke'
Presue at intake valve = 0.$ 6s ; Tennrature at iutake valve 350 K Inlet valve flow ccffrcient o Fig. 8.1 shows a two-stroke petrol engine (used in scootes, motor cycle etc.) Refer Art.
= ; 2.12 also.
= 0.325.
Assume : Fuel-air mixture as perfect gas with Spark-plug
7 = 1.4 md R = ZgZ J&g K
(iu) what would be volumetric efficiency at muimm power
speed of 5000 r.p.m., for the un-modified
engirie? tAne,(tag;n; eil47n;idiilu%;(o)55.25%l

port'
L = Cylindet;M =Piston ; C.C. = Cankcase i V= Valve ; E P. = Exhaust port ; LP' =Tlansfer
Fig.8.1. T\ro stroke cycle engine (ualkcase scavenged)'

,
INTERNAL COMBUSTION ENGINES
TWO STROKE ENGTNBS
269
The cyclinder L is connected to a closed crankese
- C.C.
During the upward stroke of the piston M, the gases
- same time fesh air and fuel (petrol) mixturu
in L ae compressed and at the
the crank claner tnrough tlre
valve V. "it"".
\hen the pieton moves dow'wards, v closes and the mixture
- compressed.
in te crank chamber is
I

- *9 8.1 (i),
-we 5. o. u prson
(/, the piston rE
is movrng
moving upwads
upwards andantl is conpressing an eplosive charge
I Tt"
supplieil r.-o i. fg"itign Lk". place at the enil of rhe sfroke.
g *lt^Y:
piston f.ously,been
The then travels down;"d. d;J;;;;;;i;;;f;;;
the end ofthis-strpke the pieon uncovers tn"
(n;:;.i A;H;",
E po"t (8.p,) anil the burnt exhirust
o- gases escpe through this porf, (Fig. g.1 (jtt)). "ra"rt
the transfer port (T.p.) then\
then\.uncoverod immecliately, and the conpressed charge
*::*.^f:t* 9IJ
i:::*IT\lh-b:l"*.")it"_tr,""ril;il;;:ffiil;;';iiil;Jl'"i;
provided on the head of the rt -a " noted that the *o*^g iir'$lr"I;;;;i
pisto-n. i
helps the remoual of gases ftom the enginJ-cyrind.er
' ; if, in o,se these uhdust gases d.o not
leave the qilinder, the fresh charge g"ls at"
\lolme _-_-___} a"i
ifficencJ of:the ;glie wi decrease,
The piston then again sarts moving from B.D.c.
Fig. 8.2. p-V aliagam for a two stroke
cycle engine. - pressed rvhen E.P. (exhaust port)
t T.D.c. and trre charge gets com-
T.p. ae covered by the piston ; thus the cycre ds
repedted, "o
o Fig' 8'2 show the p-v diagram for a two stroke cycle engine. It is onlxr. for the man
c1linder or the top sidc of the piston.
o Fig' 8'3 shows serf-explanatory port timing diagram for a two stroke cycre
engine.
In a two stroke Dieael eycre engine drl the operations are
the sotne as in the spark
ignition (ouo cycle) engine with tn" ai"i"t"i
; nr"tly in this case, onlxr ir is admtted. into
qrlnd'er instea'd of air fuer mieture and,'*ondry'toii
of a sparking plug.
ii*to, ic fifted, to suppry thc fuer instead
c Note' The top of the piston usually hqs a projection /hmp
the cylinder before flowing to the exhausipo"t".
ttis s"-". h" iru"*i"gt*o p.r"po"* i- - "--t
to deflect t}re fresh air to sweep up to the top of
;o (i) To scavenge the upper part ofthe cylder
ofcombuson products.
Pr ff, (ii) To prevent the fresh arge from flowing
3l ! directly to the exhaust ports.
The same objective can be acrieved'witrnut piston
dzflzcto bx proper srnping of thz transfer port.
ol 8'l'2' comparison between Two-stroke cyclo and Four-troke
o cycle Engtnes
o For comparison between 2-stroke cycle and 4-stroke cycle engines refer
At. 2.14.
o For all the petrol as.weil d,iesel two'strohe engnes acosmon ditadoantage greater
is
cooling and lubriatian requirements d,ue b ol.e power
stroke in each reuolution ofcrank-
shaft' Dae to higher tenperature the consumption
of lubrication oil is also high in two-
strohe engines.
8'l'3' Dieadvantages ofr\ro-soke s.r. Engine compared
to T\vo-stroke c.r. Engtne
Following are the two main disadvantages from
which the two-stroke s.I. engines suffer :
1. Loss offuel
2. Idling difculty.
;
o Ir case two cylinders are supplied the fuel after
the closure of the exhaust ports, the
Fig. 8.3. Port timing diagram. flrel loss will be nil anti the ind-icated thermaiefficiency
of the two-stroke engine will be
comparable as the four-s-troke engine. However,
in s.I. engine using carburettor, the
' scavengingls done with fuel-air mixture and only
the fuel mixed with the retained air
is rsed for combustion.
{
_f,
270 ENGINES TWO STROKE ENGINBS 271

In orfur to avod. the loss of fuel insteod of carburettor fuel injection just before Thee are open combustion chambers in the two stroke cycle engines. It would be
- the exhaust port closure may be used, extremely difficult to get proper scavenging in a cylinder with a divided chamber.
a At lotD speeds when m.e.p. (mean effective pressure) is reduced to about 2 bar, the two In some automobile engines stand,ard-type superchargers are used ond. the air is input
stroke S.l. engine rzns irregularlX and, may euen stop. This is owing to large anount of through intahe values with no fuel add.ed. The compressed air scavenges the cylinder
residual gas (more than in 4-stroke engine) mixing with small amount of drarge. At low and leaves it lled with air and a srrall amount of exhaust residual. After the intake
speeds there may be backfiring due to slow burning rate. valve is closed, fuel is injected directly into the combustion chamber by injectors mounted
Fuel injection improves il.Iing and. also elmnates bachfirng as thcrc is no fuel in the cylinder head. This is done to avoid HC pollution from fuel passing into the
present in the inlet system, exhaust srsterr, when both exhaust and intake valves ae open. fn some automobile
engines, air is injected, with the fuel. This speed.s euaporation and.'mixing, which is
In case of CJ. engine there is neither fuel loss (as the charge is only ai) nor difrculty in idling
required, becavse of the very short time of the compression stroke.
since there is no reduction in fresh charge (air).
Fuel injecion pressure is oforder of 500 to 600 kPa, while air injection pressure is
8.1.4. Reasone for Use of T\po-stroke C.I. Engines for Marine Propuleion slightly less at about 500 kPa.
T$o-stroke C.I. engines lind wide use in marine propulsion for the following reaons : For "S.L enginz" fuet injectian occurs early n the compression strohe, immed.i.
1. More uniform torque, the ideal requirement for the propeller. ately offer the e*haust ualue closes, In 'CJ. engnes'the injectinn occurs late in
the compression stroke, o short time before combustion starts.
2. More cooling is required in two stroke engines, plenty of sea water is available for cooling.
In just about all two stroke cycle engines, due b cost, crankcose compression is used,
3. In C.I. engines there in no loss of fuel in scavenging. Hence they have higher thermal
to force air into ond scauenge thc cylind,ers.
efficiency.
In these engines, air is introduced at atmospheric pressure into the cylinder below
4. Propeller imposes the cond,ition that maximur power nust be deueloped at about the piston through a one-way valve when the piston is near T.D.C. The power
100r,p.m. Two strohe enginzs may be mad of slow speed,, and with large d.isplwnlent uolume
stroke pushes the piston down and compresses the air in the crankcase, which has
(over 60 cm bore) and of capacigr 5000 kW and above. These slow speed engines can be coupled.
been designed for this dual purpose. The cornpressed a then passes through an
directly to the propeller of the ship, wihtout the necessty of gear reduction. input channel into the combustion chambers, ln mo&rn automobiles engines the
For marine propulsion, two-stroke C,L opposed. engine (cross-head type) is nainly used, fuel is then added with injectors, as with supercharged engines the fuel is then
added with injectors, as with supercharged engines. ln small engnes the fuel s
8.1.5. Reasons for the Use of Two-stroke S.I. Engines for Low llorce Power T\ro
Wheelers usually with a carburettor to the air as it enters the cronkcase. This is d,one to
-
keep the cost down on small eugines, simple carburettors being cheap to build.
r When applied to S.I. engines, the TVo-stroke cycle engine has certain disadvantages The fuel injectors will probably become more comrtron as pollution laws become
which have restriced its uee to small low horse power engines. more sringent.
L In S.I. engines the charge consistq of:q 'nrixture of air and fuel. During sco venging In case of two stroke cycle engines srg, cranhcase compresson, Iubrizating oil must
both, inlet and. ex.haust ports arc open simultaneously for sometine. Some port of be add.ed to the'inlet oir. The crankcase in theee engines caDnot be used as the oil
the fresh charge escapes with ethaust which results in higher fuel consvmption ond reservoir as with most other engines. Instead, the surfaces of the engine components
lower tlermal fficicncy. are lubricated by oil uapour canied by tlu intdce cir. In some engines, lubricating oil
o IJor small two-wheeler engines the fuel economSt is not a aital factor. Herc light. is mixed directly with the fuel and is vaporised iu the carbuettor alng with the fuel.
ueight and, low initial cost orc the main considerations, which are the mdn Other engines have a separate oil resenoir and feed lubricant directly into the intake
characteristics of two-strohe S.I. engines. air flow. Tto rugatiue resulfs occur beacuse of this method of lubrications : (i) Some oil
vapour gets into the exhaus llow during valve overlap and contributes directly to HC
8.2. IIVIAIG FORTWO STBOKE CYCI,,E ENGINES exhaust emissions ; (ii) Combustion is less efficient due to tbe poorer fuel quality of the
oil.
a In two stroke qcle engines inlet air must be mput dt a pressure greater than atmos-
whiah use superchargers or turbochorgerc generally use stand.ard. pres-
pheric. At he stat ofthe intake process, following blowdown, the cylinder is still filled - Engnzs
srised lubricatan systems,,with cronkcase serung as the ol reseruior.
with exhaust gas a amosphec pressure,,There ii no elhaust stroke, Ai under pres-
sure enters the cylinder dnd pushes most ofthe remaining exhaust residual ot ofthe
o In order to avoid, dn eucess of erhaust resid.aal no pochets of stagnant flow or d,eod.
zones cdn be ollowed in the scavenging process. Ths is controlled, by :
sill-open exliaust port. This is called scavenging. When most of the ert-haust gas is out,
(i) The size and positiotr ofthe intake and exhaust slots or valves ;
the exhaust port closes and the cylinder is filled, with
9tf1 (i) The geometry of the slots in the wall ;
At pa throttle inlet pressure is low, and this r'esults in poorer scauctrying.
- (ttt) The contoued flow deflectors oi the piston face.
o Generally following two methods are used for putting air into the cyllnders :
(i) Through normal intake valves ;
8.S. SCAVENGING PROCESS
(ii) Through intake slots in the cylinder walls.
The intahe air is pressursed using a supercharger, turbochorger, or crankcose
o In a two stroke engine because of non-availability of an erhaust stroke (unlike four-
stroke engine) at the end ofan expression stroke, its combustion chamber is left full of
compression,
272
INTBRNAL COMBUSTION TWO STROKB ENGTNBS
ENGTNES
combustion prducts' The process of ctearing
the qtlnd:r after the
";,':;i,"f,jfil";i-r-':::' ;4;J"5"'"."?. is.the-reptacemeit
exp,,nsinn strohe is
or combustion Adabac

ir
o coarprsSort

"#:ti?t{?::ipJ""'#'tr;:Ti j""i+:;ifi '!{:ff ^"{#m,'!,i:i",#:;r


E
_9
o

E fo*
' lffflXX"'*o stroke engine fe;;;ff;:'H fi:":ff-11'j;fl?'." ,. -
o J

x
U
Inadequatelpoorlbad. scauengry
teads tn the following :
^II
i*il.r' resur-' in high weight and high cosr I

:'i:I-fi,"ili1n$,T"""T:.1?:"f
(lt) Low amount of
o
htshe^o*r#'f:i:::u;l;fI *t" results in incomplete combustiou leading
E Pi=1'335bar
I'

, bContamination
(jij)
o
E
of lubricating oil to a sreate o-
P" = 1.0132 bar

,rr'iiii'#"4"*ot*uv"d'#;":;;;;f*Y::;:;::"'"Y";t:H!;:::*
oscavengingo"".";f;'ff ";;#i::':;"Xii"ff::;:,::"I;::,:,;:,^ -70. -0" 0 50. 70.
(i) pre.blowdo*:_gl tn" Crank angle(o) ---------+
op"ning oir,"i'O"", the.gases expanding in the main
Fig. 8.4. Scavenging process onp-0 for a loop-scaveoged
ff f:*'*::c:.,l1t"."",,"ig"'*,mainrord.
(jj) Browdown. with ti" opuni";;;;;;J#ff:l"Tf* ** cylinder ofa typical two stroke engine.

"li::i",*i:fiTfti",1i:f"l,f in tbe cyrinder ar Whereas the gases flow into the inlet ports, the exhaust gases continue to flow out
lffi ifi :T"T:*:",*i:li#*hr,'?,uou,rjiob;;;;i;;;;,d;conse_ of ehaust port, due to the fact that these started in this direction at high velocity
during blow-down. Also due to the fresh mixture entering through inlet,port there
:ffi i;,il";j"i1'111.u1e*.a*";il::?3,iJ*"*J*"T#:iTil:;::::
yltnder attains a value
slightty less than
is building up ofpressure in the cylinder higher tban the exhaust system pressure.
enge manifold, air pressure inside scav- Scavenging angle. It is defined as the crol angle during which both inlet ond,
(lli) s"cavenginE'ff:tt":""ltt1 exhaust ports ore open.
q3lensinc nrocess srarrs ar rhe moment the sponta- Scavenging period, It is the time perod. tahen for scauenging angte.
r,.o"tpo"tu1";'1Tr"."ril:Ht'"LT:,t:##"'.".ena'"im-Jl".""tt""*- o Afte the closure ofthe two ports, cycle is completed by compression, combustion and
the min qtinier at the end o sponton"ou!t^:y::!! resid'ual gases remaining in expansion in the cylinder as in 4-stroke engine.
pletely as iossible erhoust and replaces then as com-
with fresh Theoretical Scavenging PFocesses :
tt'ft,fl"ltl;|ff-qlffi^T"_lthe":";;;."",-*"^
c"ornpletion orscavenging phase, Following are tbe three theoretical scavenging processes
the rresh charge :

ni,ru.ipr,".l'"::ii"'Tffit"rli,Hil"J$,ilf"r":d;;;h;;;1.derris. 1. Perfect scavenging.


2. Perfect mixing.
' llfi;ii1"l"ir;:ii;i,T#'T ";;;;il;l"u,,k ",'gru
(p-0) ror a roop-scavenged 3. Short circuiting.
It shows the adiabatic 1. Perfect scavenging
is 1.013 bar "ompsioo curve frorn B'D'C' when the exhaust pressure
(a-tiosprreric). o In- this type of scavenging, fresh air pumped into the cylinder by the blower through the
- l:ilrlti""I|:H3.:r]..oj"ns at 70'before r.D.c., the pressue inlet ports at the lower end ofcylinder pushes the combustion products ahead ofitself
in rhe rylinder falls and ofthe cylinder through the exhaust vah,e at the other end.
r.^nipii;*::i,.iitr;"nw:irf;::":;i"::i:!i""tri
exhaust pressu,e).
The air and combustion products d,o not nixtogether,
In practice . t"
l""rii" liect ofthe gases after the blowrrown o so long s any products remain in the cylinder, the flow through the exhaust valves
consists of products only.
;["i',ffi ::,:};,;"]il j:l#':?;;j{":"*,Vflt-,n".""'i""ii)1i",}Ji,"r"*
ope'.rheinrervarmaybe";.*;;ffi:""t",i::1""'ii"jr","":1":lirl:?ff 2. Perfect mixing
cylinder pressure below scaven8:ng In this
rhe cytinder and continues
pi"""-;, the fresh charge ges introduced ,r; process, the incorning fresh charge mires complete\r and, instantaneously with

pressue beconres uoe than


* ;;-tii;;l:! in
o:* is open, and rhe rotal inler
the grlindcr contents and a portion of this mixture passes out of exhaust ports at a rate
p"rrr1.l" ti" equal to that entering the cylnder.
"i]rr"o"". I'he outgoing (homogeneous) mixture consists init!ally ofcombustion products only but
then gradually changes to pure air.
INTERNAL COMBUSTION BNGTNES
TWO STROKE ENCINES 2'15
o gice the reak,of tr'is procese closely
approximates the result of mny actuar scaveng-
ing proeres, thelrdore, it ia often useil, as a basis of amparison
Presur loee ccici.rn. It s defined as lhe ratio between the nain upslream ond
3. Short.etrcdiry daunstream pressures during tlu scauengng periad, an'd' rcpresenta tlw pressure loss to which
the scauerqe o;ir is etjectcd whn it crosses tle eylindcr.
In-this prooees' tlere fteh chwge coming from the scavenge maniford diretrx goet out
exhdust ports witfnut remoaing combusiian prodwts / gases.
of Exces air factor (}"). The value (Eo.-l) is called the excess air factor. Thus if the i?*,
(delivery ratio) is 1.3 tbe excess air factor is 0.3.
e It resuls iE e deod loss End ite occuence must be checkeevoided.
Measrrensrt o( 6cavenglng Efcency'
6.4. SCAVEIrrGnrG PrE*EtDRs The followingFroccdure is adopted in d'iesel enghes for measuring the scaeqgi-rry effiziercy :
c For the same puer gencratinnl more o,ir input is required, i a tun stmhe cycle engirc o A small samph of the conbusli,on products is drawn just before the .exhaust vlve
operrs or during f.h eadier part of blowdown.
tltan in a, four strolte qcle engirc. This is because some of the cli is lost inihe ooe-rlap
period of tle *aoenging proeess. o The samplo is aaalysed.
A number of di&et intke and performance efficiencies ae defied for the itake Ttre results obtained are cornpared 'ith stantlad curves ofexhaustrrodu.cts us. FIA
' procese ofwo sboke cycle engine.
ratio. This deterulinee the FIA ratio tbat.must have existed in the cylinder before
' combustion.
o volumetric efficilcy of * four-stroke cycle engine can be replaced by either detiver.r r Knowing tbe quantity of fuel injected per oycle, the quantity of esh ai retained in tle
rafo (s"*) or c.Largtng efftcteney lior : cynilerpe cyde is detonni."d. Air present in the esidual,gas is ant consider.ed as it
rerreseut a contaot quatif ryhich lnc 6f, participate in combusf,ion pocess.
Dthe4rrriot&ir. = % ...(8.1)
E.5. SC.AVEr*iSG .Srtr'D*rS
t*rfngcetcney . \* ffi,
= ...(8.2) Diferent scavengig systems/anangements bosed on charge flow ale enumerated and de-
scribed below :
where, na = Mags of ei-fuel miture ingested io the cyliader, 1. Uniflow scavengiog
tra, = !fss$oai-futr mixture tmpped. ia cyliader after all valves are closed, 2. Loop or reveree scavoging
% =Sre'pt voluntq and 3. Cross scerengiag.
p, = Deeity of air at ambient conditione,
Ilpical 1. Unifow scaYeging:
vulutes: 0.65 <
< O,gd
It is th most p.f mdh.od, of xauenging,
ft*< q,* < 0.?5. o lhe fre ehaqe i adpied sl oae ond of tbe cylioder arvl tb *rhauet escapes at the
- Parynrtb ig'c'e tfut crwr*q ffiienq beeaise someof
iBwtei{tEeryl&deris ro't out of hs *s prt be{rei{ ieelroaed,
tb air-frel mixsrc ofei's.t6rir$w is oF nd o eo4 sn lde So*dcsiOigetwern he intake
and ee*t aDqrg ;is *eable.

- f ioiort fuer a&s thcvtrvesse orose4 M.nn's of mirtare ,o T . t$*ee qaqe arrmgeoera Sr 'uaiflor c*'eqgig cc fuvn ,in fig.8.6.
Softmccmglgstke
rn the eqF'lar'6 (8,f e 8,2) Bfocw fu reflaad with the nw g trl{/csta ilr.
Saocc_
titnast *rfu dir nei& s reptwd ey tiu density of air in tn intet ratner doun-
sreant, {tlr. *pe

trM
OthreffieircP:*:

Brptug oiency = qq. = = ..{8.3)


H *--. . NSE T
g&.&tutfutuncy
= nn. =
ff ...(8.4)

. hlrtitegare'=C = ttqb
4t rw tW
where za* = t\asa of orar.ff"r"-.#
fYpieal udvas :. 066, < t6ao. <
tF;tt*-I,
=tlcrt'

incrudius exrrau., **o;:'t*t a'


A, tE

4ru"ffi
OIEO
815<q6. < 0:90 r
O,b< O'. <0190:

(a) (b)
ffiN (c)
-

Fig. 8.5. {Jniflow scavengiag.


276 )11
TNTERNAL COMBUSTION ENCINES TWO STROKE ENCINES
Apoppet ualue is used [Fig' 8.5 (o)] to admit the
inlet charge or for the exhaust, as Owing to the absence of cams, valves and valve gear, loop or reverse scavenged eng'ines
the case may be. i
ore simple ard. sturd,y, They have a hgh resi.stonce to tlermol stresss and, are thus
In Fig. g.5 () the inlet.and.exhaust por:s both controlled. b! sepdrdte pistons rnuch suited to higher superchorge.
:r.e
that rnove in opposite directions (oppos"o prston
engines) In a loop scavenged two stroke engine, the major mechanbal problem is at of obtain-
In Fig. g.S (c) the inlet and exhaust ports are controlled, ing an adequate oil supply to the cylinder wall conaistent with reasonable lubricating
of piston and, sleeue.
b! the combined. motion
oil consumption ancl cylinder wear.
c All uniflow systems permit unsymmetrical port tizlings
. Due to absence of any eddiea o turbulence (at least and supercharging. 3. Cross.scavenging
theoreticaily it " , In this system tbe inlet and exhaust ports are
uniflow scavenging system to push the conbuston "o"i",with-o
products out of the cytinder located. on opposite sid.* of the cylinder @rE. 8.7).
out mixing with it and short circuting. Thus
this ,:wt"^', iiJ lisi.r,
ing efficiency. sca.reng- The incoming flow is dirbcted.upwards by the de-
o since this systems requires either opposed flector on the pistou, and the cylinder head reverses the
systems, poppet varves or sleeve valve (alr
which increase the complication) its-constrction -'-*"' of direction offlow, so that exhaust gases are forced through
s rroi rUfi".- the exhaust oort.
2. Loop or reverse scavenging : In this type of arrangement the engine is
rn loop or reuerse scauenginc,
fresh air first sweeps across the piston top, rnoves up
structurally simpler than that with the
and then down and finaltv out throlsh!h1
*" uno"tt.' ln" system auoids the ihart-circuitng uniflow scavenging (due to the absence of
cross-scauenged engine and thus improues upon its scauenging
effi.ciency.
of the valves, distributors, and relative dive de- ,/
Inlet port
o In the MAN type of loop scavenge, Fig. g.6. (o), the
exhaust arr,t
ces).
same side, ihe exhaust aboue the inlit. inlet ports are on the The main demerit of this system is that scu-
In the Schnuerle type, Fig, 3.6. (), the ports are enging air is not able to gzt rid of tle layer of
side by sid.e. exhaust gas near th woll resultng in poor
The Curtis type ofscavenging, Fig. g.6 (c),
is similar to the Schnuerle type, except that scavenging, A small portion of fresh charge
upwardly d,rected nlet ports are plucecl also opposite goes directly into the exhaust port. These fac-
the exhaust ports.
tos contribute towards poor b.m.e.p. of tbe
cross-scavenged engines,
Fig. 8.7. Croes sevenging.
8.6. CRAI\KCASE SCAVENGING
This type ofscavenging arrangement is ernployed in the sinplest type oftwo stroko engine,
and is shown in Fig. 8.8.
In this engine, the charge (fuel-air mixture in S.L engine and air in C.I. engine) is con-
pressed in the crankcase by the underside of the piston during the expansion stroke. There are
three ports in this engine-intake port cit the crankcase, transfer port and the ethaust por. The
compressed charge passes through the transfer port into the engine cylinder flushling the products
ofcombustion. This .rrocess is called scouengng, and this type ofengtne is calleil cranhcase scau-
enged, engine.
o As the piston moves down, it first uncovers the exhaust ports, and the cylinder pres-
sure drops to atmospheric level as the combustion products escape through these ports.
a Further downward motion of the piston uncovers the transfer ports, permitting the
slightly conpressed mixture or air (depending upon the type of engine) in the crank-
case to enter the engine rylinder, The top of the piston and the ports are usualTy shaped,
in such a way that the fresh chorge is d,irected. towards the top of the cylinder before
flowing toward.s the exhoust pors. This is fo the purpose of scavenging the upper
part of the cylind.er of the combustion products and also to nnimize the flow of fresh
charge d.rectly through the exhaust porls. The projection on the piston is called the
deflector.
(b) r As the piston returns from B,D.C. the transfer ports and then the exhaust ports are
closed and the compression ofthe charge begins. Motion ofthe piston during compres-
Fig. 8.6. Methods of loop scavenging sion lowers the pressure in the crankcase so that the fesh charge is drawn into the
crankcase through the inlet reed valve.

._l
Read valvs
opsraled
intako port

Fig. 8.8. Crankcase-scavenged two stroke engine.


Fig. 8.9. Piston type pump. Fig. 8.10. Roots blower.
o Ignition and expansion take place in the usual way, and rb.e cxcb is reputed.
Demeritg :
'
1. This system is very unccornmical and ircfftcent in operdtian. This is owing to the fact
that amount of air which can be used for scavenging is leea than the swept volnne of the
cylinder due to loar uolumri efficincy of the cmnhcose whih antoins a bge &d
spoce. Thus the delivery ratio (Rrr) is always less than uziy and as such it is not
possible to scavenge the cylinder mpbtely of the combustion products and some re.
sidual gases always remain iir the cylin<ler. Consequently the aankcase-scavenged en-
gine has a lower m.e.p., typical valves being B to 4 bar. since the charge hansfened
through the transfer port is only 40-50 percent ofthe cylinder volume, the-engine output
is strictly limited.
ofthe oil vapours from the crankcase wih the scavenging air, oil con-
2. Due to mixing
surnption is increased.
In view of the above demerits tlte crankcase scauenging is not preferred and, a scavenging
pump 4 essential for a high output two stroke engine.

8.?. SCAVENGING PI,'MPS AND BLOITERS


\
Since the piston ofa wo stroke engine cannot carry out the pumping acion, therefore, a
7@;,
separate pumping mechanism, called the scauenging pump, is needed tb sup'ply scavengiug air to
the cylinder.
Following types of pumps are used : crankcase compression Gefer At. g.6), piston, roots
and centrifugal blowers.
Pison type pump shown in Fig. 8.9 is used for low speed, and, single or two qlind.er
engines. Fig. 8.11. Ceuhifugal blower.

I
INTERNAL COMBUSTION ENGINES
281
o Roots blower shown in Fig. g.10 is TWO STROKE ENGINES
used fo stnq.ll and, mcd,iun
o centrifugal brower shown in Fig' g'11 engines.
16. In......scavenging,thefreshairrstu',eepsacoesthepistontoP,movesupmdthendownandfinallyout
r"
"*ploi o" rarge and. high output engines. through the exhaust.
L7. In ...... scavenging system the inlet and exhaust ports are located on opposite sides ofthe cylinder.
1& Crmkcase smvenging mangement is veryueononical mdinefficientin operation l
t 19. Piston tpe pmp is used for high speetl engines. \ /
*'r*I";t':*""t1ffi.";Tg-H'-:"#ilted in two strokes' ie. one worution orthe crankshaft m. Roots blowe is used for small and medim engiaes.
, from whi the two stroke
as

S.r. engine suffer are :


.,Tff; i:rlisadvantases
g' rhe two stroke cJ' oppose engine cro."-o""u,r'o"tr'."-t1ffff#?;"*e I
4 Theprocessofclearincthecyrina"""n."trrJ"il""a"'#*.*"*ed,scouengingprocess.
propursion. 1. ports 2 Yes 3. Yes dNo Yes
6. ScaveBngpmcess consisrs of e superchargers T.Scaveriging & Scavenging I Scavenging 10. Yes
(i) ke-blowdom
the foltowing foui ;i.o*;;;;r.., 11, more 12. No 13. Prssure 14. excss 16. Uniflow
(iii) Blowdown(ii)
Scavengu (tu) Additional charging'
16. loop 17. cross l& Ye.s 19. No 20. Yes.
6' scauengin.l ar,la r. auno"athe crank
angle during which both inlet
7' scauenging p.rid is the tin and exhaust ports are open.
THEORETTCAL QTJESTIONS
E. Thoreticar smr"rrro orr]"Tflltaken for scavmsing augle.
(i) Perfect seyengng
; (ii) Perfect mixing; 1. "In it is bettr to have deflector top tyPc piston". JustiS the statment'
a 2 stroke eogine
(ili) Sl6""ui;nt.
L Disrus the two min disadvmtages of twestroke cycle S.f. engine. How are these disadvantages avoided
S_cavengingsystems, based in the two stroke rycle C.I. engine ?
on charge flow, are classified
(j) Uniflowscavenging a follows :
(li) Ioop or reverse smvenging & Discuss briefly Mist lubrication system'.
(iii) Cross scav l-- L Why do ihe two stroke C.I. engine fnd wide usein mrine propulsions ?
a Why are two stoke S.I. engines more commonly used in low horse power two wheelers ?

ffi
L
OBJECTTVE TYPE
AUESTIONGI
6.
7.
Why is crankcase scavenging used only for low power engines ?
Why are two stroke diesel engines, for large power, moe common than two-stroke S'I' engines ?
What is the reason that two-stroke engine is not used in car even though it develops theoretically twice
Fill in the rt*k" o. s."ry.ffi &
power than that offour-stroke engine ?
1. In a two stroke engine instead
ofvalves, ...... ae used.
z For marine propuJsion, twostoke Explain with suitable sketches the following scavengingprocesses :
C.I. op!"."d;;;;; (moss-head type) is mainly (i) Uniflowscavenging (ii) Imp savenging.
& Forsnat two-wheerer engines used.
ght-*"ight
4 In a two stroke cycle engine inlet -a ; ioiit"i"""* "* rn" -ain consideations. 10. Explain briefly aankcase scavenging.
airmust be input at ,Or"r*"" **
,rran atmospheric. ll, Explain the scavenging procss in two stroke engine. Disucss three scavenging processes used in two-
D. In case of two stroke cycle engines
usi"g
".*t"ir" *l"r*iubricatingo'must be added
stroke engine,
to the inlet
a 12, Defrne scavenging and scavenging efciency. Explain with sketches different scavenging arrangements
*1ffi:LtX;;;;ff;:y" *" stardard superchrge pressuised
lubricarion sysrems, with omkcase
based on charse flow.
1& How the valve timings of a two stroke engine difrer from that of fou stroke cycle engine ?
;f:::ffir:T;?ilffiilii:*};stioa.products in the cvrinder rrom previous power t4. what is the difTerence between the valve timing ofa crankcase-scavenged and supercharged two stroke
stroke with engine ?
&
during which both inter and Compare the relative nerits and demerits ofdilferent scavngingsystems'
exhausr porrs are open 15.
9. :""r,":i"tl""Tr*:::l:
.. period is the time period takerr for
..uuuogirg uogi; How is the supercharging oftwo stroke engines done ?
16,
10.
lh*:t;.::.*X"ti"_"1l1g.ru.r"t,u,g.""oi,i, ji.o*tr," scavenge manifold direcily goes
* *if i"s *-iJ''' out of
tl. ;* iH'.: :'-::1-"i1
*"er generation]...... """il;'iHi
l,lili:'#:
cycle engine "t;;"rJ;;;;.;d in a two stroke cycre ensiae rhan
in a rour stoke
12. Deliveryratio is smaller
than chargrng efficienry.
13. isdelined as the "tio butr
;;]ffi;;[xxll th" .ain upsrream and downstream pressures
durins
t4. The valve (Rrr. l) is called
- the ...... air factor.
15. ...... scavengingsystem has
the highest scavenging efficiency.
CHEMICAL THERMODYNAMICS AND FUEIS 283

has a molecule which consists of two atoms of hydrogen and bne atom of oxygen' The atoms of
different elements have different masseg and these values are important when a quantitative analysis
is equired. The actual masses are infrnitesirnally small, and the ratios ofthe masses ofatorne are
used. these ratios are indicated by atonic weigt quoted on a scale which defines'the atomic
weight ofo:rygen as 16.
The symbols and molecular weight of some important elements, compounds anil gases are
given in the Table 9.1.

Tabte 9.1. Syrnbols nd Molecular weights


C hemie al Therrnod.ynamic s and. Molecule Atom
Elmnts I Goses
Fuels ( C onu entional and Alternaa
e)
I Com.pounds
Symbol Maleular Symbol Molcular
wight uewht
9'1' chemical thermodynamics-Generar aspects-Basic
equations-Theoretical ai and chemistry-Fuels--combusion I{ydrogen H2 , H
excess ait-sbfhioneti-c a-fu f;el (A/E) nti*Air_fuel
anarvsis ofprcducts-Analwis ofexhaust ratio from orypn o2 t2 o 16
fl"; g;;;;;;*"rgy
ln andenthalpyofreaction-
Enthalpv of fomau-on (aHi!-Heating""i"* Nitrogen N2 28 N t4
equilibrim-Actual ombuition ara$r,. tempemture--{hemicl
"ini-l'atLticname
C""*tj";i;ifor
s.z- I.C. .ng-*F_Ihoductie Caon c t2 c t2
Desirable prcperties of gmd r.c' "gi"" r""rs.il.J*-t"r"-Liq"id
petroleum-Petrcleun and compositioi orr-" fuels--structm
oil- il, sparr.-igiioo engines-Knoc*of
Sulphur s. n 32

rating of s'I' engines fuers-Miscellan"our Water Io 18


prop"Ji"r-oil.r. fuel. fuers-Dieser co 2t
9'3' Altemative tuers for I.c' engines--Ge-ne""I "ogio", Cabonnonoxide
altemative fuels-Alcohol-AlJohol-ga"ofi". 6;arAd;;L* md dieadvantage of using
Carbondioxide CO, 4
n gu (rnethae}_
LPG and LNG-Biogas. worked Exa;p-ritlii"5ijxre
"l'l"oa"jiiroe"o_{"tural
rlpe euestione-rhortricar Sulphurdioxide SO, u
Questions-Unsolved Examples. cH.
Marshgas (Methane) 16

Ethyleoe c#{ %
9.1, CIIEMICAL TIIERTUODYNAMICS Ethoe crlt 30

9.1.1. General Aspects


In chemical thermody-namics the study of systema 9.1J. Fuels
important topic' A chemict *""t;;-;;'"""in" involving chemical reactions is an nuclear.Ilere we shall consiler brie{ly clumical fuels only,
as the rearrangernent of atoms due to Fuel may be chemcal oir
redistribution of erectrons..rn a chemical rel.eases-tpat encrgy on combustian.The principal com'
a substoce which
;;;"-lil
frequently used ''Reactants' comp"" ori"iu"i.o""'tiii""trterms, reactana and the pr"oducts are A clumbat fut i,s

*ni.n stu* t"."""uoJ wle,prcdwts, busible elenents of eadr fuel are carbon atd lrydrogen.lbough sulphur ie a combustible element
comprise of final constituents wbich
are ro"-"J ty tu" too but its presence in the fuel is considered to be undcirable.
principles which will b" re basic
t"::lTg ti ti. .f,"pt""'"pply "1"-iJ ""..iioo-lioogr
to any chemical reaction, here main Fuels can be classied according to whether :
attentio will be focused on an irnportant type ;f che;ical reaction-scombustion.. (i) they occur in natue called primary frels or are prepared called secondary fuels ;
9.1.2. Basic Chemistry (ii) they are in solid, liquid or gaseous state. 'lhe detailed classication offuels can be given
Before considering combustion problems in a surmrarr form as follows :
use of chemical formulae.
it is necessary to understand the construction
and
rhis involves Type offuel Natural@rimry) .Preparcd, (tuondarX)
which are discussed Iow bri,efry.
Atoms: "b*;*;;;;cepts SoUn Wood CoLe
r is not possible to divide the chemical Peat Chacoal
can take part in a chemi.car crlonge is
elemen ts indefinitety,and the srIles t particl.e whieh
caileJ n .;t"oi,. rr." , ,plit
Liga,ite coal Briquettes
d,iuided at o nt do e s |zot re ta.in
th;"ris;i- ;iJl*il"u tt '
"t"^
r. ""'ii.jla. ""a"too, " Li4uid Petroleum Gasoline
Molecules: . Ketosene
It is are to find elements to exist naturalry as single Fuel oil
which exist in pairs. each pair formirs;;";;i41"*yg"r,l, atom. some erements have atoms Alcohol
and the atoms of.each molecure are
stronger izer-1.,*ii.r""'-.-in i$i;"" Benzol
l.:l'j:l*h"r,lv of a molecure of oxygen would be tedi-
ous' but possible ; rhe isoration ol,'l
some substances are formed by
urlT or*vs";;;;i;il;;#;:;:'.r"ri;" -"recules or
Shale oil
the mating up'o? atoms of different water
";;.-F;;xampre,

\
.. TNTERNAL COMBUSTION ENGINES
CHEMICAL'I'HERMODYNAMICS AND FUELS
Gaseous Naturalgas peholeumgas
producergu The above equation ofcombustion ofhydrogen tell us that:
Coal gas . (l) Hydrogen reacts with water to form steam or water.
Coke-oven gas (ii) Two moiecules of hydrogen react with one molecule of o:rygen to give two molecules of
steam or water,
:1T"ffi# i,e., 2 volumes H, + 1 volume Or----;2 volumes IlO
sewe gaa . The HrO may be liquid or a vapour depbnding on whether the product has been cooled sufii-
solid fuels : ciently to cause condensation.
The most important sol-id. proportions by r)ass ae obtained by using aomic weights as follows
fuer is coal ad its various types are divided
to thei chemical and physicar p.op"*i"". into groups according
T)he : )
erenents in the fuel is called the ultimate
a' o"" u^i" anarysis by mass, of the mportant 2Hr+Or+2HrO
1lnr*ot
rhe etement .'. 2(2 xl)+2xl6-----+2(2x1+16)
hvdrosen' nitrogen and surphur.
percentages of moisture, volatle
;il;r ""Jt"i; "Jr"llt"." dbeinggives
ql]"u o;;;";;;;"sis,
carbon,
i.e., 4 kg H, + 82 kg Or-----+ 86 kg HrO
m.atter, "?*", (calted fixed carbon), and
the
or
carbon is found as a remainder "o^Ortil" "old
by, d"d;Jirc;;"*"."o*o, osl. lhe fixed I kg H, + 8 kg Or------r g kg HrO
matter includes the water derived from of the other quantities.
gases (.r.
th"-d;oii;i aecomposition of the coar, the The volatile [The same
proportins are obtained by writing the equation (9.1) as:l
\, CHo, CrHu etc.), and ta. combustibre H 2 + f.O2------+ H2O, and this is sometirnes done.
L I
Liquid fuels: It will be noted from equation (9.1) that the total volume ofttre reactants is
Most of the rioui.d fuers ate hvd.ro-carbons H, + l volume Oz = B volumes.
2 volumes
conditions' Petroleum oils ag aqnple which^gxist.in the riquid phase
at atmospheric Thetotalvolumeoftheprod,uctis only2volurres.thereistherefore auolumetriccontrac-
necessary information to the sErrs'r -i*" or"-etimes hundeds of dlfferent fuels, but the tion on com,bustion.
engineer i, th"
tre relatve proportions of C, H2, etc,
ultirnate analysis. ""h;;; as given by the Since the oxygen is accompanied by nitrogen if air is supplied for the combustion, then this
Gaseous fuels : nitrogen should be included in the equation. As nitrogen is inert as far as chemicl reaction is
concerned, it will appear on both sides ofthe equation.
These fuels are chemically the
-_
ranv at armosphei" conltions,
simplest of the tl with one mole of oxygen there are 79121 moles of nitrogen, hence equation (g.1) becomes,
fuels are manufactured by tbe various "."d;;dii,Hlii"fliii;,ili"T:::;:"%','r::lll"1T;
treatments ofcoal. 2H"+ or+ ?*r--- zuro +
a constituent of other gas mixtures,
i,
co ie an important gaseous fuer which is fiN" ...(9.2)
of the incomplete combustion of 2. Combustion of carbon :
9.1.4. Combustion Equations
"o.t "h;;;;ol'uct carbon.
(j) Complete combusion of carbon to carbondioxide
In a combustion chamber proportionate C + Or----+ CO, ...(9.3)
msses of air and fuel enter
reaction takes place, and then tt
muss the mass tow remains-constant " "o-"rtioo
p-li.p"., t" the exhaust. Bywhere the chemical
the conservation of
and including the nitrogen,
(t.e. totaimass Jf products=
t"tJ-*" r"r"*ons), bu the * l?
reactanrs are chemicallv different
ture' The totar number of atoms of
from h;;;;;,; trr" pi"".F1"??" ffg"r rempera_ By volume:
c + or+ fi , -----+ Co, N, ...(9.4)
each ere;"; ;";;"*d in the "i " constant, but
the dtorns are rearranged. into. combustian remains
prou4s hau"g irir"i'"nemrcal propertias.
pressed in the chemical equaon
*fri.f, This information is ex_ 0 volune C + 1 volume O, * trl volumes N, ----+ I volume CO, + lf volum.es N,
(i) the relarive quantities "ro-*.-flj'iiu """"qrr* and the products of combustion,
of th" .;;;;;.'"J'o*r"r- The uolume of'carbon is written as zero since the uolume of solid is neglgibte in comparison
consistent, each having the same The two sides of the equarion
number ofuto, of.u", element must be with that of a gas.
The oxvgen sueelied involved.
is usllv prwided bv atmospheri.c By mass:
r:.::Tbustion
sary tu use accuate and consistent
analysis ofair by"zt s andby uolume.
air, andit is neces-
calculations ro rake air as 2s s% It is usual in combustion 12kcgt.(2-x1.6)kgo, * 14)ksN2-) e2+2x16)kgCo, * H. ex
Oz,i.,;;";; ilfr" d
?r% O2,?97o N, by votume. The small
ffi fzx 14)N,
in dry air ar i".tua"'in'nili'Jin, rvhich t.e., L2 kg C + 32 kg O, + 105.3 kg N2
':;::;"::,."n"'gases ir'ro,n"ti-?."-t,ot*orph*i, 44 kg CO, + 10b.3 kg N,
-+
Some important combustion equations
are given below :
or lkgC+
3uro,.r1#kgN,----+fr.rcor*{f; krr.
1. Combustion of hydrogen i (i. The incomplete combustion of carbon, The incomplete combustion
of carbon occurs
2ftr+ O"= 2f[rg when there is an insufficient supply ofoxygen to burn the carbon completely to carbondioxide.
...(9.1) 2C + Or--;2CO .(9.5)
286
INTERNAL COMBUSTION ENGINES
CHEMICAL THERMODYNAMICS AND FUETJ 287
and including the nitrogen,
A weak mi.rture is one which has an ercess of air.
2c + or* Nr-----rco * N, A rrch mixture is one which has a d,eficiency of r.
By mase :
ff fi The percentage of excess air is given as under :
' - Actual A4lar$9-:toichiometric A lF ratio
(2 x 72)kgC + (2 x 16)kg o,*#,(2 zoage excess ...(9.10)
x t4)kgN,----+ 2(r2 + t6)kgCo *fi<r,14)kg N, Stoichiometric A /F ratio
24 kg C + 32 kg O, + 105.3 kg Nr,-___+ 56 kg CO + 1O5.3 kg N, (where A and F denote ir and fuel respectively).
1.05.3. __ ? 105.3 The ratios are expressed as follows :
1 kgC + L*O, * kgNr-___r Ls CO * kcN,
; 5 ff For gaseous fuels By volume
If a further supply ofolrygen is available then the combustion can continue to completion. For solid and liquid fuels By mass
For boiler plant the mixture is usually greater than 20Vo weak; for gos turbines it can be as
2co + o"*lJ zco, + N, ...(e.7)
f,| much as 30O% weak. Petrol engints have to meet various conditions of load and speed, and operate
By mass: "r---- over a wide range ofmixture strength. The following definition is used :
Stoichiometric A /T ratio
s6kgco +B2kgo,t11# kgN,----+88kgco,* E x,ll, Mixture strength =
Actual A lF ratio
...(9.11)
r:4
o n , ]913 ,__ *, 1l , The working value range between 807o (weak) ad l2OVo(rich).
or lkgCO + 105.3 ,
-
rkg0r* ff kgNz+f tu.-r+-u-Kgr\2. Note. The reciprccal of the air-fuel ratio is called the fiul oir FIA retio.
9,1.5. Theoretical Air and Excese Air 9.1.7. Air-Fuel Ratio From Analysie of ilducts
The mnimum arnount of ar that supplies sufficient orygen for the cornplete combustion When analysis of combustion products is known air-fuel ratio can be calculated by the
of
all the carbon, hydrogen, and any other elements in the fuel tLt may oxidise is called following methods :
the "theo-
retical air'. When complete combustion is achieved with theoretical- air, the products contain 1. Fuel composition known
no
oxygen.
(i) Carbon balance method (dd) Ilydrogen balance method
In
practice, it is found that complete combustion is not likely to be achieved
unless the (iii) Carbon hydrogen balance method.
amount ofair supplied is somewhat greater han the theoretical amount. Thus
150 per cent theo- 2. Fuel composiion unknown
retical air means that air actually supplied is 1.5 times the theoretical air.
The corrplete combustion of methane with minimurn amount of theoretical air and 150 per Carbon-hydrogen balance method.
-
cent theoretical air respectively is witten as : 1. Fuel composition known:
(i) Carbon baldnce tnetlod.When the fuel composition is known, the carbon balance method
cH4+ zo2. r(X)Nr-----+ co, + 2r!o + , is quite accurate ifcombustion takes place with ercess oi and. when free (solid.) carbon is rt,r:,t present
(?) *, ...(9.8)
in the prod.ucts. It may be noted that the Orrat analysis will not determine the quantity ofsolid
carbon in the products.
cH4 + 2o.E) o,., (X) (1.5) N, ---r co"+ 2n"o +o, . a (lf) u, ...(9.e) (i) Hydrogen balance method.lfis method is used when solid carbon is suspected to present.
(with 150 per cent teoretical air) (ii) Carbon hydrogen bolance method, This method may be employed when there is some
The arrount ofair actually supplied may also be expressed in terms ofpercent uncertaintXt obout the nitrogen percentqe reported b1 the Orsat onalysis,
excess air.
The excess air is the amount ofair spplied ovr and abov the
theoretical air.-Thus l5o per cent 2. Fuel composition unloown:
theoretical air is equivalent to 50 per cent excess ai. When the fuel composition is not known the carbon-hydrogen balance method has to be
Note' For conplete conbustion of fuel we need air. As per theoretical basls thue is minimrr employed.
^airwhiclr
of is required by the fuel to bum completely, but ilways, air h lmss is usd,
a amornt
becaw whole d air
supplbd for co-mbustion purposes does not coire-in 9.1.8. Analysie of Exhauet and Flue Gas
ml be-lefr unbum\. But if a large quanity of exess "oiio.t -itn the fuel nmptetely and as such portion of fuel
airis used it exercis es acoolingefutoncombuetinnpruess
uhich haueuer canbe auoidzdby preheating the air, tn",jiio*"ss
The combustion products are mainly gaeeous. When a sample is taken for analysis it is
ar supplicd. canbe dztemired, fronthe usually cooled down to a temperature which is below the saturation temPerature of the steam
yetsht of 97ysen which is teft unused.. Tini amout of exceJs aii suppliea varies wfth the type of tuel and the
firing conditions. It may approach a value of 100% but moilen practice is to present. The steam content is therefore rct inludd in the analysis, which is then quoted as the
use 25% to 507o excess' ai. onalysis of the d.ry produs. Since the products are gaseons, it is usual to quoe the analysis by
9.1.6. Stoichiornetric Air-Fuel (A/D Ratio volume. An ana.lyss which ircluds tle stean in th e:haust s called a wet analyss.
Stoicltiometric (or chemically conect) ni*ture of air and fuel is one that contains.z Pracical analysis of corrbustion producte
st suffi_
cient orygen for cornplete conbuston of the The most common means of analysis of the combustion products is the Orsat apparatus
fuel
which is described on next page.
288
INTERNAL COMBUSTION ENCINES CHEMICAL THERMODYNAMICS AND FUETS
289
Construction. An Orsat s apparatuo consists of the following :
(i) A burette thus absorbed is ead on the scale ofthe rneasuring bottle. The flue gas is
then forced through the
(ii) A gas cleaner pipette 1 fo a number of times to ensure that the whole of the
Cb, is absorbed. Further, the
(lii) Four absorption pipe ltes t, remaining flue gas is then forced to the pipette 2 which contains pyroglli. acid to
2, 3, 1. absorb whole of
or' The reading on the measuring buette will be the sum ofvoiume ofco, and or. The oxygen'
The pipettes ae interconnected by means content can then be found out by subtraction. Finally, as befoe the sample
ofa manifold fitted with cocks s1, sr, s, and
and contain different chemicars t abso s, of'gas is frced through
(or). Each piperte is also fitred
with
*it""irriii" (cor), carbonmonoxide (ib) and oxyged
t,rue, which provide a greater amounr
theipipettes 3 and 4 to absorb carbonmonoxide completely. \
of surface. These tubes are wetted " ";;f";;iili"", The amount of nitrogen in the sample can be determined by subtracting
sis. The measuring burrette is
by th"
"bJlils ancr.are exposedto the gas under anary- of gas the sum of COr, CO and O, contents. \m. total volume
:y*":T* AV ir";", ""Ar, to prevent, changes in temperature
and density of the gas. The pipettes "
r, 2, g, atntain-te rollowing che'icals :
orsat apparatus gues an anarysis of the d,ry prod,lcts of combustion. steps may have been
Pipette I : Contains ,/lOIl' (caustic s oda) to absorb CO, (carbondioxide) faken to emve the steam from the sample by condensing, but as the sampru is collected
over
Pipete 2 : Contains cz alkalne.solution ,pyrogalc water it becomes saturated with water. The resulting analysis is nevertheless a true
of d, to absorbO, tor.yg"rrl analysis of
Pipette B, 4 : Contain ao a.i.d solution of ,";p;o; the dry products. This is because the volume readings are taken at a constant temperature
and
chloride, to abso C (carbonmonoxide) pressure' and the partial pressure ofthe vapour is constant. This
means that the sum of the partial
has a lzvettins ioute and athree way cock to connecr rbe appa- p.ressu-re: ofthe remaining consituents is constant.
""rr. "il[:'"n"J::: :xf:|ffi.H The vapour then occupies the same propor-
tion of the total volume at each measurement. Hence the vapour does not affect the result
of
the analysis.
Note. Quantitatively the dry product analysis can be used to calculate Al8 ratio. This
.
ing the Aff ratio is not so reliable as direct measurement ofair consumption
method ofobtain-
and fuel consumption ofthe engine.
More caution is required when malysing the products bf commption of
a solid. fuel since some of the products
do rot appear in the flue gases (e.g ash ad mbumt.carbon).
The iidual solid must be analysed as well in order
to determine the carbon content, ifany. With al engine using petrcl or d.iesel
fuel the exhaust may include
unburnt particles ofcarbon and this qumtitywill not appear inihe analysis. The exhaust
from intenal combus-
tion engines may contain also some CH, and I! clue to incomplete combution. Alother piece of equipment called
the Heldane apparatus measures th CH, cntent as wellis
Ce, O, and CO.
9.1.9. Internal Energy and Enthalpy of Reaction
The first law of thermodynamics can be appried to any system. Non-flow and
steady-flow
energy equations Ceducted from this law must be applicable to systems
undergoing combustion
processes.
It has been proved experimentally that the energy released, when a unit mass of a fuel
undergoes complete combustion, depends on the temperature at
whch the process s carried out.
Thus such quantities quoted are related to ternperature. Now it will be
shown that if the energy
eleased by a fuel at one temperature is known then it can be
calculated at other temperatures.
The process ofcombustion-is defined as taking place from reactants at a state idcntified
by
the reference temperature ?o and another p.op".ty,-"ith"" pressure
or volume, to products at the
same state.
Let uh = Internal energy of the redctants (which is a mixture of fuel and air) at rn,
Upo = Internal energy of prod,ucls of combustioir at ?6,
Un, = Internal energy of reactanfs at temperature ?r,

U, = Internal energy of prod,ucrs at temperature ?1,


Fig. 9.1. Orsat apparatus.
Procedure. 100 cm3 ofgas
U& = Internal energy of reactanfs at temperature Zr,
whose anarysis is to be made is drawn
the levelling bottre. The stoo cock,s, into the botre by lowering
is then p"""J"" trr" whole flue gas U. = Internal energy of products at temperature ?o,
The gas remains in this pipette is forced to pipette 1.
forsmetime.itJ-"J"itrr" cabondioxide is absorbed.
Iing bottle is then lowered- to allow te The level- AUo = Q6st volume heat ofcombustion,
cemi"at io.o-" to its original level.
The volume of gas Q = Heat transferred to the suroundings during thc process, and
W = Work obtained during combustion process.
290
INTERNAL COMBUSTION ENGINES
CHEMICAL T}IERMODYNAMICS AND Ft'EtS 291
Analysis for a non-flow process involving conbustion at .constent
volume, : It may be noted that in case of real constant volume combustion processes the initial and
when the combustion process is carried out at nstont uorune thenthe
equation, Q = non-flow energy final temperatures will not be same as ?o (reference temperature), The change in internal energy,
@z- Ur) + W, can be applied to give
for analytical purposes, between reactants at state 1 to products at state 2 can be considerecl in the
q = (Up" -U4) following three steps (stages) :
...(e.12)
where, W = 0 for constant volume combustion (i) The change for the reactants from state 1 to ?o.
Ur= u\ (ii) The constant volume combustion process from reactants to products at ?0.
(Jr= upo (iii) The change for the products from ?i to state 2.
The entire process can be thought of as taking place in piston-cylinder device as shown in
. The internal energy change is independent ofthe path between the two states and depends
only on the initial and nal values and is given uy the quanuty Fig. 9.4.
p:olertv diagram of Fig. 9.8. The heat so transferred Q. This is illustrated in Fig. 9.2 .nd
cdleithe "irr-l iirg-ireombustian at Internal energy
To (or constant uolume heot of combustian), and
is denoted b AUo. Thus, (D (iD ()
,r,.--_-..}- U"o - U""
LIJ|= upo -u4 ..(9.r3)
U"o

is a negatue quantity since the internal energr ofthe reactants o --t-


, . {%enerry
chemical and heat is transferred frorn the system.
includes the poential --}- +
I

Roactants at Products at
To and Vo

Fig.9.4
xxxxx
xxxxx Thus , - Ur, the cbange in intemal enrgr between sates 1 and 2, can be witten as
(Up" -Ur) to show the chemical changes involved and this can be furtber expanded for analytical
purposes as follows:
Upr-Unr=(Up, -Ur)+ (ilpo -U4) +(U"-Ur)
F'rC.9.2 U p" - Unr= (Up - Upo) + + (U 4 - U r)
t.e. ...(9.14)
^Us
Products Seactants
(ii (rr) (r)
The values of (Un - U,)and (Up, -Upo) can be calculated from the following relations :

uh-uo, =Inifuio-ur,) ...(e.15)


where, z, = Tabulated value ofthe internal energr for any constituent at the required temperature
To or T, in heat unit per mole,
= Number of moles of the constituent, and
\- = Summation for all the constiuents ofthe reactants denoted by l.
)
x
Ifass base is qsed for tabulated values or calculation. then
Fig.9.3 U\-Ur,=Z^"i,-+,) .(9.16)
I

l
I

\
292
INTERNAL COMBUSTON ENGTNES
CHEMICAL THERMODYNAMICS AND FUEI.S 293
where, u, = Internal energy per unit mass.
in terms of tke specific h-eats(average It may be noted that nCo= nicpi,
t"." .rll;*lf,o"Jl}ssion values for rhe required tempera-
From the definition ofthe enthalpy ofa perfect gas
u\ - u r,= (q
- Tt)\r+c,; H=U+pY=U+nRoT
I,"r",,r{z - A) = So if we ae concemed only with gaseors mixtrres in the reaction then for products and
For products,sirnila" exp*ssious '..(9.17) reactants
may be written aJ
pr-Up9=)n(u H po =U po + nFTo
-uo)
p . .., -n male basis and Hn"=An + nf.To )
where, nr and n" are the moles ofproducts and reactants respectively and the temperature is the
u r, -a ro =\nfu - u") ... on reference temperature ?0.
mass basis
Thus, using equations (9.13) and (9.19), we have
Ur" -Upo =lnrc,fl.2-Ts)
AII'= LUo + (2" - n")Eo?o
If there is no change in the nurnber of moles durng the reaion or if the reference tenTperd.ture
\-r is absolute zero, then AII o and. LU o will be equal.
ez-T Lmicui ... in rerms of mean specific heats
It may be noted that nCvi mf 9.1.10. Enthalpy of Formation (AII')
= o,
Analysis for a steady io, o"-to"t.rrt A cornbustion reaction is o particular kind, of chemical.reaction in whch produzts are formed.
pressure, combustion process
In such an analysis the changes n entnabi : from reactants with th release or absorption of energy as heat s trattsferred, to and from the sur-
above will give the following urp.essioos tt):are importaut. An anatysis carried out as round,ings.In sorre substances like hydrocarbon fuels which are many in number and complex in
skucture the heat of reatton or combuetion may be calculated on the basis ofknown values of the
Hrr-ERr= (Hpr-Hpo) + AIfo + (HR'-H") enthalpy of formation, AI, of the constituent of the reactants and products at the temperature ?o
(9.18) (reference temprature). The enthalpy offormation (^fr) is the increase in entholfi whn a con-
pound is forrned fron its constituent elements n their itatural form and in a standad state. Tir.e
Products Reactants standard state is 25'C, and 1 atm. pressure, but it must be borne in rnind that not all substances
where, Arlo = Hpo - Hao, and is always negcjue can exist in natural form, e.g. HrO cannot be a vapou at 1 atm. and 25'C.
...(9.19) T?re expression ofa particular reaction, for calculation purposes, may be given as :
u.n*Tl:T y change of enhalpy o;""*;;;fffiil":J:
S[Ji:i:l{":r!::#i::':^'^\::'^::"":o*ssureheatorcombustionatl uto=\nir, .Ln,*r, ...(9.25)
Reactants: P
Tlpical values of AII, for different substances at 25"C (298 K) in kJ/mole are given below :
Hao - H*r= )rrr{Ao -i;r) ... on mole basis ...(9.20)
S. No. Substance Formula State Nrl
H*o - H*,= ,) ... on mass basis
)zr(6 - ..(9.21) l. Orygen
fo Ctr 249143

H*o Lo, Gas zeto


-Hr,=|nprlrs-ry) 2. Water HrO Liquid -285766
Vapour -24L783
= (To - T t) lr*," 0r Carbon C Gm 7L4852
... in terms of mean specfic heats
P Diamond 1900
Products: Graphite zeto
H ,, - H ro =\ni&i, - hro) ... on mole basis
...@.22) 4.' Carbon monoxide co Gas - 111508
D. Carbon dioxide cot Gm - 393443
H p, - H po =l*iTi" - hro) ... on mass basis ...(9.2s)
6. Methane cHo Gs -74455
Methyl alcohol cH3oH Vapour -240532
Hr"- Hro=\npo,Q2-rs) 8. Ethyl alcohol c2H5oH Vapour -287102
9. Ethane cJla Gas - 83870
10. Ethene c'Jd.o Gas 5r780
= Q2-fo) in terms of mean spectfic heats 11. Propane cHt Gas - 102900
1^*rr... L2. Butme cnHro Gas - 125000
...19.23 (a)l Octme csHtt Liquiil -247600
294 INTERNAL coMBUsnoN ENctNEs CHEMICAL THERMODYNAMICS AND FT.'EI.S

9.1.11. Ileating Values of Fuels


Ifa fuel contains hydrogen waterwilt be formed as one ofthe products ofcombustion. Ifthis
water is condensed, a large amount of fuat will be released
than if the water exists in the vapour
phase' For this reason two heating ualues are defined; -Thr|omtar
the higher orgross heating valueand the
lower or net heating ualue-
higher heating value, FEN, is obtained when the water
pletely condensed. formed, by cornbustbn is com-

The lower heoting varue, LI{v, is obtained when the water formed by combustion exists
completely in the uapour phase. Orygn valw

Thus: (IIHVL = (Ll:fy)o+mhrr ...(9.26)


(HHV), = (LHV), + m(u, ur)
- ...Q.27)
where, m = Mass of water formed by comustion,
/zr" = Enthalpy ofvapourisation ofwater, kJ/kg,
c= Specific intemal energy ofvapour, k l/kg, and
Relsase valve
f = Specific internal energy of liquid, kJtkg.
In almost all practical cases, the \rater vapou in the products is vapour, the lower
-
the one which usually apples.
value is

Fuel Calorimeters:
heating value of a fuel is defined as the quantity of heat transferred
in ord'er to reduce the temperature ofthe prod.ucts ti the initint from the calorimeter
reaction tenrperature. Heating values
are reported as positive quantities and are used wiilely
in the calculation ofthe thermal efEciencv Fuse wlro
of power systems.
TWo types offuel calorimeters used for the determination
ofheating values are :
1. Bomb type calorimeter Cotton
2. Junkrs-type calorineter.
Sealing water
Bornb-ype calorimeter. Refer Fig. 9.5. The bomb-type calometer,
a constd-vorume
slstgm' is initially charged with oxygen and a small sample oi fu.l.
Sob."quent to ignition and
combustion, the heat is tansferred from the products
to a surrounding water bath. Ihe heating
value is calculated essentially from the
-easured temperature increase of the system msss. The
calculated result is usually reduced to a standard heiting
ualue ot 2s.C. A heaiing value deter-
mined in a bomb-tpe calorimeter is clesignated as
constont-uolume higher heating ualue.Watcr Bhsostat
vapour formed during the reaction is completely ^
condensed especially when a few drops of water
are placed in the bomb prior to sealing in order to saturate Fig. 9.5. Bomb calorimeter.
the gaseous atmosphere.
rfunkers'type calorimeter. Refer Fig. 9.6. Junkers-type calorimeter is Notc Although the constatrt pressurs heatingvalue autl the enthalpy of comhstioa are dweloped frou
designed to burn a
gaseous fuel under a stead'y somewhat difierent colcepts, thcm ir a general eimilarity betweon these two terrns. Whe the draracteristics of
flow cond'ons it atmospherb pressure, Ireat is transferred from the
products to_water flowing teadily through the reaction are identical, ttre contant pressure heating value and the corresponiling enthalpy ofcombustio
the outer jacr.* ott ."io"i-"; ft" ;;;r;"*
*.: adjustql to obtain a gas outlet lemperature-equal to the" inlet temperature value will be numerically equal but of oppoeite sign. A correcpondiag similarity exists between the consto,nt'
l'-"":
combustion air' From observed water temper'atues
of the fuel and voluttu heoting aolue and the hbmdl energ of combustian.
and measured quantities of fuel and jacket
water, the heating value is calculated and reduced to the
correspondlng value i- zS.C operation.
Some of the water vapour in the products
condenses and drains from the calorimeter into a collec-
ing vessel' This measured quantity ofcondensate
is used in the subsequent conversion ofthe calori-
metric heating value to the constant pressure higher
and lower heating values that are based,
upon complete and zero condensation of the water vapour formed
i:^"||c^'1":.l", during the combus-
ton reactton.

\
TNTERNAL COMBUSTION
ENGINES
CHEMICAL THERMODYNAMICS AND FUEI,S 29't

9.1.13. Chemical Equilibrium


The calculation of the a.d.iabatb flame temperatzre is based, in part, on the assumplion .that
the reaction goes to completloz. Owing to dissociation, complete conversion ofthe reactants to the
products is not accomplished. As a consequence of the failure to achieve cornplete conversion of the
reactants, the maximum reaction temperature cannot attain the level of the theoretical adiabatic
flame temperature.
The cornbination of CO and O, produces CO, together with a release of energy. In an adi-
abatic system no heat is transfened to the s.urroundings, hence the temperature ofthe mixtue of
the products and reacting substances rises rapidly, As the mixtue tmperature increases to higher
levels the rate of dissociation bfthe CO, becomes increasingly more pronounced. Since the dissocia-
tion of CO, requires absorption ofenergy, a condition is reoched, where tlrc rote of evolutinn ond the
rate of absorption of energy are in balance. At this point nn further ncrease in tcmperature can be
obserued. and the reaction is in chemical equIibrium. For this condition
C+Or:CO,
At each temperature ofthe equilibrium mixture the substances participating in the reaction
exist in. unique proportions. For the combustion of CO thd right-hand side of the equation

CO+ j or=(1 -) COr+xCO+ f, O, ...(9.28)


represents the distribution ofthe equilibrium products resulting fom a reaction between CO and
Or. In this equation denotes the fraction ofdissociated COr. At low temperatures the fraction (1 -)
approaches unity while at high temperatures (1 - ) shows a substantial reduction irt magaitude.
For the combustion of H, with O,

Condensat n"+ $ or-+Hzo


Fig. 9.6. Jun_kers gas calorimeter.

9.1.12. Adiabatic Flame Temperature


Hr+ | or= (1 -) Hro + r ur+ f, o, ...(9.29)

In a given combustir It is essential to distinguish between the effects of dissociation and the losses resulting from
in.kinetic ;,;;;#;ll#ffi;lll':;:"i:,:::"ftabaticauy and wth no work or changes
of the products is referred
incornplete combustion of fuel. Incomplete combustion, which may be attributed to a nurnber of
factors, results in a discharge from the system of combustible substances. Dissociofio, on the
'a1iab?t: flome tempe,atuJ
potential energy,
wr ,iu'u.il;;ffi::I| work,ad to aslhe other hand, is of transient noure. Usually any appreciable degree of dissociation extends over a
this is the mamum tenperature no changes in kinetic or
because anv heat transfer rt lo very short time interval at the highest level oftemperature attained in the reacrion. The gaseous
rrom rhe "lt and for the given reactanrs products are likely to be discharged from the system at a temperature that is indicative of a low
tend to lower the temperature ""J;; ;;::t"t::o:i"lt"ved
any incomplete combustion
oftheproducts would degree of dissociation. For example, dissociation does not nfluettce the heating ualue determined, n
The following points are worthnoting: a fuel calorimeer. Although the maximunr temperature attained. in the calorimeter is limited by
(i) The maximum chemical equilibrium, the combustion process moves to completion with the decrease in the tem-
te
with the type of reaction through ad'iahatic cornplete combustion perature ofthe products. The reduction in temperature is a result ofheat transfer to thejacket
?c.lieved uaries
"::o:::'"': water. Dissociation of the products is negligible at oom temperaure, which is essentially the
Anncrease,",o""r.,7;"::,:::::i;:;r"arsupptied calorimeter reaction temperature.
.. . (ii)For a given fuer and given_pressu""
adiabatic flarne temperd'ture thal r'o ;;;i:"X:r:"::::?::;:tlJr^r^ The temperature ofthe products dischargeil from the combustion charnber ofthe gas tur-
can ;" bine power plant is limited to approximately 8?0'C by introduction of a large quantity of excess ar.
(ttt) The adiabatic flame ";i;;";; ;;it o,stoichiometc, mixture.
temperature can be controued Absorption of energy in the water walls of a boiler furnace limits the outlet gas temperature to
by the amount of excessolr that approximately 1100'C. The quantity of dissociated products at temperatures ranging upward to
:;?;,tr j,:y,ffi *rthemaximum.p"-*,","tempera- is
1100'C is not appreciable. In the cylinder ofI.C. engine, considerably higher maximum tempera-
i:?,fi"T.1i"1Tl";;]*,::;,fff
ture ofthe
prod"nt i urbine, and close control ofthe tenTpera- tures-that is, in excess of 1100"C are attained, hence in the analysis of this thermal power system
"tt""ir, consideration rnust be given to the effects ofdissociation. Ofparticular sigaificance is the effect of
reduced maximum temperature on the system availability. As a result ofheat transfer and work
performed by the gaseous medium the products are discharged from the system at a temperatlrre
below the level at which an appreciable degree ofdissociation is observed.

-+
298 AND FUEIJ 299
NTERNAL coMBUsTroN ENGINEs CTIEMICAL THERMODYNAMTCS

The proportions ofthe. dissociated pmducts


in chemicar equilibrium at temperature ? are 9. Low deposit forming tendencY'
established from the eoulibrum-r"l"rr.
r"-"riuuiio" ortrru equilibrium constant is achieved 10. Economically viable in very large quantities'
in accordance with the analysis presented by
V;,t ;i:
11. Easy mixing with air and low latent heat of evaporation (, )'
Actual Combustion Atrb/dg
9.1.14.
12. No chemical reaction with engne components through which it
flows'
In evaluatingthe performance ofan actual combustion
parameters can be defined dependiug process a number of diffeent
on the natuie of the process trr"-"y-"-- considered. 9.2.3. Geseous Fuels
The combustion efficiency in a gas turbine
""
forinstance can be defined as ThesefuelsareusedinS.I.engines.Thediffeentgaseousfue]sareenumeratedanddis-
cussed below

'mbutio
=ffi ...(9.30) 1.
:

Natural gas 2. Manufactured gases


3. By-product gases 4. Sewage sludge gas
where, (F/A),uol
= Fuel-air ratio required for adiabatic and comprete
combustion
-- and in which 5. Biogas.
the products would attain the adiabatic flame
temperatu;". --
In case of a sea m generator (boihr) 1. Natural gas:
HSt huftT"a to,"t"t- oltcompositionvarieswithsourcebutmainlyitcontainscH(75to95'percent)and
kg ru"1
reteao semrator = remainingCrlI"andNr.Fromsomeareas,thenaturalgasobtainedcontainsH'Swhich
trrgner heatng value of the fuel ...(9.31)
is much harnful to the engines'
In case of an intemal combustion engirc,
o It is available witb oil wells and is colourless and od'ourless'
It is found in several parts ofthe world but particularly in u.s.A It is also carried from
*-" ==_Er-
[Ieating value ...(9.32) the place of availability to the place of use through hundreds of kilometres
pipeline'

9-2. COWE\ITIONAL FUEI,$ (FOn


2. Manufactured gacee:
LC. ENGINES) The gases are nanufactured by various methods, discussed briefly below
:

9.2.1. Introduction oCooilgasismanufactured'byheatingsoftcoolincloseduessel.Thecontentsofthegas


o I'c' engines depen opon the type of coai and method of operation used in manufacturing'
can nD on difrerent kinds of fuel, including
fuels' Ihe properties and the character orlie ruet
liquid, gaseous and even solid A clean coal gas contains ; SSVoF{rand 66Vo CH'
exercise profound influence on the - Its energy cont erit is 50 percent of n'atural gas '
design, power outpu! efrciency, fuer consumption and the reriability
'the engine. and durability of
o'Watergolssfurmed'byusingsteam'Foritsmanufacture'thewaterandoirorepassed'
o The use of solid ?eJs preseut problems of
' alternateQ throu.gh a bed of hot corbon'
complicaed injection systems, as welr as
difficulties associated with so'residual *,
* hence are not popular, gaseous fuers It contains Hr, CO and Ne.
present problems ofstorage and-handling
oflarge volumes. Hence for mobil . 3. By-product gases:
gets restrited,. But gaseous fuels do ";;;;r-;;
fi ue fr stutionary pii", p""t, p";ri;;; o The gases produced during manufacture of other substances ae tnowu as
by-prod,uct
when gds s read'il1r available at tlu location
nearby. Thus uquia fuel fini abundant use goseg.
in I.C. engines. N,' It contains
a Btolst fun@ce gac is a by-product of steel plank' It contains CO and
9.2.2, Desirable properies of C,ood LC.
Engines Fuels . large mount of ust partics ; therefore, it should be cleaned by an effective method
The fuels used iu i.c. engines-are designed before its use in the engine'
to satisft the performance requirements of the
eqgine svstem in which ther.ar
Ted.
rur nl"irmJiiorru* *e following properties : 4. Sewage sludge gas:
1. High energr density (lil&g).
2. Good combustion qualities.
r It contains CHn and CO, with very small percentage of HrS'
8. Figh thermal stability.
o This gas is made available from present well developed sewage ilisposal plants'
4, Low toxicity. 5, Biogas:
6. Low pollution. o This gas is produced from the cow dung which is available in large quantities in India'
6. Easy transportatioir/transferability o It is easy to produce (with appropriate chemical reaction) and use locally'
and sorage.
7. Compatibility with the engine hardware. Except natural gas, all other gases mentioned above are generally employedfor running I'C'
8. Good fire safety. ii used locatty-to run clifferent types of equipment s like small electric genera'
""gi"u.-*-r"-"r
torc, PumPs etc.

-t
i
I
3oo
INTERNAL coMBusrroN ENcNEs
CHEMICAL THERMODYNAMICS AND FUEIS
Advantages of gaseous fuels :
(i) Easily compressed anil stored. . Naphthenzs C,Hu
(ii) Easily carried through pipes. 4. Aromatcs C"IL"-o
(r'jj) Easy starting of engines. Within ea group also, he physical properties ofindividual cornpound differ according
(iu) Er" to maitain AF to the number of carbon and hydrogen atoms inthsmolecule,
ratio in rrurti-crinder engines, as
compared to riquid fuers. The physical differences between cornpounds, even in any group, influence the way fuel
Dieadvantages: - evaporates and hence the formation of combustible mixture.
(l) High cost (onthe basis ofenergy
content) The difference in chemical properties ofhydrocarbon from different g\oups affect the
(li) High purifting cost. - courbustion process and hence the proportions offuel and air requiremetrts.
(ll'i) Storage volume per unit
energr very large. 1. Paraffins
to engines using liqui fuel-s, the
size and weight of rhe engine (i) Stnght ctnin or nonna.l panwffinc:
.""r,ul'll# rt#L*ed (kglkw) is
(u) The cost (capital and
o It consists ofa straight chain molecular structure as shown in Fig. 9.?.
running) ofthe plants manufacturing o The names ofhydrocarbon in this series end with az as in methane, propane, hexane
gases is considerably high.
9.2.4. Liquid Fuels etc.
Following are the three principal commercial r The straight chain paraflins are saturated compounds as valenry of carbon is fully uti-
types of liquid fuels : lised and therefore, they are very stable.
1. Benzol ; 2. Alcohol , t. Refined products ofperoleum.
Benzol, H HHH HHHH
1.

' I*H:filtf;":tr,"t".*],ild toulene (c,Hr) and is obrained as a by-product orhigh


l__
H-C-H H-C-C-C-Hllr rrrl
H-C-C-C-C-H
anti_kock quatity. As compared ro gasoline,
_L
H lrt
HHH tttl
HIIHH
;. ,:::f"T:.* irs hearing value is low. Methane (CH.) (CrI\)
z-Propane Butane (C.H,o)
r It has good anti_knock qualiies.
o Its heating value is low as compared HHHHHHHHHHHHHHH
It is more expensive to produce.
to gasoline.
llllllt
H-C-C-C-C-C-C-C-H tttttttl
H-C-C-C-C-C-C-C-C-H
o
r
It is used as fuel blended with gasoline.
It can be manufactured from grain, sugarcane
_L_L_L-L LL|
HHHHHHHHHHHHHHH | | | I | | | |

and waste products.


3. Rcfined products ofpetroleum : n-Heptane (CrHr.) Octane (CsHrs)
o It is the main source ofliquid fuels for LC. Fig. 9.7. Straight ain paraffin.
o It is used in the form ofgasoline, kerosene, engines. (i) Brunchd ehon or iso-paruffins :
and
- diesel oil.
The liquid fuels ae classified in t*o gro,rp", o The carbon atons are bratrched in these compound,s.
vaporised' iasilv : "Petrot' and "Atcohot'. rhese are commonry o Branched chain or iso-paraffins have an open structure which is branched as,shown in
"'?J!^'8|1:":l#":" Fig.9.8. Iso-octane, triptane etc. are examples ofthis type.
(ii) Liquid fuet which is directly
iryjected. in the conbustion
chamber :,,Diesel or fuel oil,,.
H H HH
9.2.5. Structure of petroleum
In I'c' engines the fuels which ae usually H H_C-H
I
H H_C_H
I tl
H-C_HH-C_H
fuelsareobtainedbyrefiningpetroleum. J used
Basically, petroleum is a mixture ofhydrocarbons,
ar e cornprex mixtures of hyd,rocarbos,
----''JcuutPtexntuturesofhydrocc These
_C
I I rtl
c_c_c_H
H

H_C
H
I ttt
c_ c_
H

c_H
and' hvdrogen atoms; it may also.""t"i;;;;;;rL"iiti". compounds rzode up excrusiaely of carbon
t;i::T' -"i"ii compounds having surphur, H
I -c-
H_C-H
I trl
HHH H
I lll
H-C_-H H H
lTf "ffi:i :1":"ff : l3:,ff ""-p"u
nd"rother
s s uch ..
".i'iiu* van ai u,n, I -
"trt;, "r H H
I
The constituents ofpetroleum are crassified.into ihe folrowing
four main groups : Iso-mtane Triptane
Constitute General formula
l, paraffins Cn*zn * (where is the Fig. 9.8. Branched chain or iso-paraffms.
2. olefins z- !E'u'rucr
number or
of can
carbon atoms)
c^H;;; o Iso-paraffins are also stable compounds and, highly hnock-resistant when used as s.I.
engine fuels.
INTERNAL CIf,MBUSTION ENGINES CHEMICAL THERMODYNAMICS AND FIJEL-S 303

2. Olefins H H
o These are cornpounds wh-one or more doubre
bond'd, carbon atoms in straight chain, I I
The names end with ne fo one doubre bonJ
lil'ai"n, for two double bnds. The H H-C-H H-C-H
examples are : Hexene aad Butadiene (FiC. g.g) I I
nI _c t]
HH /:, /\ /"\
H_C- C_H
H
+ FF HH HH H_C- C_H
t H_C
ttl
C_ H
ltl
H-- H-C- C- H
I

-c -f- =-" L =- =-'' H_C C- H H-C C-H


+t H
I
H \./C \,/c \./c
Hexene, CrHr, (mono-olefn) Butadiene, C.H, (diolefin) H
I
H
I
H-C_H
I

Fig.9.9. Olefins. I

o The general formula fot mono-olefines (single H


bond) is CoI!, and for diolefins(double (c) Benzene (C.H) () Toluene (CrH) (c) Xylene
bond) is C^Hu_r.
Diolefins are ,nore unstable thon mono-olefins or C.H.CH,
o olefins are present in cracked.gasorine. They form gummy
deposits as they are readily
Fig. 9.11. Structures ofaromatics.
oxidised in storage. llerefore, thei p"r*.rt g."
used in gas turbines.
*"iept low 0ess than 3%) in the fuels In most unaltered gasones, both benzene and toluene are prese4t to a modest
extent.
3. Napthenes
Following are a few specia,l propertics of aromatics :
o These are ring structured compound,s,
(i) Offer highes resistance to knocking in S.I. engines.
r The chemical formula for-the9e compounds is
the same as for olefins, C.H,- but have (ii) Suitability of these fuels for C.L engines is just rwerse of their suitability for S.I.
each carbon atom joined by single bond to t"ro
oirr"" c
structure. The examples, (Fig. 9.10) are : Cyclo-propane
;;";;,fi;;;Il,', iir"* engines.
"r
fCrff.l,'Cycfouuiri" ?C]Hrl *". Therefore, 'paraffins" d,re rl,ost suitable fuels for C.L engines and "aromatics" are
most suitable fuels for S.I. engines.
HH
ll
H-C-C-H
(iD With the increase in the number of atoms in the molecular stmcture, the boiling tem-
perature of fuel generally ircreases.
tl
H_Q_C_H
(iu) As the proportion ofllr-atoms to C-atoms in the molecule increases the calorific value of
fuel increoses. Thus, paraffins have lower calorific value whereas aromatics haue highest
tl
HH
calorifu ualue.
9.2.6. Petroleum and Conposition of Crude Oil
Cyclo-propaae (CrH) Cydobutane (C.H)
Petroleum is a dark viscous oily liquid known asroc oi'(In Greek,pera-rock,oleum-
Fi9.9.10 oil). It is formed from the bacterial decomposition of the remains of animals and plants
napthenes hsve the same formula as for olefins, which got buried under the sea millions ofyears ago. When these organisms died, they
the properties are radiaolly
HHi::: sank o the bottom and got covered by sand and clay. Over a period ofmillions ofyears,
o The napthenes are satumted compounds whereas these rernains gt converted into hydrocarbons by heat, pressute and catalytic action'
olefins are unsaturated. The hydrocarbns formed rose throgh porous rocks until they were trapped by imper-
4', Aromatics
vious rocks forming an oil trap.
a tCo_too).!3ve a ring type srructure for alt or mosr of rhe
T:: :?lLltd
aroms' t0 whcb are attached H or group of c and carbon Natural gas is found above the petroleum oil trapped under rocks. The crude petroleum
Fig. 9.11.
H atoms ; the exampies a""
"rrro
i' is obtained by drilling a hole into the earth's crust antl sinking pipes into it. When the
pipe reaches the oil deposit, natural gas comes out with a great pressure. After the
Inallaromatics,a-leuene,(cdr)morecureisfsos
centrarstructure andotheraro_ pressure has subsided, the crude oil is pumped out ofthe oil well. flrisprocess of obtain'
matics are formed by replacir-g ne or more of *re
nyarogen atoms ing crudc ol from ts sources is called mining.
\ molecule with an organic radical such as pa"affns,
(CI[) ; benzene is converred to ior"""u'fcuu6.Hr).
"pq"t".
"i;;;;;""
and orefins. By adding The crude oil s a mxture of hydrocarbons such as alkanes, cycloalkanes and aromatic
l:,"jYl,f:lt
bonds make aromatics very active and theretbr
Th;;h; lii1" hydrocarbons. It also contains a number of compounds having oxygen, nitrogen and
e they atehgiry""nrnrot"ioiorl"r. sulphur. The actual composition of petroleum depends upon its place of origin.
3A
INTERNAL COMBUSTION ENGINFS
CTIEMICAL THERMODYNAMICS AND FUEI,S
Fractional distillation of crude oil : JU)
The crude petroleum obtained by mininq
is a dark coroured viscous liquid called crude except-asphalt vapourise. As.the mixture ofhot vapour
oil Before petroreum can be for;tfililp"r rises in the column, it cools.
ous components' Th process^use it must be separated into vari- fnergfoS, the component with the highest boili;; poirrt tiqo"f"s first and is col_
*"-" --
of separatng petiteun ito useful
frutions and, remoual lected. Then a little, higher in the column, the
of undesirabte irnpuriiies x ,"itr'ienli;;* boiling point liquefres and so on. The residual g""*"i'po""nf
having slightly lower
is carried out by rbe process of ftacrional *hi.h do not ndJnse escape
lffi;:filgfiperroleum disriuarion s from the upper part ofthe tower. The
fractiois are separated, at different boib)g
points and' are thus collected' ot d.ffirent heights in
the corumn. Tire importan't
fractions ofpetroleum refining are given in tale g.Z.
Bell cap \
Table 9.2. Differen fractions ofpetroleun refining.
\blalile gasss Fraction Boling range epprcrite (Ises
GO (K conpostian
1. Gaseous 113 to 303 Ct{ As gaseous fuel, for producing
mbon black md is also used
Gasolne for preparing ammonia, methyl
alcohol and gasoline
2. Petroleu ether 3O3 to 363 c14, As a solvent for oils, fats, rubber
or ligroin
Kerosene ol and also in dry cleaning.
3. Gasoline orpetrol 343 to 473 ci-4p
(Sp. gravity
Mainly as a motor fuel.
= 0.? to 0.8)
Dieseloil 4. Keosene 448 to 548 crr{t As an illuminant fuel md for
(Sp. gravity
= 0.8 to 0.85) preparbg petrcl gas.
5. Gas oil, fuel oil, and diesel 523 to 673 cr{rs In fumace oil, as a fuel for dsl
Lubricating oil oil
engnes and also in cracking.
Lubricating oils, greases 623 and higher C,, and higher Used mainly as lubricants.
and petroleum jelly
Paraffin wu melts between Cro and higher Used for manufacturing candles,
Cruds oil 325 to 330 waxed parem md for water
proofmg.
I
8. Petoleum coke residue Cro and higher Used as artificial asphalt, fuel and
Steam---- also in making electrodes.

9.2.7. Fuels for Spark-Ignition Engines


Fig. 9.12. Refining ofpetroleurn. Gasoline, a mixture,ofvarious_hydrocarbons (such as paraffins,
aromatics) is the maior fuel used for S.I. engines. The olefins, napthenes, and
The refining ofpetroleum is done in big refineries. The first step in the refining co-position depends opo" trr" souce of
crude oil and the natre ofrefining process.
process is neutarisation of crude
oil by washing wit acidrc or basic sorution as The following are the requirements of an ideal gasoline
needed. :
1. High calorific value
Then the oil is heated in a funace to about
625 K and the vapour thus obtained ae 2. Knock-resistance
intoduced into a fractionating tower. t"
to*"" r"-i"ided into a number of com_ 3. Easy to handle
partments by means of (trays) having holes (Fig. 9.12). The
l.reJve_s.
ered with bubbre caps which holes are cov_ 4. Easy availability at easonable cost
a'ow t-rre ugnteirrore-volatile components to pass
the column whire the heavier,-less volatir"e up 5. Quick evaporation (when injected by carburettor in the
comp-orr"nt" and flow into trays curent ofair)
below. Each shetf is provided with^an "*a"rrse 6. Clean burning and no deposition ofthe esidue.
o"."fl;; ;i;" which keeps the liquid to a.
certain level and a'ows the rest to flow 7. No pre-ignition.
down to ti lower sh erf. Therefore, during
fractional dist,tation, the wth tower loilrg pont, rise up the tower and. 8. No tendency to decrease the volumetric efliciency of engine.
-fractions
cond'ense at d'ifferent reuers rrepending
upon the boi-ne por". pi-",.r.pi,1i" 9.2.7.1. Votatility
crude oil js fed at the base at about eZS f et
this temperature all the componenrc o "Volatility" is commonly defined as the euaporating tend,ency of a liqud.
fuel.

_ti
CHBMICAL THERMODYNAMICS AND FUEIJ 307

o This quality ofthe fuel has grea signifrcance for carburetted engines. This will decide offuel. T:he pressure indicated by the pressure gauge will indicate t}re pressure rise
the fuel vapour to air ratio in the cylinder at the tine of igrition. As F" = 0.6 is the d.ue to uaporisation of fuel and, increase in volume dve to increase in temperature. If
lowest limit for satisliactory ignition and flame propagation, therefore, volatility offuel the latter is subtracted from the total pressure rise we ge the Reid vapou pres-
l
must ensure to give at least this fuel vapour to air ratio at the tine ofignition under all sure (the increase in pressure due to vapourisation ofa given quantity offuel under
conditions of operation including starting from cold. a given quantity of fuel under given condition of temprature. .

Thevolatilityofgasolineisgenerallycharacterisedbythefollowingtwolaboratorytests.

-!
(;) ASTM distillation test (iD Reid vapor test.
ASTM distilltttion test : ./
Fig. 9.13 shorvs thc apparatus used for ASTM dietilation test: ,

.w
r 100 cubic centimetres of gasoline fuel is taken in the flask and heated. The flask is
fitted with a thermometer to record the temperature ofvapour being formed and col-
lected in a graduated measuring cylinder. 300
.
j")' 'fff-
r \dh dr"tt\{
+ 2s0
I
I

I
,"4 l"/t
9 zoo
7
=
E

$
E

$ rso
-*P;
4
.o

t00
100 cc
graduated
cylindr
Z. O|g ses

tn

Fig. 9.13. ASTM stillatioa apparatus. NL


-o 20 40 60 80 100
a when the frrst drop of.conilensed vapour drops from condenser, the temperature is re- Percont s\raDorated
-----)
corded. This temperature is called inital boiling pont.
Fig. 9.14. Distillation cunes (ASTM).
c The vapour ternperature is recorded at each successive 10 percent of condensed vapour
collected. When 95 percent has been distilled the burner flame is increased and the Equilibriiun air dlstillation (EAD) :

maximum temperature is recorded as the ?nd poinf.T:he mass of the residue in the . The ASTM distillation curve is not a true boing poit cune of the fuel. Therefore, it
flask is also recorded. cannot directly relate to fuel performance in the engine. In this case, the fuel is allowed
Fig. 9.14 shows distillation cuwes (ASTD for various producis of petroleum refining. to evaporate into an air stream moving throrrgh a long tube with low velocity. The exit
uapour-air ratio is measured as a functionof fuelair ratio. The tube should be suffi-
Reicl v,apour pressure I
ciently long to attain equilibrium. The tube epresents the intake manifold ofthe en-
The oolatiliy of petrol is also defrned in terms of Reid uapour pressure. This is a meas- gine and equilibrium ofvapour-air is reached before eniering the engine.
ure of the vapour pressure of oil at 38'C expressed as millimetres of mercury or in
pounds per square inch pressue and indicates initial tendency ofa fuel to vapour-lock.
r In ASTM-est, the vapourisation of fuel is carried out in the presence ofvapour of fuel
so these curves can not be used directly, as actual evaporation offuel takes place in the
. The apparatus used for determination ofReid vapour pressure is shown in Fig. g.15. current oi air. Therefore, to correlate the fuel performance in the engine, EAD test
A chilled fuel sarrrple is placed in the Reid bomb and then immersed in lvater baih apparatus is used as described above.
held at 38'C ; the air ehamber contain s an air uolume equal to four times the uolt me
INTERNAL COMBUSTION
ENCINES
CHEMTCAL THERMODYNAMICS AND FUETJ
3@i
0.2.7.2, Effects of Volatility on S.I. Engine perfonrance
volatility of a liquid, is i's. ter-radgncr to evaporate under a given set
chaacrerisric of S.r. e-ngine r.ri.rl
of conditions. It is an
ff:l"rTr",li,::::tant pe.foan* .ia n u"oo"_y
"ir""t. ""giie "r
coldstartingofs.I.engineisimprovediffrontendvolatilityishigherbutitrrayleadto
- increased problems ofhot starting and vapour loct.
* The mid range (20 80%) portio should be volatile
.to to giu" saisfactory air-fuel
atios under a variety ofoperating conditions. "oough
- Low tail end, uoratility wiu help in good. mxture d,istrbuton ad, hence good fuer economy.
., .{ f9* important effects of volatility on s.I. engine performance
scribed below : and enumerated and de-
1. Sarting and warm up
2. Vapour lock
3. Evaporation loss
4. Cankcase dilution
5. Operating range performance
6. Spark plug fouling
7. Formation ofsludge deposits.
1. Starting and warm up
Fig. 9. 15. Apparatus for determinirg Reid vapour presure o For easy starting oftheengine a certain part
ofthe gasoline should vaporise at room
Fig. 9.16 shows the EA-D temperature' To fulfil this condition I to 1b per
test curves cent ofthe distillation curve must have
flor a typical gasoline and also a.low boiling temperature. With the wa"ming
A/f, ratio volatility curves. up of the engine, the temperature gradu_
ally increases to the operating ternperature.
o For best warm-up, low distilration temperatures
are desiable throughout the range of
distillation curve.
2. Vapour lock: $itclrrro Fv\AlpO(
r vapour rock is a ,n,.,iql
"i;,",#:kt\ni2{fri,y#;l),!i,". ru" uoto-
motive fuel pump shourd. handle both liquids
and vapours. If the of fuel evapo-
ratod in the fuel system is very.high the fuel pump "-oorr-i
is mainly pumping vapour and very
I 20
little liquid will go to the engine. This resuits in very
maintain engine output.
weak mixture which can not
; 10 Vapour loch causes the following :
E (j) Uneven running ofan engine
d ;
E 0l (ll) Stalling while idling;
F
(jjj) Irregular acceleration
;
(iu) Difficult starting when hot
;
(u) Momentary stalling when
unnins.
The.vapour lock tendency of the-gasoline is
rerated to end, uolattity.The vapour-
10 20 so--il---g---ft-; Iiq'id (vll) of a gasoline directly corelates rurthfront
th" degree of vapour rock likery
"uo
to be experienced in the fuel system. At va
80 90 100 ratio of 24 vapour Iockmay start,and at v/L
% age vaporised by mass ____+ ratio of 36 vapour lock may be uery seuere.
Theefore, trte uotutilty of tie gasolne srrcurd,
Fi& 9. 1 6. EAD curyes for a typical gasoline be maintaned as low as practcable to preuent
this type of dfficulty.
3t
310 INTERNAL ao""rrrto* ENctNEs CHEMTCAL THERMODYNAMICS AND FUET.S
r

3. Evaporation loss : o Fig. 9.17. shows the basic process ofwhat happeus when self-ignitiol occurs
o The evaporation loss (from carburettors and storage tanks) dpend's on thc uapour pres,
sure llhch is d function of fraction components and initial temperature. These losses can
be as high as 10 to 15 per cent.
a T'he evaporation loss not only decreases the fuel econom!, but olso d,ecreases its anti'
hnoch quality as the lghter fraction haue higher antt'krtoch propertes.
The evaporation loss is a reason for restrictng the low end uolatility of the fuels.
4. Crankcase dilution:
o Ifvery frequent starting ofthe engine with low engine temperature is necessarr, very
rich mixtues have to be supplied and some of un-evaporated fuel leaks past the piston
rings and goes to crankcase. Consequently lubricating oil gets diluted. This d'ilutinn
decreases the uiscosity of the lubricating oil and also washes away th.e lubr.icating oil fihn
on engine cXlinder walls. lt is found that the tendency offuels to dilute the lubricating
oils lies in the order of907o ASTM temperature. Thus control of90% ASTM temPera-
ture combined with proper ventilation of cranckcase reduces the dilution of crank-
case-oil.
o In the engines using heaui.er fuels like kerosene and other distillates, the problern of
dilution and poor lubrication ofpistons and rings may be seuere. Time-----f
5. Operating range performance : S.I.T. + Self-ignition temPenture
r The acceleration ofan engine depends upon its ability to deliver suddenly to intake an I.D. -r Ignition delay
extra supply offuel air mixture in a sufficiently vaporised form to burn quickly. Good Fig. 9.1?. Self ignition charactristie of fiels.
acceleration occurs when air-fuel vapour ratio of 12 : 1 is supplied' above the self-ignition temperatue (s.I.T,),
If the temperature of a fuel is raised
r The ability to accelerate falls offas available mixture becomes lean. the fuel w1ll spontaneously ignite after a short ignition delay (I.D')' The hgher
6. Sparkplugfouling: above S.LT. which the uel is heated, tIrc shorter will be I.D.

Spark plug fouling is caused due to deposition of some high boiling hydrocarbons. Lower the
Igrrition delay is generally avery small fraction ofa second (generally ofthe order
oi thousandthi of i seeondl. During this time, pre-ignition reaction occurs, includ-
tail-end volatility less are the chances of spark plug fouling. ing oxidation'ofsome fuel componnts and even cracking ofsotne large hydrocar-
7. Formation of sludge deposis: bo-n components into smaller hydro-carbon molecules, These pre-ignition reactions
The sludge deposition inside an engine is caused by certain types ofhigh boiling hydrocar- raise th temperature at local spots, which then promotes adtlitional reactions
until, frnally, the actual combustion reaction occurs.
bons. These dep osils cdn cduse piston ring plugging and, sticking and. ualue stickng resultng in poor
o Fig. 9.18. shows the cylinder pressure as a function of time in a typical s.I. engin-e
operation and. poor fuel econom!.
coirbustion chamber sirowing (i) normal combustion, (ii) combustion with light knock
9.2.8. Knock Rating of S.I. Engines Fuels and (iii) combustion with heavy knock.

9.2.8.1. Self ignition characteristics of fuels


. When the temperature of an air-fuel mixture is raiseil high enough, the nixture will I
' . self-ignite without the need ofa spark plug or other external igrrites. ?e teperature
o
o
aboue which this occurs is called the'self-ignition ternperature (S.I.T.)", This is the basic E
f,
principle of ignition in a compression ignition engine: The compression ratio is high o o E
o
enough so that the temperature rises above S.LT. during the compression stroke. Self- o
igrrition then occurs whe the fuel is injected into the combustion chamber. On the
@ po
.E
other hand, self-ignition (or pre-igrrition, or auto-gnition) is not desirable in an S.I' o
engine, where a spark plug is used to.ignite the air-fuel at the proper time in the cycle.
Time Time Time ---------}
When self-igntion does occur in an S.I. engine hgher than desirable, pressure pulses are --)
----''
(fl Normal combustion ( Combustion with (iii) Combustion with
generated. These hgh pressure pulses can cause darnage to the engine and. quite often are heauy knoch
with no hnoch leht krcch
in the audible frequenqr rozge. This phenomenon is often called knock or ping.
Fig. 9.18. Cylinder pressure as a function oftinein a typical S.I. engine combustion chamber.
312
INTERNAL coMBUSTIoN ENGINES CHEMICAL THERMODYNAMICS AND FUEII
9.2.8.2. Ilighest Useful Compression Retio (II.U.C.R.) 313 .;
The highest useful compression ratio o There are several different rating octane numbers, each ofwhich wirl give
ests used for
is. the highest co.mpression ratio at which a slightly different oN varue. Tbe tuo
used without detonatnn in o specft'ed a fuel can be
of rating gasoline and other
tion a.nd mixture strength b"r"s ;;ij
test engine unler specified. operoting cond.itions ard ""iZn-^"tnods
automobile s.I. fuels ae the Motor mehod
the gnt- and the n"".r""rt L'uli".-d
^;;;'
t"' ;;;; ;;;;ffr"*""r. the tnotor octan nzzr (MON)
9.2.8.8. Ocane number (ON) and engine knock t_ ";;-;;;;ilne
mon rnethod is the Aviation method which
,ro_be" (RO*r, *orn". i"t"""?rll /
I- is used for aircr"ft il ;i;;;; an Avia-
o The property of a fue.r whch describes tion Octane Number (AON). ,,/

octane number or just octane. This is fuer w.irt -or w,r not self-ignite is caed the
how
anum_eri*r"r" g"""r#f'ioo,p*irrg The engine used to measure MoN anil RoN
self-ign*ion chaacrrisrics trr" was deveroped in the 1980s. It is a
i"eii" ilio"t
"a"r] i"'l"H;J;.
single cylinder, o:1tu"9 valve. engine thar operates
"rtt'"The two ;1-;;r;
"; from 3 to 80. tiJ .y"le. rt
at specific operating conditions. resr engine
stinar rri"*i"ifi"i, lrl",*" has a variable compression ratio ihi"h
can b'e adjusted
, ery hish resistance to knockand rherefore is given Research Oetaue Number (ROM Method
n";:,:"tf#J3#l,lirl * " i
Normal heptane (czHre ) which isuery prote to knocll andis therefore i.
I
-;#;r"todmeasuesanti-knockperformanceunderrlotivelymiidoperatingcon_
giv errazero
Blends of these reference It is considered to be similar to the detonation
tendency ofa fuel when the engine is
fuets the knock resistatrce of intermed.iate
d,efine
arrdthusablendof l07o-heptane octane numbers,
and'goq,ir;-";;;;;t"yrrolumehasanoctarenumberofgO. Tcce!3r9ting ftom'low speed' in top gear with a wide open throttle und,er ,medium
o The hgher the octane number of.fuel the less ti.ketjr I load" rnlet r t"-pe"aiu"e sz" coor""iiemperature
it wiil self-ign,".'iriro", with low I
= i
600 r.p.m. ; ignition advance setting lg.C BTDC, =
100.c ; engine speed
compression rotios.can use =
fuers with rower octane numbers,'blt hah-cmpressian Motor octane number (LION) mcthd.:
gnes must use high-octane fuel to en_
_
Test procedure for fi.nding octane "roiill-gnitian
and knock.
I
I
This method measures an*knock performance
number i6N) of fuel: wder relatiuery'seuere operdting
To frnd ON of a fuel, the foilowing ertp-"rJrrJ, conditions.
J
o""a , It is considered to be sieilar to the detonation tencency
The test_engine.is run at specified conditions i ofa fuel when the
using the fuel being tested. (The specl- driven at 'medium speedi intop gear wit u wiae-op"" -iuy
engine is
fied | fixed cond.tions to gi;u -""i-;; ;;ockresponse are : Air-inret temperarure, Inlet air tenperature = 150"c ; ohnt tempeiatue
t,*ttl" ,r". toa,.
coorant remperarure, ensine rpe"a, iiiio"
conr.pression ratio is ad.justed unt,
ud"d;;;; -i*iri?Jt""r,gtl, r.p.m., ignition advance setting 19_26. B|DC. = 100"c ; engine speed = 900
sity of knock is measured *ith
i standard. reuel_of hnoch is experienced. (rnten_
"t..1 Fuel sensiivib (FS)
=

" ^;;;;i;ruton nioil"i";;;;r..,,., o The difference in octane number betueen the Research nethod
The test fuel is then replaced with a mixture ofthe two standard fuels.
The intake and the Motor method.
system of the engine is desigrred s""t tfrai octane numbers is hrawn as the
tfre hlend of the two standard fuels fuet rcneitivitSr ; thus
be vaied to any percent from all iro_o"lu* can
b
all n-heptane. Fuel sensitiuity = RON _ MON
- $ir?l,fli.:f::i: t, vaied unril the same knock chaiacterisrics Fuel sensitivity is a gnd, tne@aure of how sensitiue knoch cha.racteristcs
are observed as wiII be to e'gine geotnetry. A low FS numuer witt usuatty of a fuer
The percent of iso-octane in teristics of that fuel are insensitive to engine geometry.
tr""" lrr.t r."""k-chu"ac-
the fuel blend' is the.oN giuen to F-s numbe", g"rrliully.rrrgu
stance, a fuel rhar has rhe same'knock the test fuer. For in- from 0 to 10.
rt iso-ocrane
and, ISVo n _heptane would have an
O ""*t..u"J";;;";";i, Antiknock index:
"ig.
between octane number and compression The average of the.two- octane number rating methods,
tr;Hifl:Tp ratio is approximarely
good antiknock quality indicator which
RON and MON, is very
is krroin the antiknock index ; thus
",
Antikncrcla. mdex
- RON+MON
= ---z
Advantages ofhigh-octane fuel :
t The ad.uantages of high-octane
fuJ are as follows :
1' The engine can be operated at high compression
E ratio and therefore, with high efli-
ciency without detonation.
o 2. The engine can be supercharged to high output
E without detonation.
3. Optimum spark advance rnay be employed raising
o both power and efficiency.
r High octane fuels (uoto 100) can be produced
by refintng techniques,butit is done more
cheaplv, and more fr-eouentrv, bv the us;;i;;-k";;;
(An addition of 1.1 cm3 of dtra;thvr d.ditiues, such as tetraethyl lead.
s I""d t" ;;J;;e "of 80 ocrane petrol increases the
6 7 8 I 10 11 octane number to g0). Fuers hve-ben dever"p"
*i.rr have a higher anti-knock rating
Compression rato _-> than iso-octane and this has lead to an erte.rlion
oiJhe o.t".r" ."rlu.
Fig 9'lg Relationshipbetweenoctanenunrberandthehighestusefurcompressionratio.
314 INTERNAL COMBUSTION ENGINES CIIEMICAL TTIERMODYNAMICS AND FUELS

9.2.9. Miscellaneous Properties of S.I. Engine Fuele high enough to ignite the air-fuel mixture, This requires a certain amount of time, known
as ignition deloy.
Miscellaneous properties ofS.I. fuels ae described below :
Though ignition delay is affected by several engine design parameters such as compres-
1. Gum conten
sion ratio, injection rate, injection time inlet air temperature etc., it is also dependent
. There is a tendency in some gasolines to deposit gum, a solid oxidation product, in fuel on hydrocarbon composition ofthe fuel and to some extent on its volatility characteis-
systems and on valve guides. Excessive gum forrration often causes sticking ofvalves tic.
and plugging offuel passages.
The cetane number s a nurnerical tnedsure of the influence the diesel fuel has in deter-
o The gum formation is reduced by mixing inhibitors (special chemicals) with gasoline.
mning the ignition d'elay.
c Tbe ordsed gasoline shows o loss of anti-kmch quality.
a Higher the cetane rating of the fuel lesser is tlle propensity for diesel knoch.
2. Sulphur content
a Ignition quality is usually determined by an engine bench test which zeasrcs lhe
The presence of sulphur content in gasolines is objectionable since it may lead to the forma-
gnition tirne delay under standard carefully controlled. conditions.
tion ofsulphuric acid in the presence ofmoistue. The sulphuric acid has corrosive effect on engine
parts. In such a test, the unknown fuel is rated on a scale between 0 and 100 against a
pair ofpure hydrocarbon reference fuels. Getane (Cr.Hrn) (n-hexadecane) a straight
3. Tetra-ethyl lead
chain paraflin which has a very high igniton quality (short dela!) al;.d does not
. It causes deposits on cylinder walls, spark plug and valves etc. rvhich lead to the corro-
read.ily hnock, is assigned to the top ofthe scale by a cean e number of 700, whereas
sion of spark plug and exhaust valves. These troubles are minimised by adding ethyl-
heptamethylnonane (HMN) whicl has a very low ignition qualitx Qong delay)
ene-dibromide (CrIIrBrr).
andreadily knocls, is represented at the bottom end ofthe scale by a cetane number
It is a very dangerous poison acting on the skin and in vapour form, the lungs. of lS. Originally, the low ignition quality referenc e fuel w as alpha methyl napthalane
9.2.10. Diesel Fuel @ il H i which was given a cetane number of zero. However, heptamethylnonane,

o Diesel fuel (diesel oil, fuel oil) is obtainable over a large range ofmolecular weights and a, ori stabte compound but with a slightly better ignition quality (CN = 75), now
physical properties. It is classified by varioue methods, some using numerical scales replaces t.
and some designating it for uarious uses. Generally speaking, the greater the refining Hence, the cetane number (CN) is shown by,
done on a sample of fuel, the lower is its molcular weight, the lower is its viscosity, and Centane number (CN) = Percent centane
the greater is its cost. 0.15 x percent heptamethYlnonane
"Nurnerical scales" usually range from 1 to 5 or 6, with sub categories using alpha-
A standard single-cylinder pre-chamber variable compression ratio engine is used
betical letters (e.g., A!,2D, etc.). TItc lowest numbers hue the lowest molecular -
ueights and. lowest vscosiryt. These are the fuels typcally used in C.I. engines. Fuels operating under fixed conditions : Inlet temperature = 65.5'C ; Jacket temperature
pressure
with the largest numbers are uent vi.scous dnd. can only be used in large, massiue = 100"C ; Spegd = 900 r.p.m. ; Injection timing = 13" BTDC ; Injection
heating unts. Each classification has acceptable mits set on various physical prop- = 103.5 bar. 'ire engine is run on a supply of commercial fuel of unknown cetane
erties, such as viscosity, flask point, power point, cetane number, sulphur content number under standord operating cond,itions. with the injection timing fixed to 13"
etc. BTDC, the cornpression ratio is vaied until combustion commences at TDC (by
Another method, of classifyittg d,iesel fuel to used in LC. engines is to de s,gnate t for observing the rapid rise in cylinder pressure) thereby producing a 13" delay period
- its intended use. These designations include, bus, truck, railroad, marine and sta- of 0.0024s at 900-r.p.m.. A selection of reference fuel blends are then tested, where
tionary fuel, goiog from lowest molecular weight to highest. ogain the compressioit ratio is djusted for each blnd. to obtain the standard. 13"
o For convenience, diesel fuels for I.C. engines can be divided into two extrem,e categores ; delay period.. The percentage of cetane is one of the blends of reference fuels which
(r) Light diesel fuel (molecular weight 1?O appr.) gves eractly the same ignition deley (ianition quality) when subjected, to the same
(il) Heavy diesel fuel (nolecular weight 2) appr.) compression ratio is colled tlw cetae number.o.f /e fuel. Thus, a commercial 4o
. Most diesl fuel used in engines will t in this range. cetane fuel woulil have aa ignition delay performance equivalent to that of a blend
of 40Vo cetane ad 604o heptamethylnonane (HMN) by volume.
Light d,iesel fuet will be less viscous and easier to pump, will generally inject ino
- spaller droplets, and will'be more costly. For higher speed, engnes the ceta.ne number required. is 50, for med.ium speed. engne
about 40, and for slow speed engines oboqt 30.
Hiavy d,iesel fuel can generally be used in)arger engines with higher injection pres-
- sures and heated intake systems. cetane number is the most important single fuel property which affects the exhaust
Often an automobile or light truck can use a less costly heaviei fuel in the summer, but rnust emissions, noise and startability of a diesel engine. In general, lower the cetone nu'nber
change to a lighter, less viscous fuel in cold weather beause ofcold starting and fuel line pumping higher are the hyd,rocarbon emissions and noise leuels, Low cetane fuels increase igni-
problems. tiot'delav so that start of combustion is near to top dead centre. This is similar to
\ retariling ofinjection timing which is also known to result in higher hydrocarbon levels'
9,2.10.1. Cetane Number (CN)
In general, a hgh octane uolue mplies a low cetane ualue.
o The cetane number of a diesel fuel is a eosure of its ignition quolity. When a fuel is
injected into the hot compressed air in the rylinder, it must first be raised to a temperature
INTERNAL COMBUSTION CHEMICAL THERMODYNAMICS AND FIJEI.J
E{GINES
o The.relation between cetane.numbe 31'1
and delay peric t particular engine at a par-
ticular set-or numbes only dictate that there-will be a great d.emand.
"unijis is illustrated ," 'r"'$.f;J.
"l"o*,o* for fuer in the comin! decad,es.
Gasolne will become scarce and costly. Arlernate
fuer techrirogr, availabiliti, and. use
mtst and wll become more cotnmon in the coming decad,es.
o Although there have always been some I.c. engines fuered with non-gasorine or diesel
oil fuels, there numbers have been relatively ri,^I. B".u,r." ofthe b-igh cost ofpetro-
^ leum products, some thid-worrd countries ave for many years been using
manufac-
I tured alcohol as their main uehicle fuel.
-8- o several pumping stations on natural gas piperines use the pipeline gal to fuel the en-
o gin_es driving the pumps. This solves an terwrse
compricatid p"oru- of delivering
firel to the pumping stations, many of which are in very isolate regions. Some
-d' largi
oo displacement engines have been manufactured especially for piperinJwork. These
con-
sist ofa bank ofengine cylinders and a bank ofcrnpressor cylinders connected to
the
same crankshaft and contained in a single engine blck simill to a V-style
engine.
o Another reason motiuating_the d.euelopment ofarternatiue fuers for the LC. engine is con-
Fig.9.Z0 cern ouer t-he emission problems of gasorine engines. comLined with
other ai-r poiluting
t invese relationship between systems, the large number of automobiles is a major contributor to the
air quaiity prob--
cetane (cN) and octane (oN)
ff#:::-tate number is lem of the world' vast improvements have been
-"d" i., ""a""i"g u-ir"l;;gi"L" orry
an automobile engine.
CN=69- ON o Still another eason for alternative fuel development in India and other industrialised
.2
(accurate within countries is the factthat-a rarge percentage oicrude oil must be imported
from other
g.z.Lo.z.Dieser r' 5%) countries which control the large oil fields.
rndex (DI) ,Sone alernative fuels rrc can replace conuentibnal
The diesel index is a chean fuels in LC. ,ngiin", or" ,
- -'---n tnetlad
t of predicng ignition quality. (l) Alcohol (Methyl and ethyl)
Tr.i^ . ;- possible
-^^r- rs made
.'D Dcare
*^":1j." jgli* quality is sensitive to hydrocarbon compo_ (ii) Hydrogen
high isnitian;";rit;;;;
illii";li|tii,if[7ffi"'0",, "o^poudi iiu,
","matic
and (ili) Natural gas
o Thedieselindexg^uesa^nind,icatonofthegnitianquarityobtained.fromcertanphysicar (iu) LPG (Liquifred petroleum gas) and LNG (Liquifieil
natural gas)
characteristics of the.fu:t (u) Biogas.
; ,pp.r;";:",,fr^!:!
*r"rynaon.in a test ensine. The 9.3.2. Advantages and Disadvantages of using Alternativc Fuels
*i,r,1],"i?j;,'ffi;,i:A;lii'iii*"i"'e^e"icanpetroi".,*minsrture(Apr)
Advantages of using alternative fuels :
Diesel index (DI) Ailine point ("F) API eay (deg) I Alcohols can be produced
= x from highly reliable and longJasting raw materiar sources
like sugarcane, starchy materials, corns, potatoes etc. Thus they are renewabre energy
| - Theanilinepointofthe tselistheten 100
sources.
pure anitineis.sot," i"-;;;;;;;,".;Jl:::;:::;"2:?:t::^::;::2ff^{!i!:r#
composition of the tuel since the 2. Biogas plants can be conveniently and economically installed in village
and farms.
fr;riifirlr the sorution
mo.u o".rffi.,i""_th. Manufacture of biogas from cow-dungwill give, as a byproduct, manure of very
high
ii,T'1llll'1"^,!i,T:i:ffiJ:-"1ilf, '-fl::':l ::fi;;;.;-i"*".;i'#c gravity and quality for use in the-farms.
to a good ignition Natural gas is available in plenty.
Note. The correrarion between rhe dieset-index *::j:::":*:ds d"lity. 4
coal gas is produced by coal gasification, and coar is abundant to last much ronger
!;:',";;';ff:;i';;;;;;;t";;;;;;';T#:;::'T"::f::',:2fr:1,#;:,7ii"?:::,,:#ii'; than
the liquid petroleums.
9.3. ALTERNATIVE FUELS FOR I.C.
ENGINES - Alcohols, biogas and naturar gas hare much higher octane number, and. are sutable
for use in s.I. engines u)ith tittte nrodification. There s ress probrim of hnoching or
9.3.1. General Aspects detonation-.-Higher compression ratos can be used, to giue more power
ond, increised.
o The crude oil attd petroleum products, thermal efficiency.
sometit 6'
; " ; ;",;;;;" l,ffi Hil: :ff t *.""i: ii,Hlr,
The exharrst from engines using gaseous fuels contains ress poilutants.
u e
ry s c d, c e a n d c o'st ty h fr; use of hydrogen
gives absolutely clean exhaust.
increase in the number
ril;#" l.c.", ;? ?;
engines
Q Brearrr improved
"i^"r"-"..
fi"- th;;;;i;;; wirr prJbabry
"ngi"o".-irrffi;h ffi economy of Disadvantages:
;;;;]"".;,ffi" improved, use ofalcohols as an alternative fuels have the following disad.uantages :
1. social problerns due to prohibition, as it can be consumed as riquor by human beings.
319
INTERNAL COMBUSTION ENGINES CHEMICAL THERMODYNAMICS AND FUEIS

2. Thecarburettorwouldneed,nod.fi.cation,asthestoichionetricAir-fuelratbwithalcohols 8. Alcohols have almost invisible flames, which is considered dangerous when handling
is quite low (of the order of 10 : 1) fuel. Again, a small amount of gasoline remove this danger'
3. The calorific value ofalcohol fuels is loa as corrpared to that of diesel or petrol. 9. Low flame temperatures generate less NQ, but the esulting lower exhaust ternpera-
T}:'e m.e.p. and power output from a given size engine will be low with coal gas and tures take longer to heat the catalytic converter to an e{ficient operating temperature'
(due to
biogas. 10. when refuelling an automobile, headaches and dizziness have been experienced
The handling and fransportation of natural gas is costly. the strong odour of alcohol).
High compressor power is required to compress it for storage otherwise it will need Note:
large storage space. a Alcohols are comidered as clem buming renewable altemative fuels which can come to our rescue
Hydrogen is highly explosive, and its handling is risky. to meet the a challenge ofvehicular fuel oil scarcity and fouling ofenvironment by exhaust emis-
sions.
9.3.3, Alcohol
I Alcohols make very poor diesel engine fuels as their'cetane number'is coasiderably lower.
Alcohol is an attractive altrnative fuel because it can be obtained from a number ofsources, Alcohols cm be use<l in duel fuel engines or with assisted ignition in diesel engine. In a duel
both natural and manufactured. Methanol (methyl alcohol) ad, ethonol (ethyl alcohol) ae two fuei mo<Ie, alcohol is inducted aloug with the air, compressed and then ignited by a pilot spray
kinds of alcohol that seem most promising and have had the most development as engine fuel. of diesel oil.
Advantages: Methanol
9.3.3.1.
Ttre advantages ofalcohol as a fuel ae :
r of all the fuels being considered as an alternate to gasoline, methanol is one of the more
1. It is high octane fuel with anti-knock index numbers (octane number on fuel pump) of
pronising and has exrerienced major research and development'
over 100. High octane numbers result, at least in part, from the high flame speed of
alcohol. Methanol can be obtained from many sources, both fos;r and reneuable' These include
coal, petroleum, naturol gas, bomass, wood, Iandfills, and euen the ocean' However'
any
Engines using high-octane fuel can nn more efciently by using higher compres- manufaciuring or processing raises the price ofthe fuel
source that requires extensive
sion ratios.
and requires an energ.y input back into the overall environmental picture, both unat-
2. It can be obtained from a number of sources, both natural and manufactured. tractive.
3. It has high evaporative cooling (l.) which results in cooler intahe process ond compres-
o Methanol beleaves much like petroleum and so, it can be stored and shifted in the same
sion strohe. This roiss thp uolunZtric efficiency of tlw engine and, reduces tl* rcquired
manner.
work nput in the compreseion stroke.
4. Genrally less overall emissons when compared with gasoline. , It is more flexible fuel than hydrocarbon fuels permitting wider variation from
- ideal A/T ratios.
5. Low sulphur content in.the fuel.
It has relatively good lean combustion characteristics compared to hydrocarbon
6. When burned, it forms more moLes of ahaust which giaes higlur pressure and, more power
fuels. /s wdcr inflammabitty limits and higher flame speed,s hove showed' higher
in th erpansion strohe. thermeil efficiency and, lesser erhaust emissions cotnpared, wth petrol engines.
7. The contamination ofmatter in slcohols is less dangerous than petrol or diesel because
o Depending on gasoline-methanol mixture, some changes in fuel supply are essential.
alcohols are less toxic to humanS and has a rcogizable taste.
simple modifications to the carburettor or fuel injection can allow methanol to replace
Disadvantages: petrol easily.
7. Low energy content ofthefuel (Almost twice as much alcohol as gasoline must be buroed fuel
Some important features of methanol as :
to give the same energy input to the engine).
1. The specifrc heat consumption with methanol as fuel is 50 percent less han petrol
2. The exhaust contains more aldehydes. Ifas much alcohol fuel was consumed as gasoline,
aldehyd,e emissions would, be a serinus e*,hdust pollutons proplern. engine.
3. As compared to gasoline, alcohol is much more corrosive on copper, brass, aluminium, 2. Exhaust co and HC are decreased continuously with blends containing higher and higher
rubbqr and rnany plastics. This puts some restriction on the design and manufacturing percentage ofmehanol. But exhaust aldehyde concentration shows a reversed trenil.
of engines to be used with this fuel. 3. Methanol can be used as supplementary fuel in heavy vehicles powered by c.I. engines
Methanol is very corrosive on metals, with consequent saving in diesel oil and reduced exhaust pollution'
4. In general, the ignition characteristic are poor. Advantages of methanol :
5. Vapour lock in fuel delivery system. 1. Owing to ts ercellent onti-knoch characteristics, it is rnuch suitable for s.I. engines.
6. Owing to low vapour pressure and evaporation, the cold weathering starting characteris- 2. Methanol Jse maintains goocl air quality (Methanol emits less a\mount of CO, and other
tics are poor. ' polluting gases as compared to gasoline fueled vehicles)' \
7. Due to low vapour pressure, there is a danger ofstorage tank flammability. Air can leak
into storage tanks and create a combustible mixture,
, 320
I INTERNAL COMBUSTION ENGTNES
:d.
3. Tertiary butial alcohol is used as iE. CHEMICAL AND FUEIJ 321
an octane improving agent. TERMODYNAMICS
4. One percent methanol in petrol is l
i
used to p""rr"ot freezing offuel in winter. l+i r Fig. 9'24 to 9.2? show the of equivalence ratio on all important objectionable
I
5. Iso-propyl alcohol is used as anti-ici.,g,g;t It emissions, CO, HC, Ne -effect
and aldehydes respectively, for petrol and Methanol.
I
io.".brr""tto..
i 6 Addition ofmethanol causesrethanol gasorine blend to iil
than pure gasoline below its boili"g p;i;;--' evaporate at much faster rat
Performance of methanol as I.C. engine n Lean
+
I

The performance chaaceristics


ae
fuel :
consideed as : Lean f rooo
f- Rlch

Thermal efficiency verses A./F ratio I --+ I


Pqrol
Effect of speed on power output and
:
n I
o
I Rich I

specific heat consumption with petrol


Effect of A,T ratio on exhaust emission. engine ;
6
E Methaiol
F tzoo "/
o Fig. 9.21 (a) shows the effect ofA/T ratio
and speed on brake power.
F >4 t, I
I ;
T I
li Boo

Lean
<- --.>

u2 lt Petrcl

V
,Methaml

I
-4
50
I
Rich
\ \ ::-
^ 40
Methanol-
I 0.8 1.0 1.2 1.4
I
t-.' t I
u.o
Equivalence rao ---l
0.8 1.0 1.2
Equivalenceratio
1.4 .1.6

a
o
30 I
g
----)
Fig.9.24
=
o
*i Petro
=
8.
o Fig.9.25

o 5000
E 101
E
(D
,'\ Petro
+
OI | 4000
0. 7 0.8 0.9 .t.o
r.1 1.2 0.3 2000 30@ 40oo
I
l
Equivalnce ratio
----) Spesd (r.p.m.)----+ E.ooo I
(a) : I r=8.5&r.p.m.=2000
(b) 1\.
Fig. 9.21 Effect ofA,/T ratio and speed
on brake power. z zooo
I lt r\ L E
a Fig 9.22 shows the effect ofload o
',"/,L,n^^.,
a Fie e. 2s show s th e effect Ji;T"','* ", "0" H: :HilT:j:* 1 000 \, o
!
7 o

0.8 1.0 .t.2 1.4 1.6


+ Equivalnce ratio --.>
1.0 1.1
I
t2
I
/i lhanol
I
Fig.9.26 Fig.9.27
a 11 + I
B |
I
0.8
'/ 9.3.3.2. Ethanol

E
F
Petro
\ o Ethanol canbe made from ethylene or from
fermentation ofgrans and, sugar. Much of
o
E 0.7 it is ntade from corn, sugar beets, sugarcane, and. euen cellulose (wood, and, paper).
I

Full Load
The present cost of ethanol is rtigh d.ue to the manufacturing and processing
This would be reduced iflarge amounts ofthis fuel are used.
requirecr.

2000 4000
i o EUranol has less HC emissions than gasoline but more than methanol.
Speed (r.p.m.)----> rooo 2ooo 3ooo 4ooo sooo 600 o Gasohol is a mixture of90% gasoline and rovo ethanol. As with methanor,
Speed (rp.m.) the develop-
Fte.9.22 \
---.> ment ofsystems using mixtures ofgasoline and ethanol continues.
Fig.9.23

I
FJEIJ 323
CHEMICAL THERMODYNAMICS AND
322 INTERNAL COMBUSTION ENGINES

o- TIte ma.rmum efficnncy of ethanol engine is hlher tb,an petrol engine. The
tlentul
T\vo mixture combiations that are important are E 85 (85% etbanol) ad E10 a wide range of speed which indicates that
oi"i*.engin'e is flat for
(gasohol). E 85 is basically an alcohol fuel with 1570 gasoline added to eliminate "ff.i"rr"y.,rr""
the part loail efciency is mush compared with petrol engine'
soire of the problems of pure alcohol (i.e., cold starting, tank flammability, etr.). oThebnginetorqueisconsid/roblyhigtwrfotethanolascomparedtopetrolengine.
E 10 reduces the use ofgasoline with no modification needed to the autonobile
engine. Flexible-fuel engines are being tested which can operate on any ratio of Properties of Msthanol and Ehanol
9.3.3.3.
and ethanol are listed in the
ethanol-gasoline. The physical and chemical properties ofgasoline' methanoi
Performance of engine using ehenol : Table 9.3.
etha'nol
Table 93. Important properties of gasoline, methanol'
The effect ofspeed on power output, brake specific heat consumption and thermal efficiency
of an engine using ethanol is compared with gasoline engine as shown in Fig' 9.28 to 9.30' Gasoline neorlY Methyl alcohal Ethyt-alcohal
C"II rr(Iso-Octore) (cHsoH) (cstsoH)
S. No. ProFert!
A 1. Moleolarweigbt 1L4.2 32 &
I
43 to l70 66 78
c 2. Boing point at 1 bar "C
Freezing point 'C - 107.4 - 161.8 -ll',I.2
0.?9 0.79
I Specifi c gravitY (150'C)
E 0.72 to 0.75
4.
Petrol 1110 900
e ? Latent het (kl/kg) ,rcO
o
o I b-
6. Viscosity (ceotiPoise) 0.503 0.696 0.60
E= q
I3 { Stoichiometric A: F (ratio) 14.6 6.46

-r /
7.
g3 8. Mixture hating value (kJ/kg) 2930 3070 2970
a
o
(for stoic-mixture)
8to19 2.15 toL2.8 3.5 to 17
o Ethanol 9. Ignition hnits (A/T)
E 335 574 oo
E 10. Self ignitionTemP.

2000 4000 6000 Octme Number


3000 5000 70@ 90oo 8000 11.
(o) Resear 80 to 90 112 111
Sped (r.p.m.) ------+ Speed (r.p.mJ----> a1 92
(b) Motor ' 85

1t CetaneNumbe 16
Fig.9.28 Fis.9.29 L97& 26880
Lower C.V. (lal/kg) 4100
13.
14. Vapou pressure at38'C (bar) 0.,18 to 1 0.313 o:
ro. Flme speed (rnlw) 0.rtil 0.76

Auto-ignion temPerature ('C) t22 467


lo.

9.3.4. Alcohol{lasoline Fuel Blende


methanol in rac-
1
Norrnally straight alcohols are not used in automobile engines excelt
i;;;;:Th;"obolr .rtt be used as blend with gasoline as this has the advantages
i ttt t" .,,istrg engines twt be motlfid' and' TEL (Tetro'ethll leod) can be elimnated'
E
o fiom gosottne ao" tl tn" Octane enhancing qual'ity ofalcohol' major mod'ifications are
Tf the engine s to be operated' using only pure alcohol,
then sorne
F
requrei in tle engine dnd fuet sstem as listed' below :
corro-
(i fne materiats useil with alcohols have to be changed since both alcohols are
sive to many of the materials that are used with gasoline'
1000 3000 5000 7000 9000 (ii) Ailjustmenofcarburettor and fuel injection system to compensate for leaning effect'
SPeed (r.P.m.)
(iii) Introduction ofhigh energy ignition system with lean mixture'
--> (iu)A]terationinfirelpumpandcirculationsystemtoavoidvapourlockasmethanol
Fig.9.30
vapourisation rate is verY high.
a The power outpl ofethanol engine s hgher comparcd to gasoline engine at all speeds' (u) Incre'ase in compression ratio to make better antiknock prperties ofthe fuel'
a the brake specific heat .consumption is irnproued with ethanol engine compared with (ui) Arldition ofiletergent and volatile primers to reduce engine deposits and assit in cold
petrol engine. starting.
11t

INTERNAL coMBusrloN
EN.INE'
(uil) use ofcoorer runningsnnrk CHEMICAL THERMODYNAMICS
nhr- r^- ^-- .,. AND FUEI.S 325
o rhe ,endsb;;;ffi,1ffi?*JlTffiTffifj''"l
(i) The engine can
be statd
--'*"-%s over pure ethanol
eas. ehano, :
li
ti!
9.3.5. Ilydrogen
(dl')
t* A number of conpanies have built automobiles with proiuipe or modifieil engines which
No r[oor-"1 corro. F operate on hydmgen.
,,1,t Pb;;;;;;il,,H;:ffi illl i ffslil"
(iu) some benzene ."_" F - AEngineering
Hr-powered car being developed in Melbburne University Departrrent of Mechanical
when brends are used,
is a1d3 ro n**r;rh.;;;"iria. of perrot and alcohor. is achigving 40 percent energy saving over conventional petrol engine. The
,,"rar"., are
the fououing car is a converted Ford Cortina Wagon which carries enough fuel in 4-cylinders to travel
(l) The cabuettor jei
ueede t" b"
\;;;;;;;:;iili)
i""";;;;;; "a,, required :
upto 50 km at a speed of 180 km/h.
petrol. crease the flow 1.56 mes
that of F )
German carst are so developed that they can be converted for driviug either gasoline or
(li) The float is to
be weigbed down, to co"rect - Hr by making a few simple adjustments. Presently a few vehiles aie runnittg on road
(iii) Modificatioo leoel ( specific graviry. i#
oi^iri"*-- -- ,"t
6- v* air as ,,""0
less d ligher
Dleno requires less
as there are very few public filling stations for tiquid Hr. Many more are planned for.
tion than petrol. " air for complete combus_
.(lu) ""j:,"_l future.
Pevton of a specific_arra:ryr..nl Advaniages ofhydrogen as I.C. engine fuel:
as rowe vapour pressue of heating the carbuettor
I

e Fig' 9'31 shows the change Jiriit! Jir"r*u oimculrand


or."or,oi intake maniford
berow 70.c.
L. Law emissions. Essentialty no CO or HC in the exhaust as there is no cabon in the fuel.

o indistill"r* itlu"rl-t*" of { Most exhaust would be HrO and Nr.


Fig. 9.82 shows rhe reaning
r Fig. 9.33 shows
effect 'ethanol and eaaol.
.r"";;i;;i;;,rr" ,o p"t-r. 2. Fuelauailability. T1,terc ae a number of diffeient ways of making hydrogen, including
"f of
the antiknock quality electrolysis ofwater.
methanol-gasoliae blends.
200 3. Fuel leakage to environment is rnt o pollutant.
4. High energy content per volume when stored as liquicl. This would give a large vehicle
range for a given fuel tank.
+ 5' Hydrogen-air mixture burns le n-tirnes faster compared to gasoline-air mixture. Since the
Ethanol
/
f r50
I
200/"
I
I
1.tl
burning rate is considerably high, it is more preferred. in igh speed engines.
^ 6. Hydrogen-ignition limits are much wider than gasolines. So it can burn easily and giue
E
E consid,erably hgher efficiency.
a 7. Hydrogen has high self-ignition temperature (s.LT.) but very little energy (l/50 th of
E
a Methanol
'-o a 1 gasoline) is required to ignite it.
E 1oe/o E
.@
.9 8. The exhaust heat can be used to extract H, from the hybrid.e reducng the load engine.
20%
300a
.9
o 9. Besides being a relatively clean burning renewable source, f! as I.C. engine fuel is uery
g r.os efficent as there are o losses associated with throttlng.
E
Disadvantages :
L I T\e.!a7dl!nS^of.!, lt !*" dfficutt and, storage requires high capital and, runntng cost
zo _ o --i---*-io 1.0
parttcutarly for liquid. H,
Distlfaton, V.V---+ 10 zo so-t 2. Dfficult to refuel.
Proportion of admixe 3, Poor engne tolumetric efficiency. Aay time a gaseous fuel is used in an engine, the fuel
Fig. 9.31. Change in distillation comrnenis
bv
Fig. 9.32. Leaning effect
will displace some of the inlet air and poorer volumetric efliciency will resilt.
admixture ofmethanol ofalcohot
an ethanol. 4. Fuel cost would be high at present day technology and availability.
admixture to pertol
5. Can detonate.
6. High NQ emissions because of high flame temperature.
7 ' In hyd'rogen engines there is a danger of bach fire and. induction igniton which can melt
the carburettor, Therefore in Hr-fuel system, flome traps,
90 flash bch arresters are neces.
sary. Ad'ditionally, crankcases must be aented, to preuent accumulation of explosiue mix.
tures.
9.3.6. Natural Gas (Methane)
o Natural gas is a mixture of components, consisting mainly of methane (60-gg7o) with
small atnounts of other hyd,rocarbon fuel components. In add.iton it contains uarious
d..rnounts of yr,
9Or, He and, traces of other gases. Its slphur content ranges from very
ltile (sweet) to larger amounts (sour).
INTERNAL COMBUSTTON ENCINES
CHEMICAL THERMODYNAMICS AND FUEIJ 32'I
An ideal compositian of cNG as an automative
fuer is as foilows :
Methane = 90% (ninimun) ; Ethane content 9.3.7. LPG and LNG
(madmum) ; c, a'nd hgher 0J% = 47o (maximum) ; propane content = L,,Vo c LPG (Li4uifud. Petroleum Gas) is mainly p ropane but may also contain a small propor-
= imaximuml i r- neh"r lco 2 + N) =
0.27o (maximum) ; Hydroeen
= 0.1% (naximu ;; | ;r;;"
--
""i
'
1!'i"io'r".i;
naonoid = 0.1% (maximrim ) ; Orygen
tion ofbutane and possibly, sorne ethane and a little pentane in heavier vapour form.
0.57o (maximum); Sulphur = 1096 ppm (maxim;t.- = The heaer fractions tend to occur in LPG produced by distillation ofcrude oil.
o It is stored as Collyag Nturel Gas (CNG) at pressures Propane has a higher octane number, burns more clearly and saves on mainte-
remperature around _160"C. of 7 tn2!bar and a nance costs.
o As a fuel it works n an engine systetn with a singre-ttvottre body Propane is gaining as a gasoline substitute because it costs 607o ofpetrol and gives
giues a longer ",st time, ,nn *iia"liy-rn*
mng fintinjector.
This 90% mileage ofits fellow gasoline,
Tests using cNG in various sized vehicles
W. o LNG (Liquified' Notural Gas) cones fom dry natural reservois mainty CH o with very
continue to be conducted by government
agencies and private iudustry. small percentages ofethane and propane.
Propertiee of CNG: Therrajordiffrcultyencounteredin theuse ofthisgasis its boiling temperature
Tho properties of CNG are almost simila to that _ 161.5"C.
of methane :
o Methane has very g-ood antiknock qualities which means it does 9.3.8. Biogas
no t i,nite readily. Anti-
hnock oclane number ol c!'rg i: ir""b iso,-to
compared. with petrol unledd. (Octane o
it burns at much higher temperature 9.3.8.1. Inroducion
Sgl or nl whch
o owing to better anliknock quality of cNG= it can be safely usedlwve low etane number.
in engines with
o The biogas is generally produced from by dung from different beasts as cow, buffalo,
pression ratio as high as 12 : I compared ' a com- goat, sheep, horse, d.onhqt arrd, elephant, Some other sources are :
with petrol (maximum 10, it--
The cNG fuer used engincs houe higher thermar (i) Sewage ' (t, Crop residue
efftiencies tban those fuelkd. by
gasoline' In addition to this, the reuction (iii) Vegetable wastes (iu) Water hyacinth
in the fo[utants ernitted by cNG en-
gine is noticeable. (u) Alga (ui) Poultry droppings
o cNG is non-totic and wter than air so when reakage
xrurs it qubkly disappears un- (uii) Pig manure (uiii) Ocean kelp.
like gasoline which padles,.a '- -
o Biogasisproducedbydr.geslion,pyrolysisothydrogasificaioz.Digestionisabiological
o """po"ui"r.
The presence of ethane and propane even in small percentages (5% and,zVo, respec_ process that occurs in absence of oxygen and in the presence of anaerobic organism at
tivelv) affect the burning p-pe"ti". of cNG. ambient pressures and temperatures of35-?0'c. the container in which digestion akes
Both tie g*ur-t y io lo*"i ln" o"t"rr"
characteristics and causeg pre-ignition and place is known as t}lre cligester. Biogas plants have bee built in various dsigns.
reduced ruet*emciency.
Advantages ofC?{G:
9.3,8.2. Composition and Properties of Biogas
(i) Hibh octd,t number makes it
avery good S.I. engin fuet.
Gi) Lout engiw emssbns. Lcss aldehydes than with
o Its rnain combustible component is cH. aid another major component is coo whbh re-
methanols. du.ces ts u:tott nutnber. The componentJof biogris with composition are givei below :
('ii) It is cheap (It costs about 25
to 50% less than gasoline
and more than E0% less than other Component
alternative fuels, such as methanol u"a etU"ioi. Composition
(iu) It is engine friendly. (% volume)
(u) It is safe in operation. cHn 5H0
(ui) Fuel fairly abundant world-wide. CO, 3H5
Natural gas is the secnd most abundant fuel available H, and N, 5-10
in India after coal.
(uiz') Easy to tap. IIS aad O, Traces
(uiil) Jt is odourless. Octane rating: 110 with CO,
() It is clean. 130 without CO,
Disadvantages ofCNG: o Biogas possssea excellent antiknock properties with an equivalent Octane number in
excess of 120 compared with 87 for regular petrol.
e*"gy densiy resulting in low engine performance. o lts auto-ignition temperature is higher than petrol which makes it a safer fuel
_-(i] lotn
(ii) Low engine volumetric efhcienry b"cuu." it i" gaseous
(iil)
a fuel. r Being a gas it mixes readily with air even at low temperature, therefore, there is no
Need for large pressurised fuel storage tank. need to provide rich mixture during starting or idling.
(iu) Inconsisent fuel properties. o Although its calorific value is lesser than petrol, it is possible to use higher compression
(u) Refueling is a slow process. raio for the same size engine thus making it possible to generate the same amount of
(ul) Th-s-.o""tu cyrinder takes a rot power.
of.space as the gas once fiiled has to travel at Ieast
400 km' But now a days there are byfuer of Use of biogas in S.I. engines :
and du-er.fuel engines which can run on cNG 'S.I. engines can be operated on biogas after starting the engine
and other fuel. by using petrol. Biogas can
be used in these engines in two forms as :
328
NTERNAL COMBUSTION
ENCINES CHEMICAL THERMODYNAMICS AND FUELS
To un the engine entirely on biogas. 5v
_(i)
(li) Duel fuel engine where v6re
engine wr run on both fuels.
can r'un (This Example g'2' Determine grau-imetric analysis of tlrc prod.ucts of complete
these days). fu rrrngement is preferred ,the
acegrlene with 200 per cent stoihioietric combustion of
air.
9.3.8.3. Advanages Solution. The Stoichiometric air equation (Example g.1)
of using biogas as fuel in is witten as :
C.I. engine
The biogas ca be used
in C.L engine as a duel fu"I
The following areth ad,uantrcc";"::*::11"" ;; :mpoves engine performance. c + 2.5o, (#) -----, zco, + rr"o. r.u(fi *,
T: 13.5
(,)Aunironi;;;;;ff;:::i:ffi llffi Tflil".;Hij?;:*"'""'
"H,
rf 200 per cez stoichiometric air ",
is used, the combustion equation for cr*"bpcomes
)
to ctean operarion ort"
,fii]3:"
(iir) rt;reiJvi*#; c,H, + (2) (2.5) o,
;""t"*;;;1",n"rfilffi;."*T"
when biogas is used as a "ogiou
tuel, N4 +@ e.5, (X)N, --+ 2co, + Hro + (2) (2.5) (#) *'ir,,r.u, o,
t-'"';,",1iJ';Tilio1H,1xXT"""1;*;{,*ff,f;"'"
r;", p,,n,;;;;*-.'i,n tut (2 x !2 + 2 x t) ke CrHr+ (2) (2.s) (2 x
16) kg o, + (2) (2.5)(#)
"
14) kg N,
"x
WORKED EXAMPLES -----+ 2 (r2+ 2 x 16) kg CO, + (2 x
1 + 16) kg Hro + (D e.s)(H 14) kg N,
- \r.j
Air-Fuel Ratio and Analysis
of producs of Combustion "x
o" kq crH, + 160 kg o, + s26.6 kg Nr.-; 88 kg co, + 18 kg Hro
';tlj
(,"P f;t3:l:"JY;g" of theoreticat air required, - Jrillf;ti,Jrl:,i}-"1,
or 1kg-CrHr+6.15kgOr+20.25kgNr--r3.38kg-COr+0.69kgH2O+20.25kgNr+B.O?kgO,
_r.u
acetylenz co, and Hro. " """t for the combustion of 1 ks of
Solution. "For combustin Thus for I kg offuel, the products wjll consist of
ofacetylene (CrHr) the
stoichi, ometric equation
ror->. a
C,H. + O- cor'+. r rr
t1o^ is written as CO, = 3.36 t
lz\+
Barancing the carboi, aom,
0"" ,ili HrO = 0.69 t
"
2C-aC
2C=aC ;" "r"r#.""_0".r,
.e.
tonequation(l)'weget
Oz = 3.07 kg
Now balancing hydrogen a=2
atozls onboih sides, we get Nz = 20.25 kg
2H=Zttt Total tnass ofproducts rr3, kg
=
.'. Mass fractions are :
Thus, equation (j) becomes
3RR
C"IL + : O"___-_+ 2CO, + H.,O co2=; __o.12s
Now, [f si g oza !en" at o ms'in tt rou Jioo nAq
" "
i.e. t
H2o=;;=o.o2s
Hence, the fi.,,r = 2.5
"o-bu"tio?;ffiJ,]"t
CoH, +
^ 3.07 = O'ttZ
ur=
2.502
____+ 2CO, + H"O ,t.es
Thus, for combustlon-of
CrHrin air, *" "t^^r" 20.25
Nz=
z?.g9 =0'739'
zto, + H,o +,, (#) Ifence the gravimetric analysis of the complete
'. this
un a mass basis, ,: .'(#)N'-' o combustion is :
""?l:-';ll becomes _ CO z = l2,BVo, HrO = 2,5Vo, Oz
= l]',2Vo, N = ZB.97o. (Ans.)
calculate the theoretical airlfuel ratio
(2 x 12 * 2 x.1) CrH, for the combustion of octane, c{r,r. The
+ 2.5 (2 x 16) oz + r, (X) (2 x t4) ".*ur:;1Tl::9'3'
N,
cerr,e + 12.s t (X)iv, -----+ 8co, + elro
o, + ,, . tz.s (fi) w,
Solution. The air-fuel ratio on a mole basis is
or
x:,rrr,:i;;;F,;';:i#:;:}il:,;r,.+:i,.',',(-f J;'"',o
i.e., Amount of air 8.026
+i0.t
= + I012 k;N2 tz.s +tz.s(!9)
Hence amoun, A/F = = b9.b mol airlmol fuel
";,;";:;*,'"THL:iltrii*:t::f;kgrLo
ls.l96 kg. (Ans.) 'ustion of I kg acetylene air-fuel ratio on , ,'u.. bui, is found by
= introducing the molecular weight of
,n" ,,.11irL""?retical ---\?ll
330 ENGINES CIEMICAL THERMODYNAMICS AND FUELS
JJ I
-t
I

59.5 (28.97) By balancing co rbon atons on both the sides, we get


M =G f * . le) = 15.08 kg air/kg fuel. (Ans,) 2=a+b ...(r) I

_ Example "9.4. One hg of octane @dIr) is burned. uith 2001o theoretical air. Assuming com- By balancing orygen atoms on both the sides, we get
I

i
plete combustion determine ; (0.9)(35)(2) =tut+b+3 ...()
(ii)' we get
() Air-fuel ratia
Substituting the value of 1= Z - a) from eqn' (i) in eqn' I
(ii) Dew point of the prducts ot a total pressure 100 kpa. (0.9) (3'5) (2) = tut + 2 - a+3
Solution, The equation for the combustion of CrH* w.ith theoretical air is 6.3=o+5 I

a=1.3 I

crn,, + t2.5or+tr r (#) Nr-----+ 8co, + ego + rz.s(lf) n, and b=2-ai-2-l'g=O'7


For 200o/o theoretical cir the combustion equation would be Thus the combustion equation becomes :

/7e) -_
CrH,, + (2') (12.il Oz + Q) Q2.$[#) C,II. + (0.9) (3.5) O, + (0.9) (e.s) Nr
[1J
" /29\
-----+ 1'3 CO, + 0.? CO + sHrO + (0'9) (3'5) N,
-----+ 8CO, + eHrO + (1) (12.5) o, + (2) (12.5) (,'e) *, [aJ
Mass of fuel = (1) (8 x 12 + 1 x 18) = 114 kg/mole Total number of moles of dry products of combustion
r7e)
- (0.e) (3.5)
Mass of air = (2) (rz.s) 28.92 = s448.8 kg/mole of fuel = 1.3 + 0.7 + [ 21J
[t #) of fuel
(') Air-fuel ratio : = 1.3 + 0.? + 11.85 = 13.85 moles/mole
Volumetric analysis of dry products of combuttion is as follows
:

Air-fuel ratio. A/T


Mass of air p++a's
= Mass =
fuel a = 3o'2s 1q
x 100 = 9.3M. (Ans')
of co, = ,fi6
1.e., A/F = 3095. (Ans.)
7
(ri) Dew point of the products, tun : x 100 = 6.06%. (Ans.)
Total number of mole.s of products "O
= fr6

=8+e + r2.5 + (z)(1r.5) = 128.5 moles/mole fuel


[;1) sExample 9.6 . Metlwre QH ) is burrcd with otmosplwrh air' Tlw arwlysis of
tbe prod'ucts

Mole fraction of tlO = = O.OtrS on a'dry' bass is as follows :


fr CO2= 10'0Mo, 02=2'37%' CO = 0'539o' Nz= 87'1Mo'
Partial pressure ofHrO = 1) x 0.0728 = 2.28 kPa
the oir'fii'el ratia;
The saturation temperature corresponding to this pressure is 39.7'C which is also the dew- G\ Deterrnne the comiustion equatinn; Gi) Calculate
point temperature (ii) Percent th'eoretical air'
Hence tp=39.7'C. (Ans.) Solution. (i) Combuetion equaion :
be wtten, keeping in mind
Noe. T?re water condened from the products of combustion usually contains some dissolved gases md From the analysis of the products, the following equation can
therefore may be quite corrosiue. For this reason the products of combustion are often epl c boue the ilew point that this analYsis is on a dry osrc.
until discharged to the otmosplere. r CHn iy O, + z Nr---+ 10'0 CO2 + 0'53 CO + 2'37 O,+ o H'O + 87'1 N2
for each of the elements'
_ Example 9.5. Onc hg of eth.ane (C2II) is burned with glEo of theoretical ar. Assuming com- To determine all the unlown efcients let us find balance
plete cornbustop of hydrogen in the fuel detennine the vol,urnetric analysis of the dry prod,ucts of
Nitrogen bolance : z = 87'L
combustion. \
Since all the nitrogen comes frorr the air,
Solution. The complete combustin equation for CrHu is written as :

CrI\ + 3.5 Or---+ 2CO, + SH,O t


-79,n
2L " = --9f'1-'= 23.16
The combustion equation for CrHu for 90% theoeticalqir is written as :
t QslzD
Carbonbalance: r= 10.00+0'63 = 10'53
crHu + (0.g) (3.5) o, + (0.e) (3.5) (#) O-- a cor+ Co + BHro + (o.e) (3.5) (;?) -, Ilydrogen bolonce: a'=2t=2 x 10'53 = 21'06
55
INIERNAL COMBUSTTON CHEMTCAL THERMODYNAMTCS AND FUELS 333
ENGNES
(
provides a check o" tn" tbr'
ror, and in this case rhe The nass of 1 mole of oxygen is 32 kg, therefore, the mass of O, supplied per kg of coal
balance u..*.ylffi";:'T:1":TT
,".r*""??,fli:#ffi'f#:iHtl-j.::,"":i"'itTl:::-o:"1sorved eternined by an oxygen
the
= 32 x 0.093 = 2.976 kg

r = 1o.oo + 9{9 + 2.sz * 3lS = z.ro i.e., Stoichiometric Al F rotin = ?tryl


0.233
Substituting these values for '*, y,
z and o, we have, (where air is assumed to contain 23.3Vo Orand,76.7Vo N, by mass)

lliiil:i1Jft;,Ti.1il"::*:::i^'-::
Diding boh sides by io.sa we
CH, + 2.Zo^ + n,, Nz
r" combustion
N ___,.:"^r.":p" "l-iI3-I^::-:"
.oiii;ffi#Hri:l
"""r
+.2Br
"
equation
o,+ 2t 06rr,0 + 82 ,
z''t u2+2r'06H2
*r,_"a
\ Total moles of products = a + b + 3.76 = 0.068 + 0.05 + 3.76 x 0.093 = 0.467 moles
(ii) Analysis of the products by volume : )
;,T,:r;1i,#22
(ii) Air-frel t"to m,
o.st co,.- "r*j ( Ane.) The analysis ofthe products by volume is :
The air_fuel ratio on a mole n oA
bos.s is CO' = x 100 -- L4.664o. (Ans.)
' 0.4tl1
rheair.tuer"^r,",::X1u;,:'i"-.[ffi -nn<
il.J.]T#:l"Sj;],**",r0," Hr=
iffi x r0o = lo.74o. (Ans.)
LO.47 x28.97
'vr' = l:2llllf = 18-96 kg air/kg tuet. (Ans) (3.76
x 0.093)
The rheoretical air-f,
uel raio is fouud by writing Irl= --Z xl0o=71.8%. (Ane.)
the combustion equation for theoreticar
ai.
ctn + 2o,., rFExample 9.8. Calculate the stoicltiometrc air-fiul ratia for the combusti.on of a sample of
(#)N, ____+
co, + zl"o +rzl (,,eJ rv,
dry onthracite of the following composition by mass
Carbon (C) = 88 per cent Hydrogen (Hr) = 4 per cent
_ArF
fr.edf)l*n, Orygen (Ot) = 3.5 Per cent Nitrogen (N) = 1 per ceni
(dii) percenr *h"""";;;ff ,
(12; D- = 17'24 kg air&g tueL (Ans') Sulphur (S) = 0.5 per cent Ash=SDercent
If 30 per cent excess air is supplied. deternne :
percent theoretical (i) Air-fuel ratio
ai 18.96
i "t0o=17qo' (An') (i) Wet dr1 analyss of the products of combustion by volume.
Example s.z.Thepi,
- - ' ' ' "- 6' uimetric analysis of a Solution, Stoichi<jmeric air-fuel (A/F) ratio :
ash. catcutatle . sample of coat is gioen as g2% c, la{o H" and g7o
fn case of a fuel with seveal constituets a tubulnr metlad. is advisable, as shown below.
G) The stoichiometric NF-ratin; (ii) The
-'"- anolysis of the products by uolume.
Each constituent is taken separately and the amount of orygen required for complete combustion
Solution. (j) The etoichiomeric
f ratio: is found from the chemical equation. The oxygen in tbe fuel is included in the column headed
1 kg ofcoal contains 0.g2 'oxygen required,' as a negatiue quantity,
kg C and O.l0l;;,
... 1 kg ofcoal ,ont"irr, 0.82 ^
r^ Mass per Combustion eouation Orygen requircd per Products per hg of coal
Le*h e oxygen ."n",,i 11? .ilff :l:-"::: ::,", kg coal hg of rcal

Then the nitrogen supplied C 0.88 C+ Or---+ CO, o.8s 0.8s, g.2B kg Coz
with the oxygen = r ,
# = B.Ze, mole. ' ff =2.346kc, #=
For I kg ofcoar the combustion l2kg + 32kg---s Mkg
equation is therefole as fo,ows
:
.lt c_*
0.82 0.10
U, x +8,26* N, ___+ o Co, + IIrO
H2 0.04 2I1r+Or+2HrO 0.04x8=0.32k9 0.04x9=0.36kgHzO
t + CO"
+ 8.76 N.,
1kg+8kg----r9kg
Then,Carbonbalanee : o2 0.035 -olunu
Y =" ... a =0.06gmoles N2 0.01 O.Of f.g ftf
,
Hydrogen balance : z,
ff = zt .., = 0.05 moles S 0.005 Or+
S+ SO, o.oo .
;; = o.oo5 kc o oos x
S = o.ol kc so2

32 kg + 32 kg+
Orygenbalance:2x=ya6 .,= (::Ueiel) Ash 0.03
64 kg

=o.oes_or.,
Total O, = 2.636 kg
JJ+ CHEMICAL THERMODYNAMICS AND FUEIJ
INTERNAL COMBUSTION ENGINES

From table : Example 9.9'The following analysis rel.ate to cool gas :


O, required per kg ofcoal = 2.696 kg Hz = 50'4 Per cent CO = 17 per cent
CHo = 26 Ps ..' CFa= 2 Per cent
.'. Air required per kg of coat =
= 11.g1 ks Oz = 0'4 Per cent Nz= 6.2 Per cent
##
(where air is assumed to contain 23.8% O"by mass\ COt=40"'""n''
N, associated wit this air (i) Calculate the stoicltornetric NF ratia.
= 0.?6? x 11.81 = g.6Z kg
.'. Total N, in products (i Find. also the wet and dry analyses of the produtts of cornbustion if the actual mtture is
= 8.62 + O.O1 = 8.6g kg
The soichionetric AriF ratio. 1l.3Ul. (Ane.) 30 per cent weoh.
When 30 per cent acess air is used,: Solution. The example is solved by a tabular rnethod ; a specimen calculation is

(i) Actual A/F given below :


ratio :
For CII. :
Ro CIJ.+ 2Or-----+ COr+ 2HrO
Actual A/F ratio = 11.31 +11.31 , =t+.ltt. (Ans.)
i.e., + 2 moles Or ----+ 1 mole CO, + 2 moles HrO
1 rnole CHo
(ii) Wet and dry analyeee of pruducts of combustion by
volume : There are 0.2 moles of CHo per mole of the coal gas, hence
As per actual A/F ratio, N, supptied 0.?62 x 14.7 11.27 7*, 0.2 moles CHo + 0.2 x 2 rnoles Qr----+0.2 moles CO, + 0'2 x 2 moles HrO
= =
Also O, supplied = 0.283 x t4J = g.42kg .'. O, required for the CHo in the coal Sas = 0.4 moles per mole of coal gas'
(where air is assumed to contaiu N2 76.7Vo and
= O, = 28.3) Theorygeninthefuel(0.004moles)jsincludedincolumn4asanegativequantity..
In the products then, we have
Prcdrcl Moleslnole Combustion equation Ormolzsl Prcd-ucts
Nr = 11.27 + 0.01 = U.28 kg mlefiul cot HrO
and excess O, - 9.42-2.6A6 = 0.2g4 kC fuel
The products are entered in the following table and the analysis by volume is obtained z 4 o
1
:
In column 3 the pe_rcentage by mass is given by the mass ofeach product divided by 0.252 0.504
HrO 0.504 2H.2+o.2+zHzO
the total mass of 15.66 kg.
CO 0.17 2CO +O"-+2CO' 0.085 ;
cH. 0.20 CH. + 2Or+ CO, + 2HrO 0.400 0.20 0.40

- . In column 5 the molee per kg ofcoal are given by equation


r = # : The totar of c.Hs 0.02 C.H. + 60, --+ 4CO, + 4II,O 0.120 of 0.08
column 6 gives the total ofwet products per kg of coai, and by subtracting o2 0.004 - 0.004
-moles
the moles of Hro from this otal, thelotatmoles oflry proucts is obtained as N2 0.062
0.5008. CO, 0.04 0.04
Column 6 gives the proportion ofeach constituent ofcolumn 5 expresse as a per-
' centage ofthe total noles ofthe wet products. Total=0.853 0.49 0.984

Similarly column ? gives ihe percentage by volume of the dry products. (i) Stoichiomeric A/iF ratio :

Product Masslhg cul %byrres M . 0.853


Mole: lhg cool % by ooL wet %bXvol dtXt Air required = = 4.06 moleVmole of fuel
f 2 3 4 5 6 " contain
(where air is assumed to 217o O, by volume)
.co, . Stoichomtfic A I F ratia = 4.06/1 by volune. (Ans.)
3.23 20.62 u 0.0734 r4.10 14.66
;

(ji)WetanddryanalysisoftheproductsofcombustioniftheaciualmixtureisS0To
HrO 0.36 2.29 .
18 0.0200 3.84
weak:
SO,
i 0.01 0.06 u 0.0002 (say) 0.(X 0.04
Actual A./T ratio with 307o weak mixture
o2 0.78 4.98 32 0.02,u 4.68 4,87
N2 11.28 72.03 28 0,4028 77,84 80.43 .Oe +*IUU x4.06=1.3x4'06 =6'278t1
15.66ks tffi.oo roo.oo6ns. Associated N, = 0'79 x 5'278 = 4.!7 moles/mole fuel
- H2O = 0.0200
Excess oxygen = O.Zl 5.ZZe - 0.853 = 0.255 moles
"
Total dry,= 0.5008 Total moles of N, in products = 4.17 + 0.062 = 4.232 moles/mole fuel'
INTERNAL COMBUSTION ENCINES CHEMICAL TT{ERMODYNAMICS AND FTJEI.S
J) |
' This means that 1/0.8 or 1.25 times as much air is supplied as is necessary
for complete
combustion. The exhaust will therefore conain 0.25 stoichiomeiric
oxygen.
(tq\
i.e., CrHuO+ 1.25 +axfrNzJ __+2COr+BH,O+0.2S x BOr+ 1.25 x e *lf N,
{.AO,
i.e., The products are :

2 moles CO, + 3 moles IIO + 0.?6 moles O, + 14.1moles N,


Totalwet=5.899 w'w The total moles =2 + 3 + O.Z5 + 14.1 19.85 )
100.00 loo'oo (Ans') =
- H2o = 0.984
Total dry = 4.9f5
Hence wet analysis ie :
I
/9
Example g,r0' Find, the stoichiometric CO, = x 10o = 10.08eo. (AnsJ
air-fuer ratio for the combustion of ethyl arcohor
;;5
[?;1,:!;;;*'::::E:r;:,:::i::",,:*.i:I::,:::,r'rorthe eztreme misture strengths or80per
IO = x
3
700 = t.ttVo. (Ans.)
193s
Solution. The equation for combustion ofethyl
alcohol is as follows
^=
u
o.75
x 100 = 3.78Vo. (Ans.)
: tSSs
'lL
since rhere *""'il::l;to"';; if3'.i lli3"rcfl,o then rhere mus be two L=
1
xl00=71.o3eo. (Ans.)
"."in
the products, cil;;l''" aroms of ffi
f,jilj "o, "rboo
on ea"n siae or tne The total dry moles =2 + 0.75 + 14,1 = 16.85
similarry, there are six atoms ofhydrogen in
since-
eacb mole ofethyl alcohol then
Hence dry analysis is:
there must be three mores oi nro in
t-he'pr"a""tr, gr"i"g , iirya.og"r, o.,
each side ofthe equation. "rori, COr=
- *; x L00 = 11.877o. (Ans.)
:- Then balancing the. atoms_of oxygen, lb.b
it is seen that there arc (2 x 2 + g) Z atoms o.75
on the right ha,nd side. of the
hand side ofthe equation. rr,.ru"qtion, h"o." *"' atoms must appear on =
the reft
c,z = ie"e5 x IOO = 4.45Eo. (Ans.)
ir onJ.io.-ofo*yg"n in ethyl arior, therefore
oxvgen a r4.1
u"
:*::rtT"^.Tff":f 'u't
'"pfi", 'o "o."-t""l, -"j". "r""ysen are Nl, = feSS x 100 = 83.687o. (Ans.)
Since the Or.is supplied as air, the
associated\, must appear in the equation, Consid.ering a mirture strength of TSOVo :
i.e., CrHuo+3or+Bx *r_-___ 2Cor+SHro*a" Now,
Stoichiometric ratio
H # N2 1.3 =
Actual A,/tr'ratio
-"*"Pa:t_1;tf="il;l.t*amassof(2x12+1x6+16)=46ks.Threemolesofoxygenhavea 8.96
.'. Acual A/F ratio e'esr' (Ans')
= ls =
.'. O, required pcr kg offuel = ff = ,.on f., This means that
I
or 0.?69 ofthe stoichiometric air is supplied. The combuston cannot be
fr
Stoichiometric t r1o compete, as the necessary orygen is not available, It is usual to assume that all hyd,rogen s burned,
.'. A,./
F ratio = 8.9G/1. (Ans.) to HrO, since hydrogen atoms haue a greater affinifir
f =
for oxygen than carbon atoms. The carbor in the
Consid.ering o msture strength of g|Vo
: fuel will burn to CO and, CO, but the relatiue proportions haue to be determined,.
Let, a = Number of moles of CO, in the products, and
Now, mixture strength !!!hig*g!Ii. lVF."tio
= = Number of moles of CO in the products
Actual A,rF
"atio - Then the combustion equation is as follows :
U.=- 8.96 / I (tq\
Actual A,rT ratio
CrHuO+ 0.769 +sxfrNzJ Co + 3HrO +0.269 s* N,
Actual A/T ratio
= o-
8.96
= rt'Zt' (Ans.)
l3O, --+aCor+ " fi
338 INTERNAL COMBUSTION ENGINES 339
CHEMCAL THERMODYNAMICS AND FUEIS
To find o and a balance ofcarbon and oxygen atoms can be made,
i.e., Carbon balance : (j) Volume of reactants per kg of tuel = 'H't = *tt m3' (Ans')
2=a +b ...G) "
and Orygenbolance: When the products are cooled to 130"C the H,o exists as steam, since the ternperature is
(This
1+2x0.769x3=2a+b+3 well above the saturation temperature correspondig to the partial pressure of the-HrO.
pressure is 99.6'C, and the
must be so since the saturation temperature corresponding to t}e total
or 2.614 =% +b ...Gi)
,i,oturotion temperature decreases with pressure, The total moles of the products is
From eqn. (i) and (ii), we get o = 0.614, = 1.386
/ ?q\
i.e., The products are : 0.614 moles CO, + 1.386 moles CO + 3 moles fLO + 8.678 moles N, =[2+s+3x'iJ=16.3
The total moles = 0.614 + 1.386 + 3 +8.678= 13.678.
Hence we analysis is: From equation, PV = nRoT
o A1
COr=ffi x100=4.497o. (Ans.) ,, _
nW _ 16.3 x 8314 x 103 i< (r30 + 273) 546.14 m3/mole of fuel.
=
P lx 10o
l.g86 x
co= 13.678 roo= lo.r37o. (Ans.)
n3' (Ans')
(ii) .'. Volume of productsperkgoftuel =#= 113?

HrO= xL00=ZL.eBTo. (Ans,)


,,k eExample 9.12. The foltowing s the compositon of coal gos supplied to a gas engine :

*r= Hz= 50.6 per cent; co = 10 per cent; ca= 26 per cent; cy'I8= 4 per cent ; or= 0.4 per cent ;
*ffi xroo=63.454o. (Ans,)
COr= 3 Per cent; Nz= 6 Per cent.
The total dry moles = 0.614 + 1.386 + 8.678 = 10.678 the air'fuet ratio s 711 by uolume, calculote tlc enalysis of the dry products of combustion'
If
Hence dry analysis is : It can be assumed tlt'at the stoichionetr NF ratio is less than 711 '
solution. since it is given that the actual NF ratio is greater than the stoichometric, there-
co,=
## x 100 = 6.75/o, (Ans.)
fore it follows that ecess aii has been supplied. The proilucts rill therefore
consist of COr, H2O, 02
and Nr.
co=ffi x 100 = 12.987o. (Ans.)
The combustion equation can be witten as follows :
x 7O, + 0'79 x 7N,
0.506H2 + 0.1CO + 0.26CH1 + 0'04C4H8+ 0.0(XOr+ 0.03CO2 + 0'06N, + 0'21
Nr=ffi x IOO =81.274o. (Ans.)
-rcCOr+I{rO+cOr+dN,
Dxanple 9.11. For the stoichiotnetric mi.xture of enmple 9.10 calculote : Then,
... o = 0.55
(i) The uolutne of the mi*ture per kg of ftul at a temperature of 50'C and o pressure of 1.073
Carbonbolance: 0.1+0.26+4x0.04+0.03=c
... b = 1.186
bar. Hlrdrogen bolance: 2x 0.506 + 4 x 0.26+ 8 x 0'04= 2

(i) The uolume of the products of cornbustion per kg of fuel after cooling to a temperoture of Orggenbalonce: 0.1 + 2 x 0.004+ 2 x 0.03 +0'21 x 7 x 2=2a+b+%.:, c=0..111
130"C at a pressure of 7 bar. Nitrogenbolance: 2 x 0.06 +2xO.79x7 =2i
Solution. As before. .'. Total moles of dry products = 0.55 + 0.411 + 5.59 = 6'65
Tlren analysis bY volume is :
CrlIrO+3Or+3x
# *r----- 2cor+ BHro * a' fi N,
co, = H; x 100 = 8.3e%. (Ans.)
70
.'. Tqtal moles reactants = 1 + 3 + 3 x = 15.3 nll
; O, = x rO0 = 6.27%. (Ans.)
From equation, pV = nRoT
Tff
A aRAT 15.3 x 8.314 x 103 x (50 + 2?3) N,=# xlfi)=85.347a (Ans.)
Y=
p = 405.6 m3/mole of fuel
\ 1.013 x 10c
Example g.1^3,, The
fottouing s the analysis (by weeht) of a chemical fuel
:
In 1 mole of fuel there are (2 x 12+6+16)=46kg Corbon = 60 per cent ; Hyd.rogen- = 20 per cent ; Ory gen =
5 per cent i S ulphur = 5 per cent and.
Ntrogen = 70 per cent.
Find the stochiometric ctmount of air required. for cornplete combustion of this fuel.
340
NTERNAL COMBUSTION ENGINES CHEMCAL THERMODYNAMICS AND FUET.S 341
solution' on the basis of 100 kg fuel let us assume
an equivalent formula ofthe form From the given analysis by weight, we can write
:

I%t=84 i.e., a=7


From the given analysis o" *"r"?iliu?H;* 1b=10 i.e., =10
12=60 or d=5 16c=3.5 i.e., c=0.218
Lb=29 or b=2O 74d, = t.S i.e., d. = 0.107
16c=g or c=0.3125 The formula of fuel is CrHrooo.rrrNo.ro,
lU - 70 or d. = O.7t4B The combustion equation is written as
Then the formura of the fuel
e = 0'1562
""ift;iu"" "1"
cuHzooo.srsNo.zrso.
crHrooo.rrrNo.ro, +.r o, -'(#) Nz +p co,+ qHro + r N,
The combustion equation is rse,
Then,
Corbon bolance : 7=p i.e., p=7
crHrooo.rrru N
o.rrrrso.rrr, + x. o, + t(#) ___-" c9z +
cHro + r so, + s N,
Ilyd,rogen balonce : 70=Zq i.e., q =5
Then. ", Orygen balonce I 0.218 + 2,a = (2p + q)
Carbon balance: or O.218+2,x=2x7 +5 i.e. = 9.39
5 =p P=5
Eydrogenbalance . 20 =ZO q=10
Sulphur balance : Nitrogen balance.. o.to7 ."(H) =r,
O.L562
= r r = 0.1562
Orygen balance : 0.3125 + 2, =(2f * q *2".
0.707 +21 9.3g xP
ZL =2ri.e..r
i.e., , =p * *"_9.3!1=5. o.3725
=35.+
I # + 0.1562_
2 = 9.99
llence the combustion equation becomes

Nitrogen balonce : 0.7148 *X, fi =X crH,ooo.r,rNo.ro, + 9.890, . r.* (#) N, ----+ 7co, + sHro + 35.4N,

_ 0.7743 79 0.7743 7q
--- + e'ee x
= t''nn 9.39x32+9.39xf9x28
t1
Hence the combustion Soichiometric A/F ratio = (Ans.)
"0r"r,"";r=;r'* ;;r,n, 100 = 12.89.
Gi) Percentage composition of dry flue gases by volume with 20 per cent excess
cuHrooo.rrrsNo.rrorso.ruu, + 9.990, . 9.99 N, ---+ ECo, + 10Hro + 0.1s62so, arr
[#) + BZ.94N,
3

g.ggxgz+g.ssx(fi If20 per cent excess air is used, the combustion equation becomes
Stoichiometricairrequired=' *'-""{.zf/^'o
.'.
]]1 rr.rn*sortuer. (Ans.)
[#) -,
100 = CrHroOo.rrrN'r0? + (1.2X9.39) O, + (1.2X9.39)
can also b-solved by rabutar
method as explained in example 9.8.).
I"j:ll"::lnre
Exampre , t:3,:::tl: 5HrO + (0.2X9.39) O, + (1.2X35.4) N,
Carbon = 84 per!:er
y has the fouowi",
o,*,;;;;-;";;;,::;;;;';",ro, ,
--+?COr+
Total number of holes of dry products of combustion
cent Hydrogen = I0 per cent
Oxygen = 3.5 per cent = 7 + (0.2X9.39) + (1.2X35.a)
Nitrogen = L5 per cent
Ash-Ipercent = 7 + 1.878 + 42.48 = 51.358
(i) Dctermine the stoichiometric .'. Percentage composition of dry flue gases by volume is as follows :
air fuel ratio by mass.
If 20 per cent excess oir is supplied.,
,oru*llt, nd. th pnrc"ntage composition of dry CO, = x 100 = 13.6J7o. (Ans.)
flue gases by
Ak
Solution,0) Stoichiometric air fuel
ratio: 1.878
on the basis of 100 kg offuer let us O = x 100 = 3.66Vo. (Ans.)
u.ru-u equivalent formura ofthe form Sf :SS
c"HQNo "n .

N"=
- #9
cl.a
x700=82.7rta. (Ans.)
342 TNTERNAL coMBusrroN ENcINES CHEMICAL THERMODYNAMICS AND FUEI.s 343

Example 9.15. orsat analyss of the prod.ucts of combustion of a hydrocarbon fuel of un- Solution. (i) As per analysis of dry products, the combustion equation is written as
hnown composition is as follows :
Carbon dioxide (CO) = 6qo
/tr\
Carbon monnrid.e (CO) = 0.5Vo o CrHr, + 78.1N, + 78.1 | ii I Oo + 8.9COo + 8.2CO + 4.3H2 + 0.5CH, + 78.1N, + r HrO -
Orygen (Or) = 6.3Vo Nitrogen (N) = 85.21o \79l
Determine the following : Carbon balance : 8 = 8,9 + 8.2 + 0.5 = L7.6 i.e. o = 2.2
(i) Air-fuel ratio t1
(ii) Percent thoretical air required for combustian. 78.1x28+?8.1xJx32
Air-frel (A/F) ratio
solution. From the given orsat analysis the combustion equation is written as follows :
2.2(8x12+1x18)
2186.8 + 664.3 2851.1
11'37' (Ans')
o c + H+ c or+
[?) "*, =8coz + 0.5co+ 6.30r+rHro+85.2N, = lffi =
(ii) In this case the combustion equation is written as
Then,
Carbon balonce: =B + 0.5 = 8.b i.e., a = 8.5
o CrHr, + Or+
l?e\Nr---r
alJ 8.9CO2 + 8.2CO + 4.3H, + 0.5CH, * b
(7e\
N, +.rHrO
70 IAJ
Nitrogenbalonce: 5 C =95.2 i.e., c =22.65
zl Carbon balane : 8o = 8.9 +8.2 + 0.5 =17.6 i.e..a=2.2
Hyd.rogen bolance.' 18o = 4.3 x2 + O.5 x 4 + 2&
Oxygen balance: c=8+T+e.s+j or LBx2.2=8..6+2+2 i.e.,x=L4.6
or OrXgenbalance: 2b =8.9 x2 + 8,2+ x
22.65 =8 + o.25 + 63 + i.e., x = 16.2
f, or 2 = 17.8 + 8.2 + 74.5 i.e., b = 20.25
Ifud.rogen balance : b =2t, = 2 x 16.2 = 32.4 i.e., b =32.4 /7o \
(l) Air-tuel ratio: (20.25 x 32\ + (20.20 x 28
I,J
The air supplied per 100 moles ofdry products is .. Air-fuel (A/F) ratlo =
2.2(8x12+1x18) =#=11'oe' (Ans')

r79 \x sExample 9.17. The erhaust from dn engine running on benzole was measured witlt the
= 22.65 x 32 + 22.65 x 28 = 3u0.6 ks
ln ) help of Orsot apparatus. Orsat analysis slwwed, a CO, content of 727o, but no CO. Assurning that the
3110'6 remaind.er of the erhaust contains only orgen and niirogen, calculote the air-fuel rato ofthe engine.
Air'fuel ratio =,e5 23.1 kg of air/kg of tuet. (Ans) The ultimate arnlysis of beruole is C = 88lo and Hz= 12%.
"' + BZ4 ,. 1 =
''2
(ii) Per cent theoretical air required for combustion Solution. 1 kg of fuel, consisting of 0.88 kg C and 0.12 kg Hr, can be written as 0.88,/2 moles
: C and 0.1212 moles Hr. Therefore, coDsidring 1 mole of dry exhaust gas.(D.E.G.) we can write the
12 x 85 combustion equation as follows
Mass fraction of bon
carT = rfres*a2;T =o'zg
12 2 H")
].10.18 c *
0.12 79 ___
--\ -) *YOt+ iYNr--+0'l2CO2+oOr+(0'88-o)Nr+HrO
Mass fraction of hydrogen =m =O.Z4l
f I,et tt D.E.G. contain a moles of Or. Ihe moles of CO2 in 1 mole of D.E.G. ae 0.f2. f'here- I
Considering t kg offuel, air required for cornplete conbustion is "
lfore the D.E.G. contains (1 - o - 0.12) = (0.88 - o) moles of Nr.
tlrre
I
f rct loo'],f0.241)<arlool where, X= Mass of fuel per mole D.E.G.,
i lx#
"%3J'L
|+ |
0'za1r8x.-l =-,16.96kg Y= Moles of O, per rnole D.E.G.,
= l0'75exl
L-.'--"\3i 233J-'
. a = Moles of ecess 02 per mole D.E.G., and
.'.Percent theoreical air required for combustion = 23.1 r 100 = 136.29o. (Ans) = Moles of HrO per mole D.E.G.
ffi Now,
. Example 9.18. The following is the uolumetric analysis of the dry exhaust from an nternal
combustion engne : Carbonbala.nee: S X= o.tz .', X= 1.636
CO, = 6.9Eo' CO = 8.2Vo' H z = 4.37o ) CH = g.gEo on N
o z = 7 8. l%o. Hyd.rogen balance r 0.06X = .'. =0.06x 1.636=0.098
If the fuel used is octane (C, H ,) determine air_fuel ratio on mass bosis :
(i) By a carbon balance. (ii) By a hyd.rogen oxygen balance. Orygenbalance: 2Y=2x0.!2+?a+b or 2Y = 0.24 + 2o + 0.098
... Y=0.169+o

I
INTERNAL COMBUSTION ENGINES
CHEMIdAL THERMODYNAMICS AND FUEIJ 345

Nitrogen
-2rbalance, 3 r= (0.g8 - o) :. Y=0.234-0.266a Inernal Dnorgy and Enthalpy of Combustion
Equating the expressions for y gives Example 9.19, LH 0 Qnthalpy of combustion ot reference tenperature T) for benzene uapour
(CyII) at 25'C is - 3301000 kJl mole with tln HrO n the liquid phase. Calculate LIIofor the HrO in
0.234-0.266a=0.169+o .'. o = 0.0513 the vapour phase.
L.., y= 0.169 + 0.0518 =O.ZW
.'. O, supplied = 0.2208 x 32 kgi/rnole D.E.G.
Solution, IfHrO remains as a vapour the heat trasferred to the surroundings will be less
than when the vapour condenses 6y the arnount due to the change in enthalpy ofthe vapour during
. 0.2208x82 condensation at the reference temperature. )
i.e., Air supplied = 80.26 kglmole D.E.G. AII. (vapour) = AIlo fliquid) *
where, fl" = Mass of HzO formed, and
^ohfro
Since X = 1.686, then, the fuel supplied per nole D.E.G. is 1.636 kg
hfro - Change in enthalpy ofsteam between saturated liquid and saturated vapour
ratio = !96 = 18'6/r' 6ns')
.'. A./F at the reference temperature ?o
rffi6 2447.8 kJ at 25"C
=
Example gJa. The analysis of the d.ry ex.haust from an internal combustinn engine is as For the reaction :
.follows :
CuHu + 7.5Or----+ 6CO, + SH,O
Carbon doride (CO) = 15 per cent Carbon monoxid.e (CO) = S per cent 3 moles of HrO are formed on combustion of 1 mole of C.H5 ; 3 moles of HrO
Methane (CH) = B per cent Hydrogen (II) = 1 pe" cent-
O*ygen (Or) = 2 per cent =3x18=54kgHrO
Nihqen (N) = per cent (vapour)
calculate the proportions by mass of carbon
Z6 .'. AHo - 3301000 + 54 x 2441.8 = - 3f69143 kJ/mole, (Ans.)
=
to hyqen n the fuer, assurning it to be a pure Example 9.20. Calculate LUoin kJlkg for the conbuston of beruene (CdI) vapour ot 25"C
hydrocarbon.
giuen that M o = - 3169100 kJlmole and, thc HrO is in the vapour phase.
. _ Sglut]91 Let 1 kg of fuel contain kg of cabon (C) aud y kg hydrogen (Ho). Then consider_
ing 1 nole of D.E.G. and introducingXand i, w can write Soluion. Given: AII' = - 3169100 kI
The combustion equation is written as
(xC v-- \ to
12 in" ) *y or+ fiyNr__+
* o.tsco2 + 0.03CO + o.oscH4 C.H, + 7.5O, 6CO, + SH,O (vapour)
"l -+ = 8.5, n"= $ a t = $
nn= | + 7.5
+ 0.01H2 + 0.02Or+ a HrO + 0.?6N, Using the relation, AUo = AFI. - 1o" - no)RoTo
Then,
-
= 3169100 - (9- 8.5) x 8.314 x (25 + 273)
Nittogen balonce: 79 =- 3169100 - 1239 = - 3170339 kJ/mole
n Y= 0.76 .'. Y = 0.2O2 (It may be noted that At/o is negligibly different from Af/o)
Orygen bolance: Y= 0.15 *ff *o.oz,
i
1 mole of C.Hu= 6 x L2 + 1 x 6 = 78 kg

--
3110339
. auo = - 40645 kJ/kg. (Ans.)
0.202 = O.75 + 0.015 + O.02 + 9 .'. o = 0.034
2 Example 9.21. LII o for CO at 60"C s giuen as - 285200 kJhnole. Calculote 1'IIo ot 2500"C
X" giuen that the enthalpies of gases concerned, in hJ / mole are as follows :
Carbon ha|ance:
-=0.15+0.08+0.08
L :. Xt = 2.52 ...(r) Gos 60'C 2500"C

Xv
co 9705 94080
Hydrogen balance :
- =2 * 0.03 + 0.01 + = 0.06 + 0.01 + 0.084 o 9696 99790

XY = 0.208
co, 10760 149100
Dividing equations (j) and (ii), we get Solution, The reaction equation is given by

)(x, 2.52 f CO+fOr-+CO,


xv= a2ol or ; = L2'r Refer Fig. 9.34.
It can be seen from the property diagram of Fig. 9.34 that the enthalpy of combustion at
i.e., Ratioof CtoHrin*"=; =+. 1lne.)
temperature T, AHrcan be obtained from AIlo and ?o by the relationship
347
CHEMICAL THERMODYNAMICS AND FUELS
346
Example g.28, Calcutate th lower heating value of goseous octone at constant volurne if
(LIl)25"c=- 5494977 kJ for the reaaton:
- LIIr= - 670* @R. - HR) -(HP, - HP) '..(i) [CsIn(gas) + 12-5Or(gas)] = [8CO2Gas) + 9HtO Qqud)]'
where, IIq - H*o = increase in enthalpy of the raacczs Solution. The given value of^U corresponds to th e higher heating value at constant volume
from ?o to 7 because the water in the products is in lqud phase'
and Hpr- Hpo = itr"."r"" HHV = 5494977 kJ&s
in enthalpy of the producfs from
(LHV), = (HIIV) u - n(us- u)
Toto T.
m = 9 x 18 = 162 kc/kg mole CrIIts
Now, from the given data, we have
(ur- u) = 2305kJ/l<g at25"C
HA = tx g?05 +
+ x 9696 = 14553kI
(IJI9, = 5494s77 - 162(2305) = 5121567 kJA<g. (Ans')
Ho, = 7x 94080 + x 99790 = 1499?5kJ
| Exarnple g.24. Calculate the lower and, higher heating ualues at constant prcssure per hg of
Hto = | x 10?60 = 10?60 kJ lo I T mrture at 25"C, for the stoichiometri'e mxtures of :
() Air and benzene vapour (CtIIt) and'
Fig.9.34
Hr, = lx 149100 kJ = 149100kJ (i) Air and octane uapour (C rII t|.
Using equation (i), we get Guen that the enthalpies of am.bustion ot 25"C are :
NIr= 285200 + (14ilvr5- 14553) - (149100 - 10760) CrHr= - 3169500 hJlmole
= 285200 + 1294?2 - 138340 = 276282 CrH rr= - 5116200 hJlmole
.: th.e cose where thewater in the products s n the uapour phase.
AII, = -276282 kJlmole. (Ans.) Both the aboue fgures are fir
Solution. (i) Air and benzene vapour:
HeatingValuec of Fuels For benzene, the combustion equation is ae follows :

Example 9.22. The lower heatng ualue ofpropaw ot constant pressure and.2"C is 2044M9 kJ
tu
/?e\N2ftr') __l?e)-NzG)
per hg mole. Find the higher heating value at cr,nsto,nt pressure and at constant uolume. c6H6G) + 7,5O2@)- = 6CO2k) + sHroG) + z.s Irt J
Solution. (i) I{igher heaing value at constant pressure, (HIIV), :
['iJ
Since the water in the products is in uapour phase, therefore, the
given value of enthalpy of
The combustion reaction for propane is witten as pressure'
to l},e lotoer hcating value a constant
ca",'-iri;;:itk;;*ri combustion corresponds

Now I
I

L.e. (uI9, = 3169500 kJ/mole


3169500
where , HIIV = Higher heatingvalue at constatrtpressue
LIfV = Lower heaing value
' tn = Mass of water formed by combustion
3169500
=4x 18=T2kgperkgmole = 2861 kJ/kg. (Ans.)
78 + 240 +79O
hfs = Latenlheat ofvapourisation at given temperature per unit mass ofwater
Now, (HIV)' = (LlI'I)o+ mh,
= 2442kJlkg at 25'C
I

.'. (HIfV)n = 2044009 + 72 x 2442 = 22198tB UJ/kg. (Ans.) where, (HIIV)p = Higher heatingvalue at constant pressue'
(ii) Iligher heating value at constan volune, (IfV), :
i
(LHVL = Lower heating value at constant pressure,
i/ m = Mass of water formed by combustion'
Now (AtD = LH - Anna?
or = 3 x 18 = 54 kg/kg mole ofel, and
-(HIID,= -(HIMp-An8o?
or (HHV), = (HIIV)p + AzRoT hit = Latent heat ofvapourisation at given temperature per unit mass ofwater
where,.Ro = Un\versal gas constant = 8.3143 kJ/kg mol K = 2442kilke at 25"C.
Ln = np_ nR .'. (HIIV)' = 3169500 + 54 x 2442 = 3301368 kJ/mole
,p
= Number of moles of gaseous products I 3301368
| 1l5, (HIfV)n per kg of mixture ='28 + 20;?90 = 2980 kJkg' (Ans')
L
n = Number of moles of gaseous reactantsJ
Now, the reaction for higher heating value is (ii) Air and octane vapour:
CrHr + 50, = 3COs + aH2O (liquid) (LIIV)e = 5116200 kJ/mole of CrHtt
Az=3-(t+5)=-Q
.'. (HIIV)" = 2219833 - 3(8.3143X25 + 27$ =2212400 kilkr (Ans)
348
INTERNAT CO/tsUSTION
ENCINES
For octane, the combustion CHEMICAL AND FUELS
equation is written as follows
:

cHrr@) + 12.50"@)---+8qo"k) gIoG) (79\ Since in case ofhigher heating value, HrO will appear in liquid phase
+ * t2.r
'lzi./\r"r
(LIrY)e per kgi of mirture (HIIv)e = 456?0 - (# #) x 8.3143 x (25 + 273)
u, 1) + rz5 x sz + r2.E xfrx n -- 45744kJlkg. 6ns.)
::..^::
5116200 (LIfDp = (Iry)p - 1.08 x 2442 = 4s7 7 4 -']'.O8 x 2442
= 1+d00+l't? =ZZS4kJtkC. (Ane.) = 43107 kJks. Gns.)
(HI{v)r=Q,LN)P+ mhr, Example 9.26. The reaction equaton of a fuel s represgnted. by
Cn!^+ aOl + bNz = \CO, + 0.9CO + 8.8O, + dHrO + 82.3 N,
(HIIV)e
= 5116200 + rcz x++z= 5511804 Determine :
(i) The actual air-fuel ratio and the ehemical formula of the fuel ;
Hence, (H[f\fo per ks of mixtue
(ii) The stoichiometric ar-fuel ratia and. the percent theoretical air used,.
= ffi, = 8010 kJ/kg. (As.)
(Bombay
You may assume Nr/Orratio in ar = 3.76 : University)
Solution. CoH-+ aO"+ N, = 8CO, + 0.9 CO + 8.8O, + dHrO + 82.3N,
",",,"Ili,?,k';1,7"1:;#::#if;i"i,,#;;#;:;:#::;:,,:::H::;::';:##
ualues. Equating coeffrcients, we have
Solution. Combustion of I kg C: n=8+0.9=8.9 ...(r)
of fuel produces the following
products : H: m =%l ...(rr)
CO"=
#x 0;88 = g.rg tg oz: = ^0.9 d
U*-+U.E+; ...(iii)
zz
Hro = Nz: b = 82.3 ...(iu)
At 25.C (u,
#, 0.12 = 1.08 ks
Assume Nr/O, ratio in air = 3.76
:
- ur) i.e, ur, = ZBMl<JlkS
hn=2442kJlkg _h
(i) (LHv)o: It means 3.76
a =
(LIW), = (Efry)o- m(u"_ u) o---toa
On solving eqns . (i), (i), (ii), (iu) and (u), we get
4'182k'/kg
.T::::_
i,Gtryl":, (Lrrv)p:
(LHv)" ==ff##;t ?fl1; a=2I.89,
=18.56. n=8.9
b=82.3, d.=9.28

ne conbustion equation
is written as follows (j) Chemical formula of fuel is : Cr.rHrr.r" (Ans.)
:

l moleruer *
#,o"___rT "or. S
Actual air-fue,'"*to =ff =Jffiffif = 2a.er. (Ans.)
"ro (jj) Stoichiometric air-fuel ratio
L.,,
{ = 3.23*
'z r.oe
44 t8x2 Stoichiometric air-fuel ratio can
:
be found by finding the theoretical air required for complete
or jr combustion of 1 kg offuel. Hence,
= B.g1 kc
1 kg fuel + 3.31 kg O, g.2gCO2 Ca.g Hrg.se + 13.54 02 = 8.9 COz + 9.28H"O
= + 1.08H"O
Also, Mass ofoxygen required for combustion of 1 kg fuel
MI=AIl+LnR^T
i.e., 73.54 ,32
or
-
Iwt, = - (HIn4, i a,,nor
(HIrvL=(HlIg,-a:zo7
_
- 8.9 x 12 + 18.56 x _ , ,.^
where, Ln=np-nR Mass of air required for complete combustion of 1 kg fuel
3.456
= -:-::-
,,J
Al)
- tr,i fp = Number of moles of ga 0.23 =
15.03 ks
= |. *
_ f!.28 _ .3

[r, =No-b""or-orurof *":::::ill,lTtl


I
351
INTERNAL COMBUSTION ENCINES CHEMICAL THERMODYNAMICS AND FUEIS

Stoichiornetric air-fuel ratio = 15.03 kC il.4. Theprmesjsofseparatingntrolemintousefulfractiomandremovalofundesirableimpuritiesismlled


: refining.
iscommonly define d as the euaporating tentznc! of a tiquid tuel. The volatility
of gasoline is
Percentage theoretical alrused = * 100 = 169.487o. (Ans.) 16, volatilit|
## generally characterised by the foltowing two laboratory tests :
(i) ASTM distillation tst (ii) Reid vaPour tcst.
engine. TTre vapour locl< tendency
16. Vapour lock is a situation where too lean a mixture is supplied to the
of ihe gasoline is related to front end volaility.
pressure pulses.are generated' The
L7. When oelf-ignition tloes ccur in S.I. engine higher than desirable,
Chemicl Thermodynanics t ighur'a*'e s"r-ignition tempenture whictr ttre fuel is treated, the shorter
will be ignition delay.
A chenical reution maybe defied ag e rearangement of atoms due t redistribution of electrom. Tltehiehest urefulcompwion rutia(IIUCR) is the highest compression ratio at
whi a fuel mbe wed
'Reactants' comprise of initial constitueDts which stt the reaction while 'products' comprise of final ;;;;;;;;;r;"i"'" t""t-engine uder sipecified operating conditioro and the ignition and
constituents which are formed by the emical reaction. "p.n" to give best efficiency.
rnixture strength being adiusted
2. A chemical fuel is a zubstance which eleases heat eDerey on combustion. 19. Thepropertyofafuelwhichclcgibeshowfuelwillorwillnotself.ign'iteiselledtheOctarenumberor
3. The total number of atorns of ead element ocemed in the combustion emaim constaat, but the atoms i*t b"orr. ntstnes with low comPession ratioe cau use fuels with lower octme number' but high-
'"o*pi"""iot
are rearruged into groups having different chemical prcperties. mwt use high'octane fuel to avoid self-ignition ad knock'
""te
The difference in octae number between reseae met'hod and motor method cte
numbers is knom
The amout of excess air supped varies wi the type of the fuel and the firing condiou. It may
approach a value of lfi) per cent but moden practics is to use 25% to 509 excess ai. as the fuel sensitiuitl.
Stoichiaretric (or dremically corect) milro of air md fuel is one that contaim jut sufficient oarygen RON+MON
for complete combuson of the fuel. 21, Antiknockindex=- 2 .

22. Hish octane fuels (upto 100) caa be prorlue*d,by rcfining tcchniques,,bttt it
Stoichiometric A / F ratio is done moe cheaPly, and
6. Iux'ue strengtn = as Etr@thfl lzad'
A"t*t A/F r"h" -o?" fr"qteutty, by ihe ue of mtihocL ddifiws uch
When analysis of combustion products is hown air fuel ratio can be calculatd by the following mthods : 23. Cetane lumber ofdiesel el is a neasure ofits ignition quaty'
noise levels. In general,
(c) Fuel composition knm In general, lmer e cetme nmber higher ue the hydrocarton emissions antl
a high octme value impes a low etsne value.
(i) Carbon balance method (ii) Hydrogen balance method
API El:iJv (deg)
24, Diesel index (D.I.) = Aniline point ('r, t
(iid) Carbon hydrogen balance method
() Fuel composition ulmom
(i) Carbon hydrogen balane method. Alternative Fuels
8. Ttre most common means of malysis of the combugtion products is Omat appaatw. 26. Some altemative fuele whidr can replace conventioal fuels in I'C' engine are :
9. Theenthalpy offornatio (AlI.) i the inqease in enthalpy when a compound is formed from its constitu- (i) Alcohol (methyl and ethyl) (ii) Hytlrogen
ent elements in tleir natural ?om and in a standad state. Ttre standard state is 25'C and I atn. pes- (iii) Natural gas (iu) LPG andING
eure (but it mwt be bore i mind ttrat not all substances can exist in natural fom, e.g. II"O cannot be a
vapour at 1 atm. md 26'C). (u) Biogas.
26. The power output ofethanol engiae ir higher compared to gesolitre engine at all
6Peeils'
(HHV),= (LH9, +m r.
fulled by gasone'
(EHD,=GHD,tm(r-u) 27. n CNC fuet *etl engine have highel thermal efciencies t'ha those
" in excess of 120 com-
where, HIIV = higher he.ating value 24. Biogas possesses excIletrt antiklock properties with an equivalmt octane number
pared with 8? for regular Petrol'
LIV = lePs hsting value
n = mas6 of water fonned by combustion
= enthalpy ofvapourisaon ofwater, lcl/kg OBJECTII'E ryPE QTIESTIONS
u, = specific intemal energr ofvapour, kI/kg
ur= specific itemal energy of liquid, kl/kg. .d Choose the CotrectAwer:
11. In a given ombution prccess, that takes place ailiabatically and with no work or changes in kinetic or The smallest particle whiclt can tate part in a ch'emical change is calleil
potmtia(energyinvolved, the temperahre ofth products is refened to asthe'adiabaticflm Enpera- (o) atom () molequle
ture', (c) electro (d) compound.
For a given fuel and given pressue and temperature of the reactants, the maximum adiabatic flame 2. A chemical fuel is a substance whidr releases .....' on combustion'
temperature that can be achieved is with a'stoichiometriC mixtue. (a) chemical energY () heat energy
(c) sounil energY (d) magnetic energy.
Conventional Fuels v.
I

13. Th.e constituents petroleurn are classified into the following four groups :
I S. l,l:emostimportantsolidfuelis
(a) wood (b) arcoal
(i) Parans (ii) Olefis i

(c) coal (d) all ofthe above.


(iii) Naphthenes (iu) Aomatis.
J
INTERNAL COMBUSTION ENGINES CHEMICAL THERMODYNAMICS AND FUET^S
J]J
For each mole of o:rygen, number of moles of nitrogen
requirr for complete combution of cabon ue B. Fill in the Blanks or Siry "Yes" or fio":
(d)20/2I
@)2nl l. The liquid fuels find ....., use n LC. engines.
(c) 77/21 , ,r i^h j
@)79n1' 2, ...... gas is manufactured by heating soft coal
Modem practice is to use....., excess air. in closed, vessel.
(a) 5 to 10 pe cent 3. Water gas is fomed by using steam.
() 15 to 20 per cent
(c)
\v/!v vvzuFqr!
20 to 25 per cent 4, ...... gas is a by-product ofsteel plants.
@)25ta50percet.
6. Stoichiometric air-fuel ratio by maes for D. Benzol is obtained as a by-product ofhigh temperatue
(c, D
mmbuti""
"ip"i
() 10
i"
6.
coal crbouization.
Olefins are compounds with one or more double bonded cabon ttom
in shaisht cha\'
\
b)t2 (d) 15.05. 7, Diolefins are more ...... thm mono-orefus.
An malysis which jnclude the steam in the exhaust 8. Napthenes are ...... stryctured compouncls.
is callerl
(a) dry analysis 9. Napthenc are usatuated compouds.
() wet anlysg
(c) dry and wet malysis
(d) noe ofthe above. 10, Paaffrns are most suitable fuels for ...... engines.
8, TheOrsatapparatusgives 11. Aomatis ae nost suitable fuels for ...... engines.
(o) volumeric malysis of the
dry products of combustion L2. petroleum into useful fractions and removal of unrlesirable impuities is
() gramehic malysis of called
the dry products of combustion Lll :t*"" "tt"oarating
(c) gravimetric analysis ofproducts of combwtion
incluiling HrO 18. . ...... is the evaporating tendenry of a liquid fuel
(d) volumetric analysis ofproducts
ofcombustion including I{O. 14. The...... of petrol is also defined in terms of Reid vapour pressue.
9. In the Orsat apparatus KOH solution
() carbon monoride
is used to absorb 15. ...... is a eituation where too lea a mixtue is supplied to the engine.
(c)oxygen []::Hrr,ilI*""'*
16. The vapour lock tendency ofgasoline js related to ...... end volatility.
17' when self-igrrition does occur in s.I. engine higher thm desirabre, ...... purses are generated.
10. Enthalpy of fomation is defed as enthalpy of compounds at 18, The higher above S.I.T. which the fuel is heated, the longer will be iguition
(o) 25"C and 10 atrnospheres ilelay.
() 25"C mil 1 atmmphere
(c) 0'C and 1 atmospherc 19. Igaition delay is generally a very small fraction of a second.
(d) 100"C and I atmosphere.
11. Bomb calorimeter is used to fid the calorific 20' The property offuel which describes how fuel will or will not self-ignite is elled
value of ...... fu els. the ...... numbe.
(o) solid 21. The higher the oclane number offuel, ilre less likely it will self-ignite.

12'
(c) sorid and
saseous
when the fuel is bumed end the water appears
l3]1J$*" 22' T\e difsence in octane number between reseach metJod and the moto
known as the......
method octme numbers is
in the vapou phase,"0",..
the heating value of fuel is called
.
() enthalpy
offormation () lower heating value
28' Fuel semitivity is a good measure of how semitive knock characteristics
of a fuel will be to engine
heathgvalue
(c) higher geometry.
. (d) none ofthe above,
13. Heat eleased i a reaction at costant
(o) entopy change
pressure ie called
24. Anriknock ira"* =
I9[#9I .
() enthalpy ofreaction
(c) intemal energy ofreaction 26. The cetane numbe ola diesel fuel is a measure ofits
(d) none ofthe abov ignition quality.
(e) all ofthe above. 2A, Higher the cetane rating ofthe fuel lesser is
the propensity for diesel knock.
14' when the fuel is burned md water is eleased in
the liquid phue, the heating value 27. In general, a high octane value implies a high cetane
(a) higherheatingvalue of fuel is caltetl value.
() lowerheatingvalue 28. Methanol is very corrosive on metals.
(c) enthalpy offormation
(d) none ofthe above. 29, Ethanol has less HC emissjons than gasoline but more
15. Choose the correct statemm[: than methanol.
30, The CNG used engines have lower themal effrciencies
(a) Number of atoms of each corotituent than those fuelled by gasoline-
are not consewed in a chemical eaction.
() The mass of alr the subetances
on one side ofthe equation may not be equal
substances on the other sirle. to the mms of an the
(c) he numberofatoms ofeach
constituont are conserved in a chemical reacon.
(d) The numberofmoles ofthe l. abundant 2, coal 3, Yes 4. Blastfurnace
reacants in a chemiel equatioa ae equal to 5. Yes
products. the number ofmoles ofthe 6. Yes 7, unstable 8,ring 0. No. 10. c.I.
rl. s.I. 12. refrning 13. Volatility 14. Volatility 15. Vapour lock
I ANSWERS I 16, front 17. pressure 18. No. 19, Yes 20. octane
I (a)
21. Yes 22. fuel sensitivity 23. Yes 24. No
2. (b) 3. (c) 4. (d) 25. Yes
8. (o)
5. (d) 6. (d) 7. (b) 26, Yes 27. No 28. Yes
e. () r0. () 11. (o) 12. (b)
29. Yes 3O. No.
15. (c). r3. () 14. (a)
CHEMICAL THERMODYNAMICS AND FUET.S

IHEORETICAL QI,IESTIONS 36. Give the advantages ofusing.alternate fuels.


37. Discws different properties of ethanol and rnethanol and compe them with gasoline.
Chenical Thermodynamics 38. Why blends of either ethmol or methanol are prefened over pue alcohol fuels ?
l. What is chemical themodmamics ?
39. Give the advmtages of alcohol as a frel.
2, What is a cheniel fuel ?
40, List the advantages ofmethanol as a fuel.
3. What are primry fuels ? st some important primarr fuels.
41, What modihcations in agine are required when blends are ued ?
4. What ae seondary fuels ? List sore importmt *condary fuels.
42. State the advantages and disadvantages ofhydrogen as I.C. engine fueL
5. Write a short note on'exces ai/.
a1{1. What is natural gas ?
6. What do you mean by stoichiomtric air fueI (A/T) atio ?
44, What are the prcperties of CNG ?
7. Enumerate the metho& by whidr air fuel ratio can be calslated when analysis of combustion products
45. What ue the advutages and disadvantages of CNG?
is known.
8. How is analysis of exhaut md flue gas eried out ?
46. ExplainbrieflvLPGandLNG.
9. Derive rel.atioro for internal energr md mthalpy of reaction.
47. What is Biogas ?
lo. What is enthalpy offormatio (AlIr) = ?
48. What are the properties ofbiogas ?

rt. Define heating value offirel.


L2. What is the diffeene btween higher heatirgvalue (HIIV) and lower heatingvalue (LIM ofthe fuel ? UNSOLVED EXAMPLES
ts. Describe with the help ofneat sketches the following calorinetem used for e deterrination ofheating
values : Detemine the gravimetric analnis ofthe products of complete combwtion of aetylene (C.H") with 12b
(i) Bomb calorimeter (ii) Junkers gas calorimeter.
per cent stoichiornetric air. .
[Ans. CO, = ]rg.S%,IJ'.O = g.fr%,z= 4.4V;,r= 7Z.Z%l
2. One kg of.eihane (CHJ is bumed with 80% of theoretical air. Assuming complete combustion of the
14. What is 'adiabac flame tenperature'? hydrogen in the rel detemine the volmetric analysis of the dry products of crnbustion.
15. Write a short note on chemical equilibrirn.
[A,ns. CO, = 4.8%,CO =1L.2%,Nz= 84Vo)
Convsntional Fuels 3. he gravinetric analysis of a sample of coal is given as go% c, Lz% H, and g% ash. calqjate the
stoichiometric A./F ratio and th9 ana\rcis of the products by volume.
16. What are the desirable properties ofgood I.C. engines fuels ?
17. Enumeate md describe briefly the gaaeous fuels. [Ana COr = 13'6%' IL = 72.2%,N"= 74.2Vol
Calculate the stoidriometric ai fuel ratio for the combustion ofa sarple ofdry anthracite oftie follow-
18. How ae sonstitunts ofpetmleum clssified ? ing composition by nms :
19. Explain briefly the chemical etructure of peholem. C = 90 per cent i
20. Whatarefiveprimaryhydroeronfhrriliedfountlinpetmleum?Whicharedraintype.s?Whicharering 4 = 3 per cent ; N, = I per cent ; Sulphur = 0.5 per cent ; ash = 3 per cent.
If 20 per cent excess air is supplied determine :
typesWhidr of primary.fmilies tends to b beftr S,I. engine fuel and C.I. engine fuel
? ?
(i) Air tuel ratio
21. What are difierent kintlg of fiels used in an I.C. engine ?
(ii) Wet analysis ofthe products ofcombustion by volume.
22. What are the important pmperties which S,I. mgine fuel possess J
2{1. What are requirements of ar ideal gasoline fuel ? fAns. lr.2y1 () 13.5/1 ; (it) CO, = 16.3%, IIO = 0.0A%, Se = 3.5196, 80.3Vol
=
24. What is volatility ? o. the following is the analysis of a supply of coal gas :

25. Discuss the signifimce of distillatim curve.


H = 49.4 pe cent ; CO = 18 per cent ; CH. = 2Q pe 6mt ; C.H" 2 pe cet
= ; q
= 0.4 per cent ; N2 = 6.2
per cent; CO, = 4 per cent.
26. Why volatility is an important quaty of S.I. engine fuels ? (i) Calculate tle stoidiometric AIF ratio.
27. Explain briefly the following ln regrd to a fuel : (ii) Find elso the wet and dry malysis ofthe proclucts ofcombustion iftle actual mixture
(i) Vapour lock chamcteistic. is Z0 De cent
wealc [-{n& (t) 4.06in by volume ; (ii) Wet anstysis : Cq = 9.0%, I4O =t7.5%, Oz! 3.OBE,,
(ii) Crankcase dilution.
N z = 7 0.4Vo. Dry atalysis : CQ
28' = 10.996, O z = 3.7 2%, N z = 85. 4Vol
"IVhile volatilitr of the fuel is a iletermining factnr in the selection of fuels for S.I. egies, ignition 6. Find the stoiiometric a fuel atio for the combustion of ethyl alcohol (C"H.O), in a petrol engine.
quality of the fuil iri the primary deciangtactor'for C.l. mgines'. Disruss briefly the statements. Calculate the air fuel ratix for the exteme mixture strengs of-90% and lzo. beierntie also t-e wet
29' Distingqish clearly between'Octane Nunbers'and'Cetane Nurbey'. What is fhei eignicance in rating and dry analysis by volume ofthe exhaust gas for each mture strength.
offuels lbr S.I. and C.I. engines ? [Ans. 8.96/1 ; 9.95/L ;7. Ztt,lilet
atralysis : CO, = 11.2 %,HrO = 16.&Vo, Or= l.8Vo,Nr= Je)/
80. What are the refeence fuels for'Octane Number'? Dry analysis : CO- = 13.45%, O z = 2.22%, N, = gl, ll .
31, What are the reference fuels for'Cetane Number'?
Wet analysis i CO2= 6.94qo,CO = 6.94lo,Hz=20.8Vo,N"= gg.SV
32. What is perforinance number (PN) ?
Dry analysis : CO2= 8.7Eo,CO = 8.7%,N2= 82.5%l
33. What is the significance ofASTM distillation orve ? 7. For the stoiiomekic mixture of Example Z.l0 calculate :
34. Explain the effect of fuel viscmity on diesel mgine perfomece. (i) The volume of the mixtue per kg of fuel at a temnratue of 65.C md pressure of 1.013
bar.
35. What qualities are desed in fuels to inhibit detonation ? (i) The volume ofte products ofcombustion per kg offuel after cooling to a tempemture
of 120.C at a
pressue of I bar. tAns, (j) 9:226 m3 ; (lj) 11.58 msl
356 INTBRNAL COMBUSTION ENGINES

8. The chemical analysis ofa fuel by weight is as follows :


Carbon=50pecent;Hydrogeo=2Spercent;Orygel=l0percent;Sulphu=5percentandNitrogen
= 10 per cent.
Find tlre stoichiometric amount of air required for complete conbustion of rri fueL [Ans. 14.26 kg]
9. The percentage composition ofa fuel by weight is as follors :
Carbon=89.3percent;Hydrogen=6percent;Orygen=4:2percent;Nitrcgen=l.Spercentandthe
remainder aslr" Determinc the stoidriometric air fuel ratio by mass.
10
If30 per cent excrss air is supped, find the percentagu composition
ofdry flue gases by volume. Fuell Air Minture Re quirbtnents
[Ans. 11.74; CO, = 14.3%,O,=4.9%,N = 80.8%l
10. Orsat analysis of the products of mmbustion of hyrlrocarbon fuel of uknowu composition is as follows : 10.1. Introduction. 10.2. FueUAir mixture requements for steady rumiag. 10.3. Optimum
Carbon dioxide (COr) = 9% Carbon monoxide (CO) = 0.6% fueuair ratios. 10.4. Idling md low load. 10.5. ormal power range or cruise iange. 10.. Ma*i-
Orygen (Or) = 7.3% Nitrogen (\) = 83.1% mrm power range. 10.7. Transient nixture requirements-starting and wming up nixture
Determine the following : requirements-Mixtwe requirements for acceleation. 10.g. Effects of openting variables on
(i) Air-fuel atio (ii) Per cent theoretical air roquired for combustion. mixtue requirements. 10.9. Mixture requirements for diesel engines_Highlfuht"_b"*iu" Typu
Questions-Theoretical Qusons.
lAns. (i\ 22.1, (ii) 146.21o1
An Orsat analysis ofthe exhaut from m engine running on benzole showed a CO, content of 15 per cent,
but no CO. Assuning that the remainder ofthe exhaust contains only orygen and nitrogen, calculate the
ai-fuel ratio ofthe engine.
IO.1. INTRODUCTION
The ultimate alalysis ofbenzole is C = 90 per cent ad 4 = f0g6. . In geDeral, we have already discussed in previous chapters about the profound influ-
i [4ne.15.2/1] ence ofF/A ratio on S.I. engine po\er ou[put and thermal efficiency. The discussion of
12. lhe analysis of the dry exhaust ftom and internal-conbustion engine gave : adequate F/A ratio for each particular Bet of operating conditions iS studied under the
COz--72per@nt;CO=2percent;CII.=4percentiH=1pecent;Q=4.Spercentandthe following two heads, namely
remainder nitrogen. (t) Steady running;
Calculatethe proportioru bymss ofcarbon to hyilrogenil thefuel, assumingit to be a pure hydrocarbon. (ii) Transient operation.
lAns.7.35/1)
13. The following is the percentage analysis by mass ofa fuel :
a Steady run t0,ing is defi,ned as mean continuous operation at o required. speed. and, power
output with norma,I temperatures,
Hydrogen (Il) = 10 per cent Oxygen (Q) = 2 per cent
Sulphur (S) = 1 per cent Nitrogen (Nr) = 3 per cent
o Transient operation includes stafting, warming up, and changing from one speed or
Determine the following : load to another, specially for automotive vehicle engines during acceleration and,
(i) ltre amount of air required to completely bum I kg ofthis fuel decelerations, and. also id.ling,
(ii) The products ofcombustion s a percentage by mas.
10.2. FI.IELIAIR MIXITjRE REQTIIREMENTS FOR STEADY RUNNING
[Ane. 13.1? kg ; COr=21.1% tHrO = 6.357o ; SO, = 0.147Vo;N"= 7L.75Vol
14, An Orsat analysis of the products of combustion resulting from the buming in air of a hydromrbon fuel The specific torque or b.m.e.p. (brake mean effective pressure) is required to be devel-
yielded the followilg: oped at specific speed. Frthe it is desired that fuel consumption should be lowest,
Carbon dioxide (COr) = 12.2% Oxygen (Or) = 1.17o ensuring reliable and smooth operation. These requirements cn be met with by using
Carbon monoide (CO) = 0.5% Nitrogen (Nr) = 86.2% optimum F/A ratio.
Determine : (i) The mass fraction of carbon in the fuel. o Fig. 10.1 shows the graphs ofi.rn.e.p. (nd.icated mean effective pressure) and, s.f.c. (spe-
(ii) Air-fuel atio. cific fuel consumption at fi*ed. engne speed. at fuil throttle open-.
(iii) Percent of air theoetically needed for complete corbustion. The left portion of the curve exhibits the limit of lean misture which causes etplo-
lAns. (i) 80.47,, (ii)16.57/1,(iiil r03.9Eol sions in the intahe systern, usually hnown as back_firing.
15. If the higher heating value at mmtant pressure (C.Hr) at 25"C is 3298354 kJ&g mole, determine its Depending upon the type ofengine, type offuel and operating conditions, the limits
lower calorifrc value at constant pressure. [4ns.316647 kJ/mole] on lean and rich side and also F/A ratios may vary.
16. The lower heatingvalue ofpmrane at constant pressure and 25'C is 2042055 kJ/kgmole. Find the higher
heating value at constant presure and at constant volme. The curve shown in Fig. 10.1 is a representative one since its shape is same for all
S.I. engines. It may be noted that, for getting these culves optirnum spark advance
fAns, 2217816 kJ/kg mole, 2210333 kJ/kg molel
is used for each F/A ratio. Ifthe spark advance is kept fixed- fo the best F/A ratio,
The higher heatiagvalue ofkerosene at constant volume whose ultimate analysis is 867o carbonandT4Vo
hydrogen, was found to be 46890 kJ/kg. Calculate the other three heating values.
this curve of lean and rich ends will get modified.
[Ans. (LITV), = 43987 kJ/kg ; (HHV), = 46977 kJ/kc ; (l,HV), = 43900 kJn<g]

J3t
FUEUAIR MIXTTJRE REQI'IREMENTS 359

7.5
I.m.e.p.
o IthasbeenobservedfromexperimentsthattheF/Aratiowhichgrvesthehighesti.m.e.p.
(Refer Fig. 10.2) is more or less the same ag the F/A ratio which gives the highest b.m.e.p.
7.O
(Refer Fig. 10.3). This indicates that f.m.e.p. (frictional mean effective pres8ure) is nt
A( affected by F/A ratio. Hence for best power under all operation conditions the F/A ratio
is same subject to the condition that distribution offuel to various cylinders remains
unchanged. In catburetted engines this requirement cannot be met with easily. At lean
f+6.0
tl
ll0-4
mixtures, the effect oflower flame speeds is small when, at each F/A ratio, spark timing
is adjusted for highest m.e.p. (meaD efrective prssure)'

5.;
"* s.f.c. / Full lhro ills

S#o'3
0.2

-<-
i**
+
| 3.0
I
lrn'm"
Fn ---} b :.
I
,{
3 z.o

Fuel/Ar ratio rfi

Fig. 10.1 --\a< Irrtrtt"

Full Throt(
1.2 1.4 1.5
7.0

Fn -----'-)
+I 3/4 Throttl
| 6.0
I
I
0.04 o.o5 0.06 0.07 0.08 0.09 0.10 0.11 0.12

12 Throft Fuel/Airrallo -----*


.i s.o
Fig.10.3
Fig. 10.4 shows graphs between specific fuel consunption versus F/A ratio at different

3.0 L
0.6

Fn
1.0 1.2
a -lll Throie
throttle openings.
-

-
The minimum specific fuel consumpion occure at a point where, as the F/A ratio is
reduced, the rate ofincrease in efficienc5r due to thermodynamic factor, is offset by
rate of decrease in eficiency du e to increasing time osses .
With optimum spark timing, the time losses becone very large only when crank
angle occupied by flame travel exceeds a certain value. As the mixture is made
-------* leaner, this value will be reached at high F/A ratio when the flame speeil has already
0.06 0.07 been slowed by throttling. Thus ony fa.ctor that tends to increase the crank angle
0.08 0.09 0.10 0.r1 occuped bX the combustion (.e., reduce the flamc speeds), wll tend to ncrease the
. FueUAir ratio _-f F I A ratio for best economg,

Fig. 10.2
L
--__

INTERNAL COMAUSTION BNGINES


FUEUAIR MIXTIJRE REQUIREMENTS 361

o Fig. 10.6 shows a plot between load (ratio of actual b.m.e.p' to maxirnum b'm'e'p') and
F/A ratio.

t| . t.2s
At all spr ds
Bt rsl Power
I
\L
ldling
rangs
tl
{s
r'0
*
E o.7s

{ +
0.07
/
# 0.5 |
I \ \ B Bes t praclic

E ^^^
u.uo
/ /
s
T
I
0.'l Bsst e( ,lCnomy
Fuet/Air ratio ____|
Fig. r0.4 0.5 o.7 1.0

rO3. OPTIMTIM FUEI./{R RATIOS


Load (brake basls)
o Fig' l0'5 *tX1n:.li-leen b.m.e-p.
observed "no*t
that alr the curves are sim'a Td T/t ratio for different speeds. rt may be
which leads to the concrusion that
Fig.10.6
-'
economy FIA rotio is independent
of speed.,
the es Startiug with idling range, the curves show the F/A ratios in the iclling iange, best
power, best economy etc, These curves are prac tically independent of engne speed'.

10.4. IDLINGAND LOW LOAD


o The no load. running mode of engine is called' idling. The air supply' during idling, is
restricted by the nearly closed throttle and the suction pressure is very low' This condi-
tion of low pressure giues rise to backflow of erhdt,st Eases and ar leakage from uarious
parts of the engne intake systern.
o During idling and low load operation backflow during the valve overlap period occurs
since the exhaust pressure is higher than the intake pressure, this increoses the amount
of resitluol gases. These gases expand during the suction stroke, thereby reducing the
fesh mixture inhaled. The increased dilution causes the combuson to be errotic or
even impossible which leads to poor thermal efficiency and. higher e*haust etnissons.
At idling and low loads the problem of dilution by residual gases becomes more
pronounced because the exhaust temperature reduces with decreasing load, i.e. the
density and hence mass ofresdual gases increases. Further, dilution ofthe charge
occurs due to ar leakage post ualues, etc., at low intake manifold pressures ob-
tained at low loads and idling.
B.n.e.p, bar

Fig. 10.5
362
INTERNAL COMBUSTION ENGINES FUEUAR MIXTURE REQUIREMENTS 363

The Al F ratio used for idling.and. MD(TURE REQUIREMENTS


Iow load,s (up to about 20% of full load) s/o ld 1O.7. TRANSTENT
for smaoth operation (F/A ratio 0.08 be eh
or A/T ."U.-= l!.i , f l.
Trte richening of mi.rture increas_es o Transient conditions a,re those conditions at which, speed,, Ioad, temperotures or Pres'
thz probabitity of contact between sures are abnormal or changing rapidly, like in :
paicles and thus improues combustion, fuer and air
The lower idling speeds demand increasingly starting ofan engine ;
richermixtures with consequent incease
in CO and HC emissions. In rich region, warming up of an engine ;
Cii t"";;;";" ; o 2.8%,per unir decrease in
A'F ratio' Thus c/ose torerances acceleration ofvehicle (i.., increased in load) ;
on carburettor jet sizes and. on i-dring contrors are
rnand,ed to ovoid, pollution of ar. d.e-
deceleration ofvehicle (.e., decrease in loarl).
-
r0.5. NOT',MAL POWER RANGE
OR CRUISE R,NGE
o The transient mixture requirements are differer,t from steady running mixture require-
rnents because in the former case the euoporction of thz fuel may be incomplete, the
is running at part Ioads (from 2Ogo to TEvoof qua,ntty of liquid fuel in the nlet manfold moy be ncreasng or d'ecreasing, a'nd' dstri-
tne :_tl.:-le
marmum ecorutm!, Thuq the FlA ratio
rared load) theobjectiue is
butian of fuel to uarious cylind.ers may be diffcrent.
specific for best thermal effrcienqt o,
fuel consumptinn s selected and incidentryiii, ,otio gir"" ^n^u*
carbon) emissdos. *inimum HC (hyd,ro- 10.7.1. Starting end Warming Up Mixture Requirements
As shown in Fig' 10.6, the F/A ratio 0.06
is the best compromise for various part loads It has been observed that a very cold engine generally requires abnormally rich mixture at
operation ofa modern S.I. engine (Actual the carburettor in order to secure ring mture in the cynder. Thus the carburetion system must
best ecorrooty li*tu." laboratory level works
out to 0.055, but best practical is chosen), "t supply very ch mixtures for starting and the F/A ratio must be progressively reduced from this
point during the warm up period until, the erigine will run satisfactorily with the normal steady
10.6. MAXIMUM POWER RANGE
running F/A ratios.
The maximum power range lies b etween During starting a very rich mixture must be supplied, as much as 5 to 10 times the
full opened, the F/A ratio has to be nade
7E(% to ro'vorated power. when
the throtre is - normal amount of petrol (ALF atio 3 : 1 to 15 : 1, F/A ratio 0.3 to 0.07). With the warm-
rich"., -.."lrJ-r*imum torque is required at
a given speed, or maximum-b.m.e.p. ing up of the engine the amount of evaporated fuel increases and hence the mixture
coolan.t to prevent ualue
is
"uq"i"".
M;;;; er, tne richer mirture serues qs ratio should be progressively made leaner to avoid too much evaporated F/A ratio.
failure_u"a, tn"."fo.",
thottle valve is wide open as shown by
f-"i""la be made richer before the
C-D p;;;; olir,"
-' .r*o in Fig. 10.6. Incid.en, 10.7.2. Mixture Requirements for Acceleration
tally, too rch mi.rtures inhibit NO,ani
*oroton-.' o With regard to engines, the term'acceleration'is generally used to refer o an increase
The rnixture requirement for maximum
power is a rich mixture of A.T about 14
: I or in engine speed resulting from opening the throttle. f|s in purpose ofopening the
FlA = 0.07.
throttle, however, is to provide ao increase in torque and whether or not an increase in
o In multi-cylinder eugines the A/n ratios are srightry lower (i.e., speed follows depends on the nature ofthe load.
to overcome maldistribution of air fuel sligh'y richer mixture)
mixture in different cylinders. With constant speed engines the throttle opening increases torque or b.m.e'p. at
o In case of super-charged engines the best
economy mixture is leaner than its naturally the governed speed.
aspirated counterpart over the whol.e
operational range. o In order to impart acceleration when the throttle is opened, the manifold pressure in-
Steady Running Mixture Requirements
in practical Operation creases, and fuel must be supplied to increase liquid content of'the fuel ofhe manifold.
o Refer Fig' 10'6' s-.I. engines using gasoline show best Ifthe carburettor supplies costant F/A ratio, the F/A raio going to the manifold will
to one shown in T"
the figurq particurarry, in the absence
economy F/A ratio very close
becomes less during the time the liquid conteut of the manifold is being built up to a
good distribution of fuer-air mixture ofsevere detonation and with
fiom manifold to various cylinders at alr speeds larger value when the throttle is suddenly opened. The recluction in the F/A ratio to the
ad loads. cylinders can be such as Lo cause msfiritq, backfiring, ot even complete stopping of the
o The followrng are the reasons due to which
the departure fom the best-brake economy engine.ln order to uoid such a situaton it is often found essential to increase the supply
curve shor$n in Fig. 1o.6 may . p.".tiJlil""u ratio by i4jecting into the manifold a quantity of fuel hnown as uAccelerating charge'.
,
(i) To compensate for poor distribution. The optimurn atnount of accelerating charge is that which giues best power F / A rdtio in
(il) fq 1., for possible errors or variations the cylind.er.
in carburettor metering.
(iil) Te uor" the temperature of hot spots
such as exhaust varve, splrk plug points or
piston crown, to help in cooling.
(lu) Te sr.. or eliminate detonation,
364

FUEUAIR MIXTURE REQUIREMENTS 365


g"r."al, the required F/A ratios for various running
b"lo,o ,' conditions ofthe s.L engine are ristcd
9.0
2.O
8.0
1.75
0.2: 4

0.15 :
L

I
I t'o
1.5
I

0.085 ; I |
I
6.0
,/ 1.2
Running with maimum thermal efficiencXr (gM
throtth) I

Runnitq zuith deuelnping mzzimwn power (gT_Co


throttlz)
0.06 to 0.07 : 1
0.0775 to 0.08 : I
I u'o
// /, .0
,
FulI thrcttle
0.085 : I
I
4.0
,l x
Acceleration
0.1 :1 :F
"E
3.0
2
10.8. EFFECTS OF OPERATING
VARIABLES ONIWXTURE REQUIREMEI{IS
E
2.0 ,r,
,l v
variables such as intet and exhaust pressures,
**rr"T"? "O[".",[JlT"*tl, spark timing and 1.0

1, Inlet and exhaust prebsure:


o Reduction in inlet.pressure d,ue to.throttling
or opterating at dltitudes lead,s to red,uction 0 0.2 0.4 0.6 0'8

o
in flame speed, and increase tn F t e, ratio'fir
Incease in exhaustpressures result in
best economy. =-
reduced flame speeds and increase
for best economy. in F/A ratio Fis. 10.7
r With constant throttling, simultaneous dec:ease
ofinlet and exhaust pressures at high In diesel engines the fuel injection spray cnvelope contains in its evaporated portion, a
altitudes also affects th F/A ratio a"p"nio-g--opon heterogeneous mixture n'ith local F/A ratios varying from F" = 0 to F = -' Thus con-
the net effect.
2. Spark timing presson ignition of most fauourable local FlAratio occurs initioting the flame which
later serves os uery hot source and. ossists in flame stabilisaton and burning of fuel'air
timryg wirr tead. to the intease
,",r,, #:"f";i;t:#Z:,Azi!#,K::arh of best economy FlA mi*tures. The practical high limit of overall F/Aratio, however, is set by smoke and
deposits.
3. Friction
Tbe smohe free conbuston is seldom obtained aboueF *= 0'8 and' most diesel engines are
i'm'e'p' constant, the increase in f.m.e,p. will neuer rated. aboue this limit of F I A ratio for continuous operation,
r", u""flffi}.rthe result in the increase of F/A ratio

10.9. MD(TURE REQUIREMENTS


FOR DIESEL ENGINES
o An engine, under normal operating conditions
and with iqiection timing frxed and 1. Steady running is defmed as mean continuons operation at a required speed antl power output with
b.q.e.p., will require o.r" F/A one riormal temperatures.
fir" .71 ,
""tio. j:;#,,JJ:#i;,:ru 2. Transient operation includes starting, warming up, and chmging fom one speed or loail to another,
*niH.mmf:l*fur:::,.;i,ffii,li:H specially for automotive vehicle engines during acceleration and decelerations, and also iclling.
Fic 10'? shows graphs between 3. The dchening ofmixture increases the probability ofcontact between fuel and air particles and thus
- i.*'".p.
F* for various factors of the products
tY]:]f..:lp" l?r: "" urrd
of po x tuor.
.r,or.ii for elow anJ aioJ" utn'orpr,"rt. 4.
improves combustion.
The smoke free combustion is seldom obtained above Fn = 0'8 and most diesel engines are never rated
in order to covei high attitude
l"?*!:*' ""-";;;;;";"i ii1"0"."r,,"r.a above this limit ofF/A ratio for continuous operation.

The dieser engines do not show any definite


low limit on F/A atio and can burn fuel-air
mixtues with very lean F/A ratio. OBJE CTIVE TYPE QUESTIONS

Fill in the Blanks or Say'Yes" or'T.Io".


l. ............. is defmed as mean continuous operation at a required specd and power output with normal
temperafues.
366 INTERNAL COMBUSTION ENGINES

Any factor that tends to increase the crank mgle ccupied by cornbustion, will tenrl to increase th F/A
ratio for best e@nomy-
3. The no load ruming mode of engine is called ............. .
4.

D.
Ihe increased dilution ofarge dw to exhaust gases causes tle
sible.
lhe
combustion to be enatic or wen impos-

A,/F atio used for idling aud low locls should be ........,.,.. for smmth operation.
11
6. Thericheniugmirture............. the probabty ofcontact between fuel md airparticles ancl thus.-...........

?.
combustion.
The mixtue requirenent for maxinm power is a rich mixture of A/F of about B : 1.
Carburetion and C arburettors
8. Tlansient conditions are those mnditions at which speed, load, temperatues or pressures are abnormal carburetion 1l'4'.Mixture
or changing rapidly, 11.1. Introiluction. 12.2. Induction system' 11'3' Facton inlluencing
11.6. Distribution' i1'6. Transient mture requirements' 1l'7' A simple or
9. The trsient mixture requirements are different from steady ronning mixture requirements. "-Jfoir"-"nt..
elementary cubuettor. 11.8. U";;;; c;"""tt"t' f f'S' CsburettoeEssmtial features of
to. During starting, a leu mitw should be supplied. of
go;J-"-o*"..itf carburettor foi aiutomotive enginee-$pes of carburettors-Desciption
mburettor
tt. Reduction in inlet pressure due to throt0ing or operating at altitutles leads to eduction in flame speeds i^po*-t -akes of cabuettrs-Solexcarurettr-4arter carburettor-S.U.
3o,'"
Petrol injection-Drawbacks of
and inaease in F/A ratio for best eonomy. io*i*t u".oo* vdable ctroke)-Aircraft erbuettor' 1 1' 10' injection-Indirect injection-
12. Any deviation from the optinuur spark timing wi lead to the increase of best economy F/A ratio, since it *oaero to fuel injection-Direct
of petroltjectio and caburetted firel supply
systems-
will lcrease the ..,.......... lm. i"..1i- """or"ttors-Introduction
fuel i{ection. f f .f f. 'Iheory of imple Lr,bmttr. Worked,Examples-Highlights-
Electronic""*ia"rations-Comprison
-t""tft" fyp" etestions-Tteoreti Questions-Unsolved Examples

Steadyrunning
1. 2. Yee 8. idling 4, Yes 6. rich 11.1. INTRODUCTION
improves
6. increases, ?, No 8, Ye g. Yes 10. No air mi'rture
tl. yes ' . Carbueti ol.- The process of preparitq in the S'I' engine' a combustible. fuel
12. time.
outside th engine cylind.er , cill"
i.irburitioo". This complicated process is achieved in the
ind,uction sYstem.
IEEORETICAL QrrESTroNs d device whih atomises the fuel and. mises t with
air' It is the
Carburettor. A ca'burettor is
l. What do you mean by "Steady runuing" ?
nost important part of the induction system'
_ as tlie means of adding
2. Whatisa"lansientoperation'? For several decades, carburettors wee usd otl most S'I. ngines
works is extremely
8. Explain briefly fueair mi:ture eguiraments for steady milg. fuel to the intake air. The basic pnciple on rf,hich the carburettor
when rel ioiectors nally replaced i,t as tU maln fuel innut
4. Describe briefly optimurn fuel/air ratios. simple, but by the'19g0s,
expensive systern'.Caiburettors
6. State the fueVair mixtue requirements for the following systlem, it had evolved into a comilicated, sophisticated'
engiries use simpler, better
are still founcl on a few automobiles, but vast najfity ofcar
:
(i) Idng and low load.
more flexible fuel injector systems' Many small engine like those on lawn
(ii) Normal power range or cruise ra.nge. controlled,
much simpler ones than
(iii) Msrcimtt tt*er movers and moclJ airplanes sll use carburettors, although
raDge, 1970s. This is to keep the cost of
6. What are'Ilauient contions" ? those found on the automobile engineE ofthe 1960s and
while fuel injectors
7, Explain briefly the following : these engnes d.own, mple carbirettor being cheap to nanufocture
Euen on some ofthese small engnes, carburettors are
(j) Starting and warming up niltue requirements. require;ore costly aontiol rystems.
polltirln become mnre stringent
(ii) Mture requirements for accelenon. being replaced *in fuA iaors as laws '

8. What are the effects offollowi4g operating variablee on mixture requirements ?


(i) Inlet md exhaust prBure r1.2. INDUCTION SYSTEM
(ii) Spark timpg The schematic arrangement of induction sJrstem is shown in
Fig' 11'1'
(iii) Friction. to the engine cylindes is called theintake
o The pipe that carries the prepared mixture
9. Write s short note on fiixure requirements for diesel engines". manifold..
' . The carburettor is the focal point ofthe induction system'
and
o The fuel system, comprising the fuel supply tank and necessary fuel pumps' lines
filters supply liquid ful to he carburettor'
'oDuringthemotionstrokevacuumiscreatedinthecylinderwhichcausestheairtoflow
throughthecarburettorandthefueltobesprayedfromthefueljets'Duetothevolatility
36'l
INTERNAL COMBUSTION
ENGINES
of the fuer' most of the
some of the rarger drops
fuer vaporises ad forms
a combustibre fuel-air
CARBURTION AND CARBT'RETTORS 369
mixture. However,
cioa"";;ilJ;;,"-
mav^reucn tr,"
r""m and must
wirh ai during the *.p""".ior, bevaporised For a S'I
engine, he design ofcarburetion system is very corqplicsted
il:rTJ:o "d-k;;;; rhe erecrric spark ignires
the that the air-fuer ratio by it varies iid"ly olr"" " tJg;;f
owing to the fact
particu-
automorive"equrred
engine. Fr idring as well s fo maxi-,ii "p"ration,
lr1r.l::'"
requred. f"n-"i J"n mixture is

1 MDrruRD REQUIREMENTS
1.4.
Fig' 11'2. shows the variation of nixtue requirements from no.road
to {l-roacr in a s.I.
engine.
9.1
s.1 (0.1oes)
\ Idle Transbr power
il system ldte and systsm and
12
iTfqrt main melering main
o f;iiil system L1l:'Ie
svslem l
1 12 (0.08333)

s15 tiili-- Z stoichiomerrtc A


63
.i
---F1-* 15 (0.06667)

<19 i:.t'rj


o
Fig. ll. l. Schematic anangcurent
The throttle located in rho o.l.,._s^_
There is a lirnltua l!
the carburettor, regulates
of the, Inductioa System,.

the quantity of the mixture.


ffii,f Main metering sysrem
i.i.,;
i:tj-i'
h1 ,
1 I (0.05263)

23 (0.(X348)
i" s. r. e;;;;;l: :T::t": lli,ililllf mixrure within which combu
nous.
s rion
Load
sustain flame propagation. th:.
e ."s qr useur A/!'ratio "ltio is too rich or too tean ro Fig. lf.Z. Mixture rcquiremets of automotive S.I. engine.
";;;f ;dlig";
Thi,
g:1 (rich). is from approx. 20 : 1 (lean)
to 1. Idling and Iow epeed (From no-load to about 204o of rated. piwl):
1 T .3. FACTORS INT:LUENCING Id'ling refets to * y?:, Dlring idling air suppry is throttled and residual gases
CARBURSIION **"
make up a large fraction ofthe charge
suction period. In addition, duringvalve
The uarous factors whih
influence tlw process of "ttft"br"t
overlap period some exhaust gases ar drawn "rioilie
loto tt e cinr. re r"sultlr irt"t chemically
1. The engine sp""o
carburetion are as correct (stoichiometric) mixture ofair and
, ,l_"_lirrl: ."rll.;;"l"lpa"auon follows : fuel (= 15 : 1) would be so ililuted by residual" gases that
combustion would be erratic or impossile.A rjj
2. The vaporisation characteristics
offuel.
of the mixrure.
(say Al F ratio 11 : 1 or
it tin, therefore, must be supplied,during id,tng
3. The temperatue of t2: I). The rich*r, ,h;:;i;;;;uaily change to slghtty lean for the second.
the incoming air. range as shown in Fig. 11.2.
4. The design ofthe carburettor.
2. Cruising or normal powet (from about 25Vo
o In case of modern
to about 75Vo of rated. power) ;
lifn .*:d.*qnes, the time-duration ava'able In the normal Dower rdnge the main consid,eration is
fuel econotny. Because mixture of fuel


ij;T:'fi Jilii#jf$H
Atomisation, mixing and
[.3 ff Jffi:tation ro -t.r*; i;
fo the fbrmation of
;;i;i and in du c.
and ai is tt"\'"" .o*pl"tuly horiogeneous trt"
completely and
correct (= 16.5 : "o-u
"iii"-*ric
fu"l *ill be wisted. For thi, t"u"oo
mixtue of fuel and air will not burn
ess ofair, say 707o aboue theoretically
vaporisation 1), is supplied. in ord,er to ensure coiptit""rr"r"
,* , Ourning th" iel
occur' The time available
""" which require a finite time 3. Maximum powet (From Z5% to 100Vo of
for mi*to"" roilr-"i'lo"rri:Tesses to rated power):
example, in an engine ru;;l;;;; in high speed engines (For
.q;;:':T"n '.t "."T
0.02;.;;;;.;.;'"""fi,Tffi'??ff":3t'"J""".k1'" "mall
induction p'*"-".lLiJr"r . Maximum power is obtained when all the air supplied iqfully utilized. As the mixture is not
rhan completely homogeneous aich mixture
is required in clesigning such a small period a great
ress
-.r.t ltroppii"d to assurl utilization of air (though this
would mean wasting.some fuel which touta
. the.*b;;; r;";;rn ingenuity
lor marimum power is about IJ : 7.
prr"lrr'"llaust in unburned state). The air-fuel ratio
*.::fi::".i."Tfi1n*f5: ru*:*;jt nsured
in such a shorr rime requires
good va_ Running on the weakest ,nirture, This results in high
efficiency and, there is fuel econorrty.
drocarbons in the iuel. by presence ofhigh volatile On normal loads engines work on weah
hy- msture,
Running on richest
" ;l""Jj.ffillT;; l3::"-" *lt'h effectivetv contrors vaporisarion
process orthe ruer.
mirture, Engines run on rich m-xture during idling and during the
ouerload, The effect s towerins of efficr";";;; p;i;";n
rr p"oot"^".
rh._i*il;;1;;;*li:1'+::ii:"":i,:lf o Automobiles ensines generally operate well below full power and
result in reducing power i;il,llilIi,il,,,,,,}"";"Tf *ij,",,,.mti for making the lixtu-re a complicated system
due to reduction in mass i." ;;i;;;; r- nor economically advisable, although
rrow ate_ some means are employed "i.hto enrich the""Ii"
mixture, A more representative curwe for an
automobile engine is shown in Fig. 11.8._
)tl
CARBURE-f ION AND CARBURETTORS
370 INTERNAL COMBUSTION ENGINES
since in a carburettor complete atomisation and vaporisation of theirel
is not achieved,
o a certain
theefqre, the mixture passing through the intake manifold generally contains
0.1 1

ll
Wide open lhrotlls k
amouitofpptrolintheilropletform'Thesedropletspossess.gteaterinertiathanthe
mixto"e. Thus when there is an abrupt change in the tlirection offlow the drop-
a ;;;;;;r vari-
0.10 lets continue to move in their original direction (Refer Fig. 11.4) and consequently
ation ofA / F ratio between cyliners takes place, the outer cylind,ers getting richer mir'
\ e
ture than the inner cYlinders-
t 0,09
I o The uneuen d.istributon is also caused owing to the ex,istence of thin fiIm of liquid, film
I 0.08 ad,herng to the inner ualls of the inrcr manfold"
,rh
.9
6 \ The imbalance in air-fuel ratio in different cylinders canbe partially corected as follows :
to uaporbe the d'roplets of^Iiqz. By doing
.!6 0.07 ' - By heatine the misture n the intafu manifotd
()
'T-
j
o
a
L 0.06
-l
LP^, t throtlle
c
., ho*"uu", the mass ofthe charge is reduced, resulting in reduction ofpower output.
(ii) By supplying o rich overoll air-finl mkture so thot the leanest cylinder receues the re'
qiiua'"i-i"l rio. This would, however, result in other cylinders getting a richer mix-
i ture than requireo.
0.05
-sit pl carburetlor 11.6. TRANSIENT MD(TTJRE REQIJIREMENTS
0.04 I it I
0123456788.5 The function ofa carburettor is not only to provide a suitable mixture for sdy-running but
also to supply mixture for transient conditions.under which load, speed, temperature or
pressure
Airllo, m3/mln
alter rapidly. The main transient conditions ofoperation ae :
FiB. 11.3
-) Starting and warming uP ;
---- Acceleration, and deceleration.
The portion ofthe curve from d to e shows the fuel-air requirements for wide open The mixture requirernents under transient conditions are different from those ofsteady run-
throttle (W.O.T.) and the load is furthe increased. ning due to the following redsons i
The Fig. 11.3 exhibits that a simple elementary carburettor is incapable to provide (i) The evaporation may be irtcomplete ;
the F/A ratio as desired at part tbmttle as shovllbya'b-c'd orW.O.T. given byd.e./' (li) The quantity of liquid fuel in the ilet manifold may be increasing or decreasing ; lli
' simple carburettor gives the cune as shown by s-h-h.
.The (ii) The distribution offuel to various cynders may be different'
11.6. DISTRIBUTION 1. Starting and warming up requienents:
When the engine is started from cold its speeil and temperature ae low and as such much of
Icleally, a carburettor should supply mixture of the same fuel-air ratio to each cylinaler of a .heavy ends' (The hydrocarbons with high vapour pressures and low boiling points are called 'lig
muiti-cylinder engine ; this condition is very difhcult to achieve practically.
end.s'ad. those which are less volatile are called'heavy ends') supplied by the carburettor do not
lnlt
vaporise and remain in liquid form. Further vaporised fuel may recondense on coming in contact
wiih colil cylinder walls and piston head. lbus, even when the F/A ratio at the carburettor is well tf,

within the nomal combustion limits or petrol-air mixtures, the ratio of the'euoporated'fuel'to air in
the cylind,enay b, too lean to ignite. Cnseguently it is zecessory to supply a rich mi.zture d'uring
t- Fuel-air stariing, as much as 5 to 10 times the normal amount of petrol (A/F ratio 3 : 1to 15 : I or F/A ratio
mixlute 0.3 to .?), in orde that 'light ends' are available for proper ignition. With the warming up of the
engine there is an increase in the amount ofevaporated fuel and hence the mixture ratio should be
Cylinder progressively made leaner, loo tich evaporatd F/A ratio is avoided,
block
2. Acceleration requirements :
a With regaid to engines, the termsacceleration generallyrefers to anincrease in engine
speed resulting froin opening the rottle. The main object ofopening the throttle, how-
ever, is to prouide an increase in brque and wtrether or not a increase in speed follows
depends on the nature. ofload.
o Under steaily runing conditions, t}re fuel evapoated in the intake manifold moving
much faster than the liquid film formed on the induction system walls, does not cause
any problem. But when the throttleissud.denly opened e.g. during acceleration, the liquid
Fig. 11.4

l-
TNTERNAL COMBUSTTON ENCINES
CARBURETION AND CARBURETTORS JIJ
fuel lags behind and, the cylinder receives temporarily a lean
rnixture whilst actually, to
produce more instantaneous pol{er for accereration,
a rich mixture is needed. Irence, a o It co4sists of float channber, nozzle witb metering orifice, uenturi and throttle volue.
a
suitable mechanism (acceleration pump) is required to provide the float chanber is meant for storage of fuel. The fuel supplied under gravity
rich mixture dring the
acceleration period. action or by fuel pump enters the float chamber through a filter. The arrangement
Note' Ttre petrol to be ued should be mrefullymade to suit the engine is such that whe oil reaches a particular level the needle lfloat ulu blocks the
md the climate of the place inlet passage and thus cuts offthe fuel oil supply. On the fall ofoil level, the float
since too high volatility or tm low volatility, both create difliculties
in operation.
Too high uolatility may form bubbles in the carburettor descends down, conse{uenily intake passage open and again the chamber is frlled
- the engine temperatures are high, which interfee with the r*pr"
and fuel lines particularly when with oil. Then the float and the needle/float valve maintaius a cogtant fuel oil level
of the in the float chsmber. There is amzzle (d.ischargeje) from which Fre fuel is sprayed
fuel and may disturb tbe F/A ratio so seriously that engine *"iii"p.""ti"g. ".iJ-*ering into the air stream as it enters the inlet and passes through the venturi or throat.
Too low uolatilifi may cause petrol to condense on the cylinder
- moving the lubricating oil film ; ultimately the petrol may reach the
walls, diluting and re- Thz fuel level is slightly below the outlet ofthejet when the carburettor is inoperative.
crnckcase past the o As the piston moves down in the engine cylinder, suction is produced in the cylinder as
piston rings and dilute the engine oil. Condensation ofpetrol '
on cylinder walls also causes well as in the induction manifold as a result ofwhich air flows through the carburettor'.
carbon deposits. The velocity of air rcreases cs i pcsses through the constricton at the uenturi and. the
pressure decreases d'ue to conversion ofa portion of pressure head. into kinetic energy. Due
r1.7. A SIMPLE OR NLEMDNTARY CARBURETTOR, to decreased pressure at the venturi and hence by virtue of difference in pressure (be-
rn order to undemtand a modern carburettor (a very complex device) tween the float chamber and the venturi) the jet issues fuel oil into air stream. Since the
it herps first to study a jet has a fine bore, the oil issuing from the jet is in the form of finc spray; it vaporises
simple carburetfor which supplies fuel-air mixture for cruising
or normal range oiJfeua a.ra trru., to quickly and mixes with air. The air-fuel mixture enters the engine cylin d.er ; it quantity
add other devices or attachments to take care of other function
like startit$, idling, accelerating, being controlled by the positon of the,,throttle ualue,'.
decelerating and other variable load and speed operations.
Limitations:
Fig l 1.5, shows a schematic diagram of a sirnple or erementary carburettor. (i) Although theoretically the air-fuel ratio supplied by a simple (singlejet) carburetor should
remain constant as the throttle goes on opening, actually it proaidzs irrcreasingly richer
Fuel-air mixture
to engine cylinder
mixture as the throttle is opened. This is because ofthe reason that the density ofair tends
to decrease as the rate offlow increases,
++,1
Induction
manifold
Il Throttle
valve
<-- Fuel from pump
This fault is corrected by usi ng d number of compensating deuces.
(ii) During idling, however, the nearly closed throttle causes a reduction in the mass of air
flowing through the venturi. At such low rates ofair flow, the pressure difference between
the float chamber ad the fuel discharge nozzle becomes very small. It is not adequate
Nozzle enough to cause fuel to flow through the jet.
(dscharge iet) This fault may be corrected by z sing an idling jet which hetps, in running the engne
during dling.
(iii) Carburettor does not have arrangement for providing rich mixture during starting and
warm up.
Venturi
or throat
This limitation is taken of by using a clr ohe anangement.

r1.8. COMPLETE CARBURETTOR


Float
chamber . lo" meeting the demand ofthe engine under all conditions ofoperation, the following addi-
tional- devices/systems are added to the simple carburettor:
1, Main rnetering system
2. Idling system
3. Power enrichment or economiser system

tt
rtl l
1.
4. Acceleration pump system
5. Choke.
Main ruetering system :
Air inlet
The main metering system of a carburettor should be so desigrred as to supply a nearly con-
il, Fig. 11.5. A simple or elmentary carburetto stant fuel'air ratio ouer a wd.e range ofoperation. This F/A ratio is approximately equal to 0.064 (A/
F ratio = 15.6) for best economy at full throttle. In order to correct the tendency ofte simple carbu-
lr
ii

r
CARBURETION AND CARBT'RETTORS
374 INiERNAL coMBUsrIoN ENcINES

rettor to give progressively richer mixtues with load speed, the following cutomatb compenutitg
d,euices are incorporated in the main metering system : 0.07s
1
() Compensating jet device. I 49
(ii) Emulsion tube or air bleeding device. | 0.07
.9
(ii) Back suction control or pressure reduction method. E 0.065 -
Combined st
(iu) Auxiliary valve carburettor.
0.06
.z- _L_t- --
(') Auxiliary port carburettor. :
Theie devices are explained below:
(i) Compensoting jet device:
Io o.oss
Lf
\s*k el
o.o5
A schematic diagram ofa compensatingjet device is shown in Fig' 11'6'
0.045
I 8.5
Air-fuel mixture Air fow, m3/min
to cylinders Aif spsed pal et incrgasng -------)
jets'
Fig. 11.?. Variation of F/A ratio vs' aA flow with main and ompemating
111 res of air flow, when the compensating jet has been emptied, oir is
At even higher
- bled throulh tc compensating Jet to continue the leaness effect, and incidently to
assist in ooisation of fuzl.
(ii) Emulsion tube or oir blced.ing daice t
Compensating The mixture correction in modern carburettor is done by air bleeding alone. Such an arrange-
iet ment is shown in Fig. 11.8.

In this device, in addition to the main jet, a compensating jet is prwided which is in Atmosph6ric
commqhication with a compensating well. The compensating well is also vented to at-
pressufe
mosphere (like the main float chamber) ; it is supplied with fuel from the main float
chamber through a restricting orifice' Wdl
jet
As the air flow increases, the level offuel in the compensating well ilecreases, thus re- Main
submerged
ducing the fuel supply through the compensatingje. The compensatingjet thus progres-
sively makes the mixture leaner as the main jet progressively makes the mirture richer, Fig. U.8. Corection in nodem carbwettors by air bleeding'
the sum of the two remaining constant as shown in Fig 11.?. The main jet and compen-
sating curves aro more less reciprocals ofeach other, o A main metering jet is fitted 25 mm below the petrol level in the float chamber and
therelbre it is ca1ld subm"rged.ef . The jet is situated at the bottom of a well, the sides of
INTERNAL COMBUSTION ENCINES CARBURE'IION AND CARBURETTORS
which have holes which are in comnunication
the hores and the petror is emulrjfi"dl with the atmosphere. Air is drawn orifice is (pr-p2) whenp, is the pressure at the throat. Ifthe valve is closed, the float
through
;;;Jo"u -
not as great as that in the simple caibuJbn:* - i,*"""r,"" ucross the petrol corumn is chamber communicates only with venturi throat and pressure on the fuel surface
r rnitialry the level of petrol in the float
chamber and the well is same.
will bepr. Then the caburettor depression ap will be zero auil no fuel can flow. By
opened the pressure at the-vento when throtre is proper adjustment of control valve any pressure betweenp, andp, can be obtained
d"a."ra"a the pet"ol is drawn into the
This results in progressively u"""""i"s "rrd ai stream. in the float chamber, thus altering the quantity of fuel disciargeiby th e nozzre.
ti"-'r.iJ.; the central tube Ieading to decreas_ (iu) Auriliary
o. decreasing richn""" aal,ve ear:burcttor:
;ffiilff:l "r-th;;;l;;;. No.."l now then takel piace rrom Fig' u-10 shows an auxiliary valve carburettor. when load on the qngine increases, the
(iii) Baek suction control vacuum at the venturi thmat also increases. This lifts the valve agalnst the spring force
or presrare reduction method.:
This method is commonly used to change the and consequently more air s odmitted and the miure is preuented.
ftom becoming ouer-
air-fuel ratio in large carburettors. ncn-
o In this device./arrangement (Refer Fig.
11.g) a relatively.large vent line (u) Auriliary port carburettor.
carburettor entrance with top of tfr" connects the
nt i,
orince rine, tiu top ortn" n*, "i"i". o This method is used in orrcral carburettors for altitude cornpensatinn.
""";;;
valve is placed in the large vent
line. "r,,..",il;11?Jrff:ffti;:i,1.:r."'#"1i o Fig. 1 1.1 1 shows an auiliary port carburettor. when the butterfly valve is opened, addi-
tional air is admitted and at the same time the depression at the venturi throat is re-
duced ; this results in decreasing the quantity offuel drawn in.
Fuel-air
mxture lo en(ine
Fuel-air mixture
Ful-air mixture lo engne
to engine

111 ttt
AI\ Throttle
valve

*fth
wH-
l-
Inr tt
Air Air

Fig. 11.10. Aa auxiliaryvalve carburettor. Fig. 11.11. Aa auiliary port carbuettor.


Fig. 11.9. Bac&-suction control or pressure 2. Idling system :
reduction methocl.
When the valve is wide open, the vent r As earlier discussed that at idling and low load an engine requires a rich mixture having
line is unresticted and the pressure about air-fuel ratio 12 : 1. The main metering system not onry fails to supply enrich the
float chamber is atmospheric say p,, in the
and the pressure differentjal acting mixture at low air flows bualso cannot suppfu any fuel during idring ope"tioo. It is due
on the
to this reason that a separate idling jet must be incorporated in the bsic carbure*or.

r-
I
378 INTERNAL COMBUSTTON ENGINES CARBURE'TION AND CARBURETTORS

. F'ig. 11.12 shows an iilling jet. It consists of a small fuel line from the float chamber to a Fuel-air mixture

tt
point on the engine side of the throttle ; this line contains a xed fuel orifice. to erEne
When throttle is pratcally closed, he full manifold suction operates on the outlet
to thisjet. Besides local suction is increased due to very high velocity past the tbrot-
t
tle valve. Fuel therefore can be lifted by the additional height upto the discharge
point, but this occurs only at very low rotes of or flow.

tt
To engine

ldle adusl

Float
chamber
ttt Air

Fig. 11.13. Skelton outline of a meter rod economiser'

4. Acceleration PumP cYstem:


the engine rapidly' a very
1t
Air
o- Acceleration is a transieut phenomenon' In order to accelerate

"i.h -i*to""
is required *litt'
"
simple carburettor may not be able to supply'-Rapid
be immediately followed by an increased air flow, but
the inertia
"p""i"s "tn*ttre-wiu lean mixture. Thus acceleration mixture
. Fig. 11'12. I.rligiet. i riqrr fuel (gasoline) will give rnomentarily
thie difficult situation an accel'
required may not be met wiih in practice' To overcome
tually becomes ineffective. eraton PutnP is incorPorated.
(a needle valve control'i.g the air bleed, which is manually oper-
Tlrte idte ad.just o- Fig. ll.l4showsancccelerationpump'Itconsistsofaspring-loadplungel'Alsoispro-
vied a linkage mechaism so that when throttle is
rapidly opened the plunger moves
ated) regulates the dsired. Al F rao for the d.Ing jet. jet into the venturi. An arrangement is
into the cylinder and forces an atlditional of fuel
3. Pou/er enrlchment or economisor eystom :
alsoprwidedwhichensuresthatwhenthrottleisopenedslowly'thefuelinthepump the float
At the rnaximum power range of operation from 757o to L007o load, a device should be avail- i. into the venturi but leaks past plunger or some holes into
no forced
able to allbw richer mixture (F/A about 0.08) to be supplied despite the compensating leannegs. "irra",
chamber.
Meter rod. econo'riser shown in Fig. 11.13 is such a dwic. It simply provides a large orifice opning
In some carburettors, instead ofproviding mechanical linkages,
an arrangement is
to the mdinjetwhen ttre throttle is openedbeyond specied limit. The rod maybe tapered orstepped. this vacuum
made sothot thpump plunger is helil up by manfoldvouum'Whenever
A economiser is a valve which remains closed at normal cruise operation and gets opened by rapid of thottle a spring forces the plunger dbwn pumping
- to supply enriched mixtue at full thmttle operation. It is reduced
regulates the additional fuel ""i"g
the fuel through the jet'
supply for the above operation. (The term economiser is ratherrisleading.ftslems from
the fact that such a d,evice prouides a rich uneconomical ni*ture at high load. d.emand.
witlrcut interferng with economical operotion n the normol power range).
L-_-.-

380
CARBURETION AND CARBUREfTORS
381
't

r1
Air
Fig. ff.f4. Acceleration pump.
5. Choke I
Starting ofa vehicle which is kept Fig. 1 1. lS. Choke valve with spring-Ioaded by-pass.
s,tationary for a long period (may
cool winter seasons, is oft'en noie be overnight.) during
difficult. iir;;';;'..rLrg.peeds and o The provision of ouxliary fuel jets that are opened manually o automatically only as
inteke tempera_ required, is an alternotiue to the choke,
,ffi;":"#J:"1":ii,Hl',I1"t:"9 * "ut"h;;;;;ilon. sometimes as high asfue ro
r"""tion-oin';;;:ffi
vapour f,raction can provide a.fii[:.,:*i:'::1ft :T"",,i*rrjl:* j""xl,r:
c-uurti -]rt""" uiil*. The
11.9. CARBURETTORS

providing such mixture is by the u." """'


of"i"rl"-^"
most popur*
-uthod of 11'9.1' Essential Featuresof GoodConmercial CarburetorforAuomotive Engines

fJ** ii::T,t* H:T tt",F,t;:ii:i;ffi*" the enrrance to the carburenor Carburettor is a mixing dcuice to supply the engne uith air-fuel mrture. It atomizes the
fuel
and rhe and mixes it uith air in uarying proportions to meet thi changing operatng cond,itions of
When the choke is partly closed, autotnotiue
large pressure drcrp occus at engines. It is required to provide the following es sential
the veDturi throat, features :
would normally
y#1?.1y.*:l:"*: fi.- rha .*^,.- v ^: P4ssrng Enrough the
ff : very large carburetto,
The """r1 :*_=iryffi ;.ffit"Tilffil lii Jlilil ll;:il:
i-- -'*-"" ^ 4r
!r throar
venturi throat. 1' To meter and supply the proper quantity and proportion ofair and fuel at correct
strength
pr, a
fom the !roaf, rnoucts
inducrs large amount
il;;;;;ii;"jof fuel under all conditions of load and speed of the engine of the car for
rhe main nno"zle ar^,::T:.:i:l_1t,the
ozz,te and
ated
rated fuel ro air in rhe l*^1L*re.so th.at the iati" "r;;;
wiffiii;';#ffiil""i11,T:
rh" cylinder is ;;;;;- (i) starting it easily from cold.
""",i:11""tj,:f3
'H:l'"mff
carburettordepressionande jnrr;';;;';;'-;'::'L",,.u."tha'arge
lPrrrts-uaqeqDy-passtoensurethatlarge
(ii) providing a rich mixture for slow idling.
(iil) providing a rich mixture for acceleration,
":*i:"""#""Xilx,::*#ll
and reached a desired speed ;---': :.v.uq6 uws ru perslst
,"""0:ff:,H:l:T-1rj^T^y l""sist atter e,,-gi;; i".
ri"" the engine ,Li"o,
has started,
t9 onerare (iu) providing a rich mixture for high speed, and
y-il"u.,.
"h;; ;; ;ur";;T".:"rf,1"r:
of a thermosiai *-ri.r""n?"11"":,T::::ll_.
thermosrat .o th.i i. Tlde by means
when the engine warms colcl ",rto_utjly
l"romaricary
and goes out of operation (u) providing a ich mixture for low speed when moving up_gradient.
op flpr crq+--
wams un
"R". "t".Uig. 2. To operate satisfactorily when cold, or when hot
3. To operate satisfactorily both on level and hills
-.___-T-
INTERNAL COT{BUSTION ENGINES
CARBURETION AND CARBURE'TTORS 383
4, To overcome air-cleaner reskictios.
5. To withstand vibrations aud roadjerks.
o The updrought varety is now osolele and is only used where neither of the other types
can be accommodated..
11.9.2. Tlpes of Carburetore t The downdraught and horizontal ypes ofcarburettors are most widely used.
Carburettors, basically, ae ofthe following two types : The aduantage of downd,raught uariety is that the mixture is assisfed by grauity in
Open choke t5pe
1. its passage into the engine induction troct, and d,t the same time the carburettor s
u s ually reasonably acces sib Ie.
o, Here, the main orifice known as the choke tube or uenturi is of frrcd. d.imnsians, a.d
metering s affected by uarying tle pressure d.rop across i. Almost all carburettors, except The horizontal type of corburettor has some ad.uantage when under-bonnet space is
S,U. carburettor, belong to this Iimited..
-category of carburettors.
o The importdnt ernmples of thie type of carburettor are : 11.9.3. Description of Some Important Makes of Carburettors
(j) Zenith carburettor Following carburettors will be described here :
(ii) Solex caburettor 1. Solex carbuettors
(jii) Carte carburettor 2, Carter carburettos
(iu) Stromberg carbuettor.
3. S.U. Caburetto.
2. Constant vacuum t4re
I 1.9.3.1. Solex Carburettor
o In this type ofcaburettor the area of the air passoge is uaried. autorrwtically whit the
pressure d.rop is kept approtimately constant, This carbuettor is made in vaious models"and is used in Fiat, Standard and Willy's Jeep. It
o Example: S.U. carburettor. is famous for the following charaeristics:
(i) Easy starting ;
Basic forrs of crburetors: (ii) Good performance ;
Refer Fig. 11.16. Carburetor may be of the following ree os r
forms. (iii) Reliability.
(i) Updraught (ii) Downdraught Fig, 11.17. shows the schematic arrangement ofa solex carburettor. The unique featuo ol'
(iii) Horizontal. this carburettor is Bi-sartcr for cold starting.
The various components and the cicuits for air and fuel for various ranges of operation are
explained below :

1. Normal running:
a Solex carburettor comprises a convLntional float ( 1) in a float chamber.
o The fuel is provided through the main metering jet (2) and the air by the choke tube or

re o
venturi (3).
The fuel from the main jet passes into the well of ai-bleed emulsion system ; (4) is the
emulsion tube which has lateid holes.
o Air correction jet (5), calibrates the air entering through it and ensures automatically
the correct balance of air and fuel.
The metered emulsion offuel and air is discharged through the spraying orifice or nozzle
(6) drilled horizontally in the vertical stand-pipe in the middle of choke tube or venturi.
111 (7) is the conventional butterflv valve.
Air\
(f Updraught (df Downdraught (jdd) Ilorizontal

Fig. 11.f6. Basic foms ofcarburettos.


t

184
INTERNAL COMBUSTION ENGINES CARBUREON AND CARBURETTORS 385

, appliedtothestartingpassage(11),suckingpetrolfromjet(9)anilairfromjet(10)'Thl
jets and passages are so shaped that the mixture Povided to the carburettor is rich
enough for starting.
Acceleration pump After starting the engine, the starter lever is brought to the intermediate position, bringing
the snaller holes in the starter valve (8) ito the circuit, thus reducing the amount of
petrol. Also in this position, the throttle valve is partly open, so that the petrol is also
coming from the mainjet. In this situation, the reduced mixture supply from the starter
system, ho\ryever, is sufficient to keep the engine running till it reaEhes the normal run-
ning temperature, when he starter is brought to 'bff-position".
3. Idling and slow running:
. From the lower part of the well of the emulsion system a hole leads off to the pilot jet
(13).
At idling the throttle is practically closed and therefore the suction created by the engine
on suction stroke gets communicatetl to the pilot jei. Fuel is inducted therefrom, and
mixed with a small amount of air admitted through the small pilot air bleed orifice (14)
and forms an emulsion which is conveyed down the vertical channel and discharged into
he throttle body past the idlingvolume control screw (15). Theslow running adjustment
screws allows the engine speed, to be varied.
By-pass orifi.ce (17) provided on the venturi side ofthe throl.tlevalve ensures the smooth
transfer from idle and, low speed, circuit to the moin iet circuit without occurrence of flat
Accleation
spot.
pegal
4. Acceleration:
In order to avoid flat spot during acceleration, a diaphragm typeacceleration is incorpo-
Fuel-ar mixture rated (also known as economy system). This pump supplies sprrrts ofextra fuel needeC
lo engine fo acceleration through pump injector (18).
Pump lever (19) is connected to the accelerator so that on pressing the pedal, th-e lever
1. Conventional float
2. Mainjet movs towads left, pressing the membrane towards left, thus forcing the petrol through
3. Choke tube r ventui
4. Emulsion tube pumpjet (20) and injector (18). On making the pedal free, the leve moves the diaphragm
5. Air conection jet
6. Spraying orifice o nozzles Lack towards right creating vacuum towards lef! which opens the pump inlet valve (21)
z. corr".uo. J.ibrfly valve and thus admits the petrol from the chamber into pump.
8. 'lat disc wi0l holes ofdifferent
9. Stara petrol jet sizes
10. Jet
11. Staring passage 1 1.9.3.2, Carter Carburettor
12. Starter leve
13. Pilotjet
14. Srnall pilot air bleed orifice Acarter carbureor is an American make carburettor and is used in jeeps. ltis a standard
15. Idling volume control scew equptnent on cheuorolet and Pontiac series of cars.
16. Idle port
17. By.pass orifice
18. Pump injector Fig. 11.18 shows the schematic arrrngement of a downdraft type Carter carburettor. The
19. Pump lever
20. Pumpjet brief description ofthe components and circuits is given below :
21. Pump iniet valve.
Fig. 11. 1?. Schernatic arrangement of
o The petrol (fuel) enters the conventional type float chamber (1).
2. Cold starting and warming:
a solex carburettor. o The air enters the carburettor fom the top, a choke valve (2) in the passage remains
open during normal working.
The prousion ofa bi'starter or a progressue
starter is the unique This carburettor has a triple venturi diffusing type ofchoke, i.e. it has three uenturies,
. The starter valve is in the.form feature ofsolex cd.rburettors.
ofa-flat aisc t8l wit;t oles ofdiffeent the smallest (3) lies above the level in the float chamber, and the emaining two venturies
connect the starte petrolj.et (9)n sizes. These holes (6) and (5) are below the fuel level (in the float chamber), one below the other'
rtr"te" ai"j;;;;",
a hole just betow th" th.ottle v"l.,re;;i;F'.ilb-i'1g"" to the passage which opens into
I
o. r,nult"r holes come opDosrre At very low speeds, suction in prirnary venturi (3) is sufficient to draw the fuel' The
the passage, depending upon the position
I ofthe sta?ter lever (12). The starter lever nozzle (4) entes the primary venturi at an angle, and throws the fuel up against the air
operared bv flexible cabre.from the is
dash b";;i-;;;;;. Initia'y, for starring richer mix_ stream evenly, thereby providing frnely divided atomised fuel. The mixture from'enturi
li ture is required and afrerthe engr"" (3) passes centrally through the secondary venturi (5) where it is surrounded by a blan-
Jil;l;;;;;ss
postion when .the starter.control i, p"ll";;;; required decreases. In the sat
ir. isg* lr"l", ;." tl" ."""""ti"g ket ofair seam and frnally this leads to the thid main venturi (6). where again the
holes' The throttle valve being l"
.l;*Jp*iiioi'?t u *nol. of the engine suction fresh air supply insulates the stream from the secondaryventuri. The fuel-air mixtur(l
rs
I

i
386 INTERNAL COMBUSTION ENGINES
CARBURETION AND CARBURETTORS f87
enters the engine in well mixed atomised state. The multiple uentur gues more homoge-
neous and better mi,xture at very low speeds resulting in steady ond. smooth operation at When the engine starts/fires, the spring controlled halfthe choke valve is sucked
- open to provide correct quantity ofair during the period ofwarm up,
low speeds, This arrangemcnt also ensures od.equately forrned. miuture at high speed.s.
2. Idle and low speed circuit:
o For idling rich m&ture in small quantity is required.
o The throttle valve (8) is almost closed in idling condition.
Fuel level in float chamber o The entie suction pressure created by the piston in the engine, cluring suction stroke, is
,..6 To acceleration
Filter
exerted at the idle port (9). Consequently, the petrol is drawn through the idle feedjet
pdal (10) through first by-pass (11) and a rich idle mixtue is supplied. The throttle valve is
To acceleration
pedal opened further in low speed operation.
Fuel from
pump At this stage the fuel is delivered both by ihe main venturi and low speed pot (12) through
the idle passage.
3. Acceleration pump circuit:
o The acceleration pump is employed, to ouercon'Le flat spot in acceleration.
r The pump consists of a plunger (13) working inside a cylinder consisting of inlet check
valve (14) ancl outlet check valve (15). The pump plunger is connected to accelerator
pedal by throttle control rod (16).
o On rapid opening the throtle by pressing the accelerato pedal, the pump is actuated
and a small quantity ofpetrol is spurted ito the choke tube by a jet (17). Releasing the
accelerator pedal takes the plunger back by spring for0e and in the process sucks petrol
from the float chamber for next operation.
Metering The acceleration pumpdoes not supply fuel continuously for heauy load. but only prouides
ldle screw rod jet an extra spurt of fuel d.uring rcceleraton to avoid, flat spot.
adiustment
11.9.3.3. S.U. Carburettor (constant vacuum variable choke)
Fuel-air mixture In general carburettors are of'constant choke' type ; examples being Zenith, Solex
to engin and Carter. S.U. carburettor differs from them being'constant vacuum or depres-
sion'type with automatically variable choke.
l. flsf, clbe 2. Choke valve It is used in many Briish cars and was used in Hindustan Ambassador car.
3. hinaryvenri 4. Nozzle
Fig. 11.19 shows the S.U. carburettor schematically. The various components and cir-
5. Secondary venturi 6. Ihid (main) venturi
7, Metering rod
cuits are descibed below :
8. Throttle valve
9. Idle port 10. Idle feedjet 1. Normal operating condltion :
11. By paes Iaw speed port
12. o The full metering is accomplished by a tapered needle which is raised or lowered in a
13. Plunger 14. Inlet check valve jet to alter the effective arinularjet orifrce, and hence fuel flow. The needle projects from
15. OutletchecLvalve 16. Thottle ontrol rod
undeneath the flat face ofthe cylindrical air-valve, which alters the choke area as it is
17. Jet.
Fig. 11.18. Schenatic arimgernent of a downdraft Cart aised or lowered. The upper part ofthe air-valve is enlarged to forrn a piston which fits
srbuettor. into the Iower open end ofthe vaorum chamber. A spindle situated in the centre of the
t
a In Carter carburettor mechanical metering method is used. In the fuel circuit there ig a air-valve guides the assembly into the rylindcal lr""urra chamber. To improve the ac-
metering rod (7) (having two or more steps of diameter) which is ctuated by a mecha- l curacy and tirne response ofthe air-valve vertical movement with very small changes in
nism connected with the rain throtle. The amount of petrol d,rawn into the eginz is i engine demands, the friction between the air-valve spindle and guide is sometimes re-
governed. \ithe area ofopening between the meterng rod.jet and metering rod.. I
duced by installing a ball-race between the two sliding surfaces.
I
Starting circui: o While the engine is running, the effect ofthg depression above the piston in the upper
a In order to start the engine, achoke ualue l? is incorpoated in the air cicuit. The choke I

chamber, and the atmospheric pressure-underneath, is to raise the air-valve and piston
valve is ofbutterfly type, one halfofwhich is spring controlled. The valve is hinged at the assembly against its own weight and the stiffness oftbe return spring. Since the doward
centre. load is almost constant, a consant depression is needed to keep the air-valve stationary
When the engine is firlly choked, the u'hole ofthe engine suction is applied at the in any raised position.The amount the air-ualue lifts d.epends on the flow rate of air wh.ch
mainnozzle, which then delivers fuel. Since the airflow is quite small, very rich passes through the rnising chamea this being controlled by the engine speed and throt-
fuel-air mixtue is supplied. t1e opening position.
CARBURETION AND CARBURETTORS 389
INTERNAL COMBUSTION ENGINES
i
2. Mixture adiustment and fuel temperature compensation 3

r Thejet height initial adjustment and hence mivf,q." strength can be made by altering
the tilt ofthe right a,ngled.leuer which is attached to aspring-loaded retoining screu and
Control spring
a bimetallic strip which extends to the petrol jet. To alter the jet height, the horizontal
jet adjustment screw is screwed inwards to lower thejet and enrich the mixture, and
Pslon damper outwards to raise the jet and weaken the mixture.
Vacuum or
sucton
a In order to compensate for the variation in fuel viscosity within changing ternperature
chamber Guide
. and the reluctance of the fuel to flow througb a srifice as its visposity rises, a bimetal-
spindle
lic strip subm.erged. in the fuel senses s temperoture change ad. alters thc effective jet
and ball size accordingly. When the fuel temperature rises the bimetallic strip curls upwards and
face pushes thejet futher into the tapered needle. Conversely, ifthe fuel becom'es coolcr, the
strip bends downwards and lowers the jet to increase the annular jet orifrce.
3. Part hrottle by pass emulsion system. Refers Fig. 11.19 (o).
o This system is a passagaway which bypasses the mixing chamber, it spans the distance
(a) Part throtlle by pass between the feed duct at the jet bridge and a discharge duct at the throttle butterfly
emulsion systm edge.
Over run Acceleration Transfr Throttle with o The bypass passage, with a small throttle opening, delivers a quantity ofmixture in a
Posilon posltion ovsnun vave
duct well emulsified condition frorn the jet to a high depression pint nea the edge of the
(c) Acceleration throttle. Since the bypss passagaway is much smaller than the mixing chamber bore,
piston damper dsvice Control
the mixture velocity through this pas$age will be much greater and therefore the air-fuel
laner arm
Ar duct mixing will be that much more thorough.
Air valve 4. Overrun valve. Refer Fig. 11.19 ().
I Underouerrunworkingconditons,theclosedthrottlewillcreateaveryhighdepression
Air intak6 on the engine side ofthe throttle and in the induction manifold. Consequently, the effec'
Spring loaded tiue compression ratio will be low, burning will be slow and, erati.c, and. the exhaust prod-
tapered needle ucts wIl contain high volues of hyd,rocarbon. To improue the burning process so that rnore
Petrol of the fuel is doing useful work and less is passed through to the exhaust as incomplete
inlet combustion products, a spring-loaded plate-valve is incorporated in the throttle but'
pipe terfly disc.
Floal needle o When the engine is operating at overrun conditions, the manifold depression at some
valve predetermined value will force open the spring loaded plate-valve to emit an additional
Petrol jet quantity of correct ai-fuel mixture. The increased. supply of ar-fuel mixture wiII reduce
Air Jel adiustment
assembly the manifold. depression with the result that the denser and better prepared, mixture charge
jet scfrv cold
(b) Over run valve in start wiII improue combustion, ond, hence less unburnt prod.ucts wiII be passed through to the
open position Valve
device exhs.ust.
5. Hydraulic damper (acceleration enrichment device). Refers Fig. 11.19 (c).
o This device is incorporated to enriclt. the ni,xture strength when the throttle is opened
rapidly but it d.oes not interfere with the normal oir-ualve lift or fall as the mixing cham-
ber depression changes with respect to stead.y throttle opening.
Birnetallic
assmbly
4,6\
t/.ml rrffiI
I hU lIt I I r;rl7l1! I
\ \\t3Pl,/ \ \Y tCK
{
The damper valve is mounted on the lower end of a long stem inside the hollow
guide spindle ofthe air-valve and is submerged in a light oil. The damper consists of
a vertically positioned loose frtting sleeve, its underside resting on a spring clip
attached to stem, while its upper end is chamfered so that it matches a conical seat
\v/ vf-_/-\.
J/v lnlel formed on the central support stem.
Valve body
o On rapid opening of the throttle, the sudden rise in depression in both mixing chamber'
and air valve upper chamber tends to jerk up the air valve assembly. Simultaner:usly,
(d) Enrichment culoff maximum enrichment
the viscous dragofoil in the hollow spindle will lift the sleeve and press it against its
cold slarl enrichment dor'ice
seat, and so the oil is thus temporarily trapped beneath the damper so that it prevcnts
19. S.U. carburettor (constmt vacum variable choke).
390 INTERNAL CISMBUSIION E.GNES CARBURETION AND CARBURETIORS 391

any further upward movement of the ai valve. For this brief period a temporary in- 1 1.10. PETROL INJECTION
crease in the depression over the jet orifice is achieved, and more fuel will therefore be
drawn to enrich the resultant mixture strength. 11.10.1. Drawbacks of Modern Carburettors
When the change in engine speed steadies, the depression iu the upper air-valve The modern carburettors have the following drawbacks :
vaccum chamber will also stabilize and there will be a slight leakage ofoil between 1.The mixture supplied to various cylinders ofa multi-cylinder engine varies in quality and
the sleeve and its spindle bore. Consequently any oil pressure created underneath quanttyt. Also, due to fuel condensation in induction manifold, the mi*ture proportion is affected.
the sleeve damperwill now be released enabling the sleeve to drop down onto spring 2. Due to presence ofseveral wearing parts, the carburettors operate at a lower effrciency.
clip-Oil will row move freely through the annular space made between the sleeve
3. Reduced volumetric efficiency due to non-availability of a ftee flow passage forihe mixture
and its seat so thatthe ar-ualue uertical mouement can again react to small changes
owing to the presence ofchoke tubes, throttle valves, jets, bends etc.
in d,emands of the engine.
4. At low temperatures, freezing can occur (ifspecial means to obviate this are not provided).
6. Cold start device. Refer Fig. 11.19 (d).
5. When the carburettor is tilted or during acrobatics in aircraft surging can occur (if means
o Acold start-deuice is in the form ofa rotary-ualue cnsisting ofa qlindricol valve bod'y, to avoid this are not provided).
which has an annular groove in the rniddle region with a sngle radial hole drilld in its 6. In the absence of flame traps, backtiring may occur which may lead to igrrition of fuel
side. Fitted inside the valve body is a spindle which has an axial hole bored half-rray
outside the carburettor.
aiong from one end, while at the other end a control level is bolted. A double taper otched
radial hole intersects the axial hole in the spindle. The whole assembly of the valve body 11.10.2. Introduction to Fuel Injection
and spindle is positioned in a larger hole made in the side of the floor chamber. o The functinn of a fuel injection system is :
o In oder to cold start the engine the chohe &ob situated on the instrurnent panel is (i) To monitor the engine's operating uarinbles,
pulled out, the interconnecting cable rotates the control lever and spindle to a position
where the radial hole for both spindle end valve body are aligrred. When the engine is
(ii) To transfer this information to a tnetering control, then
cranked a high depression is created in the mixing chamber formed between the jet (iii) To discharge and etomise the fuel into the rcoming air stream.
bridge and throttle valve, and this depression is conveyed to the axial hole in the control o The position whee the fuel is injected into the air charge coDsiderably influences the
spindle where it then passes to the annular groove on the outside ofthe valve body. Iere performance of the engine.
it divides and draws the fuel from the dip tube and atmospheric air from the float cham-
ber by way of the air jet. Ilre emulsifred mixture is then drawn into te hollow spindle 11.10.3. Direct Irfection. Refer Fig. 11.20.
along the discharge passage duct and out into the mixing chamber. o In this type of layout the fuel njectors ore positoned. n the cylind.er-head so that fuel is
o With waming up of the engine, the choke knob can be pushed back steadily, this rotates directly d.ischarged into each conbustion charnber (Tig.11.20).
the control lever and spindle so that the notched. hole possageway becomes progressively . With this arrangement it is essential that injection is timed to occur about 60' after
smaller and. thus restricts thz quontity of oir and, petrol emulsion trying to enter the ni*' T.D.C. on the induction.stroke.
ing chatnber. . Because of the sborter time period for fuel spray to mix with the incoming air charge,
1 1.9.3.4. Aircraft Carburettor ' increased air turbulence is necessary. To compensate for the shorter permitted time for
The ma,jor dfferece between an automobile carburettor and an aircraft caburettor is that injection, atomising and mixing, the injection pressure need,s to be higher than for indi-
whereas the former operates at ground level conditions, the latter operates at varying altitudes. rect injection.
With the increase in altitude the density of ai decreases and A/F ratio whidr is proportional o More overlap of exhaust ond, nlet values ca,n be utilized, compared vrith other carburetted
to air density/fuel density decreases, i.e. the fuel-air mixture goes on becoming richer with increase or injected systems, so that incoming fresh ai can assist in sweeping out any remaining
in altitude. The mixtue will be about 407o richer at an altitude ofabout 7000 metres (since at this exhaust gases from the combustion chambers.
altitude the air density is nearly one-halfthat at ground level and hence A / F ratio about 0.7 times o The injector nozzle and valve have tci be desigrred to withstand the high operating pres-
the valve at ground level). sures and temperatures ofthe combustion chamber, ths means that a more robust and.
In view ofthe above, it is imperative to provide in aircraft carburettors an alitude mixure costly injector unit is requred,.
correction device to reduce the quantity of fuel progressively with altitude. For this purpose, the o Gerlerally,d.irut-injectionairandfuelmi*ingsmorethorough.inlargecylindersthanin
following methods are gmployed: ' small ones beca,use fuel d,roplet sizes do not scale d,own as the mixing sporc becomes stnaller,
(i) Air bleeding\; o All conilensation and wetting of the inductioD manifold and portsiselminated but some
(li) Back suction control; spray may condense on the piston crown and cylinder walls.
(lii) Incorporating a metering pin.
11.10.4. Indirect \iection. Refer Fig. 11.20.
The aircraft caburettors entail the following other specal features :
r For fuel level system a specal floot charnber is required. o Intbisarrangementthefuelisinjectedintotheall.streo.lr-priortoenteringthecombus-
tion chamber. Fuel spray may be delivered from a single-point injection (S.P.L) source,
o For controlling/eliminating the formation ofice in the choke tube and on the throttle which is usually just upstream from the thottle (air intake side of the throttle), or it
valve due to low ternperatre an automatic de-icing unt is {'ted or hot engine oil s may be supplied from a multi-point injection (M.P.I) source, whete theinjectors are pos-
. drrdnged to flow around the carburettor barrel and through the hollow throttle ualve. tioned,ineachnd.utionmanfold.branchpipejustnfrontofthenletport. (Fig. 11.20).
INTERNAL COMBUSTION ENGTNES
CARBURE'TION AND CARBURETTORS .., 393

A najor feature with petrol injection a tlnt there is seporote air and. fuel metering and that
fuel meterng s precise und.er a engne operating cond,ition's.
1 1.10.5. Injecion Considerations
The fuel can be discharged into the air tream,using indirect injection arrangetncnts,by the
foilowing froo methads :
1. Continuous injection.
2. Intermittent or pulsed injection. )
1. Continuous irfection :
In this arrngement, the injector nozzle and. value are permanently opez while the engine is
operating and t}ae amount of fuel discharged. in the form of a sprdy is controlled by either uarying the
metering orifice or the fuel d.ischarge pressure, or a combination of bth of these possible variables.
2. Intermitten or pulsed irfection:
In this type ofinjection, /uel is deliuered frotn the injector in spray form at regular intervals
with a constdnt fiiel discharge pressure and. the amount of fuel dischorged s controlled by the time
period, thc injector nozzle ualue is open.
o Timed. i4jection This where the start of delivery for each cylinder occurs at the sane
angularpoint in the engne cycle, this can be anything from 60" to 90" after T.D.C. on the
induction stroke.
o Non-timed. i4jection- fn contrast to timed injection, this is where all the injectors are
prograrnmed, to d,ischarge ther spray at the same tne, therefore each cylinder piston will
be on a d.fferent part ofthe engine cycle.
11.10.6. Comparison ofPetrol kfecion and Carburetted Fuel Supply Systems
Merits of petrol injection :
Following are the merits of petrol engine system :
1. In petrol injection system ,d.ue to absence ofuenturi there s thc minmurn of air restriction
so that hgher engine uolumetric efficiencies can be obtaned with the corresponding improuement in
power and torque.
Fig. 11.20. Ihree-principal injector positions. 2. The spots for pre-heating the cold air and fuel mixture ate elmnated so that d,enser air
o Indirect injection can be discharged at relatively low pressure (2
to 6 bar) and need not
enters the cylinder when the engine has reached normal operating conditions.
be synchronized to the engine's.induction cycl.-el 3. As the manifold branch pipes are not greatly concerned with mixture preparation they
can be discharged simurtaneously
to each induction pipe where it is mixed an sbred unrir can be d,esigned, to utlize the nertia ofthe air charge to increase the engine's uolutnetric efft.cency ;
the i"l"t ;;;;;;;..-'- (this does not apply for single point iqiection).
since indirect injection does not need to be timed, it requires
only row d.ischarge pres-
sures and the injectors are not erposed to combuo.n-, 4. Because of drect spray discharge into each inlet p ort, accelerotion response is better.
n" -
mechanisms can be greatly red,uced, whch consid,era.bly ""iiiril;";;';;;;r"rr"g
towers iost.
6.Atomzation of fuel droplets is generally improved over normal speed and load driving
r The single-point injection system has the same air and
fuel mixing and distibution
conditons.
problems as a carburettor lavout but without 6. It is possible lo use greater inlet and, exhaust volve ouerlap without poor idling, Ioss of fuel
ueiiri restriction aliun"";^s*"
' uolumetrc efficiencies are "i
High injection pressures,compared.with or increased ethaust pollution,
rettor discharge method of2b1ai-n1d, the carbu- 7. The monitoring ofengine operating parameters enables accurate matching ofair and fuel
fuel derivery, speed up'and itnproue in" otoilti-
liquid spray. o tn" requirements under normal speed and load conditions which improues engne perfonnance, fuel con-
o The multpoint njection layout, in contast to the
single-point injection method as o
sumption and, red,uces exhaust pollution.
fuel dstribution difficurtes since each injector discharges directly-into its 8. Fuel injection equipment is precise in meteri.ng injected. fuel spray into the intake ports
port and the mixture own inoctio' over the complete engine speed, load and temperature operating range,
then has onry to move a short distnc" "o"u if".ri"r.
Since the induction manifold dealo mainly with-only induced
l" 9. There isprecise fuel distribuion between engine cylinders even under full load conditions
air, th" b.;;;l;;;"i ""11"*."
enlarged and exrended to maximize the ram effect"oithe with multi- point injec tion.
inc"-i.i;;il;;;:-"' "..,
10. Multi-point injection dnes not require tme for fuel transportaton in the intake manfold
and, there is no rnanifold. wall melting.
11, With fuel injection, when cornering fast or due to heavy braking, fuel surge is eliminated,
394 TNTERNAL coMBusrroN ENcrNEs
CARBURETION AND CARBURETTORS
395

The single point as,well as rnulti-point injection systems areprticularly 1. Fuel delivery system
. . .12'
uita b le for supercharged
ad.aptable and 2. An induction system
s e ngines. 3. Sensors and air flow control system 4. Electronic control unit.
Demerits / Limitafions of petrol injection Fuel delivery sysem :
1.
:
Petrol injection system entails the following dern erits I limitatans Itconsists of an electrically driven fuel pump which draws fuel from a
fuel tanh. The
: pump forces the oil through a fiIter into a line at the end of which is situated
1. Initial cost of equipment is high replacement parts a pressure
; are also expensive. regulator,which in tun is connected to intake manifold.
2. Increased care and attention required.
3' In oder to diagnose fuel injection system faults and failures, special servicing o The pressure regulator keeps the pressure difference between the fuel pressure and the
equipment manifold pressure constant, so that the quantity offuel injecterl is depdndent on the
is necessary.
injector open time only.
4' It is necessary to-have considerably moe mechanical and electrical knowledge to diag-
nose and rectify the faults offuel equipment. 2. Air induction system :
5' Injection equipment may be elaborately complicated, delicate to handle r After passing the air filter, the incoming air flows through an air flow meter, which
and impossible to generates a voltage signal (depending on the quantity ofair flow).
service.
6. More electical and mechanical components to go wrong. r Just behind thethrottle ulue is fitted a cold startrnognetic injection ualue, which injects
7' Due to pumping and netering ofthe fuel thee is increased mechanical additional fuel for cold start. This valve also supplies the exta fuel needed during warm-
and hvdraulic
noise. up period.
8' Due to the fine working torerances of the metering and discbarging
components, very o An auxiliary uolue (which by-passes the throttle valve) supplies the extra air required
careful frltration is needed. for idling (in addition to rich-air-fuel mixture). This extra aii increases the engine speed
9. To drive the fuel pressure pump or injection discharge devices, power (be after cold start to acceptable idling speed.
it electrical or
mechanical) is necessary. o To the throttle valve is atached athrottle switch equipped with a set ofcontacts
10. More bulky and heavy (than that of a caburetted fuel supply which
system). generate a sequenc ofvoltage sigrrals during the opening ofthrottle valve. The
voltage
I 1.10.7. Electronic Fuel Iqiection sgnals result in injection of add,itionat fuel required, for occeleration.
Fig' 1l'21 shows the fuel injection system-LJetronic with air flow metering (developed 3. Electric control unit:
by
Robert Based Corp.) It consist ofthe following znils : o ltle sensors are incorporated to reasure the operating data at different locations. The
data measured by the sensors are transmitted to the electronic control unt which com-
putes the arnount of fuel njected, d,uring each engine cycle. The amount of
fuet injected is
uaried, fut uarying the injector opening titne only.
a T'tre sesors used are :
Manifold pressure ;
Engine speed;
Temperature at the intake manifold.
4. Iqiection time :
I For every revolution of the camshaft, the fuel is injected twice, each injection contribut-
ing halfofc fuel quantity required for engine cycle.
r The injectors, at different phases of the operating cycle, are operating simultaneously.

1 1.U. TTIEORY OF SIMPLE CARBURTTTOR


During the induction stroke, the air is sueked through the carburettor by the pressure differ-
ence across it created when the piston moves. As the air passes through the venturi,
its velocity
increases and reaches maximum_(section 2-2, see Fig. 11.22) at ventui throat, this being
the mini-
mum area in the induction tack (unless the tbottle is sufticiently closed to prvide a smaller
aea).
As a result of suction created in the venturi fuel is suckecl through the noze. The tip
of the nozzle
\ is z metres above the float chamber level ; this arrangement prevents spilling
ofpetroi when vehicle
is stationary. Let us find expressions for air flow neglecting and considering the compressibility
of
i air.
I

Fig. 11.21. Fuel injection system-LJetronic with air-flow meteine.


396 INTERNAL COMBUSTION ENGINES
CARBURETION AND CARBURETTORS
39\
To engine
where, pr= Constant density offuel, and
C = Velocity of flow of fuel.

Wpr p, - srp r,
^ r---q-=
',= ...(11.6)
Dscharge [It may be noted that due to petrol surface being lower than the top ofthe jet by z metres the
nozzle pressure difference becomes (LO"-gzpr)inste_ad of4p"l
with jet \
Mass of fuel per second, rn, (theoretical) Af
= p* Ar Fp f@; c"p ...(11.7)
where, Ar= Cross-sectional area ofthe fueljet m2.
.'. Air-fuel (A/tr') ratio.

r'==-b!zgPz-=42E
nf W ...(11.8)
Ar J2pt $p" - ezpt) Af \pf \(Lp" - Czpr)
If c* and c
*arc tl'e coelficients ofdischarge of venturi and fuel jet respectively, then
"
Lf=9^
C A'tlO '
4, tp" ...(11.e)
+++
1-t-t
If z=0, o l, E
Ai
ti\f=c^
Ca' A tlO ...(11.10)
Fig. ll.Z2. Principle of a simple carburettor.
Case II'
Taking into consid.eratinn the compressibitity of air in account-Exact Analysi!.
Caae . Neglecting the compressibitity of air-Lpproximate Analysis, the compressibility ofair is aken into account, the air flow will change but the fuel
Applying Bernoulli's equation at sections 1-1 and 2-2, the equation for air flow is given by ^
flow
IFut
will
remain unchanged. Applying steady flow energy equation (s.F.E.E.) at sections 1-r.
and
2-2,we get,
Pt *Ctz -Pz tCz2 ..(1 1.1)
Pa2Po2
where, p" = Density of air kglm3, and nr*{+e=h,*91*s,
z-2
pr, pz = Pressure at sections 1-l and d-2 respectively.
C 1 = C = Velocities at sections L-L and 2-2 respectively, m/s,
2
or e-w =(h"-h,)* cr'-cr' 2
Assuming initial uelocity of air to be zero (Cr= O), densit! of ajr (p") to be consfonf, since air is
assumed, incompressible, we have
whee ht, h2= Enthalpies at sections 1-1 and 2-2 respectively.
SinceQ=0,W=0 and Cr=0
Pt
- Pz *Cz2 ...(11.2) c2= J4h1-tt)
PoPo2
Substituting
c"=@=
'11 Po
...(11.3)
h, = coT,
and. h"= crTr,we get

I
tl
where, Lpo=pt-pz
Mass of air per second rizo = C2A2po= Az JZprLp" ...(11.4)
Cz=
Pr'fA
Since the flow process between the atmosphere and the venturi throat is isentropic,
...( 11.11)

il where A, is area ofventuri thoat in ?-1


r2.
(The above equation $ves theoretical mass flow of air. The actual nass
flow is obtained, by
... rz
Tt la.);
= ...(11.12)
multiplying the co-efficient of d.ischarge of uenturi) lp,J
l
Similarly, for the flow of f.el, we have Substituting eqn. (11.6) in eqn. (11.S), we get
'i
Pt - Pz *"r' * ,"
,.
"'L \rrJ J
I
ll Pt Pr 2 -
..(11.5) "2 - fr-l.)''-""1 ...(11.13)

l
-*r
CARBURETION 399
398 INTERNAL COMBUSTION ENCINES l

Now, the mass offlow ofair is constant from inlet to ventui throat, and is given by I WORKED EXAMPLES

2"=AtCt-AzCz ...(11.14) Example lt.l. A four qilinder four-strohe engne hauing dameter and length of strohe as
u1 2
I
100 m and 120 mrn respectiuely is running at 1800 r.p.m. Its carburettor uentur hss a 28 mm
where ur, uz = Specific volumes at sections 1-1 and 2-2 rspectively, throat. Assuming co-efficient of air flow 0.8, density of ar 1.2 kg I m3 ond uolurnetric fficiency of the
'l/t t tV'l engine as 75 per cent, determine the suction ot the throat.
Sinceprurl=p2u2r, i. uz=rrftl-
' '\n) =It[al
pt
...(11.15) Solution, Giuen : D = 100 mm = 0.1 m ;.L = 120mm = 0.12m ; N= 1800 r.p.m. ;
\p2)
Substituting the values of C, and u, in eqn. 11.8, we get
Throatdiameter,dz=28 mm=0.028 m;Co= 0.8;po= l.2k9lml irluo.= 75%.
Suction at he hroat Ap. : .

- u")l ' ll
-l.A.J
l r, )k Stroke volume =
4
x (0.1)2 x 0.12 x 4 = 0.003?? m3
l Actual volume per strokes = q"or. x 0.003?7 = 0.?5 x 0.00377 = 0.00283 m3
.'. Actual volume sucked oer second
t t2lt r(l+l)/1
_ AzPt
-RE lel -l:'l ...( 11.16)
=o.oo283*
180r I
\rrl \rrl 260 =0.o4245ms/s
tito = 0.04245 x 1.2 = 0.05094 kg/s
As the initial temperature and pressure are not given, the problem is solved by approximate
method i.., neglecting compressibility ofthe ai.
tir^ Cr^ A" x-+x
Dt ..(11.17)
titf =x-
Cdf Af RJft ti'to = Co, A2= J2A" Lp" ..tEqn. 11.41

Comments on Air-Fuel (A / F) ratio supplied by the carburettor : 0.05094 = 0.8 x I(0.028)2


1. From eqn (11.8), it may be observed that no fuel flow will take place when Lpo < gzpt'
$xLzx Ap"

When Ap" > gzpr,the fuel flow wlll stat and the mixture becomes progressively richer. = ?.63 x 10 -1 16-,
The minimum air velocity at throat to cause fuel flow, with given value of z, can be calcu- tt2
lated as follows : I 0.05094
ttp-=l -'---- - z I| --4457 N/mz ber' (Ans.)
= 0.04457
| 7.63 x 10-' J
From eqn. 11.4, ...(11.18)
Example 1L.2. A spark ignition engine on test consumes 5 kg I h of petrol when runnrg on an
air.fuel ratin of 16 : 1. The engne uses a sngle-jet carburettor havtg a fuel orifice orea of 2 sq mm
2. At higher air flows when dpo >> gzpr(i.e. fractioogzprl\p,becomes negligible), the air-fuel
and the tip of the jet is 5 mnt aboue the leuel of petrol in thc float chamber, when th engne is not
ratio approaches
running,.Colculate the d.epression inthe uenturithroatto maintdin therequired. fuel flow rate through
"
tili =h.%
A Ca lp
E ...(1 1.19)
the carburettoi. Assutne specifc gravity of petrol es 0,75 and. the cofficient of d.ischarge of the fuel
orifice as 0.8. Wha.t orea of venturi throat wll be required. to maintdn thz d.esred flow rate ? Densit5r
of ar is i.20 kg / rn, ond the cefficient of discharge for uenturi throat is 0.8. Neglect compressibility
3. A reduction in the density of air educes the value ofA/.F, i.e., the mixture becomes richer. of air. (Roorkee University' AMIE' S-20OO)
It happens at (t) hish air flows when apo is large, hence c, is large and p2 becomes small' (ii) high
altitudes where density of air is small. Solution. Giuen: tit, = ^= = 0.001389 kg/s; A/T ratio = 16: 1 ;
JTJUU
Thus in a simple carburettor (he air-fuel tni.ure becomes progressuely richer wth ncreas' Fuel orifice aea,,4"r = 2 mm2 - 2 x 704 mz ; z = 5 mm = 0.005 m ;
ing air flws and. increasing al.titud,es ; this is a big d;utback. The other drorbachs are :
Sp. gr. of petrol = 0.75; Cq= 0.8, po - 1.2 kg/m3; C* = 0.8.
o It cannot \supply rich mixture required during idling and low load operation.
Depression in venturi throat, Ap. :
r It cannot bupply rich mixture for staring from cold.
The'actual fuel flow rate is given by,
. It cannot supply extra fuel needed during acceleration.
*i =cdf .Ar tFra; ...tEqn. (11.7)l
where Apo is in N/m2.
or 0.001389 = 0.8 x (2 10-6)
"

*l
400
INTERNAL COMBUSTION ENGINES
CARBURETION AND CARBURETTORS
491
ot -iq1391.
0.8x2x10-o
=ss.?s G;=6re
/ o.oorsas ),
or

or
^p"
_ s6.ze
=
los ; J
= L1.44.89 (df)2
Ap" = 639.2 N/m2. (Ans.)
Throat area, Ar: "' *;4=-l
d
,=
' (
13600x tt44.S9
= m2 x ro-3 m or 1J2 mm. (Ans.)
J

Example 1-l'4' A simpre.carburettorund,"i o


Air flow rate, ,h-- = 5
16 = 0.02222 kets with anair-fuetratio of rs. rn" certan conditian d,eriuer\ s.4s kg t h of petror
f:"ii,"";;;;;;;;;;;;/i;;;;;';i;;;iiliio.ru.
"h
8606 " of the fuel jet is 0'635 im aboue'th rf the tp
Also, tito = Co* e, ,|fr$ ...(Eqn. (11.4)
lya . p"ro|i, ''"
cient of discharge is assumed to be 0.g0, ""nt"t
t' chambr inil the untri thr.oat coeffi-
,'
Q) The uentur depression in cm of"|""ul"i"-
rate. Hro necessdry to cause'air ond, fuel flow at the desired.
& = 4)
o.o2z22 =0.r, o na**u$* (i) ?" r"n ur' throt d,iameter.
... A.
' =---9"
0'8./2x1'2.ffi =7'72o9 x 10-am2 I
(ii) The uelocity of air
across the uentur throat.
You lnay take density of ar 1.29 hg/
= mr and. specific gravity of period. 0.72.
1
=
=7.7205cm2. (Ans.) (Madras University)
Example ll.8.The following data relate to a petrol engine : Fig.11.23
Solution. Giuen : rir, =
Petrol consum,ed per hour = 0.001514 kg/s ; A,rF ratio = t5;Ar=2mm2
' = 2.2 hc **
The speciftc grovity of the fuel -.
= 0.75 _, L.29
lO^*.
-l ; C q, = 0.7 5 i z = 0.GScm = 0.0068b m ; Cda = 0.8 ;
The ternperature of air = 27"C
p" = kg/ms ; Sp. Cr. of petrol 0.22.
=
The air fuel ratin (l) Venturl depression, Ap.
=I:15 :
The dameter of the clwk tube
= 24 mtn r = ca.Ar J2pLp; c"pl
The height of top of the jet aboue the petrol leuel 4.2 mm 0.0M2 ...[Eqn. (11.7)]
= = m in the float chamber where Apo is in N/m2.
The co-efficent ofdkcharge for air
= 0.g
The co-efficent of discharge for fuel 0.0015f4=0.75 x2x10a
= 0.7
Atmospheric pressure
= 1.013 bar
Calculate the diameter of the fuet jet of a simple carburettor. 0.00r514
sz.e5n(4p;:44sd
Solution. Giuen:
7' ffi =
r =
Oy":p=9.7g
x 1000=?50kgims ;Tr=27 +278=800K;
A/Fratio = 1 : 15; dr--24mm = 0.024 m ;z = 4.2mm= 0.0042 m: apo - 44'85 =
f o.oorsr \,
w'sz
C"= 0.8;Ca,= 0.7 ipr = 1.013 bar. l.r* ffiir* ) =

Diameter of the fuel jet, d, : :. Lpd=752.22N/mr=ffi mofwater =7.67cmofHr0. (Ans.)


We know that. o = Pr =_ 1.013 x 105
= 1'176 kglm3
(jj) Venturi throa diameter,
D,
"'- Rl (o r8 :
,F
Ajr. flow rate, o = AzCd" J2p.Ap, Air flow rate = -== x 15 = 0.02271 kg,ls
. 15x7.2 1r - Also,
x(o'024)2 xo'snEx l-rzex4P.
ffi=; rizo = CoA, Jrp"Ap"

= 5.b5 x to4 JLp" 0.0227 L = 0.8 x A, Jrx Ln x ?lzn


:' (
ts rz.z \' A,=6.444x10{m2=IO:
^P'=lr***u'rt)=29221^z
Fuel flow rate, rh = ArC* ppf Lp;- g"pf] , I a.++ax to-a x 4lv2
...[Eqn. 11.7]
' = l------:-l =0.0286m=2.86cm. (Ans.)
L'IJ
INTERNAL COMBUSTION ENGINES CARBURETON AND CARBURE'I-TORS

(lii) Velocity of air across the venturi throat C. : Throat diameter d, :

C, (or Cr)
WEf rj ...(Eqn. 11.18)
Applying S.F.E.E. at sections
l-l and,2_2, we have
IPo ,hr+ c,2 c-2
2 x 9.81x 0.00635 x (0.72 x 1000) =-+q=h2+\+W
= 8.34 u/s. (Ans.)

Example 11.5. A carburettor ; tested in tle loratory has its float chamber uented to atmos- or c,2
ll1= h2i a
phere. The main metering system is ad.justed to gve an oir-fuel ratio of 15 : 7 at sea level conditions.
The pressure at the uenturi throat is 0.8 bar. Tle atmospherir pressure is 1 bar. The sanne corburet- c2= JzW:t.r)
tor is tested agan when an air cleaner is fitted. at the intet to the ca,rburettor, The pressure drop to air
cleaner s found to be 30 mm of Hg whzn air flow at sealeuel cond.ition is 240 hg I h. Assuming zero tip =,tug:q
and, consiant coeffcent of flow, calulate (i) the throat pressure when the ar cleaner is fitted
and. (ii) air-fuel ratio when the air cleorur is fitted. , (Bornbay Universit)
Solution. Giuen: AIF ratio = 15: 1 atsealevel conditions;
Pr=1ba;P2=0.8bar;
(i) The throat pressure when the air cleaner is fitted : t-]l-,El,.J
Quantity of air flowing is same in both the cases. =
(ix) = C& A n)
il".41'_lfrj
Jr+"@t
^"u"t
When there is no air cleaner,
(Cr)""r = C
&"= Pt- Pz = I - 0'8 = 0'2 bar
When the air cleaner is fi.tted,let prbe the throat pressure, then
llo^\l
p,'= ll-f
L \ 1000 ) '"J| ur
1000x 13.6x9.8rt=*= x ro-5 l-

= (0.96 -pr) bar ('.. 1 bar - 1d N/m2)


For the same air flow and constant coefrcients,
, I p, )o*t
Lp.= &i '-ll-orgj
0.2 = 0.96 -r,
pr = 0.76 ben (Ang.)
or
(ii) r-fuel ratio when the air cleener ls fited '-(,6)""'=(#l=oor66e
Without air cleaner, Lp= Lp"-- 0.2bar'
With air cleaner fitted (with float-chamber still vented to atmosphere), or Throat pressure,
| 'l
p, = (1-0.01669)o.zasz
| lx r.or3 = 0.955 bar
&t=1-0.76=0.24bar LJ
As Aat has increased more fuel will flow makig the mixture. richer. Pr 1'013x105
o,=
- E4
tsr =(org?" = 1'1844kg/m3
lp without air cleaner looo)-rr8
"
New A / F ratio = A / F ratio when air cleaner is not fitted
" i- qotrh .1"r"* - Now pur = constant
IJ "t"
( tu' J# = 18.6e. (Ans.) $ = consant

Example 11.8. A simpte jet carburettor is required, to supply 4.6 hg of air per minute' The or Pt-Pz
pressure and.lemperature of air are 1.013 bar and 25"C respectiuely. Assuming flow to be isentropic pry Pzr
and. compressibti and. uelocity coeffrcient'os b,8, calculate the throot diameter of the choke for air t. lJt I
|ou uelocity of 80 m I s. jo" pz=pJezl =1.1844 f0'e55) = 1.1856ks/mg
Solution. Giuen : mo= t= 1'013 bar, T t = 25 + 273 = 298 K ;
= O.OZAZ kgls ; p
I
^arl \r.0rg/
ff
. Cr=0;Cz=80rn/siC, =0.8.
\,

INTERNAL COMBUSTION ENGINES CARBURETION AND CARBURETTORS


40s
We know that, rh" (= pAC) = 9242C,
0.0?67 = 1.1356 xA, x 80 (wheeAz = throat area) Now, a"=bcz
or A"= 8'443 x 10-a m2
q
.'. Throat area,
or Az= 8.443 x LO-1 =+ d22
A"=oxuz - 0.1x0.898
C"-=- n
-
=976x10-am2=9'?6cm2
But er= nz2 =o.za
... nroatd.iameter, *= [*5*J"1 =o.orrro, S.28 sm. (Ans.) |
D, (or e) 9.76x4
Example L1,7. A simple jet carburettor is required to supply 6 kg of air per minute and = = 3.525 cm.. (Ans.)
0.45 kg of fuel of density 740 hg lm3. The air is initialQ at L0I3 bor and.27'C. (jj) Orifice diameter, d.:
(i) Calculate the throat diameter of the chohe for o flow ueloaity of 92 mls. Velocity coeffi- Pessure drop at venturi 1.01g _
= 0,987 = 0.0?6 bar
ci.ent = 0.8. Pressure drop atjet O;75 x 0.0?6
(i) If the pressure d,rop ocross the
= = 0.052 bar
fwl meterng orifice is 0.75 of that dt th choke, calculate
Now,
the orfice diameter ossuning Co = 0.60. (AMIE, 5-2001 ; Nagpur University) rh = AC,!2p-$

Solution. Giuen : tit o= Ot fuls ; rit.= = O.OOZS ke/s ; p = 7 40 kglns ; o.oozs =Arx 0.6 ;;t;oo5?;lou =
* =
H Af = 4.50q x 10-6 m2
t742.68
11 = 1.013 bar;?, = 27 +2'13 = 30OK; Cr=92rils i C" = 0.8; or 4.804 mm2
But, t
cdf = 0.60. tLr=
(i) Throat diameter, D, : idf- = 4.304
Velocity ofai at venturi throat, . tl,lscr-
ot= (Ans')
I f r-rl V--;- =2'34mm.
E*t-ote !7'8' The following data rerate to a 4-strohe petror engine of
c,=c* ,o", Hind.ustan Ambassa-
lz",r,l'-(f)t l Copacity of the petrol engine
re
tLl

t-[.l tt
Speed at which nat'
rheuotumetric"rfi
"',::;,":oT:;#"'",;:"ff ='r?T"i!;I
= Hg9 c.c.

| \rrl
e2 = 0.8 .fz" r.oos' 1000x8001 Theair-fuelrato
ll
I

l Theoretical air speed. at chohe (at peak =B:1


power) = BS ml s
The co-efficient of d.ischarge
for uenturi' = 0.g2
-,-lrr)H
l. pr ./
=r!)'"
\0.8/ 2x1.005x1000x300 = 0.021932
The co-efficient of d,ischarge of
The specific grauty ofpetrol
the main petrol jet
= 9.6g
= O.Z4
t0.2857 Leuel of petrol surface below the
choke
... lz = 6 mm
{.fJ =o.e7lor, pr =o.szs Atmospheric pressure and, temperdture
,ifi"iut,on,"u,,=,;;o::#X::;:;fi::::;::,"henas40
P2 = 1.013 x 0.925 = 0.937 bar ,,,"",f:,iti::;:;:;y:l"t,-"i"
Now, PPlr = P2u21 Calculate the sizes of a suitable choke and main jet.
z r'/'t
lPrl
Solution Guen : V" = 1489 c.c. = 14g9 x 10-6 ;s 0.0014g9
= m3; N = 4200- r.p.m
" '\n)
U^=Url-l Quo1. = 7 Vo ; A,/F ratio
= lt : t t G C z) =85 m/s ; C " = 0.AZ : C
",(=po)='10rlbar;p,(=p,)= *= i O.AS
- ;

I Jl%lr? ?;r,(=r")=20 +278= 2esK;


nf' f 1 )14 " =. X',T:t:r1:|'-3;P,
=-l-l Volume of air induced
p1 \0.925J = Iuol. x %

I
28?x300 f I \07143
=--.-''.---..----'''-..-l-l =u'vm"/kg -_-O.75x 0.001489 x 4200 = 0.03909 m3/s
I 1,013 x 10o \ 0.925 / 2x60
li
i

r-*
f
INTERNAL COMBUSTION ENCINES CARBURETION AND CARBURETTORS
40 407
I

Pr' 1.013x105x0.03909
A=2.209 x 10-6m2 or 2.209mmz
. Mass flow of air, i'." - = u.u4709 kg/s
RTt 0.287x10'x293 Ar= D"t2 =2.200
For compressible flow, velocity at throat, |
D. - 1.68 mm. (Ans.)
[ , .]lll Exqmple Lt,9. The following d.ata refer to a siimple cdrburettor :
cr= ,*."t,_lfj ' ...[Eqn. (11.13)l Throat d,annetr 78 mn =
il Diameter of fuel orifi,ce = 7.2 mm
:l o.oo5 x 103)
Co:effi.cient of air flow
Co-efficient of fuel flow
= 0.82
'= 0.65
ss =
{z*zss*
{t ;,}-l Leuel ofpetrol surface below the
Density of air
throat = 6 mm
= 1.2 hglms
7 Density offuel
10.2857 = 750 ke I m3.
85 = 767.4
r-lLl
\P" )
Calculate :
(i) The Al F ratia for a pressure drop of o.06s bar when
the nozzle rip is ncgrected, ;
(ii) The Al F ratio uhen the nozzle lip is tahen into ancount
;
(iii)The minimum uelocity of air or crticar qir ueracibt required. to start
,
pa | =lr-(lo-fl*
\767.4) )
=oe5?6 nozzle lip is prouidzd.
the fuer flow when

Solution. Giuen: d"= 18 mm = 0.018 m ;


p, = 1.013 x 0.9576 = 0.97 bar dr= 7.2 mm = 0.0012 m; C*= 0.82; C*= 0.65;z mm 0.006 m =6 = ;
Volume flow of air at choke, Po= L.2kglm3 ; P= 15O kglmg
I
/- \; (j) A / F raio when the nozzle lip is neglecteil :
u.=0.03909'l&l'
' \P,/ ...tEqn. (11.4)l

1'013.l1
= 0.03909 xr = 0.04032 m3/s
\. 0.97 /
u.' n/Qo
**:",1
A=
-r
-
C, x Co= 85 x 0,82 =0.0005?85m2=5?8.5mm2

1t (D2 _
Now, d\ = 578.5
4
1l - o ^l
; lD2_ (0.4 D)21 = 579.6 u=%'
x 0.84 Ds = 578.5
+ ia,, l+)l
=(e"\" I

or 7
The air fl.ow will remain satne.The fuet flou willbecom,
or Chok dia., D = 29.61 nm. (Ans.) :
, : f -.ctxA,,tgd$JQfi . -,: ..[Eqn. (r1.7)]
Mass flow of (uel, z, =+=qgP = 0.003622 ksls
thf =Cdf .Aj ,l2pll,n"-czO) ...[Eqn. (11.7)J
A/Fratio =%-.4' trf
uaf AflP\Lt"-szO
0.003622=0.65xA; -o.82xlo.or8f
0.65
EZE
y750
\0.0012/ ]f0.065_(9.81x0.006x?50/105)
= 7639.75 Aj

=11.85x@- 0.0004414E =
l1'$9' (Ans')
v 0.065 -

-\_ _/
408
INTERNAL COMBUSTION
(lll) Minimum velocity ENGNES
CARBURE-ON AND CARBURETTORS
of air, Cr: p
of fuel when tip s proui.dkwilt
to ^_ Thufloy
,^ create start onl,r
dti;;;';"ffff;':f:r1:t::y::!e
requiste pressu," minimum uerocity of air required, Thus, air required for combustion of I kg offuel
"";.;";;d"::x,^;:::;(::,y:;:
cr = 0, :;:*x:;:w#iJ^e of venruri
leo\
=[o.ea'
ff * o.rox 8Jx
ff = rs.a e
-L=A*gt TLusA/Fratio=%=15.8
POPO2
The volume ofone kg ofair at 0.C and 1.018 bar,
or 9u=c"" \
*{
Pa2
cr' gzP "
, =
P 1.013 x 10o
= r'?l!,* = 0.778 mstks
Similarly volume of 1 kg of fuel vapour at 0.C anil 1.013 bar,
2Po
r1t. 0.2614
' =Y=
o" ,,
P rt1l
1.013xrOP
= ms/kg

.. **.rhenozzrerio . ":=ff=re=8'd8m/s' (Ans.) . Thus mol min x ud + mr / min x u" - g.40g


15.3 m, lyin , uo +
*j I min, ,r = g.Oe
r:trn:;**;iaii#,!,#,{r:"y;!",y,1#}:xnffi*{;":,;:^,F::t:;;r:::^:: Thus. :
Example ll.IO.The
following data refer to an etsht-ctlndzr
""" -=---?'408-=,
-rlmin lfi "J rr 15.3= o??g.9.408
. 02614 = 0'778 kg/min
Bore four-stroke petrol engine :
"
.'. Fuel consumption = 0.728 x 60 = 46.6g kg/h. (Ans.)
Strohe = ll0 mm (it) The air velocity through the tube,
Cr(= Co)
compostion of the = 110 mm Density of air at the throat.
fuel used
Throat d,iarneter of:the
chohe tube = c-^= 84v" ; E, = 16vo
Volumetric efficienqr at = 42 mm o-=-&-
The pressure depr"ssnn
J00 r.p.m.
= 75% (refened to 0"C and 1.015
&r,
I
= 0.12 bar
bar) _ h- apo
The temperature at the
throat RoTz ("' P1-P2= LPo)
Characteristic gas constant
: For air = 15"C
(1.013 - 0.12) x tO6 _
- - 1'08 kg/m3'
rrchemicary*-*,*-*r*,1*',:::;,i:rr.,,"^r,j:,F,"*, .'. Velocity at the throat in
287 x (15 + 273)
Fuel consumption in kgl . . "eq tv' w\ m,/s,
.G)
(ii)
h; m- 153m"
/ _7"
ot, uelocity through the tube. ^
Solution. Giuen:D= A"p"
lx@.a2)z xr.O8
n"", =
ffi".mi*#
The volume of mixture
i# i,,;tr\{,;!=;l}%.?3;^}: '=I.;"o;l,mm = 0 042 m ; t5.Bx(oiz8l60)
=;------^- = f32.59m/s. (AnsJ
supplied at 0.c and 1.018 bar per minute ; x (0 042)' x 1.08
- - isE=fPle ll.l].. Determin^e the air-fuel rato supplizd. ot 4500 m altitude by a carburettor
=!pz"8*{r,
+ rvol'
Lhich adiusted to giue an air-fuel ratio u
t I oi ,tr uu"t where ai.r i.s 25"c andtr^p"*ii"
2 pessure 1.013 bar.
' ='t' x (0.11)2 x 0.11 9q00
,' , r
g x o.z5 = 9.408 ms/ min. The temperature of air decreoses with altitud as giuen
by the etpression,
Also C + fr
= Coz
-z
12 32 44 where h is the height in metres
2Hr+Or=2IIzO The pressure of "'r;?";o;!"!Tr!t t"^p""otur" in "c.
air decreases with altitucle as per relotion :
43236
r69 h= rsrootosrci#l
\ v '/
whre p is erpressed. in bar at attitud.e,

r
I
410 4tl
INTERNAL COMBUSTION ENGINES CARBURETION AND CARBURNTTORS

Solution. t t,- O.0064 h


=
oBJECTTVE TreE QUnSTfoNS
- 0.0064 x 4500 = - 8.8"C
= 25
/r
tto FilI in the Blanks or Say "Yes" or'tlo" :
Now = 19300 los.^ | t ^,^\ I
l. Carburetto is used for ............ .
\p) 2. For maximum power of S.I. engines the fuel air mixture ratio should be ....-....... .
/r
4500 = 19900 log,n | ''"o
^rd\ 3. Tlpical pressure in the induction mmifoltl at te start of induction stroke of S.I. engine, uder idling
--t p i I
conilitions is ...........- .
.toe.fI r.ors'\ 45oo 4. Relative.fuel-air ratio (Fr) for muimum power\in S.I. engine may be ."......... .
Relative fuel-air ratio (f'") for nuimum therml efficiency of S.I. engine may be ............ .
p J=l93oo=0.2332 5.
6. For best thermal efficiency of S.I. engine, the fuel-air mixture ratio should be ....."..... .
_1.013 = 1.71r
7, In S.I. engine the prmess of preparing a ombutible fuel-air mixture outside the engine cylinder is
p called ............ .
1'013 8. The pipe that anies the prepared mixtue to the engine cylinders is called the intake manifold.
P = L?11 = 0.592 bar 9. The A / F ratio for naximum power is not the game as the A / F ratio for maximun economy .
10, The maximum poweris obtained at about............ A/F ratio.
A / F ratio at altitude
Now, _
- 6"" 11. Atfullthrottle,rnmimumefficiencyocmrsatanA/Fratioofabout......'......
A /F ratio.t sea tevel {l*[ 12. In stationary engines the designed air-fuel ratio is that wich gives the maximum economy.
13. The riening of mixtue incremes the probabity of contact betwem fuel and air particles md thu
A/F ratio at elirude = 11 /orJ8&" improves ..,......... .
p"olRT""
I 14, In the wbuettor, complete automization and vaporization of the fuel is achieved'
15. Thevolatilityoffuelsigoificantlyaffectsthestartingadwarmupcharacteristicsoftheengine.
=ro/E;;?:*
p"* x ?t. 16. The term'ac,celeration'with regrd to engines, is gaeny used to refer t an increase i engine seed
I resulting fron opening the throttle.
ose2*(zs*2zB) 17. The petrol mgine is ............ govened.
=14/1.018 x (- 18. In the modem carburettors the mixture mnection is done by air bleding alone.
X 8.8 + 2?3)
19. A comon method of chmging the air-fuel ratio in large carburetton is the back suction control.
= rr-26. (Ans.)
20. Almost all caburettors, except S.U. carbwettor, are ofopen oke type.
2t. The important dilference between m,airraft anrlautomobile carbmttor is that the fomer operates at
varying altitudes whreas the latter opntes mctly t groud level conditions.
.+__ r
q, Engines fitted with petrol iqiection system can be used in tilt position which will cause srge trouble in
mrbuettors.
T:^1T:::r
in the s.t enginrc, acombustible tuelair mixture ouLside the engioe cylinder is
"?*parg,
calldcorburetian. 24. Injection systeru generate lss noise.
, A carburettor is a device whicl atonises to fuel and
mixes it with air.
24. Weight mtl bulk of petrol iriection systen is more thm that of a cuburettor.
3. The air-fuel ratio for maxirnm powe is lB : 1. 25. A petrcl injection srstem has increased volumetric efciency.
4. The following addition devices / systems are added to the
simple cuburettor :
(i) Main metering system
(ii) Idlingsystem
(iii) Power enrichment or econoniser'sfsteur 1. S.I. engine 2. rid 3. 0.3 bar 1 l:2 5. 0.8
(ru)A""l"r"Oornumpsystem 6. lean 7. carburetion 8. Yes 9. Ys 10. 12.5:1
:
(u) Choke. ( ll. l7: 1 12. Yes 13. combwtion 14. No 15. Yes

6. Carburettors, basically, are ofthe following tryes


16. Yes 17, quartity 18. Yes 19. Yes 20. Yes
: 21. Yes 22,Yes 23. No 24. Yes 25. Yes.
(i) Open choke typ
Examples: Zenith, Solex, Carter md Stromberg carbuettors
(j;) Co**rt u".uum tJrpe
Enimple : S.U. carburettor,
t_

412
INTERNAL COMBUSTION ENGINES CARBURETION AND CARBURETTORS
313
THEORETTCAL QTTESTIONS
TJNSOLVED EXAMPLES
I. What do you mean by the tem ,carbuetion,?
2. Whst is a urbuettor ? l' A fou-cfinder four-stroke engine having diameter and logth 12o m respc-
of stroke as 100 mm and
8. tively is ruming at 2000 r'p.m' Its carriretto ventuJias r
tfnical inducon throat. Assuming coefficient of air
flow 0'8, densify of air 1.2 kg/m md vohmet.ic efficiarcy of e-*
a system ofa petrolelgine. so
4.
frlw
Enlist the facktrswhich afrect the process engine as Z0 perceit,-etemine ttre
ofcabuetion. suction at the thoat.
6. Why is a chokelrsed in a carburettor ? 2. A simple jet carburettor
6, Explain brie'y the essential features of good i: *ql{:{
740 kg/m!. the air is initiallv l" supply 6 kg of air pa minute and 0.,15 kg of fuel of density
at l.0rs uar a iz"c. alculate the throat aiu-qte, or"trre
commercial carurettor for automotive for a flow
() Draw a we representinr engines. veloeity of91 m/s. Velocity coeffrcient 0.g. "iote
of mirture requiren*t, (fu"I_"i, =
l::1lt,"l
load in a s'I' mgine, mark-the relative position of sriichiiitric fuel-air""Uos) fom no_load to full_ Ifthe presswe dr-op across the fuel netering orifice is 0.?5 of that at the
ratio line and then explain choke, alrulate orifrce dimetr
assuning C, = [.6
(l) m idlingengine requiree a rich [Ans, 35.25 rrm ; 2.34 mml
mixtue : 3' A 4-stroke-petrol engiae of Hindutan Ambassador has a @pacity
of 1489 c.c. It clevelops mffium
(ir) a mising engine requires an powerat 4200 r'p'm. T'he volmetric efficiency at this sped
economy mixtue is ?0 percent and the airlrel ratio is lg : 1.
(ijj) muin,,q power demarls a rich rnirture
; At peak power the theoretical airspeed at choie is
90 is. The milicient of ilischarge for venturi is 0.g5
; andthat of the main petrol jet is 0.66. Ar allo**c"
*, effects of protongett
runnig oisuch an engine on (i) be made for the emulsioi tube, the diameter
H::Hhge the weakesr mixrure ; (ij) the ofwhich cu be taken as 1/2-5 ofthe choke iameter."hould rtre petrol surface is 6 mm below the choke at this
engine condition. The soecific gravity ofpetrol is 0.74. Atnospheric
8. With the help of a neat sketch axplain the
workingprinciple of a simple carbuetto 1.013 ba md 20.C respctively.-
pressure and temperature are
9' A simple carburettor is inherently tortitrt"
tol"l-tih.yarying mixtre requirements
whar are rhe dravbacks of a.simpiec;;;;;Ji-.--11"
calcrlate the sizes of a suitabre choke and main jet.
of s.L engine. [Ans. 22.85 mm ; l.5g mmJ
bvincorporaring compensating 4' j"l"imp/arb rrettor is to supplv 6.rI kg/min. of air and 0.408 kg/min
deces ? Explin s,ith the aid of sult"1"
"*r--e
JJ""u". o".".""o. 1:Tgl"
768 kg/mr' The
of petrol, density
10' "L"tr"',
Describe with suiiable sketches the fo'owing air is initially at 1.0-22 a.r" f .s'C."cal""lat the throat tliameter of the venturi throat
sys*"" cabuetto : if the speed of air is 97.5 m/s, asauiring a velocity coeffra"it
(j) Main meterig system "ir -ou*" ftlit oo.g. "".-. ,"Ji
(di) Idling system
; :: 1'4' Ifthe drop acrms firel nitering otirr."
orifice diametcr assming a coefcient as 0.61.
il o.s pressure at"ai".ti" "*p.1"" the
ttre throaiicatculare
; "rthe
(iii) Economiser system [Ans. 2.05 mn]
5' hgvin-g a simple singlejet carbuettor consumes 6.5 kg
(iu) Ac"""."t'oo
; tmr' The level of fuel,/hour. The tuel density is 200 kg/
nump system. =tT" offuel in the floal chamber is 3 mm below the top of theet when the engine is
Ambient conditions are 1.01325 bar,and-1?"C. thejet not running.
lr' withtheherpofaneatsketchdescribetheonstructionandworLingofasolexcarburettor. dlametei is r.zs mm ana its disc"harge coeflicient is
12. How the power and efficiency ofthe S.I. engine 0'6 The- discharge cefficientof aiis 0.85. Air-fueiratiJ" is. o"t".-ire
the critidal airv-elocity and the
v"rJ *ni frl ,r.-n el atio at full throat diameter (effective). Exprss the presso"" d"pr;;;"
13, Sketct and explain the fuel consumption lo-op load ; () pari load ? air. in cm ofwater. Neglect compressibility of
in th S.i. [Ans.4.945 m/s; 19.9 mm;43.99 cm]
14. Why a rich mixture is roquired for idling ? ""O*, 6' An eight-cylinder 4-stroke petrol engine with bore antl
stroke of 100 mm each uses volatile fuel ofcompo-
15. Why a rich mixture is required for muimum pbwer sition C =-84%, Hz = 16%. The throai diameter of chokJue is
? o mn. rire volumetric efficiency at 8000
16. What do you understand by tramient mixture r'p m' is 75 percent efenedto,0'C md 1.01325 bar. The pressure
reqoir"_"rts Z peratue at throat is 16'c. Ifchemielry mnect A
depession is 0.116 bar and the tem-
17. Why multi-cylinder engines require richer mixture / F ratio is supplie for consumption, eiermine ;
than single cylinder engines ? (i) Fuel consumption in
18. What ae the basic tJpes of carbuettors ? fu / h ;
19, Explain with nea sketchs the following typee (id) The ai velocity through the tube.
of carburerors :
(i) Carter cabuettor Take characteristic gas-mnstant g for ai and fuel
as 2gZ kg K and 97 J/kg K respectively.
(ii) Solex carbuettor [Ans. 35.1 kg/h ; 116 r',/s]
(ttt) S.U. carbuetto. 7' The venturi-ofa simple carburettor has a throat
diameter of20 mm and the coefficient ofair flow is 0.g5.
The fuel orice has a diameter of 1.25 mm and the
20. Device ar expression forA/F ratio coefficient orruel flow is 0.66. The petrol surface is
5 mm below the throat. Assuming density of ancl fuei
(o) neglecting compressibility
(i) The A/F ratio for a pressure drop
as 1.2 kg/m3 and ?50 f.g/-, ti;ly, calmlate :
;
() takingcompressibilityintoaccount. of0.0? bar when the nozzle lip is neglected"u"p" ;
(ii) 1'" ," when the nozzle lip is iaken into account ;
21. State the special reqrements ofan aircaft "ur'o
(ili) The ninimum
mburettor ? velocity ofair or critical air velocity required to stat the fuel
22. What is petrol injection ? provided. flow when nozzle lip is
23. State the advantages and limitaions ofpetrol tAns. () rs.2; ,iie.zis;i;,t 2.83 rlsl
injection. 8' A carburettor with float chamber vented to atmosphere
is testecl in a laboratory without the ai cleaner.
24. Explain briefly,continued, md.timed, injection The A / F ratio as calculated is 15 at tttu ut-o"ptt*i"
or r.oos ar. The pressrpe recodcd at the
25. With the help of a neat sketch, explain briefly istems. throat is 0.812 bar. "o"iitio.r
ai p"*ol io.u*ion system.
26, What is the diffeence between Direct injection,ina This carburettor is fitted with air cleaner-and once
again tested. The additional pressure drop due to air
27' Exprain the difference between 'continuous injecion, fJirect inection,? cleane is 0'04 bar wih the.air flow at the atmospliei"
to remain unchanged at260kg/h.
and , Intermittent or pulsed
boih ...1""tior.
injection, Assuming negligible nozzle lip, sam" ar no* ir
28. Give the comparison between petrol injection coeffrcient offlow determine ;
and carburetted fuel supply systems. (i) The throat pressud with cleane
fitterl : "ra "or.tu.t
(ii) The A / F atio with cleaner fitterl.
fAns. (r) 0.2?2 bar : (ii) 13.6?l
414 INTERNAL COMBUSTION ENGINES

A 4-stroke petrol enginewhi whentsted at sea level contions of30'C md 1-01325 bu gave the A/F
ratio of 14. The same mgine wc once aga tested at an altitude of 4000 m. Determine the A,/T ratio at
high altitude if the temlrersh vies with altitude as :
t = t,- 0.O07
The pressure varies as
h;
:
=8350h(r.0l32fp)
L2
where, , = Tempeature at sea level, "C
Fuel Injection Systems for C.I. Engines
= Height, m, md
p = Pressure, bar. lAne. 11.051 12'1' Introduction. 12.2. Functional requirements of m iqiection system. 12.g.
Functions of a
fuel injection system. 12.4. Fuel injection systens-Air injection-sorid o airless
injection.
12.5. Fuel pmp and fuel injector (Ato^mteF_Fuel p.mp__buel
atomizer
causes and remedies of injector-s. 12.6. Types of norzles anrl fuel ".;j;;a;._;;;;;;
ptt"rn._fVf"lri
requirements of an injector nozzl+classifilation and desription or"pr"y
starting systems. 12.8. Fuel inj_ection computation in C.I. elgines. "or".
il.i- n"gr""
Worked exampts_-'--"
Highlights-objective ftpe euestions-lrreoretical r"anjte.
euestibm-uisotvea

12.1. NYIIODUCTION
In c.I. engines, the ai is taken in during the suction stroke and cornpressed
' - pressure (28
to a high
to 70 bar) and high temperature (520. to ?20.c) according to the compres-
sion ratio used (12 : I to 20 : 1). The high temperature of air at the end
of stroie .
sufficient to ignite the fuel.
Fuel is injected into the cylinder at the end ofthe compression stroke
- ; the pressure of
fuel injected lies between 100 to 200 bar. During the process ofinjection the
fuel is bro-
ken into very fine droplets. The droplets vaporise taking the heai from
the hot air and
form a combustible mixture and start burning. As the burning starts,
the vaporisation of
fuel is accelerated as more heat is available. As the combustion p"ogr"""urjth"
amount
oforygen available for burning reduces and therefore heat release is reducea.
- isTlreperiodbetweenthedtartofinjectionandstartofignition,
about 0'001 second
called,lheignitiond,elay,
for high speed engines and 0.002 Jecond io, lo* speed-errgines.
The
injection period covers about 25" ofcrank rotation. After the igrrition
the temperature
and pressure rise rapidry. The whole performance of engine isiotarty
dependent on the
d,elay period; the lesser the d.lay periad. better is thz
engine performince.'

12.2. FLINCTIONAL REQUIREMENTS OF AN INJECTION SYSTDM


The functional requirements of an injectinn slrsm are listed below :
I' Introduction ofthe fuel into the combustion chamber should take place within a precisely
defrned period ofthe cycle.
2' The metering ofthe amount offuel injected per cycre should done very accurately.
3. The quantities offuel metered should vary to meet he changing load
and speed require-
ments.
4' The injection rate should be such that it results in the desired heat
release pattern.
5. The injected fuel must be broken into very fine droplets,
6' The pattern ofspray sho'ld be such as to ensure rapid mixing offuel
and air.
7. The beginning and end ofthe injection should be sharp.
8. The timing of injection, if desired, shourd change as per the requirements
of load and
sDeed.

415
416 INTERNAL coMBUsTToN ENGINES
FUEL INJECTTON SYSTEMS FOR C.r. ENGNES 4{
9. The distribution ofthe metered fuel, in the case ofmulti-cylinder engines, should be un! Disadvantages:
form among various cylinders. This method is of-sed now-a-days due to the fo[owing re asons / d.saduantages :
10. Besides above requirements, the weight and the size ofthe fuel injection system must be (i) It requires a high pressure multi-stage compression. The large
number of parts, the
minimum. It should be cheaper to manufactue and least expensive to attend to ; adjust or intercooler etc. rnake the system complicated and expensive.
repair. (ii) A separate mechanical linkage is required to tle the operation
of fuel valve.
For accomplishing these requirements the following functional. elements are required in a (iii) Due to the compression and the linkage the bulk ofthe engir:,e increases. This
fuel injection system : also results
in reduced B.P. due to power loss in operating the compression and
t. Pumping elements. To move the fuel from the fuel tank to cylinder and piping etc. |inkage.
(iu) The fuel in the combustion chamber burns very near to
2. Mctering elemenfs. To measure and supply tbe fuel at tbe rate demanded by the load and injection nozzle which many times
leads to overheting and burning ofvalve and its seat.
speed.
(u) The fuel valVe sealing requires considerable
3. Metering conrols. To adjust the rate ofmeering elements for changes in load and speed skill.
(ui) In case ofsticking offuel valve ihe system becomes quite
of the engine. dangerous due to the presence of
high pressure air.
4. Distributing Iernents. To divide he metered fuel equally among the cylinders.
5. Tning controls. To adjust the stat and the stop ofinjection. f2.4.2. Solid or Airless Injection
1"
6. Mixing elemefs. To atomise and distribute the fuel within the combustion chamber. Injection offuel directly into the combustion chamber wthout primary automisation is
tenned,
as solid irfection. It is also termed as z echacal njection.
12,3. FIJNCTIONS OF A F'I,IEL INJEC1ION SYSEM
Main Components:
The main functions of a fuel injection qstem are :
The main cotnpone)B'ofa fuel injection system are :
1. Filter the fuel.
(i) Fuel tank:
2. Metre or measure the correct quantity offuel to be injected.
(ii) Fuel feed punp to supply the fuel from the main fuel tank to
3. Time the fuel injection. the injection pump ;
(iii) FueI fiIters to prevent dust and abrasive particles from entering the pump
4. Control the rate offuel injection. and injectors ;
(u) In""o, ournp to meter and pressurise the
5. Automise or break up the fuel to fine paticles. fuel for injection ;
(u) Gouernor to ensure that the amount of fuel
6. Properly distribute the fuel in the combustion chamber. is in accordance with variation in load ; and
(vi) Fuzl pipings and, iniectors to take the fuel from the pump
o The injection systems are manufactured with grea a.ccuracy, especially the parts that and distibute it in the combus-
actually meter and inject the fuel. Some ofthe tolerances between the moving parts are
tion chamber by atomising it in fine droplets.
very small of the order of 1 micron. Such closely fitting parts require special attention Main types of modern fuel iqjection systems 3
during manufacture and hence the inJection systems are costly. 1. Common-rail injection system.
2. Individual pump injection system.
12.4. FUDL INJECTION SYSTEMS
3. Distributor system.
In compression ignition engines (diesel and semi-diesel) two methods of fuel injection are
Atomisation of fuel oil has been secured by (i) air btast and (ii) pressure spray. Early diesel
used. These are :
engines used air fuel injection at about 70 bar. T?ris is sufficient not only
1. Air injection to injectihe oil, but also to
atomise it for a rapid and thorough combustion. The expense ofproding
2. Solid or airless injection. an air compressor and
tank lead to the development of"solid" injection, using a liquid pressure
ofbetween 100 and 200 bar
12.4.1. Air Injection which is sufftciently high to atomise the oil it forces through spray nozzles.
Great advances have
In this method offuel injection air is compressed in the compressor to a very high pressure beerTmade in the.field ofsolid injection ofthe fuel through
research and progress in fuel pump, spray
(much higher than developed in the engine cylinder at the end ofthe compression stroke) and then nozfles, and combustion chamber design.
iniected through the fuel nozzle into the engine cylinder. The rate of fuel ad,mission can be controlled
by uurying the pressure ofinjection air, Stotage air bottles which are kept charged by an air compres-
/ l. Common-rail injection system :
sor (drivcn by the engine) supply the high pressure air.
j rwo types of common-rail injection systems are shown in Fig. 12.1 and r2.2respectivery.
I o Refer Fig 72.1. A single pump supplies high-pressure fuel to header, a reliefvalve holds
Advautages:
\ Pressure constant. The control wedge adjusts the lift ofmechanical oDerated valve to set
(i) It provides better artomisation and distribution offuel. amount and time of injection.
(jj) As the combustion is more complete, the b.m.e.p. is higher than with other types of injec-
\
I r Refer Fig. 12.2. controlled-pressure system has pump which maintains set head pres-
tion systems. sure
(iii) Inferior fuels can be used. \ sprayPressure reliefand timing valves regulate injection time and amount. Spring loaded
i valve acts merely as a check.
INTERNAL IBUSTION ENGINES
-J
Mechanically
FUEL INJECTION SYSTEMS FOR C.I. ENGINES
419
acluatd
spfay Advantages:
Valvs (j) lhe system arrangement is si'ple and less maintenance
cost.
(ii) Only one pump is suffrcient for
multlcylinder engine.
ttt'll*lo.lltf1".:,",fiXffi:" oreither the constant load with variable
speed or constant
(iu) Variation in pump supply pressure
Hgh will affect the cylinders uniformlv.
pfessur Dieadvantages:
16lf
valre (i) There is a tendency to develop leaks in
the injection valve.
(ii) Very accurate desigrr and workmanship
are rlquired.
2. Individual pump injection system:
t an.in<lidual purnp or pump cyrinder connecrs
Hader, or "common rail"
l;'f"rl:f.;1li"1$Tj"tl:- direcuy to
cotrol injection timing. Nozzles contain
aeliveryl,arv" a
"c,ilX#:!,?ktffand
Fuel from
day tank High-prsssuro
pump

Pumpdrive
camshaft

Fig. 12.1

Injecton
nozzles

Pump with an
individual
cylinder for
each nozzle

Fig. 12.3. Invidual pump iqiection sysem.


o The design ofthis typq ofpump must be very accurae and precise as the volume
injected per cvcle is u20,000.oittr" *ci"" offuel
i'"p"";"nt ut ron ro" and r/100,000 of the
engine disptacemenr during idriog, rri.
of fuel is verv limited (abo
ii---iu,i*e'-forinjecring;;;;;ii;;;"tity
'.1/4solecond rt rs p"""iil"* l'irjl"u""
through 20. crank ansle). The p*r;";;;;;;;fi;"",y
".p.^. "tu" "rii""
iom 100 ro 800 bar.

I
t _-.---_,_FF___

FIJEL TNJECTON SYSTEMS FOR C.I. ENGINES 421


420 INTERNAL COMBUSTION ENCINES

3. Distributor system :
Refer Fig. 12.4, In this system, the fuel is metered at a central point ; a pump pressurises, Passage (P)

meters the fuel and times the injection. From here, the fuel is distributed to cylinders in conect
firing order.by cam operated poppet valves whi open to admit fuel to the nozzles.
I
Spring (S)
Nozzles

Delivery ntve (V)

Ban6l (B)

Supply port (Y) spill pon (SP)


Metering and Plunger (L)
Distribulor
pressure pump Back (R)

Control
rod

/cam

Fig. 12.4. Diskibutor system.

12,5. FUEL PTIMP AND FIIEL INJECTOR (ATOMISER) Fig. 12.5. Fuel pump.
12.6,1, Fuel Pump a when the plunger is at its bottom stroke the ports sP and y are uncoverpd (as shown in
A large number ofigneous fuel pump designs have been developed by the manufacturers. Fig. 12,5) oil from low pressure pump (not shown) after being filtered is forced into the
Only one type fuel pump will be discussed here. barrel. when the plunger moves up due to cam and tappet mechanism, a stage reaches
when both the ports sP nd Y are closed and with the further upward movement of the
Bosch fuel injection pump: Refer Fig. 12.5.
plunger the fuel gets corfrpressed. The high pressure thus developed lifts the delivery
o L is the plunger which is driven by a cam and tappet mechanism at the bottom (not valve offits seats and fu{ flows to atomise through the passage p. with further rise of
shown), B is the barrel in which the plunger reciprocates. Thee is a rectangular verti- the plunger, at a cetain
cal groove in the plunger which extends from top to another helical groove. V froment, the port SP is connected to the fuel in the upper part
is the ofthe plunger through th\ rectangular vertical groove by the helical groove, as a result
delivery valve which lifts off its seat under the liquid fuel pressure and the spring ofwhich a sudden drop in
force(s). The fuel pump is connected to fuel atomser through the passage P. SP and y are {ressure occurs and the delivery valve falls back and occupig
its seat against the spring f\ce. The plunger is rotated by the rack .R which is move, i in
the spill and supply ports respectively. or out by the governor. By cha\ging the angular position ofthe helical groove (by rotatirrg

I
INTERNAL COMBUSTION ENGINES FUEL TNJECTION SYSTEMS FOR C,I. ENGINES

the plunger) ofthe plunger rerative to the supply port, the rength ofstroke
during which
the oil is delivered can be varied and thereby quantity offuer delivered to the
engine is Elfective stroke
also varied accordingly.
commences
The positions ofthe plunger and helicar groove atthe starting and end. ofthe deriv-
ery stroke when the engine is running at full load is shown ln Flg. f2. (a). Quantity ol
The positions of the plunger andelia-I--sroo-ie a-trtF.est'rting and end. of the deliv- fuel injected
ery stroke when the engins running at part load is shown in Fig. 12.6 (b). In this
case, the delivery takes place for a shorter period,
when the engine is to be stopped, the prunger is rotated to the position as shown in
Fig' 12.6 (c). At this position, the rectangular sro is in rine with the spill port and
there is no possibility ofpressure build-up above the plunger as the uppei part of
the plunger always remains in connection with t.l-e spilr port. Therefore th"r" i. ,ro
delivery of the fuel.

Quantty of

@MffiM
fuel njected

(a) (b) {c) L plrng",

Fig. 12.6
The amount of fuel supplied by the pump under fferent loads is shown in Fig.
12.?.
W. 12.7 . kinciple of helir bypass pump.

12.5.2. FueI Atomiser or


Effective shoke \iector: ReferFig. 12.g.
commences It consists of a nozzle valve (rvl) fitted in
he nozzle body (NB). The nozzre varve
on its seat by a spring's'which exerts pressure is herd
Quantity of througfihe spin1". ,s;i.irr"
ing screw by which the nozzle valve lift'can "rrt-
fuel incted be a4ustei. us.ratty
lifl at 135 to 1?0 bar pressure. .FP is the reering pin;iich the nozzle valve is ser o
indicates whether valve is
working properly or not.
The fuel under pressure from the fuel pump
enters the injector through the passages B
and c and rifts the nozzle varve. The iuel
i."u"t. dor"o oozzre.iv and injected into the
engine cylinder in the form ofne spray.
Then the pr"r.o"" ofthe oil falls, the nozzle
valve occupies its seat under the springforce.na
n-ei ,rrppfy is cut off- Any leakage of
fuel accumulated above the valve is led
to the fuel tank ttrrough the passage A. The
leakeige occurs when the nozzle valve i
s worn out.
424 INTERNAL COMBUSTION ENGINES
FUEL INJECTION SYSTEMS FOR C.I. ENGINES
{2s
12.5.3, Faults, causes and rmdies of iqiectors:
Feeling pin (FP)

S. No, Faults Causo Remedy

1. Pressure tui high (a) Spring rate too high. (c) A4iut if possible or replace spring
with one having lower spring rate,
Adjusting or varr the number ofshims.
screw (AS) () Needle valve seized in almost () Soak in fue\ oit. Then try to remove
closed position due to dirt etc. needle valve. If necessary replace
'with new needle valve and nozzle.
(c) Carbon blockiag ir{ector holes. (c) Clean with pricker or proke. Re-
Spring cap nut (SCN) verse flush uing sringe and par-
afm.
2. Pressure too low (a) Spring rate too low. (c) Adjust ifpossible or replace spring
Spring (S)
Passages (A) with one having a higher spring
(A, B) rate.
(b) Spring broken () Replace with cmect type.
(c) Neille valve seized in position. (c) Soak in fuel oil then try to remove
(B) the valve. Ifnecessary replace with
uew needle nozzle,

-' 3. Spray distorted (c) Danaged needle (o) Replace.

is
Fuel from
() Nozzle o holes carboned
altering shape of orifrcg.
() Clea using pricker or proke.
Revese flush using syrilge and
parafn.
fuel pump t Dribble from injector Valve not seating conectly
Spindle (E) hohs ercessiue due to
Passage (C)
(a) Dirt or carbon between needle (a) Clean with paraffrn ; do not wipe
seat and nozde. with fluf! cloth, but assemble wet
from clem fuel. Had carbon may
be removed by using a solution of
\ in 0.6 litres
60 grams caustic soda
ofwater and add 15 grams of de-
tergent. Boil for 1 to $hours.
() Scoring or pitting on needle () If possible lap needle md nozzle
Cap nul (CN) geat or nozzle seat : seat together.
(c) Needle conoded, or binding (c) Replace as necessary.
nozzle due probably to faulty
spindle or misalignment of
Nozzle valve (NVj
nozzle md nozzle holder.
Excessive leakage off (o) Too much clearance between (o) Fit new nozzle and valve.
valve ad hole in nozzle.
() Nozzle cap loose. () fighten.
Nozzle body (NB)
(c) Injector body md nozzle separ- (c) Remove nozzle and examine pres-
Nozzle (N) ated by dirt or bums caused sure faces. Rcmove dirt or buros.
by lack of are in handling. Do not overtighten cap iut as a
mte.
Fig. 12.8. Fuel atmiser or injector.

-'----''.---..----.t-'-
426
INTERNAL COMBUSTION ENCINBS FUEL INJEC'IION SYSTEMS FOR (].I. ENCTNES
42'/
12.6. 1YPES OF TJOZZLES AND FUEL
PATIERNS o usuallv the h"t"" bu.t many times they are non-symmerricar
xr^"_1:11."_l.ymmetricalry
meet certain specific requirements of the ombustion to
12.6.1. Main Requirements of an nozzle chamber.
Ad.uantages:
The main requirements of an nozzb cre: (i) Givesgood utomisation.
t at-a sufrciently.iglr.pr.s".r." so thar the fuet enrers rhe
cytinder with a (ii) D,istributes
a

T,Llf.::l-T-:l
hi-sh fuel p.roperly even with lower air motion avail-
uetocitv' Hisher the ueroiitt if n"
f*t
of smaller d.roplets is less, hence, penetration"^t"iiitfi" iil,ii"i
,;;;. ;'il,;;;:r^ aDle rn open combustion chambers.
is clso. Disad.oantages:
Penetratiori should nof be high so as to impnge on cylinder Fuel in
staing.
walls this may re surt
; poor in (l) Holes ae small and tiable to claggittg.
3. Fuel supply and cut-off shourd, be rap theteshould
(ii) Dr ibbl ing behreen injections.
; be o drbbling. (iii) Very high njection pressures ( lg0 ba and
12.6.2. Classification and Descriplon of Nozzles above).
(lu) Close tolerance in manufacture (due
to small holes) and
The type ofnozzle used.is greatry dependent on the type hence coslly).
ofcombustion chamber as open type
or pre-combustion chamber' The nozzles ari classified
as per-the type oforifrce unJit" ,rr*u"" ,r"ea
for injecting the fuel in the combustion chamber. 3. Circumferential nozzle, ReferFig. 12.11.
The nozzle are classified as : o Its spray characteristics are similar to a plae type
open_
) r.
1. Single hole nozzle.
2. Multi-hole nozzle
o The. injected fuel particles tend to be projected
in the form
of plane, with wide angle cone tn" p"""e
3. Circumferential nozzle ; of l"-t i"i, i" to
obtain as large al area of fuel u, porriUil-io
4. Pintle nozzle "p.rv
into contact with the air in the combustion .h^;;; "o_" Fig. 12. 11. Cirmmferen-
5. Pintaux nozzle.
4. Pintle nozzle tial orifice.
1. Single hole nozzle. ReierFig. 12.9. Fig. 1,2.72 shows a pinile nozzle.
r This.is th_e simplest type of nozzle and is sed in open corn_ o The stent of the nozzle valve is extended to form pin
buston chatnbers. a or
pintle-which protrudes through the mouth Nozzle
o It consists of a single hole bored centrally through the noz_ ofin"'nou"
body. ltmay be either cylindiical *
zle body and closed by the needle valve. ihe size?tie
hole The-size and shape ofthe pintle"o"i"fln-"i"p"".
can be vaed ac-
is usually larger than 0.2 mm.
cording to requirement. Th spray core angle
a lts spray cone angle vaies from 5 to 15., In some cases, a erally 60..
is gen_
core is given a series ofspiral grooves in order to
impart a
rotational motion to the fuel for bettering with air.
o When the_valve lifts, the pinfle partially blocks
the ori_
fice and thus does not allow t" p..rr"r" ."o't" Nozzle
Ad.aantages: greater. As the lift ofthe valve increases
. body
the entire ori_
Smple in construetion and, operation. fice is uncovered and full area fo, no* i",uulluUlu.
Fig. 12.9. Single hole nozzle.
dribbling s auoided,
ii",
Disadoontages: Fig. \2.12. Pintle nozzle.
Thespray obtainedby thepntlenozzleishollow
( Very high injection pressure is requredbee-ause cal spray.
coni-
whole of the fuel passes through a single
hole and, also, because therelativlfuel velocityrequired is high. Aduantages:
(ii) This type of nozzle h as a tendzncXr to dribble.
(tii) A the spray angle is very narrow (usually It is selfcleaninglype and prevents the carbon
about 15.), this does .(j) deposition on the nozzre hore.
no,t-facilitate.good mixing unless higher air velocities (li) It avoids weak inj'ection and dribbling.
are pro-
vrcled- \ (ill) It results in good atomisation.
2. Multi-hole nozzle. Refer Fi g.1rz.l}i. (iu) Its injection chaacteristics are
more near the required one.
This type ofnozzle finds extensive use in automobile engines, Disadvantage:
particularly having open combustion cbambers.
Distibution and penetration poor, hence not suitable
It mixes the fuel with air properly for open combustion chambers.
even wih slow air move-
ment available with open combustion chambers. 5. Pintaux nozzle:
The number ofholes varies from 4 to 1g; the greater o rn case the fuel is injectedin a directio n upstrea.m
the d,rection ofair, the delay period is
number
provides better fuel distibution. The hole iame"ter
lies belween Fig. 12.1O. Multi-hole
educed due ro increased hear rransrer
b;;;;;;i;;,.,;;."t.';Jr.ii'*il
0.25 to 0.35 mm and hole angle lies between 2O"
to 45". tozzle. -

't

I
428 TNTERNAL coMBusrroN ENcINES
FUEL INJECTION SYSTEMS FOR C.I. ENCINFS 429

starting performcnce. However, if whole of the fuel is injected in this manner the efii-
ciency of combustion is greatly reduced by flow of products of combustion back into the
injection path. Thus in order to improve cold starting performanc without any detri-
mental effect on efffciency apintaux nozzle as shown in Fig. 12.13. is used.
1 t 1
Nozzle valw E

Nozzle body
0 1020304050 o 1020so4050 40 50
---)
Crank degrees 0 0 ---'

Auxiliary hole (i) Muhi-hole nozzle (ii) Pintle nozzle (iii) Pintaux nozzle
(a)
frg. 12.14. lnjection ate charactristics of nozzles.
Engine rp.m. -----t
Fuel spray pattern :
600 1000 ttoo 1800 2200
. For better and quick evaporation of fuel which is essential for better burning offuel, the
fuel pattern and relative direction offuel particles with air are ofsignificant importance.
+
I
o To give an idea, how the patterns help for better evaporation, two patterns are shown in
I
Fie. 12.15.
I
E
E
.--t-----'-.
.z ,s.::ll\__--
o
/+r
0 200 400 6@ 800 10oo Fuel seeks air Ar seeks fuel
PumP rP.m.----') (i) Open combustion chamber (ii) Pre+ombustion chamber
(b)
Fig. 12.15. Air md fuel movement for two tlpes of combution ambers'
Fig. 12.13. Pintaux nozzle.
e A pintaux nozzle is a type of pintle nozzle which has an auxliary lol drilled in the 12.7. ENGINE STARTING SYSTEMS
nozzle body. ILinjects a small amount offuel through ths add.itonalhole(pilot injection) The following thee are the commonly used starting systems in large and medium size en-
n the upstream slightly before the rnain njection. The needle valve does not lift fully at gines :
low speeds and most of the fuel is injected through the auxiliary hole, giving good cold
1. Starting by an auiiary engine
starting performance.
2. Use ofelectric motors or selfstarters
Disaduontages I Drawbahs : 3. Compressed air system.
(i) The tendency ofthe auxiliary hole to choke. 1. Starting by an auxiliary engine (generally petrol driven) :
(ji) The injections characteristics are even poorer than multi-hole nozzle. In this system an auxiliary engine is mounted closed to the main engine and drives the latter
Injection rate characteristics : through a clutch and gears. The clutch is first disengaged and the auxiliary engine started by hand
i o In order to auoid hnochng in the engine it is always dsirable to suppl1 less quantity of or by a self starter motor. When it has warmed up and runs normally the drive gear is engaged
through the clutch, and the main engine is cranked for starting. To avoid the danger ofdamage to
fueL
I
drive gear it is desirable to have an over-running clutch or starter type drive.
The characteristics of multi-hole, pintle and pintaur. nozzles are shown respectively in
l
Fig. 12.14 (i), (ii),(iii). 2. Use of electric motors or self starters :
These are employed for small diesel and gasoline engines. A storage battery of 12 to 36 volts
Itis obvious fom the gure the pintle nozzle gives desired characteristics as
is used to supply power to an electric motor which is geared to th Il'wheel with arrangement for
(dm/d0) which is smaller at the beginning compared with multi-hole nozzle. The
chaiacteristics of pintaux nozzle is totally diffeent because of fuel supply through automatic disengagement after the engine has'started. The motor draws a heavy cunent and is
auxiliary and main oifices. designed to be engaged continuously for about 30yonds only, after which it is required to cool off,
,/
430
INTERNAL COMBUSTION ENGINES INJECTION SYSTEMS FOR C.I. ENCINES
for a minute or so, and then re-engaged. This is done till rhe egslrr{shrts up. When
dbe engine is Neglecting Vr, being very small compared to
running a srnall d.c. generator on the engine serves to chyel battery. Vo, we have

3. Compres*d air system , vr= J%r@;;; (1q o\


,r/
The compressed air u'stem is commanly
p, startng large d,esel engines employed for
stdtionory pouer pl,a service. Compressed air;labout 1? bar suppli-ed from an iir tank or bottle is
admitted to a few of the engin_e cylinders makig them work like reciprocating air
mJors to run the
=l*i- f;7---...-

or ifVr= Actual fuel velocity ofinjection (the velocity offuel for good atomisation
...( r2.3)
engine shaft. Fuel is admitted to the remaining cylinders and ignites in the n'rmal is of the oder
way causing the 400 m/s), and of
engine to start. The air bottle or tank is charged by a motor oi gasoline engine
driven compressor.
The system includes the following: * Cr = Flo* coefficient of orifice,
(i) Storage tankfuepsel
(ii) A safety valve Then, ur="rF* .(12.4)
(jjj) Interconnecting pipe work.
The volume of the fuel injected per second
o small stationary engines ofabout I0 kw capacity are started by hand cranhing. e is given by :
Q/ = Area ofall orifices x fueljetvelocity x time ofinjection x number ofinjections
for one orifice per second
12.8, FUEL INJECTION COMPUTATION IN C.I. ENGINES
Refer Fig. 12.16. The fuel injector shourd develop a pressure which is higher than
r- - 'r ie 601 N,
dn'""01' vt *
highest pressure dsred to be obtained in the engine and also additional pssure
the
differ- 1n",", ,.
L'#J- ;d .,.(r2.5)
ential is available to irnpart heh uelocity head. and, adequate atomisan. rhere is an = Diamerer
optimum value of the fuel particle size and velocity, so that the momentum is maximum
"r*", "noljll,i
o = Number of orifices,
and hence depth ofpenetration ofspray through already compressed air in the cylinder V/= Velocity offlow offuel through orifice,
is large..orcessiue atomsotion is not uery conducive to = Duration of the injection in degrees of crank angle,
iroper mixture formation.
N= r.p.m., and
I N; = Number of injection per min.
P,,V,- r.p.m.
? 2 ... for 4-stroke cycle engine
Pz,Yz
= r,p.m. ... for 2-stroke cycle engine
T:::n:l*"r"1:!::Y:^:,:dtt
utne to auoid, the effect of engine size.
apressed. in mms I d,egree tank anste I titre cytinder vor.

Engine r Since the injection pressures are very high (to the tune of 100
.employed to 150 bar), the
cylinder assumption_ofincomprescihility of-fuel may lea to In order to account for
".ro.r.
compressibility, a factor called "Coefficient Compressibittl,,
isi"t.;u;;;:-
Coe fficz nt of comp re s sib iligr,

^
[u1"qp -uJ
' uf(oup) (Pr - P"*p) ...(12.6)
Fig. 12.16 For pressures expressed, in bar, accepted, valve
for C": g0 x 10- 8 per bar
pt = Injecion pressue, Work of fuel compression per kg,
% = Veloci8 at secion l-1, '|
pz = Pr?ssure in the cylinder when injection of fuel takes place, w" =
f
(p, - p.o-/ x [utrop - u J
...(r2.7)
Vz = Veleity at section 2-2, Wok of delivering fuel,
Pr= Density of fuel, and W= (pt_p) x volume of ful injected.
...(12.8)
u/ = Specific volume of fuel (assumed in compressible). Then, fuel pump work per kg,
vr2 v2 Wo= W"+ Wo
Then, -f*nru,=t*prrf ...(12.1)
...(72.9)

-'l
I

i
I
INTERNAL COMBUSTION ENGINES FTIEL INJECTION SYSTEMS FOR c.I, ENGINES

WORXEDEXAMPI,ES
1.e63 x ro-s =l|^,xe]x rsr.zaxl#,#], ffi =zs.as+ ao"
Example 12.L. A six-cylind.er, four-stroke dcsel engnc develops 125 hw at 3000 r.p.m. Its
brahe specific fael consumption is 200 glhWh. Calcul.ate the qmnty of
fwl to be injectcd jer qrcle .'. d.-[r'sog]-rojl"=a.rsr10-m or 0.813nm. (Ans.)
per cylinder. Specific grovity ofthe fuel may be taken as 0.85. " \ 2s.6e4 )
Solution. Guen : n = 6 ; B.P. = 125 kW ; JV = B(X)O r.p.m. ; b.s.f.c. = 2.00 g/k\fh ; Example 12.3.Fuel injection na snglecylindcr,4-strokecycl.eC.I. engine runnng at 650 r.p.m.
Sp. gr. offuel = 0.85. takes place through a single orifice nozzle and, occupies 28o of crank travel. Thfuel consurnption of
Fuel consumption per hour = b.s.f.c. x B.P.
the engne is 2.2 hg / hour and the fuel used. has a specifc grauty of 0.875. Ifnjection pressure is 150
tnn bar and. the combustion. chomber pressure is 32 bar estitnate the uoluine of fuel injected. per cycle and
= I25=25ks the damzter of the orifice. Take cofficient of discharge of orfi.ce = 0.88.
,* Solution. Giuen : n=1, N= 650 r.p.m. ; 0 = 28" ofcranktravel ;
,'. Fuel consumption per cylinder = 25 Fuel consumption=2.2kg/h;Sp. Sr. = 0.8?5;
nG=2: = 4.167 kg,lh
try = pt- pz = 150 - 32 = 118 bar ; C = 0.88
Fuel consumption per cycle _ Fuel consumption per cynder per min. Volume of fuel irfected per cycle :
No. of cycles per min.

=
.ft## = 4.68 x 1or kg = o.o46s e
F.er to be injected per cycre _- "'"t*Ti#"f;11##*i"u""
.'. Volume of fuel injected per cycle
=ry!62=1.r2Exro+ ks
rc50tD
_ Fuel consumption per cycle
Specific graviy of fuel 'volume offuel injected per cycle
=
ffi
=
T# =o.o544zc.c. (Ans.) - x 1o-a
1'128
x 1000 -= '
0.8?5
1.289 x 1o-?
., Example t2.2. A -cylinilzr 4-stroke c.L engm develops 220 hw at 1s00 r.pm. with brahe = 0.1289 cm8. (Ans.)
specific fuel consumption of 0.273 kg I hWh. Detcrmine the size of the single hole injeitor nozzle if th.e
injection pressure is 160 br ond the pressure in th.e combustion chdibe, is 40 bar. The period. of Diameter of the orifice, do :
injection is 30' of cranh angle. specfc grauity offuel 0.8s and orifi.ce d.seharge coeffici.ent 0.9.
=
#) ,".
=
Solution. Giuen : n = no = 6 ; IV = 1500 r.p.m. ; B.p. = 220 kW,b.s.f.c. = 0.2?B kgr{Wh
Time for fuel injecrion per cycle = (*'
I= 30., Sp. gr. of oil = 0,85, Cr= 0.9, Ap = pr_ pr= 160_ 40 = 120 bar. 28 60
Diameter of the nozzle orifice, do : - 360
x = o.oo?l8s
650
We know that, actual fuel velocity ofinjection, Mass offuel injected per second,

_ Fuel injected per cycle 1.128 x l0-a


Vr= C -cf ...(Eqn. 12.4)
_, = 0Jo?1g = u.ur.o Kgls
to" fuel injection
Actual velocity orp". cycte inJ",rJ]tu
2xl2Ox!0s
=0.9x = 151.23 m/s
,,=",W=om
Volume offuel injected per second,
(0S5 >< 1000)

Now, mt=Aox\trx
mfi# =,445mls
0.273x22O P,
Qr= (0.85x1000)x3600 = 1.963 x 10-5 m3/s
Also, volume offuel injected per second, 0.0157 = (x^)"144.5 x (0.875 x 1000)

,, =lior' * no)*v,,l#, #].


1500
* ..(Eqn. (12.5) .t
"o=L nx
0.0157 x 4
L44.5 x (0.875 x 1000) ]" =,.nru x 1o-4 n = 0.4 mm. (Ans.)
(lvhere iy', No. of injection/min.
=
2
FUEL INJECTION SYSTIMS FOR C.I.
434 ENGINES
INTERNAL COMBUSTION ENCINES 435
Actual fuel velocity ofinjection,
. Example 12.4. A fou-r's-troke engine using o.222 kg r kwh
. - fuel of B2"rLprduetops rs hw per
cylinder-at 2000 r.p.m. The fuel injectiai prcssui is 720 btor and the
combustion chcmi}ressv*e+
3-0bar'Ifthedura,tionofinjectianisS0" ofctznktraueland.uelocitycoefficientis0.gdeterminethe
dameter of the fuel orifice. x = 127 .48 nls
Now, nf=AoxVfxPf
rabte sp.gr. =
Solution. Guen : s.f.c. = O.Z7zkSlkWh,
##Ef
power
(Madras University)
=f,a* xv xp,
developed = tb kW ;
Lp =p t- pz = 120 - 30 = 90 ba ; Duration of injection 30" of crank angle Cf O.9
=
Diameter of he orifice, do:
; = =
;.(ffi)' x 127.48 x(0.8654 x 1000) 0.0re14 kg/s
=

sp.sr.=#HE Time fo fuel injection per cycle 0 ,. 60 32 An


=0.86s4 = -
Fuel consumption/cycle -
s'f,c' I kW
Mass or ruer i"j""bd ;:;"'= ; SfS-; Ir;e ,U:'-r;;i:'; fit.,"
cycle / hour Total number ofcycles per hor" 19Q
= 60 = 54000
0.272x15 ... Fuel consumption in kgfl<Wh
= 6.8 x 10-5 kg
l-""" lx60
\2)-- = 5.671 x 10-5 x 54000 * O.rtt kgl'<lryh. (Ans)
=
Duration of injection = Example 12'6' An hgh'cylind'er, *
-
* " # # =
# = 0.0025s 800 r.p.m. The fuer **u^itoi
four-strokd
i, o.z's.'iit ifr|."
diesel engine has a power output of Jg6.4
fn| or*;* kw at
. x 105
6.8 injection is 32 bar and th" atimum
pr"ssur" s
in the cyrindei:r t tli
o"stnntrs o
nr= -o=o'0272fu\e 207 bar and the ma.ximum pressure at "y-Iiolder ls bgr The injector is expected to be set
ihe ineitor * ,"i u" at I

Actual fuel velocity ofinjection, required' per injector ifthe injection tahes otoui sdi iirr.-"t"i'ih,
ori6", or"o
pace ouJr-ii;'"rorn ongtr.
Assume the following :
v,=c,d-=o.s,
' p :::'f;tri:!r,'{{:"J";;?;i"t":::r\?I^!!:"!,":9"ff
't = 129.8 m/s
i:;:::;:;":,:::
tnjection perad.
bar ; rhe effectiue p,",,'," ii"i;;;;;;;
rtheiniector=0.6;atmospheric
;";"';i::::;;;r;,;,:,#":fy;#
mr=ArxVrx P, adras University)
Solution. Giuen : n g; power output 8g6.4
= = kW, jV = 600 r.p.m.,
= lndo2 xv, x p, Fuel consu:nption - 0.25 kg / kWh, 0 = f2" crank angle Sp. g.
; = 0.g5 ;
Cf = O.A ipu_. = 1.018 bar.
o.o2't2 = ldoz xtzs.ax(0.8654 x 1000) Orifice area reqd. per injection,
d :
.f
o=L O.O272x4 lv? kW per cylinde" 3T'a
x 129.83 x (0.8654
= = aa.e
x rO(X)) |
Fuel consumption per cylinder 4g.S x
= 5.55 x 10-{ m or 0.655 mn. (Ans.) = O.2E = L2.O7Ske t h ot 0.2012kg/ min.
Example I'2'5,A4'str-ohc cxcle C'1. enginc deuelops 11 kW per cylindzrwhile Fuel tti be injected per cycle
running at 1g00
r'ptm,7n!ulinsfueloilof32'API.Fuetinjectionoccupes32'ofciankiravetand.nhiiipt*ltn
ougn
a fuel njecton orifice 0.47 mm d.iamctcr with flwo o o.s. Fuer is ,injected at a pressure of Time for fuel injection per cycle
1 18.2 "n6"unt
bar into cornbustion chambeir wherc the pressuri' i, J l.gg bor.
Estimote thq'quantitl of fuet njected in kg / kwh. specifu grauity of Mass of fuel injected per second,
fuer oil is giuen by :
141.5
__ -l-=o'2o72kels
5.03x tO-a
Ifis+-ApI. "'r=
Solution. Giuen : power developed = 11 kW ; iV 1g00 0 = 32' of crank travel Pressure difference at beginning
do= 0.47 mm = = 207 - 32 = 1756^,
; C, = 0.9 ; 4p = pt-pz= 118.2-S1.A8 = 86.82bar Pressure difference at end
= 595 - 55 = 540 bar
sp.er.=
141.5 t4t.5

''*;.m= *iffi
=08654 Average pressure difference 775 + 540
2 = 357.5 bar
t

436 INTERNAL COMBUSTION ENCTNES FUEL INJECTION SYSTEMS FOR C.I. ENGINES
437
Now, mr=AoxVrx p,
^
I Eh-l = 7dt'xC,tztpxp,
x lc, l-l^r, =Aoxcr,ttp.pr
=A^
" L'I o' .' o.oz 1Et rro r ro\ = T2tlrtrdo2
or o.2oL2=Aoro.offi or
=
i*'* seo
do = 5.637 x lO-a m or
0.5682 mn. (Ans.)
Example L2.8. In a d,iesel fuel injection pump, the uorume of
.1 .on:iffltil?-z or o.or36cn,. (Ans.) c:mmence-ryelt-of the effectiue stroke is 7 c.c. The diameter of the
3 mm and is 700 rnm long. The fuel in the injection value is 2 c.c.
fuet n the pump barrer before
fuel line from-pump to injector is
Exanple 12,7. A s*-cylind,er, four-stroke oil engine operates onAl F ratin \
= 20. The diameter (i) To deliuer 0.70 c.c. of fuel at a pressure of 1s0 bar, how much d.isplacement
and stroke of the qlinder are mm and, 140 mm rcspectiietX, Thc ilumetrc fficiency is 80 per- the plunger
-100
cent. The condition of air at the beginning of compresslon are bar, 27" C. i undergoes ? Assunte a putnp n[et pressurL of I bar';
() Determine the marimum'angJlll-t of (ii) What s the effectiue strohe of the plunger if i* d.iameter is 7 mtn.
fuet tttat can be injecteil in ea.ch qlinder per second.
(i) If the speed of the engine is. 1500 r.p.m., injectinn presslre Assume coefficient of cornpressibility of oil as zg.g x 70- 6 per bor at atmospherr pre ssure.
is rsT bar, air pressure d.uring
fuel injection is 40 bor and fuel niection is carricd. out for 20" cronk'ang[e, d.etermine the Solution. Guen :Tbe volume of fuel in the pump barrel before commencement of the ef'fec-
diameter of the fuel orifrce assuming only one orifire is used. tivestroke=7c.c.
Tahe, pf = 860 kgl m3 ; Cf = O.eZ. . (fuorkee University) The diameter and length of the fuel line from pump to injector B mm, 700 mm,
=
Solution. Giuen : n = no -- 6 ; A / F ratio
= 20 ; d, = 100'mm = 0.1 m, I = 140 nm. = 0.14 m ; Volume offuel in the injection valve
= 2 c.c.
\,o.=80Vo ipo = 1ba, To=27 + 2?3 = B00K,N= 1500 r.p.m. ;0 = 20. Volume of fuel to be delivered
= 0.10 c.c.
crank angle ; p, = 956 kg/ms ; Cr= 0.67, L, = 150 40 110 bar. The pressure at which fuel to be delivereil, p,
. - = = 150 bar
ii) Amount of fuel injected into each cylinder per cycle : Atmospheric pressure, pc = 1 bar
Volume ofair supplied per cylinder per cycle Coeflicient of compressibility, q = 28. x 10- 6 per bar at atmpspheric pressure
= Stroke volume x q"". Diameter of plunger, do--7 mm
(i) Displacement of plunger:
= |4""4
a2 x r,o = I x (0.1)2 x 0.14 x 0.8 = 8.8 x l0{r3
"l Coeffi cient of compressibility of oil,
Mass ofthis air at suction conditions.
Clange in volume per unit volume
C"=
t" tt#tit#jo", Difference in pressure causing compression
^"= W= =t.o22xr.r kg/cycre
!v*t-v)
V1@1-
A/Fratio=n" 20 Total initial fuel volume,
P2)
mf I
Vt= Volume of fuel in barrel + volume of fuel in the delivery
= 5'1r x 10- 5 kg/cvcle line + volume offuel in the injection valve
ri." tur..n ro" *",,"r"*,,Jl"l n:*# t +f,<o.el" x70+2 = 13.95 c.c.
(s t 601 20 60 No pressure is built till the pump plunger
le* rvJ= aoo t
up- closes the inlet port. Further advance ofplunger.
= =ooozzz'
ro,o oil and aise thc pieisure to a required value. Once the delivcry prcssurc is
lJl.""1u1""1,the\uel
.'. Amount / mass of fuel injected into each qlinder per second, attarned, turther r\ovement ofplunger results in delivery offuel oil at constant pressure.
Change in v{ume due to compression
ritr = g*# =o.o2ske/s. (Ans.) or 78.8 x 10-6 x
= C"(pr_pr) x V,
(150 _ t) x 18.95
l(Vr-V2) =
(ii) Diameter of the fuel orifice, do : | = 0. r 6379 c.c.
The mass offuel injected into each cylinder per second, Total displafement of plunger
I = (Vr*V) + 0.1 = 0.16379 + 0.1 = 0.26379 c.c. (As.)
nf =Ao, Vtx Qf (ii) Effectfve stroke of the plunger, lo :

o,ox=Xdo' *c,ff ro i
j-
Xur'xl, -g.2637o or
! x@.7)2 xl, =9.26g79
.'. ,= *:## = 0.6854 cm or 6.854 mm. (Ans.)
438 INTERNAL COMBUSTION ENGINES F1JEL INJECTION SYSTEMS FOR C.I. ENCINES
439
Example l2-9. At iniection pressure of
yfray penetration of 22 cm in 16 milliseconds
745 bar,
is obtaned. Deternine the time rquired. for the sprayfo penetrate the same dstance at an injection oBJECTTVE TYPE QLIESTIONS
pressure of 235 bar. Assume the same orifice and the
$mbustionchamber density. Combustion chan-
ber pressure s 30 bar Fill in the Blanks or Say ,Tes" or,tlo'
(Jse the relation: s = @ 1. Fuel is injected into the cylinder at the end of...... stroke.
2. The period between the start ofinjection and start ofignition !s alled ...... delay.
where s is the penetretion in cm,
3, The beginning and end of the injection should be sharp.
t is the tme in millisecond,s, and 4. The injected fuel may not be broken into very fine droplek.
Ap is the pressure difference between the njection pressure and combustinn chomber 5. Air injection system is largely used these days.
pressure. 6. Inferior fuels can be wed in air injection system.
Solution. Giaen : pr= 145 bar, = 16 milllseconds,p, = 23O bar; 7. Solid injection is also called....., injection.
Pr = 30 bar; sr = s2= 22 cm 8. In......injectionsystemonlyonepumpissuicientformulti_cylinderengine.
Here, Ap, =pt-pqt= 145-30= 115bar 9. Very accurate design md workmmship are required in air iqjection system.
Lp2 = p2- p"= 235 - 30 = 205 bar t0' In ".." system, the fuel is metered at a central point ; a pump pressurises, meters the fuel and times the
injection.
From given relation s= I ./$, we have
1l' In...'..pumpinjectionsystemanindividualpumporpumpcylinderconnectsdirectlytoeachfuelnozzle.
r 12. A multi-hole nozzle gives poo automisation.
"t = l-4a
q hl!p2 13. A '..... nozzle is a type ofpintle nozzle which has an auxary hole dried in the nozzle
body.

- qs,tt''F
t'= .y1* Tffi-Rs I

- I x 16 milli-seconds' (Ans') l. compression 2. ignition 3. yes 4. No


= 11'98 5. No
J# 6. Yes ?. mechanical B. commonrail g. yes 10. distributor
11. individual 12. No lB, pintau.

I HTGHLTGHTS I

TIIEORETICAL QIIESTIONS
t. Main types of modem fuel iqiection system re :
(j) Common rail injection sJstem l. What ue the functional requirements of an injection systcn ?
(ii) Individual pup injection systen 2' How are injection systems classified ? Decribe tlem briefly. why the air iqjection
(iil) Distributor system. system is no used now-
adays ?
'Ihe main components of a fool injection system are : 3, what are the essential requirements to be furned by a fuel iajection system for c.I. engines
? what is
Fuel tank ; Fuel feed pump ; Fuel filters ; Injection pump ; Govemor ; FueI pipings and injectors. the most comnon injection system used in multi-cylinder esil engins ? "
The nozzle are classied s follws: 4, What are the main fmctiom of the nozzle ?
(i) Single hole nozzle (ii) Multihole nozzle D. What are the most comon types of nozzles used ?
(iii) Circumferential nozzle (iu) Pintle nozzle 6. What is the difference between air injection and solid idection ?
(u) Pintau nozzle. state the merits and demerits of multiple orifie nozzre md pinde trle nozzle in
c.I. engine iqiectors.
4. Volume of fuel injected per min., 8. Bring out the differences in constructio and working of printle and pintaux
nozzles with the help of
.. sketches and discuss their relative rnerits.
fn'
e,=LI% -J u, *f-9-r, iv'l) x r. 9.
{* l* solid iqiection system clasied ?_Explain the working of common mil, invitlual pump and
where, "^.l,. Ls6o
do = Diameter of fuel orilice, rn2,
fl distributor systems with the help of neat sketches. Discuss the; rebve merits and
aenerifu.
)o = Number ofoiifices,
UNSOLVED EXAMPITS
Vr= Velmity of flow of fuel through orifrce,
0 = Dwation of the injmtion in degrees of malk aagle,
ila
r,. l. diesel engine dwelops_ 89 kw at 2500 r.pm. Ih brake specifrc fuel consumption
N= r.p.m.,md f.!-w-ti{erj-trote
245 g&wh.
ig
calculat the quantity of fuel to be injected per cye per clinder. specificgrJ;y;iih;j
N = Number of injection per min. may be taken as 0.84.
= r.p.m = for 4-stroke engine
= r.p.m. for 2-stroke mgine.
f'
I
A six-cylinder, fou-stroke oil engine rtevelops 200 kw at 1200 r.p.m. and
mine the tliameter of a single orifice injetor irtrre i"ction pressue
tAn". 0.05;;:f
consurnes 0.3 kg/kwh. Deter-
*s is 200 bar ad combustion cham-

{
*
T
INTERNAL COMBUSTION ENGINES

ler nre.sule ls-ao bar. T)re injection is canied out for 30o rotation ofa amk. Each
nozzle on a cylinder
is provided with single orifrce.
Take p/.= 900 kg/i and Cr= 0.1.
lAns. 1.005 mml
9. A 4-stroke cycle c.I. engine d*ops 1b kw per cylinder at 2000
r.p.m. rh" sp"cific fo"l is
0.275 kgr'kwh tuer of B0'ApI. The injecrion pressure is rso bar "orsumption i'
roi zs. or;rt;;T;;p]""...u
the combution chamber is 40 bar. The eedcient of velo"ity

^sp gr' =
1{1.5
= .eZS *a ne specific gravity is given by

' Assuming fuel to be incompressible, detemine the diamete of the fuel orifice.
13
l3rs;;Ap
[Ans. 0.62 mm]
Ignition Systems (S.L Engines)
4. Aa eight'cylinder, four-stroke diesel eueie.l-ras a-power output
of 86g kw at g00 r.p.m- The fuel con_
sumption is 0.238 kg&wh. The p-ressue i the cyrinder 13.1. Introduction. 13.2. Requirements of an ignition system. 1a.3. Basic ignition ystems.
at te begianing of injection is gs ar m the
maximum cylinder pressue is 60.bar.The-injector is expected lattery (or coil) ignition system. 13.5. Magreto ignition system. t3.6. Firing order.
to be set at 210 ba and maximum ll._4.
pressue at tle injector is set at about 600 ba;. 13.7' Ignitiontining. 13.8. Spmkplugs. 13.9. Limitatiors ofnventional ignition. 13.10. Eiectronic
calcutate the orifice ir ,.
injection takes place over 12. crank mgle. ".eu ".qrir"Jp.r-i]Jl, ignition systems-Highlights-Objective $pe euestions-heoretical
euestions.
Aisume the following :
specifrc gravity ofthe fuel = 0.85 ; coeffrcient ofscharge
for the idector = 0.6 ; Atmospheric pessure
o"t ; The efrective pressure difrerence is the a-verage pressure difference r3.T. INTRODUCTION
i"l;3jl over the injection
o In S'I. engines the combustion process is initiated by a spark between the two elec-
[Ans. 0.01234 cmr] trodes ofspark plug. This occuts just before the end ofcompression stroke. ?he ignition
In a dieel fuel iniection oumo the_ vorume of the fueI in the pump banel before
the effective shck-e is z c.c. nrl the commencement of process must add necessary energy for starting and sustaining burning of the fuel till
net l;f;H;p;;;ioj"i" i" s .- in au*"te, ur-io
and the fuel in the injection valve is 3 c.c. -- ro,g combustion takes place.
() Determine the pump displacement
necessary to deliver 0.15 c.c. offuel at a pressure
o Ignition is only a pre-requisite of combustion. It does not influence the gross combustion
Assme a sump (atmospheric) pr*sure of I bar. of200 bar. process.It is only a small scale phenomenon taking place within a specified small zone
(ii) calolate the efrective stroke of the pr'nger if
the diameter of the plunger is g mm.
in the combustion chamber.
Take the oefficient of compressibility of oil as ?g.g x 10-6 per
bar at atmospheric pressure.
o Ignition only ensures initiation of combustion process and has no degree intensivelr or
extensively.
lAne. (i) 0.384 c.c. ; (ii) 7.65 mml
A spray penetration of l0 cm is obtained in 15 milli-seconds when injection pressure was 180 bar. Find Enerry requirements for ignition :
the time required to penetr{te the same clistmce ifthe ir..tioi
pressure in the combustion chamber nnder both conditlons
pr"."r." is increased to 200 bar. The Asparkenergybelow10millijoulesisadequatetoinitiatecombustionforA/Frat,jo
is 40 bar. ssume the dereity offuel and air
in cylinder remain same during injection under both conclitions. - 12-13 : I (Range ofmixtures normally used) ; the duration offew micro-seconds is suffi-
IAns. 12.4 mii_secondsl
cient to start combustion.
A spark can be struch between the gap in the two elecrodes of the spark plug by
- sufficiently high voltage. There is a critical voltag e called,breakd.oun uoltagi below which
no sparking would occur. In practice the pressure, temperature and density have a
profound influence on the voltage required to cause the spark. Also, the striking voltagc
is increased due to the fouling factor ofthe electrodes owing to deposits and abrasion.
Forautomotiveengines,innormalpractice,thesparkenergytothetuneof40milliioules
- and duration ofabout 0.5 millisecond is suflicient ore" enti.u range ofoperation.
13.2. REQUIREMEVTS OFAN IGNITION SYSTEM
For an ignition system to be acceptable it must be moderately prieed, reliable anrl its pcr-
-
forrnance must be adequate to meet all the demands imposed on it by varous operating cond.iticn.s
An ignition system should /ulft.ll the foltowing requirements :
1. It should have an adequate reserve of secondary voltage and ignition energy over thc
entire operating speed range ofthe engine.
2. It should consume the minimum of power and convert it efhciently to a high-energy spark
across the spark-plug electrode gap.
3. It shoulrltr-=---ve
a spa.Ti"ition-*ich is sufficient to establish burning of the air-fur:i
m.ixture under all operating conditions.
-\
441
442 TNTERNAL coMBusrroN ENctNEs
4. It should have the ability to produce an ignition spark when a shunt (short) TGNITION SYSTEMS (S.t. ENCTNES)
is established I
443
ove the spark-plug electrode insulato surface, due posslly b carbon, oit
ori". "posits, liquid A ballast resisror is provided in series with the primary winding to regulate
fuel or water condensation.
- current' For starting purposes this resistor is by passed so that, irore
primary
5, Good performance at high speeds. "o no*
6. Longer life ofbreaker points and spark plug.
in the primary circuit. ",ioent
7. Good starting when the breaker points open slowly cranking speed. Breaker poinls
8. Good reproducibility ofsecondary voltage rise rise.
9. Adjustment ofspark advance with speed and
o The basic source ofelectrical energy is either , a genemtor or a rnagneto. Spark plugs
The battery and generator normally 6 V or 12 V direct current, while the
magreto provides an alternating cu, of higher voltage,
The low voltage (6 V or 12 V) j to a very high potential ofabout 10 kV to
20 kv' in order to overcome the spark gap resistance and to release enough
energy
to initiate self propagating flame from within the combustible mixture.

13.3. BASIC IGNITION SYSTEMS l

The following basic ignition systems are in use : I

l. Battery ignition system-Conventional, transistor assisted.


2. Magneto ignition systern-Low tension, high tension.
3. Electronic ignition system.
The difference betwc,en Battery and Magneto ignition systems lies only in the source Secondary
of
electrical energy. Whereas ,Battery jgnition systm, usei a battery, .Magneto igiition winding
system, uses
a magneto to supply low voltage, all other system components being siilar. -

13.4. BATTERY (OR COIL) IGMTION SYSTEM lgnition swtch


It is a commonly used system because of its combined cheapness, conuenince of mainte-
nance, attenti,on and. general suitabIity.
Primary winding
Construction. This system consists of the followrng componenrs Ground
:
1. Battery (6 or 12 volts) Fig. l3.l, Battery or coil ignition system.
2. Ignition switch
3. Induction coil 4. Circuit/contact breaker \trorking:
5. Condenser 6. Distributor. To start with, the igrrition switch is made o and the engine is cranked
Refer Fig. 13.1. - hand when the contacts touch, the current flows from battery through
i.e. turned by
trru ,*iict, p"r-
one terminal ofthe battery is ground to the frame of the engine, and other is connected mary winding of the induction coil to circuit breaker points an ttre cicuit
- i. ."-pr"t"
through the ignition switch to one primary terminal ofthe igniiion coil (consisting of a through the ground. A condenser connected across the terminals ofthe contact
points prevent the sparking at these points.
bieaker
comparatively few turns of thi.ch ir wound round an on core). The other primaty
terminal is cnnected to one end ofthe contact points ofthe circuit breaker and ihrough The rotating cam breaks open the contacts immediately and breaking
closed points to ground. The primary circuit f the ignition coil thus gets completd - circuit brings about a change ofmagnetic fielcr due to which a very hfih
of this primary
when contact points ofthe circuit breaker ae togethernd switch id closJd. The second- ;
tune of 8000 to 12000 V is produced u."o.. th" secondary termin;rs.-oh;-riiiu. io t"
"oJl"J" or
ary terminal ofthe coil is connected'to the central contact ofthe distributor and hence turns-in the secondary winding may be 50 to 100 times than in p-""y
to distributoi roto. The secondary circuit consists ofsecondary winding (consisting of a *irr-i. o"
to high voltage the sparh jumps across the gap in the spark prug arrd
large numberof turns of fine wire).ofthe coil, distributor.trd four siark plugs. rne air fuer -iiu"e is
ignited in the cylinder.
contact beaker is driven by a cam whose speed is harfrhe engine spee r Fig. 13.2 shows the gradually building up of the primary curent from the
$orioui strohe
en_gines) andbreaks the primary circuit one for each cliniter urin! one complete cycle points close until they open.
ofthe engine. r' Fig' 13.3 shows a typical wave-form or pattern oftrte normar gnition action.
The breaker points are held on contact by a spring except when forced apart by lobs of
- the cam. - A
point L the distributor opens and the magnetic field
of the coil-primary winding
collapses nd consequently the secondary .,roltug., indicated
y tiru ung i"u,
to point_M. The height of the firing line shows th voltage neeied
tr j#t;;;;;". "]s",
gap and to ionize the gap between the spark plug electrodes.
ner the i.
""".t
444
iNAL COMBUSTION ENGINES r(iNlTIoN SYSTEMS (S.1. ENCTNES)
45
initiated the gap becomes ionized resulting in decreased gap resistance and a
smalle Description of Components of Battery lgnition System
voltage is then required to maintain the arc across the!p. The rower
voltage and
the spark duration is represented by the height and lenlti ofthe spark rine"Np. 1 The battery:
Spark
Tlre function of battery is to store electricity in the form of chemical energy, when
re-
lne quired to convert the latter back into electrical energy.
Ml o Moto vehicles use lead-acid batteries which have a series ofpositive and negative plates
^
I n,ine-| | i j*n* aointsi which ae interpersed, the plates being immesed in a sorution o(dilute suiphuric
acid,
rn opeti
o
tf )*tr i1*" called the electrolyte. For compactness the plates are praced crosJtogether and separa-
tors are used to reduce the chance ofshorting taking place.
-
6
ii" ii rlll
" Ad.uantages of 12-V ignition system over 6-V sXstem:
'-
E
rlrt The following are the advantages of l2-V ignition system over 6-V system :
L rlrr
r!
r! 1. Considerably higher voltages ae obtainable.
2 For transmitting equal power with excessive voltage drop, the cable in a 6-v system
Dwell necds theoretically to be four times the thickness of12-V ystem, cables.
Build up time
seclion 3. Improved starting.
Fig. 13.2. Built up time for primary cunent. Fig. 13.3. Tlpical pattem ofnormal ignition action. 4' Adequate electric power to supply the increasing nurnber ofelectrical accessories
used.
At point P the major portion ofthe energy ofthe coil is expended and consequently 2. The ignition coil :
there is a drop in the secondary vortage which result in eitinguishing ofthe-spark.
o To create an adequate spark across the gap of sparking plug high electrical pressure
is
Due to spark extinction-the circuit becomes open, the current flow is stopped, needed. Electrical pressure is measured in volti and trre rI
and, suppried iy the bat-
hence the magnetic field (produced in the secondary winding, during'tile firing tery is totally inadequate. "Jts
period NP while the-.current was flowing in the secondary wining
ani across the
spark gap to ground) collapses, thereby, inducing a curent in th primary wind-
ing, which eventually flows into the condenser an charge it. H9h tenson lead
to dstrbutor cap
when voltage in the codenser becomes higher than that in the primary winding,
it discharges back in-tho primary winding. This resurts in coilapsing oith" *.g-
netic field and rebuilding up ofvoltage in the secondary winding. ThiJpulsing
back
and forth, weaked each time, continues tiil whole ofthe energyls dissipated
iRefer
Fig. l3.3-intermediate section). .
At point Q the contact points close and emain so during d,re Il period.At the end of
this period the points again open at s (there being no condenslr action during the
period, since it is shorted out across the closed ooints.)
Advantages:
1. It offers better sparks at low speeds, starting and for cranking purposes.
2. The initial cost ofthe system is low.
3. It is a reliable system and periodical maintenance required is negligible except
for bartery. Primaty
Secondary
4 Items requiring attention can be easily located in more accessible position than tbose of winding
wndng
magnetos.
5. The high speed engine drive is usually simpler than magneto drive.
6. Adjustment of spark timing has no deteimental eflect over the complete ignition
trming
range.
Disadvantages:
1. With the increasing speed, sparking voltage drops.
2. Battery, the only unreliable component of the system needs regular attention. In
cse
battery runs down, the engine cannot be started as induction foil, to oporate. Laminated
3. Because ofbattery, bulk ofthe system is high. "oi-l soft ron core

I ic. 1:l ,t. The irdr:r'


r--
I

IGNITION SYSTEMS (S.I.ENGINES) 44'I j

446 INTERNAL COMBUSTION ENGINES


A small cam, with a lobe for each time the breaker is required to open per revolution,
o The function of ignition coil s to increase the uoltage between 1O,000 and 75,000 uolts in revolves and pushes the movable breaker arm so that the contact points are separated.
some condtions, although the voltage which occurs under normal running conditions is In this way, the current in the prirnary winding of the coil is interupted every time a
of the order spark is required in one of the cylinders.
a Two coils of insulated wire arelouq{ or1 a laminated soft iron core. The inner coil, The usual circuit breaker used on modern vehicles is the closed-circuit type (Fig. 18.6).
called the scodary, has more turns than the outer primary coil. There ae about 20000 The contact points norrnally remain in the closed position, being separated only u.then
turns on the secondary and 400 turns on the primary. the breaker arm is lifted by a lobe of the breaher arm. The closed circuit type is more
o If a low voltage passing through the primary coil is switched off a higher voltage is adaptable to high speed engines because the points are in contact long enough to allow
induced in the secondary coil, the ncrease being approimately in the same proporton complete magnetisation of high tension coil. This results in especially good sparks at
to the number of turns of the two coils. The core ad winngs are placed in an iron slow starting speeds, with less intense sparks at higher speeds when the time of the
sheath. The entire assembly being housed in a sealed container Gig. 13.4). contact is shortened.
o A high tension lead from the centre ofthe coil caries the supply to the distributor. TVo
small teminals are situated eithe side of the high tension lead, one being connected to
the contact breaker and marked CB and the other to the igaition switch identifred by
the letters SW.
3. Contact breakers :
o The distibutor unit used on the modern motor vehicle contains the breaker contacts
which make and break the primary current, and the ilistributor mechanism which sup-
plies the secondary current to the plugs in their pmper firing order. Fig. 13.5 shows how
the distributor is connected into the typical battery igrrition system.

lnsulated brrinal
(connectsd lo lever arm)

Fig. 13.6. Closed circuit type cirrrit breker.


4. The distribuor:
o In order for the coil to produce a high voltage from a low voltage supply the flow of
electricity must be interrupted (switched offand on). The distributor contains the con-
tact breaker and a cam, which is of rotary switch, that intenupts the supply. The dis-
tributor caphas a centre terminal which connects with the high tension.terminal of the
induction coil, and as many terminals equally apaced around it as there are spark plugs
to fire. The cap is usually moulded ofa highly resistant insulating maeri, such s
bakelite or condensite which is rnoisture proofand possesses high insulating properties
even under excessive heat. Tle terminals ar ofbraes or metal alloy moulded in posi-
Fig. 13.5. Contact breakm. tion, terminating on the underside eier in the form of a button flush with the surface
or i the form of a pin. The distributor head is usually held in place by two spring clips
Breaher contoct points must permit the spark plugs to fire accurately at high speed. which snap on only when the head is in its proper position. Thus the head can be easiy
Therefore the moving parts must be light, yet stroDg, and carefully buil ofhigh-grade removed to inspect and adjust the rotor and breaker rechanism with no chanie of rJ-
materials. The breaker contact points are two small contact pieces one stationary and placing it incorrectly. The roor or distributor arm is mounted on the upper end of the
one on a rnbvabl arm, normally held against the stationary contact by spring tension. distributor shaft, on which the breaker cam is also locpted. The inside end of rotor
The points are made oftungsten or platinum alloy to resist burning and pitting, and are makes contact with the centre terminal ofthe head, while the outside eitd in its rotation
hard enough to withstand the hammering acion caused by the rapid closing of the completes the circuit successively with the terminals leading to the spark plugs.
breaker at high speeds.
INTERNAL COMBUSTON ENGINFS
The distributor may be considered as a rcNrrroN sysrEMs (s.r.ENcrNEs)
revolving switch located in the secondary cir_ {49
connects the high_tension,"i"" Ao,o-"?il
;}t;."OaO b the proper spark plug at proper (lu) Check that if the carbon brush is clean
and moves freely in its holder.
o The distributor rotor must be we' insulated (u) Burnt or blackened contact breaker points
to prevent should be cleaned with a thin caborudum
stone' Care musi be taken that these surfaces ;".k;pt
curent, consequently it is usuaily mourded ,grounding of the high-tension
from'an rnsuratlng flat; ensuring that the entire aea ofthe
used in the distributor cap. material similar to that points is in contact. Remove grease or metaric
dust with a petrol :noiitened croth.
r It is designed to frt over the end ofthe Iiner shaft (ui) Breaker gaps should be periodically checked,
and if necessary reset.
in one position onry, to prevent its If the engine does not fre disconect the centre high tension lead ancl hold
. installation in a position which would trr"o* ii oof or time with the breaker. .. (ui)head' close to the
o The distributor may be of^ga! type or contact type "{lt:d-*
Arrange the cam so that the points are closea]"..r, li-"11{f"irt"
separated,
used in compriting the ; classified according
to the method with the igaition switch on, a good spark should be obt.ined. "r" since the
circt between the rot#and the distributor Thls is only alrough check
head terminals. spark does not occur under the conditions preventiag in
5. The condenser: the cylinder.
condenser. The following symptoms may indicate a faulty condenser arthoufh
o when the primary circuit is broken the current usually checked by special equipment ,' ' they are
jumps across the separated contact tendsto continue flowing and a spark
breakei po,.i.. tr" effect is charaJteristic (i) Sevee misfiring causing explosion in the
inductive circuit and is undesirable o" tfre of an silencer and carburettor.
ioliowj red.sons (ii) Engine refuses to perform satisfactorily
(i) It causes arcng, burning ofthe contact breaker i
(iii) fu'si.t at contact breaker points causing
under load.
points. brackened and oitted surfaces.
Qil The effectiueness of t\ is red.uced. since a s'udd.en inte*uption (iu) Weak spark.
9o!r
essential to produce a high of the crcuit is
uoltage. Check connections and iffault still occurs replace
o The condenser absorbs and_stores this inductive the condenser.
in the coil to die awav rapidrv wbich r".i""r"l
flow ofcurrent and causes the current High tension cables. Check that connections sre clean and secure, and
irt. ltage in the secondary coil. insulation is free that the cable
l from cracks.
A condenser is constructed strips ofsheets
oftin foil insulated by thin sheets ofparaffined Plugs. (i) Inspect and clean every 8000-4800 km and
paper or mica' The arternate layers replace every 16000 km. when setting
of tinfoil are conn-ected in paraller, foiming the gap adjust the side electrode and ceck with
groups, each group provided with-
a terminal ro" exler.ral connection. Roleir
rwo ;;i;;;""g" for preference.,
cal condensers are made by winding or cvrindri- (ll) It is not satisfactory to check the action
of a plug by lying it on the cylinder head and
into a tight rol (Fig. 18.?). "it".""i"1.y""s ofthe tin foil and the insuration rotating the engine. This is because it is not ."bu.t"ai
*_Urrj .oiditioor.
(ili) Plugs should be checked using a special
machine
------' which stimulates cylinder conditions
'fin fol and automatically gives the condition,fru pl"gr.

r3.5. MAGNETO IGNTTION SYSTtsM


o The magneto ignition.system is^similar in principle
to the battery systern except that
the magnetic field in the co119f tlr-e grimai and-seconda"y
wirrdings i" produced by a
rotang permanent magnet (Fig. 13.8). As te magnet
turns, the field is iroduced from
a positive maximum to a negative maximum and back
again. As this magnetic ficld
falls from a positive'aximurn value, a vortage and curenr
are induced i., th" primury
winding. The primly current produces a ragnetic freld
of its own which keeps the
total magnetic field surrounding the primary-and secondary
Fig. lJ.Z. The condenser. constant When the permanent magrret has turned for enoug
*i"ai"g;-";p-ximately
ro tlr"i ii" to
he total field is strongly negative, ihe breaker points
Maintenance of ignition system : are opened and the "o.rt"iotio.,
magnetic fierd
about the secondary winding s'ddenry goes from a high positive
Induction coil. 0) It is only necessary to keep tive value. This induces a gh voltale"in the secondy windingvalue to a high nega-
the connections clean and tight.
proper spark plug by the disributor,
which is led to the
(ii) Never leave the iqaition switched
on without .he
points hav'e come to est nIr," engine running. Ifthe contact breake o The magneto is an efficient, r,elable, self contoined, unit
ctosea fosit#;ffh overheat and burn out. craft engines because storage batteries aie heavy and
which is often preferred for air-
Distributor' (l) Prode light gease or clean troublsomr. Specit means
engine oil on the cam, but make sure it are required, however,. as the magrreto wiil not "i""ng
not get into the contact faces ofth--e.ol"t".t does furnish enough r". ignition at
r""i."rl.-"" low speeds. variation in ignition timing is more difficult "ilt"g"
*it tt";;g";1", since the
(il) wipe inside ofdistributor cap breaker point must- be opened when th*e rotating mag.nets
and the electrodes with a crean cloth. are in the most favourable
position' It is possible to.change the engine
the r'otor arm and injec light ensine
oil into the ,oto, urigl" * *i.rr tii.--lg"eto points
iil ,0" ."?"liil?]e spindte to lubricare
'"- "r"kpoinlopening
open without disturbing the re-lationshipietween
,oug,'ui position by
designing the attachment pad-so that ir,"\u magneto body"oa
ii'
lir degrees about its own shafi. obviously trris me}oo.iJ
may be rotated a few
nor., itlri*rory s rototing a
timer cam-plate.
\
INTERNAL COMBUSTION ENGINES
ICNITION SYSTEMS (s.I.ENcINEs)
Sparking plugs 451
to limit the secondary voltage to about
400 v and. the
-- *"'
distributor is replaced by a brush contact.
step-up transforme is used o get high A
voltage.
,.--'{J>--rr' Comparison Between Battery Ignition
System and Magneto Ignition System
o In a battery ignition system a 6- 12 V battery
is used. to prouide primary uoltage,
separate 'gntion co, is required to
boost ip ins
an. a
uoltage nied.ed to'op"rot
ti" ,parh prug.
o Magneto is aspec,ial.type of ignition system
necessa'ry ener*t for the system.It
with its own erectric generator to prouid.e the
is mounte on the.engine
nents of batteryignition system excepl
tr" ""'""pi"..1"n the compo-
pf"glr"_"g.-"ili"n,ot
"o""t t"a uy
::ffi:Til#"3t"?ffi'*""nt *'i.viig"i*tt'g"od a"". i"i"".al battery as

between the battery ignition slstem and. magneto gnition sysrems


,n" *#fi"ff:nces ae given in

Current for Obtained from battery.


primary circuit Generated by the magneto.

Starting Difficult to strt when batterv


No problem ofbatery discharge.
is iu discharged condition.
Maintenance More, due to battery.
problems I*ss, due to absence of battery.

Intensity of sparh C.ood


dt low speed Poor

l=,,"^ swtch
Efficiency Efficiency of t}le sye tem dccreases
mttr the eduction in spark inten_
Efciency ofthe syetem improves
Condenser

I
as the engine speed inmeasls
sty as tjle e4gn speed increases. due
Space occupicd
to high intensity sprk
More
lss
Fig. 13.8. Magneto ignition system. fuplication In cars and gt
In mcing cars and two
commercial vehides.
Advantages: wheelers, aircafts etc.

1. The systen is more reliable as there is no battery


or connecting cable. ' l,'5"1i;3tililg"J$*:: beween't'" r.*tffi
2, The system is more suitable for medium and
very high speed
3. with use of cobart steer and nicker-aruminium "rrgio"r.
magnet metals very ght and compact a
units can be made which require very little room. I

4. With recent development this sy"te* h* I

Disdvanages:
become fairly reliable. -
gE
1, At low speeds and during cranking the voltage is very
low. Thie has been overcome by o
suitable modifications in the
circuil. t
2' Adjustmqnt f the spark timing i.e. advance o
or retard, has deterimental effect upon the o
spark vollage or energ.y. 6
3' The powerful sparks at high engine speeds cause E
o
burning ofthe electrodes.
Low tension magneto ignition system :
Spad< per minute
In a high tension magneto ignition system the main -->
shortcoming is that he wirings carry a Fig. 13.8(). Breaker
very high voltage current which may lead rJ flnent
rz i*rlra'litn, due to leakage. This trouble can
us speed for battery_coil anil
nagneto
ignition systems.
be avoided by using hw tenson magneto system. "rgine
rn ihis system the secondarl winding is changed Itcan be observed that at.start, in
case ofmagneto ignition system, the
generated is very low since the
sta"ti"g ..";ki;;"r;;;d is row. The current
currert
increases
452 TNTERNAL coMBUsTIoN BNcNEs
IGNTTTON SYSTEMS (s.r. ENCTNES)
with increase in.tlge.nging speed. Owing to the poor starting s:
chaacteristics and
effect ofthespark timing changes on vortage gene"atea, i"varLriy Most ofthe engines are fitted with a mechanism
rattery ignition which is integral with the distributor and auto-
system is preferied to the magneto ignitton sysrem. - regulates.the optimum rp."k .aurrr"" t"
Ifowever, theattery ignition slstetnis heauier an.L requires ir.Tll"
towtlrg two mechanisns ae used : """"""t
for change of load and speed. The fol-
more maintetwcethan
nagneto ignitian slstem. 1. Vacuum spark advance.
2. Centrifugal spark advance.
13.6. FIRING ORDER
1. Vacuum spark advance :
Firing order is the ord.er in.uhic,h uorious cylinders of a murti-c'tindcr engiE fuc. The
firing order isarrangedtohauepowerimpulses equ"uypir, and.fromthepointofuiew o It is necessarJ to have,vacuum advance.of
the spark timirg sirle the rean mixtures
offurancing. require an.earrier spark_timingthan the rich
Firing orders for vaious engines are given below ; mixtue. Thus with the closure of the
throttle, the spark must be uar""a to-g"fiptrmum
performance.
No. of <:ylinders Firing order . advance is obrained with the help of a spring loaded diaphragm connected
T\vo
;l"r:T:lH
t,2 so rong as the engine idles no advance is
Three 1,3,2 required. But when the throt'e is opened
Four
vacuum *."91:*, causing the diaphragm to move inwards. The diaphragm s so
1, 4, 3, 2 or !,3, 4, 2 coupled with the contact breaker that
wien induction pressure is reduced, the an-
Six 1, 5, 3, 6, 2, 4 gle of advance increases.
or !, 4,2, 6, 5,5
Eight l, 6,.2, 5, 8, 3, 7 , 4 or 1, 8, ?, g, 6, 5, 4,2. This type of mechanism is strictly an economnt
d,euice (whencar is driven properly, in-
Eight (Vee) : Either of the following alternatives. creased fuel economy is obtainedi.
(A) tL, tR, 4L, 4n,2,R,5L,3R,2L 2. Centrifugal spark advance :
(B) 7L, 4R,2R,2L,SR,W, 4L, tn t spark advance is essential to compensate for
(c) LL, 3L,2R, 4R,Sn,2L, 4L, lR ill";?::i*l the increase in speed of
(D) 7L, 4R, 4L,21.,3R,3L,2N, lR o This mechanism is obtained by connecting
the contact breaker cam to he driving shaft
(E) 7L, 3L,3R,2L,?,R,rR, 4L, 4R through a coupling which incorporates ttri
.L"t.lruc"r weights. wiirri""."r." in engine
speed, the weights fly .t causling the
z and 'R indicate cvlinder on left and ght hand side respectively. cam io turn relative to the driving shaft in the
The firing order for a four- direction of rotation and eventuafry ff."gip*L
strofe gnein-e yith its cvnder nrmb"""d ;";;;ttt iy-trom r to wiil be 1, B, 5, ?, advance.
revolution ofthe crankshaft and2,4,6, g, ...... to ( _ 1) ''' ...... to n for one
for the next r3.8. SPARK PLUGS
""rrolrriiorr.'
r3.7, IGMTION TIMING o The main function-o/a spark prug is
to conduct the high potentiar frorn the igniton
system into the combustion chambir. It prooides
o Ignition tining is-the co-rrect instant for the introduction of sparh near
the end, of com_ produced. by apprying high vortage,
tn" proi""'iop "r"'"""*nun sparh is
pression stroke in the cycre. The igniton timing to s;k;;e combuston chanber.
is fired. to oitn nta*mum power r A spark plug entails the following re qurements
the engine. from :

o The correct instant for the introduction of a spark is mainly


(l) It must withstand peak pressures
determined by the ignition up to atleast 55 bar.
lag. (li) It must provide suitable insulation
between two electrocles to prevent short circuiting.
Some of the important factors which affect ignition (iii) It must be capable ofwithstanding
timings are : high temperatures to the tune of2000.c to 2500.C
(j) Compression ratio over long periods of operation.
(ii) Engine speed (iu) It must offer maximum esistance to erosion burning away ofthe
spective ofthe nature offuel used. spark points irre_
(iii) I4*ot" strength
(u) It must possess a high
(du) Csmbr.,i.r chamber desigrr heat resistance so that the,electrodes do not
cientry hot to cause the preignition ofthe become suffi-
(u) Throttle opening charge wi(hin the ..rgirru
(ui) The insulating material must
(ui) Engine temperature withstand sa1 rhe chemical"ylirr"..
eacrion effects
of rhe tuel t ot prou"t;;-f ;";;J;;tisfactorf
(uii) The tYpe of fuel. "ni
(uii) Gas tightjoints between the
insulator and metal p..\. :--.-"*- under
*." all oper-
Spark Advance Mechanisms : ating conditions. - u."
_-- essential
I
It is of significant importance that the point in the cycle where the spark The spark plug (Fig' 13.9) consists ofa meta-l
shell
regulated automatically to ensure maximum power occurs must be made of nicket alloy with alloying elemenrs 4aving two electodes (oftenly
and economy at different loads and speeds. of rungsr{n which
are insulated from each othei wrth an air gap.
Hilrr "t;;;.;;;;;um)
--- r."r"
the supply electrode produces the necessaiy
spu.t f"..i"l"r."u"il"-ping
/
IGNITION SYSTEMS (S.I.ENGINES)

(u) Insulator's leakage resistance, The carbon and metallic oxides from electrically-con-
ductive coating on the insulator which, thus, shunt the secondary winding and educe the maxi-
Terminal nut mum voltage that the secondary can impress across the spark plug.
(ui) Voltage increase rate t the gap. When the voltage is built up at a rapid rate by the
ignition system, the effect of leknge gets minimized, and greater voltage is available.
(udi) Fuel-air ratio. The fuel-air ratio fixes the electrical properties of the mixture ; lean
mixtures have higher breakdown voltage than those of slightly rich mixtures.
Tsrminal stud
(uiii) Electrode rnaterial.
(*) The presence of ionieed gases in the gap.
Insulator (Ceramic)
Differenc between hot qrark plug and cold spark plug:
Sealing compound

Special
Rusl rBsslnt shell

Capth/ gaskst

T- "*;
Roacir elecltode
lntomalseal
(Gaskst)
t
lnsulatof riosg
(Cor6 nose)
Ground
Sperk gEp spacirig
elclrode

Fig. 13.9. Spark plug.


Plugs are sometimes identified by the heat range or the relative temperature ob-
tained during operation. The correct type ofplugwith correct width ofgap between i
the electrodes are important factors,
The spark plug gap can be easily checked by means ofa feeler gauge and set as per
manufacturer's specifications.
Fig..13.10. Hot and cold spark plug.
It is most important that while adiusting the spark plug it is the outer earthed
electrode i.e., tip which is moed in or out gradually for proper setting ofhe gap. The difference between hot ad cold spark plugs is due to the relaive operating temperature
No bending force should be aiplied on the centre-electrode for adjusting the gap as range of the tip of the high tensianelatrdes.The operating temperature is governed by the amount
this can cause crack and fracture ofinsulation and render the plug absolutely useless. ofheat transferred which depends upon the heat ransfer path from the tip to the clinder head and
tlre amount ofsurface area exposed to the combustion ga6es, A cold plug cs o short heat transfer
Porcelain is commonly used as insulating material in spark plugs, as it is cheap
poth and, a small drea exposed.to tle ambustinn gases, as cotnpared, fo c hot plug,
and easy to manufacture. Mica can also be useil as insulating material for spark
plugs. Mica, however, cannot withstad high temperatures successfully.
1S.9. I.TWTATIONS OF CONVEI*TIONAL IGNITION
Factors afrecting esablishment of arc across the ai ga.B of the spark plug :
The conventional ignition systcn entails the following limtatians :
The probler.n of establishing an arc acnoss the air gap of the spark plug is affected by the 1. Frequent servicing and replacement ofcontact points;
following factors:\
2. Weaker sparks (or sornetimes rnisfrring).
(i) Air gap length. Greater the air gap, greater is the breakdown voltage.
3. Ineflicient at low speeds.
(di g.n geornetrr. For pointed small electrodes less breakdown voltage is required.
4. Poor starting ability.
(iii) Mixture density. High densities (i.e. high throttle openings) require higher breakdown
voltages. It follows the Paschen's law ; V = Kpl, where I! p and h are a constant (for given 5' Reduced ability to fire fouleil plugs (since most of the spark energy is lost as current
substance and electrode shape), density ofthe material and gap distance respectively. flows through low-resistance,fouling deposits).
(iu) Electrode ernperature. High temperatures lead to low breakdown voltage. 6. Poor reproducibility ofthe secondary voltage rise, peak voltage and frring time (owing to
inherent erratic operation of the nechanical points).
456
INTERNAL COMBUSTTON ENCINES ICNTTION SYSTEMS (S.r. ENGINES)
I3.1O. ELECTRONIC IGNTTTON \457
SYSTEMS
The. Iimitations / disadvantages 3. Reduced naintenance of the ignition system.
^c_ ofconventional ie 4, Increased reliabiliry,
of new igrition svrt"-
using solid. stateerr"""-.T:tt:1,tft"1,h* led to the development
"oilpts
types of-electronic ignition systems *"
tn modern automobiles the following
two 5. Increased spark plug life.
"rpl"i"Ji"uco'
1. Tlansistorised coil ignition (TCti
sy"terr. 2. Capaciive dischatge ignition (CDI) system:
2. Capacitive discharge ignition (COlj
systm. Fig. 13.12 illustrates the capacitive discharge ignition system.
These systems ae described
below :
1. Transistorised coil ignition (TCI) Chargns
systen : dvico
Ttigge/box
'-'t'- -rz:W
Ensrgy
higher output voltage and uses
:::tY"tC*tilru:des erecrronic rriggering to keep
the
a It is categorised as high t . .:,'{i
-
ener5iy electronic igrrition i"i
a The circuit diagram of this system.

system is showiin fg, I


i.il.
lgniton
SYitchlng tansformer
oubut hduction

'1n" r"iH':T 'ng'li"

To nduction
tlpo pulse
Battery * gentalor
To dlstributr
T I
I

=
Fig, f3.12. Circuit diagram of CDI system.
o In this system a capacitor
ie uged to store the energy, iniluction coil is o used. The
maglitude ofstored energy depends on the capacitance and the charging voltage of the
capacitor.
r The primary voltoge developed at the time of spark by the discharge of
capacitor is
Pulse shaping boosted up by the ignition transformer, through th thstor,
Spark ptugs
to"the high voltage
clrcult required at he spark plug.
Fig. 13.f1. Circuit agram ofTCI
The cDI trigger box comprises rhe capacitor, thyristor power switch,
charging device
systern. to convert battery voltage to the charging voliage of 800 to 500 V, by'means
oi pulses
uia,the voltage transformer, pulse shaping unit and control unit.
tyi:x"il"rf,#::fi;:;::;,::::,,#Tz!4"luu""aeuhesenerator)q,stem
etectronicro,i,ot^od"Lrd;il#'.:L::i:::"#Lii:::;::!:rTri:: Advantages:
assembly of the convention"l Tlre aduantages of CDI sXstem are
The nodure switches,offtle
ic"ition .yj"i]" :

n* 1' The capacitor in this system can store several thousands times
a high voltage in the second- . more energy per unit of
"r*"*rir"d.ucing compared to that per unit inductance of the conventionar rcl
system. This permits a
;:'l#iiH":ll,'.n ,i,f:*"*"d;;;;;;k;rugs in rhJsam" r"'"r,io,,'u-"'in.o,,- i?f"]::T:
nrgh ourput vottage even at bigh spark rates (speeds).
Timing circui is eontained in the 2' Due to the internal resistance being very small (about 50
Iater
control model that closes the primary circuit e), faster voltage rise is obtain-
so that the primary current able and consequently the system is reluivJty inensitiue
build-up-c-ai-l..rr" fo" the next cycre. to sid.e tracking.
Ad.uantages: 3. As in TCI system, the breaker points sewe as a trigger
only. This avoids frequent mainte_
nance and increases life ofthe contact points.
The advantages ofTCI system f
are : 4' As edent om Fig. 18.13, at low engine speeds in cDI sysfems
1. Reduced wear ofthe components. low current is drawn
fiom the battery and with increase in speed the c-urr""i;;;;.r*l;;;6;,il#whar
2. No problem faced in burning inonventional or TCl_system where igh current is diawn at happens
Iean mixtures. low r/"eas.'rrri, .aracteristic of
wur syslem Lncred.ses the low speed efficiency and, allows
easer cold stfrtin7,

I
458 IGNITION SYSTEMS (S.I. ENGNES) 459

2. The main comporenls of a battery ignition systen are :

(i) Battery (6 o 12 V) (ii) Igition switch


+ (jii) Induction coil (iu) Cirruilcontact breaker
I
I
(u) Coadenser (ud) Distributor,
t2 3. Thebattzry ignition syslem is ommonly used beeuse ofits combined dreapness, convenience ofnainte-
G nance, attention and general suitability.
gE 'fl\e magneto is an efFcient, reliable, self contained unit, which is often preferred for aircraft engines
because storage batteries are heavy and troublesome.
;g o. Firing order is the orde in which various cylinders of a multi-cylinder engine fire. Tlie ring order is
!
c'
ananged to have power impulse equally spaced, and from the point ofview ofbalancing.
o 6. Ignitian timing is the corect instant for the introduction of spark near the end of compression stroke in

the cycle. lhe ignition tining is fixed to obtain maximm power frorn the engine.
The nain function of a sparh phtg is to conduct the high potmtial from the iguition system into ttre
u rooo 2O0O 3000 .1000 5000 6000
combution chamber. It proda the proper gap across rilhich spak is produced by applying high voltage
to ignite the ombution chmbe.
Engne 6pood, r.p.m. -+
A cold sparh plug has a short heat transfer path md a small area expqsed to the combustion gases, as
Fig.13.r3 compmd to a hot Bpark plug.
9. In modem automobiles, the following two tpes ofelectronic ignition system are employed :
5. The output voltage is relatively independent ofengine speed (Fig. 13.1,1), this eliminates
-
chances ofmisfiring ofeven the fouled spark plugs.
(i) Ilansistorised coil iguition (IrCI) system;
(ii) Capacitor disarge ignition (CDI) system..

oerEcTrvE TYPE QUESTIONS


,} Fill in tJre Blanke or Say'Yes" or fio':
I
I
1. Ignition is only a pre-requisite of ................
.

I 2. A sprk energy below 10 millijoules is adequate to initiat combution A / F ratio 12-13 : 1.


3. Battery ignition system is rarely ued.
5
q 4. A ballast resistor is provided in series with the primary winding t rcgulate ..,..........,.. cunent.
5. In a battery ipitioa system e sparkingvoltage .,,..,...,...... rith increae in engine speed.
6. The initial cost of battery ignition qystem is low.
7. The magneto-ignition system is moe reable tha battery ignition system as thee is no battery or
connecting cable.
2000 3000
8. .,...........,.. is a special type olignition system with its om electric generator to provide the a.ecesdary
energy for the srstem.
Englno spd, r.p.m.
9, Battery ignition systen occupies less space as compared to mag:oetoigrrition systern.
F9. f3.r4 10. The battery ignition system is heavier and requires more mainteaane that magneto-ignition system.
11........-..--.orderiseorderilwhivriouscylindesofamulti-cylindetenginefire.
Dsad,vontages: 12,.......-..--.istheorrectinstantfortheintroductionofsparkneartheendofcompressionstrokeinthe
ThemaindisadvantageofCDlsystemiaveryrapiddischargingofcapacitorleadingtostrong cycle.
-
6park brrt of s/ort durotinn of about 0.1 ms.,This may 1$. fire main nctiou of a ............,,.. is to conluct the high potential from the igo.ition system into the
co use igntin lroblems wheq operting with
rich mixtures. ombutiondrmben
\ 14. The spark plug nust withstaad pressures upto at least ................ bar.
15. A .....,.......... plug has a short heat tramfer path and a small area exrosed to tlrc combustion gases, or
I rrrcnrcursl mnpared to ......---... plug.
16. TCI systern provides lower output voltage.
l. The basb gnition qysreusin uses 17. TCI system is not mucl eliable.
(i) Battery ignition systm ; 18. In TCI system comnnents are subjected to less wear.
(i) Magrreto-ignition system ; 19. In CDI system a ................ is used to stoe tho energy, induction coil is not used.
(iii) Electronic ignition system. 20. In CDI systen,.the output voltage is relatively indepenilent ofengine speed.
4@
INTERNAL col\/lBusllol{ ENGINES

1. combwtion 2.Yes 3, No
6, e. prinary
Yes Yes
7. g. Magneto
9. No
6. driseases
11. Firing 12. Ignitioatirning 10. Ys
13. sparkplug IL
16. No 17. No 18. yes 19. cpacitor
16. cold, hot
20. Yes. L4
t.
t
@
What do you mea by the em agnition,
? How is it relatd rit .combustion.
Engine Friction and Ihtbrieation
What are the requements of an.ignition ? 14'1' Intoductiou. 14.2. Total engine friction. 14.8. Effect ofengine parametrs
system for ari". *gr"" , 14.4. Determination of engine friction. 14.5. Lubrication-Defiirition
on engine lriction.
8. Enumeate the basic ignition systems ald objects-Behaviou of a
4,
and clescibe any of then. jomal in its bearing-Properties of lubricants-Additives-Tlpes ofrubricants.
Describe with rhe help of a near sket.f, 14.6. Lubrication
o.
.tt"ryl[i;.;; systems-Introduction-wet smp rubrication system-Dr!'sump lubrication
system-Mist
of an isuition coil *d" lubriation snteelubrication of-different engine
SfTgl""o"* " -";";;-;'";iu.ryrgortioo"y"t"mof a nut-cylincler iars-Lubrication ofbal anil miler beuings-
oil filters. 14.7. crankcase ventilation-Higllieit*-oj";"
6. Name the various components ofa Qumtions.
ftpe euestions-Theoret-ical
battery igrition system and
State the advantages and disadvantages elrlain any thre ofthem briefry.
ofa batteryignition system
8.
9, fi,i"jll";lfl"T"H"ofr#:-'*r;"" ;** " *,"'iL***, *d disadvutages. r4,1. I}ITRODUCTION
what are the ffer.r""u
and sring orded ?
10.
**"*H^Tattmingf I In- an LC' engine almost a'll machine parts have
relative motion and rub against each
ll. \{hat do vou ude;o""#ililT#$ffi""tff*1Trli"l other' To reduce this rubbing action Ibrication is required, which increases
the life of
t2, What is the main difference between
re battery md
-r* *"* *tion
rimings. engine. The purpose oflub-rication in I.C. engine is generally
rroo _fold,: () To reduce the
t3. List variou electronic igrrition
s "t""t.ooi"*"y*rr" f
in use' Describe rubbing action between different machine paits having relative
motion with each other ;
over tne conventioo" igiuoo .i.ijems any one ofthem clearly statiug its (li) To remove the heat generated inside tie cylinder.
advanrages
t4, Differentiate between hot sprk plug r Engine friction
and cold spark plug. i.s defined. as the difference between the ind,icated pouer (Lp.) (power
16. Explain briefly spak advance mechaisms. deueloped inside the engine) and the trahe power (8.p.) (power
16. ouoilob!" at the cranh_
Explain briefly with a neat sketch th.e .Tlausistorised shaft) i.e.
t7, coil ignition, (TCI) systen.
How does 'capacitive clischarge iga.ition Frictional power, F.P. = I.p. - B.p
system'differ;;rili""toa."a ignition system,? .( 14.1)
"oil o It is impossible to totally emove ail the fiction ross but it can be reduced
by using
lubrication between the parts which have relative motion with each
other. Increase tn
friction is ultimatety dissipated as heat to tt water and. it further increases the
pump and, fan power requirements also. " "ooltlg
o The frictional resistance between two moving parts having relative
motion is mosfly
dependent on the following factors :
Lubricating oil properties
Surface condition
Materials of the surfaces
Rate of relative motion
Nature of relative motion
Quantity of lubricating oil
14.2. TOTAI, ENGINE FRICTION
The difference between I.p. and, B.P. is hnown as total engine friction loss. This includes
the following losses :

1. Direct frictional losses 2. Pumping loss


i1
3. Blowby losses 4. Valve throttling lossps
ir 5. Combustion chamber pumo loss 6. Power loss to driyeihe auxiliaries.
461
/
//
----r----* -
INTERNAL COMBUSTION ENGNES
ENCINE FRICTION AND LUBRICATION 463
1. Direct frlctionel
5. Coribustion chamber purnp loss !
_ It includes bearing losses (mein bearing, camshaft bearing), piston and cylinder friction loss
etc. In reciprocating I.C. engines the frictional losses are comparatively higher. This type ofloss is caused due o the pumping worh required, to pump gases nto and, out of the
pre-combustion chamber. Its exact value depends upon the oiifice siz (connecting the for"_.o-to._
2. Punping loss : tion chamber and the main chamber), and the speed. Higher the speed, greater is tie loss and sma.ller
o ln four'strohe cycle engines eD ample amount of power is used during intake ad ex- the orifice size geater is thc loss.
haust processes (Fig. 14.1). 6. Power loss to drive the auxiliaries :
In order to drive auxiliaries.such as water pump, oil pump, fuel pump, cooling fan and gen-
erator some power is needed. This is also considered as loss since a part ofengine power develo-ped
is used for these purposes.

14.3. EFFECT OF ENGINE PAR.AMETERS ON ENGINE FRICTION


Followingpcrameters erercise their influence on engine ftict.o as described below :
1. Stroke-to-bore ratio 2. Cylinder size and number ofcylinders
3. Piston rings 4. Compression ratio
5. Engine speed 6. Engine load
?. Cooling water tenperature 8. Oil viscosity.
1. Stroketo-bore reio
o The effect of this parameter on engine friition and economy is not very significant.
o Lower stroke-to-bore ratio tends to decrease i.m.e.p. Its lower value reduces the friction
losses as the surface area decreases with decreasing stroke-to-bore ratio n'ith the same
value ofthe stroke volume.
2. Cylinder size and number of cylinders :
when a smaller number of large clinders are used, the friction and economy improve.
Fig. 14.1 This is owing to the fact that the proportion between the working piston area and its
friction producing atreal i.e, circumference is reduced.
o The pumpin g losa is negligbh ln tuo-stroke cycle engines since the incoming fresb rnix-
Fig. 14.2 hows th eftect ofnumber ofcylinders on the variation ffriction for he same
ture ie u6ed 6 gssyengig the exhaust gases and charging the cylinder. piston displacement.
3. Blowby losaeg:
These losses ate caused, du ta tlg leakage ofcom.bustion prod,ucts past the piston 1 2
ftom the
cXrlinder into the crankcase.
+I 8
These
- These losses depend upon the nlet pressure and. compression ratio. I Frc1 )n
losses increse directly with cornpression rotio but get reduced wth an increase in
- engine speed,. o 4
(d
I
E
4. Valve throtling losses: o 0
:COnOmV

o The standad practice for sizing tlte exhaust uJue is to make them a certain percentqe 6
s
{
'smaller than the inlet valoes. This usually results in an insufficiently sized ehaust 4
320( r.p.m.-31 kW
valve and hence, resulk in c:haust pumping loss.
Ifdup attention is not given to the valve size, valve timing and valve flow coeffi- -8 13579 lt
cientd there may be a substantial loss with the increase in engine speed. Number of rings --+
The inlet throttling loss occus due to the restrictions irnposed by air cleaner, carburet-
tor venturi, throttle valve, intake manifold and intake valve. All these restrictions lead Fig. 14.2. Effect ofnunber ofcylilders Fig. 14.3. Effect ofnumber ofriirgs
on friction. on pidton friction.
to pressure loss. similarly some pressure loss is necessarJ to exhaus the combustion
products. 3. Piston rings:
The effect ofnumber ofpiston rings on friction is not significan as the selection depends
on
tleensine lightness required and material used for the rings. It is obvious from Fig. 14.3
*e,S-rings
that generally provide best fuel economy.
404
INTERNAL COMBUSTION ENCIN AS
ENGINE FRICTION AND LUBRICATION 4F5
4. Compression raio:
4. To seal a space adjoining the surfaces.
The frietiondl meen effectiue pressure ncreases as the cornpressioi ratia is increcised,. Btt
the mechanical efft.ciency nnt! eDen increose becase of improveent in i.n_e,p, 5. To absorb shocks between bearings and other parts and consequently reduce noise.
6. To romove dirt and git that might have crept betreen the rubbing parts.
5. Etrgine speed :
+ 14.5.2. Behaviour of a Journal in its Bearing
Ttte mechanical fricton ircreases with the incrase in I
speed. From.Fig. L4.4.lt is evident that theie is nearly linear o
Fig' 14.6 shows the behaviour of a journal rotating in a bearing, the cleaance between the
variation in flm.e.p. with speed. !
a
tyq being shown very mueh exaggerated. The clearance space is supposed {o be completely filled
with oil at all times, which is possible by supplying the oil as fast as if runr oo\. If th" des not
6. Engine load : cl
rotate it will sink to the bottom of the clearance si:ace due to the load w, and the"huft journal and
E
When the load on the englne incieases, the i.m.e.p. also bearing.will touch a3 shown in Fig, 14,6 (a). The way in which the rotating shaft will build up
increases and friction loss alsoincreases. However this inirease pressure in the oil sullicient to separate the surfaces i shown in Fig. <i.16 (nd (c). As
the shaft
in. friction loss is cnpeirsatd by deeiease iir visbosity of the r.p.m. -----"| starts to turn it will climb the bearing wall as shown in Fig. 14.6 (b). But as the speed.increases, the
lubricating oil due to higher temperature resulting f:rom in- Fig. 1,14. Effet of speed on movingjournal tends to put oil into the wedge shaped area between the shaft an the bearing. As a
creased load. result-the oil pressure on right side
mgine friction. $comes more than on the left side, and thejournal is,-therc-
fore, forced away from the bearing An equilibrium position is finally eached as shown in
?. Cooling water temperare ! fall.
The rise n coolng wd.ter tetnpera.ture lro Fig. 14.6 (c) with the surfaces separ{ted by a film of oil whose minimum thickness is ro. The magni-
tude ofro- and position ofthe line offentres in Fig. 14.6 (c) will depend on the load, the"fluid. proper-
reduces the frictional ioss as the uiscos. o ties of oil, the size and. speed of sht'ft, the clearance and length of the bearing.
o)
6
. ity of ol at higlrcr temperature is lower R I

uhich redvees tl* frictonal loss. o Lne ol


o
a Bearing
Cbarahce
8. 0ll viscoslty : q
o c4 spac llled
wilh ol
Higher the uiscosity ofoil gredter is the fric- c
tion loss. As the oil temperature increases, the vis- Eo
'c
cosity decreases and ftiction losses are redueed ar 30 50 7I) 90 110 130
during a certain temperatue range as shown in Oil temper-ahJre ('C)------)
Fig. 14.5. If the temperature exceeds a eetain
value the local film ie destmyed causing meal-to- Fig. 14.5. Effect of oil tenperatue on friction. Area of
oil wedge
metal contact.
Mnmum of film-lhickness = r^
14.4. DETERMINATION OF ENGINE hruCtTON (a) (b) (c)
The engine friction can be detemjned by the foltowing five methods:
Fig. 14.6. Behaviour ofa joumal rotating in a bearing.
1. From I.P. and B.P, measurements
l'rom above, the following points, about the items influencing bearing performance, can be
2. Morse test
concluded :
3. Willian's line method
I A- slippery bearing material is desirable during starting and stopping, but once the oil
4. Motoring method frlm is established the bearing material is less important.
5. Deceleration method. 2' Higher is the speed ofjournal, more will be the oil pulled into the apex ofthe wedge ofoil
For details please refer to chapter 12. in the clearance space ofFig. 14.6 (c), and as a result, more pressure will be
developed. "uppoiti.tg
14.5. LLIBRICATION 3. An increase in supporting pressue will increase the oil film thickness r- and decrease
Definition and Objects
14.5.1.
the eccentricity e. -

4 If the eccentricity 'e' is decreased, the supporting pressure must decrease because the
Lubrication is the admittance of oil between two surfaces hauing relatiue notion. The sidesofthe wedge are mre nearly parallel.
objects of lubrieation may be one or rnore of the
following : 5. Oil film thickness ro is also influenced by changing of clearance.
1. To reduce friction between the parts having relative rnotion.
2. To reduce wear of the moving part. Film Lubrication. It is that type of lubrication in l hich bearing surfaces are completely
separated by a layer of fiIm of lubrcant artd that the
3 To cool the surfaces by carrying away heat generated due to fricion. mouements of the lubricant lawrs.
frictionaL resistance rises only due to ielaiue
466 NTERNAL coMBusrroN ENGfNEs ENCINE FRICTION AND LUBRICATION 467

Boundary Lubrication ! refining, it should be emphasized that viscosity-temperature characteristics ae of


o lJnder--the hy'l'n-dpgnc{ion the oil film supports the load. If the oil film little importance for oils that are to function at approximately constant tempera-
be- ture, such as turbine oils.
comes thin enough so as nt to support the load without occasional metal to metal
contact then joumal friction developed is called bounfury Viscosity ind'ex of oil is uery irnportant where ertrene tetnperatures are encountered,.
fricton and. the lubrieation
existing in this range is known as boundary lubrication. 'ide woril boundary friction is o In order to irnprove the viscosity index ofan oil certain compounds, called V.I. improvers,
used because under this condition journal friction is neither completely dry and not are added to iL These are long chain paraffinic compounds which enable to obtain an oii
completely fluid. In the boundary state, the kind ofbearing mateI, the hadness and having easy starti[g characteristic ofthin oils combined with good protection against
surface nish ofthe shaft, as well as type oflubricant alfcontribute to tbe amount of high temperature.
journal friction. 2' Flash point. It is d.efined, as the lowest tenperature at which the lubricating ol wtl flash
o When the load acting on the bearings is very high, the material itselfdeforms elasti- when a small flame is passed' otross its surface, The flash point ofthe oil should be sulficizntty high
cally aq_ainst the pressure built up ofthe oil flm. ihis type oflubrication, called elasto- so as to auoid, flashing ofol udpours at the temperatures occurrng in cotnmon zse. High flash point
hydrodynaniclubrication, occurs between cams and-fIlowers, gear tuth, and. rolling oils are needed in air compressors.
bearings whn the contact pressures are extreme$ high..
3. Fire point. lf is th lowest temperature at which the oil burns continuously.Ttre fire point
14.5.3. Properties of Lubricants also must be high in a lubricating o1, so that oil d,oes not burn in service.
The chief qualities to be considered in selecting oil for lubrication are : 4. Cloud point. When subject to low temperatures the oil changes from liquid state to a
1. Viscosity 2. Flash point plastic or solid state. In some cases the oil starts solidifring which makes it to appear cloudy. The
3. Fire point 4. Cloud point temperature at which this akes place is called ihe cloud, point.
5. Pour point 6. Oiliness 5. Pour point. Pour point is the lowest temperdture at which th lubricating oil will pour.It
7. Corrosion 8. Emulsification is an indication ofits ability to move at low temperatures. This property must be considered be-
9. Physical stability 10. Chemical stability cause of its effect on starting an engine in cold weather and on free circulation of oil through exte-
11. Neutralisation number rior feed pipes when pessure is not applied.
12. Adhesiveness
13. Film strength 14. Specic gravity. 6. Oiliness. This is the property which enqbles oil to spread, ouer and ad.here to the surface of
the bearing. It is most important in boundary lubrication.
1. Viscosity
7. Corrosion. A lubricant slold not corrode the working parts and it must retain its prop-
It is the ability of the oI to resist internal d,eformation d.ue to mchanical stress* and. erties even in the presence of foreign matter and additives.
hence it is a measure of the ability of the ot fitm to can! o Joad. A more scous oil can
carry a greater load, but it will ofrer greater friction to sliding movement ofthe one 8. Enulsificaon" A lubricating oil, when mixed with water is emulsified and loses its
bearing surface over the other. viscosity varies with the temperature and hence if a lubricating prope rty . The emulsificaon number is on index of the tendency of on oil to emulsfy wth
surface to be lubricated is normally at high temperature it shouid be supplied with oil of waer,
a higher viscosity. 9. Phyeical stabiliy. A lubricaing oil must be stable ph.ysicatty at thn lowest and high.est
The viscosity is measured bypat1arffiEr.The importaut types of viscosimeters ae : temperq'tures between whch tlt oil s to be used. At the lowest temperature there shold not be any
1. Saybolt universal viscosimeter separation ofsolids, and at the highest temperature it should not vapurise beyond a certain limit.
2. Red wood viscosimeter 10. Chenicel stability. A lubricating oil should also be stable chemically. There should not
3. Engler viscosimeter be any tendency for orid,e formatnn,
4. Barbey viscosimeter, 11. Neutralisaion number. An oil may contain certain impurities tht are not removed
The unit ofviseosity is given as "second.s saybolt, or "seconcls red,wood," et, during refining. The neutralisation number test is a simple procedure to determine acidity or alka-
The present-day method ofexpressing the rate at which the viscosity ofan oil will change linity ofan oil. It is theweight in milligrarns ofpotassium hydroxide required to ueuhalise thb acid
with tempe-rature is by s!ting its viscosity inder (y.1.). The oil is cornpared wit}. two content ofone gram ofoil.
referlype o-ils h.auing same viscosity at gg.C. One i.s paraffinic bose oii (liscosty changes 12. Adhesiven ecr,.It is the property of lubricating oI due to whchthc oil particles stick with
consdrably wth tetnperature), is arbitrorie assigied an index ofzero atd the other, a the metal, surfaces.
napthenic base oil Aifile chatrge in uiscosigr with iemperature) assigned. an ind,ex of lOO.
13' Film strength. It is the pfoperty of a lubricating oil due to which the oil reta.ins a thin
A high ui.scosity inder ind,icates relatively smaller changes in uiscosity ofthe oil with -
- the tempera.ture. film between the two surfaces euen at high speed. and load. lhe film does not break and the two
surfaces do not conie in diect contact, Adhesiveness and filrn strength cuse the lubricant
to enter
The higher he viscosity index, the lower the rate at which its viscosity decreases the metal pores and cling to the surfaces of the bearings and journals keeping them wet when
with the increase in temperature. Although a high viscosity index is esirable in the
journals are at rest and presenting metal to metal contact
materials and much efrort expended in improving the viscosity index of oils by until the film oflubricant is built uo.

-

468
TNTERNAL coMBusrroN ENcrNEs
14' Specific gravitv. It is a m easure'of
ENcTNE FRrcrroN AND LLTBRIcAToN 4.!9
deisty ofozl. It is anindication regarding
of lubricant bv comparing one l,,.hricant
*lth ;;;;.-i ;: debrmined by a hydrometer the grade 8. Ani-oidans:
in the oil, and the gravitv is read on which floats
tlt" r*1" r Reduce oil oxidation to protect alloy earins agai4st corrosive attack.
I4.5.3.1. Additives "itl"irylomerer at the suface of the oil.
9. Oilinese inprovement :
Primarily, there are two,types of oirs :
requirements as lubricatins olt.-ttr" Minerar and, uegetabre.Both of them
futfilr some basic
o Some substances such as colloidal graphite ad zinc oxide when added to the oil are
u"g"i;i"""ii " ril*"1"" valuable in maintaining tle oil film.
H::ii'itr*::t"!;;ilL:=" 's
iris;;;;*i* to minerar il rrnd6 Eome exrreme
minerat oits ,,re uery commanty 14.63. Oil contamination and sludge formJion
.simple
"'iln'" used,

- nineral oils ,ave most of the required


charac-teristics
The lubricating oil after a certain operating period gets contamineLd to the extent that
as a good rubricant. rrow_
ever' vryingoperating condirio".
as high viscositvindei and resist""i"'i"
;";;;;;"sp.cifi" p"op"rli"" ,?rJiiJl",,o,
It becomes unsuitable for further use. Tliiconoziation of the oil takes place due to
-u"r,
and corrosion. In order ro achieve
oxid.ation, ft;trttor, woter, formafian f ca.rbon, Iead compound.s, metals, d,ust and d.irt.
some required properties, different ""il'"tron when these contaminants mix with the oil, sludge ig formed in an engine,
typ"" orro-po"nds, called addivesare added.
1. Detergents: r Sludge is a blac!, bown or grey deposit havirig the consisency ofmud. Its formation
takes place as a result of operatian at law engine temperatures diring starting, worming
r Control high_temperature deposits
such as gums. up, and, id,ling periods.
overbased, a detergent acts as effective
,"lJrr"ot.ulirer.
2. Dispersants
14.5.4. $pes of LubricentJ------=--- _:_--
?
Most lubricants areoils or greases. Ilowever, in sncial circumstaaces other fluids likerz,orer,
control low-temperature deposits such
as cold sludge and varnish deposits. air etc. and, soJs suah as graphte rnay perform the function of lubrication. Synthetic lubricants
3. Anti-wear addives: ae also used occasionally.
These additives educe wear and prevent 1. Oils:
scoring, galling and seizure.
. extra strength needed to The different lubricang oils are : Mineral, Fatty and Synthetic.
ffij|":,"l"rf, efficient Iubrication under severe op-
"nsu"e r The mineral lubricaing oils are obtained from fhe residual mass left during crude
o The additives used are : Chlorine
and phosphate compound,s.
petroleum disillation, In this treatment gasoline, kerosene oil, and gas oil fractions
are
recovered from the distillate. Paaffinic crudes generally give a higher yield oflubricat-
. 4.Rust inhibitors :
ing oils than napthenic crudes. The residue is subject t vacuum tstillation. In several
oReduce rusting by acid neutralisation
offormation by protective film. cases steam is also introduced into the system. Bubble towers are used for fractionali-
5. Viscosity index (VJ.) improver: sation ofoil into two or three fractions ofdifferent viscosities.
o It preuents undue thinnng of the'oil Fatty oils from animal and vegetable origin are sometirnes used alone but are fre-
as the
tenperature i*ea$es, ' quently mixed witJ. mineral oils. The fatty oils exhibit poor
r Ahighviscosityindexis.alwaysdesiredfort,r.i".tingoilsasitcanbeusedinsevere keeping quality and un-
dergo decomposition. Fatty oils bowever, eihibit more oiiiness than minerat oils of the

['#ff:jd;:ilTf"":,tti*' ibi-;;b""1;'ootymers a used ror rhe purpose as


same viscosity. Non drying oils such as olive oil, castor oil, rapeseed oil, lard and
are mainly used for lubricating purposes. They are generally-compouned with mineral
fish oil
o Recen,y multierade,l!!1_have.been.produced oil to the extent of about 2 to 10 percent in order lo increase oiiiness of mineral oils.
porvmers. Their speciar
which consist of ,ow viscosity oils within
io uopt to both summer and Oliue oii is ernployed for lubricating textile machinery, as it can be easily washed out
:*y"- "uiiitvi. iinter condi-
lrom !!re cloth without leaving any stain. BaBeseed oil is used for making lubricants
Pourpoint addiives for railway engines, while lad oil is used for
6. : lroJucing lubricants for internal combus-
tion engnes. caster oil is usually compounded with heavy mineral oils for making
o The base oil at extremery rowtemperatures etreme pressute lubricants,
change into butterysorids, due
of wax crystals' If left.unchecke, to fbrmation
t" ,.
ture which first inhibit and_then totalty *v.iuti, .og,.rtut. into honercomb like struc_
e Synthetic lubricante are named synthetic because they are not obtained directly from
pour point of oit by interfering
sffieis te oir frow. These additues reduce
petroleum. Various applications require oils which will have, for example, a lower point,
witn ** or better viscosity-temperatqe characteristics, or a higher degree of resistance to oxi-
c Polymerized phenols or esters "yriitiitior.
are used upto dation or isotheimal decomposition than petroleum derived lubricants do. The synthetic
t p""r"nt of the oil for thepurpose.
7. Antifoam agents: lubricants presently in use are clcss ifiei as :
(i) Dibasic acid esters ;
r Reduce oil foaming by causing
collapse ofbubbles due to air -'\
entrainment. (ii) Organo-phosphate esters ;
o Silicon esters arc userl as antifoatn
gents. (iij) Silicate esters; \
(iu) Silicon polymers ;
\
\\ INTERNAL COMBUSTION ENCINES ENGNE FRICTON AND LUBRTCATION
471
(u ) Polyglycolethers an{ related (iu) Easy starting and short warming up period.
(ud) Fluorinated and chlo\nated hydrocarbon compounds. (u) Reduced oil consumption.
Classification of lubricatin! oils (ui) Prolonged mileage between the decarbonisation.
o The lubricating oils are nonnally classified according to tleir viscosty. The sAE (soci- (uii) Due to execellent thermal and oxidation stability of multigrade oils the engine parts
are
ety of Automotive Engineers) method of assigning number to different oils is used uni- protected against rust, corrosion and wear.
versally.
2. Greesee:
SAE has assigoed a number to an oil whose viscosity at given temperaturnes falls
in
certain raoge. There are two temperatures used as eference for assigning the o Lubr|cang gteaee is a soli.d. to semi-solid. d,isperson of o thickening agent in tiquid.
number to oils - 18.C and 99"C. lubrizont. other ingredi.ents impartng specia.l properties may be inctud,ed..
sAE, 5 W, 10 llt/, 20 w grades ae defined in terms of viscosity at rg"c and are the
-
o Greaseg are normally used under conditions oflubrication for which oil is not as suit-
oils which render starting of engine in cold. weother ecsy. s'Ar, 20, 80, 40 anil 50 able or convenient. Greases preform better tlwn oils und.er cond.itions reqairing :
grades are defined in terms ofviscosity at gg"c; these oils worh (D High bearing loads aud shock loads.
satisfactority in
normal and hot climatcs. These numbers are merely used for classification of oil (ii) Slow journal speed.
according to viscosity and d.o not indicate the quality of oil since factors like stabil-
(idd)
Temperature extremes.
ity, oiliness etc. are not considered (by these numbers),
(iu) Cleanliness or avoidance of splash or drip.
Table 14.1. SAE clessificaion of Lubricanng Oile (u) Minimum attention.
(ud) A seal against external contaminants.
Viscosity Rmge
(uid) Large bearing clearances.
SAE Viscoeitv ViacoeityUnits
Number' At - l8'c ar 90'c
The various grpes of greases used for lubrication are :
Min. Max. Mi!. M. (i) Calciun soap greases ;
5W (a) Centipoise 1200 (ii) Sodium soap greases;
() sus 6000 (lii) Alurninium soap greases
l0w () Cmpoise
(b)
; 24 (lu) Mixed soap greases ;
;

sus 6000 12000


(u) Baium soap greases
20w '(a) Centipoise 2400 9600
;
(ui) Lithiun
(b) sus 12000 418000 _ soap geases ;
(uii)
?n (c) CentistoLes
() sus
\_\ o.I 9.6
Pure petroleum greases.

_ \--_
,15 58
30 (o) Centistokes
9.6 12.9
14.6. LI'BRICATION SYSTtsMS
(b) sus _ 58 70
40 (c) Cetistoles : 12.9 16.8
14.6.1. Introducion
(b) sus _\_ 70 86 Tine main parts of on engitrc whih ued,lubricatan ae as given under :
OU (o) Centictokes (t) Main crankshaft bearings.
16.8 n.7 (ii) Big-end bearingt.
() sus
85 110 (iii) small end or gudgeon pin bearings. (iu) piston rings and cylinder walrs.
o Multigrade (u) Timing gears. (u) Camshaft and camshaft bearings.
oile. It is possible to develop an oil with more thanrone viscositr at differ-
ent temperaures. Thus an oil may be in sAE.B0 grade at 99.c and in the sAE-10 w (uii) Valve mechanisn. (uijj) Valve guides, valve tappets and rocke arms.
grade at ( 18"c, oils ofthis type are known as rnultigrade oils; other possible grades Various lubricetion slstem$ used for LC. engines may be classifurd as :
arc : 5 W/20, 20 WnO, 20 Wl4O. 1. Wet surnp lubrication system.
Following are the "advantages" of multigrad.e oIs : 2. Dry sump lubrication system.
(i) No necessity to dtange the oil as per the ambient temperature. 3. Mist lubrication srstem.
(di) owing to ease ofcranking at low ambient temperatuie
the life ofbattery is extended.
(iii)Long engine life ; required viscosity is maintained under difrerent opeiating tempera-
tures.
472
INTERNAL clcMBUsTIoN ENGINES
ENGINE FRIqTION AND LUBRICATION
14.6.2. Wet Sump Lubrication System !73
These () Semi-pressure systcrn :
systems employ a large.capacity oil sump
at the base of crankchambe from which
the oil is drawn by a row pressure o'purnjand
eriie.ed o This method is a combination ofsplash and pressure systems. I incorporates
returns back to the surnp after serving ih" jo"po".. -- to various pr"t". or-;;;;" gradually vantages ofboth. In this case rnain suppry ofoll is located in the base
the ad-
of rank chanber.
(a) Splash eystem:
oil is drawn from the lower portion i irr" through a filter and is delivered by
means of a gear pump at pressure of about"u-p 1 ba to thi main bearings. The big end
o This system (Fig. 14.7) is used-on somes a, bearings are lubricated by means ofa spray through nozzres. Thus
oil also lubricates
four strohe stctiorwry engnes. rn this case the cams, crankshaft bearings, cylinder ialis and ming gears. An oil pressr,,re g"oge
the caps on the big ends bearings ofconnecting
rods are providJcl *iti-."oop, *ti"rr, is provided to indicate satisfactory oil supply.
when rhe connecing rod is in rhe rowest
diect the o' through holes in the caps to ile
post;;r;;;ilff;i;dils and thus r
\
The system is less coslJ to instail as compared, to pressure srstem, It enabres
brg end bearings. o", to higher
reaches the rower portion ofthe cyrinder wats,.crankshaft o oil it oeanng troad,s and engn speed.s to be employed as compared. to
"il."r
and other parts requiring splash system.
lubrication. surplus oil eventually flo*s back (c) Full pressure system
maintained by mens of a oil pump which
lo the oil sump. orl rlr"ifil" rroughs is :
takes oil from ii""rsi nlb".
I o fn t!!s system, oil fom oil sump is po-p"a.\u" pressure to the various parts requir-
""_0, ing lubrication. Refer Fig. ra.p+rre or is araw.t dpto ttru .o*p through frlter and purnped
Ol pessure by means of a gear pump. oil is delivered by th\ pressure pump at pressure
ranging
from 1.5 to 4 bar. The oil under pressure is to main be*ng, fcrankshaft and
camshaft. Holes drilled through the main ""ppfi{d bearing jou-rnals, communicate
oillo the big end bearing and also smalr end".unk.il"rtr
bearinls through ote ante in coniecng
rods' A pressure gauge is provided to confirrn the circulatioln of oil to the
Camshaft ,rrriou" p"rt".
A pressure regulating valve is also provided on the delivery side ofthis pump
to prewent
excessive presgure.
Connecting rod
Main bearing
bearing

Main bearing

Oil troughs
Lower oil

Ol strainer

Fig. 14.7. Splash system.


Fig. 14.9. Full pressure system.
r splash system is suitabre for row and mediun
speed, engines haung moderate
load' pressures' For high performance engines, bearing TI,is system fin d,s fau,o.ur from most of the engne manufacturers as it allows
*iicn ,,0.,oa,y operate at high bearing hgh bear-
pressures and rubbing speeds this system tng pressure and rubbng speeds.
does not serve the purpose.
The general arrangement of wet sump rubrication system is shown
in Fig. 14.9. In this
case oil is always contained in the sump which is drawn
by the pump throigh a strainer.
4'14 INTERNAL COMBUSTION ENCINES
ENCINE FRICTION AND LUBRICATION

14.6.4. Mist Lubrication System


This rystem is used. for two strohe cycle engies. Most of these engines are crankcharged,
1.e.,they employ crankcase compression and thus, are not suitable for crankcase lubri-
To baarlngE cation.
Pressurs
regulator These engines are lubricated by adding 2 to 3 per cent lubricating oil in the fuel tank.
The oil and fuel mixture is induced through the carburettor. The gasoline is vaporised ;
Oilpump --- Breahgt and he oil in the forn of mist, goes via crankcase into the cylinder. The oil which
impinges on the crankcase walls lubricates the main and connecting rod bearings, and
rest of the oil which passes on the clinder during charging and scavenging periods,
lubricates the piston, piston rings and the cylinder.
For good performance, F/A ratio useil is also important. A'FIA rato of 40 to 5o : 1 is
optimum. Higher rotins increase the rate of wear and lower ratns result in spark plug
Engine foulittg.
Drain plug cfankcas
Wet sump Advanages:
Fig. 14.9. Wet sunp lubrication system. 1. System is simple.
2. Low cost (because no oil pump, filter etc. are required).
14.6.3. Dry Surnp Lubricatlon System
Disadvantages:
Refer Fig. 14.10. In this system, the oil from the sump is carried to a separate storage
tank outside the engine cylinder block. The oil from sump is pumped by means of a 1. Some lubrication oil will burn and cause heavy exhaust emissions and deposits on piston
sump pump through filters to the storage tank. oil from storage tank is pumped o e crovn, ring gtooves and exhaust port and thus harnper the good performance ofthe en-
gine.
engine cylinder through oil cooler. Oil pressure may vary from B to g bar.
Dry sump lubrication system is gencralty 2. Since the lubricating oil comes in contact with acidic vapours produced during the com-
a.dopted, for high capacity engines. bustion process , t rapi.d.Iy loses its anti+otosion propertes resulting it corrosion dam-
Venl
4e of bearing.
3. lte oil and fuel must be thoroughly mixed for effective lubrication. lbis requires either
separate mixing prior to use or use ofsome additive to give the oil good mixing character-
istics.
4. Owing to higher e:rhaust temperature aild lees efEcient scavening the crankcase oil is
diluted. In addiion some Iubricating oil burns in combuetion chamber. This results in 5
to I5 per cent hgher lubricont consumption for tuo stroke engirc of similar size.
5. As there is no control over the lubricating oil, once introduced with fuel, most of the two
stroke engines are ouer-oiled, most of the time.
14.6.5. Lubrication of Different Engiae Parte
1. Lubrlcation of main bearings :
T]e nuin bearings are lubricated satisfactorily rvith the help ofa ring (or chain) type
feeder,
The ring oiling system is shown in Fig. 14.11. It consists ofa ring which is bigger in
Scavenging diameter than the shaft. There is one groove at the bearing cap antl the ng is placed on
pump the shaft. The lower part of the ring is dipped into an oil reservoir. Due to the rotation
ofthe shaft, the ring rotates at a slower speed and caries the oil from the reservoir to
Fig. 14.10. Dry sump lubrication system. the bearing. This system works on the prircple ofadhesion. In this system the lubricat-
ing oil can be fed, only when the shaft s rotating. Instead ofdngs, sometimes oins are
also used.
476
INTERNAL COMBUSTION ENCINES
ENCINE FRICTTON AND LUBRICATION
Bearirg cap

isq,/20!

Oil resenoh
Fig. 14.lB. Splash lbrication system.
Fig. 14.11. Ring oilingsystem. Note' Gecse cup method is used for lubricating rocker arms and reciprocating parts with a jerkey
'iit:tx::#i:\#r::i:::!t{ffi
This type
oflubricatiou ir motioh' It is also employed on parts that are mt readily acccsible and cm be lubricate only at fairly long
;i:#"y#i:ffx#x,1:;"":::::,;,:;:; interuals.

Lubricaion of Ball and Roller Beariags


14.6.6.
2. Lubrication of cylinder and emall end bearing --':!he
of connecting rod ball and roller bearings are lubricated for the following purposes :
:
r rne cyunder,
The cylinder, small end bearing (gudgeon pin),
pin). 1. To reduce friction and wear between the sliding parts ofthe bearing.
valve gear pins, rocker shaft, crantpi" 2. To prevent rusting or corrosion of bearing surfaces.
et". a"e
ruDrrcated by drip system. 3. To protect the bearinpi surfaces from water, dirt etc.
o In drip system oil is fed to ghins parts
drop 4. To dissipate the heat.
by drop, from an oil cup. Although f;;;';; Filling hole
For lubcating the balt and roller bearings, gencrally oil or light grease is used.. Only pure
efficient method yet it is often tfr" .Ll-"*_
mineral oil or a calcium-base grease should be used. Ifthere is a possibility ofmoisture contact,
venient way oflubricating the exte.nal par
of then potassium or sodium base greases may be used. Another additnal advntage ofthe grease is
engines and machi.es, Valve gear pi"",
that it forms a seal to keep out dirt or any other foreign substance. The temperature shoul be kept
shaft,s, main bearings of r_"fi"n*Io, "*f".
o"ii_
pins, cross head pins, line shaft -"art"er.l"i
below 90'C and in no case a bearing should operate above 150.C.
many other machine and engine pas;e lu-
14.6,7. Oil Filters
bricated in this fashion.
,tril the lubricating oil (used for lubrication purpose) from the oil sump, must pass through
Fig. 14.12 show s a drop feed. oil cup. It comprises of an oil filter before it is supplied to the engine bearings. Bearings maintain very close tolerances
a oil reservoir or cup made of glass. At ihe t"o and are likely to be damaged by any foreign abrasive material entering the lubrication line.
tirere is an opening called filting hrl" *,
th;;;l;:"ti" The filter arrangement may be of the following two types : (i) By-pass ope ; (i) Fuil-flow
lubication oil is poured into th; cup. type.
The
the inner-chamber through the openings "rf ""t"*-fr
lt;i; j;;; 1. By-pass type filer arrangernent :
o"op due to graty by the nozzle. The ""dpassage
:J
through the nozzle is controlled y
of oil o In this arrangement only a small portion of the lubricating oil is passed through the
vuL,"l D6!
.int t filter and the remaining lubricating oil is directly supplied to the bearings by fhe oil
" "eu1"
feed glass is provided to see the a-"op.
ofoil.'--'v"' pump at pre-set pressure, determined by the pressure regulating valve. Consequently a
3. Lubrication ofcrank and gudgeon pin Fig. 14.12. Drop feed oil cup. portion of the oil is continuously filtered.
Fig' 14.18 shows spras lubriation o Since quantities of oil flowing through filter are small, a uery fine filter or a specal
filter
sysezr. The connecting rod is impregnated with resin to duert d,isintegration d,ue to moisture is used,. Such a fine
crankcasg' At the time of rotation dipped into the oil of the
he oil is rpl"rrrJar" to paper/filter will remove all harmful contaminants.
different parts requiring it. rhis- type centrifugal force and it reaches the
olo"i"tiJ il ,iitrru ro" a crankshaft speed
The oil must be fiitered or renewed p"ri"di;"tv. of 200 r.p.m. 2. Full-flow type filter arrangement:
,T,n:r: Also rhe revel of the oil shourd be
main- o In this frlter arrangemenL whole of the oit is fihered before it is supplied to various-
bearings. Thus, the size of the filter is comparatively large.
o In this case, it is hardly possible to remove very fine particles because of high pressure
required to pump oil through such filters.
o All the lubrcating oil, in normal course, should be filtered approximately every half
minute' A pressure relief valve is used to prevent excessive pissrrr" buili up after a
cold start.

l:,
< 479
I

i erlcrNe FRICTION AND LUBRICATION


478 ENGfNES I with compression ratio'
INTERNAL COMBUSTION
4. Blow by losses decrease directly requireil to pump gases into
and oui
pumploss is caused due to the pumping work
o. Combustion chamber
14.7. CRANKCASE VENTIIATION of the pre-combustion chamber'
r'm'e'P'
Iwer stroke-teboe ratio tends """ ""' *"d tfte
to
Crankcase ventilation is required owing to the following tuo reasons i 6. eonomyimprove'
small number of large ftinaut" t'" !o]1n 1d
(i) The various contaminants such as water, gasoline, blowby gases etc. enter the crankcase When a
piston rings on friction is not siCt*l
due to several reasons, and may cause sludge and corode metal parts. E. *" t""-*r of
ratio is increaseil'
"*" preslue incremes as the compresion
(ii) To elieve any pressure build-up in the crankcase which may cause leakage ofthe crank- 9. 'Ihe frictional mean effective inaease in speed'
with the
shaft seal. 10. T]:e mechaniel friction .........-.'
the frictional loss'
In practice, following two types ofventilation systems are used 11. The rise in cmling water temPeratue """""" also increass'
:
engine incremes' the i'm'e'p'
1. Open system 2. Closed systern. t2, *n*ii"i* "*ne friction loss'
13. **O", *" **"ity of oil lower is the r-n-a tion.
I. Open syotem: of oil b"t*een two surfaces havingrelative motion'
14, :.::::.. il;;it*"" are completely separatecl by a
i1I wh:*Iaring surface
typ""o:t-iot'l"ati9n movement o{ the
In this system, fresh air'supply is inducted into the crankcase during the compression 16. ............ lubricatioo i" t.}rut' b relative
taver of filn of lub."o, *o Jrrrt
ir," frictional resistan"" "trJ".-;;i;;;;
stroke (due to creation of small vacuum). The entering air picks up the contaminants
the pressue
(water vapour, gases and HrSO, vapour) and discharge them to the atmosphere during itslrderormselssticallv against
16, liiTH"til':";" the bearings is v^ery qgh1".n:Tlfi lubrication'
expansion stroke.
of the oil flm' ffris type of lubtion is cdted """"" '
itis a
built up due to meaical stesses andhene
The main disodva,ntage ofthis system is that the noral ventilation is quite inad.equate oi];;;'J't**t*t"t-ation
t7, ............ is the abity ofthe
d,uring id,Iing or running at low speeds.
;;;f;;;;bttY of the oil film to mrry a load'

2. Closed syetem: The viscosity is measured


by """""" '
18.
o In closed system the fes h air supply is token to the crankcose from the carburettor. o'wi *alge :nT t-neenture
o Air cleaner and the breather outlets are connected to the intake manifold through a li :tt: m:xinffi':ffin:':":"*
t"d". t"diJ;;.";i";;;; .-"1t""
:le
vrscositv oran
lo vis'cositv of the oil with the
temperature'
when a small llame is
PCV valve to ensure the burning of all the crankcase gases in combustion chamber. 21. A low scostw "tr"rgu"
*r'i"r' *'" rou;"atitg oil wiu flash
zz. ............point is the t"*"';;;;;;;;; ut

continuousrv'
,"-perature at which oil bums
IIIGIILIGHTS ,r. ::::*.;-1*T,l1il[Ti;t
lowe"t tltierature at whi t'he
lubricating oil wiil Pour'
24. ............ Point l" t" l

L. Tlre diffeence between I.P. and B.P. is knom as total engine friction loss, 25.............isthnprop"ttvor--oii*rtilenablesittosp'**"'t"artoetothesurfaceofthebearing'
of ttt" t"od*"y of an oil to
emubiry wi water'
, Lubricetion ie lhe admittance of oil between two surfaes hang relative motion. 26. The ....'...'..' oornU"" * "o-iolo n*ui* "n* *th the metal
surfaces'
oil due towrticrr e
3. Film lubrbation is that type oflubrication in which bearing surfae are completely separaCed by a layer 27. ............ is tlt" propt"ty
"rli;*]ti"g "i
offilm oflubricant and that the frictional resistnce arises only due to relative movenent ofthe lubri- of density of o
'\\ z, Sp."iA" gt",tity is a neasue purposes'
cant layers. used for all lubricang
29. The '........... oilE ale very commooly increases'
If the oil ln becomes thin mough so a6 not to support the load without occssional metal-t-miial------ *p9t*;;;;;;;; t'himing of the oil as the temPerature
3o. Viscosiry ind""
contact then joumal friction developed is called oundcry friction nd the lubrication existing in this sti foam agents'
i. sili*o *"t"rs are used ae-o'1"'
rmge is known asboundnry lubrimtinn.
32. .....'...... i6 u tttt, 'o*t i"n*i'tt"""t the consistency of mutl uuile petroleurn distillation'
Viscosily is the ability ofthe oil to resist internal.deformation due o mehanical stresses and hence it is l"t *g
rr""tirg oir"""lJ;io"a ro- *r. r"rio"rlt""
D.
a measure ofthe ability ofthe oil film to cany a load. 83. The mineral engines'
small four-stroke stationary
6. The mireral ols are uery commnly used, for all lubricang purpo*l 34' Splash system is used on some
a combination of splash and prssure sr$ems'
Lubricoting grease is s solid to semi-solitl dispersion of a thickming agent in quid lubricmt. 36. Semi-pressure system is uaopt" fu high capacity
engines'
8. Lubrication system ue dassified s follows : 36. ............ sump *
t"O"*tt""l"i"^ g"o"t'ly
for twcstroke cycle-eneines'
(i) Wet smp lubricaticn system 37. .........-.. lubricatiol systenis uged tkm to the aattkcree frcm the
cuburettor'
(ii) Diy smp lubrication system 38. I .....'...... t"oii"-tl"o-""*m fresh air supplyli
cr-ttt""
(iii) Mist lubrication systen used for tuo strcf engines. 39. In............s"'o"*tt;;;;;;-s'ystemfrehairsopprvi'i'ti""*intthecrankcaseduringt'hecomi
--
pression stroke' - r, r.:--^--:^- agent in liquid lubri
^r o +tienins
40.Lubricatinggreaseisasolidtosemi-solirlispersionofat}rieningagentinliquidlubricant.
OBIIECTWE TYPE QIJESTIONS

Fill in the blanka or Say "Yes" or ,t{o': lmswnns I


5. Yes
1. Engine friction is defined as the difference between I.P. and.,.......... . S.BlowbY 4'No
q.
The pumping loss in two-stroke cycle engines is quite significant. 1. B.P. 2' No
8. Yes 9' Yes 10, incrases

3, ............ losses are caused due to the leakage of combustion products past the piston from the cylinder 6. decrease 7' Yes
int the ran.kcase.

--l
INTERNAL COMBUSTION ENGINES
ll. reduce lZ. yes 13. No 14. Iubrication ENGINE FRTCTION AND LUBRICATION
16, elasto-hyodynamic f5. Fikn
17. viscosity 18. vicosimete
20, viscosityindex 21. No 19. Yes \481
22. Flash 23. Fire 22. What is the role oflubricating oil lters of automobile engines
25, Oiliness 26. emulsification 24. Pou ?
27. Adhesivenese 28. Yes 23, How the lubrication of two wheeler is itone ?
30. Yes Bl. yes 29, mineal
32. Sludge 3ll. Yes 24. Explair briefly "Mist lubrication s5ntem".
35. Yes 86. Dry 34. Yes
S7. Mist 18, closed 25' whatarethevaiousdesiredpropertiesofalubricant?Explainhowadditiveshelptoachievethede-
<[0. Yes. 89. open
sired properties ?

_ . 26, E_xplain briefly "Elasto-hydrody.amjc lubrication"


anrl Toudary lubrication".
272 what do you understand bv full-flow
I ruronrrrcAl auESrroNJ
ia/ other ?
tpe and byaass type
vr,s vt
oil frlters
' ? when one is prefe*ed over the
I
\
l. How is'engine fti"rror, U"** r 9----t zs. What do you understandby "craakcaseventilation
, State the importance ofengine friction. 29. What is the difference hetween open crakcase
?
anal closed crankcase systems ?
3. Enumerate the factors on which the
frictional r*bcdrce
rqsistance reween
betwe two moving parts having relative
motion is dependent.
4. Discuss the components into which
the total eagine friction can
5. Explajn brieflythe following; be divided.
Ii
i
(j) Direct frictional losses
(il) Valve throttling losses
(lli) C65o.r1oo *amber pump
loss
(iu) Blos5"o""o.
State the effect ofthe followin g mgine
i' parameters on engine friction
I (i) Stoke_to-boe ratio :
(j) Ctod"" *d number
(lii) Piston rings of cylintlers
"t'"
(iu) cotnt*"'ottuoo
(u) Enginespeecl
(utr) C;ing;a;remperaiue. ,:::|fiTil::i;
Enumerate the methods by which
engine fric'on ;;;*.
8. Defi ne the term .lubrication'.
9. What are the objects oflubrication ?
10. Discuss the behaour ofajournal
in its bearing.
tl. Explain briefly the following:
(i) Filmlubrietion
(r'i) Elasto-hydrodynmic
lubrimtion
(ill) Bounry o.""ton.
12. Enumeate and disoss
the chief qualities to be comidered
13. ExJ'lain briefly the foowing in selecting oil for lubrication.
properties ofa lubricant :
(r) Viscosity -;;;,
,
(ii) FlashPoiat
(lii) oiliness
(u) Emulsifimtion
(u) Neutrarisation number
14. what
are additives ?
(ui) Ar""ir*""r.
15. What do you understand by bil contamination
ad sludge formation, ?
16. How are lubricating oil classified ?
77. What are "Multigrade oils', ? What
are their advatages ?
18. What is a grease ?
t9. What is the imporiance oflubricarron
in I.C. engines ?
9A Enumerate lubrication systems and
exprain weisump rubrication
2t. Wrere is dry sump lubrication system system with the help ofa neat sketch
prefened md whv ?
ENGINE COOLING

Table 15.1 gives the heat balance ofprime movers.


Table 15.1. Heat Balance of Prie Movers

15 % ofFuel euergy
S.No, Prime mover Topower To cmlmt To exhaust To radiation Total
Engine Cooling 1. 4-strohe S.L engine % 30 t2 100

15.1. Necessity of engine cmling. 15.2. Aeas of heat llow in engine. 15.3. Gas temperatue Diesel engires:
variaion. 15.4. Heat trmfer, temperature distribution md temperature profrlerHeat trmfer- (j) 2-stroke 30 2L 37 12 100
Temperature stribution-Tenperature profiles. 15.5. Efrects ofopaatingvariables on engine
heattrmfe. 15.6. Coolingairmdwaterrequirements. 15.7. Cm)ingsystemAir coolingsystem- (ii) 4-stroke
WaterJiquid cooling system. 15.8. Components of water cooling system-Highlights-Objective
Tpe Questions-Theoretical Questions. (c) Naturally 31 % 30 13 100
aspirated

15.1. NECESSITY OF ENGINE COOLING


(b) Tubo-arged o 22 n 100

Gas turbine :
In an I.C. engine, the temperature ofthe gases inside the engine cylinder may vary from
35'C o less to as high as 2750'C during the cycle. If an engine is allowed to run withou external (l) Simple cycle IO 70 ID 100
cooling, the cylinder walls, clinder and pistons will tend to assume the average tempeature ofthe
(il) Regenerative to 65 20 100
gases to which they are exposed, which may be of the order of l(XlO to 1500"C. Obviously at such
cycle
high temperaturc ; the netals wiII loose their charaderstics and, piston will expand. consierdbly
and seze the liner. Of course theoretically thermol efficiengr of the engine will improue wthout
cooling but actuolly the engine will sze to run. lf the cylinder wall temperature is allowed to rise Demerits of overcooling. Overcooling of the engine is harmful because of the follo*'ing
above a certain limit, about 65"C, the lubricating oil will begin to evaporate rapidly and both cylin- reasons '.

der and piston may be damaged. Also high temperature may cause excessive stress in some pats
rendering them useless for further operation. In view ofthis, part ofthe heat generated inside the 1. At very low temperature, starting ofengine becomes difcult.
engine rylinder is allowed to be carried away by the cooling system.Thus cooling rystem s provid.ed 2. Due to overcooling, engine life is reduced due to corrosio.
on an engine for the following teaaons: Ifthe engine is overcooled some ofthe heat which could be used to expand lhe gases will
1. The even exllansion ofpiston ln the cylinder may result in seizure ofthe piston. b lost.
2. High temperatures reduce strength ofpiston and cylinder liner. 4. The fuel will not vaporise properly and some of the gases produced by combustion will
3. Overheated cylinder rnay lead to preignition of the e.harge, in ease of spark ignition engines. condense on the cylinder walls. This lead.s to d.lution of the oil in th.e pump and the
4. Physical and chemical changes may occur in lubricating oil which may cause sticking of additon of hartnful corrosve acids. Removal ofthe oil film from the cylinder wall by
piston rings and excessive wear ofcylinder, unvaporised fuel leads to increased. cylinder boe wear.
5. If the cylinder head temperature is high the voluqnetric efficiency anal hence the power 5. Inadeque lubrication of the engine, due to oil not being warm enough to flow freely,
output ofthe engine is reduced. \ ' esults in greater ftictanal losses.
Thus engine cooling is required o keep the temperatue ofthe engine low in or{etffi In general, due to overcoong the economy and life of the engine is reduced.
.(i) Loss of volumetric efficiency and hence power ; Demeits of undercooling :
(ii) Engine seizure ;
The following are the demerits ofundercooling :
(lii) Dangel of engine failure.
1. Undercooling can cause engine sezure, or dt least-shorten ualve life ond possble dts-
' . Almost'2 to 35 percent of totat heat supplied, in the fuel is remoued. by the cooling tortion of the cylinder block head. or goshet.
medium.
A hot-spot inside the combustion chamber may be suf{icient to cause pre-gnition, .e.
. ;Ieat carried away by lubricating oil and. heat lost by radiation amounts to 3 to 5 per
cent of the totdl heat supplied.
to igrite the fuel before the spark plug does, thus causing loss of pouer and possible
darnage to the engine cotnponents.
It must be noted thal heat carried away by the coolont is a d.ead, Ioss because not only
no useful worh can be obtained. from it but a part of the engine power is also used, to 3. Wate in cooling system may boil and evaporate, ar'd should the ol fiIm burn away
remoue this heat. Hence, it is of paramount importance that this loss is epl minimum add,itionol friction and wear wll occur between cllinder and pston.
by the designer

-t
I
I

I
484
INTERNAL COMBUST]ON ENGINES
r5.2. AREAS OF HEAT FI.oW
IN ENGII\ES ENCINE COOLINC
t nlce due to dirference
485
;TrHt*",!tr}j:: l;tes in temperarure and frorn bigher Fig, 15.I shows the gas temperature variations for
one cycle (of a 4-stroke engine) and two
intalestrol-an;i,*S*J,#&ri""::,T".,i":*:m,"f;.f compresson ratios (i.e. 5.4 : 1 and 9 : 1) for chemicaily
correct mixture of crHr, nd air, prus
and expansion processes the heat
;;;"f* H;;ii"" ro* .the gasesJt;,f m*Hf H clearance gas.
stion to the warts.
ga'ses give part or tnu, nuat iolhe 15'4' H-EAT'TRANSFER, TEMPERATLJRE DrsrRrBLrrIoN
ir,
(ij)
ffiffjfj" rolrowing components :
FILES AND TDMPERATTjRE pno-
Piston and piston rings \
(ili) Cylinder head 15.4.1. Ileat Transfer
(lu) Exhaust valves and Following are the processes by whe heat transfe
e:<haust ports. takes places :
- H:il:: i:#?i:'"ruH,.il.:r.J.l the cooring sysrem whle some
is rosr by direcr
Conduction' 'Cond'uction' is the transfer ofheat from one part
part of the samejlbstance, or one substance to ofa substance t another
another in physical contact with it, without
The heat going to the.surrounding appreciable diSflcement of molecules forming th"
air,'to-the.structure su"t^rr"".
rubricating o'is usuarv ofthe engine and to the Convection' 'Conuection' is the transfer ofheat within a
fluid by mixing ofone portion of
goes either to the cooling "-"iiii;;
syste*
107a),, whereas most of
o" i"i"), ,n exhaust gases.
tie heat rejected, the fluid with another.
Fuel or natural convection. It occurs when the fluid
r53. GAS TEMPERATURT VARIAIION circulates by virtue of the natural
differences in densities of hot and cold lluids. ile e-n""" po"tiorrs
of the fluid move downward
because of the greater force of gravit *o,p*.,uilr, tu force on the less dense,
orthe
cvcle there is an appreciabre "" is done to
,"."" r3"T:'fni'fl?#"?iffi:::s remperarure difference or Forced conuection. when the work blow or pump the fluid, it is said to be forced
convection.
At the beginning ofinduction stroke
- the temperature is that ofcrearance Radiation'
As the cool mixture is inducted gases. 'R d'iation is the transfer of heat through space or matter by means other than
in the engine
cylinder the ,"*o"rur".u *lrs rapidly. o convection. This phenomenon is not very sigaificant
During comprei.l?i,ff:::,: te :::i.*t"r with regard to reciprocation I.c.
remperure increases and atrains
value at the end ofcombustion process. its maximum
o At least g5 per cent of the heat transfer between the working
- irilifi:;:;ffi;:X.T:::::the temperature decreases and rhen
drops very rap-
and. the engine components ond, cooling fluid and engine components
fluid. is effected"iy ,,Fo,ii"iZinoecton,.
The transmission oflheat per unit ime from a surface
In actuar engines' there is some by convection is given by:
tempeature drop during the exhaust
process.
where, 3 = [ii;?iconvective hear transrerred,
= Coefficient ofconvective heat transfer,
. = Area ofsurface, and
n OL- t) = Temperature difference between the fluid and the surface.
I The units of coefficent of heat transfer arc :
2000

:E
E h=-a .= Y -=wm2K
1 500 A(\-t) *2K
o The coefficient of convective heat transfer ,,(also
kno.wn as film heat transfer coefficient)
E may be defined as the atnount of heat transmixed
fluid and unit area of suface in unit time. it" r"1". aunit temperature difference between the
Iq 1000 fo,r
,h,
o depends on the typcs or.flud.s, their
uelocities and-tenperatures, dinensions
(5
500 tr tn" ip"'""i"rthe trpes of problems. since ,, depends
upon several factors, it is difficult to fram
a to satisfy all the variations, howcver
for "irgi"
a dimensional anarysis gives an equation "qr"tio.,
the purpose which is given as under :

to'" 600'
cranr,u.it"e"1e; _--_r +="(Y)"(+)'g)" .( 15.1)

Fig. 15.1. Gas temperature variations


n.. ^cycle
tions for one ^c ) -,-1 ongue.
a,^\^ of4_stroke or Nu = z(tu)" eD, (?)
where, Nu = Nusselt ,ro-u" l4?)
\/
ENCINE COOLINC 487
INTERNAL COMBUSTION ENGINES
486
8r
tr_h=
/pvD) ...{u)
-
Re = Reynold's number lL I,
\P/
pr = prandtl numuer . l") ,-u=# ...(ui)

[?J, Adding equations (iu), (u) and (ui), we get

= Di.rnut"" to length ratio,


^f 1 x 1 I
- t6=Q
lo" u- ha,- hb. AJ
to
f
C = A constant to be determined experimentally,
cp = Specific heat at constant pressure' O=-r:Jn- ..(15.2)
/= Thermal conductivitY, un l' hu
P = Density' Ir u is the overall coerricient orheat
= Dynamic viscositY. and
V = Velocity. :"T:::"1 , Aeo_]D_
e=uA(L_A)=i r L
The overall heat transfer coefficient : 4* h* hu
boundary, it is
while dealing with the problems of fluid to fluid heat transfer across a metal ,, I
heat transmitted' per unt. u= ...(15.3)
usual to opt ur Jverall heai transfer coefficient U which giues the | I
on each side of the
area-unit time per augr"" lpir:o,ture difference between tie buth fluds "
hohhr
metal. -+-+-
It may be noticed from the above equation that ifthe indiviCual coefcients ffer greatly in
Cold magaitude only a change in the los will have any sigaificant effect on the rate ofheat transfer.
Hot
tluid tluid 16.4.2. Ternperature Distribution
'B'
Since piston cyliniler liner and cylinder head come in diect contact with hot combustion
gases, therefore, these components are subjected to very high temperatures. The temperature
stribution

I
Main
lluid temp,

Refer Fig. 15.2


Let, ho= Heat transfer coefficient from hot QgLd to rnetal surfaca'
hl = Heat transfer coeflicient from metal surfaee to cold fluid' and
& =\ihermal conductivity of metal vall'
as follows
The equations ofheat flow through the fluids and the metal surface are
:

Q ='h"A (t,- t)
q= hA(\-tz) ...(i)
x ,;,., 540-560
A = htA (tz- t) -. .ILLL )
(a) Cylinder liner (b) Exhaust valve
By rearranging the equations (i), (it) and (iii), we get
Fig. 15.3. Temperatue distribution.
_, Q
, -r.=
-a huA
488
INTERNAL COMBUSTTON ENGINE COOLINC
ENGINES {8e
f;i:f."'l#:'il,llH'."i;':T5:"tTffl""l',"i"*.,r."Fi".Is:.15.a (a). rhe remperature iso_
f1i
,i
temperature at differeniexhausr
varve
"t".i*iil"Jji,jit;,i3;1:?l#;:i" '""u","""i
tili
15.4.3. Temperature prolee
o Fig. 1S.4 shows tbLe representative
temperature profiles
ofs.L engine. c
ll across the cylinder barrel
wall O
ilil El
Gas tc = 650.C
til ol
.gv
l{t
;
ts
ii = Mean surface tem_
Psatue of re metal Waterflm
Cylinder plale on tllegro Eide
wall Gas flm
ilil t* = Mea surface tem-
perature ofthe metal Fig. 15.5. Schemtic diagram ofheat traufer,
il',I plate on the coolan
side. The general equation fo heat transfer, using overall heat transfer coefficient, can be writ-
ten as :

fl'r
Q = UA (Ts- T")
where, U = Overall heat transfer coefcient, Wlm2K,
'
= Surface area, m2,
Air-cooled ?c = Gas side temperature, K, and
Water-cooled ' I" = Coolant side temperature, K.
(a) The overall heat transfer coefficient U is given by
(b)

J
Fig. lS.4 Temperature proflles

(tq the Fig. 15.4)


aross cyrinder banel wall. u =-ri--r
_+_+_
t
...(15.4)
i
liffi":r"rtjj1:d th_ar rh,ere.is a very large temperarure rall
in hEkh"
where, , = Heat transfer coeicient on gas side, Wm2K,
iirt'!;#$f il:rffTi?rriJi,",iTf'iti":#ii;i"'""i.# h" = Heat transfer coeflicient on coolant side, kJ/m2K,
- *":1':":lH,::lTr:"lh'i :rih'li-*tu*r,ur". . J' o,, the gas side offers
Z = Thickness ofthe cylinder wall, rn, and
surface areas ay ,ot '! compensated largely by providing ft = Thermal conductivity, WmK.
i"*l;r;:"-" or
The maximum rate of heat transfer to external cooling medium is obtained when the follow-
In case of water cooling ffig.
coolant on the ourside of the
ri.ail tl Javer of tbe tiquid ing requirements are met with :
cylider
the resistance of this b";;;;;:'":""' #f ::.:Y
to to.uldarr
:-
uwrnghish conductity of wa'tel (i) The thickness of the gas film is minimum
s small and as such ;
drop in temperarure occurs. comparatively small (ii) The surface film of oil is minimum
;
(iii) Best conducting materials like aluminium and its alloys are used
tfre same in borh the. cooring ;
::n,nT:*'ffi",|$,'m;Xl::Ti.,:;';:;,larselr
tiallv the same with eitheiiype or n transfer fom the sys-
gases should be ,.,.tr,-
(iu) The wall ofthe cylinder is ofno greater thickness than
strength and wear consideration
sylil ."'*o require
' "ooling

f;l:lll', lln:,".0'*:thebvcvlinder
rorced convecrion or by
;
(u) The external temperature is as low as possible relatively
to the cycle rnean temperature
l lta,"di:lJ?T"lTi?mji
rouded by gas and coolant films, qrq (metal) wall sui- of the cylinder contents ;
oolant side.
,r?""
su ii;',]^:,::.:*t
re varlatron oftemperature (ui) Surface deposits such as those of carbon which deposit on piston
from gas side to crown, corrosion and
scale due to hard water etc. are reduced to minrmum.
r'v,.r.31 :i::;Hi:,:T,:ii:l:n?ffrm (srasnanr gab iaver)
and oir rm 0ubricaring oir

lr
--*Lll
490 ENGINE COOLING
INTERNAL COMBUSTION ENCINES

r5.5. EFFECTS OF OPERATING VARIABI,ES ON ENGINE TIEAT TRANSFER


1. Compreesion retio :
o Fig'15.6showsthat,.in_general,asthecompressianratoit*reoses,thereismarginal
+ I
300
I
reduction in heat rejection.
.e a

>E
J
200

300 o
^ o
I
o
I o
.g
:100
6
o
s 200 I
d
Ia OL
'Io roo
0.6 1.0 1.2 1.4 t.o 1.8

6
o
Relative fuel-air (F/A) ratio --+
Fis.1S.8
The temperature of cylinder gases qnd the flame speed are affected by F/A ratio
67 changes.
Compression ralio -----t At relatiue F/A ratio of 1,12 matimun ean ga.s tenperdture occurs, and maxi-
,num heat rejection tahes place.
Fig. 15.6. Effect ofcompression ratio on heat rejected from the engine.
It is worthnoting that at leaner mixtures more heat is rejected and the engine has a ten-
As the compression ratio is increased the gas temperature near the T,D.C. in- dency to oveheat.
c-reases slightly only. But due to greater expansion ratio, there is
considerabre 3. Ignition timing:
decrease in gas temperature nea the B.D.C. where large cylinder wall
is exposed.
Due to greater expansion the exhaus gas temperature is also reduced to a much When the sparh advance is different from the optimum uolue th heat rejected. to cooling
lower value, and the heat rejected during blowown is also less. system is increased. It is due to the fact that any value of the spark advance other than the
2. Fuel-air ratio : 'minimum spark advance'for best torque will reduce he power output in S,l. engines resuling in
a Air-fuel ratio, inlet temperature and exhaust pres'rre are the only variables rejection ofmore quantity ofheat.
which
appear to influence gas ternperature (t), lt is. illustrated in trig. 15.i. 4. Load and.speedzrCl
In the case of Fl'engines the mass of air inhaled remains same, only fuel supplied is
increased with the change of load. This leads to excessiue ternperature of gases n the

t oo 4- \* 2.0
/ 1.6 1.3
cylind,er with incease in load, In cose of S.I. engines tIE tenperoture aariation is not
much with chnnges n load.
o
I

:. 300 7 -
't.o /
P*
lbar
o Gae temperatures remain ot a higher auerage with increased speed.. lf the load is
constant, the heat input per cycle with the fuel increases with speed, atleast in the
0.7 upper range, because ofincreased friction loss.
F 2oo
There is an increase in terrperature ofpiston with speed.
-,
3 loo 15.6. COOLING AIR AIYD WATER REQUIRTMEIYTS
Fig. 15.9 shows the distribution ofheat loss in for a four-stroke engine with respect to
0 o.2 0.4 0.6 0.8 1.0 1.2 1.4 f .6 1.8 2.0 L / D (stroke / boe) ratio.
Flelalive fuel-air (F/A) ratio -----+ Fig. 15'10 shows the amount ofheat rejected to the coolant for small automotive diesel
aird petrol engines with cast iron and aluminium cylinder heads (as dependent on
Fig. 15.7 speed).
ENCINE COOLINC
f NTERNAL COMBUSTION ENCINES 493
30 I 10 i
\
t
I J
I

3 I
c"rorln"
"nsii''"
I
@
o
20
.E
E
\
o
do E 4-Cycle des el
E IR
:do )(Eetne

x
o
c 4
o o 2-Cycle desel

o
10
E
6 \ <""s5.
o 2
E rr.lroocnar!eo
5 o diesel engine
o 0
0 s0 100 150 200 250 3oo
0.5 1.0 1.5 2.O 2.5 3.0
Ratercl-power, kW ---->
UD (stroke/bore) rato -___) Fig. 15.1 1. Cmling air requirements.
Fig. 15.9. Distribution ofheat_loss
in four_stmke engine. . cooling water requirements. As is evident from Fig. 15.10, the heat rejected to the
coolant greatly depends on the type of the engine. It can be
seen that for a small high speed engine
the heat rejected in the coolant can be as hilh as 1.3 times the
B.p. developed, while fbr an open
chamber engine it is only about 60{,o of the .p. developed.
The quantity ofwater (e.) required for cooling is given by
^
4... =
ZxB.P.
..( 15.5)
where, Lt, = pey1"tble ncrease of temperature of cooting water, and
3 :'i',e"::iJ:iEm -
Z = Constont which depends upon fuel consurnption ond the compression ratio.
The heat flow to water jackets, on an average, is about 4200 kJ/kw-h
for large englnes
'
E 1.2 and 500 to 5700 kJ&W-h for small engines.
o
o
r Large heat stess is avoided ifthe temperature rise is limited
to 10_12.C.
e o The outlet cooling water temperature for various types ofengines
is as follows:
3 vadoud diesot engines eTirres -! For large engines about 50.C
- o.B
rirrel I cyrker
For mediun engines 60 to 65'C
For automobile engtnes ...........,......... g0.C.
I5.7. COOLING SYSTEMS
o.4
There are mainly following two methods/systems of cooling I.C.
2500 3000 3500 engines :

Re\olutions / mn.(r.p.m.) __r 1. Air cooling


2. Waterliquid cooling.
,. Fig. 15.10. Heatrejected byvarious I.C. ensine.
Cooling air requirements : 15.7.1. Air-Cooling System
In this system, heat is carried away by the air flowing over and aound the cyrinder.
' Ii;"lr?*t ;f;!firrffi"JlXX"H requirements ror air-cooled radiator
sysrem or repre_ o Here fins are cast on the cylinder head and cylinder barrel which provide additional
It is obvious from thre,fig11e conductive and radiating surface (See Fig. 1b.1r). The fins are arranged at right
that the gasoline engine requires to cylindcr axis. The number and dimensions should be adequate angles
much more air than to take care ol thc
;':',':*i?,ff $1.*:: :T.'J *;*#; i
". "l
*o
"; ""
o;;;;; ;;i in g, i. surplus heat dissipation. Fom all points ofview, the tuncated
edges, as shown in Fig.
conical fin with roundecl
15.13, accomplish the purposc.
,///.

I
-r-*

ENCINE COOLTNG
4g5
t shows a tvpical well designed air-cooled
cylinder (rhe Bristol ,,MercLry,,s.I.
*t;i"tt]t
It is worth noting,that the fin surface area which is function of fin
spacing decreases down the cylinder height and
"omf."ed
der head cornprising the exhust u"lu"
to the fin rr.f."" the cylin-
"."r-io
"i""" and spaces between the valves.
T\e effectiue aalue coolng requires methods
of conducting the heat,from the
ualue head' to the qtrind'er, imposed guid'"s
and metar ; arso-to tniro-", part of
the value ste\.?perdting uni",
conductivity of the exhaust valve, "oo",
ar conditons. In order to enhance the
r-u
_J_
the practice of making tr," "" .t"- and
the heads hortow.is-becorning popul""'in even
iu"g" n lir*-lt"^ is firled
with sodium' which nelts iz;c ond. boti at"rrgin"*.-rn"
"r with cond.ucting
temperatures, the valve is filled
eso"c. Th".;;;;;" operaring
materiol.
Fig. l5.12.Airmlng Fig. 15.13. Tlunmted coniql fin. Applications. Air cooling system is used in
the following
A fairly good relationship fc 6n proportions is as follows
\' "small engnes" and.engines whose application gives engines :
extreme itnportance to weight
-T
:
such as 'aircraft engines,.
Z = 0.35 D;p=O.10D ; r= 0.018 D ; b =2to 2.5 mm.
wbere L, D, p, r and are fn length, cylinder diameter, fin pitch, rounded edge radius and width
, engnes" whee there can be
::i;"r:::r:i:agricutturat a strons objection to use of
at the lin tip respectively.
Advantages :
Rocker 1' The design ofthe engine becomes simpler
Hollow as no waterjackets are required. The cyrinder
arm lugs
exhausl valve uoa indil,idu.lly detachabre
:il"f,:t:t"ji"S.lu,;1r""r;.". *,u""ro". cheaper ro
",.
2. Absence of cooling pipes, radiator
etc. makes the coohng system simpler.
Valve seal 3. No danger ofcoolant leakage etc.
nng
4' The engine is not subjected to freezing
troubles etc. usually encountered in case
cooled engine. ofwater_
Sparking
dug ring 5' The weight per B.p. ofthe a-coored
Copper engine is less than that ofwate-coored
6' In this case engine is rather a selfcontained engine.
nng
Aluminium e.g. radiator, headers,. tai etc, : unit
----' as it requires no external components
Seclion thru Section
inletvalve \traust alloy head ?. Installation ofai-cooled engines
is easier.
Screw valve S The contror of cooling sJsteh is much easier
thread 146
Shrunk
than in waer-cooled system.
9. An air-cooled engine can
stel rng g: :p
much while a hole in the radiator
some degtee of damage. A broken fin
does not affect
10' Eigh mean cylinder tenperatures
-.y,i"i-. *rter_cooled engine.
Steel mean educed carbon deposits on combustron
ber wall. This gives sustained cham_
barrel p".fo"^-;.".
tt'lHn::*p performance of ""gio"
air-cooled engine is better, this results
in low wear of
Disadvaniages :
1. Their movement is noisy,
2. Non-uniform cooling.
3' The output ofair-cooled engine is less
than that ofa liquid-cooled engine.
4. Maintenance is not easy,
5. Smaller useful compression ratio.
Fig. 15.14. A-cooled cylinder (lhe Bristol "Mercury" S.I. engine).
t erficiencv of an air-cooled engine
ll;#i:n::lc is iowe due ro high cylincrer head
: rr 496 rN rE RNA L cqwlt$dlpEi,rEfryFlN Es aH8iVrff,PldNr8r u'rer' ,:i'-4e7

.I.? "1.rur-lE,.&iW*efd[+gtqlpgitfr-*Eqsngiash IIew tsciq,J n eworta $r.dr .si{ c ir, q,)r slJiJ:eru85+g'tBF.9l"iPgllB{nilor.: rrurr;v,rrr.rdi :,ri.r ;r'cr! (riui iJ j'{.-.. e

;i;il**,ffii$ffigrdi'ffi :::!:y:::{;b:#iTjifi"T,?.*lTff:ljl*reference'i.
.d rr :l $ r{r rroiiom rlciitv cj ;irb ,1"t eisj_ r1 rsoii 'ri: cif I .;i 1o;:, o,i s:j:+
a "-' sdi roi bsbi.ro:q
ti o1 sb zsnig*'r tuqti:o :\gi.d 'e,-or,;:c\: rril- 'r
'\c trs1s-\'rr!$s.. ,r ): n-rilltrr,i
ns,: Lln6 ff3jli}'I{$ilrfiLeolp-&terepbflg#&
grrloor r:olrlq'a. 'io a+i3*iim!,!
srI3 .s,rlev Jt.sd{xe srf}1o.JiviJcnbnos
!:.;li!l ai irrst:(:$ohigh'bojnBpoiu$i:l rri relrqoq gnimocsd i uollo. ebesd .l:srqa s.nigns sdl 1,o tl fLrsqs.rt "{t 10 r\irqr rr\ir.r": +,i-l'fi;
edJ
':nile-.eqr: gr.{}olargllate'Bbddtu$uris16fl.io eilsn res llno JrsJs rlJsw'lo ntii i:iTiur.td-lis vo lc sjr:r or{'i'(i!,)
"{ci.r\, .msrboz
rirl
(iu) Non-cdAbBlgi 31!trljb(o1 riijvr fsifil i *l'ls. crfu ,er:ulsroqrcol ilr,:i'rs.i!( rt .li d,gsiilr:r 1r .:'lrigirl srll
(u) Easilf Ebs{$/e{ellEb-bfri bsu ai msJegz gniioor ri .anoirrcilgqA cj 1sJ't?i'to vrcii rJi"n1'rri! i'i::fl.: ,tti i;cl rrieii!l 3ih )jat-2 i.i!'1' t\i"
i:r.gr.;:x cl gr4|rEss'esoecszeeryianat;js.:,ltr.dpisatudstltsrfraJbd,i,ji{rbclldnff,slsrem. .:.n ilrfr

RoIe of anti-freeze solution in water-coolinCy*t$.}ntqf[pi'&h$ne is kept in gi!-(;:l 0r=


bcan ai qriruq e .mgj?ta I.ai.ICbi[ rs1cfl.g :o isr:r)'{ .S

\+ .ontffitedlarghs Anif is ErttriE)oqtio4rhacthtigeElbenrsF.bbonfrirtS,.Se water in ,scigns erli mod rcvirb ifsd i lrr noiJlIrcr ic sviJi:.r.i<r
the cooling system is liable to get frozen. Tt.e solid'mass of watenls.orytgo (10 percent ?s i1:!1\e"'ii 1.Ji)riri ir-lss i or rriT .jirsM
increase in uolume) may cause fracture of the cylind,er b.tock, water,jgc$6fig1p$jgts. The ra.dia. 11{",1rsqo 10
sol ution are added, la "unter i.tctreme win ter Heat
9frr9lr9 e?rcllgo 9lrllo ffgf9D 9L .-
!"f
"3846?triI{}'i,}','?#'{Bo}#Juieao'nt'reeze
10 etr1' i v' gr::
(i) Denarqred alcohol . . () \,i3dtflJfr?
(tt Gli,'[l'ite't'5rY? :r:troo'r'rtil z'i"r;r "td3Jlx[ft,SBgiq griifrro:iii rsai]- .s
. ,.
(u) susar solurion (riltflSflf'# t$'i'\t"]bitr*lBKaj
Lt'
', tiitIl"li8Ujltoi iiii''loiy1l*{'n;'",til ot b}ta.)lu tuii ai nnn.rr rd

Radiator core

. ubbryiniwns:drti'ofibarbhrA:Wtj 7twd,i luaBe'.irynbfuait f Mreoaedp. ..r


:'iii:: r,i,nileaaffdtdniHiatqFa}por+igsiofifrrol qsrgt ic*b}f"iS*erinetstuck to the
i{,
cold wallsfufi iio0cdnaifqld,{cyrhrderandgfon dpfi&o ftdiry hrqrhaust without
,,: jJ :r l;,,,i,,9r,suS4;ie+hpntioy.2ln!1il1,"vw;m#p"wrq4\+&k\,wp?p.kgnewmrqtion
Lower lank
of fuet,
tnereo! tmproulnc lu!'&fi?99ly(l rJni.']n:) bttir;tzu
L.ti,.t irT.tisvr :s{
i,, ,,i,.N ,+u.!jir,1-9.-s1Pf,rY?Il!b,WopF,i9il,i1,;g,",,4WqktUg;l,dde.r,W1c41gff1nf"sc Hence in
"fi 'modern engnes, hea,tlng of resh charge d,uring idlingis'prouifuflfp water_cooled
. engines thermostat preuents water circulation during *qr"lA&4{rtkhich a7ain
reduces the wann up period,. : arEsJ'lAv ji
:;i ili: j,) rr iii,i.lijl, *,,;,n: IF,lfi,-ll'to,?,Tn:+p,.q:;-lliql",-r8u,S,""y,11,",S.,,,11. i r) ,e:; ?r.,.is'ii !
.eyrii;1ar
Methods used for circulating water around tHoHCiiirgl$ftirdT I Heat is supplied to the fluid in the taik A. ?he hot fluid travdlsiupfits,plac.bsihg:takn up
Vaious methods are used for circulating water aroun8{ihU,tymf,iii"a*lviinder head.
.
..",b$ ep4cP4r-a*ivptv'seld'sgtd,ftoreftls lan$Fitbpueb;Sh- p.Be*. iXbe,bofluid Bows,ttt.ough the
These are nicg brioo:-i;i;pi;r'lo ir;dr crl,j lr:l ::i sniao Svfucr:-:ic tr: Jlg:srr +r{T .t pipe)ffin$1tg qq.aa&:Frhpe it:Ee tqpepl.ediflhuo:tle,fluid auaulates-Ietghrh.esys+em,i the
1. Thermo-syphon cooling .gaee jon i c.lnsr3JrrGl{ .}. ,urr&!a,PlgglbY,gc$ggfBlEtr;'r,r .; ..;: ii, ir. ;(,.: -j .r -i:! r: , -. : ri.:,:' ., ::: r,., r r:,ii;;,., :;; i'j. ::r!
, ..1 ,

2. Forced or pump cooling .i: iis': i:c aaotq rrro:


i I l.r'i.a r r:ll;2 . For engine application tank A.r_presn ts;sylinde6,jashets-,whilq;tirnh.B irdpr:sriiilis,ar tadia .

irr,;ri 'r:ri,irBy-C@lidgrfryi.lbthosabatatiqseguietAr, .iin e'ii ,rnri:iiis rir;rernlo-r srlI . lor, and water acts as a circulating fluid. In order to ensuro that co,blest water is always made
4. Pressurised water cooling .2$lCi.3a9qf 5J available to cylinder jackets, the water jackets are located at a lowellevet than the rodiator.
5. Evapolative cooling.
I
ENGINE COOLING
498 INTERNAL COMBUSTION ENGINES 499
Comparison of thermo-syphon and forced cooling eystems
r Fig. 15.15(p) shows the thermo,syphon cooling arrangement of an engine. The op of :
radiato islconnected to the top ofwaterjacket by a pipe, and the bottom ofradiator to
the bottom of the water jacket. water travels down the radiator across which air is
passed to cqol it. The air flon' can take place due to vehicle motion or a fan can be Cicrlation of watr by a centrifugal pump, belt Cirmlation of water by natual convection. No
driven from the engine. pump is ued.
provided for the purpose.
c Tlte thermo-syphon caling system eannot be used in hgh output engines d,ue to its Cooling is independent oftemperature, but de_ Cooling depends only upon the temperature, and
pends upon the engine speed.
inabIity to meet the requirement of large flow rates of water. is independent ofengine speed.

Limiations of thermo-syphon cooling system Rte ofcirculafion is fast. Rate ofcooling is slow ad insufficient.
:
() The cooling depends only on the tempemture and is ind.epend,ent of thz the circulating water pump needs maintennce.
engine speed. Simple, automatic, and no maintemre is required.
(jj) The rate of circulation is slow and insufficient. The circulation ofwater starts onlv after
the engine has become hot enough to cause thermo-syphon action. Cmling is eroured uder ll conditions of ooera- The circulation ofwater starts only aftr the en-
(iti) This system requires that the radiator be above the engine for graty flow ofwater to tim gine has become hot uoorgh to cause
engine, themosyphon action.
2. Forced or pump cooling. Refer Fig. 15.16. In this system, a pump is used. to cause Radiator pcion with respect to mgine not re_ The systen requires that the radiator be placed
stricted, en be placed anJnrhere.
positive circulation ofwater in the watcrjacket. Usually the pump is belt driven from the engine. above the engine for gravity flow ofwater to en_
gne.
Merit. The main advantage of forced cooling is tbat cooling is ensured unr all cond.itions
of operation, Itis cmtly. It ii cheaper.
It is widelyused. It is not widely used.

3. Thermostat Cooling :
o Too lower cylinder barrel.temperature, may result rn
severe corrosion damage due to
condensation of acids on the barer wat. To avoid such
a situation it is custim.ry rc
use a thermostat G temperature co_ntrolling device) to
stop flow ofcoolant below a pre-
set cylinder barrel ternperature. Most mod*ern cooing
system emproy a thermo_static
device which prevents th.e water in the engine
ack-et irom cir;railg;;o'''gn tl,"
radiator for cooling until its temperature nai react ea to
a value suitabre for effrcient
engine operation.
r Fig. !5.17 shows a systematic diagram ofa thermostaticaty contro'ed
cooling system,
Aleo shown is a tvpicar thermgstai(Fig. rs.re). It
sists of bellows whi are made of
thin copper tubes, partially filled with a-volatile liquid like
e" o" -uit g r"
volatile liquid changes into vapour at the correct working "tt "f."i"1.
temperat""lir-r""uti"g
enough pressure to expand.the bellows. The temperatru
urt *rri"h th. th;;;;;;;
ates is set by the manufacturers and cannot be artered. "p".-
The movement of the bellows
opens the main valve in the ratio oftemperature rise,
increasing o..".t"i*inftll no*
ofwater from engine to the radiator. Hence wh"n ihe normar
temperature ofthe engrne
has been reached the valve opens and ci"cotation
oiwater com.Eences. when the unit
Fie. 16. 16, Forced or pump mling. is closed the gas condensee nd so tn" p"u.s"iu iuil.. m"
u"tto*. un trr"
stops."otiffi
thermostat seats n its seat and circuration u.oo.rd'th".-ostat
I)emerits. The cooling is not temperature dependent. Under certain circumstances, the
nostat valve is not open and the engin-e is-running the water wni"-trr" trr""_
'engitre may get overcooled.
pressure and causes theressure reriefvalve U"i p"_p"j.i", l"
to oien. Thus the *itii"-pl"i* it.
'Qil The cooling requirement, while moving uphill, is increasecl because moe fuel is burnt. circulation through the by-pass as shown in Figs. 15.1?
and 15.1g. Now when the
The coolant circulation, however, is reduced which may lead to overheating ofthe engine. temperature of water around the engine-cylinder
rises upto a certain limit, it causes
(iii) The cooling ceases when the engine stops. This is undesirable since cooling must con- the thermostar valve to open. The p"-"r"o" oi*t"i"i"g pr-p;
reliefvalve closes. so the flow of cooiing wat"r i., irr"
;r;; p"I..u."
tinue till the temperatures ae educed to normal values. oo".r circit co-mences'through
the radiator. This accelerates the rise"of tem;;;;;
of the cylinder walls and water
and more power is developed in a few rno-"ri" ofthe
starting ofthe engine.

)
ENCiEreiJit,ff.ld r_rt Mo) Jrrv fr!i'f i.'rl

i bns noilq'-om:srf! 1o noeirsqrno3 -rcrr ry$x6rypffiWvlfffrrBtuurr)raitt0prnSffipftrytkrmarml,tet*F*itulbfs yi


e9Ya ffi6] utilising the shutter in the radiator in order to restrict the incoming air thid!ettie'iii:
acdqv.aadidi kfndhomg'hF\ilqiblsp0ibffi!Va*sqhttt d.redfdtcfusun3oltha.!
the desired rate of cooling is achieved' : r-nrji:;'3 :liril
ilsd .'arur lrg;ilirbs: c'd -alswlo aoiiskeriS
4. Preesurisdlirtud -ooliHrfua;o I.;;o'I 'JrlcJ lislo Saiir:oi sTiSiact l;: i'ii:-jiill ir)
Thermoslatire rrIJ o:l gvi;b
naibr',ixY$tptr8awffidry.l:gadriyriad,ts5ofa,E"s lfiarss :nolrevo gfrii ;ia: JI (ii)
bnt .eruto:egmoJ ei{I ,srrtsrsgrtrsJlo tsbnsqsbai gdlooC
.hrryt
tte ra*wn&tittalerroaffias'r$nit+rvtdt6eidCa+8flffi?iusbs4llrtvrliiedium,
.bseqa sains srll aoql abcsq
erivorclh ordaod e$cedJed{$Eldgreerotii,i$nibrrddalelftcodq{rdistlfrdider
.Jteiril'lq4 sJsfi bsc.ot oriduae{iredidecrftraiFordptheceo@fingsrtBtfilitslesled*('lb$hsr!i!fuspherre, and
btriux.x ai the system ip allowed to be under certain mount of pressure. Thu$:l,e qdtatltage is
J'fl1' it
zscr!:sialtd'ofdfui fabh]Ftheffipfirotweofit*allrrilllgl.ewtuni*tpdceisrllwr{'rd3$l\ie'in
Air is raised. Ths dltows a greater heat-transfer to occur in the rad,iatorrdim,tots larger
'rr+ erfl refie ylric
sj2Jli:r ot dHsorlg) .utaryssprysdifiqg*ti*$i4* grBryrc.iwt.bts**elpjry.$aKd8{#flry'ttF'!t(Plo 2
bdi are comtnonly use" .r{)at;1c,lrr.3lrrrsrriilo tC i-iJr::'f i1} :;r:i 'rl''1 i: :l
Methad, of prc$uring
19l:;jaI ?O fCJSrbi gfLi -| ,ssqeer dl jw troiiiaoq :oieibs8 . As shown in Fig. 15,19, a cap is fitted on the radiator;igFiig8-+VS"fr& f,u"
-(r3 oJ IrJc's'1o r./cil 5eoIq sd rc.5+isa}a gniloocudrithdslodrikbycd gopesClrr spri4gtand gT*aadrlvs.l{llbeaitbe eid}ant iseold
sdr sr.gtil hdqalfuaddsshrdfFu0A{I*r&cengiireis@wdsor$)the odanlterni:erdfure rises
-'I;!acr ai JI I .'rst:ur kntilitniseheb{srcginprF{vahcao*ptmlqulls aestre resnlihen the
al nrlssafdtynlarc @hslgbuhif herbolgmat'teinpdriaturelfaltsatdnirrg tlnodn$ftddeition the
.$ruvlgbiw aitl
_i oa rglrvaqvcqtiiitcld$ .hgailn untiiqthesbmbtrabulerdlaiilrsistsu$te shuivlbii{rlessure
value,
Fig. 15.17.'Ihermostaticallyontrolledcootirysytiil{'r11 ;t?orrrs{T .t:
{,-i11!ii :r!ctfri:rit :1$'l1it.t.:".!a\ar tt; r'ltrzst 1::.,oTijji),1.:}arf'rl, l,r,rrSrj sl,riti.lV:, rgwol :oT, 3 l.{ -- -----
i vrn.il zi:_ti :t,ile;-J r: lt,; i;io,;i o1'.ifs; is o0 gbit:'lr :oi1i;eagbol
-:'rq r; v,'.:rri 16f".;1j.r ''Il qoj
.,7it9li;ilrilnrfrrrc
r.
,i.trrji,.' ,;;r,rrir r. v-oiriry:s 1J:,.;;.r;iiuo:
[lrfcrsqmtt s)Jri3eorr){J G {,au
J:r' c.;iiuo: r3bgrf-Tffi ri! uFtqitlx Ignr
r3bot-T??iir,lJylr,;c,itt* :qhrilvr
Ig n t :rh;: irr
ilv: Jr
,rtl rl;i,r'r.ij :irrijr,ru:,rii i,i,;tT?S{9Hf
,,,Y1.'IlqF;rf,ltsrrl?]4!3|$rl'ern{rc.,q i}irJp rli-,b
iloo: :o1 rolibar
relif vah/s .t'!oijriq0 $rii:rrJ
. rf JJa!:. :liilrioi i:: jlc:,'r
1llssiJn$ao arror{a l-Ldf .i1iT
ir:r 1+lr:rrr qr r ll.jirilv ewaBJdFs rjf,iir1:r?j ,(8I 'J r: ai n';rorla oalA
;ii'l' ..lrr'{,i:ri; i1r{jeirr ro rer{js r:lij i:ilrrii .ii ,esi{u.t lsqq13 flidJ
1;ir i Jn: I r iiI rlJ .:r rJi:r:llcru*l 3iif'l-rrriv f ;orrr.n rris !:iurii siiisic;,,,
. r,;:.1o l;r:.li:r;rntiiJ ,;lJ rl:rid. J.r; 5.JJ-B.r9q9,i trrtagrq ib'rtr
: rull.;,i :,id.i li; ji;eovox :;il' .bg:gjir lci 'l o:{i vC Js i at:rs
'roJ] :ri; liir;rJro::i) -gti:3-r1rr; ,sai'r 91uit u cr'r gJ,t arl'ijo
:lri.1tl:- :ir.i io sr;;SsisqirreJ Irrcr rh ':rw i1:ns mo;? r+isyi l0
iiir;r lij Jleaa,.r/,J:rr(r' jlo:) i,cdrer c:gC :d
sdi br: :zrJsilor zrclbd ri'L' .flo'i srr;:as: as eril blaoh zi
r1ii Diir nrrilrr' a;Xi,.1:: Jsj?ri'1.r.l ji!ot ;:J$a isiosdJ
rIi r::)).1\1 i'atitr|lt:i :tilr':j l,;j;r':)&9[?f ]FE .Jviav.lriJzrl
il ,r.-illlqi-n:r 'r:r.is. sli ;llr:'l' .tltrlit irl *;vin s) nri g1naBrq o When the engine is switche4,gf[,.egd,,the,gp.qla,SJ,cog,] down vacuum begins to fbrm in
frri {.rtlu/'..,r;.1-l-ii irllr i,'1.d.!. .a7i\ ni rr the cooling system, but when the inter4al'pqessure faJls bqlow a!ryospheric the vacuum
?irlurjr, :ij ,3ii;:ii rr?c"r,'" ;.i r;lrt,r.r ;+i.l ls[Irili,..r:rni]+i-:a .'1D 19j6 ;/ iir tTJ61.lq jl- j.r vatve is opened by the nighul'dulsia'it'i ftd'hd6,tli,s'dt'itiit'tlieh'att"i''.
f t t:i:1:\' i,.I bfl al j::; I hcrn uq lr ri igd I$JL?/ 10 :.f r:e:c[ j
tl":.{1 ?i1
{ i}:}
I r r,l l.lriV .jrj..Jfrtgrij 3{J atmospheric pressure, 1

. A saftti-ilnfeijsr.aiibiphiaibdsil$rHe Etleirxp,m diat if draterripi'.is:uiad'ttlnbffew


rr.r,;;jitwhilb*h;systbmipnilgiiptdssqlb,/th6.ftrsfrmov-egiebofdhe:cap^at:ohcerelbvsthe
pressure and thus prevents the emission of steam or the blowingrof,f':'{he-'cap due to
higher internal pressure.
502 I\.TERNAL COMBUSTION ENGINES ENGINE COOLING

_ Advantages of prcscurised, engine cnring ouer conaentional thermo-sxtphon coor-


3. More even cooling of cylinder barels and heads due to jacketing makes
ing system :
reduce the cylinder head and valve seating temperarure.
it easier to
Following are theoduantog*ofpressurised engine cooling over convenional thermo-syphon
cooling system : 4. In case ofwater-cooled engine installation is not necessary at the front ofmobile
vehi-
cles, aircrafts etc..as the cooling system can be convenientiy located
(j) Effective and positive cooring ofall parts. Local overheating wherever required.
is avoided. This is not possible in case ofair_cooled engines.
(ii) It can take overload easily because as the engine speed increases the
water circulation 5' The size ofthe engine does not involve serious problem as far as design ofcooling
also increases, and same effective cooling can be maintained at a[ the speeds. system
is concerned. rn case.ofair-cooled engines pariicularly in high horsE power range
(lii) In thermo-syphon sysem, the radiator should be kept well above the engine, provide diffi-
to cultr is encountered in circulation of required quantity of aiifor cooling purposes.
a height for natural circulation. lhere is no such requirement for prJsurised forced 6. Volumetric efliciency ofwater-cooled engines is higher ttran that of air-cooled
pump system. engines.
(iu) With pressurised system the coolant'temperature is maintained
Disvantogea:
higher. This reduces 1' This is dependent system in which suppiy ofwater for circulation in thejacket is re-
corrosion.
quired.
(u) smaller coolant passages can be used. Ttris reduces weight
and bulk ofthe engine. 2' Power absorbed by the pump for water circulaion is considerably higher than
(ui) No loss of water by boiling and evaporation. that for
oootng ans.
5, Evaporative cooling : 3. In the event offailure ofcooring system serious darnage may be caused to the
engine.
a In this system, also called steam or vapour cooling, the temperature of the cooling 4. Cmt ofsystem is considerably high.
water is allowed to reach a ternperature of 100'C. This metho ofcooling utitises thi 5' System requiree considerable attention for the rnaintenance ofvarious parts ofsystem.
high latent heat of vopourisation of water to obtain coolin uith miniium of water. 6. The performance is rreather sensitive.
Fig' 15.20 shows such a system. The cooling circuit is srrch that cooldnt is olways ?' Ihe warm up performance is poor and has siarting problems particularly
liquid but the steam formed, is flashed off in tle separate uesser. The make up waer so in cold r'eather.
formed is sent back for cooling. r5.8. COMPONEAITS OF WATER COOLING SYSTEM
o This system is used for cooling of many types of indzs rial engines. The components of water-cooling system are enumerated briefly described
below :
1. \ilaterjacket
2, Water pump
3. Fan
4. Themostat
5. Connecting loses
6. Raators
7. Radiator cap (pressurised).
1. \f,aterjacket:
r The cmling system starts with the water passages, parts and jackets
which are usu:
ally cast ino cylinder blocks and heads in the manufacturing process. Fig.
15.21.
shows a section ofa side-valve engine with arrows indicating heat irom
the combustion
chamber being transferred (conducted) into the cooling water pessages.
water is universally used ag the coolant in the automobires. onry crean
soft water
should be sed. Hard water contains rninerars which forms a scale on
the in;ide;ur-
faces ofthe cooling system, reducing its efficiency. Incubators
are auaabre to red,uce
or prevent the formation of scale and rust and. should. always be used,.as
the coolant.
Above or below a temierature of4'c, water expands. In the liquid
state this expansion
can be taken up by the movement of the water into a vacant space,
including the addi-
Advantages and Lisadvantages of liquid cooling tional volume.created by the expansion ofthe cyrinder block, when
: o""ol the
Advantages : mass erpands and exerts a force against the internal surfaces of "otid
the cooling system.
1. compact design ofengine with appreciably smaller fontal aea is possible. The result could be a fractured cylinder block or damaged radiator,
although irr" t"tt""
2. The fuel consumption ofhigh compression liquid-cooled engine is rather lower than for is less likely, due to the mass of water involved, aniengine cracks will
be revealed
air-cooled one. when turns back to a,liquid. Antifreeze lowers the
freezing point of the water a,nd.

i

564 tOa
;

TNTERNAL coMBUsr{ltlESoEsD14s
1 erdrittl C66iirfir.lof .t .v ttttv.l 505d0t
; oJ rsiese Ji as;an gniJsl:af oj erb bssd bns :sbnilrr'lo gnilooc nsve erolr'I .t i
.erir,rrgq:rgJ Bfi ij,e9 ba bssd rsbnilgc slJ ecube:
-!r{sv slidom }o Jnor} sdj Jr rrsascsa Jor ei noija soigns bslooc-rsJwlo ser nI .$ elluo roiffbber
-be'riirpsr rs's:erlv hs.io:rol Xlineinsvroc sd er ailoos sdj s .cJs efiers:iB .ael 6qtq
sfdiaaoq foo zi eidT bsolc'
rrsJala gtif ooc 1o oieeb
sdjlo ssia srfT .d
-lib sgnsr rswoq ,"J[]&gr<
i
nI .bgfisooc ei
.aseoqruq gnilooo
I

I
ai rJluc
zscigns bslooc-1ie lo rirJsmuloV .
i Waler sal
: esgntnoulneiG
etr ai Jsdcsi srfJ ai
t,
ei idT .I Shaft
,bs:igp
rol JdJ nsdl rsrlgirl gldr:rsbiznoc rsYro9 .S
.znsi gnifoor
li. roics edJ oJ ]o Jnsvs edi nI .8
ti; msJala 1o Jao3 .$
.msJara 1o alrsq auoirav }d JcsflrrrC
as:iupsr mslag? .d
bns Jasnrcvom sninc gvcor ro1 a
Ili' srII .d
xsli .noiJnrdiv
.rsdJssw bfoc ni qumrewsrlT.l
'hel
-saor{ nsbrsg rsJsnrsib dJiw Iscrrfi,risr , rllsrfuiro gr:,w ysdT o
lrl' w 10 aT:4gvofltrloo aoqsd,e +di o.i b::blromcrq uoilv bne onsrqoch
ii,
.8.;lt
nci}llf rl.i:;;r i rr ll.l :rit Se ri rrpr,r agv ur brr n
woled bcdi C i#arg 'n1o aloenoqrrol cdT
. 1

lnl;1:':r hes roJeiltrr trlJ ri, r;j'..rliq:Fl6il6lqh,Wet$pii.p.ltt:r,it :.rr: :orf :rf.i'lo rhnl lrl'f r
Fig. rs.2r.waterjac&et. iil,"J ]illW .i 3. Fan:
.erii:r ;ulirr,,'r, rji:;l'r,rlrrri 1if *r:ir ni i:rfrri

gives a margin of safety in cold condition. lte proportion of antifreeze torxst for
various degree ofprotection is usually quoted by the manufacturere. fggfofugg.glyco.l
bosed antifreeze combined with chemicals to reist corroson t+EflpEpO+*lnfr
.l
2. \[ater pump : aoJiibS .i)
r Water pumps entail the following merits d,nd functiabphueasrq) qs:r oJsibs8 .t
1

I
(j) Enables more efficient circulation and the radiator can be placeidj#plmlOWrlt.
-'r g'rsu&idhdodg{tftmf utrsrqehh{6,ocGstydffi}ont.
I
j&. .
'rs'Pt pift +diciehed'eeIrrd6*leusiqde"or.dic
itccric trfi.bia+bfiane ",r
e.cio"
Itoi.r*ud$[htldlqnXldxj.nsi$,he3qreauarrflittodiu *rrta*etbi slo ,,or"", o u.rr. efficiency of the cooling system and the dynamo output will 6"'gf;[{gs nil t tii
il o warer pffiffef1$3Blvib.xaehpdeiup"c!,rs{jBfl'$4osaha'rird ?sgfi#om the o On no account should the engine crankshaft be rotated by the fdi{dfus%tihis will
ruirru {enJrlbf\}dileypthbdsgfre o{,tirrcrdalinae&Edlr.|."'rgttbpGotiorie torffi engine .i?ri:)ia,utdBt!ttbftbr0b,Stj&saadrrdesli{blt,tnflbthlefti$f.eeruUlr }S.dI .!'{
-rua sb'&*cdt no sjsc e rrro1 tltirlv elstsnim eiJno relsw brsH .bsr st\ tlo\ l,r:r;-rir+9df oJ fioilsiii:r qci br;lrrur ai Ji r:;e ral;,. r rrrjui'-,i,r :irlfrj(rrii C-luortil Lns'rztbi:tr,w *i :,1.
:sh llhelfohellert+rgrlfwJuidblxspaiiratiegguAat,Ooald gdtiqeileddb' lb vanes rrtr lr,il'flffig.!?[rr, rA .oi]:'cnr':: J']cf r-:*'.,c{ crJJ br :{rr,J nirrl:lr.ti rrll iv cr.ri..lf) r.}rlj oJ znr!J.r'r
r -.loorr"jltto&trittleeirahc6}odhe Mrdttbmd:troucdcnd\ovt o The thermostat ie normally fitted either to the cylinder head.patatrod0let{pwsd:ge$eitli}
.
tr,suo.q .o
noi:rr;erfienfi{fir9.*hJge-F9m0srdreEns$pscg&FdoreuiCeryai{ip.61g6rj$&Wory the . r.ii ri i l'loxrh-er,qp,bcFg.oudnber.6.ppgiFJinf4.s, ngatke'ttaerq4 pq,tF;{Lr:.r \o s:{.sr!:rin\L
itri,r; :rfi{lt$l$qTips'f4pefirsed&-s e9}rlreaf.rtr*etfrcssd bilpedb.-ppgrgi0erC+ i ttr" rr*,J: l-Sherpqiboerf rliod&ipe js iflrirfldry Lbotestic n"e; ahelhtlOar bf'lleaft**o,Eiadcr':,v
biiri; srQf{l{E*rYFP$'fl''o.ffin$qtrftBtqsgHtqdif,ttxkbehypW$fi$iinft'fliqg[61ge parts 'ioul .tteadran&,lookod*riiEltlisrerafir.rrip4iodi.flhe*ryher,rddbigndf tginpifIid,iB':rlj
rnu,,r'r9$l6dP9?8'lo ,cclrr; InrsJi edi Janisps roi n etrgxo bn bncJx: rrn riir,;rr.ebhptrrtheenosta'leosrh.il$hu sf?cu]stidn'$f tratei.td]f,g ;ptfid$dietir] .i?r1,al ,

''o.;rlt sFi&{fl&s4ets'ev@rngpn,eF.f,emphLyrteee*ntveen i#liffiyiiqf'a u.o. bn'r5';'r'1!.1tsat.litufue tcfrtfi:,,


br'irs'zc{^e&IollsFffi Ifl}9r0ghIP$lf&Fie{&giltBettpfit]nBrgnffiEgsgyetreating 'uhwqfi'rligt6d{,'i 1ir'r&Fl?.,.iriCdjEhhli?doti fi
rir! .nil6ifurys/urrmm.brubru*srgtriadary@is'},*dl'atrdstrli'uffbii
r.i'crsscffi sff iidii t ?ii,',.
t$tf{ffi mtr#tb#p$*:WB4*ii.i|$'d*HP.tlr*T*'; pr.Caateithmatfn$eldlii.i?i{ggll:ruin,;r ia'.:;,r:rr,;''ri g;{l c'1,,,;ris:r-;jniscj (,ii :r:, .,
casing scraped to remove scale.
]-_-
INTERNAL COI\4BUSTION ENOTNES
ENGINE COOLING
To radiator 507
Watr outlet
PiP Thrmostat
dosod
Thermostat
opn Gylinder
head

Thrmostat
To watr pump

Fig. 15.23. ThemcaL

5. Connecting hoeee:
o For convenience of conDection and to isolate the radiator from engine movement and
vibration, flexible hoses ae used.
. They were originally ofrubber, reinforced with canvas like large diameter garden hose-
Neoprene and various plastics are now more suitable and are premoulded to the shapes
and curves required for the installation,
o The ends of the bose ae forced over the connecting spigos on the radiator and engine Fig. 1624. Radiator.
held in place by bore clamps or spring clips. 7. Radiator cap (pressurieed) :
6. Radiators: a The greater the pressure acting on the su-
o The basic requirement ofa radiator is to prouide a. sufficientl! large cooling area
for
fa_ce of a liquid, the higher the temperature
at
transrnission of heat from the calont to the air. The construction of the centre of the which the liquid boils. Ifthe cooling system is
sealed of[, the expanding water will irr"reas
radiator or core varies, but in general the water passages terninat at a header tank at
the pressure of air in the header tank ; any
the bottom. In addiion to an opening which enables the cooling system to be topped up
steam formed will assist in raising the pres_
the header tank allows for expansion and contraction ofthe coolat within the svstem.
sure, and in consequence will require a higher
o The principal ypes ofradiator core ae : temperature to cause the water to boil, which
(i) Film type means ihat any ard.uous driuing, wi not
(jj) Fin and tube result in woter loss d.ue to euapo:ration.
(lii) Pack type. . Fig. 15.24 shows a radiator pressure cap. A
. Fig. 15'24 shows a radiator made ofthin sheet brass and is typical ofmost cooling systems.
As the water descends through hundreds oftubes or passage it is cood by radiation to the air and
returns to the engine via tbe bottom tank ad the lower Ieft connection. An oyerflow pipe fom the
filter neck can be clearly
;xg'f:l';,iil:,T;;rfl"il :l:
movedduetohotwatera"".-.eji!
moved due to hot water a ste
..i^,^_^^^.-__
under from rL,
pressure 4_^_ ,.'} Tt"C.elfbq
the radiator.
:tr
-
fn a'a&tion)
""*,'ffixr*
vow
tr
spring spring
seen. a small lightlyJoaded valve operating in re_ Fig. 15.25. Radiatorpessrue cap.
Mointenanclof wdiator. Periodically the radi-to should be flushed. This in- verse direction to the main valve prevents a vacuum being
- reuerse
volves removing the rbdiator hoses ad thermostat and flushing up through the radiatot andd.own cools down, so preventing collapsd or damage to ir"
created in the system as it
raiJo" tr* ;"i"".ti"g'rr"ir.
9o hW"petrot
through the engine block. In othe words flushing in the reverse directibn to normal water flow. For
'or
reverse cases a solution of 0.45 kg. ofwashing soda to 4.5 litres ofwater can be used in the coong oh uato ;. in;-^) i- ^----,-.--
"riii*7 "rrtr""
i7!!:"*":*:::,'::*':!:::".:"'::rb;;i";;;;';4;;;;;;e";";;;";^";;::;:l';::
,k::::::,:f_:!_:":lul
system, but the system must be fll$hed after the solution has been drained off, and this method soins n .coototi, n';;,*;;Z;;';f;t;;;7;;;;:;;i\::::,":q;:,
should not be used for aluminium blocke. Choked core leads to bad circulation and overheating, the oni t"r"f
efficiencizs of peirot "" ls-&";;;;;; ;:;ri:;;'r;:;;;i:;.
latter also being caused by the air passages becoming blockett with dit and dead insecrs. Solution. Giuen : B.P. ="nsi".i"
90 kW i N, = 27"C i rl*6= 2|%,looa S0%
=
Petrol engine !
Work developed
Efliciency, n' _
Heat suppliecl

I
r0508 r NTERNA L cQM9Jd5oNhlsrtsN Es
3 /EN6INE(6@@i8lglf ,lAy1tflr,ll 0 i 509

ork qn
J360 kW or kJ/s & As the compimmraiul*,irrea*4if$i lmloatrftnssDtihq$B$tinnlc$s*licl$rpnll0
o.25
bnc esioe,{rfDtirrdi{ffe*itqp&yndbrgasaailfiatsesA*F gt{ftiaffqvptltgd}reti ebfl4FRfwoH '!I
Eneigi')heat going to 0.32 = tt5.2 kJ/" 10. At retative fuel-air ratio of 1.12 maximum mem gas temperature @ors, arrildlirgnin{pat rejetion
Also, xN. takesplace, .meiaregnilm-:ialoasgeJaavbeaibbnsasgsJasvbn,unoirsi{qqnsrbsrnii.Sr
(where, m, = mass of cooling 11. when rhe spuk advance is=different from t\sssirysggyelcglbi'tffFf{t?ggd{e*g}4ff""}F """ '
heat of wate at constant pressure)
r asr{i;lro.{actc}rg,qtmM$ssq*rt*wlrfi**[flpr,iggiq,sr.sgCags{'6qfedig4pu .
= 1.019 kg/s or 3668 k/h. (Ars.) 1& The ....... arsnwpdetdcrggBlsb&yhd*d$eiul*ic l"rol srIJ drlqila s [liw nietqrs .i
14 Instaltatio ofai4@lqdd{UtftFi$stqispiflo asgdruvboaib bon eseJnhvbs g{J e:s rari1ff .0I
1 Ai-cooled e"gieeptfSA9.iloo:1o borltsrrr lnilrc3 telsomsb' dij3{ taetr s rftiv triafqxs 'tI
lG The output of air-cag|9$rggj.1qrtusorfhpdhgtdt6{9&f;991t ggtilg Jsec e dii'r edii:g i
17........mixtuesorsolutioroaeadd"tq-ry&Ck3$4?BefglSf'fpnlelg#ih{F,f$ry}mffatures
belowthedangerpoint. unifoo:rslswbshssrq (i)
1& The......molingsystmcannotbewedinhighoutputenginesdetoif,,11,f$lily,Jkq""9ttltiilfqurement
---''",,"dj"f1lt?i6#j.{i3ffi;.i1{3#tisff fi.n#lt,dgJ,frF8srff*|l,211,-:shrBqn#$e4ffiflgf thens:ine
or 2874.9 kglh. (Ans.)

zo. ilfil-......;t'iflyig?fr18iHslSflff"trffi"'rafig{t"Sbb"e.r-if,3X"Tg+rf""B$r#.qY,!.&1tqqlw"'4t"c'"".
21. In....... mlingthere isnoloss
22. ....... cooling utilises thehi{lila'th1terrnap-ori56ti"r.T'9itl,b".T:iy":iSj*flgiiiEi'Hffi .nt
l. Almost zo
amoa 25 fo
to ro'/o
35% ooftotal
otar heat supplied Io
near supprred fuel is-F!ffifvbd by the cooling medim.
in the tuel 2& Evaporative ooling system-is uqed-for
z 24. In a water-coolirg ry"t"- -fiS3Pflifo1 f"S#ttotrooti"g
lJ#:i"Tf$ii##l:i:::ffi?LH#:B{tr#fi1*1X}:*:.*:*""r*T"".::rsandtheensine fam. .mgJct tflialqrG .Af
& spark advance is tlirerent from the zzurzio 'ts
avstem is 25.s1tffiffSft'tnrog$JtfWt-+AEt8ffilgts9.f$.qilhtrrifiRfrAh.{tl6Eii$fi&f-i
m""*: "pti"l$B-Iql98at,fdlqslnrq##*gfli?c !lrcr{ailqnn:nliairvolilnrs.ta,'1:,[r:loo:-birp.ifrrairlaarrqhraoLruqerl!iisrflr.$3
4 In cir-cmJrng sysrem, hear is mie*efiala#ffie8H1flf,46{ffis*u?ff}iBEyEdi$r. ?lere fns are ln.,rs: ni sbsn arroi,i',r,:q ';Ci pacreifi f lnroncl le|@@Qtryipirr;q qu-nrtrt io l:r'llg rij ", lfi C:l
head and cylindetmclnrrnodiru*e"*ugn, arnA"r;;*. I ANSWERA,'trcr! ir!-nrrAv/ q rihgr ol nietts .l
:":p$flpq' m of cooling enii@*Aftffhiqn&Urdiie*bie ."1!tit,'tt,tCoryto'tbcvolrtr-,imgtz.,rgrri{oor'iolroi,'sr{i"*Jasnfurs.J.}rtll,tuaor--,rlgrrioiili?d;V.O*
iJtrCryf*, jacket through
gcancirnlatosc:rai ilivr re,:e w gn.ibneqxs sdJ ,Tlo bstsea f. ijf,i56ficili:r rt;rqrif3cltnarI rdl oo bgiqprr': f:,1:"" i:s.rylg er Jrilifi f a:li'-lnr&rYtb
waarbrd*rs ehn&p&ni*alBsqqasrq crJl 6 No 7, Yes & Yes 9. No lO Yes
-eerq edJ prriaie,r ai Jie iliw hcrrrol m:j Fins 15. Yes
11. inreased lZ air f3. 14. Yes
rerlgid n ericpsr lliw slneupsano i bns .o.rs thermo-sphon
16. less l?, Antifteeze 1& thermo-syphon 19. temperature 2O.
ffimostaticregul{ir{w ,liod oi isJsv sdJ slrric ol er;,lsrsqins 21. pressurised 22. Evaporative 23.Yes 2' No 2l, higher.
watercooling tos \liu! ,gsriuirb soubro 1nc ler$ aoesra
.Jtoit1oqnu$ rt ssb ol r:o, ni tlt:r
A .qsl r'riiee1q rolcibsr i: wora !S.'j.t .pi'{ c THEORETICAL QUTSTIONS
i r::i'gb vJllr:;'r
.,iair gd3 sri 1. Why is cooling necessary for I.C. engines ?
2. Why overheating and overcooling of LC. engines is harmful ?
s. State the demerits of overcooling and udercooling.
is}sehiinei.rn'*r*"i;;rEFi,ffi;iH"il" ourpur orrhe 4 Discuss briefly the areas ofheat flow in engines.
i e.r* :1",ir__{'fqg{qddssriBt8{d$*"tue
rnsJagng!ii..bajserr gaisd mrc:rav e alcoverq s.lv rie:n ol I Expiain briefly gas tempeature veriations for a 4-stroke cycle engine.
.asaodani*lnrd988daqereridtotrldDwHidtr Ereift#r,iscioredsryle ",r "*iiiui-L1o"
qodrnlnldium. 6 Explain briefly with a neat sketctres the representative tempeature profiles across the cylinder banel wall
'a:riss$se.Hatnarld'qci{B'SdaF'ldoi+aa&beada$ btiadisqckmuEf
0}0A6Pi-rfe*s{he total heat ofS.L engine.
r:ilsa,rs !o'ril\lP. Dedits*ro!.sr\i rlilsori.i g*iearq s: f'!3
xd sillic,isu::j! si! ;;r?,:l-i a! rsin,.ri i-rii,: sr Explain the eflects ofoperuting variables on engine heat transfer :
sstT .irrr&g4teasri{trt!lhedoielkderqli.t iirrl,rt ri gnlr.g qi5ro* (i)
o-, sX,;t**,:,..,ir .,\\ Compressionratio
I AtwlyJitrqryeluhrepg:iltiJgda"dro.rrcrres.diGritFe !"rib (,i* !rq:1., (ii) Fuel-air ratio
6, Undelpg@iegllg,Fniv&eqei,lf i O.i'S ".ri,,,,,.i,{,
= .,,V: ; Wi Ce = .t.J : r:ril -oii:le (iii) Ignitiontiming
" 3""il:::1,.ij*:Tl".f.:::"f:::l.cx.":1h^"-P1r.r.1:mittederunil,asisi$ri{peffidecree
perature difference between the bulk fluids or.Qgrtlsi{pJggqietot
tem. & Describe briefly 'cooling air' and 'cooiing water'requirements for I.C. engines.
o What is the lilm coefiicient ? On what factors the film coefhcient generally depends ?
5io
TNTERNAL coMBusrroN ENcrNEs
10. What are the two main tpes of cooling systeme
? Where tese sFtem are used ?
ll' How engines are ai-cooled ? what is the purpose ofthe fim
in m i-cooled system ? what is the size and
spacing of fms ?
12. Stte the appcations, allvmtagea and sadvanags ofair_coolingsysten
13. Name the various types ofliquid cooling systerns.
14' How is the circulation ammplished in a themo*yphon s)8tem
? what is the draw back of this system ?
15. Explain with a sketch the forced circulatiorr sysem. Stat ts merits
l& What are the advmtages and sadvantages ofquid_oolingsystem
aad demerits.
16
17. Explain with a neat sketch.themmtat cooling, method of cooting ?
I.C. engines.
Supercharging of I.C, Eniines
l& Dessribe with a neat skecl the corotructio ad worAig
of a thcmetat-
19, Explain briefly the following methods ofwater coolings 16.1. Purpce of superarging.,l6.2. Supercharging
of S.I. engine*Naturally aspiratecl cycle
(i) Pressurised water ooling of operution-Supercharged cvcle or op"iatio"ioifJ"-oil?
actul mturally aspiratcd md
(ii) Evaporative cooling. supercharged engine pressu*vorm" airgtu**aoili*"i."
ofpressure ratio on air charee temperatue_Ihermodyoa_i"
*a p"o"ue ratieThe effect
t' *" operation of the themostat what Superctrarging Limirs of S.I. -engin. 16.3.
ura supercharging power_
l;"ffi5
? is the mar advantage of using a themosiat in the cooling S"p"d;;;;;;i;.i.
"ycf"
engines_supercharging limits
of c'I' engines' tG'4. Modification or a" gi";lli-""oJt.it"rst"g. 16.5. superchargers.
21. What is e advantage ofpresurised cooling ? How pressurisiag SpercharSinq*-C"-*F:-tO Z. TfrUocargeeli-t"rJo"t*_ttitude comperoation_
is acconplished ? 1.9.6.
2. What is evaporative cooling ?
2& Compare the merits and demerite of air md water coolilg syctems. H*I#:},'*HiI*A;J:R1! "rt*b".h";"s-ri;;ons orturbocharging. worked
Exampres-Highrights--objecrivelp"e"*ti";ih;*;lJ;"ii"i,i:,triilffi;illiij"l
24, What is the function ofa raator ?
25. l{hat are the variou trpe of radiators ? Explain any one in detail.
26. Explain the role anti-freeze solutions in water_cooling systm. 16.T. PURPOSE OF SI,JPERCIIARCING
2?' Disms the advantages ofprssurised engine cooring wer convetional
hermo-syphon cooling. of superchargi ngis to raise the oolwnetric effrcbnqr
2& what is the purrcse ofpressurising a liquicl*ooled system ?
How is it ammplished ?
, obldinedly normal
be
aboue that ualue which can
29' what is the effect of warn-up period on S.I. engine fuel eonomy -Th".|utpose aspiration.
S.L engines to reduce warm-up period.
? Discus the provisions made in recent o the engine is an air,pumD. Increasing the air consumption permits greater
fuel to be added, and res ults in a grea*ter potential quantities of
3o' whyiscoolin-grecesaryforlc'engines?whatkindofcoolingsystemisemployedformobileunitslike o"ip"t.fn"r"arzlia lo*r, proau""a
automobilee ? what are the effects of load and speed is almoqt d'irectly proportianal to ih.e engiie air eonsumptian.While
on the rreal oss thn"*-"y""i *"ir" r brake power is not so
closelv relared to air consunption, it is
congumed' It is desiable, then, tht ""u";h;l"r;,-a;.n".i|;if*-u"" or,i.
rhe engine tahcs in greatest possibre
mass of
air.
o Three possible methods which might be udlized to increase the air
consumption of an
bngine are :
L' Increasing the piston dsplacemez, but this increases the size and
weight ofthe engine,
and introd.uce s add.itional c ooling'proble
ms.
2' Running the engtw higher speeds, which results in increased
.at fluid and mechanbal
ftiction lospes, and. imposes griater iirti stresses on engitte parts.
3' rncreasing the d'ensity of the chorge, such that o greote, mass of
ch.arge is intro-
d,uced nto the same uolum or sam toltai jiston
displatement.
The los mthod of incteasiug the air capacity
supercharglng. ofan engiire is widely used,, andis terned

Supercharger 3

o Tlu apparatus used. to increase the air density is known


as.a tuperchanger, It is merery
a oornpressor wht:l^1":]]g:" a denser
sumption of a greater mass of charge with +g," ,. rhe engine, tt"*y li.rug the con_
ihe oame total piston aisplacernent. During
th process of compressing the .har-ge,th"
prduces the forowittg effects :
(i) Provides better mi.ring of.-the "up"rchcrger
air-fuer The turburent effect created by the
^Ltuo.
supercharger assists in additionai mixing of trr-" rrr
ment of cerrain rypes of superch.rg".", p*i ""a "r" f;i;r:',ih. a*ange_
age8moreevendistributionofthecharietothecylinders."
^;l;;;;";;riftrg;;;;;",'"tso "r,"or._
5il

I
I
lllt Gi) The temperat.ure of the charge is raised as i.t is compressed,,
perdture within the wljyderx This is partially
resurting in a higher tem- 16.2. SI,'PERCUARGING OF S.I. ENGINES
beneficial i" tl"i il"rp" b produce
better vapourisation offuel (in case ofS.I. engines) but
deterimental in that it tends
The schematic arangement for supercharging S.I. engine is shown i Fig. 16.1.
to lessen the density of the charge. the incrase in temperatuie
or lie crra"ge also Fuel
irffects the detonation ofthe fuel.
Superchargng.tends to increase the possbility of detonatinn I
in a S.I. engrw and lessen
the possibilty ii o C.L engine.
,l
(iii) Power is required to drive the su-percharger.
i Ttris is usually taken from the engine
*toves, from over_all engine output, som" of t gain in pwer obtained
l
f_11:1:b"
through supercharging. Fig. 16.1
The theoretical operratiirg cycles for both the natural aspirated and supercharged petrol engines
Conpressors used are ofthe following three types :
can be compared on preasure-volume (p.q) diagrams as shown in Fig. 16.2 and 16.3 respectively. ?e
(i) Positive disprace.m'ent tlpeused with many
reciprocating engines in stationary prants, Iarger upper lap is a measure of thc positiue power d,eveloped in tte ethder whiJe the lower loop
vehicles and maine installtions.
Iill Q Asial flou twe serdom used to supercharge reciprocating
represents thz negatiue power needed to fill the cytind,er wth fresh charge.
.-
the compressor unit ofthegas urbines.
engines, it is widery used as 16.2.1. Naturally Aspirated Cycle of Operation. Refer Fig. 16.2.
(iii) Ce^"tol type wi-dely-zsed as the supercharger
compressor for gas turbines. It is almost exclusiveiy for reciprocating -rfin-.u.ip"o"u6.rg
engincs, as weII as
used s tl" r"p"r"tr"rg"?
power plants for aircraft because it is reratiuerx tight
ond compa,Lr l"a*", *ntinuou" tow
rather than pulsating flow as in some positiue iispl*"int
typu.
o Acoftectll matched, supercharger will raise the cylind.er's brake mean effecive pressure i
I
A
(b m'e'p') to well aboue thot ofa naturally I
ospirated. engine without creairng excessiuely E E
high peak cylindcr pressures ; the actualinciease in thl o
brake mean pressre s_ a
busically determined by the leuel of boost pressure the superchargga ffiactive o
deliuer.
q,!i^i a""igr"a to o @
E
cl
o Large commercar uehre dieser engines are r)
frequeniry turbocharged, with the objectues
of roising b'm.e.p' (and' therefore torque and power output) =
andatlhe sometime reducing =
the engine's marimum. slt.eed. other benefits of ,"*i"g' th" ;oo-pressure
o
.The
dec-reasing the engine's limting speed s tht the "yii"i", losses and, noiseand.
engine mechanical are 0
red,uced and there is an improvement in 0
fuel consumptinn, normally, an added. bonus of .
prolonged ergine lfe expectanqr.,

object of supercharging. The objects of supercharging include oneomoreofthe T.D.C.


-+
B.D.C. T.D,C B.D.C.
Swspl volume Swpl volume --->
following:
L. To increase the power output for a given weight and, bulk Fig. 16.2. fteorecl naturally irspiratd Fig. f6.3. Theoretical supefcharged petrol
ofthe engine.......... relatesto
li; aircraft, ma.rin and automotive engines. petrol engiae (constnt volume)
presswvolme diagrm.
engine (constant volume) prresure
volme diagram.
1l 2, To compensate for the loss of power due to altitute ........., Relates to aircraft ond other o {he lorge areo enclosed in the upper loop (2-8-4-5-6-2) i s proportional to the useful work
engines which are used at
high attitudes.
perforrned by combuetion on the.pieton whereas thesmall lap area (0-L-2-7-O), which is
3. To obtain more power from an existing engine, below the atmospheric line, is a measure of the work d.one in inducing the fresh charge
i
Effects of supercharging on performance of the engine : into the qlndcn
l
I' The'power output' of a supercharged engine is higher than its naturally aspirated o The fur phcses of the naturally aspirated engine's cycle are represented as follows :

terpat.
coun- Ind.uction .. 0to 1
2' The'mechanical efficiencies' ofsupercharged en gnes are Compression .. I to 3
slightly elr than the naturally Power .. 3, 4 and 6
aspirated engines.
3' Inspite ofbetter mixing and combustion due to reduced delay
Exhaust .. 5,6 and 7
a mechanically supercharged 16^2.2. Supercharged Cycle of
otto engine armost always have tp ecific Operation. Refer Fig. 16.8.
fuer consumption' highcr than a natiralry aspirited
erryine. a With this pressurised charged system the large upper loop orea (l-2-S-4-L) represents a
r measrre of the work done in moving the piston to nd fro so that the crankshaft'rotates.

r
514 TNTERNAL coMtsust.toN ENcTNES

Conversely, the small loop area (0-1-5-6-0) rich is aboue the atinospheric line represents SUPERCHARGING OF I.C. ENGINES
the worh done in purnping the fresh chorge nto the cylinder.
o "ftte four pases of the supercharged engine's cycle of operation are represented as . Pressure ratio is t}nerotio ofabsolute pressure to that ofthe atmospherc pressure
follows :
(Here, absolute pressure = boost pressure + atmospheric pressure)
Induction 0to1 "lhe ntensty of superchorgin.g can be broadly classified as follouls ;
Compression Ito2
Degree of charging
Power 2,3 and 4
Exhaust 4,5 and 6. S. No. Particulars Natually aspirated Iw Medium High
16.2.3. Comparison of Actuel Naturally Aspirated and Supercharged Engine Boost pressure 0.0 md below 0.0 0.5 0.5 1.0 1.0 md above
Pressure-Volume Diagrams (bar) - -
Fig. 16.4 shows a direct comparison ofhow the actual clinder prcssure varies reiative to the
cylinder's swept volume for both a naturally aspirated and asupercharging petrol engine uith wid,e-
2. Pressure ratio 1.0 md below 1.0-1.5:l 1.5-2.0:1 2.0 and above
open throttle.
16,2.5, The Effect ofPressure Raio on Air Charge Temperature
The relationship between supercharger pressure ratio increase and air charge temperature
Supercharged is shown in Fig. 1.6.5, where it can be seen that asthe boost pressure ncreases, so does the discharge
+
air temperature. The lower curve shows the theoretical temperature rise with increased pressure
150 Nalurally aspirated
cd ratig, whereas the band between the lower and upper curves is the worhing, ternperature uariaton
940 l\ Iikely to be encountered, due to the turbulence a,nd frintion resstdnce generated, by the compression
E
t process.
f t
Eso \
E

zo
\
. \
:i*l:-:?9?!
5O0O revlmin
\ \
" 3litre V6
o10
H ^120
I

gr*
E
980
E
T'o'c' B'D'c' 8.
s*"o, uolume --+ Euo
Fig. 16.4. Comparison of actual naturally spirated and supercharged o
petrol engine piessure-volme agrams.
I
40
-E
The two enclosed loops show thAl! the lower compression stroke part of the cuwes .:
for the supercharged engine is highlr than for the naturally aspirated engine, whereas <20
. the peak cylinderpressures forthe supercharged engine are onlymarginallyhigher.
Howver, the zr ain ad,vantage gained by supercharging thc qilind,ers is thot the uertical
. d,istance between the upper and, lower curues for the superpharged enEine is greater 1'2 2'o 2'2
throughout the cylind,er swept uolume, which indicdtes thdt the tean effective pressure
is greater,
#*,J;?,"3
r '. , Fig. 16.5. Relationship between charge temperatue and boost
Filaily, the loop area enclosed by the supercharged engine is much larger, the
proportional difference being a meuure of the ncreased power n the supercharged, pressure ratio ifheat is not dissipated.
engtne. Thus the minimum compression air tempergtures with an intake temperature of 20'C fo
pressure ratios L.2,1.4,1.6, 1.8, 2.0
16.2.4. Boos Preasure and Pressure Ratio ad 2.2 are 85.7'c, 4g.E'c,62"c,28.6"c, 84'c and g4'c respec-
tively. In practice, with compressor efficiencies ofthe
o Boost pressute refers to the gauge pressure recorded, when the air or mixture supply has orde of60-?57 and the churning, turbulence,
passed through the supercharger, and frictional factors, the actual output
air charge temperature can be considerably higher, particu-
larly at the higher boost levels.
SIJPERCHARGINC OF I.C. ENGINES
NTERNAL COMBUSTTON ENCINES

16.2.6. Thermodynanic Cycle and Supercharging power


the thermodvnamic cvcle ofa,,rp"ihaigea l..
*I;rl}rshows oo rhep-u diagram for
^ "ogiou
Air in
:----) Air out

-I Pr, Vr, Tl Pz,uz,Tz

Fig. 16.7. Steaily flow proce-ss. )


Assuming adiabatic compressiou of air, the work done on the supercharger per hg of air is
given by
W=-lui4=hr-r\'
=co(Tr-T)=rrrr(+-)
Pl i l.t/- .Y-1 II
= c-T,ll
n \,l' - llx- 1
' ^l\n/
Ps ,

| r"ai.
I
Fig' 16'6' thermodynamic cycle of supercharged t where, r"ur. is the adiabatic efciency oithe superch-arger/compressor.
engine on p-u diagrarn for an itleabtto cycle.
o pressure p, represents tbe supercharging
.'. Power required to drve the supercharger lP) is then gven by,
pressure and pu is 'l
'lre thermodynarnic cycle,
The
theezhaust pressure. .Y-1 l.
consists ofthe followingprocesses
: p=ocprt lIpzlr _rlxw
(i) O_1. Admission ofair at the supercharging pressure (whicl is grzater naar
pheric pressure).
than atmos_ l\rrl I
Gi) l-2, Isentropic conpeasion. where, zro = Amount ofair in kg/s supplied by the supercharger, and
i
(iii) z' Heat addition at constant volurne (for cp = SPecific heat ofair in kJ/kg Ii
diesel cycle, his will be replaced by a
constant pressure pocess, r.epresenting heat This power may be supplied by:
addition at conrtaot p""".,r'.".
i Gu) H. Isentropic erpansion. Gas turbine driven by the exhaust gas energy ofthe engine.
4-r-6' Heat rejection at constant volume (blow
- Separate drive by motor or any other piime mover driving tle supercharger'
down to atmospheric pressure). - Connecting.the superchargerto the engine output shaft'
&2. Driving out exhaust at constant atmospheric pressure. -In all the above cases the gain in the power output of the engine would be many times the
The thermodynamic cycle for tn.e superchorger consists power required to drive the supercharger I compressor,
ofthe following processes :
(t 7-G8. Adnission of air at atmospheric pressure.
Inporant points worth noting regarding supercharging of S.I' engines :
(r, &1. Isentropic compression to pressure p,. o Supercharging(ofS.I.eugines)isemployedonlyforaircraftand'racingcarengines,dwe
(iii) 1-{). Delivery ofsupercharged air, at a to [he fact that the increaie in supercharging pressure increases thetendency to detonate
constant pressurepl.
a Area 8-6-7-0-l-8 represents the supercharger ror,t (mechanically and. pre-ignte.
' airatapressurepr'wh'etheareal-2-B-4-1, driven) in supplying o The supercharged petrol engines have a greater fuel consumption than naturally aspirated
i,stheoutputoftheengine.Aea0-1-6-2-0 engines.
represents the gain n worh duri.ng thz gas
erchange process due to su;ereharging, Thus
a part of the supercha,rger worh is recouered., Increased intake pressure and temperature reduces ignition d'eloy and increase flarne
Uo**"r, tt *ork represented i, ti. u""u speed. The increased flame speeds make the petrol engine mo re sensitiue to fuel'air rato
L-6-8-l cannot be recouered, and, represents a loss
of worh. " nd, engine cannot run on weh mxtures without knocfr. Rich mixtures are used to control
i
Supercharging power: detonation, rulch further increa.ses the specific fuel consumption ofthe engine.
i Refer Fig. 16.?. o supercharging of petrol engines, because of poor fuel economy, is used in the following
cases:
i
I
p
, p Tr= Initial condiions ofair at entry to the supercharger
u

rl p2, u2, T"= pi1a,I conditigns of,air ; When a large omount of power is needed, or
at exit from the supercharger.
When more power is needed' to conpensate oltitude loss.
518 INTERNAL coMBusrroN ENGINES
519
Note' In highly supercharged engine,
-
chumfur (how-ever use of large
knocking can be controlled by injection of uoter in thz combustian
amount of liquid beoe prchibitive). Alt-e.ri.t*"iy1rt" ti" rge may be cnled.
belore it is fed to thc engire.

Supercharging Limits of S.L Engines


16,2.7.
rt is primarily rhe'knoch'which chiefryJi mits the degree of supercharging in s.I. engines.The
-knock limit is dependent
upon the following-facors :
(r) The type offuel used
;
(li) M*""" t"t'o '
(jli) Spark advance ;
(iu) Design features of the engine (important being cooling
systems and valve timing)
- Itandhaspre-ignition
been observed that for uolatle peboleum
fuels ofl-ilgh octane number the knocking
tendency is reduced at very rich and vry lean mixture, and that the
fuels ofsame octane value have different res-ponse to supeicharging. In the
case ofalco-
holic fuels the knock is reduced at rich rnixiures (because of te c-ooling effect
of high
latent heat ofthese fuels).
- Yery lean and. uery rbh-mi.rtures gn non-knoching operatons. The strongest
however, occurs ner chemically iorect mrtures. The use of rich mixture
hnoching,
results in a
higher specific fuel.consumption for a supercharged engine though knocking is control-
led). A slight reduction in lean mixture makes th-e *giie op"".ti; iiieg,rta"
inte"-
mittent. "rra
The ignitiori tnings and' thermal load, on the engine affect the hnoch
- limit of S.I. engine.
The ignitan must be retardd at high intake preisure'and temperature. B'D'C'
o Ingeneral,superchargerpreesueofl.Stol.Sbarisused,whichcorrespondstoaboutB0 ()
to 50 percent supercharging. Fig. 16.8. Superarging C.I. engine.

16.3. SUPERCIIARGING OF C.I. ENGINDS quieter and smoother.com-bustion This improvement in


combustion allows a p,ur,quality
fuel to be used in a diesel engine and it is also not sensitive to the type of
Fig' 16.8 shows.the schematic arrangement and p-u diagrarn for a supercharged increase in intake temperature reduces uolumetr.c and thernal effiiinq
fuel in"
preesure (diesel) cycle.
constan "Jia.
but itrcrelase in
lcrylU due to pressure compensates for ths and, ntercooling is'iot eisry ercept
o Unlike S.I' engines supercharging does not result in any combusf,ion problem, rather .highly supercharged engines. for
improues combustion in diesel enge. Increase in pressure and temperatue
it
ofthe intake If!! unsuPercharged engine is supercharyed it wiu ncrease tlrc reli.sbitity and durabtity
air reduces significantly delay and hence the rate of pressure rise resul tng in abetter, ' of theengine dw to smoother combustioiand. hwer exha,ust temperatuis. The
supercharging s rimted by thermar and. mechanicar road on in"
d.egree f
and strongly
Fuel depend.s on the type of supercharger used and. desgn of the engine ""gii"
16.3.1. Supercharging Limits of C.L Enginee
The supercharging limits for ac.L engine (unke s.I. engine
where the rimits of supercharg-
rng are due to combustion) is reachedbyth4;n
and cylinder causes scuffing of piston i-ings *a
art*ding. Thevery high temperature of the piston
iru.iirler *u"".
It has been observed that load on bearing is increased due to increased preesure
der. in the cylin-
\\e main considrons in miting the degree of supercharging of a c.L
engine are :
(d) Duability;
(ti) Reability;
(jii) Fuel economy.
- The reliability ofthe engine decreases with the increase in maximum pressure in
the
cfinder. This arso increses the thermar loaJon the engine due to theincie;r;
rate ofheat release.
t; il;
i
f

I 520 INTERNAL COMBUSTION ENCINES


l STJPERCHARCNO OF I.C ENGINES \ 521
16.4. MODIFICATION OF AN ENGINE FOR STJPERCHARGING
particularly at part loads, then the remaining load ofthe compressor is taken care ofby
Supercharging results in the increased output ofa naturally-aspirated engine. The following
the engine.
modificatbns mke the engine more suitable to supercharging:
o Increase n the valve oaerlap period. to permit conplete scavenging of the cleorance uolume, After
o Increose n the clearance volume by d,ecreasing the compression ratio. I I
cooler f
+L-lrM\^rr
o The injection systern ofa diesel engine must be modified to supply increased atnount of
trzl (this will require a nozzle ofgreater area than that nequired for the normally aspirated
engine).
o In case of a turbocharged engine, in order to supply more energy to the turbocharger the
exhaust value shall open a bit earlier. Furthemore, the exbaust manifold ofsuch engine
is insulated to educe heat losses (whereas in case of a normally aspirated engine the
exhaust manifold is water-cooled).

16.5. SI,JPERCIIARGERS
(;) (rr)

The following types ofsuperchargers are used to supercharge engines for various applica-
tlons : I Aflet
cooler t
I
After
1' Reciprocating compreaeor. Very rarely used nowadays except for some stationary T Ir-rWr\^ru I cooler i
I

installations. '
2. Vane blower. Due to the vanes the flow ofair is pulsating and noisy and the speed is I
?

'limited because ofthe radial motion ofvaues. Nowadays these are almost obsolete.
3. Lysholm compreaaor. It produces a constant compression, Has limited use due to its
mechanical cornplexity.
4. Roots blowet.t is suitable for low and tned.ium speed engines for stationary and, marine
instollatons:
5. Cenrifugal conpressor. It is simple, small, cheap and has a good efficiency in the range
ofpressure ratio of 1.5 to 3.0 and is cornonly used for supercharging. (tit) (iu)
The limited range of this compressor makes it suitable for constant speed type
- engines, suchspeed
as aircrafi engines, However, the exhaust driveo centrifugal compressor is
C= Compressor 1. Inlet to compressor
T = Turbine 2. Outlet from compressor
very popular because ite operational speed matches with the exhaust turbine speed sat-
isfactorily, G = Geuing B, Inlet to engine
Disad,uantage. Occurrence of surge. 4. Exhaust from mgine (and inlet tt turbine)
- 5. Exhaust frcm turbine
16.6, SUPERCIHRGING ARN"ANGEMENTS Fig. 16.9. Supercharging anmgements.

Fig. 16.9 shows the different supercharging arrangements.


r Fig. 16.9 (iu) shows an arrangement of supercharging in which engine supplies its total
power to the compressor and the exhaust gases frorn the engine run the turbine giving
Fig. 16.9 (i) shows a compressor coupled to the engine with step-up gearing for inceas- the power outrut. Such plants are calledfuel-piston engines.
ing the speed ofthe compressor. Here, a portion ofthe engine output is used to drive the
compessor, The net output increase due to supercharging is obtained by subtracting 16.7. TURBOCIIARGERS
this power from the engine gross output. The purpose ofaftercooler is to send cool air to
the engine for futher increasing the density of the intake air.
16.7.1. Introduction
Fig. 16.9 (li) shows an arrangement in which compressor is driven by a turbine which is
o Turbochorgers are centri,fugal compressors d,riuen by the exhaust gas turbines. By
utilising the exhaust energy of the engine it recovers a Jubstantial part ofenergy rvhich
run by the exhaust ofthe engine. The compressor and the turbine are not mechanically
. would otherwise go waste ; thus the turbocharger will not d.raw upin the engine power.
coupled to tbe engine. These ae nowadays ertensiuely used to superchargng almost u typ"t oftwo s:rohe
Fig. 16.9 (iii) efes to
an. rrangement in which engine, turbine and compressor all are engnes.
geared together. The wright T\rrbo-compound air plane engine is an example ofthis type o Atypical petrol engine may harness up to 307o of the energy contained in the fuel supplied.
of arrangement. Here if the turbine output is insufficient to run the compressor to do useful work under optimum conditions bu.t the remainng 70Eo of this energy iLs lost
in the following uay :
r-
i
ENcNes
il SUPERCHARGING OF I.C.
523
522 INTERNAL COMBUSTION ENGTNES
by a factor of one or two. Thus, a compression
ratio of I0 : I for a normary aspirated
engine would' be d'erated to g: 1
energy to friction, pumping and dynamic movement; for ro,wioo"t pr"ssure or euen reduced, to g : 1 if a
-TVoheat medium to hish boost pressure s io be nlioduced,.-
9% heat energ:y to surmunding ai; engine which might wrma,y hor" o
similarly, i., i ilr"ction dieser
- rati.o of 16i 1,' whcn tigiiiy turbocharged.
-l67oheat
energ'y to engine's coolant system ; "o^priiiio,
the compression ratio mav'be towered,
il; I ;i;;:;;t;hr;w!,charsed
energy to outgoing exhaust gases, be used., the compression ratio ";,haie to t" Uriugnt intet
-38%oheat may l., b something
Thus, the vast majoriy ofenergy, for design reasons, is allowed to escape to the atmosphere i{fi'ff?ri* "
through the exhaust system. c Thecompresr^":{,llr:!"-f:entering.thecellsdthe.impeller.dzpend.suponthecentrifu-
o A turbocharger utilizes o portion of the ercrgr contained, in the exhaust gas-when it is sot force effect which iureases with the squari of the'r"t.ti;;;';;;;i
the i,npero,
wheet' consequenv, under risht -;1.;
released by the opening ofthe exhaust valve towards the end of the power stroke (some-
leased with the e".houst gases ;ilt be"tili
;'"si;, ;;;;;'""iiiJ'rn" enerEy re.
thing like 50'before B.D.C.F-to drive a urbine wheel which simultaneously propels a
the turbine assembrv otrery hish tp";d*-
,*"u ond, is therefore insufficient to driue
centrifugal compressor $rheel. ;i"rpooairavl ii.Jftji'"?rv rirtle extra
The turbocharger relies solely on ertractng up to a third. of the wasted, energy pctss-
boost pressure tomakc *i
Ti'lL
put in the low-speed range of
i-p""'r"-";t to thi engine's to.q* power out-
ing out from the enqine's cjlinders to mpart power to the turbine wheel and cornpres- the e"gine. rhus, i"
operate wfth armost no boost pressu and "n".t,
ifi"'i#;;i;;"d
"n engine
wi,
wit a redu;"d
sor wheel assembly, However, this does produce a penalty by increasing the mani- with the equivarent naturalty^"spirateJ"igi.r". !;;;;;;#iitio "o.rrp"""d
rr"n"u, in the-uery row speed ronge,
fold's back-pressure and so making it more difficult for each successive burnt charge turbocharged engine may haue torqve and"power the
to be expelled from the cylinders. It therefore imped,es the clearing process in the outputs and fuer consumption varues
which are inferior to the unsuperchLrei
i";1"".
cylinders d,uring the exhaust strohes, o Another inherent undsirable characieristif,
of turbochargers is that when the engine
The ideal auoilable energy whi.ch can be used, to driue the turbocharger comes from suddenty acceteratcd there wilt be a,ili, is
the blow-down energy transfer whichtakes place u)hn the exhaust oolue opens and into the turbine nory.lF uorute can tp"i
dzta-y.befleii;;;;;';""s1 discharsed
up il" turtine whee!. Thus, d.urig this
thegaserpandsdowntodtnlosphericpressure(Fig. 16.10),flislow-downenergyis tion period' there wi.u be uery tittte ipriuilr"t transi.
represented. by the loop area 4-5-G whereos the boost pressure energ! used to the rise n qlinder brahe mian ,ff"rtir
" the.cylinder filling proce'ss, and, hence
fill the
ii"rr"*
will be rather sluggish.
cylind,er is represented. by the rectargular area 0-1-6-7.
16.7.2. dtitude Compensation
power outp'ts
,^-^^
oense .T10"u 11e ,tested and rated at seaJevel where the atmospheric air is most
; however, as a vehicle cmbs, its altitude is increa:ed
and the air beco-"! ili.rn"", trrut i",
less dense. The consequence i". d;";;;l;;"'i"-"# as less air will be drawn into the
cylinders per cvcle, with a correspoodi.rg
reducii;;;;;;"
"h.iency
p"*er since power is direcgy related to
Requrd enrgy to
1 fll cylinder undr

q
I boost pressure +I
c I
Blow-down E^-
f energy available Bvr
T +
q to drive turbocharger f
o I

E
6 c
6
o
3eo E
q
.E
CL a
o E
.g c\
o
c q
2n
ct o
*
Volume ------) J0
Fig, 16.f0. Petrol engine rycle pressure-volune diagram showing available exhaut gas energy.
1000 2000 3000
Ahitude above sealevel (m) ____
o Turbocharged engines prod,uce higher cylind.er uolutnetric efficiencies cornpdred with tlle Fig f6.11. Efiect ofltitude on rated
engiae power at sea level for *
normally aspirated. induction systems. Therefore, there will be higher peak cllinder naturay-aspirated and turochg"J""g1""".-' "'* both
pressures which increase the mechanical loading of the engine components and. could, cause
detonation n petrol engines, Therefore, it is usuol to reduce the engine's cornpression rdtio
iillt 524
tlltl INTERNAL COMBUSTION ENGINES
SUPERCHARGING OF I.C. 525
il lti th.e-actual
.mass of charge burnt in.the cylinder's every power stroke, A naturelly aspirated engine
will haue ts power output redtrced b3t approximatety'nro t;t x o The power deueloped' by the turbocharger is sufficient to drin the cot7pFessor, and. ouer-
1000 m
il ltl aboue sea-le-uel. supercharging the cyrnd,ers enabrei the
engiru,s "pi
,it"a "wlolot"ty
por"rlboue sea_reuel to be t cosw its nechaniol fricton. The turbocharger is independent ofthe engine, and it is
maintaned or euen exceeded,, tt ': only-connected to it by a sirnple exhaust pipe. The speed range ofthe turbocharger is
I
With a turbocharged engine thee will still be some power loss with he engine from 20000 to 30000 r.p.m.
lllll high altitudes, but the oss will,b^e far ress than ifthe engne breathirw
olerating at
d"p.rrrt"i orry o, naturar o For supplying adequate energy to the turbocharger,tbe ezhaust ualue is opened, much
aspiration. As can be seen @ig. 16.11) ot 1000 m ihe powelr loss
is onlyE"ip"*iith the natu_ { before the B.D.C. in contrast to a natutally aspirated engine. ?ds permits the exho,ust
Illil rally aspirated. engine where the power decrease is roiighly lBVo.
;. gases to escape dt a hgher pressure and, temperature, gving turbocharger enough energy
lil tl 16.7.3. Thrbocharging-Buchi System
ttttt

Iilil o Exhaust gases coming out from the engine, when the exhaust valve
opens, ale at a pres-
* is more then offset by better chargng and,.scauenging of the ensin;
il|ll
!tllt
sure well above atmospheric pressure, and their temperatue is
atro-trijil. A part oithe { 16.7.4. MBTTIODS br TT.BBOCIIARGING
hcat energy contained in the gases ian be utilized by fi:rther exput
lll,: ofit u g"ses in an The following are the main types of turbocharging methods
exhaust turbine down to atmospheric pressure. rh extra powe","ur" "ion r
n,o- the tur- :
bine is used to dive the co_mpressor. The compressor, in turn, will 1. Cons+et pressure turbocharging :
illll
tll
supply more air to the
engine' Such utilization of the exhaust energy to drive the r"p"".rr"ti".
i" called, Buc.i o The various cylinders discharge their exhaust into a common manifold at pressures
-: higher
ilf system ofturbocharging. than the atmospheric.
Fig. 16;12 shows the arrangement ofBuchi system. The exhaust gases (from all the cylinders) undergo expansion in the exhaust valves, without
tt il Compressed air doing any work, to an appromately constant pressure in the common manifold and then enter the
fiil turbine. thus the blow-down energy in the form of intrnal energy is converted into work in the
lr u turbine. The higher the ptessure ratio ofthe turbine, the higher is.tL recovery ofblow-down energy.
l During the whole ofthe cycle the exhaust gases are maintained at constaut pressure to make use of
lil
l

a pure reaction turbine.

l:i 2. Pulse turbocharging :


o In this method of supercharging, as soon as the exhaust valve opens a considerable part
ll ofthe blow-down energy is conv ertedinto erhaust pulses. These pulses enter the turbine
t1
(throufh nanow exhaust pipes by the shortest poisible route) wiere a major proportion
l
of the energy is recovered.
iti r In orde that exhaust process ofvarious cylinders do not interfere
Air from
with one another,
separate exhaust pipes are used.
l,ll atmqsphere
i 3. Pulse converter :
T= T\rbine, C = Compressor
This turbocharging method prmits the advantages of the pulse and consrant-pressue
Fig. 16.12. Arangement ofBuchi system. -
tubocharging methods simultaneously. The combination oi thus" two systems is doneby
cinnecting
Fig' 16.13 shows the additional energy equal to area 1-4-5 available when the different branches of ethaust manifolds together in a specially
d,esigned uentur junction, called,
the ex- 'pulse conuerter", before th.e turbine.
pansion is carried down to atmosphere.
p
4...Two-stage turbocharger :
Two-stage turbocharging is defined as the use of iwo turbochargers of d.ifferent sizes in series,
-
e'g' a high pressure stage operating on pulse system and low-pressuie .tage .r constant pressue
operation' This type of arrangement is employed for diesel engines requirlng
very high degree of
supercharging, b.m.e'p. ranging from 25 to 30 bar, which can noibe obtained iria
sirigle--stage-turbo-
cnarget.
5. Miller turbocharging :
r The system ofturbocharging is based upon the id.ea of increasing the expansion rato
relatiue to conpression ratio by means ofearly closure of nlet ualui as the toost pressure
is increased.
o TheMillerturbochargingisnotveryattractiveunlesstwo-stageturbochargingisneces-
Fig. 16.13. Diagram shows additional energy available when
sary because ofother reasons such as need to reduce exhaustialve failures.
expansion is caried down to atmosphere.

-==--+'
526 SUPERCHARCNG OF I.C. ENCINES

6. Hyperbar turbocharging :
Fig. 16.14 shows a hyperbar turbocharged en- WORI{D EXAMPLES
gine. It consists ofthe following components:
1. A low compression (?: 1) diesel engine ;
Example lB-l;An unsupercharged. petrol engine dcuelops 735 hW wth air fuel rotio 12.g. The
bsfc in 0.350 kg / kWh and mechancol efficienqr 867o. The nlei pressure is 750 mm of rnercury abso-
2. A high pressure ratio (upto 5.1) turbine ;
lute and. the misture tenTperatu.re is 325K. Theengineissuperchargedtoopressureratoof 1.6by
3. A by-pass control; a superclu,rger of ad,a.batic efficienqt 0.7 and mechanicot effiiiency o.b. Assumtng that air-fuel raio
4. An auxiliary combustion chamber/ located remains unch-anged and LP. is proportional to inlet density, calcilate the power-required to run the
between the direct exhaust valve and the supercharger, Assume that uolutnetric efficency does not ciange d.ue to suprr"horging.
turbocharger turbine.
(Madras University)
Firstly, using an electrical starer, the
turbocharger is started ; it is kept ruir- Solution. Given : P2
= 1.6 ,pt = ?30 mm of Hg abs., T, = B25K
ning by bypassing the air and inject-
ing first into the auxiliary c,ombustion l = 0.7, I". = 0.9
v-t
chamber while the engine in opera. / rr 1.4-l
^
tive.
Then after sonetime when appropri-
+=lal'
rr ,ot,/ =trel-* =1.1417
ate pressues and temperatures are Tz= 325 x LL437 = 371.7 K
reached the diesel engine is stated.
The amount of air bypassed and the lL=-'
.raor,
f, _f,
n2I
fuel injected into the auxiliary - D'l
combustion chamber are controlled in
accordance with operating coDditions.
Tr'=7r*4-\' I
Iedi.
Advontagee:
.., 41
,2
1. High power-to-weight ratio.
2. High brake mean effective pressure to the tune ofB0 bar can be obtained (peak pressure Ind,cated worh
limited to 140 bar). kJ&g of air. Fig. 16.15
3. Thermal loading-moderate.. Worh required. to d.rive the supercharger
4. Surge pre-operation (ilue to by pass control),
w".,^ = " o(T' ?
- T)
: _= ----ff
1.005(g9r.z - 825)
5. Good torque and acceleration available. = 7 4'48 kJ/ks of ait
n^*0.
DisodvantaSlet: lVhen unsupercharged:

# " 913#@
1. Due to low compression ratio, there is high fuel consumption over the entire range of
operation. Inlet pressure, ,' = = ez,3e kN/m2
2. System.is complex.
gz'gg
p""*,. = 4-
RT,=0.287 xB25 = 1.044 kg/mg
Density, uuuP
Limitations of T\rocharging
16.7.5. '
Turbocharging entails e following limtatians :
Air consumption, c(u*up). 0.35 x-7-3-5-x 12.8
1, Special exhru ' 'nanifolds are required for the turbocharging sydtem. - = 0.9047 kg/s
2. In order to inject more fue per unit time fuel iqiection needs modification. Tlthen supercharged:
3. In contrabt to a naturally aspirated engine which can digest solid particles in the inlet air
without undue stress, a turbocharged engine ca n pass only the most minute material par- Density, p,* =ft=ffim, = 1.386kg/mg
ticles without damage.
4. It is difficult to obtain good efficiency over a wide range of operations iince the efticiency Air consumption, c(op.) = n(u^rp.,
ofthe turbine blades is very sensitive to gas velocity. "p**.
INTERNAL COMBUSTION ENG NES STJPERCHARGING OF I.C. ENGINES

u186 I
at xa =1'2r43kc/s
= 0.91,.- p- x 0.02488x 1500x x 10
Power required to run the supercharger, ,dl: _ i
6
PtoP= o{stPXW"*
.'' Pu = 8'36 bar' (Ans.)
Supercharged engine:
1.2143 x ?4.48 = 90.44 y, 1r". Incr'ease of pressure requircd :
Exarnple L6'2' A d'iesel engine operating on
four-strohc qrcr.e is to be d,esigned.
operdte uith Gross power produced by the engine
following chardcteristics at l"i, *itu" tii-Jnf,ino^ ore 1.0rs2 bar and. to = 260 + 0.08 x gross power
B'P' = 266
sea
kw uolumetric. efftcienc! = 78vo (at sea leuel 10"c.
260
\
free air cond.itio*s), specfi.c .'. Gross power produced by the engine
=%J2.6kW
consurnptian = 0.242 hS / B.p.h. ; Al F ratio fuel (1- 0.08)
= 1Z ; speei = rc17 r.p.tu
calculate the requred' engne capacity andihz 18J9x 282.6
anacrpoted, brahe mean effectue pressure. Mass of air required
e.ngne,s with a sup-erc,harger so that it may be operated. d,t 19.77 kg
IO"otmosphericfr'tted
.,., ^-- the
where pressure an altitude of 2700 rn 260
0.22_bar.-The power tahen b1r a supercharger
total power produced bv the ern,ine .is is g per cent of the
and, the tiip*t"r""orthe air reaury the
The air-fuel ratio and, terml efficienqt superciarger zs s2"c. lwL
_ Mass of air taken in per cycle
remainihe same for the supete.harg"d as when running
Mass of air corresponding to swept volume measured. at outlet conditions of superchuger
unsupercharged at sea reuel, d.r: ;.h: uolum"tru efuiienqr. carculate-the "n,ine L9.77
required' at the supercharger Tto maintain th" t";;-;;i-;;put of 260
inrease of air pressure
Solution. Giuen : pr= 1.0182 bar, ?, L0 + 27g=
kw. Take R = o.iaz n"l t ngx.
0.?8=- LSOOI2 0.02636
= 28A K B.p. = 269 14r. (pr-16)"0J24s8 =6;
\j6-. = 7 8Vo, s.f.c. = 0.24? kg t B.p. h, A / F raio 1.7 : t, 287x(32+278)
=
.ly' = 1500 r.p.m.
0.02636
Engine capacity: ' P2 = 0J8 r. OJ,S4 bar = 1.19 ba
Fuelconsumption, mr=s.f-c:rJB.p.kg/min.= 0.247:260=1.07ke/min. .'. fncrease of pressure required = l.l9 - 0.?2 = 0.42 bar. (Ans.)
' 'v I \5 -
Air consumprion = r,r"l 33r,ro.prl* o, *i3 perms-Example
16.3. A 4-stroke d.sel engine hauing a copacitxr s600 cms deuelops ls kw
of
" of free air ind'uced per minute. It h.as a uolurtric 82 percent at s000 r.p.m.
"
= 1.07 x 17 = 16.19 *7-'rr. refened to free oir conditinns of 1.01J2 bar and 25'c. This "ifuu*y'of
engiie s sujerchorged. by a rotary
Air consurnption per stroke Air consumption in kg / min. colnpressor which deuelops d pressure ratio of 1.8 and has an sentropic efficency of 75 per cent.
- - No. or This compressor is coupled to moin shoft of the engine whi.ch supplies piwer to it. Estimate the
'
"va7iiil- increase in brah pouer due to surperchargng.
Airconsumption in kg/rnin. 1g.19 Assume mechanical efficiency of 80 per cent and. tle air ot intoke to cylind.er to be at the
N,_ = lS00D =0.O242kC, pressure equal to the deliuery pressure
frorn compressor ond, temperature equal to 4'C less than the
d'eliaery temperature fTom tlw cornpressor, and, cylindcr contains uolume charge equal to swept
Let V" be the swept volume, then mass of free
air corresponding to swept volume
= &]!. DOUme.
RTt Solution. Gluez ; Engine capacity = 3600 cm3 = 3600 x 10-6 ms ;
(1.01A2x105)xy-
= --lgilt$ .-
Power developed = 13 kWms of free air induced pei minute ;
=r'247 v"k'' Volumetric effrciency, \"o1= B2/o at 8000 r.p.m. and
Volumetric efficiency, uo,. = - - - Yass 9f
air taFen in per stroke Pr = 1.0132 bar, { = 25+2732 = 298 K;
ruass ot lree air corresponding to swept volume Pressure ratio (rotary compessor) = 1.8
aa _ 0.0242
Isentropic efciency of the conpress or, t1
^v't=Tl47v: or v=ffi
0.0242 r""o. = 7 5go
=o'02488m3 Mechanical efficiency, rl6qcL = SOZo
i.e., Engine capacity Increase in B.P. due to supercharging:
= 0.02488 m3. (Ans.)
Brake mean effective pressure, p_o (bar)
:
Swept volume / mini ul 3?o
= (86oo x to- ' = 8.4 ms min,
4I 10
We know that
-
s.p. " klv
Po
unsupercharged vorume
-
26e=&al-%-1frJ91y (... LxA= v) :i.!::r"j"=Ti# r,il,:

'i,l
{
,
INTERNAL COMBUSTION ENGINES

Rotary compressor delivery pressure = 1.8 x 1.0132 = 1.g24 bar SUPERCHARCINC OF I.C. ENCINES
s3l
Temperaturesafterisentroiriccompression, T
v-l B.P. = 250 kW ; Volumetri efficiency - 78 per cent (at air
/ r L: L4-l sea leuel free conditions) ; Specific
fuel consumption = 0.245 kS I kWh ;
rr=rJ Al' =298x(1.8)E =3s2.5K Air-fuel ratn = 17 : 1 ; Speed = 1500 r.p.m.
\ Pr,/
Determine the required' engine capacity and. thc brahe mean effectue pressure.
Also, Tlis... =#
t2 - t7
If the engine is run at on alttude of 270A m where the atmospheric pressure is 0.22
bar by
_298 frtting a supercharger directly and mechanically coupled. to the engine ; the power consumed, by the
352.5 supercharger is 8 per cent ofthe total powe.r producecl by the engine and, the temperature ofair
u. JD = leuing
ri,-2e8
--- the superchorger is 32"C. The air'fuel ratio and the thrnal eftciency remdin;he ;d;;
for the supe-
charged, engine as when running unsupercharged, at seo teuei, as does the uolumetric
efficiznqr.
7r' -(eszl
-\0.75) -zse'l * ,ru = s7o.7 K Determine the increase of air pressure required. at the supercharger to maintain the same
net
The temperature ofair at intake to the engine output of 250 hW. (Kerala University)
, Solution. Given: p, = 1.013 bar i ?r = 10 + 278 =Zg}K;s.f.c. = 0.245 ke/kWh;
cylinder
K Fis' 16'16 Air-tuel ratio = 17 : 1 ; N = 1500 r.p.m. ; B.p. = 259 ry,\"o1. 78%
=370.7 -4= 366.?
273 = 305 K.
= ;i Pz= O.72ba t-2
iTz= 32 +
The compressor delivers 5.4 m3/min. air at 1.824ba and 370.? K. Equivalent volume at
1.0132 bar and 298 K, @) ansupercharged.engine at sea lvel:
PtVt PzVz Engine capacity, \:
Tr = Tz' 0.245x250
hVqT, 1.824 x5.4 x 298
Fuel consumption = ------- = 1.0208 kg/min.
=
10132 306r =
7'9 m3/min'
.'.
bU

"=ifr ''
The increase in induced volure of air = 7.9 _ 5.4= 2.6 ms/min.
Air consumption = 1.0208 x 17 = 1?.35 kg/min.
Mass of air taken in per cycle
Increase in LP. from air induced
= lB x 2.5 = B2.b kW Mass of air corresponding to swept volume
Increase in I.P. due to increased induction Dressure
17.35
Ap x V/min. (1824 9500) 750) (t7 .35 |

.'. Total increase in I.P.


= doxlooo
= 32.5 + ?.3 = 39.8 kW
=
- 1..0132) x 105 x 5.4
60 x 1000 = 7.3 kW 0.?g =

R\
4-=@=t6ffi _ L7.BS x (287 x 283)

.'. Increase in B.P. of the engine = g9.g x l-"o.


Sweptvolume, %=
= 39.8 x 0.8 = 31.94 ry
From this must be ded.ucted the power required to driue the rotory compresior,
offi.:ff#' =0.02s8ms. (Ans.)
Brake mean effective pressue, phb !
Mass of air delivered by the compressor, ,
P- Z4lV x 10
g.p. - O*
,i" = {_ _ (1.824 x 106) x (5.4fl60) L-l"
RT 287 x 366.7
0.156 kg/s P-x%x1500xlxro
,UO- ('.' V"=LxA)
- , It may
calculated
be noted that the delivery temperature from the compressor is 3?0.7 K and mass is
at 366'? K because the volume occpie<l by this mass is known only,at 366.2 K being the
b

_ P^ x 0.0233x 750 x10


swept volume of the cylinder. ,UO
Power require(by the compressor Pn = 8'4 bar' (Ans.)
"'
_ th co LT !,1!q * r.OOr x (3?0.7 - 298) Supereharged, egine :
Tlmeh. = oL = 14'25kw Increase ofpressure required :
.'. Net increase in B.P. = 81.84 - 14.25 = f2.59 kW. (Ans.) n
Gross power produced by the engine
=250+ xgrosspower
. Example 16-4- A 4-stroke.d.esel engine is d.esigned to operate wth the followng charac-
terstics
at sea leuel, where the m"on are 1l0B bar and IT"C : _ power
.'. Gross 250
"ortiors - = ;-;;iu,u)
ir - -- 271,74kW

.'. Mass of air required - 271'7!1117.35


= 18.86 kc
on
INTERNAL COMBUSTION ENGINES SUPERCHARGING OF ENGINFS

n.-r = o.?8 =
18.86i(1500^2)
_ . 18.86x2x,R?"
@2v"tRT" 15oo"(a;lff) %' wbereP, is in bar 6xp-, x0.11xtx(0.1)2 x2000xlx 10
39.26 = = 8.689 p_
_ 18.86x2x28?x805
1500xpx0.0288 p'u=4.544bar. (Ans.)
18.86x2x28?xg0S (jj) Air consunpion rate of te engine;
Pz=
0.78x1500xIdx0.0288 = 1.186 bar. -":
... fncease ofpressure required 1.1g6 _
0.?2 = 0.466 bar. (Ans.) Ensinesweptvolume = 6"f+D')r.{ -r/-tr,.
=
Example 16-5. An ar cornpressor is being run
\4 ) z
b3r thc entire output of a supefcharged 2000
cycle oil engine. Air enters the compressor
ot 25.C and is p'w on to a cooler where 1210 kJ 4_stroke G x I x o.l2 * 0.11t = 5.184 ms/min.
are rejected. The air leaues the coo.rir at / min. 2
the engine which ias a uotumetric effi"i"*v
65"c iiai.li
tlr.' p*t i"
"r' lui"i io ,up"r"norg" Aspirated volume of air into
ofr;'p;;ot-basei on "rinii niriiil, ;;;1";;
condtion of "n$ou
65'c and r'TS bar' The eneine.whi;1 n* =rl,or. x 5.184 =O.72x5.L84=3.732 ms/min.
r1'-n_ti{4.1'rr
d.etiuers a output torque of rsl Nm.
r*
mm bore and II0 m stohe runs at I Aspirated air mass flow into the engine,
The mechanicat efficency of ensine is
?!i;:;;# 80 per I (t'tsli!9u)-l^.3'tsz
(i) Th indicated mean effectue pressure tI -"' = +* = = 6.T82 ks/nin. (Ans.)
(ii) The air consumpt.on rate of the
ofthe enginc ; RTz 287 x 338
engine ; (iil) The air flow into the compessor in kgihrinrm" :
(iii) The air flow into compressor
in kgl min. Actual compressor work required from the engine to run the compressor = Gain in enthalpy
Solution. Giuen : Tr= 2"5 ofai in compressor
_+_278
= 298 K; e"O = 1210 kJ/min. ;
p2= L.tb bari lvo =72% ;n=6 ;D= l00mm O,1m tir"xcoxLT
tto = ; i 3L.41 = f where " =Massof airhandledbycompressor I
(i) The
-- = 0'11 m i ?oor = 150 Nm ir-eo. = 80v. ; N
=2000 r.p.m. = rz" x 1.005 x(Tr- Tr\ tn kg/s, and
indicat.o'-; :an effective pressue, pd : | I
= m" x 1.005 (T2-298) L f"= Tempeature of outlet air from co-p""ssorl
= -3ntr- nw= 3#Slso = 81.41kw :. rh. (Tr-298) = 31.25 ...(j)
".o.
Also,inthecooler, rh" xcrx IT2--$5+2ZB)l =e,urlsec.
or " Q;- as8) - --l?lq- = 20.07 ...(rr)
Dividing (i) by (ij), we ger
T2_299 _3L25 _r.o
r,-33s=16:6;7="oo
T2 - 298 = L.56 (Tz - 338) = 1.56 T2 - 527.28
e,"i = .' Tz= 409.4 K or 186.4.C
1210 kJlmin.
rhus, *=;:Zk=#6=o.28o5kc/s
m., 1.7S bar, BASK *"= o.laosx 60 = 16.88 kglmin. (Ans.)
C = Compressor S.E. = Srrpechaged engine
mo = Aspated a mass flow rno rhe engrne.
IIIGHLIGHTS
Fig. t6.lT
B.P. 3l4l l The purpose ofsuperchargingisto raise the volumetric efficiency above that value which cm be obtaired
3e'zelw by nomal aspiration.
lF =
2. Bnst pressure refers to the gauge pressue recorded when the air o mixture supply has passed through
But, _ nPn IANk x70 the supercharger.
,.O.
6

'r
534 INTERNAL coMBUsrIoN ENGINES SUPERCHARGINC OF I.C. ENCINES
,/ 535
3. Pressure roio is tho atio gf
/absolute pressure
to that of the atmospheric pnswe.
(Here, absolute-pressure lboost pressure + atmospheric presure)
4, The superchuging linits for a C.I. engine (ulike S.I- engines where the limits of supercharging re due
o combustion) is readred by lerml lnding. l. indicated 2. No 3, higher 4, Yes 6. Yes
D. The following type of superchargers are used to supercharge engines for various appliatiom : 6. Boost 7, Yes 8. Yes 9. No 10. No
Reciprocating compresore ; Vme blower ; lysholm compressor ; 11. Ys 12. Yes 13. Ye 14. No 15. Yes
Roots blower ; centrirgal mmpressor. t6. Trbocharging 17, Yes 18, Yes 19, Thcstage 20. Miller
6, Turbochorgers are centrifugal compressors driven by the exhaust gas tubines.
7. Methods of turbeharging :
() Constmt prsue turbocharging. (ii) Pulse turbmharging. THEORETICAL QI,]ES"ilONS
(iii) Pulse converter. (iu) Two-stage turbocharger.
(u) Miller turbcharging. (ui) Hyperba turbochargbg.
l. What is supercharging ?
Enmerate the main objecls of supetcharging.
3. What is a supercharger ?
4, Explain briefly supercharging ofS.I- engines.
OBJECTIVE TYPE QIIESTIONS
o. Give the comparison of 'Actual natually aspirated' nd ,Bupercharged engine' pressure-volume
diagrams.
Fill in the Blanke or Say Te6'or t{o'!
6. Define the tms Bmst pressure'md ,Pressure ratio'
1, The ..................... power pmduced is almost directly proportional to the engine air emumption.
Explain briefly the efreci of pressure ratio on ai chage temprature.
2. Supercharging tends to lesseu the possibility of detonation in.a S.I. engine ud increase lhe Pcsibility 8. Explain briefly the themodynamic cycle of superarged engine onp-u diagram for an ieal otto cycle.
in a C.I. engine.
9. Derive an expression for the power required for an I.C. eagine supercharger.
3. Thepoweroutputofasuperchargedmgineis.....................hmthenaturallyaspiraiedengines.
10. What are superchrging limits of S.L engines ?
4. The mechanical ecieucies ofsupercharged engines are slightly better than mtually aspirated engines.
lt. Explain briefly supercharging ofC.L engines.
5. Inspite of betrs miying and ombution due to reduced delay a mechanically ruperdrarged otto engine t2. '\{hat are supercharging limits of C.I. engines ?
slmost always have specific fuel coruumption higher than a naturally aspintdd engine.
13. Enumerate the main considerations in linitiry the degree ofsupercharging of a c.I. engine.
6......................pressurerefentotJ'egaugepressuerecordedwhentheairormtuesupplyhmpmsed t4, What modifrcations are necessarlr for a euperctrargert engine ?
through the superdrarger.
16. List the types ofsulrerchargers used to superchuge engines for various appcations.
7. Supercharging of S.I. engines is employed only for aircraft md racing car ngnes, due to the fact that 16. Explain briefly with neat sketches various t1ryes of supercharging anmgements.
the incease in supercharging preue inseases the tcndency to detonate mtl pre-ignite.
t7, What are turbochargers ?
8. In highly superdrarged engine, knocking cm be controlled by i{ectim ofwater in t}e combustion
chamber.'
18. Desribe the Buchi system of turbocharging,
19. Explain briefly the main tJpes of superctrargiog methods.
9. Very lean md very rich mixtues give high knocking operatiom.
10,
20. What is Miller tubouging ?
The ignition timings md themal load on th engine do not affect t}re looch limit of S.I. engine.
2L. Explain brielly with a neat sketch Hyperbar tubochargiag. State its ailvmtages and
11. Supercharging improv* mmbustion iD diesel engine. disadvmtages ai
well.
12. The degree of superrhrging in the case of a C.L engine is limited by thermal md mechadcal load on ,, What is pulse converter ?
the engine and strmgly depends on the type of supercharger used and dcign of the engine. 23. What are the mitations of supercharging ?
13, The reabiliy of the engine deceases with increase in maximum pessm in e cylinder. 24. "S.L engines are generally not eupercharged". Jusfy this statment.
14, Reciprocating compressor is comonly ued for supercharging. 26. 'Supercharging is prefened in esel engine thm petrol engine.. Jwtify he statement.
16. Roots blower is suitable for superctruging low and medium sped enginc fm sttionary md marine 26. 'Supercharging is essential for a:n aircraft engine., Justi$ this statement.
istaations. 27, Explain the factors that limit the extent of supenharging of S.I. mrl C.I. engines.
16. ...............,....: are entrifugal compressors clriven by the exhaust gm tubines. 28. what do you mean by supercharging of LC. engines ? Explain why supercharging is essentiar for
the
t7. T\:rbochrgbd engines produ@ higher cylinder volumetric efficiencies compared with the normally as- aircraft engines ?
pirated induction system.
18, A naturally aspirated engine will have its power outpu reduced by appmximately l3 per cet if it is
operated approximately 1000 m above sea-level. I,]NSOLVED EXAMPLES
19. tubochaging is defined as the use of two tubochrgers of aliferent siru in series. I' A 3000 cmr capacity 4-stroke diesel engine develops 14 kWmr offree air induced per
20. turbmhaging is based upon the idea ofincreasing the expansion ratio relative to com' minute. It has
a.volumetric efficiency of 807o refened to free aii conditions of 1.013 bar mtl
pression ratio by mems ofearly closure ofinlet valve as the boost pessun is increased- 2i.C, when running
at 3500.r.p-.m.-It is proposed to boost the power ofthe engine by supercharging by a blower (diven
mechanically from the eagine) of pressue ratio l.? md iJentropic ellicienc!
oi?%. Taking overatt
mechmical effi.ciency as 80% determine the inaease in B.p.

I
I
NTERNAL COMBUSTION ENGINES

Assme that at the end ofiduction-t'e crlinden contain


a volume of charge eqaar to the swepvorurne,
at the pressure md tenperature of tle eive.y*o_dl;il;.
It is required to desisn a 4-stroke [Ans.2Z.? kwl
die*l engine to operate with the
where the mean coditioro are l.0l3.bar d lo.: ----- "'*
.. following characteristie at sea level,
B.P. = 250 kW' volumetric eeielcr ?g perent (at
= sea level flee air contions) specific fuel
sumption = 0.245 kg&Wh ; air-fuel ratio i7 ; ; con_
= f ; rp.a = 1599..0.-.
Determ!e the required engine capacity and anticipated
A supercharger is hen tted to theesine m thar iinay
brake mean effeetive pessuie.
iioli"lioo
be
con";;;; "*r""
,i m wir.." tt e
T7
;:T::l};j j:T:T*:-qi?3::.9:ry:::;;il ";r;.l;;;;#ffi T#:::,1I'f"T"'li
atmospheric pressure i 0.?2 bar. firJpower ""
;;rrechareer is A ner ani rrr,- +^+^r
ifn'"i::::":,t"Y:::":1T:-T3-g"t"l'po"t*"ortr'";iilil;;#",i'ffi""ilj"l'Jfii,liiii Teqting and Perfornta,nee of I.C, Engines
:T.:T:T".:1:ill':":*.:"s,1t"tth"";p;J""s;;;;iH;;:h1Tiilqr"rY;!lJiii,i
fr::tg*"*:***:"^":j.*",T3"m"i"a'v.il."i';;;";;;"w"ffis"l'#J"1"
:.1"::*":*upere"hargedais;;;;4;;;rff ;;ffi',il;Xffi iliJ; [Ans. 0.466 bar]
17.1. Introduction. 17.2. Performancs parmeters. l?.3. Basic measurements, 12.4, Engine
(run by a suerc}rarged ilieset engine) performance wee. 1?.6. comparisoa ofpetrol md dieel engines-fuel omrnption,load outits
{" ""t:T"^:l*
where ":mpresor at 2?.C and is passed on to a cmter
1160 kJ/min. are rejected. Ihe air l""r""-tt"=-il-."i'^o.^ md exhaust composition. 17.6. Goveming of I.C. engine. 12.?. Noise abatement-Worked
lvqr,,E.'r
compessor 6 is bled aner
after rne
the @orer
^- r , L-_
;H:.:"T:,5',H;::j":*t"a'ir,"-"i-r-"-.i"T;";il:-i1i'tiu"tTlilff"'#:l,".HT: Examples-HigNights-objective rypo QuestioeThretical
cooler to-sul*rcharge the engine. The.volunetic eueetions-unsolveil Examples.
75 percent hased'reqo the intake effi"i""ay orin" u-ogiou i"
manor t -p.i"t"r" oi??: lird pressure
of67'C and or.*,,"o of 1.7 x--
r z bar.
The engine has the foowing specifications: ^f
B.P. = 39 1y . gneed = 2000 .p.m. Number
17.1. INIRODUCTION
; of cylindem = 4 ; Bore 10O m .
=
Stroke = ll0 rnm ; Mectranical elficiency g0 percent.
=
o The prinrary task oJ the development engineer is to reduce the capital and running cost
Assunitg the intemal effrcimcy of the conpressor
as 90 pel.cent, determiDe the following :
ofthe engine. This involves trial ofvarious design concepts. The parameters are so enor-
(i) The dicatd meu effective pressure mous and different in natue that it is almost physically impossibl to take cae.ofall of
of e engine ;
(ii) The air corounption rate of the engine;
them during the design ofthe engine. Therefore, it is neessry to conduct the test on the
(itt) the air handling capacity of the engine and deternine the measures which ehould be taken to improve the engine per-
@mpresso. fA8. () 65 bar; (i!) 4.58 kgtnin. 11.b? kg/min.l
4' The average indicated power developed in I.c. ; formance. The nature aud the type ofthe test to be conducted wilidepend upon a great
engineie 12.9 kwmr of free air inuccd per minute. The number offactors such as, the degree ofdevelopment ofthe particular design, the ccu-
engine is three-litre four-stroke_engin"
refened to free conditione of r.0r3 """"i"g;?l;00-r-.p'-., ana las a,"lr-"il;;ft;; cy of B0?o, racy required, the funds available, the nature ofthe manufaituring company etc.
ia;-;;;:; t" nt a ow"r, Ji" .".""i""rry e"_
Fr_andeciercy
the engine' The brower has an isent-ropic 15".
orzo,i and works through a pressure ratio of r.?, at r The testing ofthe engine is necessary to verift the performance ofthe engine as per the
the pressure and temperature of-the delivery f.;;tr;-b;*".. specification of the manufacturer.
expected from he eugine. Take all t]e carculate the increase in B.p. to be
"m"i"iJe" a* e096.
mechanical
[Ans. 25.8 kWl
17.2. PERFORiVfANCE PAR.METER
Engne perforrnance is an indicaton of the d.egree of success with whih it does its assigned job
.
te., conuersion of chemfual energy contaned in the
fuel into the useful mechanical work.
. In evaluation ofengine performanbe certain basic pararneters are chosen and the effect of
various operating conditions, design concepts and modifictions on these parameters are
studied.
The basic perfonruurce pardnuers are numerated and discussed beiow :
1. Power and mechanical efciencv 2. Mean effective pressure and torque
3. Specifrc output 4. Volumetric efciency
I
5. Fuel-air ratio 6. Specific fuel consumption
7, Thermal efticiency and heatbalance g, Exhaust smoke and other emissions
I
9. Specificweight.
i
l 1, Power and mechanical efficiency:
, - (i) Indicated powen The total power deueloped. by combustion of fuel
chamber is called, inditated, potaer.
in the conbustion

np-. LANkxL0
:b I.P.= s-_=;_ kw ...( 17.1)
, where, n = Number of cylinders,
j n- = Indicated mean effect.ive pressure, bar,
TES'TINC AND PERFORMANCE OF I.C, ENCINES 539

538 INTERNAL COMBUSTION ENCINES 6. Specific fuel consumption (s.f.c.) :

Z = Length ofstroke, m, It is the mass of fuel consum.ed per hW deueloped, per Inur, and is a criterion ofeconomical
power production.
A = Area of piston, m2, and
m,
1
_________-__\\ i.e., s.f.c. = ;_= kg/KWh.
= ; for 4-stroke engine b-r.
= I fo 2-stroke engine. 7. Thermal efficiency and heat balsnce:
(ii) Brahe power (8.P.). The power developed by an engine at thc output shaft s called. the Thermal efficiency. ll is the ratio of indicated, work done to enzrgy supplied by the fuel.
brahe power.
If m r = Mass of fu:l used in kg/s, and
2,'IT C = Calorific value offuel (lower),
B.P. = Uo *,O* kW ...(17.2)
Then nd,icated thermal efficency (based on I.P.),
where, N = Speed in r.p.m., and
7 = Torque in N-m. ,^ - I'P'
-,tlh.*,_ r;1fxC
(r7.7)
The diffeence between I.P. and B.P. is called fictional power, F.P.
L.e F.P.=I.P.-B.P. ...(17.3) and brohe thermal effiriency (based on B.P.)
The ratio of B.P. to I.P. is called mechancal efftcency B.P.
n, (B) =* (17.8)
i.e., Mechancal effr.ciencl, n.*n. = ...(17.4)
H Heat balance sheet:
2. Mean effective pressure and torque :
"Mean effectiue pressure" is d.efned as hypothetical pressure which is thought to be acting on The performance of an enqine is generally giuen by hcat balance sheet.
the piston throughout the power strohe. lf it is based on I.P. it is called ind.icoted m.ean effectiue To draw a heat balance sheet for I.C. engine, it is run at constant load. Indicator diagram is
g, p,;) and if based on B.P. it is called brake mean effective pressure (B-.".0. orp-). obtained with the help of an indicator. The quantity of fuel used iin a given time and its calorific
lr:t:,ur"- u o .or
Similarly, friCtional mean effective pressure ({.".o. orp_y' can be defined as : value, the arnount, inlet and outlet temperature ofcooling water and the $'eight ofexhaust gases are
*.o.p. = /-.".o. - B-.".o. ...(17.5) recorded. After calculating I.P. and B.P. the heat in different items is found as follows :
The torque and mean effective pressure are related by the engine size. IIea supplid by fuel
Since the power (P) ofan engine is dependent on its size and speed, therefore it is not possible
Forpetrol andoilengines,heatsupplied =mrx C,wheremrandC are mass used perminute (kg)
to compare engine on the basis ofeither power or tnrque. Mean effectiue pressure s the true ind.ica-
and lower calorifrc value (kJ or kcal) of the fuel rspectively.
tion of the relatue perfortnonce of d,ffireht engines.
For gos engines, heat supplied = V x C, where V and C is volume at N.T.P. (m3/min.) and
3. Specific ouput: lower calorific value of gas respectively.
It is defined as the brake output per unt of piston d.isplacemet and is given by : (i) Heat obsorbed in I.P.
Heat equivalent of I.P. (per minute) = I.P. x 60 kJ ...(17.9)
Speciflrc output = ** (ii) Heat tahen antsay by cooling uater
= Constant x prD x r.p.m. .(17.6) If n. = Mass of cooling water used per minute,
For the same piston displacement and brake mean effective pressure (pnb) an engine running r = Initial temperature of cooling water, and
at higher speed will give more outpu.
z = Final temperature of cooling water,
4. Volumeric efficiency : Then, heat taken away by water = ,rlu x c, x (tr- t1) ...(17.10)
It defined as the ratio of actual uolume (red.ucel to N.T.P.) of the charpe drawn in during the
i:s where, c, = specific heat ofwater.
suction stroke to the suept uolume of the piston.
(iii) Heat tahen outa! by efihausf gases
The average {alue of this efficiency is from 70 to 80 per cent but in case of supercharged.
engine it may be nTor than 100 per cent, if air at about atmospheric pressure is forced into the Ifrz" = N[ss ofexhaust gases (kg/min),
cylinder at a pressure greater than that ofair surrounding the engine. Mean specific heat at constant pressure,
cps =
f" = Temperature ofexhaust gases, and
5. Fuel-air ratio :
. = Room (or boiler house) temperature,
Its the ratio of the mass of fuel to the mass of air in the fuel-air mixture.
Then heat carried away by exhaust gase - me x cr"Q"- tr) .( 17.11)
Relatiue fuI air raio is defined as the ratio ofthe actual fuel-air ratio to that ofstoichiometric
fuel-air ratio required to burn the fuel supplied.

----t
I
540
NTERNAL coMBUsTroN ENcrNEs
*e TESTING AND pmFoRMANcE oF t.c. ENGII{ES "'
llilr mass of exhaugt gases can be
obtained bv adding together
mass of fuel supped and
mass of
.541
"o ""ooTtlll'
itil The.heat balange sheet from.the
above-data can be drawn
connection to the engine. Due !o the suction ofengine, there is a pressure depression in the air box
llii Item
as follows or chamber which causes the flow tbrough the ori6ce. For obtaining a steay flow, the volurne of
chamber should be sufciently large comparedwiththesweptvolumeofthecylinder;generally
ilti Heat supplied by fiut
Per ent
Air box
(i) Heat absorbed in Lp.
(ii) Heat taken away by cooling water
caried away by erhaust gases
<- Air in
lr:t] I*t
illi \u t tteat unaccouned for (by difference)

Total
i$!r
Exhaust smoke and other emissions
:
is an indication of
Smohe in
-- --ve..veE uu'ousElon' rt
iilil pollution
,,' control
contror is rhe cpnsiderao,lli;lir^:,:::3::lon. rr limits.the
the cos' limit^s.the output ofan
of an engi
ensine if air I
I irp
til
concern and wirh ,0" unro."jl3*lll;"::?:::r_1zzs.sjo1.ha.ve or lut" ""I_u
::::::il"':ft'*:ffi H"fr* jj.:ff
necessary to ew them as performu"""-p.""-"i""..
il.tt". I To engine <-
:{':"::ilT"1':il::l';:""}X1"fr:1ili":"T"flil:
. 9. Specific weigt:
as the weght of the engine in
ilifl ,rrr""'l1fo"ed kg for each B.p. deueroped. It Fig. 17.f. Ai-box metlod for measuring air.
is an nd.ation of the
500 to 600 times the swept volume. It ie assumed that the intermittent suctin of the engine will not
ir; r 7.3, BASIC MEASURE}ENTS I
effect the air pressure in tbe air box as volume ofthe box is sufficiently large, and pressure in the box
ili
lrl, evaluate the performance of an
engine folrowing basic measurem.nrs
remains same.
irl t"k"r, ,To are usuary under_ A water manometer is used to measure the preesure difference causing the flow through the
1. Speed orifice. The depression across the orifie should, noi exceed 100 to IS0 rnm of,ater.
3. Air consumption
2. Fuel consumption Let A = Aea oforifice, m2,.
4. Smoke density

il' 5. Exhaust gas analJsis


7. Indicated power and friction po.wer
9. Heat goingto exhaust.
6. Brake power
8. Hedt going to cooling water
d = Diameter of orifice, cm,
h. = Head of water ia cm causing the flow,
Cd = Coefficient of discharge for orifice,
l
1. Measuremont of sreed: po = Densif ofair in kglms uncler atmospheric conditions, and
The speed may be measured p. = Density of water in kg/m3.
bv: Head in metres of air (/f) is given by
(i) Revolution counters :
(ii) Mss.r..t tachometer
(ili) Electrical tachometer.
n.p"=
2. Fuel measurement: fu o,
The fuel consumed by an engine
can be measured by the ". H=Lr%=Lrlooo=10.-of"i,
(j) Fuel flow method - *" following methods
,.^-^-
(ii)
:
100 po 100 po po
(lii) continuous flow meters. Gravemetric method The velocity of air passing through the orifice is given by,

3. Measurenent of air consumption


The air consumption can be
(i) Air box method
measured by the following
:

methods
(ij) Viscous-flow air
:
The volume of air passing through
f"+
.orifice,
(i) Air bor method: meter.

. Fig. 17.1 shows the a


shapedted;;;;ik#;'il"?il,:*:1ft i" =co' x Co= Co! = 74 ACd
..#ffi ff T:iij:l;,il:;t5j:ffi n1:r";,1;
542 INTERNAL COMBUSTION ENCINES
TESTING AND PERFORMANCE OF I.C. ENGINES 543
,\
' =s4oACdJ}.t---t--
- lPo
() Absorption d,ynarnometers. Absorption rlynomometers are those that absorb the power
to be measured by friction. The power absorbed in fiction is finally dissipated in the form ofheat
ener-s/.
Mass of air passing through the orifice is given by
Common forms of absorption dynamombters are :
*s2 brake brake
m-=V-p^=L4x
a a'a --Y-xC, -Prony -Rope
4xlooz brake bake
-Hydraulic -Fan
brake dynamometers
=0.0011 co^a2 ffikels -Electrical
(o) Eddy current dynamometer
= 0.066 Cd , d2 ffi Ug,hin. ..(17.r2) (b) Swinging lield d.c. dynamometer.
(ii) Viscous-flou air meter: (ii) Transmission d,ynamomeers. These are also called fr rquemeters.These are very accu-
rate and are used whee continuous transmission ofload is necessary. These are usecl mainlv in
lt is not subjected to the erors of
Alcock viscous-flow air meter is another desiga ofair meter.
automotic units.
the simple types offlow meters. With the air-box the flow is proportional to the square root ofthe
Here we shall discuss.Sope brahe dynamometer only :
pressure difference across the orifice. With the Alcock meter the air flows through a form ofhoney-
Rop e b rah e dynamomc t e r :
comb so that flow is viscous. The resistance of the element is d.irectly proportional to the air uelocity
and is measured by means of an inclined manomefer. Felt pads are fitted in the manometer connec- Refer Fig. 17.3. A rope is wound round the circumference of the brake wheel. To prevent the
tions to damp out fluctuations. The meter is shown in Fig. 1?.2. rope from slipping small wooden blocks (not shown in the Fig. 1?.3) are laced to rope. To one encl of
the rope is attached a spring balance (S) and the other nd carries the load (W). The speed of the
The accuracy is improved by fitting a dampingvessel between the meter and the engine to engine is noted from the tachometer (revolution counter).
reduce the effect ofnulsations.
Filter Honey comb

To engine or
__-----__,-___--)
intrmediate
damping vessel

lnclingd manometer

4.
Fig. 17.2. Alcock viscous-flm a metr.
Measurement of exhaust smoke :
7*A*,
The following smoke meters are used :
- (i) Bosch smoke meter (ii) Hatridge smoke meter
(rtt) PHS smoke meter.
5. Measurment of exhaust em.ission:
Substances which are emitted, to the attnosphere from any opening down streatn of the exhaust
part of the engine are tertned, as 'exhaust emissions". Some of the more commonly used instruments
for measuring exhaust components are given below :
(i) Flame iolisation detector (ii) Spectroscopic analysers
(ili) Gas chroniatography.
6. Measurernent of B.P. :
The B.P. ofan engine can be determined by a brake ofsome kind applied to the brake pulley
of the engine. The arrangement for determinaion of B.P. of the engine is known as d,ynannoneter.
The dynamometers aie classified into following two classes :
(i) Absorption dynamometers (ii) Transmission dynamometers Fig. 17,3. Rope brake dlnamometer.

I
I
I
]
I

IITERNAL COMBUSTION ENGINES
If, W = Weight at the end of the
rope, N, AND PERFORMANCE OF .C. ENGINES
S = Spring bala." reaaiog,
f,
N = Engine speed, r.p.m., 3. Electrical indicators
D = Diameter of the brake In addition Lo tq optical ind,icators are also used.
wheel, m,
Calculation of indiaated potoer (1.p.) :
= Diameter of the rope, m, and
(Du + d) = Effective diameter of the brake If p_ = Indicaed mean effective pressure,
bar,
wheel,
Then worly'evolution A = Aea of piston, nr,
= Torque , ,.rgru_;;;a per revolution Z = Leneth ofstroke, m,
N = Speed of the engine, r.p.m., and
=(w_sr. fl"tg) ,
2 ) x2n =(W-SX'D +d)xn I = j for 4-stroke engine
Work doney'min.
= (W-S) I (P, a = I for 2-stroke engine,
(lY-S)n(D,+d)N Then, force on the piston =p- x x 105 N
Work done/sec.
=.-_ OU Work done per working stroke Force x length
(tY = of stroke
_ -S) r (Du + d)iv . ...
".n. 60x1000 Kw
...(17.13) work done per second = work d;:S*iltJ"'"*ilr of working srroke per second
(W -S) r D,tr
_
-o ifd =pmixLxr{ x 10s. ,* orJls
60 x 1000 ...... is neglected fi N_m./s

or (_ ?x2nM,,..) pmixIoANkxTOs
= -60;106- ,_r,
**
...Q7,141
Roo"
!,_on, i, ,op oo) 1 Indicated power, I.p. = &i1414 10 ,-,,,
^-
coefficient of the rope "tuitix "::;:::r",:J"
temperoture ery nrn,r,-
but not uery -r changes
A^^^..-- of
accurate because
in friction -- 6- " **
If is the number ofcylinders, then
Measuremen of Indicated power (J.p,)
:
I.p. = P 4JV x 10 kW
* rn" ;::f,"ili::lT::i'i" i::,"1$;jJl'"0* or at rhe piston is necessarly greate than rhar ( L7.16)
Morse test:
This test is only applicable to multi.cylinder
engnes.
,-,,. !d!"o,"do",",i","ffirlrfiiiii;,8;*p.j.d.ia\ramtakenwiththehetporan The engine is run at the required speed and
fr;:"'f;:X:XT ll$11TilT"-'annotj^9.meas".Ll li"*r" r, imade.possibre by measuring
.?Ijl,"liir:1" prugi-f engine is
thJtorque is measured. one cylinder is cut out,
the unde test. The speed 31s,r. "";;;;il;';;;*";;;;;;;1;;H l;;
faus because of the ross of iowei
c.r. engine is
*?h",.Trrf ##""r*";k#*.r:,*::ii"#*-*T:*ii.::;ffi
'-'n the tfu ;: red'ucing the toad. The toroue is 2ith one cyuiaer ctit ;";:
measured agri., when the speed-has .;";;;;r;;igrnar
the values of I.P. of the cvrinders. ae
is restored by
value. If
i.e. p,, = IglStexllllaga-r ir mm2
pressure mean effective (m e'p )
and the power losses in ach cyrinder
d"""il';,;;;ii, r.a rn t"*.idering a four-cylinder engine),
^
x sPring u." a""oi"i'' Ltr, Lr, Lrand zn, then the value ofB.p., B at
Length of d;s;l;;;
nringl. gi" ll "r'
-
constant
.
where
the test speed with all cylinders
p,,, is in bar.
B=(It- Lt) + ez- L) + (t -.Lr) + (14_ L1)
is grven in b.ar per mm of nu^lnr.1 cylinder ...(r)
uerticat mouement of the indicator is cut out, then the cnt.ibuton.f, is lost
$Ht#:stant ^-.r,-,rf remain the same as when it is firing,
cytlnder ; and ifthe rosses due to that
. "r,l;)rr.rtu, then te s.p., B, now obtained at the same specd
The main iypes ofengineindicators is
are : Br.=.(0_21) +er_Lr)+(Is_Ls)*(In_t
1. Fislon indicato Subtracting equation (tr) fom i), we-Cet' n) ...(i
2. ]l.;_-neti diaphragm type
indicator: B-Br=Il
Tire i-arnbrrough balanced engine Similarly, B -Br=Irwhencylindernumber2 iscurou ...(77.77)
.(j) indicator .
(li) Dickirson-Newell indicator u"l B - Bs = I s when cylinder number 3 is cut out
(iii) y11' balanced pressure indicator and B _ 84 = I4when cylinder number 4 is cu our.
(lu) Caoacitan"e_type balance Then, for the engine,
pressure indicator.
, =,ra* rr+ Ir+ Io
..---='---- ...( 17.18)
,/ 541
TESTINC AND PERFORMANCE OF I,C. ENGINES
INTERNAL COMBUSTION ENCINES
546 charnber. Thc exact value ofthis would depend upon the orifice size connecting the pre-combustion
chamber and the main chamber, and the speed. Higher the speed grcater is the loss and smaller the
Assurnptions: orifice size, greater is the loss.
has to remainsame in both
1. Speed ofengine and thottle opening or fuel injection setting
I"o.ijrrg
the cases offour .ylirrJ"." thre" working with one cylinder cut out. 5. Piston mechanieal frictbn:
"L,d "inders be kept constant' It can be sub-divided into
2. Frictional power and pumping porver are functions of speed and must
:

same quantity of fuel' (i) Viscous fiction


3. Same throttle opcning is expected to supply
or only three (ii) Non-viscous-friction-() friction due to ring tension.
4. cyiinder individually will develcp sarne power whether all four are working
with cylinder consuming power' () friction due to gas pressure forces behind the ring.
.yiittd".t are working one
The uiscos frcton dzpends upon the viscosigr of the oil and the temperature of the uarious
Litnitation.s : parts of the piston,
(three or more cylinders)'
1. This is applicable to multicylinder engines only () Piston Mechanical fiction can be sub-divided into :
2. Results are liable to errors due to change in mixtre distribution and other conditions by (i) Viscous friction
cutting out one cYlinder. (ii) Non-viscous friction
system which may
Cutting out of one cylinder may greatly affect the pulsations in- exhaust friction due to ring tension.
()
the engine performance by imposing different back pressures'
signifrcantly Jhange () friction due to gas pressure forces bebind the ring.
Engine Friction: The viscous friction depends upon the scosity ofthe oil and the temperature ofthe various
Frictional losses in an Engine parts ofthe piston. The degree to which the upper part ofthe piston can be lubicated also effects the
These are viscous friction. The oil film thickness between pistonpnd the cylinder is also affected by the piston
The total engine friction can be dividetl int'o five main componens'
:
side thrust and the resulting vibration.
1. Crankcase mechanical t'iction.
2. Blorv-by losses (compression-expansion pumping loss)' Effect of engine variebles on engine friction :
3. Exhaust and inlet system throttling losses' 1.Effect ofstrokelborc ro,tio. The effect ofstroke/ bore ratio on engine friction and economy
4. Combustion chamber pumping loop losses' is very small. High stroke / bore ratio engines have equally good friction mep values as that for low
5. Piston mechanical friction. stroke / bore ratio engine. Indications are that at high speeds the higher stroke / bore ratio engine
may be at some disadvantage.
1. Crankcose mechanical friction:
2.Effect ofcaindr size artd number ofcylinders. The friction and economy improves as
It can be further sub-divided into :
a smaller number oflarger cylinders are used. This is because the proportion between the working
(i) Bearing friction
piston area and its friction producing area, i.e. circumference is reduced.
(ii) Valve gear friction, and
3. Effect of piston nnga lhe effect of number of piston ring is not very critical and this
(iii) Pump anil miscellaneous friction' number is usually chosen on tbe basis ofcost, size and other requirements rather than on the basis
The bearing friction includes the friction due to main bearing connecting
roil bearing and
oil viscosity, the speed' friction.
r,of their eft'ect on
other bearings. Bearing friction is viscous i nature and depends upn the
size and geometrY of the journal'
4.Effect of comptession tu,tia Friction means effective pressure increases as the compres-
and no general equation sion ratio is increased. But the mechanical efficiency either remains the constant or improves as the
The valve gear friction losses vary with the engine clesign variables
compression ratio is increased. Ifthe displacement is varied to keep the maximum engine torque
is available predicting them'
constant, this results in beter part load friction characteristics.
2. Blou;-by losses :
piston and pston-rings, 5. Effect ofengine apeed- Engine friction l'ncreases rapidly as the speed increoses. The best
Ii is the phenom enon ofleakage of combustian prod'ucts past the .'way to improve mechanical efficiency at high sped is to increase the number of cylinders.
tn, an{ compression ratio' ?hese
,yli"rl" ti;;;;"k;;;;.Tlto.u'i5"r a"pu"d on tire inlet pressurecomptression
the ratio is ncreased';, " 6. Effect of oil uccocity, Higher the oil viscosty greater is the friction loss. The temperature
losseS uary as the square root of inlet pressure, ond. in"reo""'oS
ofthe oil in the crankcase signicantly affects the friction losses, wear and servic life ofan engine.
Blow-by lsses arqred.uced' as the engine speed' s ncreased'
.4s the oil tenTpera,turc increases the viscosty decreases and, friction losses are red,uced, iluring a cer-
3. Erhaust antd inlet throttling loes: , tgin tenPerature range.
a certain percentage
The standard practice ofsizing the exhaustvalve is to make them " 7 .Effect of cooling water tempera.ture, A rise in cooling water temperature redzces engine
than the inlet valves. This usually rults in an insufficiently sized exhaust valve and hence
frictionthrough its effect on oil viscosity.
in exhoust p umpng Io s s.
During starting operation the temperature ofboth the oil and the water is low, hence, the
4, Cotnbustion chomber putnpng loop losses viscosity is high. This results in high starting friction losses and rapid engine wear.
"
case ofpre-combustion chamber engines an additional loss occuls.
This is the
In tlie
the pre'combu
occurring due to the pu^ping worh requited' to iwnp gasses into and out of
r||r i 548
IMERNAL COMBUSNON ENGINES
|il i
g' Effect of engi*'"*.As
tendency to increase stishrty.
the load increases the maximum
pressure in the cynder
has a
AND PERFORMANCE OF I.C. ENGINES 54\
This results i;;;;;;;;r
friction uatues. For a throttled engine if such a trial s eorricd, out, the throttle position has to be uoried from
lilli Measuremen of
The frictional power of ao
frictional power (F.p.) : closed at not load. to full open at maximum load, to keep the engine speed consn. Therefore
nping load vrill be bigger at no load, and reduce gradually as the load is increased. In other
be determined b the following
"1{n"jln ' methods : the pumping pouer and, therefore the F.P, wIl not renain constant, as is the assumption in
o;i;;- -
1. Willan's line method (used
for C.t.
lliir 2. Morse test "ogin.. Iine method.
Limitations:
ililt 3. Motoring test
4. Difference between
The foilowing are the limitations of William's line method.
\
I.p. and B.p. 1. Applicable mainly to C.I. engines only.
ilfli 7. Willan,s line nethod :
At a ccrstant entr":r:o the road
2. The fuel-consumption- brake power line is not straight line, but turns up slightly at low
is.educed io increments and considerably near full load. Unless sufficient d,ata are taken to accurately plot th stright
gross fuer consumption-readings and the corresponding B.p. and
are takel e gr.pt iJrr."n ne portinn of the curue, the results will be signiftcantly in error,
llili 0f fuer consumption against
ll:ili ii+l_l',-'"ili
power
ffff:tiffi i:.Tjl1"*i,'H1'ine
u"j". uiy"*:g
{raw3
Gnarosou" to
B.p.
w'ranl iine ro a steam
2, Morse test :
In 'Morse test' (already discussed), frictional power can be found by subtracting (B.p.)^ from
ross ortr,u
n . tu;i;;;;;,o;:'rXt"lfff.T$:i?lHr?:l:
llild x?::illii,l,:|;tr?ffi::il.H?r* ;;;;;. r""l'Til. to be rinear, ten a er consumprion F'P'=(I'P'"-tg.p.
is the number of cylinders.

llill 3.

Motoring test:
In this test the engine is first run upto the desired speed by its own power and allowed to
,}
I remain under the given speed and load conditions for sometime so that oil, water and
lii'li I
c
engine component temperatures reach stable conditions. The rowe ofthe engine during
this period is absorbed by a dynamometer (usually of electricrl type). The fuel supply ii
.o
ii a then cut offand by suitable electric switching devices the dynamomel,er is converted to

{ill
,.1 I,
q
c
o
o
E run as a motor to drive o'motor' the engine at the same speed at u'hich it was previously
running. The power supply to the motor is measured which is a measure of F.p of the
englne.
o
lr o Motoring test gives a very good insight into the various causes of losses and is much
more powerful tool. ?lis test giues a higher ualue cf F.P. as compared, to that giuen by
ir'ii WiLIian's method..

lii' 4. Difference betueen I.P. and B,P, :


Tle method offinding the F.P. by finding the difference betrveen I.P. as obtaine{ from an
indicator diagram, and B.P. as obtained by a dynamometer is the id.ea[ method. However, ilue to
difficulties in obtaining accurate indicator diagrams, especially at high engine speeds, this rneihod
is usually only used in research laboratories and its use at commercjal level is verv limited.

r?.4. NNGINE PER,FOR}ANCE CURVES


B. p __-_ o The following parameters are of importance
- Torque
Power
Fig. 17.4. Willan's line method. - Specific fuel consumption and
This method is used oaly jn case_of its inverse
unthrottled engines as discussed - I.C. engine efficiency
The Willians line is plo below : -
load) or constdnt ror;!f"d^l:1^T-"] ::tt"Tttjon versus-load (from no load to rnean effective pressure is defined as brake porver/sv,'ept volurne x cy-ci;c
;; ;;H;;; "ff
;:"';?;X'""ff 'i"":,"i,l$',i::.:ifl" #,:lii:";i*l
assumed constant from nor^oad-t"
rui Ir"J 1i1"11:
full -Brake
frequency. In this relation swept volume is constant. 1r.lso bralic power = torque x einrlul:ir
speed' thus at constant speed, the b.e.m.p. is directly proportional to torquc and ritirer
not only the mechanical friction, .o"ed. The F.p. incluoes may be used. The taQio of brake mean effective pressure r,o the irdicatecl mean effectrve
U.,t also "iiJri"n.r".r,
pumfin" oo*u". pressure (from indicat\diagram) may seem to bc oqrral to the ratio of hake por,,,cr to
the indicated power whic\s defined as mechanical efficiency. Thus inclicator dirgram
and the output torque ma\be connected with a suitable llorvance foi- inrchrnjcal
efficiency.
\
I

=j"..
TESTINC AND PERFORMANCE OF I,C. ENGINES ))l
550 INTERNAL COMEUSTON ENGNES

The torque-speed relations (Fig. 17 bit a curve even though it would seen reasonable ignition setting or speed may by ascertained by
to expect the mean effective press to be constant at all speeds. However, at low speeds producing a new family ofcurves. An altemative method
leakage through valves etc ofgreater significance so that the m.e.p. falls and at of plotting these prameters is to use the air-fuel ratio
high speeds the volunetic causes the induced mass to fall with a parallel fall as the abscissa. Here it can be seen that maximum
in m.e.p. The power curves are product of the torque curues wth speed. economy occurs with a slightly weak mixture' This i
means that there is excess air and combustion is
complete. Maximum power occurs with a slightly rich Z
mixture when all the available oxygen is used. The I.C.
engine efficiency is the inverse of the specific fuel
consumption with the constant calorific value as a
factor. Thus the curves of specific fuel consumption
o
E (s.f.c.) also epresent efftciency. The maximum value of
brake I.C. engine efiiciency for S.I. and C.I. engines are bmep ---->
o
3 of the oder 357o ad 407o respectively. Fig. 1?.7. Specifrc fuel consumption-
C.I. engine. The flat curve of Fig. 17.7 illustrates brake mem effective pressure curve for
that atpart load the compression ignition engine is more the C.I. engine.
economiol than the spork ignition er4ne. This is the
benefit of quality control rather than quantity control of power.

Soeed -------> 1 7.5. COMPARISON OF PETROL AND DIESEL ENGINES_FUEL CONSUMPTION LOAI)
OLTTPUTS AND EXIIAUST COMPOSITION
Fig.1?.5. Power-speed and torque-sped curyes for the I.C. engine'
L Fuel Consumption:
Specific fuel consumption relations :
Fig. 1?.8 shows fuel consumption loops, for both petrol and diesel engines, plotted on a base
S.I. Engines : Refer. Fig. 1?.6. The cuwes are plotted for constant throttle opening,
of brake mean effective pressure (b'm.e.p).
constant speed. and constont gnition setting, The only uariable is the air-fuel raio. The eflect of
Bch o = Excessively rich mixture gives slow and unstable
combustion.
! Petrol engino
= Muimum b.m.e.p. with something like 1G-207o rich
{
=o mixture.
c = Conect stoichiometric mixture of 14.7 : 1 by weight
+
E d = Maximum thermal efficiency with something like
I
IO-2|Vo weak mixture (approaches ideal constant
E
volume combustion)'
c
o '$*, e = Excessively weak mixtue gives slow buming and
popping back through a intake.
Stochiometric
.9
=oo
f
'b& \s4
-\-------i't /= Maimum b.m.e.p. with satisfactory clear exhaust
requies mixture streng of about 18 : 1 by weight.
bmep ------) bmep -----f g-h = Muimum thermal efliciency, minimu specic fuel
(a) (b) consumption ranges between 5O-85% of muimum
b.m.e.p.
0 i = No-load Oow speed idle) requires mixtue strength
o 25 50 75 100 10G-75:1 byweight.

\ Fig. 17.8. Comparision offuel consumption


loops for petrol md diesel engines on a
base of engine load (b.m.e.P.).
In case of a diesel engine, Ioad. and, speed output is controlled entirely by uarying the
quantity of fuel injected. into the cylinder wthout misft'ring occurring, that is, from
0-700Vo of the maximum b.m.e.p- developed.
Wsak Stochometrc Rich with the petrol engines, however, if there was no throttle (full thottle position) the
(c) effects ofvarying the mixture strength from the richest position (o) to the rveakest position
17.6. Specific fuel consumption-brake mean effective pressure cunes fo the S.I. engine.
INTERNAL COMBUSTION
ENGINES TESTNG AND PERFORMANCE OF I.C. ENGINES 553
(e) poduces a variations ofb.m..e.p. Qoad) on i
that is, from 75-L00% o only 259 with rnixture strengths greater than 207o rich, that is 18 : 1 by weight, so that no carbon
o u tp u t c o n t r o t o ^*'u.' ' -' p. ?r''i;"i?.!TlT"HtT
n n ot u! achieued olone "' i}"rli lff ;t monoxide (CO) is present in the exhaust (FiC. 17.10).
'oryl,s *" ^t ii"-ti""
bv
throttting th" " '**p*
^i*trru "oi
:i::t:ix,x:f:i:#ir:{'"i!'frf
17.8. ;ne !:";;:""";::::;;;;"".'::*,,""
listed consumption.loops are
on
18
I Diesel engine i Petrol engine
the right
"ij" "iii" ai*. i co.'
II. Load Ouputs: 15
21i
i\r/
o fn case ofadisel ensine al
io \
to no load.wherer. r orlh^9frnping renain reasonably constant
losses 12
from full load.
is re.duced due to the.^ c,mprng lossesfor apetrol erwine progress.iuely rtr",* tiJl.^,".p,
-".i,u,,i".i";;ffi :".ffi1":t'i1*1;*::::J1l*ll"l;:o'"i."iil'iJi"i"
l"z O
ttr.
i0
\
\
\
o,i l.o
:,:fi.1J.',?:311"':j#,i!:'1.:'"r" oi"'""1u-"iti;::Tl"T;T"'*::n:Tilf,i:,".7 .t:..; ;o
iv,
'i. i.,
;* ljn Hidf *ffift,Tr ::lff nff : jl;J";:;xs;ff t
jilrr"* l\
ico,'
! \,'
il'
0
i\ i.i',
4 100 19 17 15 13
I Diesel engine pehol engine
g 120 -,_ Air/fuel rato by weght
o
a
c) 100
Fig. 17.10. Comparison ofcomposition ofexhaust gases for petrol
aE' md diesel engines on a base ofair-fuel ratio.
9: 80 The carbon dioxide (COr) emission produced by the diesel engine relative to the petrol
f?' I - engine is always much lower, particularly as the engine load is reduced, whereas the
oc o petrol engine in the stoichimetic (14.7 : 1) band operates with the highesi level of COr.
60 e
(- o
9e
Ev o 17.6. GOVERNING OF I.C. ENGINE
o
J
40 Exhaust
emmission
-F-}l a
E I The function ofa governor is to keep the speed of engine constant irrespective of the changes
d)
20
I liml for i jn load on the engine. The governor is usually ofcentrifugal type.
1
dies'et i
engines i In petrol engine, the control is exercised by means ofa throttle valve which is placed in intake
0
manifold. The quantity ofmixture entering the cylinder depends on the amount of opening ofthrot-
I
t9 17 15 13 1Cl tle valve. The position of throttle valve is controlled by the governor (centrifugal type). In diesel
Airl tuel ralo by weight ___> engines, the flow offuel is controlled by centrifugal governor which actuates link rods which in tr.rrn
Fig' 17'g' comparisou ofload (b.rn'e.p.) operate some device on the fuel pump and consequently portion of the fuel by passes. The governor
fo petrol and diesel engine
on base ofair-fuer ratio.
o A petror engne can
effectivery under steady
in plunger type injection pump alters the relative angular position of the plunger.
.op-erate
ransins from 20 : I.ti t0 : conditions oyer a nixture strength Following are the methods ofgoverning I.C. engines
L;;;;;;"'";;;iJi i"h cannormauy otnly utilize 80vo
:

"J';:tir;:Yf;rith o '"o'nobiv "t,,J,l'i',',,, "ngir".*t


,iu operate from r8 : ri that is 2avo weah (i) Hit and miss method
lt 2o;Eo hishe; m'a;i^1{i.::::! fiXtri'Ji,ljy; ll:j:;"' e".1e
*i!'"1*"s have a 15 to (jj) Qualitygoverning
oesel engine (Fig r7'9)' (iii) Quantity governing
Irr. Exhaust composition
(i) Hii, and miss method. Refer Fig. 17.11. When the speed increases the permissible value
,"
" ul)r.ll,,l!,iffiil","f,T:':: ::{;ff:i;!i:,l":##1::f::1s for pe,ot and dieset engines
the governor sleeve S gets lifted up, as a esult of whiclt'he lever lifts the distant piece B, so that
the peckerK misses it. Thus the gas inJgfal*lfl---oo not open and the usual charge does not eni,er
- orcabon monoxide (co) res the cylinder. This continues un-ti}-thlspeed is reduced andB occupies its initial position. Explosions
ff'.iT*:3'i":iTr:"l:',Hii}:,nlllr
10:lrhisb.i"s1;1!,^13'";;;,sr":iJ-;t;rt,:J.n-.t:'r::llHlilXffi
5 h.

and full power nditions. In


contrast-th" diur"t
j are thus missed intermittently but every charge is of normal strength. This method is commanly
used, in gas engines,

"nginn
":iiil
under fufl loail n-ever opeates
T-__
554 INTERNAL COMBUSTION ENGNES 555
OF I'C' ENCINES
TESTINO AND PEI(FORMANCE
ensine vibrations for foequencies
to give the flywheel the proper stiffness to absorb
wit"" J"J"."tion.occurs the flywheel fluid flows to other locations,
at that condition.
stiffness of tne flywheel and making it more absorbent to the
changing the overalt
new vibration frequencY'
connecting the engine-to the auto'
Some automobiles have lrydrou lic engine mounts
bod.y. Fluicl n thes'e mttunts aJts b absorb and. danpen engne uibrations and'
mobte
'""ir1n"
fr"* ;";;;;;;;"r compartment' Engine mounts using' electrorheo'
better vbration dompening at
;"ii"l i" * uni, a"uaol^ent which wi-ll altow by as much as a
all frequencies. T}te vis""sity of these fluids can be changed
voltage. Engine noise (vibration)
with the application of an extern-al
factor of 50 : 1
engine manage-
i, .""Jy o"""tnri'tyii, which feed this information into the
and proper voltage is applied
ment system ervrsl. ri"t" lUe frequency is-analysed
time is of the order
Response
to the engine noun; tol;;tdtmp"tt thut frequncy'
of 0.005 second.
'an!inois,e-:1.:?:::L-":t-,""*t""
.- Active noise abatemen is accomplish edbv generating analyztng the lrequency
This is done by sensl ng the noise-with a receiver'
but out^of.phase,with the origi-
the noise, ona tnro g"ii.lo]iiii nas"e of equal freq-uency'
"*ftu"tf""ise.
of
nol noise.If the noises "";;td" tutu i""'q"""tyThis b 180' out ofpase' the wave fronts
tte noise is eliminted' method works well with constant
concel eoch other and
fi *lttir"s aaditiottal electronic equipment (receiver' fe-
speed automobil"
".,gi""t'
quency analyzer' transmitter) than that used with normal
EMS computers'
automobiles have receivers ancl transmitters mounted under seats in the
Some
Fig. 17.11. Hit ard miss gweming. passenger ao-putt-"ttt t u" engine noise abatement system' Similar sys-
""tit"
ofthe tail pipe, a major source ofengine-related noise'
(li) Quality governing. ln this method ofgoverning, the mixture strength is altered.In gas tems are us"d near the enil
are now equipped with a
engine it is effected by reducing the amount of gas supplied to the engine. This is accomplished by Noise reduction has been so successful that some automobiles that the safety switch is
,tu,io' l idle the engine is so quiet
varying the lift of the gas valve. In oil engines, quality goveming is carded out by varying the safety switch on trr" speed,
engine when it is already running'
quality of fuel oil entering the cylinder per cycle, it is done by changing the angular position of the required to keep drivers from trying to start the
helical groove ofthe pump plunger.
In this type of gouerning, the ignitian is not alwa.ys satisfa4tory and thermal efficiency is WORJ(EDEXAMPLES
reduced.
(iii) Quantity governing. Hete mi.rture strength remaning tlrc sam.e, tlte quantty of mx-
ture enterng the cylinder is altered. When the speed is too high a lesser amount of charge is admit- Exampleil1.!.Atwo.strokecycleinternalcombustonenginehasameanffictivepressure
piston and' stroke are 170 mm
ted into the cylinder. The compression ratio and air standard efficiency remain unchanged. The bar. The speed of the engine i tOOl ''p'^' If the d'inmeter of
of 6
pressure after compression and during working stroke is lower, the less work thus obtained during and 140 mtn respectuely, fi,nd' the ittd'icated power dcueloped'
the cycle reduces the speed. This method is preferred, for large engines. Solution. Mean effective pressure (indicated)' Pn = 6 bar
Engine speed, 1t/ = 1000 r.p.m.
17.7. NOISE ABATEMENT
Diameter of the piston, D = 110 mrr = 0'11 m
A lot of research and developement is in progress towards reducing engine and erhause L=140mm=0.14m
noise. This can be accomplished by the following three ways: Stroke length,
1. Passive 2. Semi-active 3. Active Indicaed Power develored, P:
,.".= s'1#ll9 **
o Noise re{uction is accomplished passively by correct design dnd the use of proper
materials.' Indicated power,
The use of rbs and stiffners, cotnposite nterials, and sand,wch construction is Here, = No. of cYlinders = 1'
now routtu. This type ofcoostruction reduces noise uibrations in the uarious engine
components.
and ft = 1 ,.,.. for 2-stroke cycle engine.

t In semi-active nose abatement systems, hydraulics are often used. 1 x 6 x 0-14 x 4 x(0trf x1000x 1 x 10
(Ans')
Some engines are equipped with flywheels which have hydraulic passages through ... r.p.= #-=13.3kw.
which fluid flows. At idle and other constant-speed operation, the system is designed
NTERNAL coMBusrtoN TESTING AND PERFORMANCE OF I.C. ENGINES
ExqDple r7.2. A 4-nli.. r^,.-
-. petrol er4
ENctNEs
mean effectiue pressure
is u.!':P":P*:strohe
bar. calculate rh;i;";;;;fi'e deuelops 14.7 htil at
1oo0 r.p.m. The D, L, displacement :
Ls r.o ttmes the
bore. strohe of the enginc,
if the length of stroke
Solution. Number of cylinder,
Power developed, n 4 =
Powerdevelopeo=
' -W!t kw = P-H*"10
60x1000--" kw
6
r"giourp.Jl ' P=14'?kw
2nNTb p^LANhxI}
rndicaa rreln efrecti .iv = 1000 r.o.m. L'', 6011000 = --
vepressrne' 6
Lengthofstroke, P"=5'5bar Substituting the values, we get
Z=1.5D(bore) )
For four stmke cycle, k=
2x3000x160 s6xDxlxD2 xB00ox j x10
+ 60 1O0o =
6
L:,D r
"
50.265 = 18849.6 '3
Indicated, power d.eueloped, I.p, = !L. Wk rto kw t -^^^- ;U3
6-----.- D= (,#ffi] = 0 1387 m or 138'7 urm'
+ x s.s x t.so x(I x n2)x j
noox x ro L=D=138.7mm. (Ars.)
14.7 =
'-........-.....".- n-a
6 Displacement
D3_ 74.7x6x4x2 *,, = 7Dz x L = i x 0.13822 x 0.1387
,< r.si xl6oo- xld. = 0.0006806
s.s x = 0.002095 m3. (Ans.)
and =
I'3 = 1.6
-0._08?9
or 8Z.O mm. (As..r " tuits
Example 17.5, A turbocharged. st-cylind.er desel engine has the fottowing performance d.e.

]".^XTjr:
t7.5.A singte-cylind.er,
x 87.9 tgl.g mm.
=
tot'r'strohe
fo_ur-stron"
(Ans.)
f
; ( Work done during compression and, expanson g2O kW
=
Xt1r" tr"k;;;;;;;T'7:I"rtuer' cvs! o;;f
oiil sae is (ii) Worh done during intake and, ethaust
!1yit71
!I,!:,!':tu
is 200 N and
*heet s 600 mm and, the "yrt" "n"
roDe d,iantot.-,--]!'_l:
fi2:d with a rope brake. The
fitted =50kw
power
tk"
oower of the engine ' -'4 ev tv' tr ii; r";;;;;"::iff;,Ji
:"fl?!,:::j::,i;';:;:::::,K;;il,^,;;;:::;i:::::::4:i";;#;;i::i
"o,i";;;;;;;;":#;;i:?;;;::::i7y: tne engine runs at?u.y: the brake (iii) Rubbing frctnn n the engine
= 150 kw
enginz ?? 4s0 r.p.m., ,h;;;iu "" (iu) Network d,one by turbne
be the brahe
Solution. Diameter of =40kW
the b If.the brahe mean effect_iue pressure is 0.6 MPa, d.etermine the bore and. strohe ol. the engile
nop" aia-ut"",;;# Tl;;T",,D = 6oo mm =0.6m tahing the ratio of bore to strohe ai 1 and engine ,p"r o" fi00 r.p,m. (GATE_lggg)
Dead load on the brake,
W= 200 Solution. Guen : p^u= 0.6 Mp, - o7
spring balance readin,
S = g0 N
6 bar ; = t t * =1ooo r.p.m
Engine speed,
l/ = 450 r.p.m. D, L:
Brake power, B.p. Net wok available = 820 (50 + 1S0 + 40) = b80 kW
: -
nx p* IANhxtO
=," -iJ:,130.r," o*
Brake power is given
by, B.p. B.p =
6

_ (200 - B0)n(0.6 + 0.026)


6x6xDxID2 x1000 xl xtO
x 450 580 =
rodo-...---
= 2'5 kW. (Ans.)
= 23562 Ds
four -. ,. , -
.Example 17,4.4 60
"
four-stroke, spark ignition
torq^ue of rco im;;;;;;: "t':der, engine d.euetsps s matmum r s8o \v3
r-'"=\zssn)
errectiue pressu,, brahe =02908m or 2908mm
",ii, ^,I#::;:: l:;::;;:,,;:,X;;i:::lXif:;r:.*:";yii,i;:: HenceD =L=290.8mm. (Ans.)
Solution. Giuen : n 4 (GATE-1992) Example 17.6.A sparh-ignition engine, designed to run on octane @
= ;h_ j tuS:irr" being 4-stroke
cycle), T, metho_ne,(CH)- Estimate the ratio of the power npit o rH r) fuel, s operated, ctrt
= 1i0 Nm, the engne wth metLaii fuel.to that with oc-
N = 3000 r.p.m.,pa tane' In both cases the fuel rato is stoichiometric, the
mixture ii supplied to the ergir{ot the ,o^"
= 960 kPa = 960 x 10s N/m2
= 9.6 bar; D = | ditios, the engine runs at the sarne speed., and. has the
satne uofu'etric and thefrat efficiencies. "or-
The
heatingualueofmethaneisS0ts0hJ/kgwhitethotofoctanein4teeolJili,-,r--i.p.S.C.-rSsl
558 INTERNAL COVIBUSTION ENCINES TESTING AND PERFORMANCE OF I.C. ENGINES 559

Solution. In spark ignition ne, the air standard efficiency is given as : So,lut'ion. Torque due to brake load, ? = 1?5 N-m
I Engine epeed, N = 600 r.p.m.
-1 Brake power, B.P.:
(r)' -

where is the compression ratio which depehds on engine parameters and has no relevance to fuel
e.9., quantity, type, calorific value etc.
Brake power, 3.p. = -?4v!- =
?%*!9fr#A= e.l6 krv. (Ane.)
Indicated power of engine is given by : Eranple 17.8. Following servatinns were record,ed during a test on a singte qlinder oil
,.".= e-Trlo r.w Bore=300mm;strohe=450mm;speed.=J00r.p.m.;i.m.e.p.=6bar;netbraheload=
when tr (length), A (area) and N (r.p.m.) depend on engine construction. 1,5 kN ; brake d,rum ditmnter = 1,8 mctres ; brake rope d,iameter = 2 cn.
(mean effective pressure) depends on engine operation, Calculate: (i) Ind.icater power ; (ii) Brahe power; (iii) Mechanicat efficiency.
OnIy p
^
For air-standard (AMIE, \Yinter lg06)
Otto cycle,
Solution. Bore ofengine cylinder, D = 300 mm = 0.8 m
Stroke length, L=450mm=0.45m
'^=#*,*'r4;=*# Engine speed, N = 300 r.p.m.
Indicated mean efrecivepressure, p_ = 6 ba
where Qn is the energy supplied.
Net brake load, (W-$ = r.5 kN
Since Q, in proportional to the mass of the fuel supplied times calorific value of Diametr of brake drum, D=1'8m
fuel, therefore Brake rope diameter, d =2cm = 0.02 m
(i) Indicated power, I.P. :
(Power)*rn"o Cfftr,.n"
(Porver)06-"- Coa* = +g = l.1l?. (Ans.)
n p ,IANhxlO
44880 1
lP. = -* [where = ;...... four-stroke engine and n = no. of cylindersl
Example 17 .7. A large diesel engine run on four-stroke cycle at 2000 r.p.m. The engine lns a
e
displacement of 25 litres and o brahe nean effectiue pressure of A.6 MN / m2. It consums 0.018 kg /
1x 6x 0.46 x 1 x 0.32 x 300 x f x 10
s of fuel (calorific value = 42000 kJ I kg). Determine the brake power and. brake thermal effi.ciency.
= 4 2
(GATE-re9e) 6 =4Z.ZlkW. (Ans.)
(ii) Brate power, B.P. :
Solution. Giuen : N =2000 r.p.m. ; I = {a-"t-k" cycle engine) ;
|
(W
- S) n (D^ + dW 15 x n(1.8 + 0.02) x 800
o...= -------E- = - 42.88 kW. (Ans.)
Displacement (Ax L)=25 litres= 25 x 10r = 0.025 m3 ip- = 0.6 MN/m2= 6bar - 60
r-*r:
(jij) Mechnical eciency,
r = 0.018 kg/s ; C = 42000 kJ/kg
q.*" =
B.P. 4288 .

Brake power, B,P, : Lp, = 4m = 0.8987 or 89.877o. (Ane.)


p^IANh xlO Example l7.lo. The power output of an I.c. engine is measured. bjr a roped, brahc dynamo-
B.P. = mter. Th d.iameter of thc brohe pulle is 700 mm and. the rope diameter is 25 mm. The loail on the
6
1
tight sdz of the rope is 50 kg mzss and, spring balance readsi1N. The engine runnirq at g00 r.p.m.
6x0.025x2000xix10 consumea fuel of calorific oalue of 44000 N I hg, at a rote of 4 kg / h.
= 250 kIV. (Ans.)
Assume g = 9.81 ml {. Calculote :
B.P. (i) Brahe specifrc fuel consunptian.
Brake thermal 6ffi ciency, nool= rrC (ii) Broke thermal efftcency. (GATts-l9S7)
Solution. Goen:Do= 700mm = 0.? m, d=25mm,= 0.025 m,
33'o?%' (Ans')
= *#ht- = o'3307 or W= 50 kS, S = 60 N; iV= 900 r.p.m. ; C = 44000 kJ/kg, tn- = 4ksh
Example 17.8. A rope brahe was used, to masure the brahe power of a single cylnder, four- (i) Brake rpecific fuel consumption, b.e.f.c :
,

strohe cycle petrol engine. It was found that the torque d.ue to brahe load. is 175 N-m and the engine
rnakes 500 r.p.m. Determine the broke power deueloped by the engine. 'r =
Brake power, n.p =
9:9*9*L{' kw
60; looo
-

I
I
NTERNAL COMBUSTION ENCINES
TESTING AND PERFORMANCE OF I.C. ENGINES
(5Ox 9.81- S0) x (0.2 + 0.02.5) x 900
_
60*1000_ = 15.0SkW .Dxanple l7.l2.A single'qrinder four-strohe d.ieser engine running ot Ig00 r.p.m. a bore
Brake ppecifiq fuel consumption of 8s ny^a1!ta glyke d Il0 mm..n.tahbs 0!6 hg iii, per minute and d,euerops a has power
brahe
output of.6 kw while the ar-fuel-ratiois 20J. nv calo;fic ialue ofthe
m (kc/ h) fuet usd, isizsso nJ rkg, and.
b.s, .c,=- < tlw ambient air density is 1. tg kg I ms, Calculate.
= mE = 066 kJ/hw'h. (Ans.) () The volumetrc fficency, and
(ii) Brake thernal efcienc5r, ror,
: Gi) Bruhe specific fuel consumption (GATT_19O6)
Solution. Giuen;N = 180Or.p.m.;D
(where mr= Fuel used, in kg/s) =g5mm = 0.0g5 m iL=lL}rn\n = 0.11 m;
Air flow rate, n = 0.66kg/min, ; B-p.:6 kW; Air-iuel ratio 20 : 1
= ;
r5.05 o = 1'18 tu/rns
=Ah600)x44000 =0.8028 or 80.?8%. (Ans.) (i) rhe vorume*r" .:;"::l,Y*.'
EyFle !7.ll.Afour-qtry:!1":*r* S.I..engine lws acompressionratio of g andbore Volume displacement tl ^" - /v
=7"-xtx,
{:"':#k:::"y::::'y::!::::?";;t;;;;;:#;;&;;":"1'";r#n:l"Yl",';;
The ens::p opeytes at aipeed of 4800;.;.;;;i';;';;"#;i;.
= (o.oesP x 0.11 x
?:X:H :**f* ::,_": :(p! _: !? ltn s :.*!1, r*"u a"*;,, = r. I 2 ts / r, man Mass of air
f, ' Y = 0.561?ms/min
f;,:r:#;n:#y"g:;*:",:"1:;,yl;tr;,;;#";':r;3&;"'':#",'n#:;
the indiated. thrmal efftciency and,
the b;;" pr;;, (GAIE.IC96)
= 0.5617 x 1.18 = 0.668 fu/rnin
0'56
..: Volumetric effici
ency= =0.845or&l.6vo. (Ans.)
Solution. Giuen : Number t
(ii) Brake cpecic fuel concudptlon
- (b.s.f.c.):
r = 8, D ='* 1#;:T: :";::;?:::.=T; T.=it":, _, 0'66 056
Air-fuel ratio = 15 ; C = 42 Mrkc; p t.l}k7ls
= I ,O Or.,
;p,o, rl = 8M.
Fuel consumption = 8"tu"1 *tt" = l- = 0.028 kg/min.
Idicated thermal efficiency, ra :
n, .'. Brake specific fuel consumption = &H$3* ks4w-h
Indicated power, np-il1Vx10
J.p.= 0.028 x 60
6

. .
r!*.ompre r1.r'. Forowine.*^
",u1,"
cylind.er d.iamcter 200 mm and piston stroke g50 m;.
'v'a'T ]I"',,*r,*
"[-,*,;l'#rY;li -*ae' o",,*
n.e.p. on couer side
= 6.5 bar
Air consumption itN ,n.e,p, on cranh side
=nx ZDz
*L" *n"o,. =7 bar
Speed
= 420 r,p.m.
=4.I x0.12x0.1x
ffxg.zs
Diatneter of piston rod.
Dead. load. on the brake
mm =20
= 5.655 rn3/min = 0.09425 m3/s = 1370 N
Spring balance reading
Mass flow of air, o = 0.09426 x 1,12 0.1056 kg/s =145N
= Brahe whzel damcter
=1.2m
Fuel cons"mption, ; 0.1066 0.1056 Brahe rope d,inmeter
=20mm
"7 = E tu;l = -r;'- = o'oo7o4 kc/s Calculate the mechanical efficenc1 of tlrc engine.
'"ti" Solution. pni(wer) = 6,5 bar,p.uo"nu = ? bar, D = 0.2 m, L 0.85 m,
I.P. =
''trD - , xC N = 420 r.p.m., d, = 20 mm = 0.02 m, W= 13?0 N, S
=t N,
_ 126.66x103 _,0.428 ot 42.6%. (Ans.) Dt = 7.2m, d = o.o'z n, n = 4-stroke cycle engine
- 0.00?04;td = f,...
Brake power, B.p. Mechanical elticiency i lq. :
Area ofcylinder on cover end side,
Brake power
= Indicated power x noeh A*u p2
= trl4 = (tc14) x (O.2)2
= 0.081+i m2
= 125.66 x 0.8 = tm.53 kW. (AnsJ
li
INTERNAL COMBUSTION ENCfNES
TESTING AND PERFORMANCE OF I.C. ENCINES
Effective area of cylinder on crank end side,
p. (ba4 ii
A"""*= trtl (D2 - d1.n) = t4 (0.22 - o.o22) = 0.0311 m?
?
Indicated po\\rer on cover end side, il

Ptcreo 11z4Ni x 10
I'P'(*"") -
li

6.5x 035 x 0.03141 x 420x I x10


=+=2skw
Indicated power on crank end side,
ii,
P-<ea*l ILAJVi x 1o
I Pr*' rl - 6 iit

? x 0.35 x o-0311 x 420 x I x to


= : 2 =26.6?kw
6
Total I.P.=V5+26.67 = 51.6?kW

Now, brakepower,
t"-:jil?#'* =
(1370 - 145h(L2 + 0.02) x 420

"".= 60 x 1000
= 32.86 kW Fig.17.12
Mechanical efiici.ency,r1^^= 6s.59za (Ans.) p2= p1x 8.78 = 0.9 x 8.78 = 7.9 bar
?* = ## = 0.68s9 =
To find po, considering erponsian process 3-4, we have
Example 17.14. The following data refer to an oil enginz worhing on Otto four-stroke qrcle :
Brahe power PsYs73 = PnVrl3
= 14.7 hW
Suction pressure 2.. r1.3
= 4.9 bar !!-l\l
Mechanicol efftcienty -,".,,.,-o,
=80% Pr- lvs)
Ratio of contpressian =5
Ind,er of compressian cupte p.=
Inder ofetponsion curue
= 7.35
= 1.3
#= ff =r.se0""
Matimum explosion pressure Work doney'cycle = Aea 1-2-3-4
= 24 bar
Engine speed'. = Aea under the curve 3-4 - aea under the curve 1-2
= I(N0 r.p.m.
Ratio of stroke : bore = 1.5 _ - pa,V _ pzVz - ptVt
psVe
Find, the d,iameter and strohe of the pisnn. - 13-1 1.35-1
Solution. Refer Fig. 17.12. psVs - pa,Va, _ pzVg - ptV
_
= o.3 - 0.36- Vt=V+and'Vz=Vs)
, B.P. = 14.7 kW, pr = 0.9 bar, l_*r = 807o, = 5, ps = Z4bar

rV= lflDr.p.m., _ 1o6e4va-2.9v) _ 105(z.9v3 -0.9v)


* =r.u
0.3 0.35
[=f' L*i = 106 (80y3 - 9.86V4) - (22.57Vs- 2.57V))
\
v,
Compression ratio, -- v"-
'- -vo
v,
= 105(80V, - 9.86V4- 22.57Va + 2.57Vr)
= 106(57.43Vs -7.29V4)
To find pr, considering cornpressiott process -l-2, we have

o
P1Y1r'36 = PrV"r'36

tP1= f+1"'=(b),,35-8.?8
= 105(57.43Vs -
= 105 x 20.98 V, N-m.
7.29 x 1Vr)
Ia'l
\vz)
INTERNAL COMBUSTION
Mean effective pressure (theoretical), ENGNES
TsflNG AND PERFoRMANCE oF I.c. ENGINES

,.=#t#ffi (idi) Mechanical efficiency,

=rr{} ='t*#
105 x 20

= 106 x 5.245 /m2 or 6.245 ba.


n-"*.= H= # =o'*or8c'6%. (Ans.)
n.*n = f#
.'. To- B'P' t4.7
rn*r, =Tt=r8'87tw.
To find D and L: (ii) Brake thernal
Solutiou. Diameter "ffici"ncy,
p^,IANhxt0
*_;--kw of brake pulley, Do = 750 mm = o.?5 m
Rope diameter, d= 50mm=0.05m
Dead load, !t/ = 400 lV
5.248 xL6D x I 1 x.D2 xto00 Spriag balance readh g,
,n., _ x x Lo S = 50iV
6
t Consumption of fuel 4.2kcth
=
Raed speed,
D3 =
*a*i*ffi;,= 0.002856 Calorific value of fuel,
({-f]'!?lldw
N = 1000 r.p.m.
C = 48900 kJ/ks

= l;T,jnol?::,onm. (Ang.) Brake power, sr.= ,.w


Exanpre
(Ane.) -n' 60 x 1000
17.16.rhe fouo;,:::]:=i8
**t" (400
- 50)r(0.?5 + 0.05) x 1000
=-=Ia.66kW.
!d1ate/ wwe" :;;';;|n
a petrot ensirc : on
Engine speed. mT;:t
. :.T Py*- = 26 kW ; (j) Broke specifrc fuel consumpion,
60x 1000

cahritu uanu of ,i r|J !:!^ ;.^^^pnr brahe'power hour = o.ss'kc ;


= 1000 r.o

".,*io,,, 6 i' ;#";;i ;, #"H:h,


c e'f.c. (brake) = 0.286 kg/ktvh. (Ang.)
(ii).,The brahe #=
thermal effuiency, and (i) Brake thernal efficiency,
t mcc hanical efficicncy.
:" ! :"
Solution. Indicated
Brake power,
power, I.p. = B0 kW n*,",=t'*=#h,
B'P' = 25ql
Engine speed,
Iv = lo(x) r'p'rr. f
F p"" l;;;.power hour (
= Fuel used in kg/s)
c"rojn..,auJ;'#ft",
"l
: liH#rt = 0.286 or 28.6%. (Ans.)
Now, fuel consumption per oTple
h " = o'* x 26 - e.l ks/h. .r7. A si'x-cylinder, four-stroke 'petrol engine, hauing a bore g0
17
til IndicatJiffi;,Tffi:L"", of 700 mm has a cotnpresson-ratio of of mm and, strohe
7. Th" rebtlue efficiency with referenee to ind,icated thermal
effitienq is 55vo tttltei thc ind.icated-1p."-n"-f""i
,"^#ofion is 0.8 kg t kwh. Estinate the calorific
n*u=iib=rEl3;
- I,P.
=o.27 or 27%. (Ana.)
u_alue of the
r'P'n'
fuet and fuel consurnption (in i|/h), giv"r'thot the inup is g.S bar ond speed is 2500
(AMIE Summer, 1999)
(md )x 439oo
Soluon. Number of clinders,
lji) Brake therrnar n=6
"*"r.o"", Bore ofeach cylinder, D=90mm=0.09m
Stroke length, tr=100mm=0.1 m
*tu'=
t'*, =
E*"* o.284 or 2s.4%, (Ana.) Compression ratio,
Relativeefficience, n _ilEo/^
'UUUxaBI Indicatesspecifictuelconsumprion'I6lfor"ri
:
i
Indicated mean effective pressure, imep g.6
I
- bar
't Engine speed, 1y = 2500 r.p.m.
)
56't
56 TNTERNAL CMBUSTION ENGINES TESTING AND PERFORMANCE OF I.C. ENGINES

Calorific value of fuel (C) and fuel consumpion (in Lg/h) : 37.5x6x4
Ds= = 0.0002387
4xBx1.6xx2500x10
D= 0.062mor62 nn. (Ane.)
rlirtnrad=t-
#- =1- #: =0.5'm8 or
and L=62x1.5=93nm. (Ane.)

Now, I'ehv = or rluo =fl'tu*x Qt"t' d*a (ii) Fuel consumpion:


#; (m, = Fuel used in kg/s)
Brake therrnal efficiency, rl*,", = #*
or Indicated thermal efficiency, rl,r = 0.55 x 0'5408 = 0.297
I.P, 1
o'29
30
But = trt $9oo
"c = (03/9600),..C
tl'<n =

r (AneJ
or =404o4kJ/kg. 6ns.) ... = m#ft55o =o.oo241ketaor8'78keh'
"= #*
o.2g? = -1991
piston displaenent of 700 cm|
no.IANkxl0 I Eranple f ?.19. A si'x'cylind'er, 4-strohe S! engin hauing a
Now, indicated power, t.P.= :lff [wherel = i- four-stroke cycleengine]
oer cylin&r d.eueloped 7g nw t 3200 r.p.m. ond. coumed.27 hg of petrol per haur. The calorific
'ualu
of petrot is
'U MJ lhg' Estimate :
6 x 8.6x oI x x 0.092 x 2500x xr0 ()Thevolumetrbeffi.cincloftheengirciftheair.fuelratinis12and,ntoheairiAat0.9bar'
_ * = 68.89 kW
6 32'c.
. Fuel consumption = 0.3 x 68.39 = 20.62 kg/h. (An6J (ii) The boke thrmal efficienq, and'
Eranple lr7.t&. A 4'eflindzr two'stroke qcle petrol en4ine &velops 30 kW at 2500 r'p'm' Qi The brake torque,
(AMIE Summer' 1998)
The mean effcoe pressure on each piston s I bar and mehanicol effriznc s 8Mo Calculate th For air, R = 0.287 hJ I hAK.
dianeter ald. stroh of eah cllind.er of stroke-to-bore ratia 1.5. Also calculate th finl consumption of Solution. Number of cYlindere =6
tte er8inc, if brahe thermal efftaizncx is 287o. The colorific ualue of thc fuel s 43900 kJ I ka. Piston displacement per cylinder = 700 cms or ?00 x 10-6 mi
Solution. Number of cylinder, n=4
Power develoPed, P=78kw
Brake power, B.P. = 30 kW N= 3200 r.p.m.
Speed ofthe engine'
Engine speed, JV = 2500 r.p.m. nr=27 kelh
Mass of fuel used,
Mean efective pressure P- = 8 bar Calorific value of fuel, C = 44MJtkS
Mechanical fficiency, I.s. = 807o Air fuel ratio ' =L2
Leugth ofstroke, Z,=1.5D(bore) pl=p") = 0.9 bar
Intake air Peasure'
Brake thermal efEciency, n*rr=28% Intake air temperatue' Tr= (To) = 32 + 273 = 305 K
Calorific value of the fuel, C = 43900 kJ/l(g R = 0.287 kJ&C K
For air.
h = L ....,, for 2-stroke cycle engine (i) Volumetrlc efllclency of the engine, r1-:
(i)L-?,D-? Mass of air, rc = Air fuel ratio x mass of fuel
B.P. =t2x27 =321kslh
rl_"*,. = [ mRT-
30 AIso, PoYo=mS,oT"*r"='t=t"
o'8 = LP
.'. Volume of intake air'
_--d6il6il-
yo= 324 x0287x 305 -= gt5.126 ms/h
"... !q kw
o'8 =-
r.p. =
\ ''''-
BZ.5
M
hour x no' of cylinder x x 60
nl .trAN x 10 Swept volurre per = Piston displacement per cynder 7
Also, I.P.=
- O
. ?oo x to-6 * 6, !?99 x 60= 409.2 m3/h
4 x I x lD x n | 4D2 x 2500.x 1 x 10 2
or.o=T Volume of intake air
.'. Volumetric efficiency, l'l*r. =
-;6t;lo-
IITERNAL COMBUSfION
ENCINES TEST'NG AND PtsRrcRMA{CE OF .q E{GINES
315J26
0'781 or ?8'174 (Ane') Number of cylinders, n=4
('i) Brake hermar rfP= Dngine to rea axle ratio
=89:8
"m"i.,.oy Wheel diameter wih tyre fully inflaed 650 mm or 0,65 m
=
n*=H"-*lffifr" 78 x 8600 Petrol consumption fo a distsce of 8.2 km at a speed of 4g kmth kg
r^fu= = O.ZZ7
= Mean effective pre$rt,
27 x(44XIOB) p_ = 5.6p5 bar
^ -3600 Calorific value of petrol, C = 49470kilk9,
(itt) lhe brake torque, T"
= 0.2SG4 0123. *o h 112 ,..... for 4,stroke engine
:
^"lr' Indtcted pgwer trp. =
2e 2nx3200xT"
:
P=
60
or Zg= - Speed of the car = 4s kn , n = nt
60
:l*o = 800 m/min.
,"= In N, are the revolutions rrade by the tyre per minute, then rDN. g00
m= osz8kl{/u (Ans.) =
t'= #* =392r'P'm'
*'.,"3-i,*^ilil;'!:iZ';H;!;."fyt:tr#:tr:.:*:_::-tuy"tumeorrTstitresdeuet.
tnsflres in one n inute. r tnc auerage nutnber of times As the rer ade ratio is 3g: g.
'rw each cyiinder
Solution. Number of cylinders, ..' jV" (speed of the engine sUan 4f;!9
Stroke volume, =6 = = 19lt .p.m.
1.75 tres !.75 x L0-
_IZ, =
s
Indicated power, = mg
I.P. = 253 qr P' /'{JVx10
Engine speed, u*
JV= 604 r.p.m. .P. - 6 --"
Mean effective pressure,
P;=6ba
Average number or rimes
.""r,
*""". _
4x 5.625 x 0.(D x /4 x 0.0752 x1911x
*
x 10

"u.lalr?*:1-H*: 6
r.p. = %ln'vx10 kI = 146 kW. (An.)
6_ lndicated therual efficiency :
6 x 6 x l.Z5 x 10-3 xN x]./ 2xIO
26.3 = To find indicated hermal efciency, let us
t'.' LA = V" = 1.?b x 10- 3 m3l find rz, first :

26.9x6x2x1000
f_-
." Speed ofthe car 48
oxbxls,lo- =5oor'P'm' = 60- = 0'8 km/min'
Actual number offires in one
minute =
ry "
6= 1SOO
Time for covering 3.2 km =
ffi = a.io.
Amount of fuel consumed in 4 min.

i:ffH:;:Jff:'
*" minure = g x 6 = 1512
.., Fuel consumeser=
=0.227 kg
0'227
kgs
= o'oooe46
:

= t5l2 _ 1500 _ 12. ' ;T'


I Auerage number of timcs each Now, Indicated, thermat fficizncy,
;I qtlind,er misfires in 12
Exampre r7.zr. r he I.P.
rtnor = ililC L425
fouo-wing data refe r to a *" :;,::T: ; f *=,k)!", = 0000146 x 4g4?0 = 0.346 or 34.67o. (Ans.)
I ,:t::i::?:\T;ili:J ;',1##;:n::il:iy;rtia 3e : I ; wheei diame,er with
tyre ru,y Erample 17,22. The following readings were taken d,uring the test of a sngle-cylindcr
48 km per hour ** km whn car was rnouing at four-
f;"".;;; be 0.227 hg. .''3'2 a speed of strohe oil engine :
. If fhe mean effective r Cylinder d,iameter
Itri "i,""y bou,ip"-;;x;;;;;:;:;::"::,';:1::::"rfru;H7:ndicated power and thermat effi. = 250mm
liri Solution. Bore,
Stroke length
= 400 nm

itrii
sroke rengrh, i:ffff,T;#r^"'
Gross m.e.p,
Pumping m.e.p.
=7bar
O.6 bar
=
TESTING AND PERFORMANCE OF I.C, ENGINES 571
570 INTERNAL COMBUSTION ENCINES
Brahe mean effectiue pressure,
Engine speed = 250 r.p.n.
, .- work done - 125a0 x 1090 7.69 bar. (Ans.)
Net load. on the brahe = 1080 N 'mo Cylinder volume 16.31x 10c =
Effectiue diameter of the broke = I.5 metres
Eranple 17,24. In a test on single-cylinder four-stroke qtcle gas engine wth erplosion in
FueI used per hour = 10hg euery cycle, the gas consurnption giuen by the meter uas 0.216 ma per minute ; the pressure and
Calorfic
tenperature of the gas beine 75 mm of water ond 17"C respectiuely. Air consurnption was 2.84 hg /
ua.Iue of fuel =4$MhJlks
Calculate : (i) Ind.catedpower ; (ii) Brake power ; mn., tle ternperature beng 17"c and barometer reading 745 mm of mercury. The bore of the engine
(iii) Mech.anical efficiencl ; (iu) Indcated thermal efficiency, was 250 mm and stroke 475 mtn and r.p.m- 240,
Solution. D = 250 mm =O.25 m,L= 400 mm = O. m,p*= Z bar, Find.uolumetric efficienca of the engincreferredtovolumc of charge atN.T.P'AssumeR for oir
as 287 N ml hg K'
p^o= O.lbar, V = 250 r.p.m., Do = 1.5 m, t, = O.OO2Z7 kc,ls Soluion. Gas consumption, V, = 0.216 m3/min.
*h =
C = 44300 kJ&c, n = 1, (W - S) = 1080 N Pressue of the gas = 75 mn of water
Net Pn= PiB-Pnp = 7 - 0.5 = 6.5 bar. Temperature ofgas, Tt=l'l +273=290I(
(i) Indicated power I.P. : Air consumption = 2.84 kg/min
Temperature of air =17 +273=29oll
x:ro lx 65x 0.4xl 4x 0.252x250x ]2 x 10 = 745 um II8
l.r.= np*LANh
"' Barometer reading
66 = kW = 26.59 kW. Bore ofthe engine, D=250mm=O.25m
(ii) Brake power, B.P.: Stroke ofengine, .L = 475 mm = 0.475 m
N = 240 r.p.m.
1080;#lr!!9 Engine speed,

"" ='taoil'#
kw= =212kw. R for air = 287 N-m/kgK.
(iii) Mechanical efllciency, Volumeric efiiciency, r"or. :
: 4-"*
B.P. 2L2 Pressure ofthe gas, p=?45+ JL = 750.5 mm of mercury.
=o'7e7 orTe',,o' (Ans')
(iu) rndicared r"",,-J];:,j*-*;tr AtN.T.P.
ofmercury
P2 = ?6O drm
I.P. _ 26.s
Tz=O+273=273IK
'u(r,= rxc o.oo277x443lD=0'2160r21'67"'
n ..-. (Ans')
vr=?
Example l723.The brake thermal efftciencX of a ilieset engine is 30 per cent. If the air-to-fuel To find volume of gas used at N,T.P. (Y2), using the relation :
ratio by weight ii 20 and the catorific ualue ofthc ful used is 4180 kJ I kg, what braki mean effetive
pressure na! be expected at S.T.P, conilitinns ? L= d"
Solution. Brake thermal effrciency, lttrst = S0Z, Tt Tz
Air-fuel ratio by weight = ZO 750.5 x 0.216 ?60xV,
Calorific value offuel used,
_
C = 41800 kJ/kg 2N 213
Brake mean effective pressure, p_u: 760.6x0.276x273
work pmduced Vz= = 0.201m9
760 x 290
Brake thermal efciencv -
Heat suPPlied
Gas used per stroke i49L 6.61575.t
i 0.S - Work Produced = =
418(x) Volume oceupied by air at N.T.P. (V)
.'. Work produced per kg of fuel = 0.3 x 41800 = 12540 kJ
pV=mRT
:

Mass ofai used per , offuel = 20 kg


S.T.P. co.nditions refer to 1.0132 bar and 15.C v-. mRT =?t!"z:,zi:ts
1.o132xrF- =2'196mYmin
nRT 20 x287 x(273 + 15) P
vo.Ume- Ol arf USed
r,^r.-*
P = 1.0132x 10' = 16.31m3 Air used per stroke =
jl{ 0.0183 m3 at N.T.P.
240t2 =

- I
I
I
I
i
TNTERNAL COMBUSTTON
ENCTNES TESTING AND PERFORMANCE OF I.C. ENGTNES
Mixtue ofgas and air used per
stroke
(iu) Indicated themal elficiencVr
= 0.0O16?6 + 0.0188 = 0.O199 r3 e, :

Voluwtric &tud volume


effzciency,
-. = dIlylpg
tlsit3re *roke at p. rrun = = #* = 0.2i! or23%' (AnsJ
N. T.
(u)
fh
Relative ecienc 4p.""
0.0199 :
;a;@;a4.,5 o'868 or 86'8%' (Ans')
Erauple r725.Th
- '- fotbw,-- -- :
-''-)ttg llarticulds
rl,"r* = j!&t*.t
werc obtained in a trial rlait
*bl '
Duration of on a 4-stroke gas engine rtdad
= r hvr -I
;
Reuotuo l.+a.oo = -
1 = 1
1
- G;i":i = 0.527 or 52.7?o
Number-of missedcycte
:f Gt-i
Net brdke load
rl,o.t'.=
Meaneffectiuepressure = 147oN #, =o'6o143'6%. (Ans,)
c^ = 7'5 ba Example l726|.The compression
,:C:V"or,uu^ptli,r---'- curue on the ind.icator diryram for a gas engin foows the

-"fu*i'supply corditUn :;iT:r';T:


Cylindcr
law pVI s = qsnstant. At two points on the cunte at ! strohe and.
sttoke ttn nressures are I.4 bar
d.ianzter I
strohe = 250 mm and 3'6 ba'-r respecvelx. Determine the compressoln ratio of the engine, Calnulate thr therna,l effi.-
cicnqr and. the gas nraumption per I.p. hoyr, if the rerntii,ve
Effectiue b.rake circumferene :'rY^^ calarific ualue of 788O0 kJ I a3.
ffttiznel is 0.4 and, the gas has thc
wompression ratio
Solution. Refer Fig. 17.13.
c"l*lttr,' ifinu"trdpower = 6'5 : 1
(ii) Brake power
( Mechanialffiiencl, p(bar)
(iu) Inor"O ,rermat
(v) Relatiue efficency. effzcizntA

Soluion. N= 11009 _ ?00


60B r.p.m., W_S 1420 N,
=
p^=7.lbar; V"=
D =250n
ffi =5.56tres/s,
= 40o mm = 0'4 m
(i) rndicaed
"
*yl::":;T:':";;'=?
np^,IANkx1.}
lP. -
6
Nk= (Y - *llsooo =
6fl
working cycres /rnin.
L x 7S x 0.4 x | 4 x 0.262 x(6500/
I,P. = c
- 6
60) x t0

(li) Brake power,


26.69 kW. (tns.)
B.p, :

@
r.r.' = 19)o?rv Wzll(zoo t a) 22.86 kIV.
1000 -
60 x =
-_;106:- = (Ans.)
() Mechnicl efllclency, Fig. tZ.18
r."* :
Compression law, pltr'3 = constant
I
,l B.P. 22.8rt
-'''*h _- iE Pressure at'o',
= Po = l'4 bar
I

t6;6" =o.869o185.996. (A.) Pressure at'',


ii P = 3'6 ba
l Volume at'a', Vo=V"+ 0.76V,
Volurne at'', V=V"+ 0.25V"
INTERNAL COMBUSTTON ENCINES

ESNNG AND PERFORMANCE OF I.C. ENCINES 5'75


PoVolg =PoVrLi
3 Mean effective pressure develored, p- :
lo=(a)u" lg{)^
vb ln") =
\t1l =2.067 Air-standard efficiency in case of Otto cycle is given by
It
lai.stedad =I - Gt:I =1 - G., = O'457 or 47 1Vo
AIso 5
V, -=W
V" +025Y" =r.*,
-'''
Tlerul
or (V" + O.75V") = 2.O67 (Y"+ O.25Y") Also, .'rrono. _
rlai-stndad
or V"+ 0.75V,= 2.06?Y. + 0.516Ir. l16*d = 4.do x rlei-st.nrtrrd = 0.65 x 0.475 = 0.308

o.2s4V"=1.06?V. or I.P. or0.308= IP.


fr =o.su. But, tthemar.) = r0.-il1800
^N 3600
Cornpression ratio =+# =f - r= 4.56 + 1 = 6.56. (AnsJ -'. 0.808 x 4t8) x 10.5
IP = ------S600- =37.55kw=37.5x ldN-m/s
Air standard effciency, ",-"m = 1- .'. Net work from one cycle per cylinder
# = t - (rr#r= = O.49G or 49.GVo
3?.5 x 103 x 60
_ = S00 N_m
flth.ur 6x(25OO I D
n.. = rbir-rtadsd V" +V"
Arso, ,= =u
0.4 =
4tho- -:
O-,96 V"+trs=5%
"' |
l,"- = 0.4 x 0.496 = O.198 or 19.8%. (Ans.) V,= 4V"= x 0'000115 = 0.00046 m3
:, Mean effective pressure deuel.oped,
But n,r-r = #* (Vs. = Volume of gas used in m3/s)
'*^" W,r", per cycle
p^=:V:- 900
bar=6'52bar. (Ane.)
I =
,troo-
-''-" - ,a'aF-
V, x188fi) ls&cple t7.2A. A 2.cylinder C.I. erigine with a compressbn ratin 13 : 7 and. q,linder
of 200 mn x 250 mm works on tuo strohe cycle anil consunts 14 kg I h of fuel while
'' vu=
6ffi+I6dd me/s = o;rrfirroo x 3600
8 cf 300 r.p.m. The relatiue and. mechanial fficiencies of engine ore 65Vo ond 76Vo respec-
fhe ful injection is effected upto 5lo of strohe. If the calorifiq ualue of the fue! used is giuen
= 0.98:l ms/I.P. hour. (Ans.)
11800 W I hg, calculate the mean effectiue pressure deueloped,.
Example L7.27.A$-cXlindcrpetrolengfunhasaoolumecompressinnrati.oof 5: T.Theclear-
ance uolume of each qrlind.er is 0.)0115 mt. Tlv engirc consutnes t0.5 kg of fuel per hour whose Solution. Rfer Fig. 17.14.
calorftc value is 41800 kJ I kg. The engine runs at 2500 r.p.m. and. the effi.ciency ratio s 0.65. Diameter of clinder, D = 200 mm = 0.2 m
Calculate the auerage indicated mean effetive pressure d,eueloped,.
Shoke length, Z = 250 mm = 0.25 m
Nunber of cylinclers, n=2
Solution. The ideal cycle referred to the petrol engine working is Otto cycle.
Conpressionratio, r= 13
Nunber ofcylinder, =6 Fuelconsumption,
Compression ratio, =5 =L4kglh
Engine speed, .iV = 300 r.p.n.
Clearance volu'ine of each cylinder - 0.000115 ng Relative efficiency, I,"6;o" = 657o
Fuel consumed = 10.5 kc/h litechanicalefficiency, \^.,=76Vo
Calorific value of fuel, C = tf1800 hJ/kg Cutoff = 5% ofstroke
Engine speed, N; 2500 r.p.n. Calorific value offuel, C = 41800 kJ&g
Efficiency ratio = 0.65. k = 1 .-.... for two-sboke cycle engine

Cut-offratio, v"
P- tr
f2
TESTING AND PERFORMANCE OF I.C. ENCINES
INTERNAL COMBUSTION

o= --izL
ffi x a1800
o'4 x14x 41800
I.P. - 3600 =65kw
B.P.
Now, 4'"". = fE
0.76=+ OD
B.P. = .76 x 65 = 49.4 kW
Mean effective pressure can be calculated based on I.p. or B.p. ofthe engine,
np IANkxlO
_
IP,=:,
where p; = indicated mean effectivepressure
2x p^, x 0.25 xnl 4x 0.22 x 300 x lx 10
b
65x6x4
P.t =
i*T-zl* n"o2riIo"r0 = 8.27 bar. (Ans.)
Fis' u,14
Also and brake rnean effective pressure,
Vs-Vr= 0.06y, = 0.05(yr _ y2)
p*=0.76x8.27 = 6.28bar. (Ans.)
or Example 17.29. Following data relate to 4-cylinder four-strohe petrol engne. Ar-fuel ratio by
Vs-V2= 0.05 (19V2
or
Vs-Vz= 0.05 x l2Vr=
- y2)

O.6Vz.
[ +=.=',J weight = 16 : 1, caiofifit aatue of the fuet = 45200 kJ t kg, nechanical iffi"A""y = 82Vo, air.stand.ard.
efficienq = 527o, relatiue fficiency = 70Vo, uolutnetric efficiency = 78Vo, stroketbore ratio = 1.25,
suctian conditions = 1 bar,25"C, r.p.m. = 2400, power at brakes ?2 hW.
=
' ='.,
vr
t
Cd,Iculate : (i) Compression rato
(iii) Brake specific fuel consumptian
(i) Indicated thermol efficiency
(iu) Bore and, stroke.
Solution. Air fuel ratio by weight = 16 : 1
-.+ l-d=l
I
rl.i,-.t
d.,r =t - No. ofcylinders, n=4
r(r)Y-l I P-t -J
Calorific value of fuel, C = 45200kJtks,
Mechanical effrciency, \^n.
= 1- ----l---:-ir'c" -rl
= 82Vo
r.o_r Air-standardefliciency, rl6;."nu=52Vo
1,4(14).4_1
[ _J
Relative efliciency, lln""" = 70Vo
1- Volumetric eIiciency, \"oy. = 78Vo
= 0.248 x 1.55 0.6l5Vo or
= 6L.5Vo
Stroke / bore ratio,
Also, =7.25
r'lclative = --Ieg.-el- Engine speed, N = 2400 r.p.m.
,tai_!tldd
Suction conditions p = I bar,T=25+273=298K
9.65 _ ltenal Stroke / bore ratio
0.615
= 1.25
Brake power, B.P. = 72 'r.
Ith"m"l = 0.65 x 0.615 = 0.4
(i) Cornpression ratio, r:
But . I.P. For petrol engine, air standard efficiency is given by
'ramal (I) = :
C
^f " I
n .=l-
'M-s@dq - (r)y_I
I

578
INTERNAL COMBUSTION ENGINES TESTING AND PERFORMANCE OF I.C. ENCINES -T
0.62 = 1- or -i- L, 4, #*
-i-
(r)r.*. ()r +_r = 0.48 = Pz x !.25D" = o.ogeg
or (fr --:- =2.08 or
1
r=(2.08)v0.4 =e.b8')z.i =6.2 ,'= 99f$3;z^r,$ = 0.001264
i,e., Cornpresson rato
= 6.2. (Ans.)
.'. D=O.108mor1gnn. (Ans.)
(ii) Indtcaed thermal efficiencyr
lu.o: and = 108 x L.25 = 135 nm. 15".
lehti"e = or 0.7 = -!$@4 Bacmple 17.3Q. A single-qlindzr four-strohz gas engine has a bore of 180 mm and. stroke of

.i ffi 340 mrn and is governed, on hit-and.-mi.ss principle. When running at 400 r.p,m. at fuII load, ind.a-
lth"_ rr = 0.7 x 0.52 = 0.8&t or 36.4% tors cards are tahen which giue a worhtq l.oop mean effective pressure of 6,4 bor, and, a pumping laop
1.2,, ndiated thermal efficiencXr 36.4%, (Ans) mcon effectiue pressure of 0.36 bor, Dingrams from thc dcad, c1lcle gve a mean effecve pressure of
=
(iii) Brake specific rel coneumption (b.s.f.c.) 0.M bar. The engne wos run li.ght at th same speed, (i.e. with no load), and, a mechanical counter
:
recordd,46 fi.ring strokes per minute.
Indicated power, B'P' Calculate: (i) FuIl load brahz power
LP. = l-*L =
ffi =ez.rr.w (ii) Mechanical efficietrcy of the engne.
Also, LP. Solution. Number of cylinders, n=7
nu,o= llC Bore, D=180mm=0.18m
Stroke,
0.86- , 87'8 'where' rn, = Fuel used i kg/s
Z.= 340 mm = 0.34 m
m. x 45200 Engine speed, lV = 400 r,p.m.
Working loop mean effective pressure = 6.4 ba
:. "'f =_364 45'z00 = 0'00533
it, Bz.E
ke/s Pumping loop mean effective pressure = 0.36 bar
Brake specific fuel consumption, " Mean effective pressure (dead cycle) = 0.64 bar
Firing strokes/min. =46
L ^r-
u.b.r.c.=
Fuel used /sec. 0.00599 Refer Fig. 17.15.
---EP- = n- kg/kws
0'00533 p
- 72
x 8600 kg/kwh = 0.2665 kg/kWh. (Ans.)
(iu) Bore and troke :
Mass of air-fuel mixture 1 + 16 1? kS/kg
= = of fuel
.'. .For 0.00533 kgls of fuel supplied to engine the mass of air-fuel mixtue
= tZ x 0.00588 = 0.0906 kC/s
.'. Volume of air-fuel mixture supplied to the eng;ine per
sc.
mRT 0.0906 x 287 x(25 +278\
P ----l;F- =o'07748m3/s
Mass of mixtue supped /sec.
TlwL =
Swept vol"me
o.07748
0.78 =
i S."pt*ltrn"
.'. Swept volume u.t|t't4r1
= 0.0993 ms/s
0.?,
But swept volume/sec. xt,)x
=
l:'Oz no. of cylinder. , H# Fig. r?.15
.BSTINC AND PERrcRMANCE OF T.C. BNGTNES
IMERNAL COMBUSTION ET{GINES
(i) FuIl load brake power.
B.p. :
Net indicaed mean effective pessure,
p;1," = Working (or power)
loop ean effective pressure
_ rumping loop
meau effective pressure
Also, working cyclevmin ] lun - o'* = 6'04 b;
Dead cyclea/min - ( +0O ,^)
Therefore, since rhere ,. 0""* lj""-"?*,tun
Frictional power, F.p. ""(Net LpJ _ (p,rrrroi[
= oor,r,"" of dead cycles)
Now, nP^(net) xZ-lf x10
Net LP. =
6
1x6.04 x0.84
_ xrrl4 x 0.182 x 46x 10 l
6 t..' Nk = 467 Fig. 17.16
= 4kW
Pumping power of dead cycles Example 17 .31. During the trinl of a gas engine following obseruations were record.ed. :
Bore =320mm
npnild) LANV xll Stroh = 420 mnt
o ;
I
Speed = 2M r.p.m.
lx6.04x0.S4xn/ 4x0.1gz x 154 Number of uplosbnslmin. = 90
&
x 10
6 t'.. N = 15a Gas used = 11,68 m3/h
= L.42 Pressure ofgas = 170 mm of water aboue atrnaspheric pressure
Substituting he above values in eqn. (i),
we get Borometer = 755 mm (mercury)
F.P.=4 -7.42=2.58kW Mean effective pressure = 6.2 bar
At full load the engine fires regularly Cd.orifrc ualw of gas used. = 21600 hJlhg at N.T.P.
every two revolutions, and there
.r"
----'- *2 = 200 firing
Net Imd, onbrake = 2040 N
strokesperminute. Brake drum d.i.amter = 1.2 n
:' nP^(*t'IANhxlO Anbiznltemperature = 25"C
I.P.= Colcula.te : (i) Mechanical efficiency, and, (ii) Brahe thermal efficiency.
6
Sofution. n = L,D =0.32m,L=0.42m,N=200r.p.rn.,
_ 1 x 6.04 x 0.34 x l4 x 0.182 x 200x 10 fv = oo * 1l i

6 |L =200
2l Ni = eo, n" = 1# = o.oos24 ms/s,
= r?.42kw l
Hence brake power, B.p.= (I.p. _
F.p.)
Pressure of gas = 7S5 + S = ?67.5 mm Hg
lJ.b
= 17.42_2.88 = 14.94 kW. (Ans.) p = 6.2 bar, C = 21600 kJlkg at N.T.P.
.
(ii) Mechanical efficiencg (W-$=1840N,Dr=1.
r-o, :

11- B'P' 14'84 () Mechanical efficiency :


*t' = Jpl = =0,852or85.2Vo. (Ans.l
nn As the number r.lexplosions per minute is given as 90 per minute and r,p.m. of engine is 20O
Note' The F'p' is ven nearly-cons{mt,at it shows that the engine is operating on four-stroke cycle.
a given engine sged-; and
ifthe
rH:ij:""j}il,,,\".;ilffi ;"#:l;* jlJ*:#ff
road is decreased giving lower
Indicated power (LP.) is given by the relation :

;"#*5*Fl"Tlli?1"""_u"p"ua,n" np.l,ANkxl0
I'P.=-3L6-
.._J
582
INTERNAL COMBUSTION
TESTINC AND PERFORMANCE OF I.C. ENGINES
583
Ix 6.2 x 0.42 x n/4 x 0.822 x gO x l0 Calorific value of fuel, C = 43900kJ&g
1...?v= Brake power, B.P. = 2A tW
= 31.4 kW Speed
= 2500 r.p.m.
(W-S)rDolV
"'^'-_ --60"1000- _2O4OxxL.2x2O0
".". =--=1000 =25'6k\4t
I
n= i .... for 4-stroke cycle, engine.
- 2-
.'. Mechanical effici.ency,r B-P. 25.6 (i) Volumetric efciency on the basis of air alone :
.""h.= I.>. = gf-J Characteristic gas equation is written as
= 0.815 or 81.52. (Ans.) pV= mRT
(id) Brake thermal efficiency:
Volume of gas at N.T.p. : p,n 1.0132x105 _roL-l*3
pl= 767 .5 mm Hg, yr = 11.6S mt/ h, T, 25 + 273
V = RT - 287 x(20+
= = 298 K
-Now, to find Vr, using the pz = 760 mm Hg, Tz= 0 + 27A 2TgK
= Also 150 mm of H.O = n x 1000 = 150 kglmr
relation :
" t'o;
Thus head ofair column causing flow,
Tr =
P"vz
T2 r= +
L.L
= t25m
767.5x7L68x275 Thus ai flow through the orifice
v-- PtvtTz =
" P{t 760 x 298 = Air consumption = |, , Jzgn
= 10'8 m3/h
Brahe thermar efficienqr, = 0.62 x
I ,0.OSZlt, .E x 9.81 x 125 = e.0247 ms/s
B.p. Therefore, air consumption per stroke
Tlu,.
G)= tla [Vg = Volume of gas in

25.6 = 0.O01185 m3
= =0.3e5or89.dVo. (Ans.) \2)
#."*_ Air consumption of sttbke
.'. Volumeric efEciency, r-. =
Piston displacement

= =o'666tffi 66'5%' (Ans') or


(ii) Air-fuel raio:
Mass of air drawn into the cylinder per min.
() Th, ,olu^"tric efficiency on the
basis of oir alone. = 0.0247 x 60 x 1.2 = 1.??B kglrnin
(ii) The air.fuel ratio. -
(ii) The brake mean effectiue pressure. L.778
.'. Air-fuel ratio = 0Jr5 = 13.67: f. (Ans,)
(iu) relatiue effrciency on the brahe therma.l
-The zffrciency boss. (iii) Brake mean effective preesure, p_o :
Solution. Diameter of circular orifice, d, = 3.2 cm = 0.032 m
Coeffi cient of discharge, P^JANKXIO
C= 0'62 B.P,=NX
Pressure qcross orifi ce, 6
, = 150 mm ofwater
Temperatre ofair in the room
=20"c p_* x 0.00128
I x '4o x 2500 x I x t0
----'--'--'-
Piston displacement 28- 2"'- t'.' 1,A = 0.00178 m3l
= 0.00178 m3
Compression ratio,
r=6.5 ZBx6x2
Fuel consumption :' p'= (Ane')
= 0.135 kg/min 0Jo1?*25od1o =7'65bar'
.. INTERNAL CbMBUSTION ENCINES
(iu) Relative effieiency
: dFsrrNc AND PERFORMANCE OF r.C. ENCTNES

tlrr*t 1 1
o.d =.r- -,I
= -
t- Gs)u:T =o'527 or 523% I x p^s x02xn 14 x 0.162 x 400x I2 x l0
Brake thermat
"*";;.
8'04 = 6
28
. rlt.rt
".".
= ;i>c-= =0.2835 or23.35%
:' __q!fa-6,14T_?
P"b= -oxx0.16.x4fi)xlo =6ber. (Ans.)
T"ar* (iii) b.sf.c., q,o*), lt.ol :
rlgenal(B) o!2s5-o. Brake specific fuel consumption,
Tldative=
= .s.lc = l'l consunption per B.p. hour
,,,,":T,*,,".o*;m,;"H::::;:;':,#:;""s,i,su,hiterun. =
2.8
8"04 =0.348kg/B.p.hour. (As.)
Area of ind.itabr card.
= 30O mm2 Brahe thzrmalfficienq,
Lengthof diagram
=40 mm 8'04
Sprngconstant I rl*.rur=
Speed of the engine
= barlmm
mf xu = _,.9_
-' ** o* y
41969
=o.z47sot24.7vvo. (Ans))
= 4W r,p.n,
Loadonthebrah I nd.icated thermnt efficizttcy,
=370N
Spring balanae rcad.ing
= S0 N B,B' 10.05
Diarneter of brahz d.rum
= 1.2 m
rltr.r
0= ,t = 80.g1%. (Ans.)
or80.91%.
= o3l)91 or (Ans
Fuel cowumptian
= 2.g kglh
N ffi;"rr_
=o.8r)91

lalorifuuatuzoffuet =4tfifu|hJlhg Exople 17.M. A 4-cylind,er, four-strohc qcle enginz, 82.5 mm bore x I3A mm stroke d.euel-
Diameter of the qtinder ops 28 hfri uhile runnng at 1500 r.p.m. and using a 20 per cent rich miure. If the uolume of the air
= 160 mm
Stroke of tlu piston in the calinder when masured, at 15.5'C and. 762 mm of mercury is 70 per cent of the swept volume,
= 2(M mm
cahulate : (i) Indiaay!,1l1l the theoretiol air'fuel ratio is 14.8, heating value of petrcl itt ISSAO n"ltnA and, the mechancal
pressure. (iii) Brahe specifrc ?ffe}ive yreysure. (ii) Brah porer ond brake mean effectiue
ftul consumption,i*p"
effuiercy of the engine is g0%, find : "sd
ti"o^ ord. itd.icated tternar effi.cic^cies. (i) Tle indicated. thermal efficiency.
Solution. lV=400,r.p.m., W=870N,
.S=50.iV, Dt= 1.2m, (ii) The brdke mean effectiue pressure.
n= 2.Bkg I h, C = 41800 kJ/kg,D
= 0.16 m, L = O.Z Take R = 287 N-mlkg K.
,1 ^,
n= ...... for 4_stroke cycle engine. Solution. Number of cylinders, n=4
i Engine bore,
(i) Indicated meen effective D = 0.0825 m
paaue, pD: Stroke length, L=0.13m
Aea of indicato-r diagrar_ Brake power,
o .= 9r card x spring costant B.P. = 28 kW
Length of dia$am Engine speed, N = 1500 r.p.m.
300x1 Theoretical air-fuel ratio = 14.8
40 = 7.5 br. (Ans.)
Calorific value of fuel, C = 45980 I'el/kg
Mechanical efciency, 116"' = 907o.
Indicatedpower, Lp. 1x7'5x0.2xl4x0.162x400x jx 10 (l) Indicated thermel efficiency, rlo
=npnIA{kxLo - o, :
Swept volurne, V" = 7r/l fi21, = tr!4 x 0.O8252 x O.1B = 0.000695 m3
= 10.05 kw.
(ll) B.P. , p.o:
votume of air drawn in = .70 x 0.000695 = 0.0004965 m3
Brake power, _ (w -s) DoN _ (g!0 - 5o)z x r.z x
-
".". 60 x -
100O 60 , 1000-
4oo
= 8.04 ktff. (Ans.)
,p
762
= 260 x 1.0132 = 1.015 bar
Also, - n2n x INk xl} y = 0.0004865 m3 (calculated above)
".r. 6
R =287 N-m/kgK
INTERNAL COMBUSTION
TESTING AND PERFORMANCE OF .C. ENCINES 587
T = 15.5 + 278 = 288.5 K
Brake wheel diameter
i. n = Mass of air/stroke/cylinder
=1.6m
Broke rope diameter =2cm
pV x 10b x 0.0004865 Gas used = .c m"
-_ nf _=
1.O15
28dr885- = 0'0005eG kg Colorific ualue ofgas = 15900 hJ / m3
Theoretical mass of air used per minute Calculate : (i) Ind.icated. power, (ii) Brake power, and
(iii\ Indicated and, brahe thermal efficiencies.
= 0.000596, 1!9q , 4 = 1.Z88 kg ai30
Solution.D =o.2m,L= 0.4 m,N,= 9400 r.p.m.,N = = ros
Theoretical air-fuel ratio = l4.g
.'. Theoretical mass offuel ssd/in (w
RK
- S) = 540 N, D b= 1.6 m, d = o.02 m, v, = rUftU = 0.00s54 m?s.
1.788 (i) Indicatcd power, I.P.:
= t4f = 0.1208 kg/min
Indicated mean efrective pressure.
When using 20% rich mixture, theu
Area of indicator diagram x spring number 550 x 0.8
/ = Mass of fuel burnt / sec ' 4' Length of the diagram 72

0.1208 120 = 6.11 bar


= _ 60 _*i00 = 0.002416 ke/s nn .IANkx70 1 x 6.11 x 0.4 x tr I 4 x O.22 x 105 x 1O
IP-'-' =
Now, _ B.P. 28 o o
'1.*l= r, = LE = 13.4 k1{.
28 (ii) Brake power, B.P.:
0-9 = LP.
(W-S)r(Du +d)N 540 x r(1.6 + o.or,.
.'. B.P.= ---Td;rooo _ [gffJ
I.P.= 3
0.9 =31.11 kW. 60 x 1000
Indicaed thernl efficiency, = 10.76 klY. (Ans.)
(iii) Indicated themal efficiency :
rrtuo=
+h=
(ii) Brake mean effecve prcstrrq
md#m =o.:r,orz*%. (As.)
n*'o=
t'#= m65#1E6* =o'238ot23'8zo' (Ans')
p'b: Brake themel efficienc
nx p*IAltlhxl0
B'P-=- B.P. 10.76
6 rtn.G = = 0O0Bb4 15900 = 0.191 or 19.17o. (Ans.)
4; C
1x pa x O.tAx r | 4 x e.08252 x 1500 x 1 x 10 "
Erample tZ.S6.fhe oowing obseruations were record,ed during the test on a 6-cylinder, 4
28= 2 stroke Di.esel engine:
?lx6x4x2 Bgre = 125 mm
"' P-b=
4 x 0.13x r x 0.0gZO, x1500 x ro = 8.06 bar. (As.) Strch = 125 nm
Eranple Engine speed = 2400 r.p.m.
17.35. During the tett of 40 mnutes on a singte-cyrindzr gas
engine of 200 I'u.d, on dynamometer
cylinder bore and 400 mm st*, uorlW on tne-fiir-strohe =490N
method. of gouernyr.g, the following *dirr;" ,*" $cteL"a s"r;"T; ht and, Dlnamamzter constant
i;" = 16100
Total number of revolutians gnT Air orifice dinmeter =55mm
=
Total number ofexplosions C oefficient of d.ischarge
= 0.66
=120O
Area of ind,cator diagram Head causing flow through orifice = 310 nm of water
= S5Aninz
Length of ind.icator d.ia.gram ?2 mm Bororneter reading = 760 mm Hg
=
Spring number Ambient temperature = 25"C
=0,8barlmm
Brake load Fuel consumption
-&40N = 22.1 hglh

I
I
rlti INTERNAL COMBUSTION ENGINES

lil; Calorific ualue of fuel


= 45100 hilhc
TESTTNC AND PERFORMANCE OF I.C. ENGINES

Per cent carbon in the fuel


=85% Po = density fair at 1.013 bar and 25"C.
llllr Per cent h3tdrogen in the fuel
Pressure of air at the end, of suctian strohe
= I57o P 1.013 x rd
ill = 1.013 bar = RT = 2B7x@+27 =1'18k9/mu
ilt Temperature at the end, of suction stmhe
= 25"C
ilt Calculate : (i) Brake meon effecue ptussure,
(i) Specific fuel con sumpto n, I or
(ii) Bqp. ..*aI efficiency, % = 8O x 0.0023? x 0.66 11J8 = 6.73 m3/min
Volunre.of air,
ilt l (u) Percentage of excess air
supplied-
(iu) Volumetrc efficienqr, and
Actual volume ofair per cylinder I
Solution.
_!=
6lD = 0.125 m, L = 0.L%m, |I= 2400 r.p.m. 6'73 #F 7e
4r =490IV, C, = dyaamometer constant 16100 nb = 1.12 m3/min
ill, =
do = orifice diameter 0.055 rU C O.66,, Air supplied per stroke per cylinder
= = = 310 mm
t;i . ,t1 t.t2
- = ffi =o,oo614ke/s,c=451ook/ks, (2400 | 2)
= 0.00O933 m3
fil Volume of ai actually supplied
1 ,. ,twl. _
h= ..... for 4-stroke cycle engins. Volume of air theoretically required
;
(i) Brake mean effective pressurg p* :
0.000933
(Ans.)
ilf' g 49ox 2400
oJol6s =o'609o16o'97o'
Brake power, s.p. -
tfit,
= -%"l[ - - ".
=73kw (u) Percentage of excess air upplied :

fil: Quantity of air required per kg of fuel for complete combustion


Also %4t.ro
- loofc*9*a^'9'l
n.p. =
iliri 6
23L 3 . 1l
6x p^b x 0,L25x t 4 x 0. 252 x 2400 x x
ilii zg= --'2"-:
1 10 where C is the fraction ofcabon andll, is the fraction ofhydrogen present in the fuel respectively.
l00l^^- 8 ^-, ^l
:. p*r== ,r==73"6t4T2 = za o.sr"-+0.15x8J =
15.0zkg/kgoffuel
iiii = 3'96 ben (AnsJ
Actual quantity ofair supplied per kg of fuel
(li) specific *t
ili
""rW
22J _ %xpox60 _ 6.73x1.18x60 =21.56ks
b...f... = 22.1 22.r
ls = o.s02z tCl kWh. (Ans.)
Gil) Brake thermal efficienc r_r, : .'. Percentage excess air - "'ug#91 x 100 = 43.0G7o. (Ans.)
i

J9
't*.rur= A, xC = 000611 ,=,
=,r, ,,45100 lt = o'2616 or 26.3670' (Ans.) IIEAT BAI.ANCE SHEET
(iu) Volumetrlc efflciency, r-.:
I
t,
Example 17.37. following obseruatians were record,ed in a test of one hour d.uration on a
The
Stroke volume of cylinder */.4 x L single-eylind.er oil engine working orxfour-stroke cycle :
l =
= ttl4 x 0.1252 x 0.125 - 0.00188 ma Bore = 300 mm
. The volume ofair passing through tbe
orifice ofthe air box per minute is given by, Stroke = 450 mm
Fuel used. = 8.8 ks
% = saoAo ,l!
"o I r
Calorific ualue of fuel
Auerage speed.
= 41800 kJ/ hE
= 200 r.p.m.
where, Cr.= Discharge coefiicient of orifice 0.66 n.e.p.
= = 5.8 bar
Ao = Aea of coss_section of orifice Brake friction load = 1860 N
= t4 doz = nl4 x (0.085)2 0.0028? m2 ofcoolingwater
= Quantity = 650 hC
h,=Head causing flow through orice in 810 Temperoture rise = 22"C
cm ofn'ater = = 31 cm Diometer of the brahe wheel
li- = 1.22 m
TESTING AND PERFORMANCE OF I.C. BNGINES 591
INTERNAI, COMBUSTION ENGINES

Calculate : (i) Mechanical efficienq, and. (ii) Brahe thermal Calorific vahu offuel = 43890 hilhs
fficicncy.
Draw the heat balance sheet. Air consumptbn = 3.8 hglmin
Solution. n = l, D = 0.3 m, Z = 0.45 m, m = 8.8 ke / h,C
= 41800 kJI<g, Speed = 79N)r.p.m.
N= 200 r.p.m.,pa = 5.8bar, (-$ = ZgE N,D = L.22 m, Torque on the brahe d,rum = 786N-m
1
h= for 4-stroke cycle engine Quantity of caling water used = 15.5 hlmin
;,...
Temperature rise =96pC
m.= 650k9, trz- t ' Exlwustgastemperaturc -41trC
r=22C.
(i) Mcchanical efficienlr-o. Ramtemperature
; =2tC
I x 5.8 x 0.45 x /4 x x 200 x 1 x
cpfor ex.haust gses = 1.17 hJlhA K
Indicated power, nP-IlVl x 10 0.32 10
I.p. = = Calcul.ote : (i) Bruhe pouer,
6
hw
(ii) Brake specific ftnl consumptio4 atd
= 30.7
(i) Brshe thermal effi.ciency.
Brake power, B.P = ffi60:ift?N
=
18qqr1#ro9 Draw heat balance sheet on minute bosis,
=23.76kw
Solutlon. n = 1, r = 15, n = t0,2 keft, C = 113890 kJ/kg, m" = 3.8 k8/min.,

(j)
n-""=
Brake thermal efficiency, r*,r,
i* =i# =o.77sor77.8to. (Ans.) .lV = 1900 r.p.n., ? = 186 N-m, m, = 15.6 kgrfmin, r - r, = B6oC,
:
, = 410'C, t,= 20C, c" = t.l'|.
= -m?!+
(i) Brako power, B.P. l
n'h.(Bl = ** --:::_ x 41900
= 0.232 or 28.2?o. (Aits.)

Heat supplied = 8.8 x 418{D = 86?840 kJ/h.


B.p.=
6#ffi =s?EW. (Ans.)
(l) Heat equivalent ofLP. (iD Brake speclc fuel consumptlon, b.afc. !
= Lp. x 3600lcl/h lO2r
b.s.f.c. = 0.2760kSfhlvh. (An&)
= 30.2 x 8600 = t10b20 kJ/h. ;:\=
(li) Heat carried away by cooling waer (iii) Brake themal efiic'lency,
ilnrc,-r(t."- trr) B.P.
' rlwr=;-;;
g7

= 660 x 4.18 x 22 = 59774kilh.


mf^v= ft-- =0J0ll6or20'76%' (Ans')
ffix890
Ifeat balance sheet (hourly badc) Heat supplid bx the fuel per minute
LO2
Item hJ Per ent = 60 * 4i890 = 7461 lcthin
Heat supplizd b1t finl (i) Hea equivalent of B.F.
367A0 r00
(i) Heat abeorbed io I.p. = B.F, x 60 = 37 x 60 = 2220 kJfuin.
u0520 30.06 -Iii) Heat caried away by cooling water
(ii) Heat taken away by cooling water
59774 16.25
(lii) Hs .... away b{ exhaust gase, = m,x cr,(!or- r, ) * 16.6 x 4,18 x 36 = 2332 kJimin.
197546 53.70
raation etc. (by diffrence) (ii) Heat carried away by exhaust gases

Total = ftr, cps x (tr- tr)


367840 100
Example !7.88. In a triar (L0.2 \
of a singte+yrind,er oil engine working on duar ,= +3.8J x 1.1? x (410-20)= 18ll kl/min.
bseruations were mad.e :
cycle, the forlowing [ff
Compression ratio
=lC
Oil cotawnption 10.2 helh
=

--'l

I
I
I
INTERNAL. COMBUSTION ENCINES TESTING AND PERFORMANCB OF I.C. ENGINES 59:
Ifeat balance sheet (minute besis) : (ir) Brake powgr, B.p.:
Item
hJ Per cent =
t*eoilj#/ 1r.zskw. (Ans.)
Heat supplizd by fuel
74161 Heat supplicd in 20 minutes
"" =
100
(j) Heat absobed in B.p.
90e = 1.52 x 43900 = 66728 kJ
(ii) Heat taken away by cmliag watr "220 (i) Heat equivalent of I.P. in 20 minutes
(iii) Heat caried away by exhaust gares
2332 31.2 -
(iu) He",
1811 24.3 = I.P, x 60 x 20 = 15.9 x 60 x 20 = 19bg0 kJ
rou"-u.u* fo (by ditrerence) 1098 (ii) Heat caried away by cooling water
74.7
Total
7t6l = , \ Cp., (trr- t.r)
100
Exampre t7 '39' From the data giuen below, = 162 x 4.18 x (55 - 30) = 16929 kJ
carcurate ind.icated, power, brahe power Total mass of air
a heat balnnee sheet and, d.raw = 32 x 7.52 = 48.64 kC
for a two-stroke dlser
r.p.m. ""ai.'r""-, z0 minutes ot furt road : Total mass ofexhaust gases = Mass of fuel + mass ol air
m'e.p,
= 350 = I.52 + 48.64 = 59.16 t
= 3.7 bar Mass of steam formed = L.4 x 1.52 = 2.13 kC
Net brake load, .'. Mass of dry exhaust gses .= 50.16 - 2.19 = 4g.03 ke
= aqN
Fuel consumptbn (iii) Heat carried awai by dry exhaust gases
Coolng water
- 1:52 hg
= 162 hc 7 tug x cps x (t"- tr)
Water inlet temperature
= so.c (305-25) = 18448
= 48.03 x 1.0 x kJ ,

Wate r outlet te mperat ure (iu) Heat carried away by steam 2.13[hr+ htu *
= = e"np_ t")l
Air usedlkg of fuel "o"
's"c
= gzks At 1.013 bar pressure (atmospheric pressure assumed) :
Room temperoture I I
= 25"C L hr= 417 .5 kJlkg, hn = 2257 .9l<Jlkl r
Erhoust ternperature
Cylinder bore
= 305"C = 2.t3 + 2287 .9 + 2.09 (305 _ 99.6)l
I4L7 .5
= 200 mm = 6613 kJ/kg ..... neglecting sensible heat ofwater at room temperature
Cylinder strohe
= 280 mm Ileat balsnce shee (20 minute baeis)
Brake dianeter
Calorfii value of fuel
= I metre
Item hJ
= $900 hJlhg Per rel
Steam formed. per hg offuet in theez,haust = 1.4 hS Heat supplbd by fwl 66728 100
Specific heat ofsteam in exhaust _ 2.0g kJlhg K
Specificheatofdrytexhaustgases (i) Heat equivalent ofI.P, 19080 28.60
Solution. JV= 350 r.p.m., pai 3.1 bar, (w_
= l.OhJ/kCK. (ii) Heat carried away by cooling water 16929 25.40
= S) = 640 N, mr= I.E2 kg, m,u (iij) Heat caried away by drr exhaust gases
t,, =30"C, t,^=55"C, m^=32 = 162 1rg, 13448 20.10
fuel, r,= 2s"C, ts=Bgb.C, D=0.2m, Z (iu) Heat eried
away steam in exhaust gases
Du = r -, c=?ss00 kJA<e, c.^=2.09,lqt"f =0.28m, 6613 9.90
crr= r.oiir.gx, k"=1.....r*t*o-.t"Ju (u) Ileat maccounted for (by difrerence)
ili (i) Indicated power, f.p.:
cycleengine. 10658 16.00

il Total 66728 100.00


np^ LANkxlO
l.r.=-
6
- Example 17.4o. A six'cylinder, four-stroke CI engine is tested. against a water brake
ii dynamometer for which B.P. wN | 1z x
_ Ix1.tx0.28xn/ ld
in hw uhere w is the brake load. in newton and, N is the
ir -__=_.-lx
4xO,22 xB50x t0
kIV. (Ans.)
=
the r.p_.tn. The air consumpton was measured, by means of o sharp ed,ged orifice.
6 = r5.9 :!, test
Y.r:! "! the
uurng :ryine ,in
following observations were tahen :
iji Bore
=10cm
Stroke
=14cm
Speed
= 2500 r.p.m.
'll
:

li
594 INTERNAL COMBUSIION ENGINES TESTTNG AND PERFORMANCE OF I.C. ENCINES 595

Brake load =480N (i;i) Engine torque, T:


Barometer reading = 76 ctn of Hg B.P, = 2r.ltrl?
Orfice diarneter = 3.3 cm t:::t#3'
Coefficient of dschnrge of office = 0.62 or ror.que, ,t, = # =
r
= 2Go.GB N'm. (Ans.)
Pressure drop a,cross orifice = 14cmof Hg ^ 16-
Room temperature = 25"C (iu) Brake specific fuel coneunption, bsfc :
FueI consumption = 0.32 hel min.
C alc ul ate t he follow ing : bd"=!# = H# =0'272kglk\r'h' (Anc)
(i) The uolumetric fficizncy ; Gi) TIu brake mcan effectiue pressure (bmep) ; (iii) The engine
torque ; (iu) I'he brahe specific fuel consumption (bsft). (AMIE Summer,20OO) t*nnle 17.4L. During the triol of a single-octing oil engirc, eXlin'der diameter 200 mm,
strohe 280 mm, workng on wa-stroke qcle ond firng eoet! 6lcl, the following observations were
Solution. (i) Volumetric elificiency, r,o,:
nnd:
% = SwePt volume, Durationof trial = T lnur
= -: D " L x ^/ x No. of Wlinders, for 4-stroke engine. (whereN = r.p.m.) Totalfuel u'sed = 1.22 hs
afr; Calorifu value = 41d70 hJlhg
2500 Proporti.on of hyd'rogen in fuel = 75%
(0.1)2 x 0.14 x x6=0.137m3/s.
60x2 Total number of revolutPns , = 2r0a0
Meon effectiue pressure = 2.71bar
Barometer = ?6 cm ns = ra,6 x r03 x
l(Fs = s.8r] x 101.4 kN/m2 diameter N
lffi " Net bmhe load. applicd to a drum of 1 m = 6N
Total mass ofcooling water circuloted. =195 hg
D =J= ==_#f _ =1.18rr4kg/m3
RoT 0.287 (273+25) Intel temperature of cooling' water = 13"C
Outlct temperature of coolng water =&ae
Air used = 135 hg
Ap= 14cmof US= x 13.6x 1000x 9.81 =18.678x 103N/mz
ffi Tempemture of air in test ram =20.c
Lp=pox9.81 xhn, Temperature of etlwust goses = 370"C
where, o = Head, trl of aii, cagsiog flow Assvme: cp(goses)=1.005t|.fuK;cegtedm)atatqtericpressure=2.093WlkgK.
, 18.6?8 x 10
"=ffi=1607.6mofair Calulate tlr.e thennal efficiznqr ond d'raw up thz hdt funce' (U.P.S.C.' 19S7)
Solution. Giue : D = 200 rrm = 0.2m iL = 280 mn = 0.28 m;mf = 4.22keh;
V" = Volume flow rate ofair, at free air conditions C = 44670kJlkg;

=c, l<aof @" r.p'm. =


# = 35o ;p., = .?4b'r' ;D = 1 m ;
(W-S) 600N;rn, = 495 kg/h i r, = 13'C, tru=38"C;
; (#)'
= 0.62 x !q x s.slx r60?^s = 0.0e4 m'/s. =
mo = !36kglb.,l,= 2O' C, t" = 376'9 ; c* = 1p05 kJ/kg K ;
V nno/ cp, = 2.099 hTkc k
t- t*= H#
qon",t= x 100= 6s.6ea (Ans.)
Ihenl efciency, t.t :
(ii) The brake nean efiective,Itresrre, tr'ob-:
' ," = Mr 1t 4'gq#lq Indicatedpower, I.P, - PnlA]lkxlo
x 10-3 = ?0.588 kw 6
' 1v =
2.74x0.28x!x0,22 x 350 x 1x 10
p^b LA- - x 6, for six-cylinder, four-stroke engine
=
# ; 6
?0.588x60x2 = 14.06 kW (i = 1, engine being 2-stroke cyclel
f mb - = 518.5? kl{/m2. (Ans.)
0.14x:(0.1)2x25oox6
f*-l._-.----'---_-

INTERNAL COMBUSTION
ENGINES J AND PERFORMANCE OF I.C. ENGINES 597.

Thernat fficienq (ittdicated.), tloa = Heat balnce sheet (minue basis) :


J*.

=_d_"
=
#""rr
=0.268 or 26.87o. (AnsJ ffi
# (i)
Item

Heat supplizd by fire|

Heat equivalent of B.P.


hJ

3141.8

659.4
Perenl
100

20.99
Brake power, B.l
-.r.= -o;6- kw c
(ii)
(iii)
Heat caried awa by cooling water
Heat caried away bi dry exhaust gases
863.4
782.6 \
27.48
24.9L
(p) Heat caried away by steam
_ 6ooll_r:s5o
300 9.55

= to.99kw W (u) Heat unaeomted'for (by differmce) 536.4 t7.o7


Heat balance sheet (minute
basie) S' Total 3141.8 100
Hea input 4.22
= E- t {4670 = gr4t.8 kJ/min. Example 17.42, During d test on a two stroke oil engine on full load. the following obserud-
tD, recorded:
tions were -^^-.t-.1.
(j) Heat equivalent of B.p. = 10.99 x 60 659.4 Speed
= kJ/min = 350 r.p.m.
(di) H"", ,ort ,o cooling water'
=,xcP.t (tr,-t r) Net broke load =590N
Mean effective pressure = 2.8 bor
495
= jO-- r 4.186 x (38 _ 13) = 868.4 kJlmin. Oil consumption = 4.3 kelh
Mass ofexhaust gases (wet) Jachet cooling woter = 500 hslh
= mase of air/min + mass of fuel /min Temperature ofjarket water at inlet and outlet = 25"C and 50"C rspectuely
135 422 Air used per hg of oil
= n -E6- 2'32 kg/min =33h9
E- =
Temperature of air in test room
Steam in exhaust gases = 25'C
= I x It x mesg of fuel usemin Temperature of erhaust goses = 400"C
Cylinder diameter = 220 nm
t*
Mass of dry exhausr gases / rnrrr]
# "# = 0'095 k*/min Stroke length = 280 mm
E ffe ctiu e brake d.ameter = l metre
mg = Mass oferhaustgas (wet)-ur. Calorific uolue of oil
of $oproduced/min = 43900 kJ I hg
(iii) Heat caried away by = 2'226 kstmin' Proportion of hyd.rogen in fuel oil = 157o
uo,
"*lft:?;:*
hs""wx(tr_tr)
Mean specific heat of dry exhaust gases = 1.0 kJlk4 K
. Specificheatofsteam = 2.09 kJlks K
= 2.225 x 1.005 x (BZ0 _ 20) ?82.6 kJlmin Calculate : (i) Indicated. power, and. () Brake power.
(du) Assuming =
that steam in exhaustgases Also draw up heat balance sheet on minute bass.
and exhaust gas temperature,
the enthalp]
-kg;;;-
exist as superheaed steam
at atmospheric pressure Solution. n = 1, N = 350 r.p.m., (W - S) = 590 N, p-, = 2.8 bar
i a! armospheric pressure
"f
1.013 ba - 1 ba ancl 3?0.C mf= 4.}kgfti, m, = 500 kglh, t*,= 25"C, t.,= O'C
mo = 33 kgitkg of oil, t, = 25' C, t = 40O. C, D = 0.22 m
k*l = sensible heat of water at
room temperarureJ.
e
L = 0.28 m, Du= 1 m, C = 48900 kJ/kg, co" = 1.0, cp" = 2.09
= [hr+ hrr+ cp, G6op - t) - h] h= | ..... for two-stroke cycle engine.
= 417.5 + 2257.9 + 2.099 (S?0 _ 99.6) _ I x (i) Indicsted power, I.P. :
4.18 x (20 _ 0)
Hear caried away by stearn jrtH: np^,LANk xIO
Ii#= 800 kJlmin l.r. = '"
o
f x Z.e x 0.28x n | 4 x 0.222 x350 x 1 x 10
6
17.38 kW. (Ans.)

t
**J!tj.._
T_

:

INTERNAL COMBUSTION ENCINES TESTING AND PERFORMANCE OF I,C. ENGINES i!


tl
f
(id) Brake power, B.P. :
trIeat balance sheet (mtnue basie) :
ff-S)rD'IV 590xrxlx350
D.r.= _--
il
= --O;lm- =10.81kW. (AnsJ Itm hJ Per ent 1l

Heat supplbd, by fiicl


Heat eupplied per minute =
# r ** = 8146 k.llmin,
3146 100 il
(i) Heat equivalent of I.P. r042.8
(i) Ileat equivalent of I.P. lZ.B{t x 60 1042.8 kt/min. 33.15
= = () Heqt ffiied away by cooling wter 8?0.8
(ii) Heat lost r*ttt *r:; 27.70
(iii) Heat eried away by drr gases
' x cpn x (t,r_ t.,) (iu) Heat canied away by steam
887 28.15
320.6 10.20
(u) Heat unaccounted for (by diference) 24.8 0.80
= r 4.18 x (50 - 25) = 8?0.8 kJ/min
;6- Total 314.6 100
Now, 2H, + O, = !!Ire
43236 Erample L7-t13. During a test on a Diesel enginc tle following observatinns were mad :
189 The pouer developed by tle enginc is usd for drivng a D,c. gerc'rotor. Tlw output of the
genero.tor trtas 210 A at 200 V ; thc efficzncy of generator being 8111o. The quantity of
i.e., 1 kg ofH, produces 9 kg ofIIrO fuet iupplied, to
the engine was 71,2 hglh ; ealorific ualue of fuel being 426(fi hJtkg. Th air-fiel-iatio was L8 : l.
.'. Mass of IIrO produced per kg of fueI burnt Tln alnust gases were passed' throrqh a ethost ggs calori;neter
for which the obseruotans
- g*E x masa offuel ueedlnin. were u folbus : Water circulated, through a,twust gos calorrr1eair
= 580 litreslh. Temperature rise of
water through calorimeter = ?fC. Temperature of exlnust gays a.t *it fivm calorimter ggC.
=
=9x o.t' #l = 0.096? kg/min.
Ambicnt temperature = 20"C.
IIat bst to jacket cooling water is 82% of the totat leat supplied.,
Total ma.ss of exhaust gases (wetlhin. If thc epaific heat of ethaust gases be 1.05 Ntkg K d.raut up the heat balance sheet on minute
= Mass of air/min. + nass of fueUmi.
Soludon. Output of generator =2104at200V
= !19-l)"n
i: =2.4t6kg/min. Generator efficiency
= 8Hb
Fuel used
Mass of dry xhaust gaseey'min. = ll.zlngt.
Caloric value offuel kI/kg
- Mas of wet saust gasee/min _ mase of II'O produced/min,
= 426{D
Air-fubl ratio = l8:1
= 2.486 _ 0.096? = 2.389 kg/min. Mass ofwater circulated through calorimeter, m = 580lites or 580 kgtlh
(ii) Heat loet to dry erbeust gases
Temperature rise ofwater, ,."- @r = 36"c
= m8x cx (tr- t) Temperature oferhaust gases at
exit from calorimeer
= 2.339 x 1.0 x (400 - 25) = 88? kJnin. = 98'C
(,) Ambieot temperature 20.c
Arsuming that steam in exhqust gases exists as steam at atnoepheric =
Heat loot tojachetcooling water
gure and exhaust ge temperature,
the enthalpy 'uperheated
ofl kg ofsteam at atr.nospheric = 32% of the tota-l heat supplied
Specic heat ofe:aust gases
1.013:1 bar and 400.C = 1.05 k^t/tsK
Total power generated
h^e-h (wbep = W = 200 x 210 = 4200 W = 42 kW
= i the sensible heat ofwater at room
t, = Ihr+ h*+c_(t oo_f/ _1 x4.1gx(26_0)
Power available at the brakes oftbe engine,
3 =51.22kw
Total heat sqpplied to the engine = Fuel supplied".",=
per min. x calorilic value of fuel
= 1417 .E + 2267 .9 + 2.09 (4OO _ 99.6, _ 104.6

= 8356 kJ/nin, r

.', Heat carried away by seam = 0.0962 x 8856 320.6 kJlmin. () Heat equivalent of B.P. 5L.22 x 60 3973
ff ' lzeoo = ?952 kJ/min,
= = = 7.to
Mass of exhaust gases formed per minue

= Fuel suppied/min. (f,*'r"-') , lf otio -"u"s air-Fuel ratio]


600
NTERNAL COMBUSTION ENGINES
TESTING AND PERFORMANCE OF I.C. ENCINES 601
Lr.2
= (18 + 1) B.5S kg/min.
T0
(ii) Heat carried away by evhsust gases/min.
= The uolume anafusis of ethaustgoses is :

Corbon d.arid ggo


=
- Heat eainef by water in exhaust gas calorimeter fron exhaust gases O4gen = I07o
+ heat in exhaust gases at exit from
exhaust gas calorime, Nitrogen = gITo
above room temperature. Temperatureofexhaustgases = 400"C
= 4a t cpo, (t."- t.r) + m, x coftr- tr) Specfitheatofexhaustgases = L\ShJlkS'C \)
Ambient temperoture
=
580
4.18 x 36 + 8.55 x 1.05 (98 _ 20) = 25"C
TO " Portial pressure of steam in exluntst gases = 0.050 bar
= 1454.6 + 290.7 = 1748.8 kglmin. Specfii heat ofsuperh.eated, steam = 2.1 t"IthS"C.
(l'l) Heat lost to jacket cooling Drau up heot balance sheet on minute basis.
warer
= 0.32 x 7952 = lg{t.6la^in. Solution. n = 1, D = 0.34 m, L = 0.44 m, N = 400 r.p.m,, ly 950 N,
=
Ileat balance sheet (ninuto basis) Co (dynamometer constant) ?460, mf - 10.6 kg/h,
: =
C = 49500 ltJlkg,m, = 25 kg/min, (t,r- t,r) = 25"C,
It2n hJ Per cent tr= 4OO"C,cw = 1.06 hI&g"C, c*=2.1kJlkg"C.
Heat supplied Mean effective pressure,
7952 100
(r, tteat equivalent ofB,p. Area of indicator diagam x Spring constant
3073 38.7 Pm
(ii) Heat carried away by exhaust gss
L745.3
f*netl' of i"di;to;i.e.;-
(iri) Heat lost to jacket cooling water 21.9
x o'e
(iu) Heat unaccounted for (by difraence)
25U.6
589.1
32.0 = -;d-
!65
= 4'65 bar
7.4
Indicated power, I.P. npnix I'ANkxI0
Total . =
7952 100 6

During a tral of a single cylind,er, 4_stroke d.iesel 1x4.65x0,44xl4x0.342 x4O0xlxl0


,rr^rrr#I!rr"17.44. engine the forbrr*;; _
=6t.9kW
6
Bore
itl Stroke
= 340 mm Brakepower, u.".= ttt =
*;to
=50.ekw
= 440 nm
f'p,m, .= 400 Frictional power, F.P. = LP. - B.p. = 61,9 60.9 = 11 kW
Area of indicator di.agram -
= 465 mm2 Heat supplied per minure
Length ofdingram
= 60mm = Fuel used per min. x calorific value
Spring constant
= 0.6 borlmm
Load. on hyd,raulic dynamometer
= ::::
10-6
x 49500 = g?45 kJ/min.
Dynarnomcter constant
= 950N
= 7460 (d) Heat equivalent of B.P. B.P. x 60 S0.9 x
= = 60 3054 kJ/min. -
Fuel used
= to.6 kelh (ii) Heat lost in friction
Calorific ualue offuel = F.P. x 60 = 11 x 60 = 666 7-n..
= 49500 kJlkc (iii) Heat carried away by cooling wate
Cool ing w ater circulate d
= 25 kglmin
Rse in temperature of cooling water
= fft, x cpu, (t.r_ t.r)
The mass analysis offuel is
= 25"C
:
= 2E x 4.18 x 25 = 26L2.5 kJ/min.
Carbon Mass of air supplied per kg of fuel
= 847o
Eydrogen
Incombustible
= 157o N xC 81x 84
= 7Vo = ss(co+cot = es(o+gJ =22'9kc,
Mass of exhausbgases formed
f:ir=Tff,_,
2 INTERNAL COMBUSTION ENGINE TESTTNG AND PERFORMANCE OF
I.C. ENGINES

Mase of exhaust gases formed/min. MORSnIESI


1?.46 . In o test of a 4-qlindzr, 4-strcfu engine 75 mm borc- atd 100 mm strohz,
the
Exanple
=23.ex ff =l.zztc followirrs
'fuet ""t;Ii"
uere obteined. atiutt throttle at a particulu consta speed etd wth fired setting of
Mass of steam formed per kg of fuel
' 'il.i. of 6.0 k9lh.
suWtY
= 1'35 ks with oJt eflinder worhing = 15'6 hW
... Mass orseam r".-*=J";L1u B.P. with qlind,er no. 7 cut-out = 11.1 hW
B.P. with qlind,er no' 2 cut-out = 11.03 hW
=1.35xff=o.zaaW-io. Bi.P. with eflind.er no. 3 cut'out = 7O.88 hW
Mass ofdry exhaust gases forured per min. B.P. with qlindcr rc.4 cutout = 10'66 kW
= 4.22-0.2;38 = 8.982 kg. Ifthz colorific ucltue ofthe fuel is 83600 N tfu and. clearonce volume is a.0o07 m3, calculote :
(iu) Heat carried away by dry exhaurt gases/nin. (i) Mechonbat effinicncy, GD ltdbaAd' thrmal fflcbncy' and
(i) Air stand.ard efflciencl.
= mgx cx x (t, - t')
Solution. B.P. = I.P. - F'P'
= 3.982 x 1.05 x (400 - 25) = 1668 kJ/min.
Assuming that the engine is running at constant speed the hictbnol end',putnping
losses
Steam is carried away by elaust gases. The temperature ofsteam ie also the same as that o
exhaust gaaes e.g. 400"C. Now if one |tinder is cutout it will not producg anr pgrgllut the frictional loss
remain constan.
speed ofthe engine is constant'
At partial pressure ofsteam 0.0l bar, the saturation temperature is 24.1'C. Therefore, stean an power tost in operabing te valves will remain the same as the
fL'S.p. reduction at the crankshaft due to oue cyliniler q:t out will b urctlX equal to tfu I'P.
is superheated.
pr od.u c ed by .that e linder.
Enthalpy of steam = hr+ cn|.op-t.) Therefore,
= 2!i45.6 + 2.1 (t00 -24.L) = 3934.89 kJ/kg. IPr= BP - BP1= 15.6 - 11.1 = 4.5 kW
LP. produced in cYlinder 1,
(u) .'. Heat carried by steam in erhaust gasee
IPr= BP - BP2= 15.6 - 11.03 = 4.67 k\tr
'I.P. produced in cYlinder 2'
= 3334.89 x 0.238 = 793.? kJ/min. I.P. produced in cYlinder.S' IPr=BP -BP"= 15.6 - 10.88 = 4.72 kW
(ui) Heat unaccounted for
I.P. produced in cYlinder 4' IPn= BP - BP'= 15.6 - 10.66 = 4.91 kW
= Total heat supplied - heat equivalent ofB.P. Total LP. produced = IPr+ IPr+ IPt+ IP-
- heat loet in friction - heat carried away by cooling water 1.p. = 4.5 + 4.57 + 4.72 + 4.9't = 18.73 kW
- heat carried away by dry exhaust gases (i) Me@hanical ofclncyr rlncc' :
- heat carried away by steam in erhaust gases B.P; 15.6
= 8745 - (8054 + 660 + 2612.5 + 1568 + 793.7) jF: = lrrg =O.8lllorStlS%. (Ag)
rloc-=
= 6.8LI/mi. () Indieatcd thermal efciency' q*r :
Ileat bIance sheet on mlnute bis: .
I.P' 18'?3 (Ang')
-. rr.f ^v - - = o.1344 or 1s..14%
Item hJ Per cent ffix$eOO
Heat supplied, [:t (i)AfussGandsdefflclercy'u.rudar!
87t15 100

(i) Heat equivaleut ofB.P. Strokevolume, V,=!nU=LnxO'0752x0'1=0'000411?mg


304 34.92 l;.
:
(ii) Heat lost in friction volume,
660 t.oo L
Kl.
cleaance v" = o'ooo1 m3
(iii) Heat criedfway by cooling water 2612.5 29.87 F.' v. +V" o.ooot4u+o.o0o1
(ru) Heat carried away by dry erhaust gases 1568 17.93 f' Compressionratio,'=T = "*=ffii=-: =5'1
(u) Heat caried away by steam in exhaust gasee 753.7 9.07
3f 1 -.- 1,, , ,t9q6. (A.)
(ui) Heat uaaccounted for .. qaiF.t&dad=1-ffi=t =oJl or
6.8 0.66 S: (S.4)-'-.
F a bore of A0 mm and -d-strohz of 90 mm' Its
Total 8746 100 f; Eranple 17.48; A 4'cxlinder petrol l^
: rated. sneed. is 2g00 r,o.n. nd, it ii "@Y
tsted.Athisspeedagainstbrohewhichlnsatorquearmof
604
INTERNAL coMBUsTIoN
ENCINES
0 3,7 n. The n brake load TESTING AND PERFORMANCE OF I,C. ENCINES
is 160 N and the fuel consumptt q05
netrol used. is 0.74 and it hrrs
the^cytinders are cut out
tto w
,r:?:* :*"!*.;;f;;;T;:;;0"hlT: 'f;;:i:; I:; :::::it;::il:[
in
*ii"",1,,.-b";:;;:;tiffilr!;i:t utith correspond,s7,;i;;;d-"'r?rt tr, ro4 and
IPt= BP - 3P" = 160 - 104 = 56 N
IPo= BP - 3Pn= 160 - 110 = 50 N
(i) The engit torque, Hence for the engine, the indicated load is given by
(ii) The brake thermal efficenqt, brahe mean effectue IP - IPr+ IP" + IPr+1P4 = S0 + SB + 56 + 50 = 209N
:itlTy specift pressure,
(u) Mechanical effiiienqt, lhe fuel consumption, BP
and \Lu.!.
(ui) Indated mean 160
Solution. Number of clinders, effecue pressure- Q-""r, = /p = ZOS = O.Z0S or ?6.5%. (Ans.)
Bore,
n=4
(ui) Indicated mean effective pressure, pmi
Stroke,
D=60mm=0.06rn ;

Z=90mm=0.09m
Speed,
Torque arm
rV = 2800 r.p.m,
n-.*. = *
Ymi

Net brake load = 0.37 m Pmb


P^ = = =e'66bar' (Ans')
Fuel consurnption =160N
= 8.986 litres/h
l**h- bH
Specifi c gravity of petrol =8.986x tx0.74kgth
Calorific value = O.74 IIIGHLIGHTS
(l) Engine torque, = 44100 kJ/tg
T: Perfornance of I.C. engines. Some important relations :
Dngine orque, ?= Net
brake load x torque arm (i) Indicated power (I.P.) *^#N
= r*
(ii) Brake (Ans.)
(w-'Jlz!&-+)
^"",.;;::;:;1";::j T,- (ii)Brake(8.p.)=
60x1000 kw - "
", f= jgl_kw')
(-oo*rooo.^"J
Brake power, r.r. = -:$I-000 -= 2n 1380+jg?
60;1000- _= 17.36
1. kW (iii) Mechmical efficiency, r.*
=

s.p. = "P-l4lt:lo (iu) Thermal efficiency (indicated), I.P.


6 q,".,,, =
;N
4 xp. xo:09x x (0'06)2 x 2800 x
u.g6 - ; xr0 and thermal efliciency (brake), r*.,", B.P.
; =
rhf xC
17.36x6x4x2 where uf = mass offuel used in kg/sec.
Pn
(lii) Brake thermal efficier
4-x 0.09 x x (0.00), x zs0-I6 = 7.31 bar. (Ans.)
(u).. Tltema.l
trG$ 4 :
= 'tu_shdfd
g
B.P. (ui) Measurement of air consumption by air
-,lh(B). =nrl;z 17.36 box method :
\o.Ybx1x0.74)
______;;;;__ x 44100 Volume of air passing thrcugh the orifice, V= g4O ACd
(lu) Specific frel consumptro.r, *.r... ,tUOO
and mass of air passing through the orifice,

s'r'c'=2-6'as kc/kwh' *,[ifi


: lm
= 0'383 (Ans') m" = 0-066 C x tr/mn
(u) Mechanicar.ffr.,i: where, = Aea oforice, m2

roro*,lin"
il;;;;ilJirlltlff*;;ring rhe brake roads insread orrhe
d = Diamete of orifice, cm
varues orB.p. h = Head of water of in .cm' causing the flow
as
Ipr= Bp _ gpr= _ r10 50 N
160
o = Density of ai in kg/m3 mder atmospheric conditioru.
=
IPr= Bp _ Bpr= 160 _ t0? 58
= N
606 1'ESTING AND PERFORMANCE OF I.C, ENGINES
INTERNAL COMBUSTION ENGINES
16. Voltage developed to sirike spark in the spark plug is in the rmge
(o) 6 tb 12 vols () 1000 to 2000 volts
OBJECTTVE TTPE QI'ESITIONS
(c) 20000 to 25000 volts (d) none ofthe above.
Choose the Corret Awer : 1?, In a 4-cfinder petrol engine the studard fring order is
The specifrc fuel consumption of a esel engine as compard to that for petrol engines is b) 7-2-3-4 (b) 1-4-3-2
() loer () higer (c\ l-3-2-4 (d, r-3-4-2.
(c) sme for same outrut (d) none of the above. 18. The torque developed by the engine is muimum
2. The thermal elliciency of perol engine as compared to dieael engiae s (o) at minimum sPeed of engine () at muimum speed of engine
(c) lowe () higer (c) at maximum volumetric efficiency speed of engine
(c) same for same power output (d) rame for aame speed. (d) at muimum power speed of engine'
3. Compression ratio of petml engines is in the range o=f Iso-octane content in a fuel for S.I. engines
(d)Ztg () 7 to 10 (c) retards auto'ignition (b) accelerates auto-igaition
(c) 16 to 20 (d)none ofthe above. (c) does not aflect auto-igaition (d) none ofthe above.
4, Compression ratio of rlieeel engines rnay have a range 20. Norrnal heptane content in fuel for S.I. engines
(a) 8 to 10 () ro b l5 (o) retuds auto-igaition (b) accelerates auto-ignition
(c) 16 to 20 (d) me of the (c) does not aflect auto-igaition (d) none ofthe above.
above.
6, The thermal efficiency ofgood I.C. engine at the atsd lod is in the range of 21. The knocking in S.I. enginee increses with
(c) 80 to 909o (b)ta7V% () incease in inlet air temperatue (b) incease in compression no
(c) 3O ta 35Vo (c) increase in cooling water temperatue (d) alltf e above.
6)lora2O%.
6' In case of s.I. engine, to have beet thermal elciency.the fuel air mixture ratio should be 22. The kocking in S.L enginc gets reduced
(a) lean () rich (c) by increasing the compression ratio (b) by retarding the spark advance
(c) may be lean or rich (d) emically conec (c) by increasing inlet a temPerate (d) by increasing the coong water temperature.
7. the fuel ai ratio, for maxinum power of S.I. engines, shoutd be 23. Increasing the compression ratio in S.I. engines
() lean () i (o) inceases the tendency for kno"Ling () decreroes tendency for klocking
(c) rray be lem or rich (d) (c) does not affect knocking (d) none ofthe above.
enically conect.
L In case ofpetrol engine, at starting 24. The knocking tendency in petrol engines will increase when
() rich fuel air ratio ig eeded ()reak fuel a ratio is needed (a) speed is decreased () speed is inmeased
(c) chemically cortect fuel air ratio is needed (d) any rel eir ratio wi do. (c) fuel-air ratio is made rich ( fuel-air atio is made lem.
9. Carburettor is used for 25. The ignition quality of fuels for S.I. engines is detemined by
() S.I. engines () Gas enginea (o) cete number rating () octane nuber rating
(c) C.I. enginea (c) calorifrc value rating (d) volatility of the fuel.
(d) none oftle above.
10. Fuel injector is used in 26. Petrol commercially available in Inilia for Inan pmsenget cars has octane number in the range
(c) S.I. engines () Gas engieg (o),l{) to 50 () 60 to 70
(c) C.I. engines (d) noe of the above (c) 80 to 85 (d) 95 to 100.
11, Very high speed engines are generelly 27. Octane number of the fuel used comercially for esel engine in India is in the rmge
(a) Cas engines ()SI ergie (o) 80 to 90 () 60 to 80
(c) C.I. engines (d) Steor eugines. (c) 60 to 70 (d) 40 tn 45.
In S.I. engine, to develop high voltage for sparhplirg 28. The knocking tendensy in C.L eoginc increases with
(c) battery is installed (a) decrease of compression rao (b) ircrease of compression no
{b) diskibutor is istalled
(c) carbuettor is (d) ipion coil is instaed. (c) increasing the temperatur ofinlet air (d) increasing cooling water temperature.
18. -rnst"Jld
In S.I. engine; to obtain required firing order 29. Desirable characteristic of combustion amber for S.I. engines to avoid knock is
(o) battery is installed (a) smal bore () short ratio offlame Path to bore
(b) disiritntor is instailed
(c) carburettor ie istalled (d) ierdtion co is (c) absence ofhot surfaces in the end region of gas (d) all of the above.
italled.
14. For petrol eugines, tle metltod of governing employed ir
(o) quantity gwerning (6) qutlity goeling
(c) hit aod miss goveming (d) oae of the above.
15, For diesel engines, the method of goveraing employed is r. (a) 2. (a) 3. () 4' (c\ 5. (c) 6. (o) 7, (b)

(o) quantity goveroing () quslitygwening


8. (o) 9. () 10. () 11. (b) 12. (d) r3. '() 14. (o)
(c) hit iud miss goveming (d) none ofthe above.
608
NTERNAL COMBUSTION ENGINES
i:.QF{TINC
AND PERFORMANCE OF r.C. ENCTNES 609
16. () 16. (c) 17, @) 18. (c) 19. (o) 20, () 2r. (t) '
22. (b) 23. (a) 24. (c) 26. () 28. (c) 27. (d) 28. b)
Also, calculate the brake specific fuel consumption if the air/fuel ratio is z0 : l. Take I for ai as
0.287 kllkg K and clearance volume as 0.000084 r3. [Ans. 61.4%,2I.75 kW, 0.4396 kg/kw]l
29. (.
Durinc a trial of a two stroke esel engine the following obsenatioro were recorded :
i .6.
.

.', nrgiou speed = 1500 r.p.m., load. on brakes = 120 kg, Iength of brake arm = g?5 m.
THEORETICAL QUESNONS Detemine :
(i) Brake trque (ii) Brake power. [Ans. (i) 1030 N-m; (tt) 161.8 kW]
1. (a) What do you mean by performance of I.C. engine
? A four*troke gas engine developes 4.2 kW at 180 Lp.m. and at full load. Assming the following data,
(b) Disnrss briefly the basic performance parameters.
calculate therelative efiiciency based on indicated power and ai-fuel ratio used. gplumetric efficiency
(c) Dismss with suitable sl<etch the brake mpe dynamomele.
= 87%, medranical efficiency = 74Vo, cleatance volume = 2100 cm, swept voluine = 9000 cm3, fuel
2, Describe how the I.P, of a multicylinder engile is meased ? coqmption = 5 mVh, calorific value of fuel 16?50 /mr.
= [Arte. 50.2%,2.456 : 1]
3' Dessibe the method commonly used in laboratory for rneasuring the air supplied to
an I.C. engine.
During the trial of d four-stroke cycle gas engine th folrowing data were noriled : .

4' Derive the formula used for finding the mass of air supplied to an engine using
an orifice tank.
Area of indimtor diagram = 565.8m'z
5' Explain the phenomenon of auto-ignition. Explain how auto-ignition is responsible
for knockins in S.I.
Lengih of indicator diagram
= 74.8 mm
engines. Spring index = 0.9 bar/mm
6. Explain the phenomma of knocking in S.L engine. What are the ilifferent factore which
inlluence the Cylinder dianeter = 220mm
knocking ? Describe the methods used to euppiess it. Stroke length
7. Explain the diference between (i) pre-igion, fii) auto-ignition ad (iji) detonation. = 430 mm
Number of e:glosions/min = 100
8. What is neant by igrrition delay ? Determine : (i) indicated mean effective pressue
9. What are causes ofknock in C.I. engines ? (ii) Indicated powe.
10. wla ar the difTeent methods used n c.I. engines to create tubulence in the nixture [Ans (i) 6.8 bar ; (tt) 18.5
kW
? Ttre following observations were recorded duriug a trial of a four stroke engine with rope brake
Explain its effect on power output and thermal efficiency ofthe engine. dynamometer :
ll. what do you mem by'octme numbey' mil'cetane numbef of fuels ? How re Engine-speed = 650 r.p.n., diameter of brake drum
they determined ? = 600 m, diameer of rcpe = 50 mm, dead load on
the brake drum = 32 kg, spring balance reading 4.25 kg.
=
Calculate the brake power, fAs. b.9 kwl
UNSOLVED EXAMPLES The following data refe to a four stroke petrol engine :
Engine speed = 2000 r.p.m., ideal thernal eficiency 35%, relativ effi ae cy gOEo, mechanical efli-
l' A single-cylinder petrol engine working on two-stroke cycle develops indicated power
of 5 kW. If the ciency = 655,
= =
mean elfective pressure is ?.0 bar and the pision diametei is 100 mn, calculate
eflisiency = 70%.
the average spee ofthe "oumetric
Iftheenginedevelops29.42kWbrakepowercalolatethecylindersweptvolume.
Diston. [Ans,0.001g5m]
t1. A single cylinder four'stroke gas engine has a boe of 178 mm anil a stoke of330 mm and is governed
lHint. Average piston speed = 2lN.l lAns. 109.1 rL/sl by hit-and miss principle. when running at 400 r.p.m. at full load, indieto crds are taken which give
2' A 4-cylinder petrol engine works on d mean effective pressure of 5 bar and engine speed
of rz50 r.p.n. a working loop mean-effective pressure of 6.2 bu, and a pumping loop rnem effective pressure of 0.35
Find the indicated power deveroped by the engine if the bore is 100 mm md bar. Diagram frm.the dead cycle give a mean efrective j."szurJof 0.62 bar. The engine was run light
stroke 1b0 mm
at the same speed (i.. with no load), and a nechanical munter recoded 4? frring stiokes per minute.
[Ars,6.11 kWj Calcrlate :
3' A4-cylirderfour-strokeS.I.engineisdesignedtodevelop44kWinmtedpowsataspeedof3000r.p.m.
(i) Full load brake power (ii) Mechaniel efficiency
The compression ratio used is 6. The law ofcompression md expansion is pln.a constant of the engine
= and heat
addition and rejection takes place at constant volume. Ttre pressure md temperature at
the beginning lAns. (i) 13.54 kW | (i, 84.7qo)
of conpression stroke are I bar and 50'C. The maximun pressure of the
12' During a 60 minutes trial of a single cyliniler fou stroke engine the following obsenations were re-
cycle is limited to 30 bar. coded :
Calctrlate t}le diameter and stroke ofeach cylinder assuming all cylinders have equal dimensions.
Bore = 0.3 , stroke = 0.45
Assume diarm factor = 0.8 and ratio ofstroke/bore 15.
[Ans,D = 95 mm, L = 142.5 mm] T, fuel consumption = l1.4 k!, calorific value of fuel = 42000 kJ/hg, brake
mean effctive pressure = 6.0 bar, net load on brakes
=
During the trial ofa fou-stroke dieser engine, the following obsenations were recorded : = ISOO N, r.p.m. = 800, brake drun dianeter =
1.8,n, bnke rope diameter = 20 mm, qumtity of jacket cooling water 600 kg, temperature rise of
jacketwate=55'C,qumtityofairasmeasuied=Z50kg,eJaustgastempeiature= =
Aea of indicator diagram = 475 mm2, length of indicator ctiagram 62 m, 42O"C,c,,for
= spring number = 1.1 bar/mm, exhaust grees = I kJ/kg K, ambient temperatvre 20"C.
dianeter of piston = 100 mm, length of stoke 150 m, engine r.p.m. 325. =
= = Calculate : (i) Indicated power, (r'i) Brake power ;
Determine : (i) Indicated mean effective pressure
(ili) Msgrni*1 elliciency (iu) Indicated thermal efficiency.
(ii) Indicated power.
{Ars. (i) 8.43 ba ; (iil 3.1 kWl Draw up a heat balance sheet on minute basis. tAns. (i) 47.7 kw, (tr) 42.g kw, (ttt) I9.9%, (iu) 35.86va)
A 4-cylinder, four-stoke diesel engine runs at 1000 r.p.m. The bore and stroke of each 13' A quality governed four-stroke, single-cylinrler gas engine has a bore of 146 mm and a stroke of
cylinder are
100 mm and 160 mm respectively. The rut offis 6.627o of the 280mm At 475 r.p.m. and full load the net load-on the frction bake is 433 N, and the torque arn
stroke. Assuming that the initil condition
of air inside the cylinder are I bar and 20'C, mechmical efciency of 71Vo, is 0 45 n' The indicator diagram gives a net area of 578 nmr and a length of i0 mm with spriug
calculate the ai-standard
efficiency and brake power developed by the engine. rating of 0.815 bar/mm.
610 INTERNAL COMBUSTION ENGINES
TESTING AND PERFORMANCE
OF I.C. ENCINES 6ll
Calculate : (i) The indieted Power (ii) Brake power Mean sPecific heat of exhaust gases = 1.0 kJ/kg K
(idi) Mechanical effrciencY. tAns, (j) 12.5 kW (tt 9.69 kW (tti) 7?' Specic heat of suPerheatcd steam = 2.1kJksK
14. A two-cylinder four stmke gas engine hro a bore of 380 nm and a stroke of 585 mn. At
240 r.P.m'
Specifrc heat of water = 4.186kl/kgK
torque developed is 5.16 kN-m. : (i) Indicated Power'
(ii) Brake powe
Detemine
(iii) Mechmiel efciencY'
Draw up heat balance shet for the tct. [An* (i) 17'1 kW ; (tt) f 1 kW ; (rit) 64.337,]
diesel engine developes 83.5 kW at 1800 r.p.m. with specifrc fuel consumption
2t, A 4-cylinder, four-stroke
of 0.2g1 ks/kwh, and air/fuel ratio of 23 : 1. The analysis offuel is 8?7o carbon and 137o hydrogen, and
it" value of the fuel is 43500 kJ/kg. The jacket cooling water flows at 0.246 kgdand its
"uU" rise is 50 K. The eaut temperature is 316'C. Draw up an eneigy balance for ihe engine.
iempeatur"
iJft=0.302kJ/kgKandco=l09kJ/kgKfort'hedv.exhaustgases.andcp=1.86kJ&gKfor
superheated steam. The temperature in the test house s l / -u' and the exhaust gas pressure is
1.013 bar.
247o, radiation ad unaccouted
[Ans. B.P. = 35.8%, cooling watet = 22.1%, exhaut = = 16.78"1
the following obsenations wee recorded :
22. During the trial of a single eylinder, 4-stroke, diesel engine
Bore = 350 mm, stroke = 450 mm, r.p.m. = 400, area of indietor
diagram = 4?2 mm'?, length of indicator
i.ct* = gz --, spring mnstant = 0.59bar/mn' load on hydmuc^dYnam-ometer = 9?0N, dynamometer
.or,!t*t = 7SOO, iuel usea = f 0.?8 kg/h, calorific value of fuel = 500O0 kJ/kg, cmlingwater circulated =
Calculate : (i) Mechaical elnciency 24 lites/min, rise in temperature of cooling water = 24"C' The-main analysis of fuel is : carbon = 857,,
(ii) Brake themal efficienry of the engine. udronur = l7r, incombustibles = 17o. The volume malysis of exhaust gases is : carbon oxicle = 87o,
18. A V-8 four-stroke petrol engine is requiretl to give 186'5 kW at'140 r'p'm' The brake thermal
oivg"i = ll%, nitrogen = 81%. Temperatue ofexhaust gases = 380'C, sPecific heat of exhaust gases =
can be assumed to be 82% alt the compression tio of9 : 1. The air/fuel ratio is 12
: 1 and th,e r f.i kfi.g'C, .-bient temperature = 20"C, partial presure of steam in exhaust gases = 9.93 6u.,
specific hJat ofsuperheated steam = 2'1 kJl<g'C'
elliciency at this speed is 69%. Ifthe stroke to bore ratio is 0'8, determine the engine g"-Pt-1t"t
t" tlimemions of the bore and^stroke. The calorific value.:f,tl1L"l:""10i Yff1 Draw up the heat balance shet on minute bmis'
""q"i."i*
;"";; J;;;ti;".* roi-s b;; G;c. [Ans' 5'12 litres ; 100 6 mm ; 80 5 lAn. (i) Heat equivalent ofB.P. = 34.55V0, (ii) heat lost in fiction = 8.78%,
19. During the trial (60 minuts) ou a single cylinder oil engine having cylinder diameter 300 mm, (iii) heat canied away by ooling water = 26.MVd,
-
OO working on the four sbokl cycle' the following obsenatioro were made : (iu) heat in dry exhaust gases = l9.54Vo'
^"m ""d
Total fuel used = 9.6 litres, caloric value of fuel = 45000 kJ/kg, total nunbcr ofrevolutions = (u) heat mied away by steam in exhaust gases ='1.24Vo,
on the
gross inrlicated m"* ufie"" p*ue = ?.24 bar, pumping i'm'e'-= 0'34 bar'.et load (ui) heat maccounted for = 3.107o1
] efSO N; dianeter of brke wheel clrum = 1.78 m, diamiter of the rope = 40 mm, moling
circulated = 545 tes, cooling water temperature rise = 25'C, specific gravity of oil =
0'8' 23. A4-cylinderpetrolenginehasaboreofTmmandastokeof90mm.Iteratedspeedis2800r.p.m.and
itis tlsted afthis speed agaimt a brake, which has a torque am-of 0.356 m. The net brake load is 155 N
Determioe : (i) IndicatedPowen (ii) Brake powe' petrol used is 0.?35 and it has a lowe
and the fuel consmption is 6.?4litres/h. The specifrc gravity of
(dii) Mechanical efr cieDcY.
(iii) 80' calorific value of 44z}OkJtkg. A Morse test is caried out and the cylinders are mt out in order 1, 2, 3,
Draw up the heat balance eet oa minute bsis. IAns. (i) ?? kw (ti) 61.77 kw ; 4 with conesponding brake ioads 111, 106.5, 104.2 mil lll N, respectively. Calolate for th,is spee<l :
20. fire following results were obtained on full load during a trial on a two stroke oil engine: (i)The engine torque. (i) Thebrake mean effective pressue.
(iii) The brake thermal effrciency (tu) The spcificfuel consumption.
Engine speed = 350 r.p.m.
(uj) The indietd mean effective pressure.
Net brake load = 600N (u) The mechanical efrciencY.
m.e.p. = 2.75 ba [Ang. (i) 55.2 N-m ; (ii) ?.55 bar ; (iii) 26.6% ;Gv) 0'306 kg/kWh i fu) 82.8Vo; (ui) 9.12 bar]

Oil consumption = 4.26kelIr


Temperature rise ofjac.ket cmling water =25.C
Air used per kg ofoil = 31.5 kg
Temperature ofair in test mom
' =20"c
Temperatqre of exhaust gase = 390.C
Following data also apply to the above test :

Cylinder diameter =220mm


Stroke = 280 mm
Effective brake diarneter = 1 metre
Caloifrc value of oil = 45000 kJ/kgi
Proportion ofhydrogen in fuel oil = lSVo
Partial pressure fsteam in e:haust gases = 0.04 bar
I

rl
FROM T.C. ENCINES AND ITS CONTROL
I
s.AlR POLLUTION

il. ri
ij
Mechanism of formation of CO:

18
i,l
CO is generally formed, wfun the mixture is rich in fuel.The amount of CO formed increases as
rl mi*ture becomes more and, tnore rich in fuel, A small amount of CO will come out of the exhaust
when the mixture is slightiy lean in fuel. This is due to the fact that equilibriun is not estab-
rl I
Air Pollution from f.C. Engines when the products pass to the exhaust. At the high temperature developed during the com-
I bustion, the products formed are unstable, and the following reactions take place before the equilib-
i
rrl and fts Control iium is established.
,I
rl 18.1. Intmduction. 1.2. pouot"r*-pl.t-+i^- ^r--^r .
ZH.O + Qr--------: 20 - flllrO + 24IJ., + yO,
srrength ad combustion produ3i3-P:illlgt derived from .o.Tb":I9t products-Mixture ?.where,y is the fraction of HoO dissociated.
,i'l
rl
c.""i.o;;;;;;;";:":";;:;:L.THijf; :5i';HlTt,"fi.]"l: j'Jiffi ffi t:L; C + O, --------r CO2 -----r (1
control-Modification in the-engine oestgn -r)CO, + rCO + f O,
ara ope.;;;;;g;'"*,"_RvLa,,.,
ausicn and operatingpuamerers_f*f,a""t gis
ii n_L.""t J;;;iilT:ff: -^^ o*id;lil;:
^..:r-.!
"^"1i"" conhol-Evaporation As the products cool down to exhaust temperature, -ajo" J^.t of CO reacts with oxygen to
GEcD)--c;nil ;ilft;: ;;;Ylation-Blow-bv
(NoJ ";;;J;i;'"}; form COr. However, a relatively small amount of CO will remain in exhaust, its concentration in-
-?otal emission control packages. re..'Di"""i
;li engine emission.. t8.0.
u*""-"ii- oi
oi"""itl5e:
i"d mntrol-lxhaust
ot smok+Diesel
"#;-'""#'"T""1:
gasolio" *ali"sr]"-*t"ir.'il'"i
.-oL"-c""?"" ;i ;;;k=
odour and control. 18.7. ;;;;;#;
creasing with rich mixtures.
l',.ll 2. Hydrocarbons (HC) :
contor. r8.r0. rn"*" oi Frr gas tubine and its
i:,, I "ffi";i::HfJTfl#:;fi5lhti:l ightights-objective
Questions-Theo;at."l d"*1""" rlpe o Hydrocarbons, derived from unburnt fuel emitted by exhausts, engine crankcase fumes
and vapour escaping from the carburettor ae also harmful to health,
I'li ' Mechanism of fortnation of HC
iil 18.1. IflTRODUCTION
- Due to existence of local very rich mixture pockets at much lower tgmperatures than the
Airpollutioncan combustion chambers, unburnt hydrocarbons may appea in the exhaust.
bedefinedas,i!^!!:rb:":ur atmosphere of The hydrocarbons
have a deleterious effect on life any materialwhch witt
upon our planet.Besides I.C, engines
other
-A significant portion also appear due to flame quenching nea the metallic walls.
of this unburnt hydrocarbon may burn during expansion and exhaust
as electric power stations, i"a"ri
rn;lffil"l"."nsumers also addsources such
polrution. srokes if the oxygen concentration and exhaust temperature is suitable for complete oxidation.
There has been a great concen,
in recent years, that the internar conbustion Otherwise a large amount ofhydrocarbon will go out with the exhaust gases.
engines
::T:il:t:i",,,Y,,T""Tufi"t^o,nr,u."i.
poli,,rio;:ffi;'; detirnentar ro human hearrh
is
i 3. Oides of nitrogen (NO"):
;;;;";;;;:;;,;"i)"":,iiij"i'lill""l":f;t,:#"i?::,#:X"rn::i:lJl::,::::::::: t
o Oxides ofnitrogen and other obnoxious substances are produced in very small quantities
and, in certain environments, can cause pollution ;wbile prolonged exposure is dangtr-
18.2. POLLUTANTS ous to health.
I
t
pollution Derived from Mechonism of formation of nitric oxide (NO)
18.2,1.
Combustion products t
At high combustion temperatures, the following chemical reactions take place behind
Pollutants ae produced by tl.e incomplet"
ourntng ofthe ai-fuel mixture in the - the flame
chamber' Themajor joilurns e;ift; combustion :
;;';i;ffiili due to incomplete combustion are N, + O, ------r 2NO
1. Carbon monoxide (CO) :

2. Hydrocarbons (HC) N, + 2IIrO ---------+ 2NO + 2H,


3. Oxides of nitrogen (NO_).
Chemical equilibriurn alculations show that a signifrcant amount of NO will be formed at
the end of combusion. The majority of NO forrned will however decompose at the low temperatures
Other products o.o,r,i,.^*"::i".? lene, of exhaust. But due to very low reaction rate at the exhaust temperature a part of NO formed
aldehydes etc. I{ however, combustion
tlrc only products being exoelled from is complete
the u*"u"t *ri he. water uapour remains in exhaust. It is far in excess ofthe equilibrium composition at that ternperature as tirc
cttrbon trioxide, which is an inert gas whichi" hr.*t"ss, and
rrr, airu"tty t umful to humans. formation ofNO freezes at low exhaust tempeatures.
lr 1. Carbon monoxide (CO): "" "uJit'iJiit The NO formaton will be less in rich mixtures than in lean mixtures,
o It js a colourless gas ofabout the same density -
It Smohe or particulate
il It is a poisonous Easril]1llh"",i:haled, as arr. Solid particles are usually formed by d,ehydrogenation, polynerisation and, agglontero-
that the body's metabolism can not function replaces the oxygen in the blood stream so tion.
ti
o smat amounts of co concentrations, when co.rrecfly. o In the combustion process of different hydrocarbons, acetylene (C2Hr) is forrnerl as an
breathed in, srow down physical
activity and produces headaches, *fri" and mental intermediate product. These acetylene molccules aftc simultaneous polymerisation and
ij frig..r"centration will kill.
dehydration produce carbon particles, which ae the nrain constituent ofthe particulate.
612
AlR POLLUTION FROM .C. ENGINES AND
INTERNAL COMBUSTION ENCINBS ITS CONTROL

Ald.ehydes
-
o Due to very slow chemical eaction during delay period in the diesel engnes,ald.ehyiles
are formed as intermediate prodzcs. In some parts ofthe spray the aldehydes will be left
after the initial reactions. These aldehydes may be oxidised in the later part ofthe cycle,
if the mixture temperature is high, and if there is suflicient oxygen.
/l

E
|
I
2400

20(b
7 t\ )
10
s
o At heauy loads, due to lach of orlgen, an increase in aldehyde emission in the exhaust is !fr
gA
rom
:s / sE
obserued,.
F3 12oo
\ 6F c
Following points ore worth noting: P fr
If the air-fuel 6:, ,&.1""/
1. mi.xture is too rich there is insuflicient air for complete combustion aud t'e ol
4
some ofthe fuel will not be burnt or at least only partly burnt. Since hydrogen has a greater afnity
for oxygen, hydrogen will take all the oxygen it needs leaving the carbon with a deficiency ofoxygen.
EO
r
aoo
.-i lyd catb.
m
6

4oo 2
As a result of the shortage of orygen o, percentage of the carbn will be conuerted to corbon monoxde
as well as carbon d,oxide, and, with uery rich mi*tures, porticles of pure unburnt carbon may be 0 0
erpelled from the erlnust as "blach smahe,', 11 12 13 14 15 16 17 18

Incomplete combustion due to partial oxidation ofthe hydrocarbon fuel also produces other Air-luel ratlo --->
products such as acetylene and,ald,ehydz. These products, when expelled from the exhaust, Ieave an
Fig. 18.1. Eff*t of miture strength on exhaut ompositim of a petrcI engine'
unpleasant smell and are particularly noticeable during engine \arm-up when a rich mixture is
provided.
2. If
the nixture is ma.d too weah it is unlikely that the atomised liquid fuel will be thor- r8.8. SPARK IGMTION (S.I.) ENGINE EMISSIONS
o tghly mixed throughout the combustion chamber so thatslow burning, incomplete combustion and. The following are the three main sources from which pollutants are emitted from the s.I.
rnisfiring may result. engine :

A further characteristic ofweak mixtures is tha the excess oxygen (which has no taken part 1. The crankcs ee.whcre piston bhw-by fumcs and. oil mst are uented to the atmosphere.
in the combustion process) at very high temperature is able to combine with some of the nitrgen 2. The fuel syst en. wlwre euaporatiue enissions from the corburcttor or petrol i4ieetion air
that constitutes about three-quarters of air, to form oxides of nitrogen such as nitrogen p"roiid, ntahe and fuet tanh ore uentd to the atmasphere.
(NOr). The amount of nitrogen peroridc prod.uced will increase as the m*ture weahens intii it peahs 3. Tlre exhaust systD. where tlw products of ncomplete combustipn
are expelled ftom the
at just ouer an air-fuel ratio of 15.5 : 1, beyond this pont the ternperature of combustian begns to tail pipe into the atmosPhere.
below that necessdry for formation of nitrogen perord,e so thit with further reduction in msture
fdl
Evaporative
strength the arnount f nitrogen peroxidz progreesively d,ecreases. emission Crankcas
(Hc)
18.2.2. Mixture Srength and Combustion product Charcterisics emission
Trte chemically correct air-fuel rotio b1 mass for complete combustion is known as l-Hcl
tcoJ
soichiometric ratio. Evaporativ

Refer Fig. 18.1. It shows how the theemain exhaust pollutants products (CO, HC, NOJ
vary from different air-fuel ratio operating on either side ofthe stoichiometric ratio for a
very rich mixtue (11 : 1) to a very lean mixture (18 : 1).
The anount ofCO produced in the exhaust is about 8/o for an 11 : 1 air-fuel, ratio,
but this percentage steadily decreases to zero as the mixture is reduced to just
beyond the stoichiometric.ratio (on the lean side).
r HC.produced in the exhaust gases amounts to about 1100 parts per rnillion (ppm)
with a rich 11: 1 air-fuel ratio and, as the mixture stren6h approache-the.
.' stoichiqmetric ratio, it progressively falls to around 600 ppm. A further weakening
" of the mixture to 18: 1 air-fuel ratio only reduces HC content to approximately
350 ppm.
Oxid.es ofnitrogen products formed during combustion are very low at 100 ppm with
a rich air-fuel ratio of 11 : 1. As the mixture strength approaches the stoichiometric Exhausl
emissions
ratio it rises fairly rapidly to 2000 ppm, and a futher reduction of the mixture
strength to 1.5 : l peaks the oxides of nitrogen to something like 2,800 ppm, weaken- rco I
ing the mixture beyond this point rapidly reduces it until, at an 18 : 1 air-fuel ratio, lHcl
I
No.l
it is 1000 ppm. LPB I
Fig. 18.2. Spark ignition engine emissions.
l'
li,
lr;
I
INTERNAL COMBUSTION
ENCINES
t
18.3.1. Crankcase ArR polLtrrroN FRoM r.c. ENGINEs AND rs coNTRoL .617
:ll Emission
Piston rig blow-by : (il) the air-fuel mixture, partially burnt and fuily bumt
#iil r The piston and its inss
d;:'ffg contominate th engne's lubrbating oil.
vapour firmes, will cozde nse snd
spirt and the cytinaeJwji:

o forn a gas-tight seal berween
rhe sliding piston Since it is impossible to eliminate piston blow-by completely,
;";'i;;;iJkiJ'J";[i1:::;:i:::::;t!11w,iu
"h"'c" ri.nss
atwit"-"")'^"'iil,]o,"*"o an organised, connection is d.e.
liberately created whch circulates the crankcase and, rockei o" cmshaf,cor",
piston and tizerefore enter the
l!":::f*"son and; oit controt quently carries the unwanted.fums out wh i. The removal
ofblow-by g"r", "p";;;;"d-;;;;-
m"y b" ";;"1;;;T'idlli
wav past the piston ring bett 9'' r'hese sases which find thei vapour fumes
partially ;;;;llliir'r"
parriaryburnr)produc-rs"i*_i,"r"it6;,1i;::#Jli::yj.
burnt) producis or @mbustioh en hydrocarbons, or burnt (o
from the crankcase is obtained by creo ng a portial dzpressinn at the ouellocoton "'odso that blow-by
-
,_.|]xture
^
gases under pressure (escaping between th pisbn
d cylinder wall) arb attracted
---:--' towards the
lower pressure region of the craakcase, at which point they are er;ll;l.--'
9<
Following are the two methods of cieating the ettra.tion d.epressian
:
'4:. (i) The road d,raught ciankcase ventilation system.
' - - The unacceptable limitation of this system of cankcase ventilation, due to expulsion of gas
and,fume vapour (HC and Co) into the atmosphere thereby contributing
to pollution, has made this
method ofinternal purging ofth e eng;ne obsilete.
(i) The induction moniford vacum positiue cronkcase
uentilation ststern.
o Theblow-byHCemis-si.onsareabout2}voofthetotalHCemissnnfromtheengine;thiss
increased to about
S1Vo if the rings or" .oo.

Escaping 18.3.2. Evaporative Emission


charg Esgapng
Evaporative emissions account for 15 to 25Vo of total hydrocarbon
exhausl emission from a gasoline
engine. The following are two main sources ofevaporative
gases emissions :
(l) The fuel tank
(il) The carburettor.
(i) Fuel ank losses. The main factors governing
the tank emissions arc fuel uolatility and
theambientternperaturebut,thetank designan locatiii canalso
influence the ernissions as location
affects the tentperature. Insulation of tank and vapour
collection systems have all been explored
with a view to reduce the tank emission.
(li) Carburrettor losses. Carburettor emission may
be divided with following tw ocategorizs :
(d) Running losses
; (il) parking losses.
1i o Piston blow_by increases with
Fig. 18.3. piston ring blow-by
- Although most internally vented carbuettors have an external vent which open at idle
engine
and, in particular, as the prston speed t.( throttle position, the existing pressure forces prevent orin"* orr"pour.'to
tt ,t-or-
rings and phere. Internally vented carbuettor may enrich the "
llll:: p*
Decomes more
*ears (Fig.,r'8..a), ,h;r"d; t
noticeable IntneuPPersPeed
exhaust emission
mixture which in turn increases

range. oos Carburettor


- tion. The fuellosses. a,re significant only during hot condition when the vehicle is in
- opera-
o Blow-bytakesplacebetweenthepistonring volatility also affects te cabJrettor
gap, piston-ring to piston_groove e 0.06
:r 18.3.3. Exhaust Emission
".i"rioo".
.uu.*.3 ^^^
and in the T.D.C. region "."
tt" piJt""
ili5.j'i"i*l:l:ll1'-_",n'o:
raiely follow the contour of
.";;;;;;'l ;
an ilo:ji

o.oo
0.04
The different constituents which ae exhausted from S.L engine
will affect percentages ofdifferent constituents are discussed berow
1. Hydrocarbons (HC);
and different factors which
:

mounter cylinder wall. .o


o Besideseffectivelyreducingtheenginecom_ The emission amount of HC (due to incomplete combustion)
o.o,
is closely related to :
pr-ession ratio and the powr desgnuariables (such as induction system and combustion
de"";;;; - chamber design);
effects of piston blow.by are operating uariable (such as A,rF ratio, speed, load)
(i) It can lead to a
two_fold :
o@o - mode ;
hilh concentration of ofoperation (such as idling, running or accelerating).
combustible air _fuel
-
l mkture which cout The following/octors tffect HC emisson:
ii
cause an exptosion in tie.cranh"oi".--* ,.
(i) surface/volune (s /v) ratio. Fig.lg.b
shows the effect of s/V ratio on HC emission.
",r. ":;ril"".ffirJT:r;:l;
prston and ring blow-bv. (ii) Wallquenchng.
I
619
ITS CONTROL
AIR POLLUTION FROM T.C' ENGINES AND
618 INTERNAL COMBUSTION ENGINES
is emitted from the exhaust which forms
Maior portion ofthe lead that enters the engine
(iii) Incomplete combustion. When the mixture 500 #';^sl"affi;.i".'"i"Ja"" in the atm.osphere' A portion of the lead
""Joxvt'aliies enough to remain suspended
supplied is rich or lean ; the flame propagation becomes particles falls to the grouno Je"v qtitify, "-tft"rs are sma'll
so"-"ti^", before thv fall out' usually after coagulation with other
weak which causes incomplee combustion and results in 400 il;;;*h"i"
HC emission. The incomplete flarne propagation is caused
by the following factors :
A

: I 9P dusty material in air.


It may not e possible to eliminate lead complebely from
all petrols immediately because
provided by a lead film to
g
I

(o) Law charge temperature ; 300 a large number of existing i"fv opoo th" lol"i"atioo
""*""t seats' However' a very small lead content would be
*ear of exhusalve
() Too rich or too lean mixture ; ;;;;;t"pt
(c) Poor condition ofthe ignition system

engine ;
;
(d) Non-uniform fuel in the mixture supplied to the
I
j
't00
zoo
9 adequate for the PurPose
Following Points are worth noting :
oBothLheftowratearrdpollutantconcentration'forexhaustemissions'canchangewith
- emissions'
-"ll"p"Jion. s;h -ost be consideredexhaust HC concentrations are low while the
rr" in determining
(e) Large exhaust residual gases left in the cylinder.
(iu) Sparh plug timing. lt s observed that HC emis- [Jnder constant high speed' condtians'
is low but$C concentration is
sion is reduced by retarding the spark plug timing during
45678 io* rot", oo" nel. fiie iieleratins the flow rate
slr' ---> hish.
low speed (lower than 40 km/h) but has no effect when run- losses is virtually inde-
ning at 40 km/h. Fig. r8.5 _ T1.ecotrcentration oHC in the crankcase anil evaporative
of operuti"j *"aitio"" ,but theflow ras fiom each of these sources chonge
pend.ent
(v) Compressinn rofio. It has been observed hrough experiments that emission of HC in er-
during various operations'
haust is decreased with an itlerease in cornpression ratio,
Thus, on km basis CO and HC emissions decrease
with increasing driving speed
2. Carbon monoxide (CO) nq are relatively not affected'
3
*ftif" ernissions
o Ifthe oxidation ofCO to CO, is not complete, CO remains in the exhaust. o ln o poorly maintained' engine the erhaust pollution
is more'
. It can be said theoretically that, the petrol engine exhaust can be made free from CO by An autornatic choke sticking in the closed position or a very dirty air cleaner ele-
operating it at A/T ratio = 15. However, some CO is always present in the exhaust even - ment can reduce air-iuel rat'0, generally inreasing HC and CO emissions'
at lean mixture and can be as high as 1 per cent. without combustion'
A misfire allows an entire air-fuel charge to be exhausted
c 'I);e percentage of CO ncreases durng engine idlng but d,ecreases with speed. -
Whatever may be condition ofrunning at any load or speed, and A/F ratio, it s not 18.4. S.I. ENGINE EMISSION CONIROL
possible to completely eliminate CO and,0.5 percent is considered a reosonable goal. engine emission control are :
o CO emissions are orest durng acceleraton and, at stea.d.y speeds. They are, however, The main methods, among various methods' for S'I'
design aird operating parameters'
high d.uring id,Iing ond reach ma-rimum dur.ng deceleration. 1. Modification in the engine
2. Treatment of exhaust products of combustion'
3. Oxides of nitrogen ( NO,) :
3. Modification of the fuels'
o Oxides of nitrogen occur mainly in the form of NO and NO, and are generally formed at Paraneters
high temperature. 18.4.1. Modification in the Engine Design and Operatlng
quality' A few parameters which
o The rnaximum NQ levels are obserVed with A/T ratios of about 10 percent above Engine desigrr modification improves upon the emission
stoichiometric. More air than this reduces peak temperature and therefore NQ concen- improve a-n emission are discussed below :

tration falls, even free O, is available.


1. Combustion chamber configuration :
o The following factors affect the formation of NO" : flame quenchinq zo'ne where combus-
Modication of combustion chamber involves avoiding
(i) Al F ratio ;
incomplete and resulting in high HC emission' This includes :
tio., rr,igi;t oth.rwise be
Reduced surface to volume (S/V) ratio ;
,lii]Xi;J;;, angle of advance decreases appreciabty the roma-
Reiluced squish area;
tion of "ruance.Thedecreasing
\e.
It has also been observed that NQ increases wth increasing manifold pressure,
Reduced sPace around Piston ring;
engine load. and cornpression ratio, This characteristic is different from HC and CO
Reduceddistanceofthetoppistonringfromthetopofthepiston,
emission which are nearly independent of engine load except for idling and decel- 2. Lower comPression ratio:
the quenching area'
eration. a lower compression ratio reduces the quenching effect by reducing
Lead emission ll:;us reducing HC'
o Lead etnissions cott only from S.I. engines. o Lowercompressionratioalsored'ucesNoremissionsduetolowermaximumtemperature'
and, increases fuel consumption.
o In the fuel, lead is present as lead tetroethyl or tetramethyl, to control the self ignition o Lower compression, howeve r, red.uces thlrmol efficency
. tendency offuel-air mixtures that is responsible for knock (to improve the octane rating
ofthe fuel).
620
INTERNAL COMBUSTIoN
The decrease in ENCTNES poLLUTroN FRoM r.c. ENcrNEs AND m ooMrRoL
will result in com ratio is becomingvt 62l
ie;;;;'#;:f'::":lon r"ot lt r""* desigrr parameter, because
wutdecrease ananAnl".llit-"Itt ffiiit{t*f:"tant
toberepraced,ll;;:;:;ffi!:trr"u":.tt"o,l;;";:;;,::i;:',::3;2;::##;;:t;j:::;:i
ir o Thjs arrangernenUsystem is not successful in reducing the emission owing to the diffr-
culty in sustaining the combustion during low HC emissions due to high het losses over
B. Modified induction system a lage area.
:
2. Exhaust manifold reactor:
,. {" a multi-cylinder engine it is always difficult
to o The exhaust manifold reactor is a further development ofafter-burner where the design
;:l::JJ"""'ff ll ;il1*"'*,t[i,",','::::,ff is changed so as to minimise the heat loss and to provide sufficient time for mixing of
4.
"il#:H|f;n*iffi
Ignition timing:
;jl:j:X"*ff fi :; exhaust and secondary air.
r The ignition
r Fig. 18.7 shows a General Motors, air injection system. )
timin
Anpur,p
fu;:lZrinTr,;ifu:#*i:n$:,".**H1*:H::::fi :z^;::;: I
Air in ----+
if#:r:{:;t:i#r::r:r:*fr*:i:X"*:#.''":,;#{#,iiJt!##" Air distributor
5. Reduced value verlap: to each
exhaust port

:f "*Jr;#i*:",":,,:lr;,ffi,:;X::f*2,,!ffi allows for controlled


0,,*.,,"andincreaseemission
Exhaust
; improves echedulins of valve
"y9.nt.petrol
wtrn
engines. -
ir,:"*iolsTi,l""f"tt;";:l;;-;
"1"-,lt::ot.<uot)
n. rhis
"v"tu-
timing
,'"..o"ff;"'i;,'ffi ffi:':"mt:,Jr:":l$l
valve

18.4.2. Exhaust
Gas Oxidation
The exhaust gas coming Pston exhaust Filtered ar
to reduce Hc and co emissons. stroke after combuston --|
, ;li.:;:fi:",:".'{::"i:^r,trki;:#:iif"'.*:#r, ".'" Unburnt
hydrocarbons
o An "after.burner" (Ni- to ot .
mbture is burii ,;;;1;:^':i-:: burner p," o- ' ----) Treated exhaust

::1i",.il;;;::;::::::1i"y.#::;:"y:i:r:!iy::#,.{9';:,:"{kHii:;?ii3 Fig. 18.7. General motos air injection system.


il"^ll,ot providing ai in a ""p"."tJ uo*l iiii,l'^1d"-o,ate time to burn aie further Here a positive displacement vane pump, driven by the engine, inducts air from the air
;:f;::":,:7. ff :il: jf
;111,f i
;;r ;1i fi ffi
:" "n"^
b u r n r' r\e e

"R ".'
o.n
"..
i. - cleaner or from separate ai filter.
The air passes into an internal or rixternal distributing manifold, with tubes feeding a
t"".tffi fr i$fu#i,T,1ffi,;:*,"":T''f ri1TJ,::1ii{l:"*,."::i1{jt*"$. - metered amount into the exhaust port of each cylinder and close to the exhaust valve.
Since the exhaust gases are at high temperature, the injected air reacts with HC, co
- and aldehydes to reduce greatly the concentration
By pass valve ofsuch emissions.
The injected air is closely metered otherwise it can decrease the temperature of the exhaust
secondy ah
o In earlier type of reactor developed by Du Pont the entry of exhaust gases was radial and
--+ Exhausl the air flow peripheral.
Engine out 3. Catalytic converter :
exhaust
,9niton o Acctalyticconuerterisadevceuhichisplaced.intheuehicleexhaustsystemtored,uceHC
source and CO by oxidsing catalyst ond. NO by reducing catalyst.
o The basic requirernents of a catalytic conuerter dre :
Fig. 1a.6. Dect flame (i) High surface arca ofthe catalyst for better reactions.
after_burne.
(ii) Good chernical siability to prevent any deterioration in performance.
rl
(ijj) Low volume heat capacity to reach the operating temperatures.
INTERNAL COMBUSTION ENGINES AIR POLLT'TON FROM I.C. ENCINES AND ITS CONTROL 623

(iu) Physical durability with attrition


resistence. 2. Two-way cetalytic converter:
(u) Minimum pressure drop during
the flow oferhaust gases through the catalyst bed ; In two.way catalytic converter, the eraust line has two catalytic converters in i. A reduc-
this will not increase back pres=sure ofthe engine.
tion catalyst i8 required to reduce NOr. Arr oxidation catalys is placed downstream ofthe reduction
Fig. 18.8 shows a catalytic converter, developed by the Ford company.
It consists of two catalyst to convert the excess HC and CO. The advantage of two-way converter is t}rat il allows a
separae elements, one for No,-and the other f". partal &oupting of emssinn control from engine oryrdtinns.lherefore the conversion efciencies
/co emissions. The secondary air is injected
ahead of the first element. Thehow in the converter
i" u.1. fo HC and CO are very high at normal exhaust temperatures.
3. Nobl metal catalrtic converter:
They use noble metals as catalyst materials. Platinum or Platinum and Palladium arq ap-
plied to a ceramic support which has been treatcd with an aluminium oxide wash coat. This results
in an extremely porous structure providing a large surface area to stirnulate the combination of O,
with HC and CO. This oxidation process converts most f these compounds to wate vapour and
coz'
Failures of caalYtic converter :
The failures of catalytic converter may be due to the followin g fattors I teasons :

1. Converter melt down 2. Carbon deposit


3. Catalyst fracture 4. Poisoning.

The eason for caalytic converter being so popular:


Fig. 18.8. Catalrtic onverter. ' The most common range of exhaust gas temperatureo for S.I. engines is 40o.600'C, and the
Orid'atian catollic teactionc. CO, HC and O, from air are catalytically exhaust gas may contain modest amount ofO, (when lean) and rnore substantial amount of CO
- CO, and-HrO and numbe of catalysts are known to be effective
converted to (when rich). In contrast diesel engines always operate lean. The exhaust therefore contains substan-
noble metals like plati- tial amount ofoxygen and is at low temperatures (20G-5)"C) removal ofgaseous pollutants from
num and plutonium, copper, vanadium, iron, cobalt, nickel, chromium
ets. the exhaust gases after they Ieave the engine may be either thermal or catalytic.
Reduction catalytic tee,ction* The prirnary concept is to offer the
- activation site, No molecule an In order to oxidize the hydrocarbons in the gas phase without catalyst (thermal), a esident
say-nickel or copper grids in the presence of co but not o, which wilr time ofthe order of50 minutes, and a temperatr in excess of?)'C are required. To oxidize CO
cause oxidation, to from N, and cor. The No may react with
a metal morecure to form an temperatures io excess of 700"C are required. Temperatures high enough for this purpose can be
oxide which then in turn, may reac-t with co to restore the metal obtained by spark retard (with loss in efficiency) and iusulation ofexhaust parts and manifold. The
molecule.
Rhodium is best catalyst to contror Norbut AtF rstia must residence tine can be increased by increasing the exhaust manifold volume to form a thermal reac-
be within a narrow range of
14.6:ItoI4.7:L tor, However this afrproach has limited appliiation.

Mqjor drzrtbachs of catolytie conoerter Catalytic oxiilation ofCO and HC i the exhaust can be achieved at temperatures as low as
ate as under : 260"C. Ttrus-effective removal ofthe pollutants occur over a mc wider range ofexhaust tempera-
(l) Owing to the exothermic reactions in tures tha can be achieved with thermal oridation.
the catalyst bed the exhaust systems are lro tter than
nartnal.
The only sa,tisfortory method hnown for the remaval of NO frorn exhaust gases involue cata-
, (ii) cars equipped with such converer should
catalytic actiuity.
not use leoded fuel as lead d,estroys complete lytic procesvs. Removal of NO by catalytic oxidation to NO, requires tetnperdturcs less thon 400'C
(from equilibrium conditions), and subsequent removal ofNO, produced. Catalytic reaction ofNO
(tjj) Ifthe fuel contains surphur (as dieser oil) emission with added NH, is not practicable because ofthe traneientvariation in NO produced in the engine.
ofso, is increased.
Three-utaX4 Ttoo-utay ond tnble metal coltollytia conaetter.s: Reduction of NO by CO, HC or H, in the exhaust to produce N, is the preferrd catalytic
pocess. It s onrJ feasible in S.I. engine exhaust.
1.. Three-way caalyfic converter:
Therefore, use of catalytic convertor for CO, HC and NOr removal in IC engines has become
If a engine is operated 4t all times with an air-fuel ratio close to stoichiometric,
then both
wide-spread.
No reduction and Co and HC oxidation, can be done in a single catalyst bed. The catalyst effectively
O"t*t 18.43. Exhaus Emission Control by Fuel Variaion
g composition to a near equilibrium state at these exhaust conditions, i.e.,
comPosition"{1Tt
11." a o 1be ability of a fuel to burn in mixtures leaner than stoichiometric ratio is a rough
of cor, Hro and Nr. Enough reducing gas will be present to red.uce
No, and enough o, iadicaon ofits potential emissioi reducing characteristics and reduced fuel consumption.
to oxidize the CO and hydrocarbons (HC). Such a converter
is called three_way catalyic con_
vette4 since it remoues all the three potlutants. There is a Darrow band of air-fuel r lfgasoline is changed to propane as engine fuel CO enission can substantially be re-
ratios near duced witb reduced HC and NQ ; and in changing from propane to methane the CO as
stoichiometric in which high conversion efliciencies for all
thee pollutants are available. commer- well HC emission touch zero level and only the NQ remains as a significant factor.
cial three'way catalysts contain jrratinum, rhod,ium wiih some
A, o, Nio and.c.or. et,r-in, i, ti" e From pollution point of ew, both methane and steam reformed hexne are very attrac-
preferred suppott material.
tive fuels but we are unable to use at present for want of technological progress.

1
INTERNAL COMBUSTION ENGINFJ
AIR POLLUTION FROM I.C. ENCINES AND TS
I8.4.4. Blow.by Control
CONTROL
,625
o The basc princple ofcrankcase brow-by contror systetn o Fig' 1'8'10 shows the arrangement ofexhaust gas recirculation (EGR)
system. A portion
is recircuration ofuopours bach to (about 10 to L?o) ofthe exhaust gases
the nlet manfold. i. l.t" to cylinder intake charge, and this
r Fig. 18.9 shows a positive crankcase "".i..'t
reduces the quantity ofO_, availaile for combustion.
The exhaust gas for recirculation is
taken (as shown in Fig. .10) through.r, Jti.u and passed
ventilation (PCV) system. Connectng
regulation
thri"et.*t"a valve for
hose pipe of the quantity of ecirculaltion.
The filteed air is drawn from the
air cleaner and passes on to the Closed oil Contol valve
Exhaust recirculaton line
crankcase. The air and th blow_ flter cap
by gases passes through flow cali- ^^^r,^l
;,'J';'
brated. pCV ualue before being
d,rawn into the intake manifold i-n
ord,er to restrict the air flow while
id.Iin.g when the vacuum is high at
the manifokl and, the blowby is FueFair --->
/ess. PCV valve is spring loaed. mxturE
At wide open throttle, the air flow Throttl yalve
gets unrestrjcted but flow iate is
Fig. 18.9. PCV sysiem.
meteed by tlre valve opening.
In view ofthe added browby flow, the carburetor
has to be caribrated. since brow-by
gas and air by-pass the cabuettor
in entering t" rnr"rol,lrr"l"ii,rre ofpcv will
give improper fuel ratios. Due to improper
seating of the pW .t frigt vacuum, Piston top
large quantity of air and also lubricating oil ".ir"
mist will enter the manifold. This will
read to lean mixtures. AIso, the valve geiiing
to u" to
to rich mixtures. "to.t -rrtu.i".nt , wilr Iead
Fig. 18.rO. EGR system
In case ofvalve failure, in this system, blow-by is
unable to go to atmosohere. o The effect of A.F ratio of Ne emis_
18.4.5. Evaporation Emission Control sion taking EGR parameter is-shown
Device (DECD)
In one of generar moto_rs system the filter pipe is
built iu vacuum eliefto allow air-to enter as tiulig.t. sealed with a pressure cap. It comprises
in Fig. 18.il. t
I
gooo

consumed. An invertei bowr is praced


a Itmay be obsened from the fig_
inside the tank' It traps
air as tank is
filled t" d;i;r'tiquid fuel volume expansion. ure that, maximum emission of d. 2000
released from the bowl bv two small ori.fices The air gets Ne takes place during lean rnix_ cf
in the top. The tank is ventila,r;ed to a cannister
has activated carbon pariicres. This cari rr"i
o ig Jiirer vapour upon shut down. But when
which ture lirnits when gas > 1000
engine is running, filteed air is. drawn th.o"gh the recirculation is least effecti-ve.
vapour' This mixture is sent to ajr cleaner
e l;;tom of cannister, removing the absorbed
or to'th" i"i"t u Whereas, for emission of hydro_
in proportion to
-"oirold, air flow rate. carbon (HC) and carbon monox-
18.4.6.Control of Oxides of Nitrogen (NO*) ide (CO) lean mixture is pre_ 600
The concentration of oxides of nitrogen in the
exhaust is closely related to the peak cycle ferred, 15 percent recycling re_ + I HC
temperature. The following are-the three mJthoa. duces Neby g0 percent but in_ :E 400
perature <i"u".ilg"t"d so far) for reducing peak cycle tem_
and thereby reducingNO emission. creases HC and CO by 50 to g07o. c!
1. Exhaust gas recirculation (EGR) These are two conflicting re- 1 200
2. Catalyst quirements of this emission-con_ s,
3. Water injection.
trol system and this problem has o
been solved by adopting package
1, Exhaust gas recirculation (EGR) : 6
system which have both NO- and A
o This method is *:!onlt.used' to reduce No"in petrol
as rvet as dieser engines. In s.I
HC/CO control devices. I

y4
engines, about 10 percent recirculation redules
q .been 2. Catalyst.A few types ofcatalysts have
nately, the consequently poorer combustion directly "-i.rion ;; H;;;*. unforru- tested to reduce the emis"jon ofNOr; a cop- o-
mixture enrichment to
increases HC emission and calls for
per catalyst has been used in the presene e
combustion regularity which gives a further ofCO
-restoe indirect in- ^i
creasc ofboth HC and CO. for this purpose. The research is
loing on to de_ o^
velop a good catalyst. 12 14 16 18
Air-fuel ratio --t
18.11. llffect ofrecycling ofgas on
NO. concentration.
:

--___t
NTMNAL COMBUSTION ENGTNES

AR POLLUTION FROM I.C. ENCINES AND ITS CONTROL ot


3. Water injection. It has been observed that the specic firel consumption decreases a few
percent at medium water iqjection rate. Attempts have been made to use water as a device for In this package emission of NQ, IfC and CO are reduced to a required level but at
controlling the NO,. This method, because ofits complxity, is rarely used. the cost of20 per cent less power and 10 per cent more fuel consumption.
This converter can be employed for a un of 1.5 lacs km.
18.4,7. Total Emission Control Packagee
2. Catalytic converter package :
Earlier (previous articles) we have seen that any meod whicb is used to decrease NO,
tries to increase HC and CO and ce.vesa. Thus it is of paramount importa4ce to develop a The,worhing principle of this pachqe is to control th emissinn levels of uarious pollutants by
methosystem which should reduce emissions of NQ, HC, CO to a desired level simultaneously. changing the ch.erncal characteristics ofthe erhoustgcses. The catalytic converter package as com-
After a long and detailed experirnental study of various possible systems, the following two pared to f,hermal reactor package requires non-leaded fuel as leail reduces the catalytic action.
systemdpackages have been developed to achieve the required resulte: 'Ilne mqjor ad.uantage of this converter (as compared to thermal reactor) is that it allows a
1. Thermal reactor package
partial decoupling ofemission control fom engine operation in that the convesion efficiencies for
HC and CO are very higb at normal exhaust temperatures.
2. Catalytic converter package.
o Fig. 18.13 shows the anangement of cotalytic converter package.
Using this approach, the following arethethrcebasic mzthod'sof emission control :
(i) Thermal reactors, which rely on homogeneous oxidation to control CO and HC ;
(il) Oxidation catalyst for CO and HC ;
(iii) Dual catalyst system (here a reduction catalyst for NQ and an oxidation catalyst for CO
and HC are connected in series).
Where control of NO, is required wih the first trvo methods, EGR is added to the
system.
1. Thermal reactor peckage :

t Athernal reacor is a chamber which is designed to provide adequete residence ime for
allowing appreciable oxidation of CO and HC to take place. For enhancing the conver-
sion ofCO to COo the exhaust temperature is increased by retarding the spark' Clean gas
to atmosphere
Prqgrammed Mufller
bypass valve

Fig, 18.13. Catalytic convertcr package.


Converters for HC aad CO andNQ ar arranged as shown in the figure. The NO,
Thermal catalyst is the frrst element in the gas flow path, does not cause release of any heat.
reactor The next is HC/CO caalyst, which rleases heat to such a great extent that may
cause overheating and burning ofthe element. This is taken care ofby injecting air
through secondary air pump.
Fig. 18.12. Themal reactor package (Fortl). A bypass valve ahead of converter is used to increase the converter life (to about
0.8lcas km).
Actual thernal reactm (made of high nickel eteel) that is used on a car consists of two For better control of NQ, erhaust gas is circulated via an intercooler back to air
enlarged exlraust manifolds which allow.greater reskteDee ti}e for burning HC and Co
cleaner,
ith xygen in the purrped in air. For keepfurg a flarne eonstautly borning (and there by
assuming complete corubustipn) a secondary ai punpinjeets freshir into the re-actor ; o For this system, the power loss is bout 307o ad. the fuel corisumption is about 107o
this redieq Hb and CO, .About 1,0 to ?5 percent ofthe gas ie recirculated after cooling in moe than normal,
the intercooler to reduce the formation of NOt
I8.5. DMSEL ENGINE EMISSIONS
In this packing system are also included the following :
(i) Enriched and. stage carburettor temperature crntrols ; o Emissions forn diesel engines can be classified in the same categories as those for the
(ii) Crankcase valve to conprol blow-by gases; gasone engines but the level ofemission in these categories vary considerably. Typical
(rdl) Special evaporation control valves. level, ofthe constituents ofthe exhaust products ofcornbustion in 4-stroke cycie and
2-stroke cycle are given in Table 18.1 at idling, accelerating, partial load and full load.

ili
iil
t
rtrf
lf
L__

628
INTERNAL CMBUSTION ENGINES AIR POLL{JTION FROM I.C. ENCINES AND ITS CONTROL
629
Tble 18.1 18.6. DIESEL SMOXE AND CONTROL
Concentration ae. measured in erhauat products 18.6.1. Exhaust Soke

chambeinwhich thefuetis burnr ar a retatively


|f,H,?:m':i"J,"1"i":lr.Yombustion
F/A ratio greater rhan 1.5 (FR-> r.sr ut p"*"""".-;;;i:;il:J:4fi:,iT'""*T1?:
quantity of soot formed depens or\ the following/ocors
fllowins ".i"" .'
Ttuoetrohe cAcle enline (i) the local F/A ratios
:

r. co% ;
(ii) The type offuel;
2.COr%
(iil) The pressure.
3. HC ppm
NQppm to,nnf. adequate. (o2) which <in the whole is much
4.
3j*::*^':^llt1 !, in excess or rhe
5. RCH ppm
;:lTfiff*ill:'. i,*l;:i'":'"'31i:,':':::::!:i:/!"1iii\ ;i#;';Hi:tr("r H:
I 6, Smoke Hartrige Unit ::iljr':1":#.1?:r^:,llf:.t*haust..Andirt""",L,
called smoke. The colour of th" smok" aepen
x sufficient, it witr beuisibler,",iJ)i,,,i,X
'"'l
I
;;;;;;;" ,*:r,;;;;:;:;
il
Four- stmke eXcle engine - l"*:i:::::ll! l1!1"t.."|y .
p-"",, oi";;-,;;;;res
"f
orhydrocarbon ruers
:?'J':Tl:'j:::i.ll1t-':-o::'gi";1eag,"i*J;;;;;;;;#ff d;i;#'#"-":ffi';
(Un-supercharged)
ii
ii fi, 7.CO%
0.08 ;#:X,?:::*P,?";'::-'i'"1"11",:.lu.l'"i'.i;i'"r.;;;"*ilffi :''*#ilT:T'i:;
p.*",,-ii;;;;1":fi 'dl:::ffi
l/,
il
ll
2. COrvo
3. HC ppm
3.5 l*:*::f :,"-T1".
consumed during Tl111.?,,",ti""
later part ofcombustion
E6sD ::i:illi:
uu! rs rL
330 Pyrolysis offuel molecules themselves is
I 4. NQ ppm
920 - Fue[ heated with.insuflicient o, will giu.
thought to be responsible for soot formation.
5. RCHO ppm
t. deposits. It is be]ieved that the
"heavy ends" ofdieselfuel may ";irorru."ous
fiyrolyze to yield the type ofsmoke that is observed from
l
6. Smoke, Hartrige Unit
44 the diesel engine. This is ueevea t" "-t* path
of formation of polycyclic aromatic
hydrocarbons (benzo-pyrene) found in soot.
o As shown in Fig' 18'14,.it is wor'h noting that No-
concentration in unsupercharged Many theories have been put forward for
4-stroke-cycle diesel engi.n-es vary linearly-with powl" (output).
The design *d op"."ti
- leading to the formation olsmoke uru .rot
the formation of smoke but the basic reactions
ing parameters to contor No, are the same as for s.I. ir.lty t .ro*.r.
engines
"l"uuyJi.*r"a. o The smoke ofa diesel engine is, in general,
oftwo basic types :
1.4 G) Blue-white smohe' It is caused. by
liquid. droplets of lubricating oil or
starting from cold, fuet ot uhile
owing to low lower.surrounding temperatures
rq the combustion products are at a rela-
tively low temperature and intermeiiaie p.odrr"ts
1.2 of combustion d.o not burn. .rhis
results in bruish white smoke when exhausted.
This type of smoke is also formed
when lubricating oil flows past piston rines.
t I
(ii) Blach sn ohe:
d 0.8 o It consists of carbon particles suspended in
the exhaust gas and depends largely upon A,/F
z
It increases rapidry with the increase in load and available
air is depleted.
0.4 18.6.2. Causes of Smoke

- It is known that the cas of smohe is incomplete


ar. The two major reasons burning offuel insde the combustion chant-
for rncomplete combustion are :
(l) Incorrect A,F ratio :
50 100 150 200 (li) Improper mixing.
Power, kW ----) These might result due to the design factors
discussed below :

Fig. 18.14 1. Injection system:


The following injection characteristics
substant iary incrcase the smore leuers.
o Unsuitable droplet size.
o fn:rircquate or excess penetration.
63r
INTERNAL COMBUSTION ENGINES AIR POLLUTION FROM I,C, ENGINES AND ITS CONTROL
30
In ttris meter, a measured volume of exhaust gas is awn through a
filter paper which is
o Excessive duration ofiqiecon. ta.t"rrea to u"rious degrees depending uponthe carbon present in the exhaust
g_as. The

r Secondary injection. l"iity of ""*t ' ismeaslredby timing out omount of light reflectedfrorn tlrc sooted, paper.
o Improperdisperaionatomisation. The specifications ofsample volume, filter paper size etc are well
dehned'
Rating:
2. 2. trIatridge smoke meter:
o It has been obsened that snoke limited power is reached much before the thermal load r It works on the principle ofhigh extinction.
Iimited power ; and the iuetability of smoke i exhaust is inherent in lese loads. Thus o is passed through tubes of about 0.46 m length which has
ii"""-.', smo is unavoidable in Ci engine, oaly its level can be kept as low as possible. - In this meter exhaust samplephotoce or solar cell. at the other end. The amount of light
ii-Sht at one ed and
"o"r"u oflevel ofsmoke.
3. Load: fissed through this smoke column i8 used as indication
o Smoke may be defined as visible products of com- oThistypeofmeterisusefulforcontinuoustestingandcanbeemployeilinvehicles.
,}
bustion. Rich mixtures (higher loads) results in I
18.6.4. Control of Smoke
higher smoke because of non-auaability of ory' has to run at
o There is hardly any successful method to control the soot except t]ne engine
gen for combuston. E Iower loadi.e. dc.oog and mointd,in th engine ot best possible condition.
o The smoke level rises from no-load to full-load (Re- U)
Th othr method,s which may be used for the control of emohe are :
fer Fig. 18.15). Duriug the first part (noload, say'
1. Smoke suPPress out edditives:
about half load), the smohe level is more or lesg the tem-
constant, as there is always excess air present' . It has been observecl that some bgrium comPounals when added in fuel reducethat if the
o 50 100 pe"atore of combustion and avoiil tbe soot formation. It is further observed
However in higher load range there is an abrupt
Load (%) sootisformed,tbebariumcompoundsbreaktheminveryfineparticlesandreducethe
rise in smoke level due to less available oxygen' -> smoke.
Fig. 18.15
4. Fuel: r Theuseofariumsols,however,enhznt*sthedcpositlbrmatinntendenciesofengituand
o The white smoke pmduced in an engine depends upon the quality of fuel' red.uces the life of the fuel filter'
o Generally, more volatile fuels give less smoke than heevier fuels of similar cetane numbe' 2. Fumigation:
oTtrecetanenumberexercisesnoeffeconproductionofblacksmoke. o It is a metho d of introd.ucing small amount of fuel with the intahe manifold.Thisnitiates
5. Fuel-air ratio: pre<ombuian reatianstforean<t during the compression sfroke resulting in reduced
aldehydes re-
chemical delay (because tbg intermediate products such as peqodes 8nd
il;;; irprri*u oxJgen than original hydrocarbons. The shortening in chemiol
d"toy p"rio ttn rr-t whir;h is r*ponsble for,soot formaon'
ence of ercess orygen, tliesel engine 65 miring problem' "uib"ofabout 11"rvl"Hrg
r Fumigation rate to 15% gives tbe best smoke improvoment' '
Engine ype and aPetil:
6.
o The smoke levels at higher loads are higher in naturally aspirated engines 3. Catalytic nulllers:
turbocharged engines tause thelatter have adequat'e oxygen even at fuIl-loail' The use ofcatalytic mftlers, uulike petrol engine, are not very effective'
o At low as well as high speed the smoke is worse. o Much development is needed in suchdwices before they can be put to use'
7. Maintenance: 18.6.6. Dieeel Odour and Control
r the condition of the engine. Good maintenan
:The emoke levelg geatly depend upon
--riotet*." It has been observed through some experimen ts tbat thc products- of partia,l^o*i'dntion are the
affects the injection chaacteristics
a muet for lowr sioke leveis {th" main use of odour in diesel eziaust. This partial oxidation may be because of either very lean
the quantity oflubricating oil which pmses across the piston rings and thus urixtures su as during idling or due to quenching efrect'
significant efrect oa engiue tendency ib generate smoke).
T}ie foUowng fators a,ffect od.our productian :
18.6.8. Meiaeurnent of Smoke 1. Fuel-air ratio
T$o basic types of smoke metera for measuring smoke density are : 2. Engine oPeration node
(i) Filter darkening type. (Examples : Bosch smoke meter ;Vem Brand smoke meter)' 3, Engine tYPe
(ii) Lght extinction type. (Exagtples : Hartridge smoke meter ; UTAC srnoke meter). 4. Fuel consumPtion
The,Iight egrrcton metel rjal-be used fot contirunus neaaurements wbile tbe'filter type' 5. Odour suPPressant additives'
be used only under stea.dy state cotd'tians.
1. Bosch smoke meter: :"::tJ"::lfficturers chaim that odour aadirive compounds can reduce the intensitv of
have any effect on
It is a frlter dakening type of smoke meter. odour, bt it has beeu found in practice that these additives hardly
odour formation etc.

I
I
INTERNAL COMBUSTION
ENCINES 613
The control ofodours by AIR POLLUTION FROM I.C, ENGINES AND ITS CONTROL
mentalyrharafewoxidan;r,;d;l'i,1illi?iTilj;lj,l
mentatttrrJ J#j;::":l?]Il'-' Y'u"l oevelopment. Ir has been
The comparison between ECC and conventional CC is given in the chart below
r8.7. COMPARISON
OF GASOLINE AND DIESEL Conventional Erhaust
EMISSIONS S, No. Electronic Catalrtic Converter
Fis. 18.16 shows the comparison Catalytic Converter
tfemissions and odour
from gasoline and diesel
engines.
Used as a pre-engine fuel converter. Used as an exhaust convert.
Cleans the process of combustion. Cleans the exhaust gases.
Improves fuel pmperties for better combwtion, Does not improve fuel pr\perties.
Works with all fuels-diesel, leailed, unleaded Does ndt work with leaded petrol.
gasoline, CNG and LPG.
Deorbonizes the engine combustion chamber Does not decarbonize the combustiou chamber.
with use and kwps it clem.
o. Reduces engine noise and improves engine Does noi reduce engine noise or engine smooth'
smoothness, ness.
li n Enhances fuel economy, Does not enhmce fuel economY.
t'
8. Works on micowave electronie, pl*ma chem- \{orks on chemical oxidation and reduction
istry, particle acceleratr principle. principle,
il o No Nobel metal or chemiel is used. Fuel flows Noble metal substrates or chemicals are used.
.1
through open-ended pipe, also acting as w&ve-
ilil guide and particle accelerator.
Fig. lg.16. Comparison ofemissions
and odour f 10. Engine pick up is improved, No chmge in Engine pick uP.
I t1 Sme size for all engines, all fuels md cubic Different size for different engines cubic capac'
capacity. ity and difrerent fuels.
:,:ii',"J#;;'"ff*"11;"',Tx",1,x,,:'};:;"J#ffi;1H,#"p"oau.t.or 72. No waming up time required for cold start l9am up time required for cold start emission
' Hiu;:nfi"ill,:-f-*t "ogir,". r,";- "-.o.u*"t simirar emission pauern, arr emission sontrol. mntrol unless provided with electrical heating
dieser for the catalyst.
."",,"tr**"*:h#.f ft;,ill:;frlil}Tff
",""-*;r,",".*;,'.uo,**r...' 18.9. AIR POLLUIION FROM GAS TI,]RBINES AID ITS COT{TROL
. -.rnordertoaccountr"""rn",,";;::__-:".."*combustionisofrelativelvhomogeneous o The pollution frorn gas turbines is lo as cornpared to pollution from convention piston
ilgs:+i#:iT:'i"1,:HT:::"::r,:ffi::#l}""ffi if; il".:ffiTj#ffj3'n:,f,:j,:? engines, due to the fact tbat former operate at much leaner tnieture used in the latter,
and still further, lbe exhaust is d.luted. d,ue to the presence of excess oir.
Correction factor
= l|/(%COz+ %CO + o As compared to S.I. engines, a gas turbine exhaust about I to 20Vo HC' 1 to 10% CO and
'lfi
VoC).
30 to 807o NO,. Besides this exhaust also contains oxides of sulphur, aerosol , smoke
..,iil;ltri]?,T:f.'.','"tri;:f
amount ofCO and :I;:l"t-havS.verxr*recoinrheirexhaust particles and odourants.
paredtoperror*t"""il;;;;ff ""iiff ilAf ff ::liili:T;:::h:'"1,"jfl '"T*: Contrcl of pollutonts emonated. from gos turbines :
18.8. ZERO EMISSION
o Reduction of CO concentration can be accomplished by using following methods :

(i) By improving atomisation of fuel.


o For,,zero emssion (i) By redistributing the air flow to bring the primary zone equivalnce raio to an optimum
value ofaround 0.85
**Tr.'f #,".,.H:H{dfi::":i,fjt{iT::JJj:!ft ;,m:,,nff:nir (iid) By increasing the residence time.
unleaded gasoline. rur auromobile f":?"ffi
fuels u ; the diesel, leaded and (iu) By reducing the frlm cooling air.
' ",ff";;:::;;"1!:,!;,#!':;l:;,with microwaue etectronics
and does motecutar ensineer.
(u) By fuet staging (In this technique supply offuel is cut-off to some nozzles and diverted to
the remainder ; it reduces emissions at low power conditions by improving the quality of
o ECC can be instaied as
a p"e-enj.re device. automisation).
. ECC s the frst catalyr" Reduction of UBHC (unburnt hydrocarbon) requires the same treatment as em-
"""r""J i" ll* iu"t ,ortrg, due to improued. ployed for reducing CO.
combustion.
o Rerluction of NO,. The NO, emission increases exponentially as : NO" ct(e)o osr, where
T is reaction temperature in K.
AND ITS CONTROL 635
634 AIR POLLUTION FROM I.C. ENCINES
INTERNAL coMBusrroN ENctNEs

Thus' for reducine No" coacentation, itwilJ IIIGHLTGIITS


be required to lower the reaction tenperature
The reduction in both flame tempratue T.
and residence dle qe_"eadily accomplished by increasing which will have a delete-
the air flow in the primary zone .t i"."r*"
n" p"J"Jti"on 1. Air pouwtinn csnbe dfmed as m adtion to our atmosphere of any material
to reduce HC and CO inceases Ne aodvice-rer". ----' of HC ancl co. Thus any attempt rnade ious effect on life uPon our Plmet.
are :
The NQ level as we'as te q The major pollutans emitted from the exhaust due to incornplete combustion
- zone.
smoke revels can be reduced by prouiding ban combuston
CO, HC, NO, md other products e.5'. acetrlene, aldehydes etc'
3. CO is generally formed when mixtue is ricl in fuel'
18.10. EFFECTS OF ENGINE EITIISSIONS 4. NO formation will be less in rich mixtues than in lean mixtures'
ON ITUMAN IIEALTII as stoihbmtri mtio
T\e effects of d,iffercnt engiw emhsions on hutnan o. The chemical mrrect air-fuel ratio by nass for complete combustion is known
health are d,iscussed. belaw : engine ; thig is increased to
6. The blowby HC emissiom are about 20% ofthe total HC emission fom the
t. Sulphur dioxide (SOr): about 30lo if tle rings ar won
o It is an irritant gas and aflects the mucous membane
when inhaled.
7, The following factors affect the fomation of NQ :
In the presence o{ (ii) r.p.m- ; (iii) Angle of advance.
water vapour it forms sulphurous and sulphuric acids. These acids
cause severe
(l) A/F ratio;
bronchospasma at very low ievels ofcooc"nt"ition. q In a poorely maintained engine the exhawt pollution is more'
o educe HC mil co by
Diseases like bronchitis and astbarna are aggrevated 9. Lcdtalytic conwftzr is a device which is placed in the vehicle exhaust system to
by a high concentration of sor.
oxidising catalyst md NO by reducing tlysL
2. Carbon-monoide (CO) :
10. RhodiumisthebstcatalysttocontrolNQbutA./Tratiomustbewithrananowrmgeof14.6:1to
It has a strong affniy (2olqnes) for combining with the haemoglobin L4.7 : L.
of the blood to
f3r1arf oyhaemoglobin. This r"au"o nu oii''e r,"e-ogroir t"-.-.y t" 11. Thebasicprincipleofcrmkcaseblowbyontrolsystemisecirculationofvapoursbacktotheintake
the blood tissues. "r'rfr "ryg", manifold.
a CO affects the centml nrwus slsten. t2. Oxides of nitrogea can be coutrolled by :
a It is also responsible for heart attacks anrl (i) Exhaust gas recirculation; (;i) Catalyst; (iii) Water njectim'
a high mortality rate. levels of vaiou lrcllut-
3. Oxides of.nitrogen CNOJ : 13. The working principle oI mtatxtic conwrb pachage is to control the emission
'r ants by changing the chemical aracteristics ofthe exhaust gases'
a nese are
known to cause occupationar diseases. It is estimated
that eye and nasal irri-
tation will be observed after expooure to-about ls p.p.m.
of nitrogen r", .iJ p""- OB}ECTIIE TYPN QUESTIONS
nary discomfort after brieferposure to 25 p.p.m.
oinitrogen oxide.
a It also aggrevates diseages like bronetritjs and rii
asthama. Fiil in the gt-.s or Say {ee'or t()' :
4. Hydrocarbon valxrur: l. ...... are proiluced by incomplete burning of the air-fuel mixture in the combwtion
a!ilbr' ff
a They are primaly irritating, 2, ate : """' il
The m{or pollutmts emitted from the exhaust due to incomplete combustion
b water vapour
a They are mqior contributors to eye and respiratory
irritation caused by photochemical 3. If the combustion is complete, the only products being expelled from the exhaust would lli
smog. atl carbondi,oxide.
ill
5. Compoundc of Inconplete conbugion: 4. CO is a colourless and....,. gas. j;

Exhaust discharge fron I!.eyqes-carrr compounds


5. CO has about the same density as air.
organic compounds and ariphatic hydrocarbons),
ofincomplete combugtion (polycyclic 6. CO is generally formed when the mixtue is """ in fuel'
wtrcn ac ls ,o Z. SoUa particles are usually foraned by dehydrogenation, polymerisation aud agg-lomeration'
are responsible for lungs cancer. """"i.rof"ni"-uJu",
8, At heevy loails, ilue to lack of orygen, a decrease in aldehyde emissiou is observecl
6. Lead: 9. Tbe NO formation is more in ridr mixture than in lem mixtures'
Inorganic lead compounds (discharged from vehicres 10. Tl|e chemically correct air-fuel ratioby mass for complete combustion is known as ......
ratio'
using leaded petrol) cause a varidty
of huma health disorders. ll. Oxidesofnitrogeoproductsfonnedduringcombustionareverylowatl00p'p'rn'withrichair-fuel
o Th:.-:trecqs jnclude
gastrointestinal damage, liver and }ddney damage, ratio of 11.: 1i
fertility add prepancy abnormality in Ihe ca.nkcase, the
erc. 12. The three main souces frod whi pollutants are emitted fiom the s.I. engines are :

7. Smoke: fuel system and ttre .'.... systen-


It 13. Piston blow-by .'.... with engine speed.
is visible carbon particles.
14. Eva?ofative emisions acsoutrt for 15 to 25% oftotalliyocarbon emission fron a gEolie engine'
It causes iritatiou in eyes and lungs, and visibility reduction.
I also eauses other respi- 16. The two m'in swces of evporative emissions are : T}e tank and the """ '
ratory diseases.
18. Ttre emission ofHC in exhaust is decreased wit'h an inaease in compressio ratio' ii

,-"- , Gut":"lly-speaking,
ntants
susceptibility to the effects of exhaust emissions is greatest
and the elderly' Those with chronic diseases oflungs amongst 17. The percentage of CO, decreases during engine idling but increases rrith speed' rl

;li
or heart are thought to be at great risk.
636 .. INTERNAL COMBUSTION ENCNES
AIR POLLIITION FROM I.C. ENGINES AND tTS CONTROL
18. CO emissions
ue lowest during aaeleration and
at steady speeds.
19' oxidesofnitrogenocmrmainlyirteo"rnoro*uq"ou.*"rera'yformedathighertempera. 4. State the mechanism offormation ofCO.
; 5, How are hydrocarbons (HC) formed ?
20. The formation of NO- is affected
2l'
by the factor: AtT ratio, R.p.m.
; ......
6, Explain brielly the mechanism of formation of nitric oxide (NO).
NQ increases with iicreasing .

22. -.iifold pr"""ro,


Icad emissio* come only from ...... - .'' -"""'' lora compression ratio. 7, What are t.Le effects of the following factors on the exhawt emission ?
",,u "od (j) Ai fuel ratio ;
23' under high speed conditioro, u*t u*t"ogi""
ic "eotatiom ae row while te flow rates
(i) Surfae volume ratio :

24. In a poorly maintained engine tt u are high. . (id) Engine speed.


25.
.*f,uu"t poUoJ* ir-Io.". 8. Explain briefly the various sources from whih pollutants are emitted fiom S.I. engine.
Lowe compression ince":e: therrnal
26' efficiency *", cons.mption. 9. Defne tle crankcase blowby aud explain how it can be contollerl \
NQ emissions ae reduced due to loweriis-ofi*-rr"J--o*tion
"rO ""i""" 10. Disms briefly the following with regard to S.I. Engines:
temperatues.
bumer where (i) Crankcasg emission
'" fr;ff*lff:;:a "t"*T"* 'th1 exhaust gasee and mrue js bumt with the (ii) Evapoative emission
28"""' is the best etalyst (ijj) Exhaut emission-
to contror No but A / F ratio must be-within
nanow range of 14.6 : l1' What are t'le souces of HC fomation in petrol engine ? Explain
l to a diflerent factors which affect HC
'. ffio"Xli'Jn'1li"*:1""::T il mi{11 t"** ,i""fi","*,iomerri*ario is a rough indication or i,.
14.? : 1.
formation.
*";;;;:H;,J;trj_::1.ilffi I"H,;*f ff r*xj:*;il.;,",hein,ake 12. Explain briefly the factors which affect the formation ofNO .
13' what are the souces of HC formation in petrol engine ? Eiplain various factors which affect the
formation.
3f. NO, emission can be reduced )y : Exhawt gas
32. A thermal reactor is a cl recirculation ; catalyst ...... .
;
14' what are the sources of evaporaive emission in petrcr engines ? How cm it be contoiled
?
appreciable oxidati ;rt6Tlit$t"
is designed to provide adequate
residence rime for ailowing
16, What do you mean by crankcase blowby ? How can it be mntrolled.
16. Explain brieflyvarious nethods by which s.I. engine emission can be controlled.
"";['.i:Ht"ilil1if,il;;,lr,fiii,i"IrXf":l""emissionreversorvariouspo]ruranrsbychansing 17, Explainindetailanytwoofthefollowingmethodsofs.Lengineemissioncontrol:
34. The quantity ofsoot formecl d"p"od" (i) Modification in the engine clesign and oprating parameters.
upon u loll F/A '(ii) Treatment
of exhaust products
::''mokui"cuo"edbvrr:l
3u. ....,, smoke consists
""
g"*:";r,"il;;;,i;H:i:1'"ffi::*","H;'*"":l;:**
__--s rq!uE
of carbon particles .*puoa"a
(iii) Modilication of the tuels.
of combustion.

Ewlrenoed t-ii"
tD the exhaust gas and depends
ratio. largely upon A,/tr'
18' Describe with sketches the foilowing methods of petrol
exhaust emission control :
37. I'he cause of smoke is incomplete burning (i) A{ter-bumer (i) Exhaust manifold reactor
of fuel inside tle combustion
38. ...... nay be rtefmed as sible p."dr"t -" " amber. (iii) Qsyti. converter system.
;";;;;;;.
"f 19. Explain brieflythe exhaust gas recirculation (EGR) device for the
li. il"".1iliT:j}irl"il:**:*:r"htJ;;;;*useorno'ava'ab'itvoroxvgenrorcombusrion. 20' What do you mean by "Total emission control packages"
control ofNe.
? Describe with neat skeiches two tJpes of total
emission control packages.

t. Pollutants 2. CO, HC, Nq


l-*.'l
3. Yes
21. Explain brieflythe following :
(i) Three-way cata\,tic converter
(jii) Noble metal catal,tic converter.
(ii) TVo_way catal,.tic converter

6. rich 4. poisonors 6, Yes 22. Explain briefly the ,,evaporation emission control dece (EECD)..
7. Yes
ll. Yes 12. exhaust
8. No
13. increases
9. No 10. stoichiometric 23, Discuss the emissions fom diesel engines. On what factors
these emissions depend ?
16. Yes 14. Yes 15. carburettor 24, Define'smoke'and discuss different factos which affect smoke
17. No 18. Yeg fomation in c.I- cngines.
20. angle ofadvance 21. yes 19. Yes
26. what is the cause ofdieser smoke ? Discuss the ways by which dieser
22. S.L 23. Ye smoke can be controlled
26. No 26. Yes 24. Yes 26. What is the mechanism of smoke formation ?
27.Yes 28. Rhodium
30. Yes 31. Water injection 32. Yes 29. Yes 27. How can the snoke intensity be measured ?
33. Catalytic convertel 84. Yes I
35, Black-white 86. Black 87. Yes
I 28. Describe two important t)?es of smoke metes.
40. increases. 38. Smoke 39, Yes I
f
29, Explain the effect of engine load on diesel engine smoke.
30. what is diesel odour ? Explain the effect ofdifferet factors on the
{ formation ofodour in c.L engincs.
31. How odour can be controlled
ti :rrEoRETrcALQUESTroNs ?
i
L I
32. Compare the diesel engine and gasoline engine
emissions.
t. What do you mean by,,Air poll"- 33. Discuss the air pollution from gas turbines and
compare its with emissions from petrol engines.
, What are the main sources ofpollutants 34. Give the comparison between "electronic catalytic
converter" and "conventional exhaust catalnrc co*
from gasoline./petrol engine verter".
What ae the main pollutants emitted
by petJ
"l ?
35. Discuss the effects ofemissions on human
"nsio;-r--'". health

-
639
MISCELLANEOUS ENGINES

intake manifold where it mixes with the incoming air to form a homogeneous mixture. In

19 2-stroke cycle engines the gas may be injected at low pressures, from 1.5 to 3.5 bar,
directly into the combustion chamber after the closure of the exhaust ports). It is
interesting to note that in the duel fuel engine, the combustion starts similar to C.I.
Misc ellarr.eous Engine s engine but it propagatcs by flame similar to S.I. engine.
Owrspesd
To automatc gas
trip lever
19.1. Dual-fuel md multi-fuel engines"-Dual-fuel engine*Multi'fuel engines. 19.2. Stratied Gas header shut otf valve
charge engin*Introduction-Claseication-Advatages and disadvantages of stratefied Fuel oil
charge engines. 19.3. Stirling engine--Stirng cycl*Working principle of stirng engin* manifold
Overspeed and
Gas low fuel oil
Differences beta'een 6Dot and stirling engines-Engine geometry and iving medranism.
governing pressura shut
19.4. The Wmkel rotary combustion (RC) engin*Introduction-Construction and working- otf cylinder
valve
Features--Constructioual and other detils of wankel engin*Performance of wankel engine-
Conncton cam
Advantage md applietions of rotary combwon engine-Why wankel rotary engine could
to govrnof To gas
not become sueessful. 19.5. Vile ompruion ntioy'VCR engines-Introduction-Meods
govsrnng
to obtain variable compression ratic-Analysis ofVCR engine-Performance of VCB engine.
valvo
19.6. Fre-piston engine plant--Higbligbts-Objective'Ilpe Questions-Theoretical Questioru. admsson
valvg Govlnor

19.1. DUAL.FUEL AND MI'LTI-FI'EL ENGINES


Dual-Fuel Engines
19.1.1.
Fig. 19.1. Dual'fuel engine.
Inroduction
19.1.1.1.
Owing to various tenical d nancial reasons, some engines are designed to ope,rate, The engine output (power ) is generally controlled by alterng the o,mount of primary
using a cornbinntinn of two fuels, For instance, in some third-world countries d.ual.fuel gaseous fuel aded to inlet manifold (the pilot oil quantity is usually kept constant).
engines ore used. becduse of high cost of diesel fuel' This type of engine is capoble of runnng on either gas or diesel oil or a combination of
Large C.L engines are run on a combination of methane and d,iesel oil. Methane is the these two ouer a wid'e range of temperature rotios.
main fuel because it is more cheaply available. However, methane is not a good C.I. fuel Fig. 19.2 shows ttrep-o diagram for a duel-fuel engine working on different fuels
(change
by itself becaus e it does not read.ily self-ignte (due to its high octane number). A small from one fuel to another being quite smooth).
amount of desel oil is injectzd at the proper cycle time. This igntes in a twrma.l tnonner
and intiates combustion n the mthane-oir mi*ture ftlling the cylinder. 000 70
1
I
a On these trye of engines, cornb ination of fuel input system are need.ed,.
a Duel-fuel operoton combins n a simple rrLctnner the possbilty of operatng a diesel en' +
1
t 60
gine on lquid, fuels such as diesel oil or gas oil and, on goseous fuels such'as natural gas, I 8oo
sewoge gas ond cah ouen gas etc. -o I ;\
The engine can be switched from dual-fuel operation almost instantaneously in case I -Die el
of emergenclt. g ooo
x. 40
, t\ti
19.1.1.2. Working of Dual-fuel Egine E
:v \
Fig. 19.1 shows a diesel engine with necessary modifrcations, which it requires when it pors 30
on duel-fuel. c
400
\ ilot
The working ofa dual-fuel engine, which woks on a diesel cycle, is given below : I \ ( 20

The mixturdof air and gaseous fuel is compressed in the cylinder similar to air compression zoo
in diesel engine. \ \\, ln

A small amount ofliquid fuel, (disel fuel), calledpilot fuel or t}:.e secondary fuel (5 to 1vo \ <\
of the fuel at full loail) is injected near T.D'C., irrespective of load on he engine. This
orllllll_+-Jo
pilot injection acts as a source of ignitian. The gas-air mixture in the vicinity of injected
60 40 20 T.D.C. 20 40 60 80 100
0 (Crank angle, degrees) r->
spray ignites at number ofplaces establishing a number offlame-front and combustion
continues rapidly aod smoothly. (In a 4-stroke cycle engine the gas is supplied in the Fig. 19.2. ppical p-0 diagram for a dual-fuel engine.

638
ill 640
. INTERNAL COMBUSTON ENGINES
MISCELLANEOUS ENCINES
rit' 19,1,1.8.Factors affecting combustion in a dual-fuel o4l
engine :
': The following factors affect combustion
in duel_fuel engine : The hnoch tendency of the engine can be red.uced, by supplying excess ar wth lou
1. Pilot fuel quantity 2. Injection timing initial temperature and. by using the ontihnoch dddu;(e.i. tJtramethyr lead).
ilii
1,,
3. Cetane number ofpilot-fuel C tntet ternperat,rre 19.1.1.4, Performance of dual-fuel engines
5. Tlpe ofgaseous fuels
rriil
6' Throttliag o Fig. 19.4 shows the relative performance ofdual-fuel engines.
T. Mixtue strength.
Theeffects of these factors on combustinnin
ii.: a duel-fuel engine are briefly discussed
below :
+.
I

iIt
I
ii):J#l,Z:::':y.)*::::::!:ofpitotfr,etresu,lii,_"i'i,],",i,*,,*,,1"'",
!#::#;:;i#J"::::;"':!"yj":"i:::i:y:y;;;;;;:::';ffir'::,xxi:;#r;T:"i
; :":;;#ffi;: ::::;::i:'::::f:* {9ro
':i;? fuel
pilot ;:,Y:Yl'
as used by unsupercharled
j ::l:.";i
lmx*::l
"l= a""l-t i;il;
"i"rgir".
?
'rjl o,w
h'r;:I:;,y':f;.7:""":::l!:(:::?{'o:.*:oilltimingisnof uery hishexceptrhat at

7i',1"::::"'#;f:,?::::,1!'::::^"-ii,o,"^"o;;;;;ffi;;|^r'uL'i|'J"[i'i":il::i,!::Tj
l

increases making the engine mnre near 0 rn


to hrwckng conditian. o rv
3. Centane number of pilot fueI: E
' I[:rf:rffl:2;:::i::"^u,r fuets resutts in poor performance of the ensne and,greatty s
o The ignition quarity.offu.el, in general,
I
exercises ritre effect on combustion a
engines as compared to the ignition q"riity in duel-fuel a
of tfru pmary fuel. 20 40 60
4' Inlet tenperat ure. The temperature 80 1oo
- of inlet charge greatty affects potLer output,
,gnition Load (% age)
,! --t
',^,,", ;:,:.1::::::":,:: ilo *rr;" *" n""nili l)its of a particutar fuet.air rnLture.
Fig. 19.4. Comparison ofperformance ofdual-fuel engine wih other engines.
tt

ir3m#,:;T":i^'::H;*:^yJ:,ir*.,::.r*r,.Ji""a",;ffi ;"ffi lii,.,,"o At paft load conditons, in case of duer-fuer engine, the therrnar efficiency is row
;::iliJ::1i,11"i;':""l""lT"li;*:3-*"*l::n4il';,,#;"'j:[:'#T"H;f:iil:1 (38.5vo against 409o of desel engine)
I EH##H*11::?T::',T,1',:l,lg..:*::l*;;;;;":;;;;;";il:;,;,::;:X*:::::::::::::::::Z:T:n
firing from cylind.er nti s"' / air itii ;;;;;,;;;;;;';W.';:;::;::"f5. and. specifi; fu;I consumption ig be-carse of
increas-ed delay periods at weak mixture oi air'and gas. This ieqtftes-either ad,just-
when throttling is used in iuel-fu ei uit marimum ment of mixture strength or injecting more pilot
grrotbl'Throttling' cylind,er pressure red,uces fuel at part loads.
At fuII load, duel-fuel engine s superior to d.iesel engine since the gaseous
T Mixture strength. with the increase in - fuel fills all
ports of the combustion chamber and allows more air to tahe pari in tlrc combustion.
mixture strength the ignition delay reduces
then again rises sharply A futther i*.""*"i"'rtr" rap-
iff,ill mixture strength leads to failure of 19.1.1.6. Advantages and applications of dual-fuel engines
r Fig 19'3 shows thep-D diagrams for normal
burning and knocking for duer-fuel
Advantages:
engines. 1. A duel-fuel engine can run on either ofthe fuel and diesel requirement is hardly 5 percent
ifit runs on gas.
to i 2. Reduced pollution.
o It is preferred when cheap gas is easily available.
J
4. owing to clean combustion in the engine the wea and tear ofthe engine as well as con-
I E sumption oflubricating oil are educed.
5' The e-ngine's utility is considerably increased due to the possibility ofinstantaneous change
50 -40 -30 -20 _10 10 20 30 40 50 over from gas to diesel and vice-versa.
-50 -40 -30 _20 _10 l0 20 30 40 50
T.D.C. 6. Best suited for low pressure liquified gas (LpG) which evaporates very easily.
T.D.C.
(o) Non-knocking 7' Very attractive power generation system because of its greater flexibility of operation
() Knocking
compared wilh conventional diesel engines.
Fig. 19.8. Tpical p_9 diagrams for
a dual_fuel engine.
Applications;
- lT,:::.tX T'"'*:tffi.:::E-e111sio1ar:.hieh 1d,'l^ilar to s'I' engine knock- it
t 1 A typical use ofduel-fuel engine is to produce synthetic gas (mixture of co and Hr) by
nio"ni,s inii;;;;;:i:;:;! fi;;y.' r"' uevond a timit can ,ou," o-,nry ,",",," t
t
burning CH, and simultaneously deveping porier.
2. A lot ofconventional fuel can be saveal by using duel-fuel engines particularly for
rrriga-
tion purposes.
643
INTERNAL COMBUSTION ENCNES MISCELLANEOUS ENGNES

19.1,2. Multi-fuel Errgines _Theengineoutputislsto20percentlesswithpetrolasfuelcomparedwithdieseloilat


L9,1,2.L. Introducion hgher speed'. The difference is less in lower speeds'
is- also higher at Iower
A multi-fuel engine is one uhich can operate satisfa.ctorily (wth substantally unchanged, The b.s.f.c. (brake specific fuel consumption) for petrol engine
performance and efficiencl onawdzvaricty of fuels ranging from diesel oil, crude o1, IP-4 to lighter
- .o-pu."a wit diesel fuel' The difference narows down at higher speeds'
"f""a
fuel lihe gasoline, and euen tnrmal lubricating oil. The use of low ignition quality fuels require
higher compression ratio for burning, therefore, diesel engines are prefered to use multi-fuels in the 19.2. STRATI}'TED CIIARGE ENGINE
englne. 19'2.1. Introduction
(stratiFred diesel en-
19.1.2.2. Requirsments of a multi-fuel engine o The straified charge engine is usually d'efined' as a S'I' engne
plug-is verl rnuch
gine has also been leelop) in which th mxture in the zone of spark
The following requirements must be met with by amulti-fuel engine : ,"it the co bustion chomber . 6ne whih burns leaner ouerall
richer than that in the of
1. Good combustion efirciency anil minimum heat losses fr.om the engine.
fuel'oir mixtures.
2. In view oflow ignition quality ofpetrol, the temperature ofthe combustion chamber should Chargestratificatior:naed'nsprod.i'ngd'fferentfwl.airmi.xturestrengthsatuari.
be comparatively higher.
ous places in the cornbustion chanxber'
3. The engine must le able to start under sub-zero conditions without any external aid. Whereas severl S.I' engines are designed to have a homogeneous
ai-fuel mixture
a engines are designed
4. The engine must bave low exhaust smoke and low noise levels. throughout the comb..rsuoln ch.mber, sorie modern stratified charge
In order to meet with the above requirements, the following design features of the engine d.ifferent air-fuel ratio at ifferent locatbns within the combustion chamber. A
tohave a
the-mo.jor uolume
need be considered : rich misture thot enitis readily is d.eired. around the sparh plug, while
giues good' fuel economl '
(l) High compression ratio of the combustion hamber i" lled, with a very lean misture that
(ll) Large stroke / bore ratio pecial intoke o," ni,""ory to supply'this !o";ho.:oc:":,o:s 7i'xture combina'
"yrti'"^" volue and njection
(lli) ton of muttpte values and, multipli fuel iiieAors, atongwith flerible
Open combustion chamber accotnplish the d'esired results'
timing are used to
(iu) Injection pump.
SonestratifiedchargeS.I.enginesareoperatedwithnothrottle,whichraisesthevolu-
I 9, 1.2,3. Difficulties associeted with multi-fuel operetion metric efficiency. speil is controlled by proper timing and
quantity of fuel input.
aduantqes ofboth pgtrol engines (very good
The multi-fuel operaion entails the following dfficulties : o The stratified. charge engine combines the
and flexibil-
1. Tendency of vapour lock in the fuel pump while using lighter fuels. full load power ch.ia"teiistics e.g. high degree ofair utilisation, high speed
iti) and eset engizes (gooil part--loarl characteristics) and o e same time auoid.s as far
2. Tendency of increased wear in the fuel pump due to lower lubricity of gasoline. ii
as Possble their disad'uantages'
3. In view of differences in heating values and compressibility of fuels, different volumes (of :
The following ar e lhe ad'uontages of burning overall fuel-air mi'xtures tl
fuels) need be injected. I
(i) Higher thermodynamic efliciency' tl
19.1.2.4. Performance of multi-frrel englne (i) Reduced air Pollution. f,
l
. Fig. 19.5 shows the performance of multi-fuel engine. 19.2.2. Classifrcation
lt
iI

According to the method of formation of the heterogeneous mixtule


in the combustion cham- ril
1l

ber, the stratified charge engines can be classified as follows :


t:
,i

+ l.Thosesingfuelinjeetionandpositiveignion(includ.ingswirlstratifiedcharge
I
I engines)
f1s (d) Stratfr.ation by fuel injection and' postive ignition :
t
I
3
I 1. Ricardo sYstem
o
to 2, Pre-chamber stratified charge engine
d 3. Volkswagon PCI stratifieil charge engine
iui q 4. Boilerson method of stratification.
I
(b) Swirl stratfi.ed. charge engine :
1. Witzby swirl stratification process
2. Texaco combustion process (TCP)
3. Ford combustion Process (FCP)
Speed, r.p.m. --+ 4. Ford PROCO
5. Deutz combustion process (AD-process).
Fis. 19.5
INTERNAL COMBUSTION ENGTNES
MISCELLANEOUS ENCINES 6T5
IL Stratification by carburetian alone
1. Russian stratified charge concept, (iil) The inherent knock resistance ofTCP allows the use ofhigher compression ratio or
2. Institute Francias Du petrols (IFp) p"o"er.. turbocharging without imposing any octane requirement.
(iu) The lean overall mixture can be used.
3. Honda CVCC (Compound Vertex Contolled
Conibustion) engine. (u) Low exhaust smoke,
Here we shall discuss in detail only Texaco
combustion process (TCp). o TCP system has proved very versatile as regards multi-fuel requirements and are used
Texaco combustion process : in military uehicle engines.
s'I' engine has several problems ad areas
of potential weaknesses, these 19.2.3, Advantages and Disadvantages of Stratified Charge Enghes
,." *r"3;""rXfuretted
(j) The end charge has a- long residence The advantages and disadvantages ofstratified charged engines are listed below
time. It may, become highry reactive, and :
cause lurock. Therefoe a r"er or r,igtr-oci;i-;;is:;."q.i.". it may Advantages:
(ll) Homogeneous mixtures with the 1. Compact, lightweight design and good fuel economy.
fuer-aiilimits of propagating a flame
enthalpy effrciencies and also relativAy
frlgf, iC
reld relatively low
2. Good part-load efficiency.
(ill) Throtiling as a means of controlling ""T. umirsio.rr.
outrut, induces a pumping Ioss. 3. Exhibit multi-fuel capability.
(iu) Flame quenching at the
walls adds to air pollution. 4. The rich mixture near the spark plug and lean mixture near the piston surface prouicles
weaknesses (and hopefullv ithout cushioning to the explosive combustion.
". ""TtJili"t#ithese adding others) strarified charged engine
5- Resist the hnoching and provide smooth combustion resulting in smooth and quiet engine
' H;Jffi:*T*$:TiffiTi,f$?" ilrushated in Fig. 1e.6. rhis sysrem was par- operation over the entire speed and load range.
6. Low level of exhaust emissions ; Ne is reduced considerably.
7. Usually no so rtiig problem.
8. Can tolerate wide quality of fuels.
9. Can be manufactured by the existing technologr.
Spark plug
Disadvantages:
1, For a given engine size, charge stratification results in red,uced, power,
2. These engines create igh noise leuel at low load conditions,
3. More complex design to supply rich and lean mixture, and quantity is varied with loarl on
the engine.
4. Higher weighl than that ofa conventional engine.
5. Unthrottled stratified charge engines emit high percentage ofHC (d:ue to either incom-
plete combustion oflean charge o occasional misre ofthe charge at part load conditions.
6. Reliability is yet to be well established.
7. Higher manufacturing cost,

19.3. STIRLING ENGINE


producls

Fig. 19.6. Barbefs Texaco corrbustion process. 19.3.1. Stirling Cycle. please efeAt 3.11. .'

In this system, the fuet spyay The stirling cycle is superior to the carnot cycle because of the
at the end of the compression strohe following reasons :
sparh plug which is entrainidls.injected,
by iwrl
toward, 1. The Stirling cycle ispracticable, whereas the Co rnot qrcle cannot be realised, n practicc
duc
near sparh plug. ";, ""ifJ, armost stochionetrc mixture to wide variation in speed during a cycle (alternately very high for.adiabatic
and very slow for thc
The charge from the injected region isothermal part ofthe cycle).
starts burning succes_suer1t but the
Iocated' between the spark ptug" flame front
tl* """iilemans armost stationary. _ 2. The workoutput per cycle and rn .e.p. of Stirling cycle are hgh. The Carnot cycle need.s a uerl
. This system entails the followingadvantages long strohe and, prod.uces d uery nanow strip of worh giuing a lou m.e.p.
:
(j) Gives good performance over,vhole
range ofroad 19.3.2. Working Principle of Stirling Engine
. ranging from premiurn gasoline to frig.h;etaie and speed with a wide range offuers
(jj) Besides giving better part load
lilsel fuel_ The basic principlc ofworking ofstirling engine is thc same as that ofconventional cgine.
ef'ciency the starting and rvarm-up characteristics ,
of t he alternate compression at low temperature and. expansion at high temperature of a worhing
TCp are very good aue to employm"rt.ir."p flu.icl
crramber.
"ilriustion

* -... .!i.i ,
INTERNAL COMBUSTION MISCELLANEOUS ENGINES
is the bas-is-for the stirling e.nTine. However, the working
fluid is heated in a radically different 19.3.4.1. Two piston mechanism
manne. It burns fuel outside the engine itself, and
continuously.
The following featutes distingujs[ the stirling engine from Fig. 19.7 shows the arrangement of the system. The whole mechanism can be divided into
other heat engines :
1' The ai and fuel are externally,burned, and, the heat generated two systems :
is transferred to the work- (i) The displqcer system (ii) ltre power system.
ing fluid (may be air or any other suitaLle gas) of stirling ;
engin-e where the worh,ing fliia worhs n
closed cycle. The functon of thed.isplacer systen is to heat and. cool the worhing fluid.
2 The cvclic flow of working fluid within the engine is achieved solely through geo -
The functnn of the power syetem is to obtain the power dvring erpansion and, supply
volumechanges,andwithouttheuseofintermittentrycr'osedvalvesorports. ' --l - p o wer d,uring c ornpr es sio n,
i

ii
3' An intermittent flow heat exchanger stores a large portion
ofthe heat ofthe working fluid
returns to th. *o"ki.,g fluid after compression, thereby accom- lii
:"1:i,:10.1.^:t:1,1d_ubseg.uently
plishing thermal regeneration.
Fuel used. for stirling engine :
Since the stirling engine is an external combustion engne, T T
it possesses n ulti-fuel capacity, It Expansion Compression
can use any petroleum fractior such asgosoline,
diesel, methanol-jasotine brend,s,Ltc., with no oc_ space (V") space (Vc)
tane or cetd'ne requirements. Ttrus the stiiling engine
h isthedesirlbte characteris;ic ofadaptabitity
to changing fuel auailability. t t
19,3.3. Differences beween Carnot and Stirling Engines
The differences behpeen Carnot and Stirling engines are given
below :

Fig. 19.7. TVo piston rneduism.


Its cycle consists of tuo sentrcpic and. two Its cycle consists of two isothemal ond. two con- The total working volume (V,) is given by :
isothermal processes. stant volure prxesses. Vr=Vr+V"+Vo
No heat is supplied duing isentropicprocess. Regenerative heat exchange takes place during where, { = Volume ofexpansion space,
constst volume processes. This is an ideal cvcle
md tlle net heat flow to the worki:rg fluid is iero V = Volume of compression space,
in the constant volme processes, o the net efrect V = Volume ofregenerator, heater, cooler and coanectiog tubes.
ofthese prccesses is adiabatic. This total volume undergoes changes due to the motion ofthe two piston.
For its operation only one piston-cylind.er is For its operation two rylinder piston are required
requed. (the displacer piston ud the pwer piston).
When the crank rotates through one revolution, the following processes take place :
The cycle is zop rrctiable as altematelv the The ryde is practirable, as all the processes are 1. The working fluid is passed from hot space to the cold space via heater, regenerator and
isothemal and isentropic processes a.e io be performed at the same s1reed. Engines working on' cooler. Tbis is accomplished by phasing the two pistons such that there will be small change in the
slow.and fast respectively. The rycle is ofaco- Stirling cycle have been in existence. total working volume. Thus, tbe heot associated wth the working flu is passed on to the regenera-
d,e nlc npo rtq nce onl!.
tor at almost constant uolume.
It requires a very long stroke, and produces, Itdoes not require a long stroke, and hence the
a low m.e.p-, dtae lo a nauow indietor 2. The working fluid is compressed when it is rnainly within the cold space and the cooler.
.uery m.e.p. k relatiuely qute high.
qraSTam. 3. After the working fluid undergoes compression, it is passed from the cold spce to hot space
through tlte cooler, regenerator ad heoter, During this operation the heat of the regenerator is
transfered back to the working fluid at consfant volume,
19J3.4. Engine Geomery and Drivingmechanism L

4. The working fluid being located mainly within hot spaee anil the heater, an expansion
The following.-are the three basic configurations/arrangements takes place thereby producing work output
ofengine rayout :
7 z Ttt o\eparate cyrnders, n"i
^Tlro Tro"
piston mechanism. ""1 "" "is
of the engine, each with worhng
'- Inthissystem,thesefourprocesses atenotdstinctandseparatebttouerlapeacinother,
whereas in an ideal cycle the heat is rejected and received at constant volume and the
Txpe 2 : ASingle cylindcr with two pistons, one s com.pression and expansion occur isothermally.
the worhing piston and.otheris a
- piston er-rgn9.
Two So long as the compression and expansion are isothermal and regeneration is effective,
Type 3 : Two cylinders wth a worhing piston n the - the thermal effrciency ofthe engine is least affected with the variation oftotal volume
cooled, cylind.er and. displacer piston with
t he heated cy lnder-piston displac." errgine. (v).
rt
ii,|
i li 648
INTERNAL COMBUSTION ENGINES .MISCELT.ANEOUS ENGINES
bqr
19.3,4.2. Two piston engine
o 3' Reduced' erhaust emission (due to
Fig' 19'8 shows a ne\f, arrangemeDt (the arrangement overall lean mixtures and exhaust gas circuration).
discussed earer in Art. 19.3.4.1 4. Part load efficiency considcrably
is unsuitable for higher caplctr engines) higher.
iiii engne, Here, compression ofth working "r"?*i"
acting suitable for hgh capdcty 5. The engine canbe safee ouerloadcd
fl"iJ on one side and expansion on the for o brief period.
l,fl
other sideofthe engine. ".""* 6. Higher thermal efficiency (gpical
vaiue is +E;q" atioo ar an eoo"C).
7 ' Noise'free e''ie (because combustion takes place
outside the engine).
ill t' enshe is relcle (becauseit u*""0"
f,l""J:rlto only on rhq isnftion of fuel in rhe
"the
l'{l n of (e.s.utomobites, shio
l,::!!,?!::a stirling
",u.1yiety .apptications
engine oroootrioo or power senera_
bas almostcnstunt torqie characteristics
?onErnce over tie ntii" ,ong"
l,l 7o' Norlubricating oil required,(the
use ofrollsock seal has eliminated
I the use oflubricating
Disadvantages:
il L' Complex engine design (due to rhombic
i, ri, drive, regenerator, heater and cooler).
2. Requres large quantity ofcooling
i,rill water and*i"r roororr.
3' The engines re quires a brower to force
the air ttrrough the preheater and the
;li, Fig. 1e.8. rbo-pisron doubre-acting in-line o-rrru* chamber. This reduced the engine efEci;;r"d;ilases combustion
*oo,illllr"". the noise.
The top space of each cylinder is an expansion 4. TIte major disaduantage of a
!llt1r
- space (hot space) and bottom space is stirling i, ir" h- .o"r.
]'', compression space (cold space). 19.3.4.5. "rgi.,
il; Comparison between Stirling Engine
The hot space ofthe cylinder is connected and I.C. engine
,i - through heater, regenerator and cooler.
to the
--- cold
vvs e'qwe ofthe
space v other c)
lrE uurer cylinder The comparison between stirling
engine and I.c. engine is given (in
tabular form) below :
il
The pistons move with a suitable phase
- shift (90. for 4-cylinder engine). S. No. Aspect6
ri; o The mojor drawbach of ,:_"1 u: engine is greater LC. engine Stirling Engine
,.!*"tn- the cooler, re,enerator nd heter. f,'d.fficulty_in obtdining a uniform flow
.,riu* of greater 1. Fuel ad.ditian
aerodynamic and
complexities involved tr'i' J.o." ,rliiiiiii, I tl:-t".9 fuel is added in arr, Continuous combustion takes place in
i
l ':;nfin:;ic "oei"" Jtpcurt r," I either before or after compressron. II comblction chamber and energy rs
I ransfemed into or out of engine
19.3.4.9. piston displacer engine. | through
I
I an heat exchmger.
o Thepistondisplaceengine.consistsofacylinder,intowhichtwopistons-oneisknownas Valve or ports I

I Valves or ports required.


Valves o ports (for induction or ex_
displarcr piston and other porue r piston."The I

cylinder of disfiaJer"il;. divided into haust) not required since the gas
hot space (expansion) and cold ,p.." (.o_p.""sion). carge remains pemanently inside
The displacer piston reciprocates
within the hot portion of the cylinder (ope-n one) tne engine only.
while the power piston reciprocates Speed uariatinn
within the cold portion of the clinder (to*eiorr"t. Achieved by contolling the Achieved by changing the mass of
o The main function of the d,isplacer piston
is to heat and cool the worhing
amount of fuel injected m the workrng gas within the engine.
main function of the power-piston i's to comprlss fluid wh,e the quantity ofmixture.
and, expand, the gas. Heat rejection
the working ofstirling engine is complelely governed by rlative Achieved by high temperature
Achieved through heat exchmger and
pi.tonJht'' motion ofthese fwo exaust. hence it requires large cooling systen.
Noise Noisy (due to presence of valves
19.3.4.4, Advantages and disadvantages Noiscless, smooth working.
md periodic explosion).
The following ae the advantages and disadvantages
tional I.C. engine. ofa stirling engines vis-a_vis conven- o. Cooling system Sealing of cooling of system not
Should be perfecfly sealed.
absolutely essential.
Advantages: 7. gealing
The sealing cmbe done by the use
1. Since it is an external combustion engine, it posse This engine does not use ols sice it j
sses multi_fuel
capabitity. I
ofoil. willcause contamination ofthe work-
2' owing to rhombic drive (this mechanism consists I |
of two crankshafts ro,'tlg rn opposte
directions, each having a crank on rt two syn_chronising
ing fluid.
; gears are gsed to time these I I
crankshafts which in tum drive the ouip"t.liunl,
ii ;;;";r;;;;y;;irJiairrrrr. ,
.
a'1.'*
,:i
650 INTERNAL COMBUSTION ENGINES MISCELLANEOUS ENCTNLS 51

19.4. THE WANKEL ROTARY COMBUSTION (RC) ENGINE o The engine having three lobed rotor is driven eccentrically in a casing in such a way
that
there are three separate uolumes trapped between the roior and, thelasng. These three
19,4.1. Introductlion volumes perform "ind'uction", "compressionu,'combustion", "erpansion" nd, ,,erhaust,
Scheffel in 1952, to get a p'atent for rotary engine, utilised the principle that oual or processes in sequences, There are three power impulses for each revolution
of the rotor,
elliptical rotors can be designed to mointan contact, while turning about fixed' centres, andsincetheeccentricoroutputshaftrotatesathreetimesthespeed oftherotor,there
and that three or mare rotors can be run to enclose between them a continuously uarying is only one power mpulse for each reuorution of the output shaft f a single banh
rotar-v
uolume. The four volumes between the rotors, with the suitable arrangernent of ports, engine.
ignition system and adequate compression ratio, could be made to execute afour-phase One complete lermodynamic cycle is completed over 360'rotation ofthe rotor; the sction
otto cycle. However, this design failed due to its complexity and great difficulties/prob- phase takes g0' ofrotor movement and so the also other three phases. One flze rmodynarnic phase
lems involved in its manufacture. is
completed every 27O' rotation ofthe output shaft, since the oulput shaft makes thre revolution
for
Felia Wankel (German inventor), in 1954, got a patent for design offour-phase rotary every single otation of the rotor.
engine working on the otto cycle principle.
19.4.3, Feaures
Later Dr. Froede made certain modifications and an engine was developed, called as
KKM (Kreiskolben motor); now popularly known as Wankel Rotating Combustion (RC) l' Sitnple constructon, less mechanical loss, smnth motnn and d.oes not requiTe a cranking
engine. rnechanism.
2. Good power volune ratio.
19,4,2. Construction and Working
Construction. Refer Fig. 19.9. It consists ofthe following parts : .. 3. No reciprocating parts and hence no balancing problem and complicated engine vibrations
eliminated.
(i) Rotor (Three lobed). 4. Due to the absence of intake-exhaust valve mechism, theco rrect timings foropening
and
(ii) Eccentric or output shaft with its integral eccentric. No connecting rod is required as closing (the ports) con be maintained euen at high speeds.
the rotor rotates directly on the eccentric shaft. T:heoutput torque s transmitted to the shaft through 5. Lou torque fluctuation.
eccentrLc. There are problems in the design, notabty ofsealing and ofheat transfer but these
(iii) Internal and external timing gears. They rnaintain the phase relationship between have been
overcome sufficiently well for spark ignition engine to be maketed.
the rotation ofthe rotor and the eccentric shaft and eventuallycontrol th orbital motion of the rotor.
I9.4.4. Constructional and Other Details ofTyankel Engine
Working;
o The Wankel engine works on lhe four-phase principle. 1. Rotor housing and housing materials :
(The word pose corresponds to stroke of the reciprocating engine) . Rotor are generally made ftom high-grad.e mailzabre spheroid,ar graphite iron.
r The rotor housing is an dlwninium siticon alloy, bonded to the cylinder-bore rralls in
lnducon Compression thin sheet metal, the outer surfaces ofwhich have a saw-ooth finish to improve adhe-
sion and thermal conductivity. This rining is then given a hard chromium vanadium
Three plating, which in turn is plated with more cLome bul in a thin, porous and
lobed oil retaining
layer. End and intermediate roto housings are maile from imilated high silicon
rotor alu-
minium alloy.
o Aper and side seal blades can be made fromccs-iro but the more popular types are
made from ard carbon material.
o Both the leaf and washer springs can be made from beryuiurncopper, which
has the
ability to retain its elasticity when operating under workiig t"-peto."s.
2. Rotor seals:
t The planetary motion of the rotor within the eptrochoid, bore of the rotor
housng is d.e-
signed' to mantain a contact between the trioigular corner of tie rotor and
ualls. Peripheral radial corner blades, known as theaper seors, are necessary to fiuod",
the
gas leakage between the three cylinder spaces createrl irevent
by the three-sided roto. simi-
larly, side seols between the flat rotor sides and the bnd ancl intermediate
lgnion hou"i"g ,ia"
Expanson and walls are essential to stop engine oil reaching the cylinders, and gas from combtion
exhaust escaping into the eccentric output shaft region.
Xig. 19.9. The Wankel engioe. r The gas-tight sealing between the roto and housing may be consideed
in-terms of pri-
, mary and secondary sealing areas.
The primary sealing areas are those between the sealing elements (apex and
sde
blades) and the cylinder housing bore and side walls,
'
i.f'
i 6s2
INTERNAL C1)MBUSTIoN ENGINES
653
,i
,l ll
- Ii:, between the searingerements and g"oou"., t Efficient combuson k produced by having a very ad.uanced,
timing, tbis can be partiarl
^:f*f":::"ffiffi"1:..tn*"
The total sealed system aounil the
achieued by positnning a spork-plug in thi traitiig region
ofthe blie walt as fai o, po"";
lllll: rotor is known as asealed.grid.. ble from the waist or minor a.r.is of ihe epitrochoii
In order to estreminmum reahage-at eath
rotor dpex,srotted cyrindricar trunnion
walt profile.
,;,i brocks 4, Cooling system:
rj alu."i, tr," ouu"iJ-"i?to p"o. - r
;ff.":T::j;"i:::.",i:l::.::lr:*r1::o
vide a much longer reakage parh ar differei-
;;;;;;;i;il#";i:;:iT"T
"oro".o".,"i, The engine block is made up of the following five separate
section housings
,til blades overlap. n".. The front and rear end housings ;
:

il
;, Jll 3. Conbustion and spark-plug location TVo rotor bore housings ;
: )
,rll o The improvernent in combrstion process
can generally be made by insia ,ing two
Intermediate housing.
' tl'l' tead, comb ustion- rp* od.t " n" ir r n"
sparh_
*oui,e,"e" * All of the abovd housings are clampeil together by bolts.
:l
,l i:::i ;:;::^e "
o r
"' "f?ti\
io^u ur-
t coolant citrukrtion is prouided, around, tbe bore wails by arial possages
A tilting or rocking action takes place when the rotor is housing secton, while the side walls of the twin cylind,ers
cast into each
near the T.D.c. position an. combus_
tion commences' This tends to expand lhe g*"" are cooied bfextend,ed, cootant
i., it ei"uding portion ortrr" passages ad'jacent to the side walls cast in
"o.i,r.tion chambe
but to compress them in the trailing regionl both the end covers and. thentermediate hous-
Lng.
Positaning a plug in the high'pressure trailng
- produce srightly more power and use ress
region of the combustion chamber tends
to
clearance and swep vorume, compression Ratio, Engine
Dispracement capacity
fuer,"whereas if the prug ;ri."""lrn rn" h"airg 1. Clearence and swept volume:
portion ofthe chamber, better starting and
idre running is obtained, Ttre. clear-ance volume (v") ic d.efirrcd as thz
uolume created betueen the curued. chanber
walls and one of the rotor flanks when in the nearest-approach postion (T.D.c.)
Triangular .fla!sid'e adja-
to the waisted portion ofthe epitrochoid bore.
rolor
swept volume (vJ
,-expan'd's3:(B'D'C') is the diffeence of the madmum vorume to which the chamber
as the rotor ieuolues ond. the mini^u^ uolume of the combustion chamber at the
point of greatest compression (T.D.C.) lt may be given by the following :

V"=B,tgffi
where, R = Radius of otor from its centre to one of the apex tips (cm),
W= Width of the rotor (cm), and
e = Distance between the axes of the output
shaft and eccentric robe, known as the
eccentricity (cm).
2. Compression ratio:
The Compression ratio (r), will be the conventional
formula, as follows :

,"= %j%
I vc
(where and v" are the swept volume and clearance volume respectively,
{ in cm)
_ GJlEwe)+v"
uootanl ---%-
jacker Rolor shaft

ffffi::"" 3. Engine displacement capacity :


In a Wankel engine there are three power strohes per rotor reuoluton, and the output shaft
Fig. 19.10. Wankel engine-sectioned view. -
completes three revolutions for every one revolution ofthe rotor.
For achieving high efliciencv it is necessary_to per reuoluton of-the-output Therefore, there is one power strohe
have rapid combustion and pressure rise, shafi. This, therefore, can be seen to be equivalent to a two-cylinder
but the backward flame propagation of the four-
rtJ"e*. in the chamber is opposed by stroke engine which also has a porver
stroke every crankshaft revolution. Because ofthis reasoning
the forward movement of the,gases as they a two-chamber swept
are s*"eit,rou'd with the rotor, which, in uolume equiualence is geneialty recognized,,
t"wn the speed or combustion ."-i"*i"*
;:ii::i::" ao.. ,rot gi',u';;;;;. The displacement caoacity (vr) is theefore
plied. by the number
flc e the swept uolume of a single chamber
ntult.
of rorors; that is
INTERNAL COMBUSTTON ENCINES b)f

Y=2V"n
where, V"= Sweptvolume, and
= Number ofrotors.

Fig.
Performance of Wankel Engine
19.4.5.
1. Power and specific fuel conbustion :
the effect ofthe speed on the power output and specihc fuel consumption for
19.1 1 shows
I.C. engine (conventional) and Wankel engine.
I

-
E

1600

12W
<\
t\
'co @
st

^+
ol

8
I

10 l
o 48
^I
I
?
800
\ /i
<F
F
400
+
NO-
\
+ E
l6
t
f+
2 l.C.
'Wank
er ine)
lengir t)

.Z
RB
6E
f,v
0
12 14 16 f8
;
o I --1 .9 A,/F ratio ---->
=I
o Fis. 19.12
a.
19.4.6, Advantages and Applicaions of Botary Combusion Engines
Advantages:
1800 22co 2400
1. Lighter in weight, t{igh power to weight ratio.
Speed (r.p.m.)
--> 2. Compact, occupies less space.
Fig. 19.11
3, Engine completely balanced, hence free from vibration and inertia forces.
The power ofWankel engine, as cornpared with conventional I.C. engine, increases 4. Less noise.
rapid.lY. 5. l,ow grade petrol can be used, as it is less knock prone'
ahrc frctonal losses ofwankel engine, with increase in speed, ae lss than I.c. 6. Low maintenance requireinents.
engines (conventional). ?. The size ofthe engine is appreciably smaller than conventional I.C. engines for the same
Thespecificfuelconsumptionofwankelenlineishigherthaal.C'engines(conven. output.
tional). 8. Itgenerates power after every rotation whereas 4-stroke engine develops power after
2. Brake thermal efficiencY: every two revolutions.
The thermal effrciency of Wankel engine is less than the conventional I'C. engine since Applications.:
lhe cooling loss is more, 1. High performance motor cycle (Comotor, Luxemberg)
The thermal effrciency can be increased, by creatng squish flow .by prouidng recess in the 2. Snow mobile (Outboard marine in USA) I
rotor.
engine 3. Outboard engine (Yanmar)
3. Exhaust emission. Fig. 19.12 shows the emission of HC, CO and No" from wankel
4. Helicoper engine (Curtis Wright) I' I
(engine speed being 1450 r.p.m.) taking AIF ratio as variable'
l- The prese$ce ofunburnt HC in the exhaust gases is due to quenching ofthe flame by the 5. General purpose engines
surface of the combustion chamber. The H-C presence in tbis layer depends upon A.tF 6. Industrial engines (Ingersoll-Rand, USA)
ratio, temperature and turbulence in the layer. ?. Passenger Cars (Several models in Japan and Germany).
The variation ofCO (percentage) is also shown in the Fig. 19.12' The- emission ofCO is 19.4.7. \[hy Wankel Rotery Engine could not become Successful ?
- higher at rich A/F raiio ant deireases with increasingA/T ratio as sufficien air is avail-
Wankel rotary engine could not become successful mainly d,ue to the foilure of aper seal on the
able to burn CO as mixture is lean.
straght line contact between the rotor apex and, the casing. Also the side ga3'seals and, corner seals
The formation ofNo, depends upon o, content and peak cycle temperature. sinbe the haue not been successful in preuenting, mieture and gas leahage. Further problems / limitations
- maximum temperatuie in Wankl engi-ne is lower than conventional I.C' engine, there'
uith the Wanhel engine are :
. fore NQ formation is low.
6s6
INTERNAL @GUSTION En-CINES
iil ENCINES
1. High compression ratio difficult to achieve. 6s7\
2. Ineffi cient combustion.
3. High surface to volume ratio.
4. Scraping ofwall layera into the exhaust ports.
5. Lubrication and cooling problem.
6. Shorter iife,
./taa
7. Higher specific fuel consumptiot.

19.5. VARTABLE COMPRESSTON RATIO (VCR) ENGINES


-,/
t-.'.
19.5,1. Introduction
It is always desied to develop high specific power
output, accompanied by good reriabil-
ity and longer engine rife' The maximum power output can be
amount of fuel can be burnt efficien'y a" obtained if maximum
un".ti""! | thernethod.s which are used to
increase power out entoil d number of probl";;.-' -'' '"
It has been observed that a fixed, compression ratio
guirements of high specific output. H";.", engine ca.nnt meet the various re_
(VCR) engine seems to be a
iL'; u*i"iop,r"t compression ratio
necessitv. "i"ariable
19.13. VCR engine by dranging the stroke length. Fig. Varinax VCR ensine
- i:::::::i!r:,"_"!it-l colnpression ratio,,.is emproyed
forsood, stabitity and tow
19.1.1.

?#.:::::'::":y^1-lo1"o'nn'"u"i""ti"
charger to boost the intnke pr
x;;7;;,";"i-;;;"'::;;;'i;,1::-
qoeq v t e"-Lv@u La ultQu tne
turbo'
Fig. 19.15 shows the VCR s),stem, as developeil by BICERI.
without increasng the peah cycle pr""rur".Iti"
wnrh n^+i-- +r^l aL^ , -_ ,.essures
:,1i:*:i3:"::?j,1t'1r: j:lvorume.of ;;;;";;;;?il;;;,,i,ll,""liil5; j,"" j
;::i::j*::".y.,jlT:y:.d"]:inr.k"f";;;;;;;;;;;;J;JTJ"","ji::
and consequently more output wiil be obtaine.
Upper chamber
admssion valye
Pressure relef
. - ..--b"'-'EIULEP!u-lvarlaDlecompresslonratio,
of variabre compression rario,
vatve
L
fl*j"::.Tff;:"-1:J::,:i,:.ygl.::gT.rhe,concepr
however,canbemoresuitab. rry used with the turbocharged
diesel u"gin" i""uu"; oiilr"
f_ollowinc .,^
following reosozs .
"^-. :
(i) The vc* con1ep.t.il beneficar
onry at part load and the part-load efliciency Non-rturn
diesel engine is higher than that oittre-gasoline of the vav
engine.
(li) The diesel engine has bettqr Lower chamb6r
multi_fuelcapability.
admission valve
19.5,2. Methods to Obtain Variable Compres ,!on Ratio J
Variable compression ratio can be obtained
:
the ctcaronce aotume. rtrequire rnisins
,n"ft or lowerins of either entire cranh-
":),:,:":?::':::u
(ii) By changing both,tle
-
requlres a variable throw crankshaft3tyrgnce
and, swept vorume of the engine.This method
for changing th" .t-.o*".
Lower chamber
The following mechanisms have been proposed
for the VCR engine. orfice
o Fig' 1g' 13, shows the mechanism by which the displacement K
the stroke length. It is not favourejsince can be varied by altering
it mecnanism is quite complex.
c Fie, 19.14, shows the mechanism ofVarimax engine
developed by Tecquirnent Ltd .(u.K.)
which uses the movement of crankshaft fo. Fig. 19.15. BICERI variable compression ratio cngiae piston.
uftJring ;lr" compression ratio. This mecha-
nism changes the compression ratio from :

.S : f to'2: f. It bonsists oftwo main parts A and B as shown in the figure. The part B, known as
o The most promising vcR-mechanism is the use ' ca,ier is mounted on the gudgeon pin and part A, known as she[, slicres on carrier
of special piston to rover or raise the
piston skirt as developed by British r"ternationai-mbustion B to vary the clearance volume. These two parts ofthe piston are
tute (BICERI). Engine, Research Insti_ so arranged that
two chambers c and D are formed between then. The chambers
are t r,-rtt or
lubricating oii via a hole in the connecting rod and non-rcturn valve F"p
from the
lubricating system. /
The gas load is caried by the oil in the upper chambe C.
ENCINES 659
ITIISCELLANEOUS
INTERNAL COMBUSTION
6. Multi-fuel capability, The vcR-engine has a good multifuel capability due to higher
When the load increases the gas pressure is increased to a pre-set value, the spring compression ratio at starting and partJoad operation.
loaded reliefvalve Z opens and discharges oil to the main sump. The piston shell _Theopposedpistontypeengineisespeciallysuitableformulti-fueloperation'
slides down to a position decided by the relationship between the oil pressure in tro
chambers and the cylinder gas pessure, and thus a charge in compression ratio ie 19.6, FREE-PISTON ENGINE PI,AI{I
affected.
Free-piston engine plants are the conventional gas turbine plants with the d'ffirence that the
19.5.3. Analysis of VCR Engine air and clrnbuition chotnber ore replnced, by a free pston engne. The working of such a
cornprssor
From various experiments conducted on VCR engine the following obseruations have been plant is exPlained in Fig, 19.16.
made : Refer Fig. 19.16. A free-piston engine unit comprises of fle cylinders-with two assemblies of
cylinder located in the
1. Both the pressure and temperature exransion curves for VCR-engrne lie above those fc nistons that move opposite to each other. The pistons are powered by a diesel
constant compression ratio engine, indicating thaltheexpansion is slower at low compression ratioa i""ti". nu"rt """"-biy has a diesel piston at the centre of the unit. These are rigidly connecLed to

2. The gas temperature for the VCR-engine is lower than that for constant compression atio :. i"iJ"-aiu*nt"" pistons reciprocating in the large air-compressor cylinders. Each assembly has air-
engine for full compression stroke and upto about 50" after T,D.C. cushion cylinders at the end.
3. Boost pressure and mean cycle temperature increase with load but maximum cycle tem- Refer Fig. 19.16 (a) :

perature for VCR engine is lower at higher loads. The pistons are at their innermost position'
. 4. In case ofVCR engines, both b.s.f.c. and i.s.f.c. increase significantlywith increase in load. The diesel cylinder bas charge ofcompressed air at about 100 bar ready for firing.
The air-compression cylinders arc frlled with air at atmospheric pressure ; the inlet
19.5.4. Performance of VCR Engine
valves fV, and fVo have just closed. The air trapped in llte oir-cushion cylinders'
The performance aspcts ofVCR-engine are discussed below : also calle bounce-cylind.ers, is at its lowest fressure'
1. Power output. It has been observed that for the same engine dimension a VCR-engine
deuelops more pouer (i.e. it isvery compatt and has a hi.gh power-to-weight rotip without any penaltl Air compression cylinders
on specific fuel consumption), Pislon
2. Thermal loads. With the lowering of the compression ratio the rst-stage of combustim
increases while the second stage (ofcombustion) decreases but the overall heat release duration is
shortened. This leads to smaother conbustian ofboth lower and. higher compressian ratios.
The following are the other effects oflower compression ratio :
(i) Reduction in cornbustion chamber temperature.
(ii) There is an increase in charging efficiency.
(iii) The exhaust temperature increases sghtly.
(iu) The ignition lag increases and the maximum pressure clecreases. The use ofVCR
ple results in lowering of thermal lood,s and. a uery high specfic output, Compressed air receiver Air
3. Specific fuel consumption. It has been observed that the thermal efliciency ofthe (a) cushion
gine decreases considerably with the decrease in compression ratio. This effect, however, is counter- cylind6r
balanced by the following factors : To Turbin I
I
() The frictional power (F.P.) increases with the increase in peak cylinder pressure. I
the peaks pressure in VCR engine remains constanl therefore, the F.P. ofV0R-engine I

constant irresoective of the load.
(ii) Lower rate of expansion during combustion provides adequate time for combustio
complete.
Owing to.a\rve mentioned factors it is expected that the b.s.f.c. of VCR-engine should not
much higher than the conventional engine.
4 . Engine nois e. The noise etnanating from the engine depends upon the peak pressure in
cylnder and, rate of pressure rse. The peak pressure affects the lower frequency noise while the (b) lV1, lV2= Inlet valves
of pressure rise affects the high frequency noise . As the peah pressure in VC&-engine remains DV1, DV2= Discharge valves
irrespectiue ofload, on engine, the lotit frquency itise is reduced, However, as the eompression ratin
red'uced' (i,.e. high rates of pressure rise) the noise frmtheVC&-engine is mainly high frequency wise' Fig. 19.16. trlee-piston engine plmt.

5. Starting, As the VCR-engine uses highe compression ratio at low loads, it has agood
'starting and, id.lng performance at low anbient temperatures. fi

!t! .l
'
f
$
660
NTERNAL CoMBUSTTON ENCTNES 661
, MISCELI-ANEOUS ENGINES
in the ar compression qtindzrs
ft".H11:::J*lll?,u,i:l1lo are herd shut Applications:
.ioo .iri.a".r ro ffit:::,t:frflH"r":-ai
receiver that connects t; tii."*pi The following are the applications of free-piston engines ;
Tle cornpressed air_charge in the
diesel cylinder is at high temperature 1. Widely used as a submarine air compressor units.
fom r 2. Suitable for power generation in medium power range. Below 300 kW diesel engines are
:j.Til,ff:",tilj"r,l".li-:0"::-*.:111,^" r"p;.a-ri","ii"*," *,*"
resultingsudder,"i""orp""".",ri"io-t"r,u;i"#ffJ"'?fl:fr$#l[H: bu"nsl
"od . indispensable as free piston engine ofcomparable sizes ae not being built on commercial
:;fl, ffi" :T jl;"ll "" _":y::,^ ?::, : :;,n,:; l" i i u, o, n " u n cov ers ri s r,,l scale.

:;.*;:fi. j##';":f ,:::ffi ;;i;-;\:\i:::fi3i1a:,"IiT,f#: 3. Free piston engines are specially suitable for pumping oil ; also the same oil can be used
fuel. \
outermost oosirinno ,u^ ,_r!^!o_r!lturbine-. As the piston iontinue t" -""" i" as
::rffi :JJ,'"*'lr,:;11iil:":"**:f.F,:";,F;T'::i:11:::;"1
cylinder to ."ru."g" oL,i
es_el
combustion producrs
"., ",i lfl"]L".l3nter.fhe
trre-cvrin;;ff; ;:'""#?:Tr:,:.,i;;:ff
Droducrs an nrr
ilti: TIIGHLIGITIS
#': :i'H"Jg;"rt""j3l'::, l*:i: ir' " ;i ;:h# ers been i''i .n a s co mpres sedi
j l'ff [ffixlill;,illff :l*::e;:"*i''il1'"",:Tn""Hi:::"fi iliffi'"t
.1"1. .r,",su t."prJt";il
" Duel-fuzl operution, combines in a simple mmer the possibility of operating a diesel engine on liquid
fuels such s diesel oil or gas oil and on groeous fuels such as natural gas, sewage gas and cook oven gas
iil
aies"r ly.; ffi ;:l]rTil_;:T,[ffi :: :,|;ff
cyrnler etc.
At full-load, duel-fuel engirie is superior to diesel engine.
T!;rx!r"{!":xiT"'lf,':::i::1;.1"-" * g"y."tiiv tr,",u pr",,ts. rhis can be
achievedbecausetr,eisoo.c"omi;,;i;;;::i;;;;"il:"ii;X:il:;ffi
2.
3. A multi-fuel engine is one whi can operate satisfactorily on a wiile variety offuels rmging from diesel
irili
air' rhe nul' ni,H oil, erude oil, JP-4 to lighter fuels like gasoline, and even normal lubricating oil.
i'fi n'l"ifi:::;:f:;?:i.ng 'l"il",ii*:,"re of gasesdoing worh, t.e Tbe stratified. charge engine is usually defned as a S.I. engine (stratifred diesel engine has also been
The temperatue to the tue of developed) in which the mixtue in the zone ofsparkplug isvery much richer than thatin the rest ofthe
tit at ._ .,I"t".
^. isnition and, qctu"ity. o"*[;;i;
1900.C en be tol becarpl +peos onty instzntareowtt mmbustion chamber i.e. one which burns leaner overall fuel-air mixtues. Charge stratifi.cafio means
u.a-"*p*air,! ;;;;";#l JJ:;"::,1,t:::"."1
m-ost or the. cvcle the cvlinder ir
ili lo u.aaitiot' tttu i"JJii'lt ?"i"s *ot"a bv relativelv coolet *t providing different fuel-ai mixtue strengths at vadou places in the combustion chmber.
iI
rii
550.c.
materials problem of the *. ,.S'
"'-o--"rrbinei."r""r"t"h."'Ji:#i;il?::tr';':fi,J,1fir1*r"I? "rt"ig" Thebasic principleofworkingof sirlingengine isthesameasthatof conventional engine.Thealter-
nate compression at low temperature ond expansion at high temperature of a uorhing fluid in the basis
for the stirling engine.
and Applicarions of Free.pisron In lhe VCR-engine a high compression is used for good stability and low load operation and a low
rli ffi::ff:::"isadvantages Engine Arrangemenr
compressio:n is used at fullload to allow the lurboarger to boost the intako pressures without inceas-
ii 1. Less air rate as co ing the peak cycle pressure.
I
2.*ispossibre,",J:ff $r:T":].#l'il::r.T j:::jl; Free-piston engine plants are the conventional gas turbine plants wi the difference that the
pressor and, conbustion chamber are replnced by free-piston engine.
air com-
3. Lighter and smallerthan a aieset
engin; Jinu ,u.n" capacity.
ll t'l,*f* turbine is about oie third rie .t""
.ii." turbine for a simple open gas turbine
OBJECTTVE TYPE QIJESTIONS
5. The free-piston is brationless.
is considerabty higher than
Fill in the Blanks or Say'Yes" or'T.{o":
I ]h,:*t1"::i:Tiency simpre gas rwbine unit. A ...... engine is capable of roning on either gas
or esel oil or a combination of these two over a wide

lf liliJJ,lllff ::::T:::1.:".1i.ii1*t,iJ'";;ff""ffjH:TfJ""
'*J'IJ*:il""f":tiilix::::::::gr"ilffi **"ffi :lH,?ff t
range of temperature ratio,
The use oflow octane number fuels in duel-fuel engine results in poor performance ofthe engine and
required for conventional engine ;echanism
,;;ff greatly affects the combution.
9. The starting^ofafree-piston In a duelfuel engine the temperature ofinlet charge has no effect on the knocking limits ofa particular
engine is u".i"".o_o""with nonwa-ri^-r :^^^r
air requireJfor st.*iie;."alv
t"" starting is hardlv so ho.a-l
- . as
lltl conventional diesel engine
p"iffi i;;ffiffi;11"T1ift:[,Ti:T:i:ilT3tr: fuel-air mture.
capacity.
;:".1,;'.'"0
so ^r;^ --- rLouat oesel englrne ofthe
same 4. At fullload, duel-fuel engine is superior to diesel engine.
t" 6. A duel-fuel engine is prefened when cheap gas is easily available.
i;::ffiIl"rff-jlT*f:tr",-1,j]"^l!":rr"..s
are much
w.eil as mechanicar ioerria, its accererarion 6. A ...... engine is one which cm operate satisfactorily on a wide variety of fuels.
l11i:;1":
Disadvantages: ""p".io" ",,iiu"li;ii"l,.:i ffiffi: 7,......meansprovidingdifferentfuel-airmixturestrengthsatvariousplacesinthecombustionchamber.
1. Starting and control problems. 8. Ttre stratified charge engine combines the advnntages ofboth petrol and diesel engines.
2. Synchronisation problem not yet 9. A statified charge engile exhibits multi-fuel capability.
full ovecome.
t is higher than convenrionar
10. The stirling engine is an extemal combustion engine.
ftliffitl;*elrconsumPtion dieser engine, parficurarly ar 11. The partJoad efficiency ofa stirling engine is very low.
- 12. In case ofstirling engine no lubricating oil is required.
;ffffilH:'r:1l}jJ:[1*i:'.t t" rimited range. Anv nuctuarion
in rhe ruer suppry
"
662 TNTERNAL coMBusrtoN ENcrNEs MISCELLANEOUS ENGINES 663

13. T?re Wankel rotary combustion engine hs a mr power volme ratio. 24. Explain briefly with regard to Wmkel eugine :
14. Rtos of the wankel engine arc mde fron high-grade malleable spheroidal gaphite iro. (i) Cleaance and swept volme ; (i) ComPression ratio ;
15. The specific fuel consmption of Wankel engine is lower than conventional LC. engine. (iii) Engine displacement mpacity.
16. Rotary combution engines are lce noisy. 25, Describe brielly the perfomane characteristics of Wankel engine.
17. The VCR concept is benecial only at par-lctl 26. State the advantages md appcatioro of mtary cornbustion engins.
18. TheuseofvcRprincipleresulteinloweringofthermalloadsandaveryhighspcicoutput. 27, Wankel ntary engine cmld not become successful, why ?
19. In a free-piston engine anangemmt it ir possible to achieve efciency noe than 40%. 28. What are the advantages of vuiable comprssion ratio engine ?
20. The free piston is vibrationless. 29. Discuss the important designs ofvriable compression ratio engines ad comment on their salient
points.
30. Discuss the performmce aspects of V0R+ngine.
I ANswEns I 31. Describ with a neat sketch a freepiston engine plant.
1, Yes S?. Sat the advantages, disadvantages and applications offree-piston engines.
duel-fuel 2. No 3. 4. Yes 5. Yes
6. multi-fuel stntification
7, chrge 8. Yes 9. Yes
10. Yes 11.No. 12. Yes 13. No. 14. Yes
15, No 16.Yes l7. Yes 18. Yes 19. Ys
2O, Yes.

TIIEORETICAL QI,JESTIONS
1. What is a duel-fuel engine ?
2. Explain with a neat diagram the working of a duel-fuel engine.
3. Enumerate md explain briefly the variow fastors which afrecLs ombution in a duel-fuel engine. ,
4. Discuss briefly the perfomance drractristics of duel-fuel engines.
!
6. State the advantages and applications ofduel-fuel engines. t
6, What is a multi-fuel mgiae ?
7. What are the rquements of a multi-fuel engine ?
L List the difcultiee assosiatd vith multi-fuel operaon,
9. Discuss briefly the perfornance charactoristics ofmulti-fuel engine.
lO, Whatis a stratifieddrargemgile?
11. Give the classification ofstratified charg engines.
12. Explain with a neat sketdr Texaco cornbueon process.
13. State the advmtages and iladvmtago of stratified charge engines.
14. What is the working principle of stirling engioe ?
15. List the features whidr distinguish the stirling engine from other heat engines.
16. The stirling cycle is superior to the camot cycle, how ?
17, List the fuels which ae ued in a stirling engine.
18. Giv the differences (in a tabular form) beteen caoot md stirling engines. ,

19. Explain briefly the following with regard to a stirling engine :


(i) Two piston Inechanism
(ii) TWo piston engine.
20. State the advantages and disadvantages of a stirling engine vis-a-vis conventional I.C. engine.
21. Give the comparison between stirling engine and I.C. engine.
22. Describe with sketches the working principle ofWankel rotary combustion engine.
23. List the features ofthe Wankel engine.

--1
iI
t

I
AR COMPRESSORS
665

An air compressor takes in atmospheric air, compresses it and delivers the high-pressure
air to a storage vessel from-which it may be conveyed by the pipeline to wherever th1 supply of
compressed air is required' Since the process ofcompressing the gas requires that wok shul be
done upon it, it will be clear that a compressor must e drivJn by some form of prime-rnover. Of
the

2A energy received by the compressor from the prime-mover, some will be absorbed in work done
against friction, some will be lost to radiation and any coolant which might be employed to cool the
machine, and the rest will be maintained within the air itself. The prime-mover cnverts only a
it receives from the source into work, and so fai a"\th" compressor alone is
il Air Compressors
fraction of the heat
concerned, the energy which it receives is that which is available at the shaft ofte primc-movar.
The general arrangement of a compressor set is shorvn diagrammatically in.Fig. 20.1.
20' 1' General aspects. 20.2. classification.ofair omprcsors.
20.8. R.eciprocating compressoe
Construction and working of a reciprocating compresso.
{.irgi" .t"g"}_"ingtu Heat to radiation
equation for work-volumetric ef;frciency-Act"a p-v ai"=g."*
for singl=e "gu "o-pr."-_ and coolant
Multistage comprasion-Efciency of compres"oiHo* ti ir.ru""" "t.gl "o-i..."or_
cleaance in compressors-Effect of crearance volum+Free
isott em--al i-
"m"i"r"y
air delivereil ancr d.isplacemJnb_
compresor perfommc*Effect of atnospheric conditions on the
output of a ompressor-control
of compresoeAnmgerent of reciproeiing compres
I ore-Int".*t"r**p*"" . -otoi-
Reciprooting air motor-Rotary typu a. m-otot. )0.. iota[-"o-p"u"so*classification-
l,i '
Displacement compressors-Root blowers-vane type blower-sleady
li flow
il, centrifugal compressor--static and total heads-veotty ai"gtu-"
and thory "o-p."".or"-
ofoperation of
centrifugal compressor+-width of brades of imreller and diifus"i-h""es
,li aoa i"".t.opt emci*cy
of the compressor-srip factor md pressue cochcient-ai
rI flo* diagrams
"o-p.uso.-v"locity
ofaxial flow compressors-Degree of reaction_Compr"s.o, .haia"t""t*_-Surgi"S
*
l 20 5 comparison between reciprocating and rotar| "rikil;.
air conpressors. 20.6. comparison between
axial flow u_-d centrifugal
"o-p..""o.i_Highligh_OUctiue Type euestions_Theoretiel
Questions-Unsolved Examnls. Heat LOW pressure
air

20.T. GENARAL ASPECTS I


Source
The compressed air finds application in the following
felds : Fig. 20.1. General anangement ofa compessor set.
I It-is widely emproyed for powering sma' engines, generalry
b-'-v'e'J those of portable nature,
^
Compressed aitis used in such diveisihed fiilds as : o-----'
() operating tools in factories 20.2. CLASSIFICATION OF AIR COMPRESSORST
;
(il) operating drills and hammers in road building; Air and gas compressors.are classified. into two main types :
(iii) excavating ; 1. Reciprocating compressors ; and
(iu) tunneling and mining 2. Rotary compressors.
;
(u) starting diesel engines and
; Accordingtowhetherornottheprocessofcompressingiscarriedoutinoneunitorin
(ul) operating brakes on buses, trucks and
- several similar units in the one machine, a compressor rrray be single-stage, or multi-
trains.
2' A large quantity of air at moderate pressure is used sroge.
- .
melting iron,
in smelting ofvarious metals such as Again, in case of reciprocating compressors, the air may be compressed in the cylinder
in blowing converters, and cuiola work. - on one side ofthe piston only, or use may be made ofboth piston faces. Such compres-
. 3 Large quantities ofair are used in the air-conditioning, dryrng, and ventilation
fields. In sors are sngle-acting and double-actlng, respectively.
many ofthese cases, there is little resistance to the flow ofair;
and hence it does not have to be centrifugar compressors, which are of the rotary type, may be single or double entry,
compressed (l'e', measurably decreased in volume). For
such cases fans serve the purpose ofrnov- - which means that the compressor is flrlled with eitheione oi two air intakes accorciirig
ing the air to ihe desired location. In other cases, particurarry
in drying work, therc is appreciable to whether it is of the former or latter type when compression takes place in one o tw
resistance to the flow ofair and a compressor ofsome
sort is requircd toluild up suffrcient pressure units, respectively.
to overcome the resistance to flow.
Air compressors may be classifred in another manner, this time frorn an aspect of the use to
The function of a compressor .is to tahe a definite quantty whch they are put.
of ftud (usually gas, and most
often air) and, deliuer it at a required, pressure.
For example, air pumps and exhausters are used to produce vacua, their job being to
- emove ai from a particular system to create a low pressure therein.
664

,
"F-
666 TNTERNAL coMBUsTroN ENctNEs
AIR COMPRESSORS 661
Blowers and superchargers are essentia'y air compressors,
- but the icrase in pres-
sr which they produce is only small, u.ri ,rpto,
say 0.7 to t.0S bar. reach a pressure slightly below intake pressure early or in the stroke. This means thai the pres-
A booster is an air or gos comprcssor whih is emproyed, to rase sure outside the inlet valve is now higher than on the inside and hence the inlet valve will lift offits
- the pressure of ar I
gas which has already been comp.e""d.
Ii i" *here a slightly higher .-*--
pressure is required, seat. A stop is provided to limit its lift and to retain it in its valve seating. Thus a fresh charge of
or where a loss of pressure has occured in a long
d"li;; il;1-.-- air will be a-yirated into the cylinder for the remainder of the' inducton stroke, s it is called.
2O.3. R^ECIPROCATING COMPRDSSORS During this stroke the delivery valve will remain closed, since the compressed air on the outside of
this valve is at a much higher pressure than the induction stroke.
20.3.1. Construction and lf,orking of a Reciprocating
Compressor (Single.etage) Compressed
Fig'
2Q'2 (o) shows a sectional view ofa single-stage Delivery valve air delivery
reciprocating compressor. It consists of closed
a piston which reciprocates in a cylinder, driven ihrough
a connectingioa mounted in
a crankcase. There are inret and delivery valves
-oorrtJd in the head rtr" "rr."rrrtThese varves
are usually ofthe pressure differential type, meaning
\ Inlel valv
valve
closed
"irra"".
that, they *ill op.""; ;;i;; result ofthe open
difference ofpressures across the valve. The *o"kingifthis
trpe ofcompressor is as folows :

Delivery Cylinder

Connecting
rod

il
(b) I
il
FiE.2o.2(b) Fig. 20.2 (c)
;l
,l
As shown in Fig. 20.2 (c) the piston is now moving upwards. At the beginning of this
upward stroke, a slight increase in cylinder pressure will have closed the inlet valve. Since both
I

the inlet and delivery valves are now closed, the pressure ofair will rapidly rise because it is now
locked up in the cylinder. Eventually a pressure will be reached which is slightly in excess ofthe
compressed air pressure on the outside ofthe delivery valve and hence the delivery valve will lift.
The compressed air is now delivered from the cylinder in the remainder ofthe stroke' Once again
there is a stop on tbe delivery valve to limit its lifL and to retain it in its seating. At the end of
compression stroke piston orrce agaio begins to move down the cylinder, the delivery valve closes ;
the inlet valve eventually opens and the cycle is repeated.
As air is locked up in the cylinder of a reciprocating compressor then the compresson
pressure for this type of compressor can be uery high. It is limted. by the strength of the various
(a)
parts of the cornpressor and. the power of the d.ruing motor.
Fig. 20.2 () Sestional view of a single-stage reciprccatingmnpressor. It may be noted that there is intermittent flow of air in o reciprocating ar compressor.
As shown in Fig' 20'2 (), the piston is mong down
pressed
the clinder and any residuar com- 20.3.2, Single-sage Compressor : Equation for Work (neglectng cleorance)
air left in the cvlinder after ihe previous .ri,pr"""i""-*i'f;;;;d;ff;irl
evenrually In Fig. 20.3 is shown a theoretical p-V diagram for a single-stage reciprocating air compres- :
so, neglecting clearance. lr
t

tl
illi
INTERNAL COMBUSTION ENGINES
AIR COMPRESSORS
6.9

the reciprocating compressor, the inciex ls lcss than'',tbe adiabatic index. Sirlcc work nrust ue
, put into an air compressor to run it, every effot is made to reduce this amount ol work inout.
2". 'l Tnspection.of p-v diagram shows the frictionless adiabatic a 1-2,, and that
if compres-si;;;
;' along the isothermal 1-2' instead of polytropic 1-2 thcn the work done, given by the area of the
diagram, would be reduced and, infact, would then be'minimum'. Isothennal compression cannot
be achieued in practice but dn attempt s made ta dpproach the isothermal ., cooling thc
"or"
com'pressor either by addition of cooling fins or a unter jackct lo the comprqssor
'f- pV'= C (Adiabaric) eryIncler. Fot a
reciprocating compesso, a cornparison tctween the uctual work done dtuing Lompression and the
ideal isothermal wor.k done is made by r,-rcans of the isothcnnal efficiency,
This is defined as,
PVn = C (Polytropic)
Isothermai work done
Isothermal efliciency
Actual rvork done
Thus, the highcr the isothermal eftciency, the rnore nearly has the actual comprcssro'
approacbed the ideal isothermal cornpression.
Total shaft work done/czcle, W = Area 4L2:J4
or W = Ara under 4-1 _ Aea under 1-2 _ Aea under 2_B

__- -- _ :r*
D,v,
t.'V., - p,lt, _ D2v2
n-I
(p,v, - :,v,) - (*Y: _it") e,v, - p"v2).
= =
[+=".)
= -,t-r) (p,v, - p,\',\
['
r= [fr) \p,v,-n,v") ...(20.r)
This equation can be modificC as follows :

,r/
* = '_l - n?_i p,y, [1 ';,t)
'.p!1- prVr\ i.20.2)
Now p1V{ = p2V2

v" ( r,'tt""
.Fig. 20.8. Theoreticalp-V and ?_s agram
(b) T-s dagram

for a single-stage reciprcating air


compressor.
' v; = rlil
and substituting this into eqn. (20.2), we have
The sequence ofoperations as represented
on the diagram, are as follows :
(.i) Operatnn
4_i : air V, aspiratJ into the compressor .t p""""rr" L 'vn)= t .ll
(ii) operatian r-2 :
I^1:l_"-:r p, urrd W = n-\''
--;
p,V,,
l'-+f
i
+l
Pr\Pz) t| - r,r', lr- lzt-zl'
n-r-'l pt\p1J I
I

[rd::.taccording_ro rhe law prz, = c p, to pressure p,.


from
votume decreascs from ( to vr. Terlperar;;;;;#; from r, ro i ' .r-li |. .r---ll
(ii) oprisn r-g 2 cmpressed air of vorume v, and at pressuep2 with temperature
=;i_i p,ri j' t.":j
\frl
"l -
,?r- r,r', ],I \Pri " i:l i 2cll
?, t ) l
:'ii JT*"-",;.
u.,'* compression,
air wili. rlosc some heat. Thus, neglecting
i"t";i;;;ct
th"
;; J**,"*,
fricrion whichof
in the
*"
is smalr
The solution to tbis equation will ahvays come out ttegotiue shJ*ing that work rnust
done o the ctrmrrcssor. Sincc orrlv them.a;rzi.ticlc of the work drne.is require
bc
from tlt: ,,*1r.u.rion
case of then it is often witton.
AIR COMPRESSORS 6'7I

is the clearance voiume and is filled at this stage with compressed air. As the pision begins the
iniake'dtroke this residual air will expand, according to some polytropic law pVn = C, and it is not
until the pressure has reduced, to intake pressure at 4 that the inlet valve will begin to open thus
permitting the intake of a fresh charge of air. For the remainder ofthe intake stroke a fresh charge
is taken into the cylinder. This volume (% y4) is effectiue swept uolume.
-
n Work done,/cycle, W = Net area L234i = Aea 51265 - Area 54365
n-I Assuming the polytropic index to be same for both compression and clearance expansion,
then.
If the air delivery temperature T2 ( I I n-r
equation n "-1
:
--,, l(p"\" ,[
r - -!- n4' ll p 1
n_I.,
W= l-l ,,,, , ,, | n-l ^'tro ll;l "
i
QO.7)
T2 lp") ' /\- n_l rri l t''-'
l-l
- = .ot,/ or To-r, lrul"
- 'tpl/ ...(20.6) But pn p1 and p, - p2, then eqn. (20.7) becomes,
2O.3.3.Equation for Work (wih clearance volume) [, r'-t I
, In practice, all reciprocating compressors will have a clearance w= -!- ,.v. 1l!21 " -rl - n PtVt _,1
-l
uolwne. Tne clearance n-l "'ltp'J r -1
volume is that volume which renains in
nward strohe.
the cylnd,er cfter the piston has ,"o"h"d-th" end, of its t'"' I

I n-r I
. Refer Fig'. 20.4. At point 1, the cylinder is fuil of intake air, vorurne
about-to
"ol-e199
its compression stroke. The air is compresseJ*l"t
v, and the piston is
*i*li"t Jlraing to some
yv'= c to delivery pre6sure?2 and volume vz. At 2 te derivery valuu tt uo.J"uy opens
= f1 r,u,- %, '
lt#J -'l
,.,(20.8)
\w
for the remainder ofthe stroke, 2 [o B,the *mprssed air is delivered anir
from the cyrinder. At 3 the
t'"' )
piston has reached the end ofits inward stroke and so Volumetric Efficiency
on, derivery of compr"""u uii Lrr"" ut s. y" 20.3.4.
Refer Fig. 2O,4. Thz uolumetric efficiency of a compressor is the rato of free air deliu-
ered to the displacement of the compressor. It is also the ratio of effectiue swept volume to the
swpt uolume-

i.e.,
Effective swept volume Vt -Vt
Volumetric efficiency -
Because of presence of clearance volume, volumetric efficiency is always less than unty. As
a percentage, it usually varies from 60Vo to 85Vo,
Clearance volume
The ratio,
Swept volume =#=+=r ...(20.10)

is the clearance rotio.


As a percentage, this ratio will have a value, in general, of between 4Vo and 10Vo. Tbe
geater the pressure ratio through a recipmcating compressor, then the greater will be the effect of
the clearance volume since the cleaance air will now expand through a greater volume before
intake conditions are reached. The cylinder size and stroke being fixeil, however will mean that (V,
- Va), the effective swept volume, will reduce as the presure ratio increases and thus the volumet-
ric effrciency reduces.

Volumetric efticiency, n,",.= W


Yl - Y3

Vs
= V1-V3)+(V3-V)
V4
ffi .v, = L + vt-%- vt-vs
v3 v, vr vs Yt Vn
=r* y,-v, - vr-vr'% =-r,tt vr4, - vr-v3'
='*
%

'I I

J
I
"l
f,'x
li' 672
INTERNAL COMBUSTION ENGINES
! COMPRESSORS
1
=r + k-h. L 673
v3
I t rvl
/ ptxTomb.
fi+n-nl4l
=r+- f &lu ln"vi = novil ;,r:
.i rluot. (amb.) =
#. | \rrl I I
...(20.16)
\h) l=f^)"" I f /., \'l
lv' \po ) _ Pt\To b. It+n-nl:2ll
- Ttx Pomb.
rr-.=r+n-nlfiJ
I tJ
|
L (Yrll ...(20.17)

irl Ps = P2, Pa = pt) .,.(20.1 1)


(This efficiency should. not be-.used.
for find.ing out the d.imensions of the ylinder. For
,.:.Ir,r!: of the cylnd,er, the uotuietric efficiency based, on sucton cond,ition
...Q0.L2) should, be !: :"sins
used).
Fig' 20.5 shows the manner in which the vorumetric efficiency
- -"" fi ",,*:rr",;:;?ff""n#;il':t1fl,-.nsion and compression is same. However, varies with delivery pressure.

sinn or the ;;r":#;::: ;:';Ti-;;T,::X.?"::Tffiil;


-"" yl:^: ,m;*;i; lings, :*
i:.,:t:I, and -y:tlylli.
efiiciency
is 100% rhen rhe delivery
in fact no compression takes place at all. It
pressure eqals that of the
"",.1,^i".1,aii; ffi;";:"'l;
expansion ="t"o,on", pressure at first, and then more slowly for increase
pr"."r."
2", te volumetric
"m""""v
*'cir* l; in derivery
"t
highur pressure.

rr,o =1 . n- , (?)'"' ...(20.13)

=1+-,rla.l'"'
\A/ ...(20.L4) I
I

='.n-n ff,J ...(20.15) I


I

In this case volumetric efciency = 1 + fr _ * f5l. Ea


In pracrice rn" ry-l:'sucked ir .uring HF
while passing through ""
rl]ii***
,rucrion) srroke gets heared up 5b
the hot-varve *trr, hot cylinder walls. There is wire
drawing effect through ,n. r.:,y:: ""a*d;Jri"L, ii']Lr"o"u. Thus the ambient conditions are
different from conditions obtarneo f:ll*.:-oF-r
at state I in Fig. 2O.4.
It pa. = pressur ofambient
air, and Environment Deliverypressure----__)
Delivery pressure
pressure
{_u. = Temperatue of ambient air
,.. Pamb.Vamb. _ h(Vt-Vi. Fig. 20.5. Variation ofvolmetric elficiencywith delivery pressure.
To^b. Tr Tbe uolumetric efficiency is lowered, by any of he
following conditions :
Thus, v*0.= ## x(v,-v,)
(i) Very high speed
() Leakage past the piston
Thus volumetri. (iii) Too large a clearance volume
"*.,""., ""ilf;i-',"r, ""rur,a* may be written as (iu) O6s"n"rtotr at inlet valves
lloot. bmb.) = :-*=+]!".,vr-vo
,l - vt rix pa^b. Vt_Vz '
t'
(u) Overheating ofair by contact with
hot cylinder walls.
But from eqn. (20.11) (ui) Inertia effect of air in suction pipe.

vt -vt f run ,,_ By paying careful attention in the design of the compressor to these causes of loss, an
ltmprovement in volumetric efficiency can be obained.
vt -vs =t+ n- *la)
,3, 20'3.5. Actual p-V (indibator) Diagram for Single-stage C.mpressor
'. i Fig' At
gscussed)
20'6 shows an actuai compressor diagram. 1284
point 4,
i; the theoeticalp-v diagram (arready
& when the cleaiance ai" hr". to atmospheric pressure. The inlet valvc
"edu"ud
75
6'74 INTERNAL COMBUSTON ENCINES
AIR COMPRESSORS
now controlled by effective iwept
in practice will not open. There are two main reasons.for this.: (i) there must be a pressure areansion being 6-?. The mass flow through the compressor is
difference across the inlet valve in order to move it and (ii) inlet valve inertia. Thus, the pressure ;;il;t - t;), which is less than (uL-v): In-the limit' assuming the compressor to be strong
place fia, where V" = clearance *t"11,:' case there
drops away until the valve is forced offits seat. Some ulue bounce will then set in, asshown by th enough, the compression wold take :lll1"h
wavy line, and eventually intake will become near enough-steady at some pressure below atmos- " no delivery. It is seen, therefore, that as the deliuery pressure for a sngle'stage, recipro'
*ool
',oti,ing
pheric pressure. This negative pressure difference, called the intake depreision settles naturall is ncrease so the mass flow through the compressor decreoses. Note, also
"o^pr"rsor olso wiII tfu deliuery tenperature incres. Refe.
showing that what is called suction is reqlly the atmospheric air forcing its way into the cylindi lhal as the d'elivery pressure s ncreosed., so
against reduced pressure. A similar situation occurs at 2, at the beginning of compressed air fig. is, Tr, Tr. If high temperoture air is not o requirernent of the cornpressed air
,-lrrg ,o 2O.7 ,
delivery. There is a constant pressure rise, followed by valve bounce and the piressure ihen setges an energy loss'
d,el"ivered, then, aiy incleo""-in teperature represents
at some pressure abovti external deliverypressure. Compressed air is usually delivered into a tant
If high pressure s to be d'eliuered' by d sngle'stoge machine th-en.it uill,require heauy
called the receiver and hence external delivery pressure is sometimes called the receiver pressqrq th; hih pr"""uri rotio through the ,nachne. Ths will
uorkng perts n order to occorntnod,ate
other small effects at inlet and delivery would be gas iturta and. turbtilence. -irru"t;"". lobr"ing
;;;;;;"in" prool"i ind. the hish-torue fluctuation wiII require a heouer fUwheel
The practical effects (discussed above) are responsible for the addition of Such disdvantages can be overcome by multi-stage compression'
the two small
shaded negatve work oreas ehown in Fig. 20.6.

External deliverv

Fig. 20.6. Actual conpigssorp-Vdiagram. Mul-stoge cornpression is very iffi.cient ail is now a days almost universally adopted
.*""pt io'. ih"""
the werail pressure ris required is small. The method is not onlv
20,8.6. MrItistage Compression g"0"" from a thermodynamic point of view, but also has mechanical advantages over
"o^iE"ro",
"""t
single-stage compression'
In a single-stage reciprocating compressor if the delivery is restricted the delivery i
will increase. If the delivery pressure is increased too far, however, certain disadvantages Advantages I
The important od.uantages of multi'stage.compression can be summed
up as follows :
Referrirtg to Fig. 20.7 assume that. the single-stage compressor is compressing to pressure p2,
intemeiliate betyeen intake and delivery pressures'
complete cycle to 1234. clearance air expansion will be 3-4 and mass flow through the compre 1. The oir canbe cooled at.pressues
will be conrolled by the gffegtive swept volume (vt - vr. Assume now that a restriction is 2.1\epowerrequiredtodiveamulti-stagemachineislssthanwou]dberequiredbya
delivery pressure.
placed on delivery. The delivor-pressure becones ps say, and the cycte is then 156?, singie-stage machine delivering the same quantity of air at the same
3. Multi-stage machines have better mechanical balance'
.,
rl
i

INTERNAL COMBUSTION
ENGINES
4. The pressure range (and AIR COMPRESSORS
I 677
aDre rni. iesurts
rimils. rhis results in-il";;;#,'"J,t\::T:;f-:r,Ty.-b:
;"1;".*;*"":"3:11i^ril" 1."-"c"i _"y be kepr wilhin
within desir.
:.:r: ITfr:
tr'on, i,i',*J,Zf eah!8e (ii) |"pt
- . Fig' 20'9 shows cycle arrangernents in the development ofthe id.eal conditions required for
due to lower temperatures sn.t/ii;\ ;--_^..-r ,:;:, *f:}f,;:)",# improued lubricu-
l
(
i3 j:.li:";
5. The_cyrinder, n:,*:::i":l{q
in a sin gie_stage ;il";
,*,,' *, * ffi ncy.
.c e multi-stage compression. For simplicity, clearance is neglected,
",1:
ery prssure. rhe doJn p"*"rr"
The down ;:' :":X,":t enough to withstand the deliv-
pressure .y;;;i;;ff::,"'ough Referring to Fig. 20.9, the overall pressure range is prb ps.cycle g156 is that of single-
::I f'"'::*. since
construction
constructian
cvrin.ro.. ur ^ rulu-sEag
^r d ---t1. maine may be lighter in
mariinurn pressure
the marimum rh-.^ iu
Dressrrrp there i_ is
._ low.
,^-18u
stage compressor. Cycles 8147 and 7456 are that of a two-stge co'mp.".ro" without intercoong
Disadvantages between cylinders, Cycles 814? and 7236 ae that ofa two-stage colpreisor with perfec! intercoolng
:
between cylinders.
I

.l *r," tff;:r"j:: mutti-stase compresso-r.with


intercooters is tiherx, to
:l
I
I

Muti
"y::,:(flrlki"#,::ry?,.a tase com pres so' rh"' ;;;*";;;.;;.' be

I
- sra ge .."ip"o. ],'j "r
"ti
,n" a-rrangemenr
which compressed
air from
20 8."rjHi:tt?.".:iT#:"JTr:-1-series
The t;;;.;; llv,rr3je^r1,in
air for the cvlinde which.foll"*r.
ft;;;i"h;ltrated in Fig. Delivery
#sne.tntake
In th ow-pressure cvlinder,lllns
::d:":d "ra rt"lii"il""uru \-olume of this cvlinder
t"t it Jil, i.
pressure
rh" .*r;;;;;#e?t tne machine, is increased. ""li"l*r.l
sin"" "*puo.io'
n liilyliode (p" or p)
:o:tlgl,
il-to,ttr" -'chii;; r# ;i#,'q" because tt_t. which
--'-'e ts greater mass flow through *ii"J'i'i"&r,"es the air
singre-stage ^*t. "yli.,".
the murti-stage arrangement than Without
^u"irr".-- the ntercooling

Intercoolef
Perlect
intercooling

L.P = Low
pressure
l.P = lsr..,. Inlermediale
ptessure
prsssufe
H.P = High (pz)
pressure \ t pV=C
L. P.

Air Intakg
inlake pressure
3-Stage compressor (P1 or pr

. rf a intercoorer,.,::r:1,:1,Pr.""";"t';:"t",
in wtrich.rhe compressed air

li:.?Tin :::: ,n:,:::::::,;i l::;in; reduced ri ;,* ;ii b.np."u,.."


is cooted be_ Fig.20.9
'Perfect intercooling'means that after the initial compression in the L.P. cylinder, wth its
consequent temperature rise, the air is cooled in dn intercooler bcick to ts orginal temperature.
';n,!!;"::,,;j":, n;',::*:,,#i:,;;:i,r:Jtr'""'":l?,:i:.'jr,'"",:T ffifl"H,,:i:;-":ii This means, referring to Fig. 21.9, Tz= Tt, in which case point 2 lies on isothermal through point
1. This shows that multi-stage comprlssion, with perfect intercooling, approaches more
* *i['"ryn::Ti,Xi,X:^:n'#t]r",',,#wo orthree srages orcompression.
?e comptex_ ideal isothermal compression than in the case wiih single-stage
closely the

Ideal conditions for multi-stage compressors : "ompressiorr.

:::::::li3i"T;ft: ;l",HTH*l*"H: i,:t'ers.


which decease as the pressure Case 1, Single-stage compreasor:
As earlier stated cycle 8156 is that ofa single-stage compressor, neglecting clearance. For
*,:f ri1"!Fi!t1""T'?:*ni::**""::*H;'""""'i.:.':'""i"'""ii"'":"'f
usuallv range This reduction
tilffi rfl$l this cycle,
accomplished by
cylinder;l;;;;;t"" i.
""lr.u l.
reducing the
| -l I
ll \- |
w=
f1 t'v, ll&1" -rl (20.18)
f\rrl ]
INTERNAL COMEUSTION ENGINES AIR COMPRESSORS

/ \- -l
f o-1 -rl
Delivery temperature, fs
= \'t I &-lj ' "'(20'1e) ,lfnl;.lol-
lpt
l\A/ \Pz)
|
Case 2. TVo-stage compresaor :
(i) Without .intercooling Hence, for minmum, W,
dw I

de, = T = O

8L47 ..,... Low pressure cycle Differentiating, with espect to p2,


7496 ...... High pressure
rycle.
For this arrangement work done.
-+- 'f+l ,,[#)-'*,o,,
(p,)T \ " )
I
*l
Pt"
' ["-t]
\n.i
o,

Bl47 ...... Low pressure cycle


7246 ...... High pressure cycle. r-2n+l\
For this arrangemen, p2-uo Pz-\ " )= (prA)
l- o -r ,--7 -l
| -un
* = -!_ ^-1
|
^
r'2 pzti- = (PrP) "
o,u, 'l\rti
lf.+)-
n
-1
p"vrll&l " -tl ..Q0.2L)
l\Pz
LJ
) |
pruf = toror)*
Delivery temperature is given by
pz2 = ptps ...(20.26)

r.=r.(u\T /\- or P2=


'[Ws ...(20.27)
" \pz) ="r ll " , Since 7, = ?, I
\P2 l and Pz Pg
Now, since ?, = ?r, then ...(20.28) i'l
Pt Pz
P2V2= PrV, ...(20.23)
or pressure ratio per stage is equal. lil
Also pt= pz P2 obtained from eqn. (20.27) will give ideal iutermediate pressure which, with perfect
l:f
...(20.24)
Inserting eqns. (20.28) and (20.24) in eqn. (20.21) intercooling, will give the minimum W.
With these ideal conditions, inserting equations (20.23), (20.24) and (20.28) into eqn. (20.21) ll
w= r-,,",
n-t - ^ lf*.)?.f.+l+-,1
-l (2025)
shows that there is equal worh per cylinder. I

ltrr/ \pr) i

Now, inspection of Fig. z.o.s shows th srnded. or"o iou, whch is the worh sauing which
Hence, w=*",ri[*)* ...(20.29)
occurs os a result of using an intercooler. 1
Conlitians fur minimum uor* Inserting eqn. (20.27) in eqn. (20.29), we get ilIt
It will be observ(d from the Fig, 20.g that as intermediate pressure fi
^ AIso
. + p,,then ares 2453
t fe, then ar.ea 2455 -+ 0. This ,n"ur,r, th".ufo.", tr,ut u,ipcntei#Jiur"
,,',f{**}"']
lrl
-+ 0'
?".p.,
p2 exists which makes area 2458 a maximum. This is iile condition when p"".ror" w_ _>r IN

lI is a minimum. n-l
. Inspection of eqn. 20.25 shows that for minimum W, l(p2/p1)"-il" + (pa/p)n-ttnl must be
minimum, all other parts of the equation being constant llli
in this consideratioo i. tt variable.
"rrp,
,,"li[#'i"']

" 2n
n-7
ii

1i

I
680
INTERNAL COMBUSTION ENGNES AtR CoMPRESSORS

: w=
#1o,v,lf*l#_,1 ...(20.30)
P4= ZPs= ZspL

thar p3 /pristhe pressurb *"" *";.


fotg ZPr= 7p,
Case 3. Multistage comprl,lcsor "o-jurro... P" r 1=
From the aalysis ofcompressor Px+l
i so far, for a single_stagecompressor, 2x -
Dt
rl - f, ro-1 ;)
w=fro'v'l]l?i '-tl I
--xlPtt-lr
= l-;- = r {(pressure ratio hrough compressor) ...(20.34)
i\lt /
lP7
Insertingthe vahte of Z in eqn. (20.88) wiil d"tenirine
For a tuo-stagecompressor, J the intermeate pessures.
In the event of interc<nling^,bgiW imnerfect we must
pressor, in which case,'in eqn. (20.91);ilriu treat each r*U, i o com-
| -1 1 witn this special;;;;;,the""Oorae
power per
w= J\,,u, ""iti, the
stage can be calculated, and finally the total power
,o_ of tn" po*;;;;; ;""" ,
lf.+.)-_,1
nt l. .r-r I| .- lt. \i:=
rr seems reasonabre ,o
".r".* ,n.r"r".",t:::;, r)uo*rmachine.
u
W=
v=
;-
"ul( U)^ -i .
o,u, n,v,ll#j ', _1 .
llJ ffi |

- l, .n-1 I
w=4,," l{#]' _,1
Heat rqjection per st@ge pel hg of air :
If the air is cooled to its-initial temperature the whole
- of the work done in compression must
and for.r-stage l be rejectedto the cooling medium.
compressor,
Hence for a single-stoge the heat rej-ected
is given by,
-l
'"-l l, l, ,4 I
i
* = -:!= r'u'
l[X;]
-t
'^ -tl ...(2o.sr) Heatrejected n=".v.
1[lzl " -tl
w=n-r.t''l[r'./
"(20.35)
This equation is ver
Lt ' l I
j,
euen to vapour it to anv tvpe of compressor
";s;;;, ;;:,)ny::2,'i;u "ru1;"" or motor, and
and since for 1 kg of air, prV, RTr and
L_l

Note that
tl
is th" p."r"o." ratio through
the compressor, in each case.
=
'.,
* = (
+)-
\3/ , rhen eqn. (20.8b) may be wrirten as

also' that since for an idea-l


compressor there is equal work
per cylinder, for an r-sage
w= J: Rrrl+_rl ou. kgofair
"o-n.uTroJi' '._L Lrr l
'"-ll n R (r2- l = 1......S.I. units
i
*n= = _r j r) heat uns I
l
w = n-L't'
,.v. ll)=" -, I
" L, =;;;..: ..*K.s. units
' l(p,J
I
...(2o.s2)
I But 4 = cP- c'
J
*..-, f,: :::Tl""r:t""tTffii't" p"u'"o"u" * , *:' r-",*"l,u"r,i.,u running under idear condi-
' Here * = -!- Go- c,)(ir_ Tr)
it is
shown that atio per srage is equar. ...(20.86)
Hence, for an x_stage::;"1',:::*" .'. Heot rejected. with perfect intercooling
ot pt pG+L)
*",
?Pt = pz - pa .....
""' -;:- = Z, sa! ...(20.33) =
f"o
(=l] (r2- rt)per kg orair ...(20.37)
Fom this,
Pz = ZPt
Ps= Zpr= 720,
f*"*. -+ ", - c,) = c
o' q1;]
INTERNAL COMBUSTION ENGINES
AIR COMPRESSORS

The first term in eqn. (20.37) represents t}Le heat rejected, d.t constant pressure in the
intercooler ; whilst the second term represents the heat rejected d.uring alone ; and
writing c, -- Rl J ('i, - 1) it may be reducecl to the form "o^pr"tiioo
T-n x work done in heat units
,J
To find the change in entropy (s) during the first stage ofcompression, we have, from the
definition of entropy,
.dwdQ=
a.s= ('..
n- T Work done = Heat rejected)
Differentiating eqn. (20.37) and dividing by I,
dw
,^.
os= - f^ *. lt-")l
T =lco+cu{-"-tll
dT
T
Integration gives the change in s as
i * r
lv-")l ros, rzrr,
l" -1 jj
(s, - sr) =
1.,

=
# ,r- c,) log. rr, [s"uut"rtr,.
=
;l)
...(20.38)

For the complete isothermal tuo-stdte compression the change in entropy,


(s, i?.lo4, pJp,
Fis.20.10
- sr) =
; eJpt = (co - cu) log" ...(20.3e)
20.3.7, Efficiency of Comprossor
But if the work done in stage ampresson is tn be mnmwn
The theoretical horse power ofa compressor is calculated on the assumption that the com-

+
11=lo)T \n /
= l.)#
tpr/
pression curve ofp-V diagram is an isothermal, Then,
Isothermal work done/cycle = Aea ofp-Vdiagram
[.
I
By inserting eqn. (20.40) in eqn. (20.38) = P1Y1logrr

Isothermal power =
dfri*#{ o* ...(20.42)
,t
(sr-sr) =fiGo-",r*[*)# units
:t
[In M.K.S. I
/. -^"'''\ Isothe"-al e5s psvrs = P loE" rxN
i{

=l-P
\ 2 )toE'P{P1 ..,(20.4L) I
| 4500 I
I .,(2o2 (a)l rl

Comparing eqn. (20.39) and eqn. (20.41) it will be seea that the one is half the value of the lwhere, N = number of cycleVmin.J
other ; hence the work done per stage is minimum when increase in enropy per stage The indicated power of a compressor is the power obtained fom the actual indicator card it
taken during a test on the compressor,
_ Isothermal
incease in entropy for whole compression
Number of stages isothermalhorsepower
Compressor efficiency _
-
and the ma.rimum tenyerature per siagb is constant and egual to ?r. indicated hoe potyer
Actual p-V (indicator) diagram for fwo.stage compessor isothermal horse power
Isothermal efficiency - shalhorse power LI
The actual indicator diagram for atwo-stage compressor is shown in Fig. 20.10. The wavy
nes during induction and delivery strokes are due to Tlatter" of the disc ves. The L.P. and where the shaft horse power is the brake horse power required to drive the compressor. A usual tl
H.P. diagrams ouerlap d.ue to pressure drop inthe intercooler, and, ofcourse, clearance effects are value ofisothermal efficienry is about 70 per cent. lil
plainly visible in the actual indicator diagram. The nertio and ftiction effects which result in The'ad.iabdtic fficiency' of an ai compressor is the ratio of the horse power required to E
valae flatter increase the area of the diagram stightly, ond, hence their
total worh of cotnpression.
ffict is to increase the drive the compressor compared with the area of the hypothetical indicator diagram assuming
jil
adiabatic compression.
:il

rfl

,'!
I,

I
fii INTERNAL COMBUSTION
ENGINES
ArR coMpREssoR,
uS,
i
tlt
il
or
flt-tJ''"'ll*i ,l
r )- ,.1(r,\T-tl. ,o
b
The direct efrect of clearance
volume, and because of the necessary
is to make the vorume-taken in per
in"""""" i.-.iJi-oi tt
stroke less than the swept
.o_pii"o;l;;;;;;H;
t!
'll Adiabatic ercie:
aw'\oitot,,=ffi 'J:"T;T,"ff " rhe ourput)
jj;*i#;:"",i.rn,^',^i*'i,^lJo,p,"",,,,"i,
!ir
t ...(2o.4g) "i;;"#""mr::'
Tbe ualue ofclearance may be expressed
.l 20.8.9. How to fucrease as follows :
ii Isothetrnal Efficiency ? t
Th" following methods are v'HwJEu
^_^^, compressors :
speed - -' employed to
.
!u acnlev
acl neaily isotheimal compression
$lT":"r}"fl:Ti::.f ::q,:'."d in macr,i,'ing-and erection, a large clearance cheapens a
i'rl
(1) Spray injecion.
for high .u'"1';,#Jl"TTffi:T: j:';:'1"1j'3;"*_,,"",,,",,or"**,o,u.o,,",".,,
:_- - This method (used dom
into the *rnp.urr* aso) assimilates the practice of
l:c
the late-r the compression inject- (iii) rncreasing the clearance.in-one
air. rt "iil;il:-r'r:liTe-veas
"'"ii;i;;irfifr["*;:er"towards "t""r"
*iidt" object orcooring stage throws more work on the stage
below. rn this way
(i) It necessitates tho rrsp nF a o" controruni the output bv throi-
-_^--.- J1T:?$:ffix':jX 1?i'L#lJo-""1n"""'
lffi
20.9.10. Effect of Clearance Volume
",,,",i1*:J;;;;;'#,"::::;;:"::i"r:;,:;;#:,,i1i,",,,
(ttil fhe water mixed
and a,,acks cytndzr wa,s
he crearance oorume is the-uorune
with air should be separated, its.inward trauer ptus ro:^i?!y!" within the cyrnder when the piston it at the
end. of
.-
ware Jrl War-er jackeins. rt
^. jacket which help "oort"ts
ll;;""ff;
before usng the oir.
iri!.;;";;"0;;:q""
uolume 8 to reduce th volune,actually u*l,ii tll,,,i"""r"",
aspirateiihe"efo""iler"ao;;;;1";*;""1d of crearance
tvpesorreciprosatins;TiTr',::#;ffi ::ffilT$#',f ##T":.:::::;Wi"':; as possible, but it cannot.be educed
t.;#;;;;;f* be as small
mechanicar reaaons, tbe piston face cannot
(l) Inter_cooling. be
_,_ high
afso ,. When the speed of the comr
with single-sta--gecomp"".rio it I":;|:Z::E::,\,f1;:"::,#;:-;"a"""i"""i"nl;r"*."T;Jil;:'Tn'nT;o."ssorisgiven
coortns is restoed to *ri""i"'";""'t'ii:t-t::t ligh and pressure ratio required is
t3.le less The p-v diagram for a-single-stage
in aition il;;;;"fiH:lgfi"Y
to,the- effecive. Th" rr";ii;;;_ and a single.acting air-compressor with
water jacket volume is shown in Fig' 20'
more stages. tt. th" n;;;;;;::Hrt"rt't"T*Lt"H"::::'l:il"TJ:"ilffi 11' a t" l'iiii"""l cleaance
the high pressure air is left in rhe
"i. "o-il-Tron
originar temperarure
i-T:'T-1" \r'Edl "i "t."ke,
c"""J'-t-L
. (41 Externar
o"":?;:'""f"1,':il"'j,fft':;''ng
nns on their external sufaces. -"'rpressors can be effectively cooled
by using
(5) By a suitable
- choice of cvlinder
ta.ge bo"e-i; Jfi;il; eroeo::o.T.-By providing a shorr srroke
and a
lT-::"::" "ril,i."r''i.""X'Jl'i:"r:: fjff:ri"ff:n,fl:::.r'o'r."than i. uuliiu"ro. und
:.:f:,t" the per-iodic mouon or trre-pi"'oi.ffiJ:H""ii:.:lf
ti"i" r". i""i""f,#:T#"#Ljffi,',"rt"1P_rT
"",tp*t
brrer
the or"ooriog,
the b-r"i
;;";;;" the air ror a
"u"r""e
iig*., clearace
uilortunately increasess th";;:"':;-ltr
ro
comPressed against the
this increasels *.[l."tt:"-"ses as the 1s
square of the bo"e, b,rt.in thJB#;:i;de
----" " "yli"a". "or"i
compressor
Mechanicar e"r;fi;:r'JJ:'.ff:iffi11il*rated varve'
cal output to the mechanicar nput. For efficiency is the ratio of the
an air mechani-
""-n""3jll
I'H'P. of compressor
Ilfechanical efficiznw .
t'truch. -=
Str;ftF;"*;;_*
20.3.9. Clearance in Compressors
TIte clearance uolz consists
of the following tuo spoces
(l) The space betwee :
the cylinder end and the piston
"om arin to allow for wear and to give
" mechani_
(jj) The space for the reception
ofvalves.
In high-class H.p. como"."*."" ,u^
swept volure, the clearance
^,^]__ volume
i";" ;ttressors mayle asrittle as 3 per cent
of.the
;ni :ffi iitffi,H tTttr,illlffilFl#
i;i,ff'i:*:.''i:il,lhs",ii,_;:l;Ti,,:? --l t" l*- swept votume (vs)
._ Dau ot putty is a measure of gap. --*-J
Fig. 20. r 1
L.:

;r

INTERNAL COMBUSTION ENCINES AtR COMPRESSORS

clearance volume and suction for the second, cycle starts only when the air pressure falls to the 2O.3.I2. Cornpressor Performance
atmospherc pressr. This is represe4ted by the expansion curve 3-4. Assuming the compreesion ,
and expansion ofthe air follorv the same law, the work done per cycle is given by the area 1-2-3-4- By compressor performance, we generally mean the mass of air delivered per minute per
1 on p-V diagram. B.P. (or B.H.P.) on the machine.
For a machine ofgiven capacity and numerial pressure the performance of a compressor is
W(area L-2-3-41) = W*o. (area 1-2-5-6-1) - W",o. (area 3-4-6-5-g)
influenced by the following factors :
f -1 (i) The pressure range per cylinder'
w = -tt- o.v. ll lzl
If \-
' Y4'4
lor)? (il) The number of stages employed.
n-7 " 'lt, I n -l l.o.J
t'"' (iii) The clearance volume.
(iu) The speed ofthe machine.
Pt= P2and'Pn= P,
(u) The cooling efciency.
[ll -I It
* = u= p{v,-v4) '
l[#-', -'l J
(ui) The air intake piping.
(uii) The type and disposition of tbe valves'
L" j 20.3.f3. Effect of Atmospheric Conditions on the Output of a Cornpressor
f n-r I
n -^n.v llr')-_rl
=_1
A low barometer and a high temperature (as encountered at considerable elevations during
day time in tropical countries) is responsible for an appreciable diminution in the mass output of
"
where Vo = Vt- Vs,
l\rrl
is the actual volume of free-air delivered per cycle.
l compressos which have to operate under these conditions. The volumetric effrciency (when re-
ferred to a standard atmosphere) falls by about 37o per 3b0 mm increase in elevation, and 17o per
ftl n-r I 5oC increase in temperature. As a result of the considerable reduction in temperature after sun-

.. n down, and accompanying humidity, power plant in tropical clinrates runs considerably better at

... w= m,Rr,ll?l" -tl


n_r...r.-r ...(20.46) night.

where rn, is the actual mass of air delivered per


l\&/ l
'cycle. 20.3.14. Control of Compressors
Compressor control may be carried out in many different ways, depending on the circum-
.'. Work delivered per kg of air delivered
stances in which they are used ; e.g.
[ -r I 1. A compressor, directly driven by a steam engine, may be controlled by a combined cen-
= f1*',l[*] " -'l ...(20.47) trifugal governor on the steam engine ?'d an air-iressure regrlator, the control consisting in an
adjustment of the speed to suit the load. The mechanism operates either the steam thottle or
LJ varies the cut-off. This is suitable where the prime-mover may be run at reduced speeds without
[From eqns. (20.35) and Q0.47) it is obvious that the clearance volume does not affect the
uorh of compressi.on per hg of air,l too great drop in e{ciency.
2. Where the drive is by means of electrical motors it will usually be necessary to keep the
20.3.11. Free Air Delivered @-D.) and Displacerrent speed eonstant (it may be inevitable with synchronous motors), aird then some unloading device
The free oir deliuered (F.A.D.) is the actual uolume deliuered, at the sta,ted. pressure re- rnay be used to blow low-pressure air off to the atrnosphere. By artificially obtructing the low-
d'uced to intahe temperature and. pressure, and expressed in m3/min. The displacement is the pressure intake and thus lowering the intake pressure in addition to the mass aspired, the tem-
actual volume in m3/min swept out per rninute by the L.P. piston or pistons during the suction perature of delivery may be raised to a dangerous value due to the higher pressure ratio, and this
strokes.
method is therefore not to be recommended. \
The free air delivered per minute is lss than the displacement of tlp compressor because of
3. A method, commendable because it affords some control over the volumetric efficiency, is
the following reosons i
to prouide uarioble cleorance control. This is,achieved by having air pochetb adjacent to the
1. The fluidesistance through the air intake, and valves prevents the cylinder being fully
charged with air at atmospheric conditions. rytinder, which are brought into communication with the cylinder by automatically ojrerated valves.
4. With mechanically operated valves it is usual to hold the suction valve open for part ofthe
2. On entering the hot cylinder the air expands ; so that the mass ofair present (compared
compression stroke.
with that at atmospheric temperature) is reduced in the ratio : (Absolute atmospheric tempera-
ture)/(Absolute temperature ofthe air in the cylinder), l In many cases a combinatiori ofthese, anil other, controls may actually be used.
'3, The high-pressure air rapped in the clearance space, must, expand o a pressure below 20.3.15. Arrangement of Reciprocating Compressors
atmospheric befoe the automatic suction valves can open ; a portion of the suction stroke is As earlier discussed the reciprocating air compressors may tre classified intti single and
therefore wasted in effecting this expansion, multi-stage machines, and they may be single or double-acting. In the latter case, air iq com-
4. A certain loss is caused by the leakage. pressed alternately on either side ofthe piston, and consequently there are two compression strokes
.
t! i'i
!

j 688
ti
TNTERNAL coMBusrroN ENGTNES
AIR COMPRESSORS
per revolution per cylinder. .or: machines
compression in'the various stages take prace
cylinders, the pistonsI in wh.ich
I
separate in
:
are independently actuated from separate
crankshaft' In others a compound cylinder, cranks upon the
ti I *rri.r, iotro. either two o even three stages, fitted
with a single piston, is employed a.r
li : tu" ;ir;;. -"y r" connected ogether each recipro-
cating in its own cylinder. "g"i",
ll
l. Fig.20.72 shows thee arrangements in diagram

form :

i)
i
L. P.
Coolant
I ou

i
I

I Fig. 20. 18. Intercooler.


section througtr an intercooleri:
in Fig. 20.13. The coolingwater passes
:h,o*_n
secured between trvo rub through the tubes
pf.tu", uia *" .i.
;r*h::: """ul"tu,
over the tubes through a system

20,9.77. Compressed Air Motors


in awide varietv of applications
For some purposcs in indusrry.
(b) (c)
"t"-"0"."i#ifiil;*?:"ffI"t"{ed
menrsrobe.",;;;;;rT;"$"r,=*:l;llTilr",i"f:?1,,*:1";i:**g.:,,T[
Fig. 20.12. Arrangement of reciprocatingcompressors.
iTi;Xli'i'T;i:"; gffij:li:1t-il,".0.t"".':;;ffi: wid.e apprications in consrructionar
Fig' 20'12 () shows a two-crank, two-stage
piston is actuated independeirtly
singre-acting air compressor. In this arrange_ rorms or poier ,""i;;;;;::ii:1,,.#f:i"rl:;ffii.tll,llo;ii.iln
simplicity and robustness of
,i:tnil";;ii::;:fi work

;fi'r::* f.o'i. ,up-".ut" crank. The canks are kept 180. out construction, l. *lrh an air-operated design.
Tbe most general types ofmotors
ae
"i"T
:
(l)
:t:;_?t:i?"!? jl:n.i_"'l-cl**11k,
three-stage single-acring air compressor. Pistop type or Reciprocaling
Here com. type.
(2) Rotary type.
il?.:",1i:i:?:'::i:: i::_lTiur
-'n" lvrin11t";;;il;;;';;*
tr,. ut.-Jii"
;,Tlhilii"_ilil;
;;ffi ;J#:H::H:::Tjf,::?J*l; 20.3.18. Reciprocating
;:.:T1"" ::T:J:::1,:.,, -:---,-::v'ruaucuurr'
Fig.20.12(c)showsatwo-cank-fhroo-cla-^-,-l^^,--^----, Air Motor
air compressor having double-acting low and
ediarc nroca,,-^ -_r;-r^-^ ^_TI: th.:*:t*e
il:.:T:i:::"::::'"":,,"r.1::1".::.lo-:*".i.gr"J1i.;'i''#":;i""u"1'"':":ffiiff:: l?"" :T'i:Ji: orrhat in the reciprocaring compres
these high pressure cylinders taking
* ""i"r",rr""*;i" lri#T,lll;il
prace i;;,i:l i:::::li:*"_,"';;;;"";,ff
ambient temperature. Th""e :- ;;,".::. j:;"#.i,TT,:l'in:
iijTij**:j';::'3"^1"-.1: T1ersen
ar arr recelver which the air is at approximately
Manv such arransements are possible )_ls as pressure ur op in
"r rr'ui. ii.".il;'odilffie orthis book.
The expands i.
r,he air exnants in ao _^+^_
motor ,
'-. Pr eou c drop ln f,ne
the alr
air line
llne between th" .;";;u;; and t},o
the receive thc motor
-^r^-
. 20.3.16. fntercooler rne expansion is internarrv,;?.1"1i1"t:::T::ll::"j*.'*e in.a manner *;j;-h ;; "-.a f,orytropic (i.e,
ll:.",,,il:m;:Tllllli*:"";11.';ft"#:',:f'i,il;H'""f'3#T::Jil:1,';ff;'"T":';,,il;
.*T,J:il"r",.J;?i
The coorer which is praced. in.between
removing moisture' cooles are sometimes
soges cared intercooler. with the object of
ds ;1TJXffn:"::1,": jt;:,":i:1i":lli:fr ::til:llffi :,kffi
-'"'t"'oLurc as rower pressures
'Aftercoolers', but it
fitted aft1r the last stage, and for this reason are
called
*:f,
reached may be sufficie.,gu'---
":?:'"iJ:$,,:,:lil;,;';{i*ll{i:Tf T:":J:'.'1""1"'Hi:'ff
are reachcd. The :",:ff r"JJ::
tcmperaturt:s
should be understood ti"r-"rtrira"^
cannot influence the utorh crone in .ond"n"ed, Jffi;il1;i"J":"j:"r:,.::_* j"^T-l"l"l.r.,n" ln"i.i,,".;".ir,ln tr,i" -"y r,o
compression. iT"*:"';i,l!,';,::i:lfjj1_dE;:I"{i'i:r:',:::f,"J'j:,T,liil:illH,T',l:
rormation orice in t}'u.yri"au" *it;';t";::"::j:::":l?
lurrcqucrcc "-t
"i,, lead to rhc
lieezing point. 'l'his may
Intercoolers and aftercoolers are simple heat rnav
may be necessary
necessa ,u to o,"-r,"lljl11"^":-:"::""""e or
,o pri-hea
ortocLeJ ua]u;"r'T";;#;r;h"is
olocKed valves.'Io prcvent this condition jt
exchangers in which heat is removed from which i. I'ig
which has bcen compressed and its t"-p"."t,r." air tce rormation.
rhis heating lr'!,",::":,':":::l]t]:]-t"lo*"jure to preuent ttle
laubcs an lncrease n "#,,ii;::i'""ffi
t o";;;" .. a resurt of compression.
A simprified ;:::#i:"":*m*li"]iXif^.:illrir,'""ff
demand from the .";;;;"J'sr "'r"!i'^
volumc at supply pressure h*i.Hr:",:::;::::{;:
and ieduces the
row,andur"*g.,a;'.1,p;r;;rff;';::#il:'":T*;#,XlilL.l"XH:,r'Jil""*
,ow, and a row grade;*,"i;llfljJlj::,:"il:'":f*:i:',:llr:i,,1t";;';;;.f*'i',.,;:r';:
90 INTERNAL COMBUSTION ENCINES 691
AIR COMPRESSORS
and even so the slots, in
The sliding blaile eccentric drum trpe requires internal lubrication,
which the blades move, wear rapidly'
The tootheil wheel type has a smaller friction and can expand damp
airwithout internal
torque
deterioration.In the "herringbone" type expansion is possible together with a high starting
'This
;;;;t;;;" mechanical siirplicity. the turbine for collierv work inspite of its
"o-m"od"
extravagance on alr.
Exa-ple 2O.L' A singte-stage reciprocoting compressor takes 7 m3 of air pe.r mnute at
lar or rc"C aod. d.elilers it-at 7 bar. Assumnj thdt the low of compression s pV1 35
1.0fi
'=-colnstart,
and the cleaiance is negligible, colculote the indicated' power'
Solution' volume of air taken in' vr = 1 m3/min
lntake pressure, Pr = 1.013 bar
Initial temperature, Tr = 15 + 273 =288K
Delivery pressure, Pz=7 bat
Law ofcompression: pV135 = constant
Indicated Power I.P. :
Mass of air delivered Per min.,

^ = H+ = "*i:T;J' = 1226 kg/min

7 1(n - l)/n
reciprocating air motor.
f ig. 20.14. Hypotetical diagram for a
Delivery temperature, r" =
' r,llzl
'rr,/
Fig. 20.14 shows a hypothetical diagram for a reciprocating air motor. The sequence of
operations is as follows : o 1{1.35 - 1)/1 3s

(j) operation 1-2 : The air expands from 1 (p1) to 2 (pr) at the end ofthe stroke (according
to the law plr = C).
=,* [t-rj = 47s2K

(ii) Operation 2-3. : Blow down (release) of air from 2 to 3 (at constant volume). nR Qz- ?r) kJ/min
Indicated work =
(ii) Operation S-4 : Air is exhausted from 3 to 4, and at 4 compression ofthe trapped or f1 i,.
cushion air begins. r aK
(iu) Operation 4-5-6 : Air at supply pressure P6, is admitted to the cylinder at the point 5 = x 1.226 x 0'287 (475'2 - 288) = 254 kJ/min
*, -
where it miies irreversibly with the cushion air. The-pressure in the cylinder is rapidly brought
upto the inlet valve, pu.
i.e., Indicated Power I.P = 4.28 kw. (Ans.) f
(u) Operation 6-1 : The supply of air. is made at constant pressure behind the moving
= # is a single-
piston to the point of cut-offat 1. The cut-offratio is given by Example 2O.2. If the cornpressor of erample 27'7 is d'riuen at 300 r'p'm' and'
to bore
V, *"g, ;;l;-;;tind'er mochine, calculate the cylind'er bore required' assurning a.stroke
Cut-offratio = ;i- -Ve ,atio' t., I. Catcutate the pwer of thc motor requ.ired to d,rue the cotnpressor f the
mechani'
ve-va,
cat of the compressor is 85% ond thnt of the motor transmission is 90Vo'
eff.ciency
The effect of cushion air is to give a smoother running motor. The position of the point 5 Solution. Volume tlealt with pr minute at inlet = 1 m3/rnin'
depends on the point of initial compression 4, and on the law of compression pV = constant. The
I
conditions may be such that the points 5 and 6 coincide. The analysis of such a diagram is best = 0 00333 m3/cycle
volume clrawn in per cycle = t*
caried out. from b1".ic principles. "'
Note. For a gilen power, a reciproeting air compresso consumes less airthm the rotary form, because i.e., Cylinder volume = 0'00333 m3
ofthe educed leakage and ofthe greater expanion that is possible. ltris however is only semred at the expense
ofa heavier, and more costly nechanism.
.. : z = o.ooaa3
20.3.19. Rotary'T\4re Air Motor
(where D = bore, I = stroke)
The ar turbne isvlveless, small in size, light in weight, and requires no internal lubrica-
tion, but air friction is tigh, and any dampness in the air causes rapid deterioration ofthe blading
at Low temperatures. I D2 (r.5 xD) =0.00333 or D2 =q#
i

!
t, 692
fl , i.n.,
INTERNAL COMBUSTON
ENGINES COMPRESSORS
Cylinder bore, D _ 0.1414
m or 141.4 mm, (Ans.)
I
l
Power input to the compress". Heat transfeted during compression,
=# = 4.9g kW Q=W+tU
.'. Motor _
pou)er 4'98 DtUr - DDo
I
= 0.9 _- J.5B kW. (As.)
|
= -# +co(Tr-Tr)

il, ,:t t2
R = 0.28T
p, in air at l^.!.1.-1and. 20.c
ly33i,!\:=';;",,1:"ri;i:f;::il,;!::;*::-b.ar-and
jni =x.'jJiJi:;:::,ro"" detiuered to a receiuer
"orrtn;."'-rY'LPressor
tahes
ot o
20"c and compresses
conpresses it accord.
accord.
"oi,tolr"|j,Ti"L" of n u".
R(n -T")
n-l
Tenperature at the end.
.(.i) of compression
(ii) Worhdone and, heat ; o'zaz
tran during compression per hg = (430 - rrr, fo.rr, - 12 I (Ans.)
l',1 solution. Refer Fig. ,o.ru.t"'"""o of air. L -11
Negatiue sign indicates heat rejecton.
Tt = 20 + 273 K j pr = lbar
i
= 2gB I bar ;: p,
p. = 10
to bar
hnr Example 2o'4. Following data relate to a performance test of a single-acting 14 cm x 10 cm
reciprocating compressor :
Suction pressu.re
=Ibar
Suction tentperature
= 20"c
Discharge pressure
=6bar

lr Discharge temperd,ture
Speed of compressor
Shaft power
= 180"C
= 1200 r.p.m.
= 6.25 hW
Mass of air deluered
= 1.7 hglmin
Calculate the followin.g :
(i) The actual uolumetric efficiency
;
(ii) The indicated power ;
(iii) Tlrc isotlwntal efficiency ;
(iu) The mechancal efficiency
;
(u) The ouerall isotltermal efficiency. (AMIE Summer,2OO0)
Solution. Giuen : p r= 1 bar ; Tt = 2O + 27g = Zgg K i p2 ?, = 1g0 + 2TB
Law of compression =6 bar ; = 45g K
(r) I'"-o""rt.rre at the
I = ;t;;tfir^_ *
: pur,2 = c; N = 1200 r.p.m., Psoft = 6.25 kW i mo = L.7 kg/min,
;

of compie""r.", i"l
For compression process "rrd
1_2, we have

?=(t\+-,01Y#
rt lp,J =lfj =1.468
OT
T, x 1.468 = 298 x 1.468 _
To =
(rr) \16 done
and r,1"t i.;r"";#;;;;:ession K or 152"C. (Ans.)
430
pcr rg oi_ii",
Wt'rh done, W mRT,
=
,-t l[ pz); -'l,
' -!_ I

lla) ..rEqn (20.5)l

tl
ii = t x 0.287 * 2% * ( 1.2
.
lro.li,r - r] ,r..r,
v(m-)

\t.z_r1l\r ) = kJlkg or air. (Ans.) Fig. 20.16


694 INTERNAL COMBUSTION ENGINES 695
AIR COMPRESSORS
(j) The actual volumetric efficiency, compressor worh obtained, from the analysis of a conven'
r"o,. : Example 2o.5. (a) show thot thz
Displacement volume (m3/min)
tional card. iith and, polytropic pio".rr., for the reciprocatng compressor s identical
"lroron"" rotary conlPressor where in certan
to iiot otton" from the analyiis of o reversble sleody flow
ih" pressure p1 and t, respectiuely to the
mass of a
vo= n2t* w gas is compresr"d, irom intiot cond,itinn of
| (for single-acting compressor)
pu, C'
ft,nal pressure P2 in accordance
to =
(t\2 /rn\ (b)Alowpressure'waterjachetedstead'yflowrotarycompressorcofnpressespolytropically
= 4' llooj " l.,J x 12oo=1'8473m3/min 6 75 k;1';;
'r';;;;;'f ii, froor'1 atm--an'd' 21"C-to o ss 9"' .syq".::d 4.3"c: !-11.Y':Y^'::,::":t^,::
of the colting-
iiu:;r;: fr"d'r;; *;ri ind ,no"" of water circulate f the temperature rise IDTI\
(P.U.)
F.A.D.
mRT, 1.7x(0.287 x1000)x 293 water s 3.3iC. Tahe co ffor air) = 1.003 hJ I hg K'
= txto:G-= = 7'4295 m3/min' Solution. (a) In reciprocating compressors the work required is
Pl
| -I
i.
I

r..^,={4*roo=1'445 t 100 = 77'3870' (An6')


v r.a73 w= -+p{,1["1
n-L
" -tl
(ti) The indicated power, f.p. / l\p1 |
: LJ
(1)
Tz- n -I ln(T"llr) = -!-nR(?2-T)
or
1 n tn\p2prt When cornpression is adiabatic, n = 1
energ! equd'tion'
Work transfer in rotary compressor is determinedly applyng steady flow
OI . ln (453/293)
(D Wih isentropic flow' Applying steady flow nergy equation' we get
ln(6/1)
or
2s'29=n",*t
n = L.32 n *9!*w
Hence, index of compression, n = I.B2
For C, = Cr,W = hr'- ht= cp(Tz'- T)
l, .^-1 I
"-1 where ?r'is the ternperature after isentropic compression
Indicated power,
'"=;-.", l|ff,J Since -P 1-t
1E

732 7.7
w= J Re; -r) '..(2)
= 182_1 x x0.287x293

= 5.346 kJ/s or kW
*
{t:l*'} Eqn. (1) is similar to eqn. (2) for unit mass'
(rrt with compression polytropic. In actual practice due to internal heating there is
incease ofwork done above isentropic wor\ and work done is
I'P' = 6'346 kW' (Ans')
(iii) Isothermal efficiency, r,"o :
W = cp(Tz- f r) = J- n{f"- f r)
Isothermal power = mRTTln (p,/pr) where ?, is the actual temperature, i.e., obtained by using the relationsbip pvn = C'
r.7 (t) wih cooled compression. seme heat is being taken away by cooling of compressor'
= x 0.287 x 293 ln (6/I) = 4.269 kJ/s or kW and so
UO
W= cp(Tz- ?t) + I
n*" = x 100 = 2e.857o. (Ans.) (b) \trork' W=mcp(Tz-T)
(lu) The rnechanical efficiency,
## = 6.75 x 1.003 (43 - 21) = 148.94 min. (Ans.)

\ 4_""o :
If the compression would have been isentropic
l4-l
r-1 11
n.""n= Hiffi , 100=
# x 100=85.620. (Ars.) Tr' = T, (rr) 1 + 320.3'K ot 47
(u) The overall isothermal efficienc
= (2r 273)
l+J"
\./ = '3"C

1oves (iso) i
Heat rejected to cooling water
loveralrgso) t100=#x100=6g.gq. (Ans.)
= n co(T,i - T2)
= 6.75 x 1.003 (47.3 - 43) = 29.11 kJ

Mass of cooling water, m* = 2'11 kg/rnin' (Ans') Il


= ##h l1
tl
^#=
tl
ll
. L-_
Iir
l!
/

696
INTERNAL COMBUSTIO ENCINES
AIR COMPRESSORS
ov/
air per ninuie
^";;;;;; il.ots;",_;";;.,.;;"';;ff1"i,.i!,::,:"i;,;:,2:;t:tr,;:",f:;[
as ,vo
Vt= 4.423,Va= 4.423 x 0.05 % = 0.221 f
ii"?' #1"='!"r:tZ:,::;:,:",'"^e "rll" "-"iffiume uith the compression and erpansian (V1- Ya) = 1.05 Vs - 0.221 V" = 0.0233
() Suept uolutne 0.0233
of the cytnder ; (ii)I'" "r'u"O
(ii) Indicated power. temperd.ture ; % = trS O:t = 0'0281 ms
-
Solution. euantity ofair to be deliveed i.e., Suept uolurne of the cylinder = 0.0281 m3. (Ans.)
fntake pressure and temperature = 14 m3/min () The delivery temperature, T, :
= 1.013 bar,
P1

Delivery pressure, = 1 +273=2gB]K Tz (p"\;


"r
Compressor speed,
Pz 7 bar
N= S00 r.p.m.
Using the relation,
4 =lAl
Clearance volume,
V" = 0.0s y (p,
Compression and expansion index, n = f.a Tz= Tt x
(l) Swept volume of the cylinder, V- : IA
Swept volme, .', Deliuery temperature =450- 273
\ = V, - Vs = Vt - V. = V, _ 0.O5 V" (jii) Indicated power :
Yr = 1.05 V"
Volume induced per cycle
= (Vt- V)
It/ -r I
|
ir
l[*] " -tl
Indicated power
vt-v4= - n-l pL(v1-v1)
-fo =0.0233m3 t'"' )

=1.3-r
(bar)
1.013x105x14
l.s ' --lor;;o-
3T"2
Pz = 57.56 kW.
i.e., Ind.icated power
= 57.56 kW. (Ans,)
reExample 2o,7. A sngle-stage, double-acting compressor has a
free ar d.elivery @.A.D,)
t.3
of 14 rn3 / min, measured at 1.013 bar and 15'C. The pressure and. temperature in the cylind.er
durng induction are 0.95 bar s2'c. The d.elvery pressire is z bar and. iid.es of compression
ond.
expansion,n=1'3.Theclearanceuolulneis5voofthesweptvoIume.CaIcuIote
(i) Indicated, power required i) Vo I u niet r ic
; ( effici ency.
Solution.
Free air delivery, F.A.D. = 14 ms/min. (measured at 1.018 bar and lE"C)
Induction pressure, Pr = 0'95 ba
Induction temperature, T, = 82 + ZZg gOS K
T.
=
Pr = 1.0 13 Delivery pressure, Pz= 7 bar
288 K Index ofcompression and expansion, z 1.S
=
Clearance volume, yB
= % = 0.0b %
V (mt) (i) Indicated power !
I

= 0.05 Vs vs J
Mass delivered per minute,
Fis.20.17 ^= #; = ### = 12.16 ke/min.
lwhere F.A.D. per minute is V at p(= 1.013 bar) and ?(1b + ZZ3 288 K)l
=
Now. / :U,
(z\"' To find Ir, using the equation,
% = 1.OS Y.' --? lPzl
"rra v3
\Pr,/ = (1r$] = 4'423 12 (or\-^'
rr = l- |
lpr/
699
INTERNAL COMBUSTION ENCINES
AIR COMPRESSORS

Pt
.e., F.A.D'lcYcle, V= (V1 - V4) p

(where p, and ?, are the suction conilitions)


288 0.95
V=0.81? v". ffi " ffi = 0.723v"

q*,.= = 0'723 or 72'gq6' (Ans')


Volumetric efliciencv, #v" = v"
'pl#L efficiencv of a^r":y:T:li:f:"^T essor

I Ex*mple 2O.8. (a) What s meant bv volumetrb ?

; Gi) dztiverx pressure, and (iii) throttling a,oss


How is it affected by tl ,p""-iin" "i*pit"o,
the ualues.
I (b) State at lec,st sir' uses of compressed air'
I (c) The free air d'eliuered' bv a sngle-stoge d'ouble-actins'"1'p::::::f..::T!1"'!l:,!7"
I u," of 1 ;:: :;; ;;" ;;;; i'il";,L*.y
l l a'' i i.,0" l1:::::::"! ^!:,:::':::::,i: ?: ;":::" *i:
s'0"c.
"J'I;:;:::#;-;;;;";:""" "iir. iln" 'i,pt i:il:'i"::!,1::
ii,i"iJ,',ii,i"?,i"1"""!'i" ::::"?,"i::'"JI3',-u,,oii;"1\i"
uoli^" oid the mean piston speed is limted' to

300 mlmin. Detertntne :


Pr = 0.95
(i)Powerinputtothecompressorfmechanicalefficiencltis90{oand'compressioneffi-
ciency 85lo; -^
(ii) the compressor runs at 500 r'p'm'
Strohe and. bore if
v"J k_-- v.
v (mt)
Take ind'ex of compression ond' exponsion = 7'3'
o is
(AMIE Summer,20O1)
as the- ratio of Free Air
vs
= 0.05
,
Solution. @) Votumetric efficiency of compressor d'efined

Delvered tr*)
uetluereo to "*
@AD) ov swepi uotu"' P/^b s t:ne uo.Iume :l "i: !:l:y'i::d^!',:!"":::l::t:?:
the "**";;;;;i;;i"'-ltrt
measured' at some reference
tn^v be the ambient condition or the standad sea
Fig.20.18 reasons :
i"*i fn it less than the swept volume due to the following
"""4iU."1.
(i) Throttling and pressure drop at inlet valve and passages ;
ry
/\--L_l
lPzl4
1.3_l
* (ii) Heating of inlet air by coming in contact with hot cylinder
walls ; ancl
t-r -
- l-l
'\A/ =305x l=l
\u.yo,/
=48s.5Ii (iii) Re-expansion of compressed air retained in the clearance
volume'

Effects of parameters on volumetric efficiency


:
Indicated power n
pressure drop.in the inlet passage
= l:-1 ^R1T'r- () Speed. of compressor.As the speecl is increasel the
1.3 ", andtheinletvalr"in""uu"""'Forthertheairtemperatureduringintake-alsoincreasesduetoless
of compressor with increase
efficiency
17.16 x 0.287 (488.5 available for cooling. notn oi ihese fa ctos reduceiolumetric
1.3-1 - 805) = 3809.4 kJ/min.
of its speed.'
.. rndicated power 3809'4 (ii)Detivert preasure' Refer Fig' 20'19' With in- P
= 60 = 6l'49 kW. (Ans.)
oi',,"ry ir!"t,t"" the pressure ratio increases
and
(il) Volunetric efticiency :
"."^"" during inward stroke o-b, the effective swept vo-lume
hence
Using.the relation, ;;;cJ. ih" volume of air clelivered GAD) is reduced
i';";V,-;;;;hen the deliverv pressure inincreased from
the volumetric efficiency is d'eteased when
vn_( pr\u"=lu
ve-lpo)
( o"\^t I z'\rtt
=[o.r5J
-r. r?. fitir"
or-"
at.u"ry pressLlre s ncreasecl"
=4.65
(ii) Throttting aoross the uolues' Throttling across
Vt= 4.65 x V, = 4.65 x 0.O5 = O.2SB the
- % Vs th" i"i"i'"th" "ed,rs the pressure in the cylincler,at and
Vl-V4=V1-O.zggv"= 1.0S % _O.2ASV"=.AfZ V" i"l"t stroke. Further, throttling at the-inlet
"" "i:tft"
;;;;;;"" increases the pressure ratio' Bothf these
_. pV p1(V, -Vo)
*==--8",- reduce the FAD, and' hence the volumetric
"if"tt"**l
efficency of the comPressor'
Fig.20.19
li
I
INTERNAL COMBUSTION
() Uses of ENGINES AIR COMPRESSOI{S 701
"ompressed air
I
:
a compressed air engine (air
fll I
i

,'.] ?tlti"*
kl) Driving pneumatic tools.
(ii, Spray painting.
motor).
= t''36xo'287 (462:4 - ^^':l ot;'it
303t' 1

I
(iu) Cleaning sufaces
L{.tr;J.
I
by air blast. = 5016.9 kJ/min = 8it.6 kJ/s (kW). (Ans.)
(ii) Stroke (L) and bore (D) :
Piston speed = 2LN
,,ii'""T?i;r::::iiliil:.TJimaterialsinpipe,ines
(ull) Pnsm. controls. 300=2x2x500
fui,il.) Inflating tyres L = 0.8 m or nm. (Ans,)
of.automobiles and tractors. 3(X)
.
(c) Refe Fig.
20.20.
)
tt
F.A.D. = :4 D2L x 2N x n..", ...... for double-acting air
'vol
compressor ...(1)

To find ,",. proceed as follows:

v4 lor)t^ (or\utt I o )ut'


% =
[tj
= ttj = lts.J =4'oe4
V = 4.O94 x % = 4.Og x 0.04 V" = 0.1637 V_
Vt- V? yr - 0.1637 % = 1.0 y; - 0.1637 V,
=,0.8763
V"

Now 4 nvt:v)
'-
^= RT -
- RTt

i.e., F.A.D./cycle, Y=(Vt-Ur,


+ t
(where p, and I, are suction conditions)

.'. v= o.8?68 %. # ,T = o.?ee v:


v(mt) o'7#%
Fis 20 20
" n*, = # ts
= t
=o'zss
. Substituting the values in eqn. (1), we get
- _(d) F.A.D. = 16 ma/mi- /-^-.
i = :il j; 4 ;;; fl ,,'="ilfi 1j_lx't'i ' 1s=|
=

(j) power input


;:,:r;f;: 3il#;il:?&*tr* Orx0.Bx2x500x0.T99
to compressor :
Mass flow rate of compressor .. D=( ., !u:l )t"=o.zsmor2eomm. (Ans.)
Inx0.3x2x500x0.799J
pV (txlO5)x16 Example 2O.9.A sngle-stage single-actng air compressor deliuers 0.6 hg of air per minute
^_ at 6 bar. The temperature and pressure at the end. of sucton strohe are 30"C and I bar. The bore

'*n*;: t.? i;: ililT; *, "*TPt - ;;';'^'" and strohe of the compressor are 700 mm and 150 mrn respectiuely. The clearance is SVo of the
#:il=' swept uolume. Assuming the index of compression and expansion to be 1.8, find :
(i) Volumetric efficiency of the compressor,
12 lr')ii (ii) Power requred if the mechanical efficiency is 85%, and
,t (a/ (iii) Speed of the compressor (r.p.m).
Solution. Refer Fig. 20.21.
' ,,= n * lSJtj = ro,, l-g*)t# Mass of air delivered, = 0.6 kg/min.
Delivery pressure, pz= 6bar
_ \Pl ) 0s6J = 462.4K
Induction pressure, pr = 1 bar
Power input to compressor J= ,. r]
= |Ln-r^* ,r,' -,'],,l_"rirl";
_ I Induction temperature, 7, = 30 + 273 = 36311
lll

'r03 I

INTERNAL COMBUSTTON ENCINES


AIR COMPRBSSORS
Bore, D=100mm=0.1 m
(ili) Speed of the compressor, N (r'p'm') : iii
Stroke length, L=750mm=0.15m iii
Clearance volume, y y"
mRT. 0.6 x 0.287 x 1000 x 303 = 0.5218 m3/min.
ll
= 0.08 Free air delivery, F.AD. = ; = ----- t't--
Mechanical efficiency, \a". gSVo
=
F.A.D. 0.5218
=0.5728 ml/min.
Displacement volume i; = ffi.
Also, O'5728 =
! O"t, N (for'singleacting compressor)

f, , 0.t'x
e2-v
0'5728 = 0.15 x N
(Ans')
Speed of compressor N = -0':7:?:+;= 486'2.r'p'm'
nf,plq2O.Lo,AnarcornpressorhzvingstrohetengthofSScmand'clearanceuol-
air at a.pres.sure t to studv the
urne of 2 pq cent of th-e swept uolume d'eliuers "f ? !::'..1:-":der
was ouerhauled' and a
effect of clearonce on lree "'""i
lli i-1 the cotnpressor
d.istance pizce of 0.55 ,Lr'li*"t"ti-un th" "a1'a1a'
cytinder head. and the cylinder. The compressor
;;;;h"; r;;;ssioned'"- und.ei the changed clearance' calculate and'
:

(i) Percentogechange in the uolume of free air deluered"


(ii) Percentage chonge in power expended'
a suction pressure 7.025 bor and' the nd'ex of
Before ond. after overhaulTng the piston ha.d. (M'U')
compressbn and. exponsion was 1'3' ^ cm
Vc=
0.03 vs
-l vs-----J Soluion. Stroke length of the air compressor' L -
88

clearance volume' v"= h x v" = o'02%' where v" is the swept volume
Fig.20.2l
The pressure at which air is delivered' Pz=
8'2bar
(i) Volumetric efficiency of compreasor,
n@ :
Inclex ofcompression and expression' n = 1'3
/ \u^
(i) Percentage change in the volume offree air delivered

Tr,o.=1-o-ol?) ...(Eqn. 20.12) :
Before ouerhoulng
=. Area of the Piston
x Stroke length
where t= =o.oa Stroke volume
fr = A (cm2) x 88 (cm) = 88 A cm3
I Clearance volume, V"=0.02x 88,{ = 1.764cm3 = V"
f 6)1r V = 884 + 1.76.4 = 89?6,4 cm3
rl,o =I + 0.03 - 0.03 l; I = 0.91090 or g1.0967o. (Ans.) Cylinder volume, V, = 4*
\I/
(ii) Power required to drive the compressor Considering etpanson process 3'4, we have rll
:
PsVsn = PoVon
Indicated power n mRr..L\A/
=n_r [o)"" -rl _
/
lD.
\vo
I
,n _If
1r ,r lj3l
l \pJ
-1 I r cq
?'" r*l'" =8.?12Acm3
r.3
1.3 0.6 l, 6\-l* tl -17RAl t
=LgL*60 x 0.287 x 303 | |
r)
|
-'1| = 1.93 kW or " U.025/ if
(89'?6 A - 8'712 A) = 81'048 A cm3 tll
L' I .'. Volume of free air sucked in = Vr- Yn=
.'. Power required to drive the compressor 1.93 1.93 '. rt
= 2.27 krff. (Ans.) After ooerhouling :
= t1** = oi5 ,
. iwhen the clistance piece is insertetl the clearance
space is iircreased, consequently for the
same sti.oke volume, the cylinder volume would increase iii
Tl

il
704
INTERNAL COMBUSTION AIR.coMPRESSORS 1.O5
ENGTNES

, Example 20.11. A 4-cylind.er double-acting compressor s required. to cornpress


'3Q m3 /nin. of air at 1 bar ond, 27"C to a pressure of 16 bar, Deterrnine the sze of motor requiretl
and cylnd.er d,imensions if the following data is giuen :
Speed of the compressor, N = 320 r.p.m.
Cleorance uolume, V" -- 47o
Strohe to bore ratio, LID = 1.2
Mechanical efficency, \p" = 82Vo
)
Value of index, n = 1.32.
Assume no pressure change in suction ualues and, the ciir gets heated by 12"C during
suction stroke,
Solution. Refer Fig. 20.11.

Net work done

30
vher:e V = suciion volume = 30.m3/nin (given) = = 0.5 m8/s
60
Clearance f t.z -r
lf lG)
votume
=Va=Vc
.'. Work done
1.32
T_52_tr1x10x0.5 il.1l
m2
-r] = ,rr.ur.n *-.
Total volume L
= V,
197648.9
Theoretical power
Cleaance volurne, Vt
F8.20.22 1000 = 197.64 kW
= e.lg + 0.55)4 = 2.31 cm3 t97.64 197.64
Cylinder volume, fzi=
ee a + 2.lt A= n;;;:;, .'. Motor ower = n** = os2 = 241 kw (Ans')
From expansion i-4, we have
"rru" | , rvnl
PeVs" =pnVn",
Volumctnc efficiency, n,,, = l1 .k-kl?l | ' **
[ \a/ I P"ti
or

or
vo
-lfl '*f/.1 =f'i-l=l-=nn,
u, _/-\v"
\r4,/ =f
\Pr \l.c28l
I
(Suf{ix'i' and'a' stand for inside and atmospheric conditions)
V' = Va ,4'95 = z.gr A x i -- /re\ut32l rx(278+27)
lt+0.04-o*[ij l . ffi
.'. Voiume of free air sickrd 4.9 = 1t.4s A n,' =
in =0.686orG8.G7o
= Vt- Vt
= 90.81 A _ U.49,4, = ?8.88 cm3 Now swept volume of one cylinder
Percentage change in free air deliuery
30
-- 4 ' 2"gn " # =oo17o8m3
= !!91M-78884
(ir) percenrase change = 2'87vo' (Ans')
rr;ffi];J,too ! -tr
D'? = 0.01708 or
t D2xl.2D=0.01708
^ 0.01708 x 4
nJ_
e=
#xp, x volumeorrree ajr sucked
in lf4l"-rl -- tt xI.2
D = 0.263 m or 263 mm. (Ans.)
and L = 7.2 x 263 = 315.6 mm. (Ans.)
,rr-rr"$r,ff;iff#::""-,:?Jii:':i:'lfl".'"xTl"'.1'.:.,3 ;;:",;;";Liinnlr:l;:x^;; Example 20-12. A two-cylinder single-acting ar compressor is to deliver 16 hg of air per
'. Percentage change in power mnute at 7 bar from suction conditons I bar and 15"C, Clearance mat,be tahen as 4Va of strohe
expend,ed. (decrease). (Ans,)
= 2.67%, uolume and the ind,.t for both compression and re-expansion os 1.3. Compressor is directlv

I
l
J
rT___......---

AIR COMPRESSORS l

INTERNAL COMBUSTION ENCINES I

ne Pb-4AN
couped to a four-cylindzr
four.-stroke petrol engine which runs
at 2000 r,p.m. with o brahe nean Now, 65.97 =
effectiue pressure of 5.5 bar. Assumift tt o"-
oo ,"tu of r.2 for both engirn and. compressor If D" = diameter of the engine cylinder,
and a mechanicat efficienqt of B2lo "
fr ,o^puuor, ,J"uIaE th; required &llii", ^"nronr.
Solution. Refer Fig. 20,11. Z" = length of the stroke of engine = 7.2 D,,
n" = number of engine cylinders,
16
Amount of air delive = I kg/min.
reo per cyMder= 4 x (5.5 x 105) x4 x D,2
x 12 D"-=4 x2ooo
; -
Suction conditions :.pr = 1 bar, ?r 15 + 27g }1g]f- Then, 659?=
= = 60 x 10o
From the gas equation, pL(Vt- i1) mRTt = 6l'9?x80xl03x4
.. Dr- =0.0009545m3
vl - v4 =
+ =*##g = 6.61 m3/min.
i.e.,
' 4x10"x55x1.2xx2000
D, = 0.0984 m or 98.,1 mm, (Ars.)
661 L" = L.2 x 98.4 = 118'1 mm. (Ans.)
= 2000 = 0.003305 m3/stroke ..... compressor being single_acting
Example 20.1S. A single-stage d,ouble-acting air compressor deliuers ar at 7.5 bar' The
vn ( pr\u" lJ)utt pressure and, temperature at the end. of suctian strohe ore I bar and.25"C. h deluers 2.2 ms of
From expansion '*%=l.Aj
curv - =l.i., =4.4$7 ftee air per minute when the eompressor is running at 310 r.p.m. The clearance uolume is 54o of
stroke volume. The pressure and. Emperature of ambbnt air are 7.03. and 20"C.
' Vr= 4.467 Vs= 0.04 y, x 4.467 = O.t7g7 V,
(where V" is the swept volume) t Deternine : () Volumetrc efficizncy of the compresqor ;
(i) Diameter and. stroke of the cylinder f both are equal, and.
Since % = Vs + V, = o.o v" + v, = r.ol v" (iii)LP. of the compressor ond' B.P. f the mechanical efficiency is 854o'
V1- V, = 1.04 Y" - 0.1787 % = 0.86tg V;
But V, Tahe : Index of compression = 1.25, and Ind,ex of expansion = 1.3.
- Vo is also equal to 0.00g805
Solution. Refer Fig. 20.23.
.'. 0.8613 %=0.003305
Giuen :pr- I bar, Pz = 7.5 bar, T, = 25 + 273 = 298 K Vo a. = 2'2 m3
0.003305 N = 310 r.p.m., V" = 0.05 %' P*. = 1.03 bar, ?o,. = 20 + 273 = 293 K, \^"". -- 85vo'
v: = o861t = 0.003837 n3
If -L" = Iength ofstroke ofcompressor, and
D" = diameter of the cylinder of the compressor, then
L" = L.Z D" ....@iven)
:, D"2 x L"= V"

or
?r ^^ 0.o0g8g?x4
i D", , 1.2 D" = 0.0098g2 or D!=
,r,x]-.2
i.e. D
,
= 0.15b6 m or 159.6 nn. (Anc.)
L" = lSS.e x 1.2 - 191.5 n-. (Ans.)
Now indicated power ofthe compressor

|, .-r
'- ,
- -n-L I( lzl "' _
^nr,'l(r,/ ']
1.3 16
1.3-1
x
6- x 0.287 x 288
=',0*
Brake power ofthe engine 54I. 54I
fr)*-,] F\s.20.23
llmeh. 0.82
= 65.97 kW

I
,iI
il
I
u
i
I 708
INTERNAL COMBUSTION ENCINES AIR COMPRESSORS
I
I
(i) Volumetric efficiency of the codpressor rl-i : where, n, = Index ofcompression,
uz = Index ofexpansion,
Volumetric effrciency = E:lL
v. Vt = V" + V = 0.05 V, + V, = 1.0S V", and
From the ezpansian curue J-4, .Vt= 0.235 V",
Inseting these values in the_eqn. (i), we get
P2Vsts = Pol,tnr' (' nz-\
| I f,
P2V"L'3 = prVnrs Vr={andpo=pr) wcycre = 2 x p,v, -4 *. *, * '4-r _l-
'l "; -r " o*em );;
v.=v {Pz\* =, (1!\'
|
rl'. I[
[\ '
_r _
| "f"""
^ |

l[AJ
[e,

' '(AJ "\1/ ={(7'5)ozes=4'7rV" [ ro<


But v=0.05% =2 x I x los x 0.00456 lffi".ou1t.s0'25tt25 -t]
V= 4.71x 0.05
{ = 0.2s5S % '1.3 t.
n -, =
(Y" + %) - 0.2355 %
lj _1
^a^a
'0.2355 t(7.bf-'"" -
lj'l
v" (5 x
I

= 9r2 1.0S x 0.496) _ 1.0205 x 0.591


I
[v" V" + 0.05
o.os V"
= t.....-............-
4- 0.2A5S V^ ) = 1825.7 Nm./cycle
\% %-- = 0.814 or 81.47o. (Ans.)
(ij) Diameer and stroke of the cylinder ---
D and L -l
:
Work done / cycle
60 x 1000
x r, o. m.

The volume ofair delivered at suction condition is given by 1825.7 x 310


Po^bVomTr 1.03x2,2x298 60" 1000 = 9.43 kW
'I=
V, - - = 2'305 mYmin'
prT*- 1. 293
The volume delivered per minute is given by .r. = t-I.P. = .
9.43
= rt.OD kW. (Ane.)
ft.meh. U.dC
Vr= (V, x l_ x r.p,rn.) x 2 Example 20.14. A single-stage_singre-acting cotrpressor d.eriuers 14 mj of free air per
2.3O5=V"x0.814x810x2 minute from l bar to 7 bar. Th.speed. of lompressir is JfO r.p.m. Assuming that compresson
2.305 and, expansion follow the law pvt.ti = constant'and clearance is |Vo of the swept uolume,
vI = find. the
0.814x810x2 - 0.00456 m3 or 4560 cmg diameter. and' strohe of the compressor. Tahe L
= r.5 D. The tempertu* ori priirure of air at
the suction are sdrne as atmospheric air,
14 D2L =4560
Solution. We know that,
rJ4 Ds = 4660
D=L)
- - l5eo x \u3 nuor =r +h-k[fr)"' = r+005-0.05 ()^" ('. r=*='oo =o*)
'= [----
wr |
./
= l7.sz cm 3 18 cm. (Ans.)
L = 18 cm. (Ane.) = 0.889 or 8B.9Zo
(rD I.P. of compressor and B.p : I
The free air delivered per minute is given by,
t;
The work done per cycle ofoperation for double_acti4g
compressor is given b i
%xlluo-x810=14

t, .,-r .l
t4 t4
I r
u" = n""rl slo = os39 r g1o = o'0538 m3

"=,. -\- #,,, {t*)f But,


f#,," {[?)T 1l
v"= | nzt

Ps=P2 and Po=P, 0.05s8=


! nr,rsn
i
l*
( n'-t
t. ,o"-t ll ^ lo.5s8 * )u3 =o'i7mor77 cm' (Ans')
=21 "1
l\-I ,rrrl(A)T "' _1J ...(i) "=l "rts )
L' [\a/ ",n L = 1.5 D = 7.5 x 7j = 115.5 cm. (Ans,)
llffj
ll
710 tl
INTERNAL COMBUSTION ENGfNES AIR COMPRESSORS t

Example 2o'l' (a) Define the term'ouerall uolumetric efficiency'wth (iii) The effective swept volume is decreased from (Vt Vn) to (r'V\. This is due to the fact
rocating co,rLpressor. Djscss the parameters in brief uhich affect it.
reference to a recip- -
that by increasing the pressure ratio the pressure of air in the clearance volume, at the end of
(b) Show the effect of increose in compression
ratio in a single-stage reciprocoting compnes- delivery stroke, is increased. The re-expansion 3'-4'occupies a large fraction ofswept volume (as
sor on p-u diagram and. giue its physicat explanntion. compared to 3-4). This reduces the effective suction stroke with increase in pressure ratio.
(c),A double-acting air compressor hauing size (D x (i) Procesc on p-V diagam
L) 3J x JS cm and. clearnce S per cent (c) :
runs at 300 r.p.m. It takes in air at 0.gs bar and 2s"c. The deliuery pressure
is 4.5 bar ond. thz Neglecting the efrect ofpiston rod the processes for cover and crank enda are shown on the
ind'ex of compression, n = 1.2s. The ftee air conditions are r.TrT bar and.
20"c. worh out the p-V diagram as indicated in Fig. 20.25.
following :
(i) Shout the process on p-u d.iagrarn (ii) The free air delivered :
- for couer and. crank end,s ; (i) The free air d,eliuered as
reckoned' from the dpparent u-olumetric efficiznq (iii) The heat
; rejected. d,uriig -- -"
compression ; (iu) V"= : D2L x 2N for double-acting
The potaer n'eeded to driue the
"o*pr""ro,
if mechanicar effrcieniy t aoc".t '
(AMIE Summer, l98g) n les\2 ,
Ans' (a) ouerall aolumetric efficiency is
(FAD) refened to the arnbient co.ntions to the
d,efined as the rotio of Free Air Deliuered
wept uolume or d.isplacemint of the compressor.
4 [1ooJ # " 2 x 300= 1?.96 m3/min.
The volume of free air delivered is less than the dispiacement volume
due to clearance. tirre lvo! c = clearance volumetric efficiency
ofthe delivery stroke the clearance space is filled *,ith air. On the inward stroke, "ri f, rV 'l
the
air. will be admitted only after the clearance air is expanded
"omp.essed
o the inlet conditions. This would =1_c lf l _11
=1-oosl(*["-']
fd:::,t1"_:,ll::rrt:.y"o.t volume..Furrher, in pracrice rhe air that is sucked in during the
rnclucfon stroke gets heated up whilepassing through the hot valves l\rt / J
and coming in contact with
hot cylinder walls. There is-also wire drawint effectihrough the valves resultiniin = 0.8765
drop in pres-
sure. Thus the conditions obtained at the end ofinduction stroke (p,, ?,)
are d]fferet
---. from the
ambient conditions (po T*). Therefore the overall etii";i;;;-
6, Vs = SwbPt rolume
"ol,r-et"i"
Vc - Clearance volums

'lu,overall -
Dl
, r.* lr_cf*)" _rJl 1 -2-3-4, procsses lot cotar snd
a
l o.* l, [\rrl Jl 5-6-7-8, processes ,ot cnnk end

where, C = Clearance ratio (ratio of clearance volume to swept volume), and


n = Index ofr-expansion.
The parametrs which affect the overall vorumetric efficiency are a given
below :
(i) Engll speed. Higher-the engine speed, greater
is the wire drawing effect, and also
higher the temperature of cylinder wallr, to*, th imci"o"y.
(id) Leakage past the piston lowers the
volumetric effrciency.
(ii Too large a cleqrance volume will lower the clearance v
-
the overall volumetric efliciencv.
volumetric efticiency, and hence -l u" l*-v"----{ vc
lr-
p
(ju) Obstructions at inlet valves wilt increase
wire Frg.20.25
drawing, and hence lower the volumetric efficiency. Pz
(u) Inertia effects ofai in suction pipe nay The actual air drawn in per min.
lower the
volumetric efticiency.
pz
=(Vr-Vr=%=t*r,c"%
__ _
() The effect of ncrease in comprr;csirln rvtio
on = 0.8765 x 1?.96 = 16.?4 m3/min.
p-V d,iagram is as shown in Fig. 20.24. Ilis volume of air drawn is measured at 0.95 bar and 25'C. The free air conditions are
By increasing (he delivery from prto pr,, 1.013 bar and 20"C.
lberdfore the free air delivered, in ms/min is
( ''
-^- O'\ ^
(
(i) The pressure ratio:-:-^-^^-^r
isincresedfrom P,
&o"
l) "l? ) = Pamb, !*-
?.r"
r 6.7a
The compression work perkgis increased from t-
- toGi)1-2'-3'-4' Ftg.20.24
0.95
x L5.74 = 14.61 ms/min. (Ans,)
2-3-4 .
1.013 "m
INTERNAL COMBUSTION ENGINBS
(.lj) Heat AIR COMPRESSORS
rejected during compression :
Mass of ai delivered per min.
Similarly for the cornpression process 2-3, rve have
(0'5;ioiu' 1-1
-" = = 17.48 wmin. 5
a fp"lr =l-*
=r-t f4ooo)-i.4
\Pz) t ?00 ./| =1.64540r?3=310x1.645=510.1
/ - \-v^
,,= ,,[!l' = r
= 2eB l-n.u )o*'* = 406'7 K
Heat rejected ou"r* **o*lr;r;: '*' lmsj
e2= ncu <rr_rrl
fi
l'4-l'25
= 1?.48 x 0.717 x (406.7_298)
t.zs_t
(iu) power needed ro drive;ltl;t-itIl,x,, ff"''
r I r -r
p= _
'rnech.
lr-^r^l(1)=
-ti"
In-r 1l
-= o.e l#" !!#xoz,7x2*kli# .ll
^"''"'""""llo.e5) -tllt^ .

": io.zs^ro-
Exampre 2o.16. Air.at-rr, ;,!;i3;.ii;:yy:F"|,
eoLpressor and is isentropicarly eompress" cytinder o,*,_,,"r"
pressure to 37"c in an intercoorer to ilipo. The air is then cobd
"r ",
at consta,nt Fig.20.26
and is then ogoin- ro*pnrud sentropcarly .'.
H'P' cylinder, and is d.eliuered, at this pressure.-Deterine to 4 Mpa.in the Work required to run the compressor,
pressor if it has to dliuer B0 mr the power required, to run the com-
of air per.n ,i, "ur" ot oLt ,onaiiii.
Solution. Refer Fig. 20.26. (M.u,) /= -f
'r -1 l^R
(Tz'- TL) + mR (Tr- Tr)l
Pressure of intake air (L.p. cflinder),
pr = 108 kpa
Temperature of intak -T, _1 , *R KT2' - T) + (Tr_ Tr)l
pressure or,;" 300 K
unt",iiJll.p. cylinder, T,==Tr;#:= r.4 35.89
Temperature of air entering H.p. cylinder,
ii = Sl * 2?B = g1O K = 14L t g600 x 0.282 [(518.2 _ 800) + (510.1 _ 310) = 4.194 kN m,rs
Pressure of air after compression in Hence power required
H.p. cyliider, ps = 4 Mpa or 4000 kpa to run the compresso = 4.194 kW. (Ans.)
Volume of air delivered Example 20'17. A trial on a two-stage single-acting reciprocating
Power = O _rA ar compressor gave the
required to run the compressor, p: d.ata
following :
Free ar deliverecl
Mass of air compressed, ,7, -
(103 x ld)
x 30 = 6 m3/min
(0.287 x 1000) x 300 = 35.89 kglh Attnospheric pressure and, temperature
= 1 bar and 27'C
For the compression process 1_2,, we have Deluery pressure
= 40 bar
Speed
= 400 r.p.m.
U -=(z\? =- f?oo)+i Intermediate pressure
=6bar
4 lptJ lroel =1'728sorT2'=300xr.728e =518.7K Temperature at the inlet to the second, stage = 27'C
Law of compresson
= pvl.s = consto.nt
Mechanical effciency
= 80Vo
Stroke of L.P,
= d.iameter of L.P. = stroke of H.p.
INTERNAL COMBUSTION ENCINES
AIIt COMPRESSORS 7t5
Calculate:
(i) Cy lind,e r-d.iatneters ; final pressure of 0.7 MPa. The intermediate pressLlre is ideol a,nd intercooing s
'perfect. to a
pressed
() Power required,, neglect clearance. (AMIE lVinter, 1998) The cotnpresson ndex in both the stages is 7.25 and the cornpressor runs ot 600 r.p.m.
Solution. Let, Dup. = dia ofL.P. cylinder, and Neglecting clearance, determine :
p (i) The internediate Pressure,
Dx. = dia of H.P. cYlinder'
() The total uolume of edch cylind'er,
L=Stroke=Dur. Ps
Assuming a volumetric efficiency of (iii) The power required, to d.rue the contpressor, and
l\Wq
(iu) The rate of heat rejection n the ntercooler.
F.A.D. Drr.r. x r.p.m. (for single-acting) Take co = 1.005 kJ I hg K and' R -- 0.287 kJ lkg K' (AMIE)
=i
Solution' Intake volume, Vr = 0.2 m8/s
6 = :4 Drt^p., Dr.p. * 400 m3/min Intake pressure, P1 -- 0'1 MPa
Intake temperature, ?r=16+273=289K
:. Dt .p. = 0.2673 m = 267.8 mm = L.
Final pressure, Ps = 0.7 MPa
Swept volume of H.P. clinder Fis.2O.27 Compression index in both stages, nt = t2-- n = 1,25
= Volume of air at 27'C and 6 ba Speed of the compressor, N - 600 r.p.m,
cp = 1.005 kJ/rg K ; R = O.287 kJ&g K
= lmr/min "'' x Zx r.p.m.
= ;4 Dp ' == 4
" x 0.2628 x 400
(i) The intermediate Pressure, Pr :
['.' p,V""=pzVu.p. or 1x6=6xV".". or Vnp.=1m8/min] pz= Jitps = ./6.1" 0J = 0.2646 {Pa ..'-'.(Perfect intercooling)
." Ds.e. = 0.109 m or 109 mm. (Ans.)
(i;) The total volume of each cylinder, V* , V", :

N
f1 n,v,ltfl- -'] . *,,",
600
Indicated work = -,] Weknowthat %,t=% or %,xff=0:2
i(f)= :, v", (volume of L.P. cylinder) = = 0.02 m3. (Ans.)
%f
= o-t-,^nr,|[*)--'] .*-",, i; PtV", or %,
PtvL
1[*)* 1 #
*
Also --PzV"" =
P2

[
t/ " -1 {H ... V", qvotume of H.p. cylinder) = ,*#f = o.0o756 ms (Arts.)
=
lor)* - rl
f1*,,1[#J" -trj " ,
rt= rs.
{t
It
r)
required drive the compiessot, P :
1"
,
"" (l) The pewer o
f "-7 I
Also, "'-
P,V
RTt
,=h,,",L[fj'-,] ......(Perfect intercooling)

6l
= roo x
66.
, 6'87(,,l; = 0.116 kc/s l. .125-1 l
2x]..25 ^
x (0.1 x 103),
ll0'7 lzxr'zs ' I
-r.J = 42'ec krfl' (As')
.'. Indicated work
(
Ls - kul:3 .a)H _,1
=os *o.116xo.ra?'9ootr/ \6/ I
= T*;
The rate of heat rejection in the intercooler :
(iu)
0.2
Llolj
= 45.94 kJ/s ftW) o,v. (01x 1d) x 0.2
Mass of air handled, = ffi = 0r8? -, %, = 0.241 kg/s
45.99 ^
Power required = OS = 58'42 kW' (Ans')
Dxanrple 20.18.,4 two'stale single-octing reciprocating compressoi takes n air at the rate
of 0.2 tn3/s. The intake pressure and..ltemperature of air are 0.1 MPa and. 16"C. The air is con- Also,

'. Heat rejected in the intercooler = rn x c, x (Tr- Tr)


= 0.24! x 1.005 x (351.1 - 289) = 15.04 kJ/s or 15.04 kW'
(Ans.) I
:ll

rl
l-_

7t6 INTERNAL COMBUSTION ENGINES


COMPRESSORS

Example 2O.lS. A tuo-gta9e air cotnpressor with complete intercooling dliuers ar to the
Now temperature Tr' at the end ofcompression in the low pressure cylinder
m'sins at a pressure of 30 bar, the suctinn conditinns being I bar and. 15"C. If both cylinders haue
the s.ame stroke, find. the ratb of cylinders d.io.meters, fr the efficiency of cornprission to be a / \d r.3-l
marimun. Assme the indx of compression to be 1,3. r=r," f .1' =(15+2?B) fryf', =426.4K.
Solution. Refer Fig. 20.28. \rrl
Frornn constant pressure process of 2,-2
\ ro /
Vt V"'
t2

t2
l

v2 t2 426.4
vt=rr= (15 + r?3)

Fom (j) and (ji), uv,r;,


i;=
v2'
v2 = 3.7 x 1.48 = 5.476

Dup. tV
uu,p. lv, rtl.+i6 = zsa.
Ratio of cylinder diameters = 2,J4. (Ans.)
Exarnple 2O.2O. In a two-stage air compressor the pressures are atmospheric 1.0 bar
intercooler 7.4 bor ; d,elivery 42.6 bar. Assumiig complete intercooling to the original tetnpera- ;
ture of 15"c and, cornpression ind.ex n = i.J, fin tn iorh d.one in copressing l"hg of air.
If both cylinders haue the sam.e strohe end the piston d.iameters are g cm and, J cm an.d the
vol-umetr.ic efficiency of the compressor s g0 pu, will the ntercooler pressure be stead,y or
""it, ? Justify you, onrr"r.
will it rise or fall as the compressor continuei working (P.U.)
Solution. Witla cornplete ntercooling, worh done,
n-L
n , l,
w=n-t Rr,lf)--lo);
.n-l
,l
Fig.20.28 'lt rrl \pz)
Volume of L.P. clinder = V, l
Volume of H.P. clinder = V, 1.3-1 I
IfD".". and Dn.r. are the diametes of the low pressure clinder and high pressure cylinder *flzs)T _r
respectively, then \7.4 J l

l
1nz,- = 358.17 (1.587 + 7.497 - 2) = 388.25 kJ. (Ans.)
v, -4',i Dup. vr Since intercooling is perfect,
r _
= ,___l_ Or
D.p.
_
- v2
D'tt.P'
=
From the curue t-2'followirrf, the law ;
PlVt= Pr'Vr' or
Ratio of effective cylinder volumes
#=# =,'
pVt = C, we have
v. 1Ul3 _ Effective volume of L. P, cylinder _ x/4 x (O.09)2 x I x 0.g
u'r
P1V1r'3 = P'Y''l'e o' lP2 l vohr-"of HJ-ilEi- -nl+ffi'- =
Yz {.tl As the ratio ofeffective cylinder volumes is more than the ratio ofthe volumes obtained
p-v diagram, more air s supplied. to hgh pressure cylinder than can hold, therefore from
Btst for maximun effciency H.p. cytn-
der would such less air frotn intercooler tian receiurl
p2= ,fprh = .'[x3o = 5.48 bar intercooler will rise.
from L.P. cylincler. Thus pressure in the
vt It may be noted that the effective volume of L.P. cylinder neglecting cleaance is actual
tl I
v2
= (5.48)v1.3 - 3.7 u) volume x volumetric efficiency. It applies to the Z.P. cylinder only s theLr
is sucked. in this
cylinder.
AIR COMPRESSORS 1t9
718 INTERNAL COMBUSTION ENGINES
T'ixr$erlure at the end of first stage compression,
Example 20.21-: A single-acting two-sta.Ee air compressor
d.eols with 4 m3 / min of eir
f -t
under atrnospherc conditons of r.016 bar and Is'c wth a speed of 250 r.p.m. The deliuery
pressure is 78.65 bar. Assuming complete ntercooling find the mininum power required, by the
..2= r,, r,o)+ =zea, (P|l=#
\ 1.016/
- 444sK I where r, =
L
12
pt)
|

compressor and the bore and strohe of the conTprcssor. Assume d piston speed. of 3 m I s, nechani-
cal efficiency of 757o and uolumetric efficency of 80Vo per stage. Assurne the polytropic indzx of Work required =2 ' -!- x mR (T"- ?,) x --l-
n- I rlron.
compression in both the stages to be n = 1.25 and neglect clearance (N.U.)
1t6 1 I
Solution. In the Fig. 20.29. l-2 shows compression in L.P. cylinder, 2-3 shorvs cooling =2 x x 0.0819 x 287 (444.9- 288) x ::;,.:*-
befoe conpression in H.P. cylinder and 3-4 compression in H.P, cylinder. =ll:=
7.25 -r O.75 1000
= 49.17 k1{. (Ans.)
Example 2O.22.In a single-acting two-stq.ge reciprocatng air compressor 4.5 hg of air per
min. are compressed frorn 1.013 bor and. 15"C through d pressure ratio of9 to 1, Both stages have
the same pressure ratio, and the law of compresson and erpanson in both stages is pVt
s

-- constdnL If the intercooling is conplete, calculate :


(i) The ndicated' power
' (ii) The cylinder swept uolumes required.
Assume that the clearance uolutnes of both stages are 57o of their respectue swept uolumes
and that the compressor runs at 300 r.p.m.
Solution. Refer Fig. 20.30.

Fig.20.29
Piston speed = 2tN
Piston speed 3x 60
, ---j-
.. Stroke length, L= 0.36 m. (Ans)
2N 2x250 =
Volume handled =ttl4d2lxNxI,o
4 = r/4 d.2 x 0.36 x 250 x 0.8
Fig.20.30
4x4
.'. Diamete, ,=[ rx0.36x250x0.8 oru, m. (Ans.)

Mass handled by compressor,


f' = Amount of air compressed,
Suction conditions,
m -- 4.5.kg/min.
P = 1.013 bar, ? = L5 + 273= 288K

Pressure ratio, -ps --g


^ = #=H# = 4.e16 ke/min. = 0;081e ks/s.
P P
Ps- Pi
Intermediate pressure, pz= tfpspr = !qm6 x 1016 = 8.94 bar Also ... (Given)
{rri
ll
:

.i 720
INTERNAL COMBUSTION ENCINES AIR COMPRESSORS
,/ 721
Compression, expansion index, 1.8
=
Clearance volume in each stage / \ltn
= 57o of swept volume
Speed of the compressor, N = 800 .p.m. rl,-,=l*n-nlLl I
,] (i) Indicated power
\P )
i, l 3 v
il and since
P _P its G D is the same as for the low pressure
- stage and ,l.o & = & then 4,o,. is 0.934
as above. P Ps
P" P
... Swept Volume of H.p. stage,
Pi2 = P"x Po- P, x 9P" = 9P"2 \
frl 0.00408
:' P,=3p"-p".e., & =s v"c.p.) = 0gB4 = 0.004367 m3. (Ars.)

_l It may be noted thr / % ) .

Now using tbe equation, t/s = ltl- = (s)ri# sucrionremperaruresare,lt",#:::::,:::::y-flwvrt'v"',


lp"/ in the atio ofthe suction pressures,
:;,;:,;Ti:;:T:Ii:;::::::
LrEreue lrre swel
t T,=? x(g)0.31.3=289x(B)0.s/1,8 =B7LK
Now as n, rn and temperature difference are the same for
both stages, then the worh done i.e., v -Vrp.
O.013f
in each stage.is the sanxe.
Exampre zo.2s.A,:.:":i:;'":*:*:;,H^=;;::'1,,i';,,
Total work required per min. , -!-_l and temperature of air at the sucton 0", minute. rhe pressu,e
=Z mR (Ti - T") I-;;; The pressure at the d.eliuery
"r;
is 55 bar' The clearance in L.p. cytin.e)ilss-tlilalso
";;';;;lc-"""pe"ttu9tl.
in H.p. cyrinder is svo of the strohe.
Assuming perfect intercooling betieen
1.3 *" t o
=2 x i;--
._l
x 4,5 x 0.287 (gZ1 _ 288) = 929 kJlmin. the compressor at 210 r.p.m. "tlr,'nnd the minimum pouer required to run
929 -. If the strohes of both the cyrnders are equar to the d.iameter
of the L.p. cylirrter,
*i i"i* uolumes ; Law of compression find
Indicated power d'nmeters and strohes' what is the
= 60 = 16'48 kW. (Ans.) ltle lTtjo
expansion in both the cylinders ts s "r o.nrr re-
(jj) The cylinder ewept volumes required nVt =-;r;;;;;;.-'"-'
: , Solution. Refer Fig. 20.31.
Free ai delivered, Vt = 2.2 ms/min, p1G p) I bal, Tt 2s + 27J
The mass induced per cycle, - = = 0.015 kglcycle. = = = 2gg r< .
# 55 bar ; N = 210 r.p.m. ; Law ofcompression and
This mass is passed thrugh each stage in turn -ll= constant.
pVl3
expansion ;
Fo the L.P. pressure cylinder (Fig. 20.30) =
Cleaance in each of L.p. and H.p.
cylinders = 57o of the stroke
vL-v4=T = r+#fr#E =o.ot22ms/cycre
Minimum power required to run the compression, p
For two-stage corpressor (with perfect intercaiog
:
work done is given by,
f

'tuol - r,v^ =r+- o (*)"' = 1 + o.o5 - o.os (s)vls w= J+,,- ,]


= 0.934 =*="*J
[*)*
(-
2n
= ---- mRT" ('.
:.
4u".

v's =
= 0.1' ,4
vt -V
- \,ot. =
Vt-V o.ol22
- m3a = 0.934 = 0.0131 m3/cycle
n -l

ptVt 7x705 x2.2


frrr'] PrVt= n7r1

i.u., But
Swept volume of L.P, cylindr
%rr..p.r = 0.0l3l mB. (Ans.) RT, - 287 x298 =z o/ Kg/mrn
For the high pressure sage, a mass.oi0.15 kg/cycle is drawn
in at 1b"C and a pressure of
P;=3x1.013=3.039bar 2x1.3
w= (1j_1) x2.57 x287 x29B
.e., volume drawn in
0'015 3q1289
= = 0.00408 ms/cycle
8.039 x t0
1119919 Nm/min.
I

INTERNAL COMBUSI]ON ENCINES 723


I
AIR COMPRESSORS
,l
14 DsL.p. = 72822'7 t ', Lup.= 2".". = D.p) (Given) i

(::zezz.\x+Ya ;

,r.". = = zs.az cm. (Ans)


rd
7 6 lff ) l

Lt.p.= Lu.P.= 25'31 cm' (Ans')

N
\ rlmr.(L.P.) = 1luol.fn.P.l

\ D2tp.Pt= D2g.p.P

D't.p.= e# P1 = P")

r.* cm. (Ans.)


u"
(H
L-
P.)
",, =
l+') = ;
I


Vl
Ratio of cYlinder volrrmes, : I
v" i
\: 1
As the points 1 and 5 are on isothermal line we have
I

PyYl= P"Vu
(where V, and Vu are the clinder volumes of L P' and I{ P')

P.)----{ Yt _ ps lL
-l v"tr-.el
l*- vs(1. ''-' (to".)
Ys- pt -= ps =U1 =2..
Fig.20.31 Example20.24.Atwo-stogednuble.actingorcompressor,operatingat.220r.p.n.takes
n air at 7.0 bar ond,27"C. Thze of the L.P. cylnd.er is 360 x 400 mm;
the strohe of H'P'
s 4Vo' The
... p l=tt'rn-t='- = 18.66 kw.
= 60 (Ans.) i in, tarne as that if ihe L.P. qlnder an the clearan"e of both the cv,linders
x 1000 "il"*
.'1.- ,ili", ais"harges the ir at o pri"rur" of 4,0 bal. The air passes throug^h the,intercooler so
the H.p. cylnd.er at 27iC and.3.80 bar, finally it is d'ischarged' from
the conapressor
Diameters and strokes : thoti
"nur"
atls.2bar.Theualveofninboththecylind'ersisl'3,cr=1'0035kJlhgKandR=0'287hJlkC
(p, = Intermediate Pressure)
We know that Pt= ,llqd K,
= .[ x 55 = 7.4bar Calculate : (i) The heat rejected in the intercooler ;
() Diameter of H'P' cYlind'er ;
The volumetric effrciency of L.P. cylinder is given by
() The pwer required' to d'rive H'P' cylind'er'

r+ft-l?.
( o,\un (pz= p) Solution. Refer Fig. 20.32.
nuor.0)=
l-J Speed of comPressor, JV = 220 r'P'm'
p1 = 1.0 bar, pu (= ps) = 3.8 bar, pz (= pr) = 4'0,bar, pu? P) = 15'2 ba
/d , \VL3
= 1+0.05-0.05 l+l =o.817 or8r.79o Tt=27 + 2?3 = 300 K,Ts=27 * 273 = 300 I!
\r,/
Clearance of L.P. and H,P, cylinders = 47o
The volumetric effrciency of the H.P. cylinder is same, because Value of''for both the cylinders = 1'3
Pd P ano %ts.P.l Valr'l
^-r i;- cp = 1.0035 kJ/kg K ; R = 0.287 kJ/hg K
- ,;-
=
\
I Ys(H
Ps Y(LP P') ) Diameter of L'P' cylinder' D"'" = 360 mm = 0 36 m

Free air or air at suction condition delivered per minute is given by, .Stroke of L.P. rylinder, L".t. = 400 mm = 0'4 m
Free air = (\- v) x r'P'm' Swept volume of L.P. cYlinder/min
(220 x 2) = rl4 x 0.362 x O'4 x (220 x 2)
= %o,.p.l luol.(L.P.) x r'p'm' = %<l.p.l x 0'817
x 210 = 2'2 = t!4 D2up. x Lr,.p. t
= 1?.91 mS/min ..... since compressor is
double-acting
2.2 ^ 2.2xL06
%rr.p.r = fffi -r = offi = 72822] crrr3 Volumetric effrciency referred to condition at 1
t :Un ,.^rUr.3
r/4 D2".r. Lt.r. = 12822'7
1. ft - = 1+ o.o4-tr, (.#.J
=
[ff,J
INTERNAL COMBUSTION
ENGINES AIR COMPRESSORS

.'. Swept volume of H.P. cylinder = = 4.71 ms/min


ffi
/4 DzH.p. x trr.p. x (2 x 22O) = 4.7I
/4 DzH.p. x. 0.4 x (2 x 220) = 4.7t t
... Lu.y. = Lr*. = 0.4 ml
- = ( 4.7rx4 )"
DH" ($ven)
"' ["*nfui = 0'1816 m or 184'6 mm
Diameter of H.P. cylnder = 184.6 nr. (Ans,)
(ili) Power required o drive H.p. cylinder :
Since the initial temperature and pressure atio in the L.P. and the H.p. cylinders are the
same, Tu = T,
.'. Power required for H.p. cylinder

=. \ mRer- ?,) = -l:q- " +# x 0.287 (4rB _ 800)


45 kW. (Ans.)
Example 20-25. A sngle-acng tuo-stage ar compressor d,eliuers air at 18 bar, The tem-
P!.ralure and pressure of the air before the compression ii L.p. cytinder are 25"c and, I bar. The
d.ischarge pre.ssure of L.P. cytinder is 4.2 bor. bh" prrrrur" of alr reaving the intercoorer
is 4 bar
and the air is cooled to 2s"c. The diameter and.'stroke of L.p. cylindlr ae 40 cm and s0 cm
respectiuely. The clearance uolume is Svo sttoke in both cylinders. -The
v (m1 speed, of the compressor rc
200 r.p.m, Assuming the index of compression and. re-erpansion in boti cllind.ers
as 1.2s, c, for
Fv.20.32 air = 1.004 hJlkg K, find :
(i) Power required, to run the compressor, and
= 0.92s8 (or 92.BBvo) | r=+=#=',,] (ii) Heat rejected in intercooler / min.
.'. Volume of air drawn, eferred L Solution. Fig. 20.33 (a and b) shows the process on p_V and ?,s diagrams
to condition at I Giuen : p" = 1 bar, p, = 4.2 bar, pi = 4.0 bar,
= 0.9238 x l?.91 - 16.54 ma/min p = 18 bar, Iu = Tt= 25 + 279 = 29BK
Thus, mass ofairlmin, 1.0x105x16.s4 Dup. = 0.4 m, .Lr." = Q.$
^= #-
nr, - o.zez*s-t;l't = 19.21 kslmin N = 200 r.p.m. , % = 5% ofstroke in both the cylinders, and
_l 1'004 kJ&g K
. n
", =
Also, Tz= Ttx lltln
pl)
="' (fi# = 413 K
- 't
= 'l-"D"t.v.Lt.t.= - x O.42 x 0.5 = 0.0628 m3
vs(1.P.)

(l) Heat rejected in the intercooler The volumetric effrciency of L.p. cylinder is given by,
:
Heat rejected in the intecooler
/ \lln
= mcr(T, - T) = 19.27 x 1.0035 (41g _ 800)
lluor. (1.p.) = | +k -k
kJlmin' (Ans.) l!L)
0r) Diamerer or H.p. .",,;;tlt* =r+0.05 -o.os(!z)""' lu=I'=o.ou)
Volume of air dawn in H.p.
cylinde per minute \1,/ r.%)
= 0.892 or B9.ZVo
V"= ^RTu _ t9.2tx1.287xS00x10a .. (q - lr,.e.r x V", = 0.892 x 0.0628 = 0.056 m3
V) = \,"t

., ^sn"9
the pressure *::1":i::y::::""-r",,"n
efficiencv of both
or!T#. o,* !rjll)"7,'l),"rro, ,"^,,
Considerinc tF,e compression curue l-2
:::;:{:::;,:::.uotumetrc "vfr;;';;;""11'ro^" referrert to"=condition at the start or T / .n-1
'2 Ipil
r' __

l&J
n
i:
ii

INTERNAL COMBUSTION ENGINES


t.t ll
AIR COMPRESSORS tl
II
ii
n -1 li
T, = r, pi\; ( 4'2.I L% = sez K lt
= 2e8 f
" '[p"] \1/ ii
li
It/ n-r
'
I
|

w,rr.y'cycte = {1 ^*r, ll'l -rl


L\o"/
| -1
I
|

I4/,,,.o.y'cycle = f1 ^nfu ll.'l--'l


l\rii l

|n=Massofairofvolume(V1-V)atsuctionconditionofL,P.cvlinder]
Asrs=r\tsi"""ll .. ,"= t;t?'Jr!r:t. =0.o655kg/srroke l
.'. Total rvork dorr"/"ycle, W= Wt".".l + W1n.r.l
[ -].' n-1

= n-L
J- mtf, -rl
(a)
L[t)-.[#)- f
l
L25-1 1.25-I

# . ,0.0655 x 287 x2e'l(lz\rF


L.zs-r "" l\ t / -l*)- -rl
L.J \4.0/ |

= 28009'8 x (1'332 + 1'351 - 2) = 19130'7 Nm


(i) Power required to run thecompeesor
W xN(r.P'm). --- l$l$Q7 v 2oo kw.
t *. = ffi'ioooo kw = 60;;
= 63.77 (Ans.)

(ii) Hear rejecred t:


ar:T::: , (rz_ rt)
=
(0.0655 x 200) x 1.004 (397 - 298)
= 1302.08 kJ/min.
(Ans.)
sExample 20.28, A sngle-actng two-strye compressor with complete intercooling deliu-
and
ers 10.5 hglmin of air at 16 bar. Thc suction occurs at 1 bar and' 27"c, The cornpression
expanslon processes are reuersible, polytropic ind* n = 1'3' Colculate :
(i) The power required to drve the compressor
(ii) The isothermol effciencY
QiD The free air deliuery
(iu) The heat transferred, in ntercaler
The comPressor runs at 440 r-P.m.
(u) If the clearance rotios for L.P. and, H.P. qlinilers are 0.04 and, 0.06 respectvely, calcu'
Iate the swept and clearance uolums for each cylird'er'
() Solution. Giuen : p1= 1.0 bar, pz= .Obar, P3 = 16'0 bar
Fig.20.33 T, -- 27 + 273 = 300 I{' n = 1.3,
INTERNAL COMBUSTON ENCINES
AIR COMPRESSORS

41.77
Isothermal efficiency = 0'8488 or 84.88%. (Ans.)
Ps = 16.0 4g.lzr =
(iii) Free air delivery (F.A.D.) :

Free air delivery, mRT 105 x 0.287 x 300 x 103

. pr 1x100 = 9.04 m3/min. (Ans.)


(iu) IIea transfenred in irtercooler: \
Temperature at the end ofcompression

r- \+
T2=\xl!21"
Pz= 4.0
- \A/ =300 flli#
\l/
= 4r3 K
Ifeat transfered in intercooler
- fn x cp x (Tz-.T) =mx cp (?2- T) t... Ts= Tl
10.5
= -6t r 1.005 (413 - 300) = 19.87 klV. (Ans.)
ir
(u) Swept and clearance volumes :
il
Pr = 1.0 Volumetric efficiency for L.p. stage,

u .l

+---
vc(L.P) v.(L.p) ____{ tuor(L.p.)= I + &- ol?1" = I + 0.04-0.04(4)vr.i = 0.92g8 orgz.BSvo
vc(H.P) +lf l.-- vs(H.p)_--+l Snilarly volumetric efFrciency of H.p. stage,
\Prl
4m. (H.pj = I + 0.06 - 0.06(4)v13 = 0.885? or 88.57Vo
Fig.20.34 L.P. Stage :
Amount of air delivered 10.8 kg/min
=
Clearance ratio for H;p. cinder, ,t (j) Swept volume, \rY(L.P.) = /r,
(Yr
Free air delivery
=.0.04 - YsJ = Sp*d;rt
Clearance ratio for H.p. cylinder, A
= 0.06
,r;
The pressure ratio per stage 9.04
= fip, = f * fO = 440,. O92g8 = O.O222 m3.
= (Ans.)
(l) Power required : (ii) Clearance volume,
Work done in two stages with perfect ntercooling % r"".1 = 0.04 (vr - Vr) or 0.04 y"(l.p.)

= 0.04 x O.O222 = 0.0OOB8B ms. (Ans.)


%t m,rr|[lo)#
=n_7 H.P. Stage :
'llrj -rl_,] (i) Swept volume
% rr.p.r =Us-v)
_ Fee air delivey
- Stage p""."o". ."tio'i.p""d ><ll-lr")
2x1.sx to.5
_ x 0.287,300 l. .S+ I
1.3_t ;; f
(16)r,rr _ll = 49.21 kW. (Ans.)
(jj) Isbthermal efficiency , t l _ 9.04
4x440x0.8857 = o'0058 m3' (Ans')
(il) C.".rn". volume, % crp.l = 0.06 x (Vu _ Vr) or 0.06 ys(H p.)
Isothermal work = *nfrn, (4\
"""|.,'./ = 0.06 x 0.00b8 = 0.0OOB4B mB. (Ans.)
Example 2o.27. The,pressure rimits of o s-stage compressor are
10.5 /rc\ 1.0s bar and, 40 bar, The
x1}o" supplies-s m3 of air per tnnute. The rowlf cornpression pvt.2s
I = q.tzkril. (Ans.)
= 0.287
G0' los" lf ::ompressor
Iateon one minute basis :
is = constqnt. carcu-
(i) Indicated u'orh done assunng
cond,itions to be those for maxirnum efficiency ;
(i) Isotlermal uorh between the
same pressure limits ;
AIR COMPRESSORS
730

(iii) Isothermal efficiency ;


(iu) Indcated uorh if the ndthine were of one-stage olll! ;
p4=36
(u) Percentage sauing in worh done to using three stages instead, of one.
Solution,
l. .n-r I
done/min. j\ p,v,llult"
n-r - -l\p") -tl
(i) work =
|
LJ
3x1.25 l.ll 4o .r.2s-r I
l"i.zs-r.l
, - -l\r.05/
x1.05x10Dx3 ll.:---l
r.z5-1. |
L]
-;.
= 1297725.7 Nm./min. (Ans.)

(ii) Isothermal work done/min = 105 p,V, log. *


, 10s x 1.05 x 3 x los- 4 = 11a6628.1
*'L05 Nm, (Ans.)
^ -l "-n
yt-
(iii) Isothermal efficiency = +93+ = 0.88:t or 88.37o. (Ans.)
1297725.7
(iu) Single-stage, work done/min. Fig.20.35
f -1 1

llo)- -'l
= f r,v,
L\P"i l Now rr=
r
".\+':
r'l7.) lwhere = no' of stages = 3]

lL25-l\ I
Lzr' J"t
1.25
,*.,1[#)* r-z = Boo l3ol. = B8o.e K ii.
= LzS _l
x 1.05 x

= 1686780.2 Nm. (Ans.)


i Mass of air handled per
Tiil
l. 1 /

,.* , ,u
,.0 ,. 17 .42 kermrt.
[1
llt
ill
= -" = o28fooox3oo =
(u)
L686780.2 -1297725.7
(in kJ/s)
ll
7o Work saved = 16362802 = 0.23 or 23Vo. (Axts.) Total work done in tbree stages
Example 20,28. A S-stage compressor is used to cornpress air frorn 1.0 bar to 36 bar, The
r- |
i

compresson in all stages follows the law pyt'zs = C. The ternperature of air ot the inlet of com- =l;^.mR(r2-41 " "
pressor s 300 K. Neglecting the clearance and. ossuming perfect intercooling, find out the ind,i-
cated power required, in kW to d.eliuer 15 m3 of ar per minute masured, at inlet cond,itions ond, _i ,,ru ,rr.n xo.28?x(380.9-soolr "
s
intermedicte pressures also. Tahe R = 0.287 kJlkg K
Solution. Glue: pz= 1.0 bar, pn = 36 ba, z =1.25,R = 0.287 kJ&gK = lii11;?,, *u,oo,,,
101'11 kW' (Ans')
n*
,i ?r=3ooK .e., Indicated power required =
As there is perfect intercoolng, gExample2o.2g.AS.staged'ouble.octingco.mpressor,operatingat200r'p.m.tahesin
cylltdz,r size,,is 350 nm x 400 rym,The
intermed'iate
air at 1.0 bar a,nd n"c-. rir"lo*'ptressure
tz = ls Pn
=l*)" lrylu' = r.ro, pressure cylnd'er and' tne nie;:'"."'7.t7"1'1 !1Y :::"::*:;!"!;X:r'!:"i":"'!tiT.7''|i;
Pt Pz = ps \Pr,/. =
\1./ intercooler
.'. Intermediate pressures, 'n"*":"w;:;:'"":;:,\7"!::!';L"':::;^Tifi";;",;;;;"'.te.nte:dture,in.the
pi"'*",of o'z,bo' " of the interc'oolers' The clearance
p2 = 3.302 pl= 3.302 x 1 = 3.302 bar. (Ans.) after each stage and tt"'" ':'a";;;p";i "i"n
th";;;P;,ts'ion ind'ices areand l'2' 1'25
uiolume in each cylinder r,
i"*oin"'ioohe uolu.me,but
p = 3.302 p2-- 3.302 x 3.302 = 10.9 bar. (Ans.)
I

732
'l
AIR COMPRESSORS 733
I

l;it'i{;';"::;,7:r1:::::},:"^":!,"{,reIs j,2n.dand-#;'""]l)l,il"",""",'):
j,"fl*;:r:;t:,,:";#,;lii".r"y";l::;r:#,-r:*x:::
"u*,
ture. ' -J-ctrea m ihc after-coorer if thz deriuere ""i ir""ii"rir,o,
,"^o"ro_
di.ameter of the intermed.iate
.(!.i:).The pressure an ht'gh pressure stage qtlindcrs.
Gii)The tnoft w*), required.
to d.riue ro"
Tahe co = 1.005 hJlhg K. - ssr with mechanical efftcicncy
"o*or",!,'he of gTLo.
solution' Fig. 20'36 tlil *o (I
shows the p-Iland ?_s
The swept volume of low pressure diagrams.
dinder per minute.
vs'p,=
:::::,::::;r,,,, .. -^ m8/min.
...--
t = 15.894
[r.p.m. is multiplied by 2
since the comr
compressor is doubte-actinsl
bar and 20.C are: ""t"-"'i;.:ffi;:T:it)
I:i:::t:::"T"t""r
I to 1
]l
't: ,al
4oor.osrstaee) = I + - f = I + 0.04 - (4\n
i
l lff)' 0.04
(T/
= 0.918 or 9t.B%

Tluor (2nd
= 1 + 0.04 -
stase) o.o+
fZo)rre = r.o4 - o.o,
)
ll9)c
\.3.8/
= 0.9186 o, n.ee;"Pu

l'luor. (s
= 1 + 0.04 - O.O+ ffrOl*
sg

)
=104-'* [#)ti'
= 0.9227 o, s2.zz)"P"
Volume of air taken in at 1
ba 20"C
(v, - v)/mn = Va,r.".y'min. X ruor (rs
sree)
= 15.394 x 0.913 = f.OS ms/rnin.
And mass of air/min.,
pt(Vt-Vt) 1x105x14.05
^_ - = 16.208 kgimin. 20 + 273

Arso Irl= = 293

,=r,l\)" =ZeB
[i)- = s6eK

+=,,(+)T =rsr flgl##


(38/ =Beo6K
s (kJ/kg K)
l.::-jrar "
rto=rgi#j =*t (rrrg)--u' =noou* (b)

Fig.20.36
INTERNAL COMBUSTION ENGINES
AIR COMPRESSORS
(j) Hea rejected in each cooler :
Heat rejected in intecooler after lst stage : ftq
I rnnlffix@60-2ss)+:0.25
1'25(Bgo.e

= mcp (Tz- ?u) = 16.708 x 1.00b (869 _ 298)


= u*r* o.ir
-2g3)+Px(o'a-293i J

= t276.lG kJ/min. (Ans.) 16.708 x 0.287


* nr, + 483.6) = 142.6 kW
And heat rejected in intercooler after 2nd. stage = --Go.a *UU

= mco(76 - Tr) = 16.708 x 1.005 x (890.6 _ 299) i.e., Shaft Power = 142.8 kW. (Ans')
Heat rejected. d'uring cornpressinn process d'uring a stage aboue *ory \o! be confused
with
= 1638.85 kJlmin. (Ans.)
And heat rejected in the after-caoler heat rejected. |er stoge, wich iictuds heat transferred during suction and, d'eliuery, if any.'Thus
heat rejected during compression process 1-2'
= mco(Tn- f1) = 16.?08 x 1.005 (404.6 _ 293)

(tr) Dr.p. i Dn.p. ,


= I8ZS.9S kJ/min. , (Ans,) = l",lt+),a
L \1- r/
' -
",rlJ
, -
Volume drawn in intermediate pressure cylinder/min. co = c, + R, :' cD= cp - l? = 1'005 - 0'287 = 0'718
c- f .005
(tt r/v8rlmrn
(v5- \/_:_ = ^ryk _
=
16.?08 x 0.282 x 298 x 103 ^t=;=-=r'4
; 3;i05- =3-69m3/min.
.'. Swept volume of intermediate cylinder/min Therefore, heat transfer

V"(i,t"r 3'69 3'69


=lo.rrr(t'.n , t:') tru, - rrr,'l x 16.?08 = 45s.86 k.r/min. (Ans.)
"i"cy'min
= ;_
,,,o.;;; = ,r", = 4.039 m8/min. | \r'4-r) I
Sinilarly fo 2nd stage-comp"""sion process 5-6
X orr., L x
(r.p.m)x 2 = 4.089 t)trno.u
Heat transfer - zssi] . ro.zos = 1te.07 kr/min. (Ars.)
r.t" It i;1?
=

'' urP = I a.oso'4 y' | .oss* fu, Similarly for 3rd stage compression process 9-10
=
L;xL;G.p.^-)x2l L"r04rro0"rj =0.17em or 179 mm r" - 2e3)] * 16.708 = BB4.6e kr/nin. (Ans.)
L'. r Dr.p. = 179 mm. (Ans.) Heat transfer =
[f,f),non.u
Volume drawn'in high pressure cylinder/min ir
Example 20.30. A mult-stage oir compressor is to be designed to eleuate the pressure
ltr mRTs 16.70g x 0.28? x 298 x 10s fron 1 bar ti l2S bar such that strye pressure ra.tio wiII not exceed 4. Determine :
\vs- vllimln = -* -
= 0.ggg m3/min. (i) Number of stages (ii) Eract stage-pressure ra,tlos
,'. Swept volume of high pressure cylinder, (iii) I nt er ne diat e pre ss ure s.
0'889 Solution. (i) Number of stages' x :
{tto'/min = ,l,o,.r*l*J = =0.9364m3/min. Assuming perfect intercooling, ihe condition for minirnum work of compression in multi-
-0889 n stage compression is
"t
Or".r. L x (r.p.m.)x 2 = 0.9g64
(p)+r f(p),*rlt'
I (pr, = L t-p), J
Du" =
I o.sseax4 )r, (P)' * r (p)" * i . t25
" lffi] = o'086s m or 86'3 mm rn tnts case
l!f,-
is restricted to 4 and (p)r =1 = r25
1.e., DH.p. = 86.9 mm. (Ans.)
Hence 4 = (125\uz
(ii) Shaft power 3

Shaft power is given by I


: log,4= - log"L25
f ro
("r0 I 1

L#^*(r2-r)+ ffi^*rru-"u)* r*1-n -re)l *


60 r rl*- 1.399= -x x4.828

,t

'lil
d
*
1
INTERNAL COMBUSTION
ENGINES
AIR coMPREssoRs
... 4.828
= ** = 3'48 sav 4
Hence the number \." r

", J
P1= p, x 4.25 = 1,0 x 4.28 4.25 bar
n' ,*".,
() Exacr ;;;;;":"ft; ]
I

TIT"T,T
=
Again using the elation Also
;=; =t'=...
tB +
lt_j
(p)+r
ir
l{il,*rlv, (t.)<\v1
(p), = l-;l.-l 0r $ _ t,1, \_1
=ll=lI
= n- - \-rio,
ll' (ai).rnrermedirr"o"lir"""L,to'J i =3343'
= (As')
i

rrxtreme pressures arc and d (t25+275) S98


already fixed ',"tio= = Erd = Lzzrs
il
lfilfif
Pt= lbar,po+r=p5=125ba
A]so
, ,
\p)+t ps
t6o *:l?1
(L2t7)\'o' r, or
(p),t = =us*t .25 = 8?.68 = (1.271b)sr
log" 37.65 = 5x x log" 7.2715
o._ Ps _
'n - s.s+s =
L25 3.628=5rx0.24
ffiA = s7'39 ba. (Ans.)
3.628
Similarly p" r=
= l;
ottr = 8.02 o say B
''tns Hence, numberofstages = B + I = 4. (Ans.)
i .. p.=;*==.rug_
3.343 - = 11.18 bar. (Ans.)
(i) Work done per
kg of air:
Pressure ratio in lst stage 4.25
3.84g =
ard p^= Pt -= 11.18 pressure ratio in too't/3
Fxampte 20.3I.
3 313 t34t : 3'344 bar' (Ans') the follow;-- ^.^-^
, ,0" -^^.'.,'-
::::,::'^::*;rxi"*i;!::"":i1!i;ir:;:::r::iyir^#.i,,;:i::;,":ff
remperature reaving 2nd, ,,J":rl; ,tI;?= ,*'-l;J::r, K (Given)
temperature cooling
brt"*-;o:; ;;:i^.?."""atso in the I n- I
catcutate , (i
t,>,^'-!-"!,o "o^p'"'-'io?"iJ,'ii,'J7'["f,i:;":!:::r::,7::"j,,:!,:;::"ffi
o' staa:s required'
Work done/kg in tsr srage = -+
n-LRT lfp, "ll/1;- _,
^l

wo'n1l' per hg of air' and LJ


(ii) Heatrnput
rahe R = o2y u.r,ol!tJ"':o '::!".intercooters.
so.,to,,. i-ui, ""
,J=
=
#i x o28z x zoa
frazsr$i _rl
L I
= ,rs s r.",
o^,,
,i:r,;.1"!l!,y=*, Wok done in the following , ;;' -)
4 ,"mpeature
=ii * rzr= ze8 r!
*::::: pressure from lst
:
=er !25 f
xo.2BT:*3t3 {a.asr-ms
r.zs-r I
ii : :"',tt""t stage = eny pessure
after everv stage'
l stage,
to 2nd f -rl =soe.ak
o' ==;'"""J"Y:::f:'"t:""
p^ressure work done/kg LJ
from (N + l)rh stase, = 143.b + 368.6 = 5t2.1 kJlkg.
_= ::,:* of stages after the
_Total
(ii) Hss rejected in the intercoolers
(Ans.)
,' , l-1-0"" lst stage, and :
I emperature ratio Heat rejected in the intecoolers
(i) Number * .;lt for 2nd stage, # ,r; ..... n th stage.
= 3cp (398 - 313)
=3x0.997x8b I"'
Thus, a -lqj
- 1",);- = 254.23kJ/kg. (Ans.) "r:"0';:
A rExample 2o's2' A.J'age
air compressor supplying air brast
= o.ss7 kJ/l<s) 0.287

rated capacity of 10.5 ms of for an o, engine has a


_ (tzs*zzs\#
_t_l aoe)5
pressure at the suction to L.p.,free,air/ntin, and, ts ortrnn t, he nain engi.re at 100 r.p.m. The
i" i b"; ;;;';; deriuery f|om H.p. cyrinder is g5 bar.
cvrncrer
\25+27s ) =
l) = a'zs fractional clearances are 0.04 o.oz o, .'1. ;;; ;;;, cvrind. rs.
"."a t bi:c'.".,Jriii'ru*cooring, Assuming the temperature .t
the end of suction in art cvlinders
TJte

t stage pressures r.n geonLetric


l
j

#,_
738 INTERNAL COMBUSTTON ENGINES CoMPRESSORS IS9
pfogr8sion and the law of compression pV1.25 constant, calculate the swept volume
= of each Swept volume of each cylinder
cylinder. The free ar conditions are 1.015 bar and, IS.C.
Volume of free air reduced to suction condition of LP. cylinder is given by
solution. Refer Fig. 20.32. since the pressures are in geometric progression (given),
PIt - Poor.Vo^b.
.P =Pz T1
=Pt = z (Pressure ratio). .To b.
Pz Pt Ps
,,= *E**;,n
Pt = zF" p = Delivery pressure (H.P. cylinder)
Piz= Pg = z2P" pz = Intermediate pressure-2 _ 1.013 x 105 x 10.5 x (273 + 25)
P= Pf = z3P" pr = Intermediate pressure-1 (273+15)x1x105 = 11.0 m3/min.
r lv3 ps = Suction pressure (L.P. cylinder) 110
.. -
" = l!l
Swept capacity ofL.P. cylinder
fl@.(L.P)x r. p. m.
\Ps./
1r n
I )utss = o.nfl-r* = 0.1215 m3. (Ans.)
-\r./
Again volume offree air reduced to suction conditions ofI.P. cylinder
1.013x105x10.5x298
= ,88 436 10t- =
2.41 m3/min
" "
Swept capacity of I.P. cylinder
tL1
P= 95 = (Ans,)
, =0.O89m3/min.
"
Again volume of free air reduced to suction conditions of H.P. cylinder
1.013x105x10.5x298
= ,s8; r'j;F-- =.0.529'ml/min
p2 = 2O.8 Swept capacity of H.P. cylinder
0529
= 0SS4 ,100 = 0,(X)634 m. (Ans,)
Example 2O.33. Show that the heat rejected, per stage per hg of air in a reciprocatng
compressor with perfect intercooling is giuen by
Pir = 4.56 lr-"\1.-
Ic, + cul-\n-rll
r-+ ll (?]2- r)
I
L
-
where (7:, - f) = Tr^piroturc rise d.urng compression,
n = Polytropc nd'e*,
| = Ailiabati'c in'dex, and.
Fs=1.
co c, = Tltso sPecift'c heats of oir.
Solution. In a compressor heat is rejected in two stages :
(i) During compressinn when beat is rejected to the cylinder walls ;
\ Fis.20.3T
(ti) D uring intercooling.
' P= zps = 4.56 x 1 = 4.56 bar
Heat rejected dwring compression
Pz= zPt = 4.56 x 4'56 *20.8 bar
f -n ^ -n
( p2r2- p1u1\
luol(L.p.) ! +k - 1u" = l,L tcompressonworK= y_l
= | "L l
= 1 + 0.04 - 0.04 (4.5G)v1.25 = 0.905 ot 9O.5/o
Simiiarly, tlor (L.p.) = Imr (H.p.) = 1 + 0'07 -0.07 (4.56)ut 25 = 0.834 or
, -n (T, -T,\
83.44o = Y- l;-1 )R"""Perkgofair

h__
l

aA^
INTERNAL COMBUSTION
ENGINES

t",r,., il"rioTf,il"illercoolin8 the temperature of ai after compression ?,


must be educed to
Heat rejected in intercooling = 105 x 6.3 (0.000198 - 0.000018)

= co (7,
... Total heat r'ejectud per kg Q
-
of air
..... per kg of air *F 105 (6.8 x 0.000198 - 2.7rx 0.000329)

T-N
f-rl ll
R
- 105 x 1.013 (0.000379 - 0.000036)

r-t - Q2-T)+ co(T2-


f ^ t
r,=lc,n
.-2-'!=Lwe R
-7_1|."_l
fv-n\'1 (?:2_Tt) _ 105 (2.4e4 x 0.000018 - 1!E !.ggoOgq
, \r (1.3 - 1)

=1,. rt=)] (r2- r1).Proved.


113.4 + 73.44
149.29 x 300
- 34.74 - 2.808 = 149.29 Nm
Example 2O.84, ?he cytin kW. (Ans.)
+. T!:
r!3 sulely -u'-'"*'' ut aIL Atr motor" has
ju:!,":0"1::,::'i:::::_: a bore
bore,of
of 6.35 60,1000 = O.746
suppty p""",u,"-i,
on"rr- _ -- "." va,, ,r.e suppry temperature 6.ss cm and
and, a strohe ol
or
1';1,:fbar. The
t.ulr
1.01J rhe ctearance,.'r.'i
ctearoice,ol rerature.24",
24"C, and, exhaust pressure Air supplied per minute :
,?tuf""l;r,!: pressure is
compressed ty tn" ,"tu,nnTl;:: "::::r!.1: ""a ",1i,",
,1" g"-"n","iiJ"*i.s.
is
mass induced per cycle is given by (rnr- rnr). It is necessary to determine the tempero-
rn, o * ture of-The
::J:":,:,",:"0:^!:;:::f
":#:;::";;:i; "?;:t:::-'!j_i.1!,:^,"1"o
:i",:ii::r!::::y^;:il#",X";;;'J;i':', The law of
air at 4, which can be tahen as equal to tht at-3. It is assumed that the air in the cylincler
at the point 2 expands isentropically to the exhaust pressure.
""o'o"o;f ;::;;r;:,il!*;,1":;i::,:,'zx!,!;ii;,"r;,::::#;f,!:j,jl
::;:i:::::"j:^;^i'"1i,irl"::':i';;i'#:;:: r-l

::i"::::.'{:,!,;,:;?::*o,";;',;;;;;;;:;'l#,,hrlhsK
Solution. Refer Fig. ZO.ia = l4t) ' - rnn.n t'ots\ont'n -, r.84.4 K
""" ",- \p2) \2.77 )
Swept volume x(0'069!)2x0.1r.4
- p^ xV^ 1.013x105x0.000036
=0.000361 m3 h_-_

= 0.0000689 kg
Clearance volume RTt 287 x184.4
= Ve = Va = 0.05 x 0.000861 0.000018
= ms
0.000361 ptVt 6.3x105x0.000198
"t = ----- + 0.000018 = 0.000198 mg Also, *r= n?t =
287 x297 = 0.001463 kg
= 0.000361 + 0.000018 0.000829 .. Induced mass/cycle = (0.001a69 - 0.0000689) kg
_ _Y, = n3
= 0.0S x 0.000861 _ 0.000018 .'. Mass of air suppliemin = (0.001463 - 0.0000689) x 900 = 0.418 kg/min.
Il m3 (Ans.)
= 0.00001S + 0.000018 0.000036
_% = mg
P'Yt" = P'V'n 20.4. ROTARY COMPRESSORS
o Rotary compressors are the machines whch d,euelop pressure and haue a rotor as
" p,= o,(3)" = u., [ooog]e8)" their primary element when compared with the piston slkling mechanisn of the recip-
= 2'71 ba rocating conpressor.
Temperarure
", ,n. ""0 .r!lol.""r;;,roo*/ c Wheneuer large quantties of air or gas are required at relatiuely low pressure rotary
compressors are employed.
r,= r,l#l = 2ez fglgqes)o'
I v \n-l

."., remperature n" 20,4.1. Classification


"ft"" "*p"l.ion' ::;i^ ,, = _|ffi{*;
Indicated power .:nn The rotary compressors arc classified as follows :
of the motor : l. Displacement (positue) compressors :
(i) Roots blower
o,= o^ (ft) = ,o,, o.ooooeo )r3
I ooooo18.l =
2'4e4 bar
(ti) Sliding vane compressor
Work done per cycle (iii) Lysholm compressor.
= Aea 1284561
(iu) Scew comprcssor.
i'"., Wok done
=p r(vt - v) *
g-1{il 2. Steady-flow (or Non-positiue displacement) corLprcssors
- ps(vs
- r,t - (eXUr1 (i) Centrifugal (o radial) compressor
:

(ii) Axial flow compressor.


'

INTERNAL COMBUSTTON ENGTNES


AIR COMPRESSORS
20.4.2. Disptacement Compressors
'Displacement compressors' are those compressors
trapped in the reduc.ed space -in which air is compressed. by being
ii
formed blt two sets surfaces.
20.4.2.1. Roots Blower ""g"g*g
sh^own in Fig.20.38, bur three and four tobe i
nrgner pressure ratios. one of the rotors is connected versions are in use for lrreversibleoressure
",_r--11:^:::^llfl,tw1is to the drive and the
drivenfrornthefirst Inthiswaytheroto^...ot"t"inprrlseandtheprofileofthelobesisofcycloidal i. g"u" 'r" rise due to ack
"""orrJ"oro. 4 flow from receiver
or involute fo'n giving co*ect making of th" lob"tl;:;;t the delivery side from
sealing corrtinues untill delivery commences. There the inlet side. This
must be some clearance between the lobes,
and between the casin-g and the lobes, t"
""d";;;;;;Jhi"
has an increasingly adverse effect on efficiency forrns a reakage path which
"le..ao"e
as the'pressure ratio inceases.
To receiver

Fig. 20.39.p-V diagram for roots blower.


(i.e., isentropic) process.
The irleal compression process fromp, top, is a reversible adiabatic
The work done per minute ideally is given by,
I .r-r I
Work done/min =
f- p,v..lf rl '
y_r--[\rri - tI
I
Then a comparison may be made on the basis ofa Roots effrciency,
Work done isentropically
i.e., Roots efciencY
Actual work done

V"(p2 - p1)

Inlel
Fig. 20.38. Roots blower, two lobe rotors. = x2 = n"u""rr." rrtio)
[*n""","
As each side ofthe each lobe faces its side ofthe
casing a,volume ofgas v, atpressurepr, is
displaced towards the delivery- side at constant p"""". cp
A further rotatioi of te otor opens this \
volume to the receiver, and the gas flows bu.k Also,
pressure The gas induced is.compressed irreversibly
r"o- tt" receiver, since this gas is at a-higher 'v-1 R
and then delivery begins,. This prcess is carrieil
by that from thu ."."ilr"", io th" p."..rr"" p,
out'/oir firnes per revolution of the driving shaft, co
il(r)'-'
1 -11I
For this machine'the p.-v diagrarn is shown in
to p, is shown as an irieversible process at
Fig. 20.3g, in which the pressure rise fronp,
constant volume.
.'. Roots efliciencY =ft1(,-1) |
.(20.49)

Work done per cycle


@z_ pt)V
LJ
.. Work done per evolution = 4(pz_ p)V In case of a Roots ai blower values of pressure ratio, r of !.2, 1.6, and 2 give values for the
If { is the volume dealt with per minute at p, and ?,
...(20.48) Roots efliciency of 0.945, 0.84 and 0.765 respectively. These values show that the efficiency de'
then creases a,s the pressure rato increases
Work done/min (pz- pt)V"
...120.48 (a)) This machine has a numbe of mperfections but is weII suited to such tashs as the scau'
engng and supercharging of I.C. engnes.
I i{

.'li1
'744

INTERNAL COMBUSTION
ENCINES AIR COMPRESSORS
Roots browers are built for
ratros of the ode of 2 capacities from 0.14'3/min.-to
to I for a 1400 m3lmin, and pressure
"ingt "tag" m;.ii""'l"o 3 to r for a two_stagc machine.
20.4.2,2, Vane Type Blower
Refer Fig. 20.40. Avt blower consisLs of u.o,:1..:ulted
und.s.uppo.tei iy;ni
ij.l" lvn" i'*--
th
"
b
"
;;, ;; ; ff ;:::1ffj:iil#:,jf ::i ff H"T f :*jJ; *" mtr" :ff l* eccentricany

:,""
in rhe body,
I
lrreversible pressure
rise due to back
flowfrom
ll^M, {
'- -^^^:.---
receiver
",*"* ,r'"'."*
vo,ume
and casing t
l*::[:X''::?:H:Tx1T,Tru'""riJ:*l":"rii:g
differs from that of the noots btower ty.pe of compression
in that some ]rfu*efl:^Ts_aee Jhl
"" "u
rfff ".$::lT":;ffi iJ"*y":ti*i*i;"J:H'r:::;:xil j:,"T;ili::ii:3:1,::

Fig. 20.41.p-V diagram for vane blower.


Example 20'35' comp.are the- worh inputs
corlpressor hauing the requred for a Roots blowr and a vane type
same ind.uced, ,otu*"'o o.oi'-s/reu., the inret pressure being 1.018
bar
ffi#;tiiii#i:!,J;!"',:j""*, tn" i"",-r'i''u*e that internat tahes ptace
"o^p,"",io,
Solution. Inlet pressure, pr 1.013
= bar
Pressureratio,
* =rU
x 1'013 = 1'52 bar.
For the Roots brower, .:ilaJ'r'r;-5
, .^irq:?9nl.ho*s rh"e-:,lL1y,;l:ir!fi.fifiume atpr.essurep,
i" ia""i io.i ro, and remperature
1^iff#;".'r'J" fiilll,flii: lessurep'
ts opened to the receiver '.,..'n.ood -^.r.,;r.,! i"ing rsentropic.
arses.the pressure i.."u"."irl8u"
"' -"' bly to P2
gT -9
The work done per revoruiion
n"*r* ..t"r."n1 the receiver
following ,- *itlr-ii"r"* j" g,rru., uy th"
"*p.u."iorr

",
w=,---I--"ll11
f, .1-t -lI
r-tvtvtl\-) -tl+N(p2-p)vi ...(20.s0)
blowers require less ro.A
ll"__:"n" rise.
pressure .ornptur"d
-v' v wv'rPar cq to
to.ootrJbl,
roos blower for the same capacity and
They are commonly used
g.5. to rJeliver upto 150 rn'
ofair per minute at pressure ratio
upto
'JLe speed lilnit of a vane blower is S000
r.p.rn.

Fig.20.42
NTERNAL COMBUSTION ENCINES COMPRESSORS

Work done/rev, = <',- i)'" Soluion. Volume of air to be compressed, V = 0.08 m3

105 x 0.03
Intake pressure, Pr = 1.0 bar
= (7.52 - 1.013) x --io3-- = 1.52 kJ. (Ans.) Pressure after compression, Pz = 7.5 bar
Actual work done, Wo"tuot = b2- tt,) y = 105(1.5 - 1.0) x 0.08 = 4000 Nm.
1.52 +
For the Vane type, pt= --- z-1.013 1.266 bar Atso ideal work done per revolution is given by,
ft \(1-lyt
Refer Fig. 20.43.
Work required = (Aea A + Area B)
w,^t= ']-
.t-, p,v, ll a I

l\Arl
1 ..frnr-r)/1-1,l |. ,14-l
Now, Arca=
*' p,%ll:l
L\frl =_--
t.4
x t.o x 1os , o.o8 llll I
*o
-,-l = ,nrr.r, *-.
L\ ''0.' l
t.4 r.or3 x
= 1rL'--
o.os flr.zee \1; .l
105
-il
ro-" itro*j
k"r/rev'
"'ncon'|pressor=W=ffi=0'8597or85'977o'(Ans')
= 3.5 x 1.013 x 100 x 0.03 x 0.066 = O.702kJlrev. 20.4.3. Steady-flow ComPressors
p (bar) The cotnpressors in which compression occurs by transfer of hnetic energy from a rotor
are called, SteadXt-flow conpre s8or8.
The centrifugal type of compressor was used in the earliest gas turbine units for aircraft.
Pz=1'52
For low pressure ratios (no greater than about 4 : 1) the centrifugal compressor is
lighter and is able to operate effectiuely ouer a wide range of moss flows at any one
speed than its axial-flow counterpart.
t For larger units uith higlrcr pressure ratios the axiol flow compressor is more efficient
Pz = 1.266 and.is usually preferred. For industrial and large marine gas turbine plants axial flow
compressors are usually used, although some units moy employ two or ntore centrifu-
gol compressors. I

Fot orcroft he trend has been to higher pessure ratios, and the compressor is usually l,
- of the a*ial flow. ht aicraft units the advantage of the smaller diameter axial flow
compressor can offset the disadvantage of the increased length and weight compared
l,
with an equivalent centrifugal compressor.
Pr = 1.013 Advantages of centrifugal compressors ouer axial flow compressors :
i.
1, Smaller length.
2. Contaminated atmosphere does not deteriorate the performance. I

vs--------l v (m') 3. Can perform efficiently over wide range ofmass flows at any speed. l
l!
4. Cheaper to produce.
Fig.20.43 ti
5. More robust.
AreaE=(pr-pr)V 6. Less prone to icing troubles at high altitudes.
DlsadvanagBs:
u,=u"(#)"
"' (ij#)"
Now, (,
= = 0 0256 mg 1. large fotal aea.
2. Lower maximum efficiencY. ij
(I.52 - 7.266) x 105 x 0.0256
,- = -------lO-- = 0.65 kJlrev. Uses : The centrifqgal compressors are used in :
(i) Superchargers (ii) Turbo-prop.
.'. Work required = 0.702 + 0.65 = 1.352 kJ/rev. (Ans.)
Example 20.36. A roots blower compresses 0.08 ms of air from L0 bar to 1.5 bar per o Centrifugal conpressors are preferred. uhere simplcity, light weight, ruggedness
reuolution. Calculate the compressor efficienqt, ore Lore important than marimum efficiency and smaller diotneter'

l
t__
i

ri 748
l
INTERNAL COMBUSTION COMPRESSORS
ENGINES
749
20.4.3.1. Static and Total Head Values
1
As compared to reciprocating c2
compressors the uerocities t' C2
encountered, in centrifugal To-T= "'(20 53) ho-
com.
ffi:ilUii,#":T:;"::i!,::;!,J"j"I:#;:;,:i:::nto,h',r;.;;;";"i,io"nu"ana,ysing *, h
2
...(20.54)

dccount the kinetic energ! of For finding the total head pressures, use the equation,
passng througi tn, the air
i' "o^pr"rro,
Let us consider, O:,:,i:1r_1 passa of varying po fzl)
20.48). Applying steady"flow area through which air is
energy equation to the s-ystem flowing (Fjg. .: P \T:) ...(20.55)
,ir external heat transfer and work for 1 kg of air flow (assuming where, p = Staticpressure,
transfer t'",1.
"irt"J] *" ,*
no
il . T=Staiictemperature,
1

utlppt.+ =ur*O"rr*9t-
po = Stagation pressure, and
?o = Stagnation temrerarure.

eu- ${= u, + Pzuz * cr2 I Adiabatic process and isentropic process


l,r, *
3

| " zsr J 2cJl In adiabatic process the system does not with the surroundings
exchange any heat
i'e., no heat enters or leaves the working fluid exiernaily. The kleal reuersble arli-
L ......in MKS uns]
abatic process is called isentropic p"o"es" and in ths jrocess entropy remans con-
stant.
ht+ ef --L r.2t
,
C,,
2 2
a2
ul Pe
C,,
"oTt 2
= cpT2+ To,
-------T- 2
2 n

crT +
C,, f2 e.
'lzco i*i1
2
= constant fr
..(20.51) 'a
rr'
(? Q=0

W=0
F8.20.44 Fig. 20.45. Total head and static quantities on T_s diagram.
Temperature ,?, is c;tlled
bv the thermometer when the .the
static temperature ot
temperd,tureof the ar measured r During adiabatic compression in a rotary cornpressor there is friction between molecules
ttrcrmomete;;;;;;' the-the
ar uerocity. If the moving air is of air and between air and blade passages, eddies formation and shocks at entry anci
brou1ht to rest under evesible
"t energy
conditions,-trr" t"i"iii"uc exit. These factors cause internal gt:neraton of h,eat and. conseqtrcntly the maximum
t"ouu.ing rhe tenrperature ofthe ir is converted into heat
ll"fq: and O;"..r"u;i;;; temperature reached would be more tltan that
air ;,"togootion" for ad.iabatic This resill.s
is known
^,
peratue and pressure
o,.,,totoi n ii;;;;;z:;":^:;i?:ff::ff:inTiHT* The heat generated by friction
"oipr"rrion.
in a progressue inc-rease in entropy. Such a process though ad,iaiatic is not isentropic.
ae denoted y, lO, *ay be removed with the result thar
"rm_,"i*i"". the process might notinvolve any "i".entropy change. The"oniinuously
process a,zld then be isen.tropc
but not adiabatic as heat has been trasferred..
where ?o is known as totar ...(20.52) Isentropic efficiency. "Isentropic effici.ency" of rotary compressor may be defined as
0"";!r-rrr"rr";'1"^ouoru* ratio of isentropic tenperature rise to actual temperctturc rse.
rre
751
750 TNTERNAL coMBusrroN ENGINES AIR COMPRESSORS

7. Curued rad.ial vones. A series of curvecl radial vanes are attached to and
otate with the
With eference to Fig. 20.45, which represents a combined diagram for static and stagna-
tion (total head) values, we have shaft.
t":!ll:ltr',!?.1"1"""", 2. Impeller. The irnpeller is a disc frtted with radial vanes. The impeller is generallv forged
rsentropic effi ciency = Actual temDerature nse "'" or d,ie-casted of low silicon aluminium alloy'
Toz' -Tot
^'lson-- ry ry- ...based on total values ..,[20.56 (a)]
'02 -
tol
r; -r,
- Tz-7,
...based on static values ...t20.56 (b)I
During compression process work has to be imparted to the impeller. The energy balance
equation would then yield :

"^T,*C" =c^Ti+C" 2
-w
or coTor= coTor- W or W = cr(To2- Ts1) ...QO.57)
Thus the work input is the product of specific heat at constant pressure and temperature
rise. This relation is true both for adiabatic and, sentropic processes.
Fronr eqn. 20.56,

Isentropic effrciency, l;"", =


u_h
-
Toz Tot
cr(Ts2' -T6)
tl,""n = =Isentropicwork
A.r*l *-k ...(20.58)
4{4rl4J Fig. 20.4?. Impeller (single-eyed) and raal vanes ofcentrifugal compressor
Thus the isentropic efficiency ofa rotary compressor may be defined o s the ratio of isentropic
compression work to actual compression worh. Ah out
20.4.3,2. Centrifugal Compressor
111t
Fig. 20.46 shows a centrifugal compressor (with double sided impeller). It consists of the
following porls :

lttt
iltl
vv vv
Air out

Fig. 20.48. Doub1e eyed inpeller

Fig. 20.46. Centrifugal compresor,


;

752

ENcrNEs
j*.oeller may be single-eyed or COMPRESSORS
-^ .,,.t1" double sided (hvi^- :l^t ::MBUsrroN
".." ^-
iliJ#;;.Tffi :Tffi "":: Cr = Absolute velocity at
"1ili$:f:**l';1""#?ili:l:.:tl|!i:"1r,tiJ'ffi
to approximately
in an axial forces Cz = Absolute velocity at
inlet to the rotor.
ouilet to the rotor.
3. Casing. The casing "qual
suround.s thp n+^+:__ : C', = Relative velocity of
air at entry of rotor,
;;,:r::;:,';;::#;::::Jf ;"fHi;1fi,1t,,n"..,,", C., = Relative velocity
ofair at exit ofrotor,
Cr, = Velocity ofwhirl at
',{+",j$ii,i.',i}iiit:;li*"n:::l**,,:t;,li:,Hi*rr?,r".ir Cr,
iniet,
= Velocity ofwhirl a ouilet,
Cn = Vclocity offlow jnlet.
at
C =Yeloeity of flow at ouilet.
p"1E'.:Ifr
'ff f !i:i:::;::#i;,::'::li":;:n|tri,:,::lifi;;:itff xgg*ri*t
ritr:{.fi
"illT:.#ilif;,j#*fu
TT:'ff ;;
dr = Exit angle from the guide vane or inlet
angle ofthe guide vane,
Fr = Inlet angle to the roto or impeller,
|f Fz = Outlet angle from the oto or impeller, and
az = Inlet angle to the diffusei.
:,i}ffi :'j"*'".ffi ;tr#n';;m ff f fl :ii}iffiiJilj",,l"j:f r Fig 20 50 shows the velocity diagrams for the
inlet and outlet of the impeller. It is assuned.
that the entry of the ar is 'aal', therefore
the .hri-component at the nret ( c,,, rs zero
) and
j
J- tl
t*:11t: = c ' The enlarged ews of inlet and
outlet velocity diagrams are shown in Fig. 20.51 (a)
at entry and erit the btad,e must be parauet to
Silr?\L,|!rr:""::;rt!:!"gh thelefore g, af{ g2 are the.impllerblaa"
the reratiue uetocty
Pressure curve
Pressure rise rhe d;#,,".. r,-r^
Tha diffuser ^__,^O must_be p4;;ll;i:";h"";;;;:;;r,
blade angle ii" irl"t and outlet.
in diffuser rnerelore ' isrthe diffuser blade angle
".rgt".
ue_tocity of r "i
f,:";;;; ;;;:tLil[i\:,
at tbe inlet rnJ q i, the diffuser "J.
the ouflet.
y;:;,*"::"i1;[::m the d,iffuser isit ""gl-"t
Pressure rise "i^",]ii"i,Ji-".^'t, brade angre ar ouuer (c,) should be as
In tmpeller

IF
Uro.n, f I

lVetociry
il:ff:'.1 I decrease
n dffuser
i
I

-L
v3
Fig. 20.49. Variations
ofpressure and velocity
r fn
rn practicp - of airpassingthrough
--^rqed'6,rrougrrmpe.lleranddiffuse.
impeller and diffuser
practice hnearly L ^ t.,,
a-,t^, half .'
j,.' tt
i u f' p;;#;;;;:":!:e:sur.e ! : :' , :r can
iss achieued
"""^""r-:d i" iyry.t1,, and
!For; higher
u
t n
i1:!!n*, ai : i a ch ieu
ratio '-" *. oe achieved and. remainins
e
retnaining hatr
"l;::J,:" ^',i",.of .S ::^tf::!!* hatf
,':4":.li"li'"",li#|,!r^,in
::Ii:::":l
::T_l*""9:. ratit
Ti.,nir::,"TIi,"i,J;i",,"ilii,i".*:""::r^y:"
w^ith tnel"-n"ii'""t.';?grr
pressors, the outlelofthe .^.'-,"'"'"'"gc
jn".;r"lXi,l?lii',?'"'i;l,Xt:;ry_1":,0.. compressors are used. i,
a,"-.,sed ln ;;i,i ;;;;;.";
ij,l,.",li';
l1'::t 1r., j,;:ffi*':Jt;:ff
r'
' o"Jii" *r"il:ff]"',ff::1|],ll:"',":::o
20.4.3.2,l.Velocity.Diagr='"--
l1:;:,".',:;. 1"":;::1::'iHi:il1:::ff
nuu-bLage centrifusal
stage and;;;;
:
i;;;,*."
^"iri_.i^*.".""r*
--'--"'vr'
of operation"ntrp.".ro...
Ler, cr, "bq = Mean ,"Il,lli
tytean blade lt:orv
velocity at errtrance,
of cenrrifugal compressors

C"
= Mean blade veiocity ar exit,

--._*-_j_
/))
INTERNAL COMBUSTION ENGINES AIR coMPRESSORS

c? - c?" c?," - ch, ( dp ...QO.62)


2 2 Jl P

where P is the densibY'


lf th" diffr."t outlet velocity is Cn' then
C*'=0 cz" -cn'
=[n 4 * Md,r.ro""
2 J2p
co,,
i.e.,thedifferenceofK.E.attheirnpelleroutlet.and.diffuseloutletintroducestheworkpartly in diffuser'
pJ.iiv it.*".rily inverted into heat due to losses
utilised for pressure irr"r"".", ".ri
Inlet velocity Outlet velocty
dagram diagram air flow in one second'
(a) (b) o Power required per impeller for m kg of
..(20.63)
Fig. 20.51. Velocity diagrms. , = ^"#;"' kw, using eqn. (20'60)
Work done by impeller (Euler's work) at the outlet ofthe irnpeller
:
Ifthe radial (ideal case)' then the velocity diagram
blacle is
The work supplied to a fluid in a stage of compressor may be found by applying the - C'' = Cub th" work d'one per kg of air flow per seconcl' fs
tnoment of momentum theoem. Consider 1 kg of working fluid passing through the impeller. The is as shown in Fig' 20'51' As '
theoretical torque which must be supplied to the impeller will be equal to the ate of change of given bY ...(20.64)
noment of mon)entum experienced by the working fluid. w = czz ,, velocity'
The theoretical torque - (C,z,rr- C,rr1), where r, and 12are the radii at the inlet and at a velocitv greater than the impeller tip
i-p"il"r
since the air cannot leave the eqn (20'64)'
of air flow per t""*d iJ given by
outlet of the impeller respectively. the mamum work supplieJper kg impeller, assuming the heat
If u is the angular velocity in ras, the work done on 1 kg of fluid will be, _ Now consider the steady flow at the inlet and outlet ofis the zero.
the impeller
. W = Theoretical torque x angular velocity i..rrr", rluring the flow of air through

= (Cr, . rr- C,, . rr) o Cur=c*,-


=(Crz,rr.a-C,r.rr.a)
W= C.rC6"-CrtCur= hoz- hot= co(To2- To\ ...(20.5e)
The above equation is known as Euler's equation or Euler's worh.
If the working fluid enters radially i.e., if there is no prewhirl, C,r = 0, then
W = C,z. C64 JIkg ...(20.60)
Using the inlet and outle velociy triangles, we have
Cr"-- Ct"
c,r' = Ctt, + c'2 -2C6,C. ...()

c?"=c3,"+c22 -2cbhc2 ...(,,)


Inserting the values of C.2 . C6" ar'd Crt . Cbt, from the above expressions (i) and (ii) in eqn.
(20.59), we get

.?:-cf *ci,:?-c"o *c'*:,ctu


cr"
. w - ...(20.61)
? Fig.20.52
Firtlm Se@ndh thirdh
The first.{ern shows tt,e inoease in K.E. of 1 kg of worhing flud n the irnpeller thot c"'
. - has to conuerted into the pressure energy in the'diffuser'. hr*+ +w=hz+ 2
* The second fer shows the pressure rse in the impeller d.ue to'diffusion acton' (as
(. c"'z)
the relative velocity decreases from inlet to outiet).
w=lhz+ .l l-
The third terr shows the pressure rise in the impeller d,te lo'centrifugal action' @s
- the working fluid enters at a lower diameter and comes out at a higher diameter). | ['.+J 1t

Thus the fraction ofK.E. imparted to the working fluid and inverted into pressure energy in
impeller is given by
=""?'.*) ,,?,.*)
i,
'i1

li INTERNAL COMBUSTION ENOINES


COMPRFSSORS 757

20.4.3.2.2. Width of Blades of Impeller and Diffuser


If the mass of the ai flowing per second is constant and is known, then the width of bla<lcs
impeller and diffuser can bd baltid -s"follws :
,,'
Let, = Mass of air flowing per second,
r = Width (or height) of impeller at inlet,

rfr C/, = Velocity offlow at inlet ofthe impeller,


ur = Volume of 1 kg ofair at the inlet,
ii
rr = Radius of impeller at the inlet,
Then,
. _ Volumeof air flowingpe second _ 2nrtbtx Cf,
. Volume of 1kg of air u1
But as the air is trapped radially,
c =c,
.
h=
2nr,-' d xC,-
...(20.77)
u1

L. - rit ut
,
"1-
h-_
...lzo.7t (a)l
l, 'fl I 2nrrtCt
Cr'="oTo,lf &lt _rl Similarly the widtb of impeller blade at the outlet
eqn. (20.71)
can be found by using suffix 2 in
l\rotl l . 2nr2 b2 xC.
m= "
.(20.72)

tr=f#.']* .(20.69) _The


w-idth or height of the impeller blades at the outlet and height ofdiffuser blade
inlet should be same theoretically.
at thc
C,= Cz
The widih or height of the diffuser blades at the outlet, is given by

a =.19*,]'- =
2trr b xCf
...(20.73)
"'Q0'70" ^ Ud

Tho. as per eqn. ,ro.rur whee suflix 'd' represents the quantities at tine outlet of the diffuser.
.-^,--.
factors :
rnlt oo*\":l;")r. depends .--^_
comn,oo.n,ro^^-^
thee cotnpressor upon the following n = Number of blades on the impeller, and
(l) Mass flow of air through = Thickness ofthe blade,
the compressor.
(ii) Total temperature at the then eqns. e0.71),(20.72) and (20.23) are expressed as follows:
inlet oflhe co_p"esso".
Toial pressure ratio ofthe (2r, - nt)/'C,
u",o.,rtjit' "o-p.""so"
*hilir J.p..ra. upon the square ofthe impelrer tip ' ' tl
rh = ...(20.7 4)
In the above analysis .
U1

ps, = Stagnation pressure . (2ro' - nt)b.C,


at inlet ofthe compressor m= 'Iz ...(20.75)
;
To, = Stagnation temperature U2
at jnlet;
po2 = Stagnation pressure
at ouflet : - (2nr,* - nt)b,C,rd
m= " ...(20.76\
?0, = Stagnation temperature
orjl"t , U
pt = Static pressure at inlet : "t
20.4.3.2.3. Isentropic Efficiency of the Compressor
Ir = Static temperature at injet :
The following losss occur when air flows through the impeller
p2 = Static pressure at outlet :

; (i) Friction betrvcen thc a,r layers moving with elative vclocities
Zz = Static tempeature at and frictior between thc
ouilet. a! and flow passages
INTERNAL coMBusrloN ENolNEs AIR COIVIPRESSORS 759
(ll) shock at entry
(iii) Tu5r1.... caused in air. 20.4.3.2.4. Slip Factor and Pressure Co-efficient
The losses mentioned above cause air increase in enthalpy In the earlier analysis it was assuned that Qr, - C6, but this condition is not satisfied in
of the air without increase of
actual practice d,ue to secondary flow effects and therefore in actual compressors C., . Cur, .
pressue therefore the actual temperd,ture of air
coming out frornihe compressor is more than the
temperature of air if it is comprssed isentropically.
he actual uor& required for the same in- (Cwr- C,") is kown as slip.
crease rn pressure ratio is more dte to iteuersibiliiies. The diffeence between
The actual and isentropic .o-p"es"io' fo"
the same pressure ratio is shown in Fig.20.b3. It is defined as the ralio of actual whrl component (C,") and the deal
Slip factor (Q").
The isentropic efficiency is given by the relation,
whirl cornponent ( Cat )
"
n. _ Isentropic work hoz' - hot
',." - -Act,r"l *oik- _
= 0, _; a"= yr" --rif c* = c6,
Toz'-Tot
= ...Q0.77J The stagnation (total head) pressure ratio is given by
provided specific heat at constant t-l 1-r
o"""3; ,"lt."mains constant. o* =(!--\T
T; _T' por \?o' / = l,r*rm'-ro')-r
?ot )
If C, = Cr, "rt n.tasn
T D ...(20.78)
\
'2-rl Substituting the value of Q* - To) from eqn. (20.77), we get
_ -1-.
Poz l', 1*" (42 - 4r) l t
- ...(20.80)
Por=L" ?h J
lncrease in work due As per eqn. [20.60 (o)], the actual work done per kg ofair is given by
to increase in volume cp(To2- To) = Cwr- C'"
Theactualworkdoneperkgofairbythecompressorisalwaysgreaterthan C6"C*, dueto
fluiil friction and windage losses, therefore the actual work is obtained by multiplying cu,c,, by
a factor 0, known as work factor or power input factor.
j. cp(?oz - ?or) = q.Cdrcy
T' vw-oJz'tb2
'02 Toz- Tot= ...(20.81)
'p
Now substituting the value of eqn. (20.81) into eqn. (2i-80), we have

Po'- f., *\uoL'c'c"l-T


Por L- "PTot J

Now substituting the value of C,, from eqn' (20.79),ve have

lncrease in work due to Ioa n*" o,i" ct'lT


Pgr= fr*
.,.(20.82)
friction, shock and turbulence
L cp ror J
Pressure Co-efficient (Qo). It is clefned as the rato of isentropc worh to Euler uork.
\Fig. 20.53. Actual md isentropic compression on ?-s diagram
... -Isejrtlopicw-ork
.vp- Eulerwok --co(Toz'-Tot)
. .- v-l
*
"Ait Cu"Cr,
Y-', [ol' -, *q1
*-{ Using eqn. (20.77) and assuming the vanes of the impeller are radial and.counter flow of
=8.=fu'"'^r'
lt

-'
t Dr I
-l5i fluitl is neglected (Cu2=CbI')

", lpt)
yh)
" _, cp rl* (4u - 4r)
^ -_
yp
Cuu,
INTERNAL COMBUSTION ENGINES COMPRESSORS
,;dlR 761
Now using eqn. (20.g1) which is
. Fig. 20.55 shows a typical vaned diffuser. Thee
co(To"- Tor) = Q, cut, cw = Q.Q"C6"2
is a clearance between the impeller and vane leading
edges amounting to about !0 to 2OVo of the diameter Vaned
diffuser
,P= 0. 0" c,r-',lu* -= 0'0" ni""o
for compressois. This spoce constitutes a vaneless dif-
q" fuser and its functions are :
CJ_- ...(20.83)
(i) To smooth out velocity variation between
the impeller tip and vanes.
"ruil33;3""Ifi f ',i:"r'1l,iill,'i[:*.?fl ;;i:ffi *,::.*,"rmance.rherorowingshapes
Dilfuser
rilr
() To reduce the circurnfeential pressure gra- vanes
dient at impeller tip.
1. Bachward_curued, blades (8, (iii) To reduce ttie Mach number at entry to the
< 90.)
W vanes.
. The flow follows an approximately logarithmic
spiral path to the vanes afte which it is constrained

a
2. Eadial-curued btades (Br= 96"1
t
A
by the diffuser channels. For rapid diffusion the axis
ii, ofthe channel is straight and tangential to the spiral. Fig.20.55.Tpicalvmeddiffirser.
Fig. 20.55. Tpicalvmed diffirser
The design ofthe passages is usually_based on the simple channel theory

6I
3. Foruard-curued, bla-des (Fz > with an equivalent degree
lilj 90.) ofdivergence ranging between 8 to 12. to control separation.
c The number of diffuser uanes has a d.irect bearng on the size and the effciency of
lli; Fig. 20.84, shows the elative performance the d.iffuser.
of these
blades, Centrifugal effects on
tne curved
tng momcnt and produce increase.d blade screate abend.-
t I
When the number
lf liffuser passd.ges is less than the number of mpeller po,ss.d.ges d
ll, the tnaximum speed at which
,rn"i"'lii"i-r"ru a
more unform total flow results,s
,hz ntpeller can run. o
Diffuser efficency ts defined as,
e Normally backwad bladeVvanes
with p, between
(p,
\d=;---:'-' -
20_28" o n')
are emploved in L.".'rii"*,l,*r,
"*".pt
head is the m"jor con"ideirtion.
E
o
. ...(20.84)
Sometimes compromise is made
between the low
;;@l -c2)
1 and 2 denote upstream and downstream conditions ofdiffser and
energy transfer (backwrd_curved E
,^.._,._lu*tut is the weight
van"s) aniigl o- oen sltv.
ouflet velocity (forward_curved
,r.".1 ll i"l"*""- flor rate
l order to achieve higher diffuser efficiency, the following points need be considered at the
Mass
dial uanes. ---| .
Ad'uantages of rad.ial.blad,e impetters Fig' 20'54 characteristics of time of design:
: 1' The entrance blade angle of the diffuser must be such
1 Can be manufactured easity. '"5:TuJ;:il1f"1X,#;"u, angle of attach.
that air impinges on it with a sm.all
and rorarionar speed, hence hishest
3. ilJ"ll,lll,liXt,.'ff:ft:,il:niiameter weight.
turned through 90'to flow in an axial direction to the combustion chamber. This
energy conversion turning may be
in impeller and diffuser, giuing achieved by vanes installed in the diffuser below.
,ffi"i":y:q^l high pressure ratos ,tith good
3' The area ofthe flow passage
lust be large enough to handre the air and it rnust expand
within certain maximum and reasonable limits.
o",,r,Ji-\ili,t"t:;*T;;:?;":;I'"'::;!,.,, uith radiat btades has been the tosic choice
of the 24,4.3.2,7. Losses in Centrifugal Compres-
20.4.9.2.6. Diffuser System sors, The losses in a centrifugal compressor may be Total losses
In a centrifugat ."ili::.,"-.:.the diffuser categorized as follows
plays a significan_t role in oveall
:

iil,;';';'n:ff.1ffi:mlX*:igned to t;*iil*:fl.tgtbe.air by increasing itscompression


verocity as tional to
1. Friction losses.
CP and hence
These losses are propor-
proportional to n2. I
.,ai"r.iaea ffir"";,;:;iil:::"1?iTnJi,;,,#::::i,il:;,:;",:::?^,,""""##.Jl',";ii:
o
2. Incidence loss, These losses in terms ofdras
+"" '"tljL",J;';iitk::;:\::-T:: :* "'"4 '"'""* the whirr;rilffiJ" higher rate
i" o
coefficient C, is proportion al to C oC2. J

n;i:i,j:i';"H;,;:::;:":;:::,E"no,'r;,'1.:*:,;:l:lir:::,:*%,; ,*:":: l* Fig. 20.56 shows the variarion of losses with


respect to the mass flow rate- Incdence losse

Mass flow rate ---->


i

irig. 20.56. Variation oflosses rvith


espect to mass flow rate.
INTERNAL COMBUSTION ENCINES ArR COMPRESSORS 763

20'4'3'2'a' Selection of Compressors Geomeirics. The various compressors, The numbe ofvanes may vary from 10 to 30 but should not coincide with nurnber of
geometrics -
are selected on the basis ofthe following: uanes in mpeller to avoid resistance.
1. Numbe of blades in irnpeller
D,
vary fom 1.25 to 1.6 and the maximum diffusion angle is around 10".
2. Blade angles
- 11
Uc ^ay
3. Impeller diameters
4. Impeller widths o Is,- ntropic efficiency of vaned. d,iffuser at design condition is ltigher than that of uaneless
but its off-clesign performance is poor than that of vaneless.
5. Impeller mateial
20.4.3.2.9. Compressor Characteristics. When flow is taking place in an impeller chan-
6. Vaneless diffuser
nel, there are certain.inlet losses, friction and separation losses and discharge losses in the dif-
7. Vaned diffuser_ fusei. Ifthese losses and the effect ofslip and non-uniform distribution ofradial velocity around the
1' Number of blades in impeller. The optimurn number pcriphery of thc impeller ae taken into account, the head-capacity characteristic for the back-
of blades which gives the best
ef[iciency can be chosen byexperience for a particular ward-curwed vanes would take the form LM as shown in Fig. 20.57.
requirement. A number ofelmpirical rela-
tions are available for calculang the optimum number
of blades.
o Vncent suggests that the opimum blade number varies from 18
to 22 for radial bladed
impeller having diameter fiom 25 cm to B6 cm.
2. Blade angles I
Among
' extent. the outlet and inlet brade angles, the fomer influence the latter ro a great
o The inlet blade angre, within he reasonabre rimit, does not affect the performance and,
its optimum value is about 30. to 35".
3' Impeller diametere- For rad.it brades,
following equations :
the tip impeiler diameter is calcurated by the I
p
6
Actual work input = {,Q Clr JAlg s
...(r)
and c,,
UL2=zN 60
After knowing the tip diameter, the inlet diameter is calculated I

from the value of diameter


) I
ratio. Generally fdt varies from 1.6 to 2.
4' Impeller width. If and , are the blade width at inret and ouet of impelrer, then
1 ,# Capacity
neglecting the thickness of the 6lades ii is .alcutute
by ttre equation
H -_> ofa centrifugal compressor
Fig. 20.57. Actual c:haracteristie
m = ndrb, Crc1=r4hC"pz
:l 2O.4.3.2.1O. Surging and Choking.
Generally Cfr=Cf,
.
Fig. 20.58 shows a typical cbaracteristics ofa
5' Impeller material. lhe impeller of the centrifugal centrifugai compressor at one particular sped.
compressor generally forged or die
is 6cEa'J
uv o
casted of low silicon aluminium allo Consid.er that the conlpressor is running at
6' Vaneless diffuser.{he function ofvneless
diffuse or space is to stabilize the flow for
point N : i
shockless entry into the braded diffiser and
to in"*t po.fio"" ;iiJ.l;;; ;"l;',rl o If now ti.:'e resistance to flow is in-
E
"Jrou Th" creased (say, by, closing the valve o
diameter ratio of vaneless to impeller tip diameter "rr""gy. I
vaies f.o- $ = V0.06 to 1.12. provided at the delivery line ofthe 3
compressor), the equilibrium point E
since the flow i the vanelss diffuser is assumed
to be 2garithmic spiral, hence og cr. moves to M.
Generally , = g = width of vaeless diftser. =
i., .o-".r"". a,, ar. . Any further restriction to the flow
7' vaned diffuser' The outlet diameter ofthe
''
vaned diffrselr depends upon the choice for will cause the operating point to
the velocity desired from the outlet of tn"
shift to the left, ultimately arriving Mass flow rate -----|
"oip*.o.-.of
By the zse of bladed diffuser the d,imensions at point L. At this point maxirnwn
the machine can be red,uced, due to red,uc-
tion in diffuser size. The ourlei diomerer t Fig. 20.58. A typical chaacteristic at one
;;;" the equation, pressure ratio is obtoined., lf the partrcula speed.
m = rdnbnCappo sin an fiow is still reduced fron this point
where n = bz= bs, then the pressure rato willred,uce.

I
l-l
i1
ii
)iii '764

INTERNAL coMBusrroN
At this rnoment- thao is ENcrNEs
L3_r
; a- higherpress AIR COMPRRSSORS
compressor 0.,:l-Ln:"t'
. At ", " ,o*, ,,),!7,],r:,rli!;jj::;,i:i{
utatn !iIj:",:rHr."y,:ffm
,han rhe
starts to increase frn- 7 ,,--. ;--"'r' 'u' strlrl
stor|^ to deliuer flud'
llui,l The
If the downflovl"^!-o!,." ,rrv lr--"lui"Lagdtn p"""'rrn 6 't00
;;;Jtd''operating point mouis tt^-"sit n "fi.
iL
Tsl
t
r
arter point
jjfi i."';;;ilXilj;i;i;"::iiff,l;l*k",,Xl
I

1
mo*""""-"ili"'J:if
""trtl1dlti",""
i";'l 1
! 80

lir auctng high pr, 3


1l
impacttodsa::;;;;;]!:ri,::;;"i:i:i^ii:#,i'!::;;:;;;,:;":;f;:;:jf I

*t- -9
60 0.60.7 0.8 0.s 1.o
l!
Owing to this particula. ; olo e (
:i

compressor canr, r,r"""."*r'J.", ,


or pumping at o
40 to design values
lr
oiorv");;;f'""r flow rates the /ou mass go ;fr;Relalive
That is, , o13""'"0 * ,i"-orli'eF of the maximum presaure ratio point' t
'r "^nnll',!"-:0"'"t"
on rhe charact"'t 1: t'"o" of the chaacteristic.
At a consrant
,"t,"*,"*-""#;;;" occurs
at higher mass
constant. with the "oro..lrtt",,tl" t"tr"*"i'"iii flou rate points:
,,""."^."-o.i"1ll-" i tn"'prr*\fi)cv component at the impeller tip remains
aecreased. Consequentlv. n
,;!:^:,^,t o 0.2 0.4 0.6 0.8 1.0 1.2 0 0.2 0.4 0.6 0.8 1.0 1.2
j,
ruEe verociry and i"'t"..J*f"tb d'e*eases wbichand bence tr,. i"t.lt],"'i.
tt, incjdence
d cnonrng state. T\e",oo.
il^lld]?l:"t::1"
tt the ditruse" v;;;;;t;t:"ttd"rably'
increases
rnus' there is rapid proeression the aso-
.ff ,",",,u" to desgn vatues mF ,o o"sgn vatues --->
tLow catnot be ncrease"r.l'l:,9ot"T'"t"ti. ti,l."r"
o1.n-tr'l towards "",u,,u"
,' and finallv aftJr
I rne characteristii rrrsllj becomes uertical. ooit't P m"sJ (c)
curve t" 7^!::'l",'ru" "lt"*"i""ji,J^ttu:P"nt
chokingpo-r:rii"""sutc Te point ?,on @)
llrl When the .o_p."."o41.1"d Fig. 20.59. performance cunes ofcentrifugal
compressor.
ll't poperly otherwise with gas tubine tl
troubles :,,f, l"^tg
" the characteristics must e Fig. 20.59 () shows the performance and constant
be matched efficiency curves.
Note.Thechohing-o""1tt'
* to
"*p"u"u?iti";oo.len
*"*oi i);;;;::"'::;:,il:,::::, errciency. such a plot does not take into ccount the varying inlet
temperature and pressure. In
zo.4.s.2. t t. p
",ro,
"
of centrifugal
" ly: addition to this, these plots cannot show the comparson
of performance for simila
r Fig. 20.59ouo*'lll compressors
t":-o:':]'ltionship between
compressors of different sizes. To account for
all ihese, the'performance curues arc
plotted' uth'dimensionless parameters'.These
uerss flow t"'" pr
.i;:t:sure
ratio' power and efficiencv
curves
dimensio.ri"r. p.".ut"". are : pressure
s1eed,, efficiency
rrlt-l-'^i1tt"..t'-h;;-;i
the as N" No etc' At a Ie ; soeed paramet"r,
ftow certain ratio,

ater which ""r'^73t^"t^os
a".o.a.,gli#;^:,;::rr:r:::"::::!:ir:;X:::i:^:i
"","irit'such Pt ff and flow parameter
' ^JT, il.irr.20.59 (c)and (d)1.
otro nrrnor"r.*uu;eoses. lri pr
Example 2o's7' A centrifugar compressor used os a supercharger
150 hglnin' of air. The suction.pressue ond for aero-engines hand.res
teiperature are I bar and.2g0 K. The suction
uelocity is 80 mls' Afler compressiin in the impeller
tI Calculate :
h condions are I.s bar s4s K and 220 m/s.
I
une of constant (i) I s e ntro p ic e
e sped fficiency.
(ii) Pouer required to d.riue the compressor.
Lo of conslant
+
I 2t efciency 6ii) The ouerall efficiency of the unit.
E I Slabity It may be assumed thdt K.E, of oir ganed. in the
imperer is entirery conuerted into pres-
.2 z.q sure in the dffuser.
g Ma
.9 g )<n
Solution. Ciuen : riz= = 2.5kg/s ip = I l.a ; Tt=290
* K; C, = 80 m/s ;
E
[ pz = 7.5 bar ; T, = 345 K ; Cz= 22C rnls.
i 1.6
(i) Isentropic efficiency,
,"",. :
y-1
Nr>N2>N3>N
!=(u)T -[ I / '"
rt [p'j =lu1 =tt22B
eU 110 13n ia^ _
| lu Tz', = 290 x 7.1228 = 325.6 K
r90
Volume flow rate, mlnin ____,

- f' * 9t-!
(a) Isentropic work done = cp(Tz,
(b) 2x1000
(220)2 - (so)2
= 1.005(325.6 - 290) +
2 x 1000

i
INTERNAL COMBUSTION ENCINES AIR COMPRESSORS

= 35.778 + 21 = 56.28 kJ/kg, I 5rR


Solu!{on Giuen : th
= t.t kg/s ; pt = 1 bar, Tt = 2O + 273 -- 293 K ;
(220)2 , =
Work done in the impeller c-(?^ - @qz
= - ,.,'' * 2xtOOO
N= 20000 r.p.m. i l,,, = 80vo i pot= 1 bar ; poz = 4.0 ba ;
c-z
Q2q2 -@of Cl = 145 = 0.9 ; d, 10 cm 0.1 m.
= 1.005(345 - 290) +
2 x 1000
^ls, Cu, = =
(i) Rise in total temperature during compression if the change in K.E. IS
= 55.275 + 27 = 76.2T kJ/ks
negligible:
"
(ii)
q*" =
Power required to drive the compressor, p
*Hffi# =# = 0.7445 or 74.457a (Ans.)
Refer Fig. 20.60. The suflix '0' indicates the total values.

:
p
= ,h * Work done in the impeller (kJ/kg) 02
= 2.5 x 76.27 = 190;62 kIV. (Ans,)
(1tt overall efficiency of the unit,
fT Iovera' i
As K.E. gained in the impeller is couverted"iriio f oz'
p."""u"", h".r"u T-
co(Tr-Tr)= c-r" -cr" I d2
2 x 1000
I
dr ,r ;
1.005(?s - s4b)
x 1000
?s = 365'9 K'
=
Q2!D2
2
- lsqz
II I
The pressure of air after leauing the Tot
d,iffanv:r, p, :

rY-l T1
A l

T2=[n)
\n)
'
Fig.20.60
I

:.
tr-e)*=(T#)# =Lz2B6 The stagnation temperature at inlet to the machine,
iil
ps = 1.5 x l.22g6 l.g43 bar. lr
= rrt= rt+
ff=rnt*t#*-
fii
After isentropic compression, the d.elivery = 3oa.b K lll
ternperdture fiom diffuser, Tr' il
, ' 1-1 r tl

r3 _l/ trb ,11 _ l.{_r fl1

II
-ltJ =l 1 l
I 1.848) r-4
=1'1e1 Now, a, =[a1)
r1 \rrl
' or D^. =,. , |,h.)-
\r'l iil
"'
Tz' = 29O x 1.191 A45.g9 K
= 1.4

l.ou"ror=
r"' - r, 345.39 _ 290 p,, = t"
[#)'"4. = 1.181 bar
4_4=;rffi*
T:T= o eA< = 0.7298 or 72.98Vo, (Atts.)
1-l 14-r
:;i":::7':;h"":y*':'::-r:::":::pc:! conlpresEor hand.tes 528 hstmin. or air. jr!-=tj!l'
4' |- 4.0
i::,:#!:":;,:L#*";r::"l::-! :ti:;"i;::"h;";?::;,*;:ffi'":i:o;f;Tl,i!,; T"t lr/ =l\1.131/)T = r.asu
:::::y;",:ry,;"ii il,,f!:,::":"t. T!: t;li;;o;:,:;';:,;;;';#;",::,"f:# ii:# "T,Ji!" .'. Toz'= 303'S x 1.435 = 435.5 K.
';:,:;";:':,j-!^,!'^'::!-,{*.il':
with ? ;i;;.',h,";:;;;;,:A;';:,;:"',!J#o'iioIjT"
no prewhirt. Assumins ,i"i nll"=ii',inii
() Rse n total temperature
"il: !l!'ff li !rl!!0",,,r:t{!t*:[,:f , .'. Isentropic rise in total tetnperature = 435.5 - 303.5 = 132"C
durns to" i.'"" ,: ;:;;;:;;:
(i) The tip dameter of the
impeller.
"o^pr"rrion "i"r'r7 Hence, Actual in totol temperature =
rise = 165'C. (Ans.)
(ii) Power required.. #=#
(ii) The tip diameter of the impeller, d, :
(iu) Bt" dianeter if the hub d.inmeter is 12 cm. Work consumed by the compressor = cp x (Lt)n"t,ot
= 1.005 x 165 = 165.8 kJlke

_1

I
$
r!i{
| . il
il
ljl 768
INTERNAL COMBUSTION ENCINES
l
COMPRESSORS
.' work consumed by the compressor is also given by Euler's equation
769
without p'rewhirr as : .'.: Tz' = 293 x 1.486 = 435.4 K
l
w=9-ffixc" kJ/kg = 165.6 t fr'_n
Now, n.
'rsen Tz - Tt
J,.
But --=- = 0.9 ... C,n = O.9Cu.
"blz r, = r, i351: J9s
ir
e?. Ya
+
T:=
2e3
"' = 4GG.7 K. (Ans.)
1A(a- -oh-'-'- (ii) Theoretical power, p:
tir 1000

lll frGS.8 x 1000\u2


^ =l-------:--- | Mass flow rate. th = PV lx 105 x
-=-28,1-2;:
(660/60)
ur" = 129.2 rrls RT = 18.08 m3/s )
ll
\0.9)
... p= rhcr(Tr_Tr)
But Cn- = 429.2- ttdzN -r4x2OO00
60 60 = 13.C8 x 1.005(466.? _ 29A) 22gB.S k\f,. (Ans.)
(ili) Impeller di.ameters at inle and outlet, =
d,
429.2x60 d' d, :
". = = 0.4098 m or 40.98 cm say 41 cm. (Ans.) For iadial blades, work input to the compressor
is'given by,
(iii) Power required, p : 'iZOOOO c?.
Wok done = .Ij-l!
^ = cp(Tz- T)
P = x165.8 = 8.8 x 165.8 = 1459klil. (Ars.) Tggg-
(lu) Eye diameter if hub diameter is 12 Here T, is the final temperature of ai from the exit
cm, d, : of compressor.
Fom continuity equation, wd h"rru
ec2 - Ti1v2
|Iooo x I olsggg.z:?gef'''
o" -_l =L -- *
c,, =
iz=: (dtz - d2) x Cr x pl ll00g*"
L = qqo.+ r^t"
=++u'+
But density at entry is given
4 J
b Also, "'2=d-r!
cu^
60
= q+0.+
Pt =.
- n7',
1x105
^ =
Pt =
281. r 2gg =
1.189 kg/m3
9:4ao'q
-. =
xtOdOO =
0.8411 m or 84.11 cm. (Ans.)
.'. 8.8 = L 6rz -0.L2\ x 145 x 1.189
.'. d, -- 4 =!4
11

) 2_
g.gx4 ' 2 -f = ''o' cm' (Ans')
dt" = + 0.722 =0.07939 m2 (iu) Breadth of impeller a inlet,
n *ffi' b, :
.'. dr = 0.2818 m or 28.2 cm. (Ans.) Volume flow rate = 2vrbrCrh", where ,io is the blade
area coefcient
Example 2o.gg' A centrfugar coinpressor running at 10000
r.p.m. d.eriuers 660 mr/min. Volume flow rate
of free air. Th'e ar is compressed, from 1 iar zo"c lo a pressure ratio of 4 with an isentropic .., b. _
""o impeiler and
efficiencl of 82vo. Btadcs are ra-d,iar at outret ' 2w1.Cr,ho
assumed throughout constant. The outer radius of flow uelocity of 62 mls ntay be
iipeller is tiice the innr n-tn" sp factor (660/60)
may be assumed as 0 9. The blade area coeffrciznt
b, orrum"d 0.g at inret. carculate : = ,"tq.4116lr) = o'1492 m or 14'92 cm' (Ans')
(i) Final temperature of air. ^oy
() power. (u) Impeller blade angle at inlet "6r",
Theoreticat p, :
(iii) Impeller diameters at nlet and, outlet.
(iu) Breaclth of intpeller at inlet. Lanp,=CL" =, 6,
-- =0.2816
(u) Intpeller blade angle at inlet. u, (440.42)
(ui) Diffuser blade angle at inlet. .r pr = tanlr (0.2816) = l5.z$.. (Ans.)
(ui) Diffuser blade angle at inlet, uo :
Solution. Giuen : N = 10000 r.p.m. ; Volume of air delivered, y
= 660 m3lmir.. .
c,, 62
pl = 1bar, Tt=20 +278 =2gSK; rr= 4,nis 0.g2 tan c"
= | C, =62rn/s; ' = Q, .Cot,= 0.9
- x 440.4
- r| lFA/
rr= 2r1; 0" = 0.9 ; Blade area coeflicient, h"= O.g.
(i) Final temperature of air, To : dz = tan-r (0.1564) = 8.9". (Ans.)
2O.4O. A tcntr.ifugal blower compresses 4.8
, . uur' tne Lndex
t' ma/s of air from I bar and 20"C to
^^_"11-Pt: ol compres.sion-n is 1.5. The flow uelocitlt at inlet td outlet of the machine is
- l\ 1.486 rhe sdme and, equar to 6s nt/s.
The inlet and uilet impeller d.iametL: ; are 0.32 nt and, 0.62
Tl respectiuely. The bloruer rotates m
at g000 r.p.m. Catculate :
llo INTERNAL COMRUSTION ENCINES
771
(i) The blade angles at inlet and outlet of the impeller.. AIR COMPRESSORS
(ii) The absolute angle at the tip of tfu impeller.
Example 20.41, A centrifugal conpressor d.eliuers 16.5 kg/s of air wit,h a _total head pres'
(ii) The breadth of bladn at nlet and, outlet. is
sure rato of^4 : 1. The speed of the compressor is 15000 r.p.m. Inlet total head temperature
It may be assumed that no d,iffuser is employed, and the whole pressure increa;e tahes ii;0.C, slip fcrcr 0.9, pouer input factor 1.04 and 807o isentropic effrcency. Calculate :
place in the impeller and the blad,es haue negligible thickness.
(i) Ouerall diameter of the impeller.
S<lution. Giuen : V, = 4.8 m8/s i pr = I ba ; T, = 20 + 278 -_ 29J K ; n = 1.b (ii) Power inPut.
Cn= Cn= 65 n/s ; dr= m;
O.B2 d."= 0.62 m ; N = 8000 r.p.m. Solution. Giuen: th = 16.5 kgls;Pressure ratio, ro, = 4;N= 15000 r'pm' iTot=20 + 273
The temperature at the outlet ofthe compressor,
= 293K
, . Y-I r.6-l Q, = 0.9 ; S. = 1,04 i r\""o= 807o.

T=[a)'=f]!)*-
lt \Prl \1/ =t1447 (i) The overall diameter of the impeller, D :

x 7'1447 - 335'4 K
r r,e p".ipt = 1.486
"."r '"r".,r?;;l1
r = Tot x 1.486 = 293 x 1'486 = 435'4 K
"!=4!
x o'32 x Sooo Toz'
60 -
c^,. = 134 m/s
60
. Toz' -Tot 435.4 - 293
The tip peripheral velocity at ouflet, -
'rsn or
- Tot
roz - Tor' Toz

435.4 -293
=259'7 rt/s
Tor- = 176 I{
""=# Tot = 03
work done = cp(rz' - r) = H*t 4,i* CIt"
... Work ao"" =
AOOe
= co(Tor- Tr)
, C* - 1'00f335'-a-?93) x 1000 = 164.1 m/s
(j) The blade angles at inlet and ouilet of the impeller, p' p, :
^ =l-lis-Cle.-&!1v'-
Cu"
f f
iooo-" r'ooql
L 1'04xoe
rzef'''
= nur., *"
L a'ot I J

ran p, = ... 25.88.. (Ans.) Also, cbh=gY - $7.2


#=* F, =

tan p, = = 0.6?ee ... gz = s4.2.. (Ans.) D'= = o'556? m or 55'67 sm' (Ans')
;h=---lg-
(ii) The absolute angle at the tip of the impeller, og
"'
(ii) Power inPut, P
p = ="#
: :

C, 65 rhco(To"-
tan a' = = 0'3961 c2 = 21'6" (As') "rJx 178 = 2951.7 kW. (Ans')
%= t "' = 16.5 x 1.005
(ttl) The breadth of blade at ^inlet an outlet, b' b, : rsExample 2O,42. The followng data pertain to a centrifugal cotLpressor :
Discharge at the inlet, V, = 2rr
rb, Cn ?otal pressure ratto =3.6:1
4.9=2nx0.16xrx65 Dianeter of nlet eye of compressor impeller =35cm
.'. b, = *#*,. 0.0?34 m or ?.34 cm. (Ans.) Atial uelocitY at inlet = 140 mls
os = Mass flow = 12 hsls
Let V"be the discharge at the outlet, then
The velocity in the deliuerY d.uct = 720 mls
\ __ n.v
vr= +xT' r^ | . ei=te
rt Tz l
I r'ti.rg
The tip speed' of inPeller
Speed of mpeller
= 460 mls
= 16000 r.p.m.
4 P2 |
I

rrl =rr|1 Total heod. isentroPc efficienq = 809o

_ lx--tes-"fftr-
x4.g.. gg5.4
105 Pressure co'efficent = 0.73
=366m3/s Atnbient conditons = 1.013 bar and 15'C.
, Vz= 3.66 = 2ttr. br.Cp=2n x 0.81 x b, x 65 Calculate :
(i) The static pressure and temperature at inlet and outlet of compressor'
Ur=
;*#"* = 0.028s m o 2.Be cm. (Ans.) (ii) The static pressure ratio,
.l
fiil I

iltllT2 (iii) IYorh of compressor per hg of


air.
INTERNAL COMBUSTION ENGINES AIR COMPRESSORS
713
(iu) The theoretical pouer required,. Calculate :
Solution. Giuen : ro (pressure ratio) 8.6 (i) The temperature rise of a,ir passing
= through the compressor.
iitil ?or = 15 + 2?g
= 2BB K; l,." = 0.g
(ii) The static pressure rato.
Assunte that the absorute uerocits of air at inlet and,
iii'i m = 12kCJs; Qo (pressure co_efficient).= 0.?B: Tahe co = 1.005 hJ /hg K.
exit of the compressor dre so,me.

Q tz
= l2o ns Solution. n rDN nx(550/1000)x18000
li{l1 (i) The static prssure and temperature at
inlet and
u, = -60- = ---j0 =, 518.36 m/s.
ou'et of compressor :
Total head temperature rise Work done per kg of air
= (C_, .Co" . Cr, ,C6r) --
l-l I4-1
ilI aro= lbr(rp-r--l) - 288(3.6) 14
-
ris, o.g
-11
= 159.l'C
But e,,=Oand O,= I
"bl t
. To = Tot + ''. Work done per kg of air
z = 2gg + 159.1 = 447.I K. (Ans.) = Clt" Q, = (blg.g6)2 x 0.82 = 220gA1 W
ilr The static temperature at exist is-' ^f0 "
itl
I
o* ..(,
if r*-* n2 (i) Temperarure rise of .",;::o;itl
i,
ii
r =
- 'cp=447.t_
= ,tL
2x1.005x103
Also work done = cr(Tz _ ? )
i
Equating (l) and (l, we get
,il = 447 'l - 7 'l = 44o K' (Ans')
!' The static preEsure at erit is 220.331 220.337
12-rr=
% =
(Ans.)
t^oo5 =2r9.23'c.
n,=no,-L ('n*"* =O?) E'-7,
n"""=;ffi
Pt x 7202
v2 - Po2-'z" po2- 0'057 p2
o'z8?t 440;10- =
But poz = 1.013 x 3.6 = 3.65 bar
i'e'
' o zo = ?-; l9o
219'23
or ".2' = 4G6.6 K.
.. p2 = 9.65 _0.O57
(l) The static pressure ratio .
pror 1.057 pz= 8.65 The static pressure ratio is given by
_ Pz=3.45bar. (Ans.)
To calculate static cond.itons at inlet , .Y 7.4
: P2 f?r')r-t f466.6)r4 -r
,F l4O2 l4O2 o,=lr') =lr*l =4'6e' (Ans')
I
'ol -''
-^
%o = ^Ir + = Tt+ 9.78
t;lOsxlo3 20.4.3.3. Axial Flow Compressor
Tr = 288 - 9.75 = 2?8.25 K. (Ans.) Construction and Working
20.4.3.4.1.
h x l40z In an axial flow compress-or, the flow proceed.s throughout the compressor
(1 - p^' - essentially parallei in a direction
2x 0.297 x 27g.25 x u. = f.013 _ O.I2B p, to the anis of the machine.
-_---_---__-:T Construction. Refer.
Pr = 0.9 bar. (Ans.) 20.61.
o An axial flow compressor-consists of adjacent rows of otor (moving)
P2 blades and stator
(li) Static pressure ratio
=p,= 3'45 oF=3'$' (ans')
(fixed) blades' The otor blades
are mounted on the rotating drum
fixed to the casing stator. one stage of the machine com!.ises ^ "tu1*
blades are
(ijj) Work done on air a row of'rotor brades
= cp, ATo followed by a row ofstator blades.
o,:u;f::"_ir:;::.iJftg of air. (Ans.) o For efficient operation the blades are of air foil section
based on aerodynamic theory.
0u) rheorericar power ."n";: The blades are so designed tlla wasteful losses
due fo shok and turbulence arc pre-
vented and the blades are fee from stalting troubles. (The
bladcs are said Lo be stalled
when the ar stream fails to foltow thc bhe contour).
Whereas the compressor blades
", ,,ofi?I!,l",Tk!,*0,!,:J,r:i7;;1|*;::,':,."':'";j,;ilir"liY;"*'jl.j",)*, have aerofoil section, the turbine blades have profrles
acs' This is so because.the acceleration process being
formecl by a n.,,ier ofcircular
Isentropic total head afficency Q./g carricci out in the convcrgrng
= bl;rde passages of a reaction tubine is
Outer di.ameter of btade tip ared rvith the diffusing or decelerating -r..h -o." cffic.icnt and stabrc proccss as com-
= SS0 mm .r.ocess being cariecr out in thc diverging
Slip factor : irssage betwcen the bladc , of rn axial fl-o*
= 0.82
"o_presso..
-\.&
INTERNAL COMBUSTION ENCINES AIR COMPRESSORS 715

Stator Delivery Note, TVo foms of rotors have been rmed nmely the dnm nd disc types' the ec trpe is used
(Casing)
vanes where consideration of lou weight is more importmt than cost xin oircmft opplicatians- The dnm tyPe
is more suitable for static ind.ustrial applications. In sonie applications, conbination of both types has been
used.
, Air
Materials. The following materials are used for the various components of an axial flow
delivery
-----) compressor I

l. Rotor bladings, The materials listed below are in the ncreosing order of weight and
Air their ability to withstand high temperature :
.
tn
------->
(i) Fibrous composites (id) Aluminium
Drive shaft (iii) Titanium (tu) Steel
(u) Nickel alloY.
>
Ar
delivery 2. Botor:
o For rotor sofs and disc ..........'se1."
o Aircraft engines may use titanium d.t the front stages and' "nckel alloy" in the rest-
Moving S = Stalor (Fixed) blades
blades Fixed R = Rolor (Moving) blades 3. Stator blod'ings :
blades o Same materials as that of rotor but stl is the most cornnlon,
Fig. 20.61. Axial flow comprsor. 4. Co*tings. These may be of cs magnesum, eluminum, steel or iron or fobricated frorn
titanium or steel.
r The annular aea is usually reduced from inlet to outlet ofthe compressor.
This is to NC (Numerically controlled) machines make dies aDd the blades are mrnufactured by
heep the flow uelocity c.onsto,nt throughout the compressor
t"ngtn.'t-lne diverging - precision forging. Blades ae also machined by CNC copying machines.
passages of the moving blades, there is rise in
temperature due io diffusion. The abso- 20.4.3.3.2. Velocity Diagrams and \fork Done of a stage of Axial Flow compre6sors.
lute velocity is also increased due to work input.
The "fixed. blades,' sere the following two purposes :
Fig. 20.62 shows the velocity triangles for one stage of an axial flow compressor. All angles are
measured from the axial direction and the blade velocity C" is taken to be samz at blade entry and
, (t Convert a part of_the K.E, of the fluid into pressure energ!. This conuersion exist. This is because thb air enters and leaves the blades at almost equal ra'd'ii.
-- - is achieued
by diffusion process carried, out in thi drrs" tioirassoges.
(ii) Guide and' red'irect the
flud flow so that entry to the ne stage is without shoch.
Working
tli
Basically, the compression is performed in a similar manner to
that ofthe centrifugal type.
The work input to the rotorshaft_is iransfered by the iovingblailes :.1
to the air, thus accelerating
it. The blades are so aranged_that the spares Letueen the blad,es
hence,th.e uela.ty of the aii rerotiue to thi brades ii-d"rr"or" or^ aiffisrl passages, and.
as the air posses through them,
and' there is a rise in pressure. The air is then
further dffused in the stotor ilades, which are also
arranged to form diffuser passages' In the fixed stator lades the
cir is turned tir.ough an angle I

so tltat its directiort is such that tian be.allowed, to pass


to a second. row of movng rotor blades.
It is usual to haue a relatiuely large number of stageli and. to maintain a cnstunt iork
stage (e.9.,
input per
from 5 to 14 stages have been use).
The necessary rduction in vorume may be allowed by flaring the
- the rotor. It is more common to use a flared rotor, andihis typle is
stator or by flaring
dhown rliagrammati_
uli k- C*.-rl+- c* ------l
cally in Fig. 20.61. ,,.Ar

It is usually arranged to have an equ ar temperature rise in the mouing and, the t;.,
-
;;";;;"; ilin""r'iir"i,o'^ir""ror. fired,
btades, and.'to heep the axiat uerocity lii'i
"t
"f similar with r"god to rnu"
of.,th.e compression is exactiy otr r"ii'ity and blad'
?::!,"r"c,9
tnlet and outlet angles,
A diffusing flow is ress stabre than a converging flow, and for this reason
- shape and profile is much more impbrtant for L compressor
the bratrr
Fig. 20.62. Velocity diagrans for axial flow compressor.
than for
,]:_!?:r1: of compressor blades is based on aerodjrnamic theorya reaction turbine.
and an aerofoil o Air approaches the rotor blade with absolute velocity C, and at an angle or. The relative
so ts used_
velocity C"r, obtained by the vectorial addition of absolute velocity Ct and blade velocity
Co,, has the inclination pt with the axial direction.

t I
ii

INTERNAL COMBUSTION
ENGINES AIR
. Due o diffusion in the diverging
passages formed b
COMPRESSORS
77'7
sure rise. This is at ttr" mtor blades,
ofrelative velocity and so the there is some pes- The second term represents the increment of K.E, in rotating cascade that has to be
""p?.rr"u'o?";iT,:::::E
creases from c,, * q".'^9*oense relative velocity de- -
;;#',;;::"J," converted into pressure euergy in stationary cascade. compaiing this equation to the

wour urtinatl .i;,
The air then enters tn"
J"'T::i: J:1["rr"ffii"::";il;; "i. work input to centrifugal compressor, ." fird that the term'centifusal action
the diverging passase ""t1.to:
rno ot:9.:
r3r: 1"".. ni"rr'iill
rh;;;;, and decereration takes prace
rn curt - c*,"f
rt. reaves_tbe staror brades with I

Ct
"l
'itr'jly"i!T,Hf."t?::,:l;:;:"::":,:,_:::':;""ros,,
r __-._ ,,,o
ceo";il, i,t.-Ii",."
wu.pr.essor stage equals the
approach uelocity
L-:r-i is missing in axial flow compressos. Due to tbis reason the pressure

rao per stege in axal flow cotrLpressor is much less than that of centrifugal
compres-
From the velocity triangles, we sor,
have
The stage temperature dse;dgardless of efficiency of compression, will be given by
c^, . the
equation
4=tancr+tanB, ...(20.85.)
and' (^T),"t= - ran F,) (20.91)
? = F2 + tan a" \rr"^g,
Assume ,l kg of "^ Pressure rise in isentropic flow through a cascade :
flow o/a# through t,,e compressor
....Consider the incompressible isentropic and steady flow through a cascade from uniform
'I --"'Ftwoou
From stage
Newton's s.econd law of molior,, condition 1 to uniform condition 2. From Brnoulli's equation, we
rl 'Iangential force per kg C,z_ have
= C,t
Work absorbed by the stage p., Pt *Ct2 Pz
I
t! of"i., p2p2 - *Cz2
I
-9 = co(r o2 - r or)*, tnut the static isentropic pressure rise may'be expressed in tems.of the inlet dynamic
and c,rare the whirr Y:
^=-!:'9,: ^) *^-,
f,:::".",, "".o"i*iu,"i;;*ft:,;,?lf,-l.,,lTi, and exit;":?:;iJl
( c,' c"'\
y:fflr::fff:t
becausee or
of rne jifl*:'J,Hi
til;i;;--"."
,vur uurponenr ar the
""#,lii\i:!-iie,entrance orthe
ur compress or iis notzero
(Ap),*n = (p2),n -
-Pl-.'-;-l
TL^
th tact
fact that
ai flows
to-ws atially
^_--- for work donr
The expression
axially an
and not
not rad.ally.)
"";;"i;;"^"ru'
ne compressor
\)
be put in terms of flodaxial
w^. = Cu a- (tan
c,, C, (t^^ dog +-- -',mar
tan ar)
velocity and air angles.
= (Pz),,n - tl(rr' + c,r2) - (c22 + c,22)l
!" -
From eqns. (20.85) and (20.8'6), .(20.88) Since Cn= C2= C
we have
(4p),.o= 2
t ".r, - C,r\ = cf &anzcr- tan2 cr) ...(20,',2)
t'.' C,r= Crtan a, and C,"= Cltan o"zl
20.4.3.3.3. Degree of Reaction
Degree of reaction (.R ) is defined as the rato of pressure rise in
the compressor stage.
_ hessure rise in the rotor blades
Wn = C6" Crz - Cbt, Crt -o=@
since C,z- c,,t =.r1, _ tiu-,
j _ Pressure rise in the compressor stage wrk input per stage and is
Futher = c,*, _ c,*, eq,,,"'"ls
C^ = Cw, C ,
= ",*> is modified
the above equation ",*))as, gu, (C,, -Crr)
=
Pressure rise in the rotor blades is at the expense ofK.E. and is
By use orverocir, ,.i"n*rli|] ;l;r;*], = cu, ac,, = c, (ar)*,
c,,t -cr"t
w"'= 9'it'i* c'":c" ...(2o.eo)
2

teh) (second
:' C"' -C"'
c, = ,)'""tt"'t' R, = 2Cil (C*, c.,) ...(20.93)
"": The _ term " -
- first on the rieit sie of.the. above
equation introduces the part Reler inlet and outlet velocity triangles
ili:L',i#;i:fi3ffi:**'"'which is ;;";J;;;";" of the work
pressure aue to dirrusion
action
:

C*, = Cu, - Crtan P,


Cr, = Qur- Crtan Bt
AIR COMPRESSORS
INTERNAL COMBUSTION ENCINES
Let us conside the compression process ofa multistage compressor on ?-s plot ofFig. 20.63.
Cr" - Cr, = C(tan tan pr)
9 - .(20.94)
Similarly fiom velocity triangles,

c,,2 = (C)' + (c, tan pr)2

So
crr'

o*ffi=;;i
##+1ft*=!
C""2 = (C)" +

- c"r' = cfz
(Crbn
(tan2 F1 -
Fz)2

tan2 gr

cr
(tan p, + tan Br)
...(20,95) IrT-
AT^ I
Kr,/o^,.,
'.'---t-
(stage)
Degree of reaction is usually kept
as 0.5, lmacnine) m/
|
IC
ub= -'C" (*0r+tanPr)
C^,

6 =r* F1+tanP,
C^t
But q1 + tan
C, = tan Fr = tan o9 + tan B, (from velocity triangles)

.. 9w
C, = tan pr + tan 9z = tan crl + tan Fl
= ta., g'r+ tan p,
From this
so with uo* Fig. 20.63. Concept ofpolytmpic efficiency.
type of set up losses in '"""Jiff;fr,!;",**rressors haue svmmetrcar brades and. wth ths
flow path
In symtnetrical bl"e., ,O
";'" ii)"r_". pop
The gas is being compressed from pressurepor hpo, in four stages ofequal pressure ratio.
po and poo are the intermediate pressures.
20'4'3'8'4' !:
**t""rt':;;lTJ;*""l"oron""
and. fluid. friction losses are mininutn.
Now by definition :
work input to a cornpressor, with usual
notarions,
Overall isentropic e{Iiciency (stagnation) fot the m.achine (mlc),
(T' \ A/n
=c^(Tno,-r^,rr(Tor-4\
P w \isenVntc) =
"''
(4"-4T)=""1;4)xQ*'-r') Stagnation isentropic efliciency fo a
-
sa,e, (s)

=
"01,""or,,(X- tl
\.or J= u- n*,
--. 4,fl&n)-
".l(pr) rl
I
...(2o.s7) n. ..=dro'
'14?4(3,
dq
Eqn' (20'97) indicates that for the same The total actual ternperature rise A?o can be represented as : I
isentropic efficiency rl"n and pressure ratio po2
, (AT"')*"ti*
the work input is proportonal. to ihe
initiat temperature. Thus in a compressor M,_ "'(t)
several stages bf equal isentropi" coosistirrf,olr 'lie (n/c)
because it has to deal with a fl-"ia "m"i"oqr,
u".-"""Ja"g stage wilr have io perform more work E(dro')w
ofitt"."rrLi"-;"."t"r" eriveredto it from tire preceding stage. or tT^ = ...(tr)
" l*(o
;H,'j""fffi:i;"J*:".*Tf':'"";rili;il.;,,1ii'*","u"" orthe overar peJrormance orthe ' Equating expressions (i) and (ii), we get
efficienq,;ii;";;";;;:":"#f;.y;::#:#n::;#r,i:!:;::i:;;"s,i"et"tio. ouerau (LTo', z (dTo'
I

the perform"n.u of.orrrp."J"o"rn"irh


)
^o"hio"
_ )
"bsc
diff"""nt stages, rhe concept ofpolytropic \isen (ml c\ 'lise (r)
""",""[rlillTflling
Polytropic efficiency ,:^r!:.:::r:or:,efficienqr of one \en(m/c\
stage efficiencv is constant stdge of d multstage compre..sor. or -(ATo')mrchire ..(20.98)
,'rl;"":tt' f"; "t;';;;;;;" q a cotnpressor with inlinite number of Iise(sr) L(dTo')"b8"
I
INTERNAL COMBUSTfON
ENGINES AIR COMPRESSORS
From Fig. 20.63, we have :
(LTo')_*h* Thus, by the definition ofpolytropic or small
= (l_/.) + (t,_m,) +(m,,-n,) +(n,_2,) stage efficienc we get :
1i
on ?.s prot, ,h" v- r'\ r f
".:(fl",;.,.1::tirJ.::_y);::r::":f ;r,-' T, dP.lt;I )P"
rl-=*'o l:
l_m, > l,-m" \
' d?. = -,^("-I\\
^
,n_h, )
m,,_r, '-\ n )po
')
say that E(dTo)ou" > (LTo,)*"u.n"
;:.:"":1
rro:t eqn 20'98, r,.n("r) ) or - _lr-r)1, ,
l:tussmall stage
fr",
_,^__ and is
ciency
ron (-r")
effiaency \",1, uhich is consto
desgnited"by \r. nt for all stages is called polytropic
",=l Jl"-r)
y
Eqn 20.103 gives the value ofpolytropic efficiency
...(2o.roa)
effi- in tems of exponent z and the adiabatic
"Pllvjropic exponent y.
.--_ efficieny' in terms of entrv s-nd deli.very pressure
tures and ttr. r"tio .i and tempera-
heats . Substituting the value of
"iecifrc
Refer Fig. 20.63. The -u from eqn. (20.101) into eqn. (20.103) we have
actual luPcssron path l-2
2 are no equlilrium;";';i"*"": is-ineuersble, bu, the end ,Il :
equtibrium and. tie on
onrr::1:'::liT":?in^1,?t:,:'*":rs.bte,-but
the same polytropic path states r and
,n"r""tnrX"J'tl*,
. = Lrf .dqn.
r'"
tt^
.
Let the corresponding reveiilu
p"uor = constant, zl podTo
i.u"t"opi.;:th
=I -
1-2, with eud states r and 2, chaacterised
n^
dTo 1 dP,

.
b rreversible Pour = conatant zz
-t;tpo
Integrating between the two end states 1 and 2, we get
or path, we may write
(m \ / \
P6 = Z1Po" '" \Tot)
n"lnl3l=r-rnfPozl
y -"[ro,i
On differentiation, we get d,po= zrnpon-t.d,p.= n le dp^
Now, the characteristic gas equation
..(20.99) TLr.fpor') ,^lrr)tl
.", O" *",r1"," ..
n,= -#=
^,"'t_t".t-l t?'*" .,(2o:o4)
t="" or Po -
p^
l"l*l rr./ lnll!l
RT" \ [4rl
on differentiarion, we get dp"= The eqn (20'704) gives the polytropic efficiency
;l+*";J:llZ1 ...(20.100)
in terms of pressure ratio @ , ,emperaturc
From eqns. 20.99 and 20.100, 'o J . T^^ pol
we haive ratb and the ratio of specific heat
,*
r01 y.

a""= 20'4'3'3'5' Flow coefficient, Head or Work


"u![ut*tu]= "
.r. *[L,ufir"] Pressure Coefficient coefficient, Deflection coefficient and
l' Flow coefficient (Q/ The flow coefficient ofaxial frow
compressor stage is defined as.
=
"l!"k"_"!e.] c",
"r_ Co,
dpo= n.dpo
-# Since Cot = C,,t + C* = Cn (tan B, + tan cr)
o . c",
,'
-#" = ndp.-dp"= dp"(n-t) 0-=
' c1(tan Bl + ran crl) - rr;;;;n
al
...(20.105)
Also,
'|)r Actual stage temperaturo, dT^
dp^ll.-1) e
C1= C2= C
=
(20 rol) O,= Cr' I
sjmirr tearment ro the,o"rr "' 'r- clt^";;; --
=
p, + ta" o, . '(20.106)
the stage isentropic "",nor!.I "l::r",
t"-fo"utu." a4, expressed :ss following tltelaw pooo"r= 7, woukl give
as 2. Head or work coefficient (Q). It is d.efined "rn
as the ratio of actual worh d.one to the
dT; = dpal
/^, ,\ -
,-'15 hinetic energyr correspond,ing to the meon peripheral
uelocity. .Ihus.
\ Y )P' (2r, 102) cd ,r
r, - 2+
"bI l.
=?9+-
eL. =z [!els,:: ", ]
\ tan p2 + Lan (J2.)
.(20.707 )
783
NTERNAL COMBUSTION ENGINES COMPRESSORS

3. Deflection coefficient (0i). It is defined as, The overall pressue ratio is given by
,, = [t.oL,*"]" ..,(20.113)
oo",=4&#=C'tC"
"' crf ct
or Q=2Q"r ...(20.108)

4. Pressure coef;ticient (0,). It s defined as tlu ratio of isentropic worh done to kinetic ,o=,ln ll(?) , ...(20.114)
eilergy corresponding to the peripheral ueloclfy. Thus, [ro)"6""1
c- LT.--- (ro)rros" varies from l.l2 to l.2'
q,=VlE =r.o 0 ...(20.109)
where
20.4.3.3.7. Losses in Axisl Flow Compressor Stage
20.4.3.3.6. Pressure Increase in a Sage of en Axial Flow Compressor and Number In actual practice, various losses occur while the fluid flows through a compressor $tage.
of Sagee The total pressure loss arises in three ways:
The pressure ratio is expressed as 1. Profile losses on the surface ofthe blades'
2. Skin friction on the annulus walls'
T,-TrT"'-'
Pt=f,*.
P,
(sei Tr ...(20.110) 3. SecondarY flow losses'
L- I The various losses represented on graph between stage efliciency and flow
coeflicient is
Ict ?, and ?, denote the temperature ofthe working fluid at inlet and outlet of rotdting shown in Fig. 20.64.
blades. Hence the temperature increase is
C't! =C'22
aTP- T'- T'
' - ZXcp
100

+
... P2=fr*n" 4"n|'tt-tt
Pr L -'41 ...(20.r11) 190
I
I

The temperature rise in stationary blods is given by, 6

c"'-c"' ao
,r5= r3- ^
tr=-1"",
c
@
. _ n/(1-1)
pl=lr+n-.951 g' 70
... g
U'

Hence pressure l, ro! ,r";;""'4J brades is


"r" 60L
0.5 o.7 0.9 1.1 1.3 1.5 1.7
_ pz= pzf{r. n" +}"1-t _ rl Flow coaffclent -------|
4p" =ps
Lt tz) l Fig. 20.64. Losses in @mpressor stege.
The pressure increase in a stage is
&"'= &*+ LP" 1. Profile lossee on the urface of the blodee :

By profile losses, we rrean the total pressure loss of two dimensional rectilinear
cascade
and LT"r= L!*1 7" r.
-
The stagnation pressure ratio is given by
;t"i;; f; tn "n" friction on te surface and due to the mixing of flow particles
after the blades.
poz ,t
n
or"T"' '
noFTIJ These losses are usually determined experimentally'
=l
Pot L- 2" Skin frlction loss on the annulus walls :

'
on the annulus
If the work done per stage is assumed to be the same, then the number of stages (N) is given r The wall fiction total pressure losses arising from the skin friction layer growth on
by, walls and the secondary losses ae dfficult io anolyse as bound'ary
these walls s a complex three'd'mensonal phenomena'
* =#_" ...(20.112) e Empirical relations (by Howll, Haller) are available for calculating drag coeflicient'
3. Secondary flow losse :
If the pressure ratio per stage b the same, then prod,uced' by
r In an axial flow compressor blade channels, certairi secondary flows are
(r) _ Poz-Pos*Po(tv+r) combned' effects of curuature and boundary layer'
'P'stdsc por poz poN oSecond'aryflowisprd,uceilwhenastreamwiscomponentsofveloctyisdeveloped

*
INTERNAL COMBUSTION ENCINES
from the deflecon__of AIR COMPRESSORS
an initially sheared
developed pipe flow:-o!".-. u ""a, flow. such secondary flow occurs when a ^l)
*hen .*"heared flow pr"r"" oo""
thickness or an aerofoil of finite
liit a aerofoil offinite on the section lMof the curve, the flow is not stabre. A fa]l in mass
flow rate wil' be
boundary level meets an obstacle
it i" "ll1" aa wind
fl;;j;; ffi normar accompanied by a fall in pessure ratio. In this situation
any small disturbance caus-
. f1,3:iltjil",j'l 't'lctr browing pasr a relegraph pole).
of secondarv-no," *'o'"',,, uiat
iirg a cbeck in rnass flow wilr cause a fall in pressure ratio and t},e
flow muy reuerse dv
machinery turbo- some point. when the temporary disturbance is rernoved,
"","':;::T:3;::;::"3":;i:
orthemchine,ou,aert,i"L;;:#-;;il:i:f:;r"tr:#i?,f*;"ohubwatis the flow wilr pick up and it is
found that small disturbances cause the flow to oscillate rapidly.
The oscillations is
20'4'3'3'8' Surging, choking and noisy and can, ifallowed to continue, cause stuctural damage
stauig-compressor characteristics in the compressor. It is
called 'surge' and the point M on the curve marks
nsurging'is the rimit of ,riuf.,l operation of the
:g':
when
qi[#lT#,X X:;?:i;:i:#;Ti;n*:"' an uwtabte timit oropera.
the-coip,i"'li"ii,-r',r",,'iii1l:""Ji1:'"-::d r11|sat of flow throush tt n
compressor. If a compressor is runni.g normally at the point
where-surge usually
commences it is possible to induce surge rnerely by passing
correspond.ins to *"'ti,,i,i!"',,,ir:l'."F"1fi "opi,,l, ihe hand across the inlet. It
value, this surge can.reach iu"r, rk^:J"*ff":^""#Kxl^("'iiyi*gili:; is found that compressor efficiency is highest at point adjacent
advisable to able to operate as close to M u oossibk.
to M and it is therefore
mechanical failures mav rl,sutL ".mrg,,ii;J"1"'##a""rr the compresso and rn many cases
r" 'stalling. "stallng" ofa stage ofaxiar ftow
"'rt"."r"tri; "'..". to which the rotor of the machine is . is defined as the aerod,ynanc stall
ily""::,:":Ht5.:Httrf, :'tril":fil{*if .i,ii{:#tr:;:r,":*:a;Hro,orbrading orthe-breakwayoftheflowfromsuctionsideofthebladeaerofoil.Itmaybe
"o^pr"""o,
than designed value or dueto non'uniforrnity'in tn" ite profite.
Thus stalling is ohead plrcnorn-
d.uetolesserflowrate
Choking. When the Dressuro rnt;^ :- ,,-:.-.-,.- enon of surging,
nr"x:i;ii'"';xifi;":[:J:*"i;t'#.-"ii!;;:ilrrfi rr!.'!:::::;:! , A multi-stage compressor may operate stable in the unsurged
stages stalled and rest of the stages unstalled. In other
region with one o more ofthe

;::;:::":";"T:,""n j::":,:xil{;::::r:;i#"#r#:':',:;",:;i";!i::,:,
word{ snrtng is a locar phenomenon
whereas surging is a complete system phenomena.
20.4.3.3.9. Performance of Axial Flow Compressor
Fig. 20.65, shows the compressor
In the compressor wh*: tl,u flow is"horo"t"-rirti"".
o Fig' 20'66 (c) shows the relationship between pressure ratio, power and efficiency uer-
. against ,i"1""".,,"u gradient the incidence ross szs flow rate for yarious values of speeds such as Nr, N;,;".-A;
rncorrect fluid angles retarive to
t,. 1"a". i".uil;l: due to i certoin
,*o::qJ.i.".r*",r.r," (r,) falls efficiency increases as the flow rote increases and,echels'
a marimum varue"pnea,
after
the desis'n p"i"i. rm.'m.,
,"1"T:'J"i#:::?::::lf"::f
ss llow rate, gives a pressure
added to the rricrion ross which
w'r ylicn it d'ecreases. Accordingry as the flow rate rncreases the power consuned, also
tncreases.
Fig. 20.65. ratio-mass n"* i"t"."i"1.. as
shown in

+
I
.e
i6
+ e
I
I
1
I
* E
E
l
I .9
:
E
u
E

o o
3
L

Flow rate
-----.} Volume flow rate ------|
(s)
Mass flow rate, ir _______) to'
Fig. 20.66. perfomance cunes of axiul flo* ao-pru."or.
Fig. 20.65. The compressor a Fig. 20.66 () shows the performance and constant efliciency
characteristie. curves,
such a plot does not take into account the varying inlet temperaturc
is cttohe! and is passrnsrrre maximum and pressure. In
: :fi:':"*":#;T:1lT:i rnass flou rate. addition to this, these plots cannot show the coriparison of performance
compressors of diffeent sizes. To, account for alr tirese,
for similar
a rise in pressure *u" *fii"","11]":i;::::j:"t#?ji mass now."i" *irr*"".,,rt i" plotted' with'dimensionless parameters'. These dimensionless
the performan"" or"
parameters are "urii"'
: pressure

2e -42 ; speed paramete., l/' flow parametcr


toJrL.,
'
Pt "u nu.. Fig.2o.67( and r.
Jt p1
786 INTERNAL COMBUSTION ENCINES AIR COMPRESSORS 787
I

Adoptability Ailaptability to Iow speed drive Adaptability to high speed,


low maintenance cost driv-
+ 1.0
ers such as tubines 'i
4
)
I
E U.
12. Operoting attention More Iss
1 co ^^ u-o

6 ? u.b 13. Mxing of worhing fluid uith AJways a chance No chance


e 3 6 /N\ 1.0
lubricating oil

o
o.
74. Suitability For low, medium and high pres- For low and medium pres-
6 sures md low and medium gas sus mdlarge gas volumes.
volmes.
0 0.4 0.8
mat., 20.6. COMPARISON BETWEEN RECIPROCATING AND ROTARY AIR
por
COMPRESSORS
mlot
Por S.No. Aspcts Reciprocating air mmpressors Rotary air compressors
(a) ()
l. Suitability Stable for low discharge ofair Suitable for handling large
Fig.20.67 at high pessure volmes of ai at low pressures.
20.5. COMPARISON BETWEEN RECIPROCATING AND CEIVTRIFUGAL Operotional speed Iw Usuallyhigh
COMPRESSOR,S Air supply hating Continuous
Balancing Cyclic vibrations occur Isqvibrations
Reciprocating
Lubricdting slsten Genually omplieted Generally simple lubrication
systems are required
cmpresaors
6. QuaLity of air dcliuered Generally contaminated with oil Ai delivered is relatively more
Vibration probkms Geatr bration problems (due Less vibrational problems clem.
to the presence ofrtriprocating since the machine dos not 7. Air atmpressor size Iange for the given discharge Small for same discharge
parts the machine is poorly bal.
mced)
have reciprccating parts. 8. Free airhondhd
Delivery pressure
250-300ms/min
Hi&
200G-3000m/min .{ rl
9. Low
Mechanical effrciznc1 Iwer (due to the presence of I{8her ompuatively (due to 1U. Usual standard of compression Isothemal compression Isentropic compression
several sliding or bearing rnem- the absence of numerous i$
bers) sling or bearing membere)
Installed. first-cost
20.7. COMPARISON BETWEEN CENTRIFUGAL AND AXIAL FLOW COMPRESSORS 1
Higher Iwer (where pr*sue md
volume conditions are il
favourable. 'l
Pressure ratio per stage About 5 to 8 About 3 to 4.5.
Type of flow Axial (Parallel to the direction Rdial ri

ofaxis ofthe machine)


Capability to dzliuer pressure .High presue @y nultistaging, Medim pressme (By multi-
high deverypressue upto 5000 Pressure ratio per stage High, about 4.5 : J. Thus unit is Inw, about 1.2 : l This is due
staging, the delivery pres- id
atm. may be achieved), conpdct to absence ofcentrifugal ac-
sue upto 400 atm, may be lg
In suprsonic compressors, tion. To achieve the pressure
achived). -the pressue tfl
ratio is about 10 ratio equal to that per stage
C apabilty
llilil
of dzliue r ing uol u me Small (By using multicynders, Geater (per uit of building but at the cost ofefficiency. Op. in centrifugal compressor 10
of airl gal, the volume ntar be increased,). space). eration is no so difficult and to 20 stages are required. lll
risky. Thus the unit is le ss conpoct
7. F lexibility in npacity and p re s - Greate No flexibility in capac'ty and and,Iess rugged.
sure ranSe pressue mnge. il
Isothermol efficiercy About 80 to 827o About 86 to 887o (rvith mod-
. Maintenonce etpenses Higher Lower ern aerofoil blades) it
9. Continuiti of seruice Lesser Geater Frontal area Inrger Smalle (This makes the jt
10 Conpression efficienqr Higher, at compression ratio axial flow compressors more
Ffigher, at compression ratio
sutable for jet engnes due ti
above 2. less than 2. il
I
,I
I
I
I

t I
INTERNAL COMBUSTON ENCTNES
COMPRESSORa
:
ArR
1 --;:
to.r:
(dye to adjustable pre_
whirl and ffuservmes) (li) The work done per kg of air, W : ^n
o. Part load.performance
Better W = Co, Cr(tan c, _ tan og)
Poo
7. Effect of deposit foimation on 240 x 190 .
Pe rfo r ma n ce not ad.uerseb
the surface of impeller rotor
affectcd
Petformne aduerselt = -;Oa-ttun 45" - tan 14") = 34.29 krtr. (Ans.)
affected
8. Star ting w Example 20'43. An axiar flow cotnpressor hauing eight stages
to rq req uire d, Low. design cornpresses air in the pressure ratio of 4 and, wth s,vo reacti.on
Hish : 1. The air enters the compressor at 20"c and,
o S uitab ilitl fo r m ultis tdging Slightlydilotr flows through it with a constatut speed, of gT'mrs. The rotating blad,es
.
More suitable for a mean speed of 180 ml s. Isentropic efficiency of thz, of cotnpressor rotate with
multistaging compresslr *oy b" ioninZ," b%. cot."ulote :
Delive ry pres s u re possible (i) Worh done by the machine (ii) BIaO", onrr"r.
Upto 40O bar upto 20 bar Assume I = 1.4 and c, 1.005 tal / hg K.
Applications =
Us.ed in blowing cngines
in stel Mostly used in jet ensines
mUs,.low prssure refrigeration- Solution,
(due to higher efficienci
org central air conditioninc and
ptants, fetiliser and
smalle frontal aea). Also
indutryl preferred in power plant
superchargilg I.C. engines, eas
g; tubines and steel mills. Also
t" Iong disrance pipe
ilI?,i:
rli Previously it was ued
rij -engrnes in jet
I
Effubncy vs. speed, curve More flat (Fig. 24.68 )

lVork required./kg = co(Tr- Ir) = 1.005(466.6 - 2gB) = 174.47 kJ/ke. (Ans.)


Now, work done/kg * Ctt(C., - C*r)
= Numbe of stages
Cmlritugal
L74.47 = 8 x Cu, Cr(tan tan crr) [Refer Fig. 20.621
% -
"' tan c,-tan o, =
{fiffi = r.rau
Ft.or 50Vo reaction blading, a2 = g, and c,,
= B,
1.346 = tan 91 - tan c,

Now, tancr+tanp,= I = # =,
-Lf9U
."., tan p, - tan al = 1.g46
...(r)
tanpr+tancr=2
Exampre 2o.44. rhe ...Qi)
fouowing axiar ftow compressor: From (j) and (ji), we get
Cut=240tn/s, Cr= ""r"::":::: "n
lS0 mls, % =45"; 2 tan pl = 3.346
Calculate. o= r0., p= i hglm'
(i) The pressure rise
1 Pr = 59'1' = crz. (Ans.)
(ii 7" worh and d. - l8.t'= R la-. \
Solution. Giuen : Co, 240 m/s . d,one per kg of air.
= ; C, = 190 m/s ; al = 45" i Exampre 2o.46. An an ouera, isentropic efficiency of BSVo
(i) The pressure rise,
Ap :
az = 74",p = I kelma ".;;rlllr;ru,Z;,,lfi)o
it in the presrur, ,otio of 4 : r. The mean blade speed. and.
The pressure rise through a ring
of rotating blades,
!:i::,:t::, ?!,Lan.d,
compresses
ueoclty are constant throughout the. compressor.
Assu*ing
f.tow
50Eo reaction Itl"rig and t.hing
blade uelocity as 180 nrls in worh nprt'g"to, a, i,"i|, calculate :
Ae = gcfftan2 c, (i) FIou uelocity
- tan2 q) ...lEqn. (20.s2)l (ii) Number of stages
(tgot"'
= 1" t(tun 4S.)2 _ (tan Taht: a, = I2', Ft = 42".
2^ l's- 14")2J = 0.169 bar. (Ans.)

,-.--*J
INTERNAL COMBUSTION ENGINES AIR COMPRESSORS lf

Solution. Given i 1".n = 85%, T1= 2O + 273 = 293 K Po


Pressure ratio, PI =,C.=180m/s Po(Fl)
Po(rH)
Work input factor = O.82
t-l L4-l
t2
ll =[ul'
;- (4)-n- = 1.486
\&/ K- C"r-d
'2 = 293 x 1.486 = 435.4K Co
Tz'-Tt ---il
Now 4i*o = Tz
-Tt
-
435.4 293
0.85 = -{i-rr3
?z = 460.5 K
Theoretical work required per kg
K- C------
= co(T2- fl) = 1.005(460 .5 _ 2gB) = 168.98 kJ <c
Q
Frorn velocity As (Fig. 20.62) ----d
F\9.20.69
9u-
g, -,
=tan or+tanFr= tan 12.+tan 42. =9.212 +0.9= 1.112 Let suffix N denotes the number of stages.
With isentropic compression the tempergture of air leaving the compressor stage is
ur=
cu 180
7 1(1-Vy) L4 - I
Tfr, = 1It2 = 161.8m/s. (Ans.) ?* = trl@l =zeox(b)T = 45e.3 K
Work done per stage = Cu(Cr, -Cr,) x work input factor \ Por ,/

Now, C,, = Crt^o ob = 16f.8 tan 42. = t15.7 mls ('.' % = 9r)
But 1*"=++
- 0r oN dj
I
and Cr, = Cr tan ol = 161.8 ten 12' 34.4 m,/s
.'. Work done
=
per stage = 18O(145 .7 _ 94.4) x 0.82 x 10-s kJ/kg
se2 = gl1::eo
To -290
= 16.4 kJkg
." Numberofstages = = to
459.3_-290
-t*= +290 = 474K
i.e., llumber of stagee
ffi "' -
o.9z
= 10. (Ans.) Tle work consumed by he cornpressor iil
Example 2o'47. In an eight stage exiat flow cornpressar, the ouerall stagnation pressure = cp(Ton - Tor) = (C.r- Cry' Cu, x N
ratio achieued is 5 : 7 with an,oueroll i.sentropi efftciencl of g2 per tan crr) Cr, . N
tenperdture and, pressure at inlet are 290 K anct 7 bar. Th work-is
cent. The nlet stagnatian co(Tr - f,r) = Ct (tan % -
tfl
stages' The mean blade speed. is 760 mls dnd.507o reaction
d.ivided, equalll, between the
..' -T.) Loos(474 - 290)x 1os
c"(T6 IH
"' = qcrl- = ---ld;166;- = r.oro
d.esign is used,. The axiol uelocity tan d2 - tan " '(')
through the compressor is constont and is equar to g0 m/s.
careurate :
angles.
(i) The bladz From velocity triangles, we have
() The power required,.
sglution'Giuqt: N=j,?-=5.: l tbar;cr,=160m/s
C*,
*" !99
. irla*o =92%;,Tor=290Kips1 =
= cr + tan P, = = 1.77s "'(ii)
; Degree of rea ction = i C = 0 m/s.
5i0Vo :!
(j) The blade angles, a' Adding (i) and (ii), we get
F' ob, gg : ; tu

- Refer to Fig. 20'69 for velociy diagrans. Since the ilegree


ofreaction is S0% reaction, the 1rrFr=1.605 + 1.778 ('.' rt
blades are symrnetrcal and hence the velocity diagrams =1.6915 ctt = 01) i

are identicar. Thts


cr=prandog=p, Fr = tan-' (1.6915) = 59.4" t{
Fr=%=59.4.. (Ans.)
I
tlr
'li'
ir
792
lt INTERNAL COMBUSTION ENGINES
I AR COMPRESSORS
i
Putting the value of tan
F' in q, we have
tan crr + 1.6915 1.??g
=
tan s1 = 0.0865 or cr1 t_r (0.0365)
= = 4.94" = 445.8 K
ar = Fz = .9". ftt .l
^
)'. .
(,,'rne
power required by compressor p
: te ."n = 0.8b = 3v'- 4r - tl'e - eoo
p ,h cp(Tar_
= To) 1r _"r _300 ?il
l,
x 1.005(474 - 290) 184.9 Kw' 445-8-goo
i
mple 2o.48. In an aia] flow
=
_r = kW. (Ans')
(Ans.) ro" - + goo
= 4?1.s K
cornpressor,;;;"r"';r'"
I

i 'th ouerall
i.s 4 with ' -- '-"* ww'!'Pt vsor' tne ouerd.ll stdgnaton
stagna,tonpressure
pressure ratio achieued
stagnation isiiroon oaio^^, or ,^-
stagnation . rninu achieued,
, 0", jTi*:::,!u--fZ"nt. ;;;;;;'^'),,,sure and ,.d
",o tii!'io^!r::"Wtt;"n,i;;:i,:';i:'tr;::*;:":"::;:,:;z
rcmperd.ture are
I
':rf^::l:::T:,!j,"#;"if1,f;:y lnlot I
pol
1

i:,f;
t ly. Tr;:".:ii'tr,?,;:"#:1i';;;;;:i::;*:ffi:':::';'!'Jh'""J1:fr:::{,'::::;::,::!i
i he w r k
u e
o d.o
"c
ne f"" ; ; 1; ;. ;. ;;;;:;
r", "' -
\ ./
...[Eqn. (20.104)]
(i) Stagnotion polytropic
effcien*. ;f4tl-
(ii) Number of stages. [4r]
(iii) Inlet ten perature t{_1
,j and. piessure. ln (4) t1
(iu) Bo" height in the _ 0.996
lill firit stage if the hub-tip ratio is 0.42, mass 'b = ;}zrsJ = o-4* = 0'8761 or 87.617o. (Ans.)
ii
Solution. Giuen: r^= 291'- flow rate 1g.5 kgls.
- po, =4 ir1a"n.= 85vo ; por= I bar; ?nr \3oo/
.ilt'ij Pol =300K (jj) Number of stages, N ;
From the configuration ofvelocity triangles,
* = ,{.;J:: -" ; Degree or reaction, 0'5. work done facror = 0.9 Hub-tip
=
; atio = 0,42 ; 9a = tan qr + tan
the inlet and ourrer verocity ; 9r = tan 12" + tan 82" = g.gg74
""""" ff:if3
,3 :LI\ul!;::""ton, diagrams are idenricar.
/ :!- = lto -= 214.e vts
c,=
0.8374 0.8374
= "
Now, C*r= Crtan \ = 214.9 x tan 12. 45.68
= m/s
C_r= Crtan rl"2= 2L4.9 x tan 32" 134.28
work consumed per stage = m/s

= C(C.2_ Crr) x work done factor


[Eqn. 20.g2)l
_ 180 (184.28 _ 45.68) x
=-.---0.9 =- 14'35 kJ/kg
Total work consumed o,
K--c" * "''l
,n;"#o
:o(To, - = 1.005(a?1.5
",r)
- 900) = 172.86 kJA<s
.'. Number of stages, I.{ tj#
= = t2 stages. (Ans.)
Inlet temperature and pressure, T,, p, :
(i)
The absolute velocitv
c, at exit t o* thl giri"t ,*.r." r.d
first stage' approaching to moving brades of

cr 2L4.g
"t = *fi =
lF = 219.7 nJs
'Te-pe"atu"e, """
K_-- c,.___N T,= T.t-S=rOO Q1g.T2
- z-_x l-|O$ilooo
(Ans.) K'
*- Cu zco = 276
-------}| Assuming eversible flow through the guide
q =D2=12:(\=91=32 vanes put ahead ofthe first stage,
Fig.20.?0
(l) Stagnation polytropic
efficiencv.
rne remperature at the end ofthe n ,
t =la)'-
Por \T"r)
stage due to iscntropic expansion 1.4
"o-pr.J*ioo is i (
r7A\- ,
I P1 = 1xi#1"--' ..0.2469bar. (Ans.)
\ uu/
"195
794 INTERNAL COMBUSTION ENCINES AlR COMPRESSORS

(iu) Blade height in the rst stage, I : /l-r\l " )


on=l* 0.88 x 1.4
The density of air approaching to first stage, or
J["-J or
n-l (L4-1)
o=ojn ' During polytropic compression fom first stage to last, we have
p, = += I=t9' = o.e42s ks/ma
RTt 287 x276 , .
n
From the continuity equation, P"lv 1irl"-I-f - 5064'1"--=5.56?
-anc
=f(4"1
Ptqpr= th = 19.5
Por \290/
Pov = 1 x 5.567 = 5.567 bar'
.'. (Ans')
o.9429 x nrr2l _ (o.42)1 x 241.9 = 19.5
(ii) Number of stages, N :
or I
rr=l_l rs.s f" =0.1g1gmor18.1gcrn As the pressure ratio for each stage is the same hence
10.9429 x z x fi _ (o.4D\ x 2aL9 l Poz Pos Po
= = =...... Por
But L = O.SZ .'. rA = 18.18 x 0.42 = 2.686 cm
Por Poz Po3 Po(N - 1)

t where suffix N indicates the number of stages.


Hence height of the blade in the first stage,
I= rt- rtN
rn = 18.18 - 7.636 = 10.544 cm. (Ans.)
lPlzl =!s!-
Exemple 20.49, A multistqe atinr flow compressor d.erivers 20 kgts of air. The inlet \ /
Pot Por
stagnation condition is 1 bar and 17"c. The power consumed
fur the compissor s 4s50 kw. Taking log on both the sides, we get
Calculate: /\/\
() Deliuery pressure. ry= r"ll=l,l&Ll
(ii) Number of stages. lpot/ \Por./
(iii) Ouerall isentropic efficiency of the compressor.
Assume ternperature rise in th first stage is rs"c, the polytropi.c effvienca of compresson
r" lau')
( por
l\ = ----7-.-/ =:---;:-;;=
ln (5.56?)
is 88Vo and the stage stagnaton pressure ratio s constant. or rn (rrb'
= 11' (Ans')

Solution. Giuen : tiz =^?9kg/"i p,r = 1 bar ; I,, = l7 + 275= 290 K p 4350 kW ln f &
\ por
I l

; = ,/ i
= I5 + 290 = 305 K ; r1, = ggg,
;
Toz
(iii) Overall isentropic efficiency of the compressor, (o""t )*o 3

(l) Delivery pressurc, p*:


with isentropic compression the temperature of air leaving the compressor is, I

,. .1-' r4-t il
r"faa); ,nl")o*' T ^i = T^,f &L I t = 2e0 (5.56? 14 = 41s.6K
Now, or = (P911 "-
[ for ./
riil

z
.,"[.J
rp^nl 10.2857
U = -\#-
'[#] 0.88

'"(#)
-
rn ..). =
"ovil'rs
Tox'-Tot
r.x - Ttt
-
473.6-290
506.4 - 290
= 0.g4g4 or g4.g47o. (Ans.)
il
tl
ii

)_ " "_*
=0.88..-.(2eo i
"hl!gq)=0.04438
(y)t"t' 1.04s4 or p",
= o.o{,r38 = =11sa5a*ht = 1.156 l. An air conpressor takes in atmospheric aif, compreses it and ilelivss the high pressure air to a storage
\not ) pot
vessel from whi it may be conveyed by a pipelioe to wherevr the supPly of compressed air is required.
The power required by the compressor, '.]i
Air and gas eompressors are classified into two main typ* :
\ P= rhicox(",v_4J .j'i (i) Reciprocating comPesso! (ii) Rotary compressors.
x 1.005 x (?,jv_ 290) .;
4350 = 20 3. Single stage compressor
435n 'a' Equation for work (neglecting clearance),
- = + 290 = 506.4 K
"* tO, rOOs
AIso, the polytropic elciency ofthe compessor is given
r'-rlf=+l
as
'
t = [; )\n- r) ..(zo los)
!
il
I
il
it
,

\
196

INTERNAL COMBUSTION
Equation for work (with
clearance volume),
ENCTNES AIR COMPRESSORS
.Ig7
2. For reciprocathg air compressor
the
'..v law ur
qw ofmmprcssion
@nprss:on desind
(a) very low speed de is isothemal and that may
be possible by
(") any sp""d a" verlhi8h sPeeds
speed does not affect the
compressio()
(d) none o{ the above.
3. Work jnput to the air compressor
of the with , as index ofcompression
tot mcreases with increase in yalue
of () decreses with
\*.=L+k-
rral (c) renains same
whatever he value ofn increase in value ofn
\

l+1"
\P1 I
=r+_fe.)t (d) first increases
and then decr eases with incease
ofvalue ofz.
\pt) w^iL ^_^:_
p,\ H::ifi:m;,ii:""1fff ;; ,i'i,,
=7+h-h
(
+h_h(v/v,)
;5*:i::j fi;jT-o,*ff
the compression pro""""
iJi""tilrrJffi ","u,*"" o"r kg orair derivered whe n
[;J=t
| I
. :.t::"'t= trffi#
u. vrultl{tage compression
=crearancerario. b,
"-t
_\o,.,l[*)- ,/ |*lf
-
(r)itlorri,r,,,
is v
I I
(c) tu4loglZ
"ilT;:T":#l1LT:"ffi i#At:T#":*""**"a,vadoptedexceptrorcompres- -- pt
t frmnloc" ?
n
5. . pj .

!Z=h The clearance volune


in re,
Forr-stage compr"."ort
o, pr= ffi (o) ro reduce ,n"
*;J;;;"T::;;Ti;:".':,1:::r*'s is proded to

"
Jo?L * . supplied, () to increase the
volumet
(c)toaccommoda,""","".;,;T;,""iu":;il:X;:":"""".:
(d) tocreateturbulenceintheaiobedelivered.
*=r\nv'lt,o..)*'l ,
6. With increase in clearance ,olume,
I the ideal work ofcompressing
(o) increases t kgofair
() decreases
ff#|ilffi;-?t#:f"t'tt"r it["ppti"" to *y {. of
"o,no."".or or motor, and eyen r.o vpe
(c) renainssame
z. In reciprocarirf u,,
""d
rhen decreases
ffi::'".Tiili:ay:Hff[T,::i;r"v],1,r"xJ,:]!,"Til1[ffi"d:,?,i]ili*ffiil;ri",.::: "o_'.u""o." th" "ruurun.u ."ti]"T;;T;"**
(") &^ql4!I9qg!Eg s##H**F
8.
cruu.u,""uor.""!-- swc

9. i:::* tmru;l "x[n:,:r;ll;.i:#


;ffj";.'"
""-nressed bv being rrapped in rhe
, , _Clearance volwle
o)

I0.
"tii#,tr"H*r:fi:Iressora
*-piur"oi'ii
are tto"u ___-v ." *ru,.
wusr ompression qccurs by
"o, tansfer ofkinetic
t"r ;*ffi^i;ffi , @;
vgrorne
Slio facrn.;"
Slip factor is ^n_ - , as
defined the rat
=--*Tan"e
l. Pressure co.erri;;;;::: *.ponent Ii'L::::':':"';;";;,T:*,'u.i",i."."u"u].Jffi,;iffi1'Jlli*uo","".uncevo,ume
12.
:l::T:' :q'of isentopic
the,ratio
and idear whr componenr. ro7 mcreae volumetric
elficiency (A) it..--_.^^..^__-
Deglee of reacri." work to Eulerwork. ()
,iri*Ti.-- (") do* rot r"i*"t.t.l*.*"""
decreases volumetric efficiency

i",,,= ""Til$fr,;i;d;.riseinthecomporstage
"r,u.lo
{d) fir.st increases volumetric
efficiency and then decreases
Mechanical efficiency ofreciprocating it.
air compressor is expressed
as
i RP
(oJ -;=- Tp
t.r.
'- _-:1-
BP
1,.p
Choose the
t
CorrectAnswer: @ j
.", s p ,r, {I. .
to air cornpressor is For the same overall pressure I Y.
miimum if the compression
ffi;::?ut Jaw followed is compared to singl"-"tog"
.o-p.Ji"tiioonfu
l"ukn8" of air past the piston
for multi-stage compressio'
() Isot}lermalpy (oJ more
zr
(c)Isentropic =C
pvr =C () less
(d) ppl =
(c) constat
(d) may be more or
". less.

.,...L r-
-
'798 INTENAL coMBusrroN ENclNEs 799
AR COMPRESSORS
11. Work done in a two stage reiprcating air compressor with imperfect cmling is given by
4. Pove that the work done/kg ofair in a compressor is given by
f"-rlfn-11
6 -r-prvrl[lzl ' -tl* -'=o"vrl[lzl " -tl - l- n-7 l
n-r W = R?, -:1lto)
'- -L " -11,
wherero= pressureratio.
L\e'l .l "-, l\nz) .J
5.
i
Prove that the volumetric efficiency ofa single stage compressor is given by
l, .-l I w ^-l(ps\%
I L-L

a 3\o,v,l[?l
n_L " -tl (c)
n-! - 1I'Iil l\pll =7+h-u(+\;,*r,"."
1

t= f
l
']
L,*, l n,"t .
- Ys
L
[pr./
6 State the couditions which lower the volumetric elficiency.
f n-1 I Explain with a neat sketch actualp-V agram for a single'stage compressor.
,o, $^u,l(3J"-'l & What doyou mean by multistage compression ? State its advantages.

12.
L"^' J
Work done in a two-stage raiprocating air compressor with perfect intermong is given by
o
10.
What is the effect ofatmospheric conditions on the output ofa compressor ?
Explain briefly with a neat sketch a reciprmting airmotor.
11. Write a shot note on'rotary type air motor.'
f "-r -l I I n-l What is a rotary cornpressor ? How are otary compressors classified ?
totn-" -r p,v,l(P2l- .lol=
ot)
-, I or t-t 4av,lfrl; ']
13. What is a centrifugal compressor ? How does it differfrom an axial flow compressor ?

l\Ptl | l\Ptl IL What is Euler's work ?


LJL 15. What is a slip factor and a pressure co-efEcient ?
|
l/ \- "-t 1
I
l- "-t -l I 16. Describe briefly an axial-flow compressor.

-I=prllll&l
r.l
n_j. ' -rl (d) -lr.Avlll
n-L
tzl
!/ \-/
" l4l"
\-

-zl t7. Draw the velocity diagrams of an uial-flow comprcsor.


What do you mean by'surging' aad'chokingi ?
\p2) 18.
L\e,,/
13. In reciprocating air
l [^,
compresso the method of controlng the quantity of air delivered is done by
] 19. Prove that the work done in two-stage compressor pr kg ofair delivered with perfect intercooling is given
by
(o) throttle control (b) blow-offcontml
(c) ciearance control (d) all of the above f -l I
t4. The effrcimcy of Vane trpe air ompressor as compared to Boots air compressor for the same pressure ratio
is
wlks= -l\r. / -rl
'v-onl(u\T
-_, |
(o) more (b) lm Usingthe above equation, prove that th! work ilone/kglofairin !'stages withperfectintercoolingis given
(c) same (d) may be more or less" by
15. In centrifugal air compressor the pressure developed depentls on
(c) impeller tip velocity (b) inlet-temperatrre | ^-r I
16.
(c) compressionindex (d) alloftheabove.
In a centrifugal air mmprwor the pressure ratio is inmased br
w*g=:+nrll-l-
/ -tl
(o) increasiughe speedofinpellerkeeping its diameter frxed
l\e" |
20. Pove that the heat rejected (perkg ofair) with perfeit intercooling
() increasing the ameter ofthe impellerkeeping its speed constant
(c) reducing inlet temlrcratwe, keping impeller dameter anil speed fixed i r.-.\l
(d) all ofthe above. =lcr+cul
L \"-rll
-ll(",-r,).
Answers 21. Explain with a neat sketch the actualp-V diagram for a two-stage compressor.
r. () Z (a) & (a) 4 (c) 6- (c) 6. (c) 7. (c)
22. Defme the following efciencies as applied to reciproeting air compressor :
& () e. (q) lC (&) ll- (c) 12. (b) 13. (d) 14 () (i) Compressor elficienry. (ii) Isothemal efficiency'
15. (d) 16. (d. (i ii) Adiabatic effi ciency. (iu) Mechanical efliciencY.
23. Write short notes on my three of the following:
(i) Clearance in compressors. (ii) Free air deliveed (F.AD.) and displacement.
TIIEORETTCAL qUESTIONS
(iii) Compressor performance. (iu) Control of comPressors.
1. Enmerate the applications of compressed a: (u) Anangement of reciprocatingcomPressors. (ui) Intercooler.
2. State how are the air compressors classified ? (uil) Compres;ed air moLors.
& Describewithaneatskettheconstructionmdworkingofasingle-stagesilgle-actingreciprocatingair
compressor.

I
I
\ t

fi
ll,
ii, 800
ij
ii INTERNAL COMBUSTION
rl ENCINES AIR COMPRESSORS
80r

l: l_YvsoLvED EXAMPLESI (u) Ifthe clearance ratios for L-P. and


H.P. cylinders are 0.04 and 0.06 rspectively, calculate the swept
u *ffiorom
clearance volunes for each cylinder. an(l
r '"rJ,::"T::tr*.".0r
pwwe requred ror r.0r' barad r5.c to 7 bar. The speed of the compressor is 400 r.p.m.
fAns. (t) 44 kW, (ii) B6.8Vo (iii) 8.266 m3lmin (iu) 15.41 kW
,i,, a tle ar delivery o10.3 m"lnir,
wh", til-coii#"io".or"""..
it ,""rr-r.---*qeeq Arwo-sraseaircompressorwithcomprereinrercoorirgi:i:"t:Tilil1ll#";iiil1ll;:'XXT::il]
(ii) Revesibleisothermal
l:
1;
(lii) Poltropic,wion=1.N5. suction conditions being I bar and i?"c. rr u"*t
h-alu t}," same stroke, find the ratio of thc
,
[Ans' I 31 kw ; 0.98 kw r.19 "yir""r"
cvlinders diameters, for the erficiency ofqompressio;;;;;;;;,-"-.
I
li ih .
Il:,:^"Tl**:: "f-the above example is ro ron ar 1000 r.p.m. If"''ecompressorissiagre-actingandhasa
; kwJ tobel.S '-"'*""')"""'"'^"^ili'".r.r.?
Assumqthe index ofcomDression
10, a.Ii"a;r
i':ff:::t1:.til*"j:""eboresizerequirer"" running at 1000 r'p m [Ans' 68 3 mm] Ipressure of 15g-bleacting
bar'
compressor is required to compress 2s m3rminof
ai at 1 bar and 25"o to a
Determine the size ofthe moto req"it"'Juni
r" t'r'* i r'd;J#*l:::ffi:
lnducton md rru.
're additional data is given: t}," dime'sions irtie iottowirg
ms/min.'rhe crearuo* ". "ontillp-t-Tsu "ylinder
^"*Ie sweptvolume and
"oru^J"'llf or
11,::1;::i",HT.";
the stroke/".".;;;;;, Clearance volum e= 5%,LlD 1.2, r.p.m. gQe,
i.'iill,lr" rn"
^u,"_"r,at1"r. "JJ;;;;il;;,:t
sionas,Bcarcur";;i";;;fiT;:ilHli*T,,il,::1,il:**: jlf ffi
;;"";#Ho,Lr*_
=
Assume no pressure change in suction
= \,techanical effrciency g0%, Value of index,n 1.35.
=
valves and the air ge," heated by r0.c during suction
=
stroke.
4. Asingle-actingcompres tAns'?-3.2mm;8?.84mn;67.6vo;2kW;6T.6Eol ll' [Ans. D = 550 mm, Z = 680 mnr j
A three-stage compresso-r^is used to compress
hydrogen from 1.04 bar to l5 bar. The compression in all
r'o--u' i'""tio"';;;""* stages follows the lawpyrd c. The
tempe."tu.u ,

ii'
f,nl,"|']f iiii#T'f"f',,1i#'f*,'ff:#1111'"':'
stages wih an q qt^:.:-l^culatetheindicatip"*".",*t"otu"C
''
Thetemperatutu't*r""toirr"
bar, ar the
clearance and
= orfrylrogen at inlet ofcompressor is 2g8 K. Neglecring
assuming perfect intercooling, fud :
(l) Indicated power required to
deliver 14 m3 ofhydrogen per minute masued at inlet
ii
expansion for borh
;;;:l,T*T,ff til"il#f#l:'""iff ;:ilfJi:::: l1:Hf.'""J,"T *; (il) Intermediate pressures. conditions.

ii.l::l:*'x1T,"""#;il::3:l*:iyiiffi ;il"":ffi
"' il::::i:I;H""",rxT::;':"'
cylinder'
Taken = 4125J/ksK
fAns.(;)96.2kw(il)B.3s4ba,
i" zso t.p.* "'h calculate the swept volumes
of the t2' A threr:-stage reciprocaring.air 10.815 barl
If the mechanicrl effi"i".'" "o-T1."..* compressor compresses *".-i'i, l,he law of
cy ofthe compressoris compression ispyre = C and is same for all the ges of "o
equired. 8570, calcu compression. ""0';;:;;;;;rr.
u Assumingperfect iDtercoolingand neglecting
and valve resistance, find the minimum pou,er
fr.ly:r.{,ra".,.i,gr"*.tiog..;"J*j:';i'i'T;i"':ffi$:T::t.ffiffi:i';};ff$i
300 r p m is driven bv
requied to compress 15 m3/min of r."" "lu^n"" ." "ili"-i"t".nrJ*i. "'"
p."."r;;;:"- -'
i fi ff :}XHf t"',Tjt :li:, g""" ui,t*"* *i".'"'"
23 kw rectri ror.a e c mo "i..

::Ti:#iiil"*,ildT;"db"l'.'"?d#X;1k#T:,i!_?:,i;i!:,,',-..Tif :il $ff j 13' A three-stage sin'le-acin. air cmpressor


delivery of2'83 n3/min The suctim p."".o."
ruming i, un ut*orph",l}ili:;;:I'J#:* lXj:l
anedxpansion is 4 r'3, -p".atrr"
rhat rhe crea'*""
'"r'-" i. i,;'ff'Jn,,i:ilil:,.,l*:1il1;: ;:::T{t{ti:
= late the indicated nower reqred-, assuming"rd o.sa bar and B2.c respectivery. calcu-
"r"
.o_oi"iu'iir"."ooling, n = 1.3, ancl that thc nachine is
6" Atwo-stageaircompessorcosistsofrh...^,:-- designed for minfiirum work.-The detivery
[Ans'44?mt/min;zTEo:296mmi frl"."lJJi" i" ,o "r.
p.us"'.eis?baranJ;;;:'":":t:ts of three cvlinders n*jl^q,:n: 14' using the data ofexample 1,3 determine [Ans. 2a.2 kw I
hat loss to ttr" cytltae.acket cooling water and heat loss to the
u"
i,."r.," "1T"^!9I" "t-oi.'rr," a"ri"".v
r ors"i,li;ifilaint"rmore-.
rntecooler circulating water, per minute.
"i'i"':'[il:;:':fii::H;"'"3,H{Tfrl]..llly"i:,t
all the hree cylinders' Neglecting 15' tAns. 90 kJ/min, 875 kJ/min.l
Afour-stagecompressorworksbetweenmitsoflbarandll2ba.ltreindexofcompressionineachstage
clearance is 1 28' the temperature at the-start
() The intermediate
pressure :(dl)- :,*t are so chosen that the work is divid.ua"i""-fr"."i""
i"lulh .tug" i" sz.c and the intermediate pressu.es
(iil) The isothenai;;#" The power.required ro drive rhe compressor uq""lly".o"gifrl Neglecting clearance find
(i) The volume of free ai delivered
7' Atwo-stagrdouble-actingaircomDressr^na,-lj--^.^^ lAn6'(i)2l9bar(ii)76.3kw(iiilg4.'lal per kWh at 1.0 bar"rrru.. and 15.C.
sr'" ortn"i.p. 200 r (il) Th" t"-Ouruture at delivery
from each stage.
"yi;;;;::qi)l:1Tt'essor'.operatingat (ili) Ths im,..-al eflicier
;11:l::,";.1'*ffi1i:h,T,"H;;;Ui;:'1'Li'X;T;lX1i:Jl"i.liXlh1l'1"',il:l'"ii;?ft:
tbe air atapressure 16' Amurtisrag""t.""-r."""i?*tubecresi*nedroe,.,"r",nif""..l:,?:Jrtn;g'rYffill"#
ifffi:J"T;":jT:T"LH11terc39l-erso that,t, *i"..'in. u'ltscharges or4.052 bar.
pressure raio will not exceed
R= o 28zkrkgi'---'^'P'"ssorat ls ,.. n'"'"i"";i:'T ;i.il,iill"il,"r1i'J;tii*it?i,{j,".;
4. Determine :
(i) Numbeofstages.
Cairulate.: (l) The heat rejected (l/)
Exaccsragepressueratio.
in the intercooler (iii) Intermediate pessures.
,l rneoameterofH.p.cylirder [Ans. (r) 4, (ii) 3.3r (jii) 36.25 bar, 10.9s bar, 3.8 1 barl
(iii)ThepowerrequiredtorlrivetheH.p.cylincler. l7' In an ideal four-stage reciprocating
air compressor, the inlct pressure is 96 kN/m?
300 K t hc ai r is delivered at a pressure and inlet temperature rs
[Ans' (t) ] 805.68 kglrhin. (tt) of 27.6 MN/m'. Th"
8' A single-acting two-srage
cor t tn.t .- iitr, ,r., u*l requircnlcnt and has perfectintercooti"g. "o.p."..o.
is designcd for r lre min imunr pou,,.r
wirh conplete inrercoolins T}ro.u;";"iuioio'-p."""io.
---"-' "' ond expansion processes both con-
sucrion occurs ar r b-;JilL'"j:"r delivlrs,, i;i-l,i";;;; lorn to the relationp! 2 C. Determinl
polytropic inrlexn expansion and compressron ro u.. rl'n = ,
J;;. il;JJ: p.ocesses are reversibre
polytropic u,ith (l) Interrrediate ;)ressures.
(l) The power required. (iri) The idcl isothermal efficiency. 0i) The air delive.y temperature.
(,,') rhe iree air dJi l ;::Jn:*T;;ii11,""",". iorair'whic! maybeassunredtobeaperfectgas,thespecificgasconstartis0.2g?02kJA<gK.
[Ans. (i) 39b kN/n,, 1628 kN/m? ancl Z is N/m, (rr) 880 K (ii) AE.t)1,1
AIR COMPRESSORS 801
ti02 INTERNAL COMBUSTION ENGINES
(u) Impeller blade angle atinlet.
Ilotary compressors (ui) Diffuse biade angle atinlet. lAns' (i) 466.85 K ;(i) 2077 7 kW t(ii) 46.745 cm, 94'9 cnr I
18. Airat1.013barand l5'Cistobecompressedattherateofs.6m/minto11.?Sbar.TVomachinesare (iu\ 12.2 cm,(v) 15.7"' (u) 8.9"i
considcred : (i) the rc,ots blower ; and (ii) a sliding vane rotary compressor. Compare the powers required, 26. A single inlet type centrifugal compressor handles 8 kg/s ofair. The ambient air conditions are 1 bar and
assuming for the vanre type that internal compression takes place throrrgh ?5% ofthe pressure rise before 2O"C."The compiessor uns at 22000 r.p.m. with isentropic efciency of 827o. Thtr air is compressed in the
delivery takes place, and that the compressor is an ideal uncooled mach.ine. [Ans. 6.88 kW, 5.?5 kWl compressor from 1 bar static pressure to 4.2 bar total pressure. The air enters the impeller eye *'ith a
19. Air is compressed in r two-stage vue tJpe compresso from 1.013 bs to 8.?5 bar. Assuming equal pressure velo-city of 150 m/s with no prwhirl. Assuming that the ratio of whirl speed to tip, speed is 0.9, calculate :
ratio in cach stage, c.alculate the power required. Assume that in each compression is complet and that () RiseintotaltemperatureduringconpressionifthechangeinK.E.isnegligitrle'
intercooling between ,stages is 7570 conplete. Calculate also the capacity ofthe high pressure stage in cubic (ii) The tip diameter ofthe impeller.
metres per minute fo.r a free air delivery of 42 mr/min measured at 1.013 ba and 15"C. The machie is
(iii) Power required.
uncooled xcept for th e ntercoolcr and operates in m ideal manner [Ans. 18? kW : 15.6 m3/minl
(iu) Eye diamete if the hub diamete is 10 cm'
2o. Arootsblowercompresses0.06m3ofairfroml.0batol.4Sbarperrevolution.Calculatethecompresso
elliciency lAns' (i) 167.6?'C; (ii) 37.56 cm; (iit) :1348 kW ; (iu) 25.9 cntl
fAns.87.117ol
27. In an axial flow conpressor, the overall stagnation pressure ratio achieved is 4 w ith overall sta5mation
Free air of 30 m3/min is compressed from 1.013 b^r to 2-23 bar. Calculate the power required (i) if the
compressicn is caried out in rmts blower, (i) if the compression is canied out in vane blower. Assume that isentropic elliciency 86 percent. The inlet sta5nation presure and temperatue ar() 1 bar and 320 K. Thc
t)rere is 257o eduction in volume befoe the back-flow occurs, and (iii) the isentropic effrciency in each mean biade speed ii tgO ms. te degree ofreaction is 0.5 at the mean radius with re,lative air angles of t 0'
case. and 30" at rotor inlei and ouUet respectively. The work done factor is 0.9. Calculate :
(i) Stagnation polytropic effi ciency.
lAns. (i) 60.85 kW (tr) 48.46 kW (ttt) 73.69Vo,92.53Vo1
A centrifugal compressor is designed to have a pressue ratio of3.5 : 1. The inlet eye ofthe compressor (id) Number ofstages.
impeller is 30 cm in diameter. The aal velocityat inlet is 130 m/s and the mass flow is 10 kg/s- Thevelcity (iii) Inlet temperature and pressure.
in the delivery duct is 115 m/s. The tip speed of the impeller is 450 m,/s and runs at 16000 r.p.m. with total (iu) Blade height in the first stage ifthe hub-tip ratio is0'4, mass flow rate is 20 kg/s
head isentropic elliciency of787o and pressure co-eflicient of0.72. The ambient conditions ae 1.013 bar ard tAns. (i) 8S.a% ; (ti) lL ;(ii\ 287 '39 K, 0.68 64 bar, (iu) 11-4 cnrl
r5'C.
a. A multistage axial flow compressor delivers lSkg/s ofair. The inlet stagaation conditi()n is 1 bar and 20'C
Calculaie: The power consumed by the compressor is 4260 kW. Caldate :
(l) The static pressue ratio (i) Delivery pressure.
(ii) The static pressure ancl temperature at inlet and outlet ofcompressor (ii) Number ofstages.
(iii) Work ofcompressor perkgofair, and (iii) Overall isentropic efhciency ofthe conpressor'
(iu) Thc theoretical power required. lAns. (t) 4.2r (tt) 0.9L7 bar,279.6K,3.86 bar, 461.07 K Assume temperatue rise in the first stage is 18'C. The polyiropic efh.ciency ofcontpression is 0.9 and
(rli) 180.29 kJ/kg of air (iu) 1802.9 kWl the stage stagnation pressure ratio is constant. lAns. (i) 6.41 bar ; (i) r0 ; (iii) 87 -2aq,'l
,8. Air at a tempaatu'e of 290 K flows in a centrifugal mmpressor running at 20000 r.p.m. The other clata
given is as follows : it
trl
Slip factor = 0,80 ; Isentropic total head efliciency = 0.75 ; Ouie diameter ofblade tip = 500 mm. "g
Determine :
(i ) Ttre ternperature rise of air passing rough the compressor.
(id) The static pressure ratio.
Assurne that the velocities ofair at inlet and exit ofthe compressor ae same.
tAns. (t) 218.62'C (tt) a.81
u. An axial flow air conpressor of507o reaction design has blades with inlet and outlet mgles of45" and 10'
respectively. Ttre compressor is to produce a pressure ratio of6 : 1 with an overall isentropic efficiency of
;:l
0.85 when the airinlet temperature is 40'C. The bladespeed md axialvelocity are constantthroughout the
compressor. Assuminga v:ue of200 m/s for the blade speed, find the number ofstages required when the
*.i
work factor is (i) unity (ii) 0.89 for all stages. [Ans. (t) 9 (tt) 10]
25. Acentrifugalcompressorninningat9000r-p.m.delivers600m3/moflreeair.Theairiscompressedfrom s
1 bar and 20'C"to a pressure ratio of4 with an isentropic efficiency of0.82. Blades are radial at outlet of sm
'
impeller and the flow velociy of 62 m/s nay be assumed throughout comtmt. The outer radius of the
impeller is twice the imer and the slip factor may be msmed as 0.9. The blade area coelcient may be {
assumed as 0.9 at the inlet, Calculate : i+
(i) Final temperatue ofair. ,;
(1,) Theoretical power.
'n'
(iii) Impeller diameters at inlet and outlet.
(lu) Breadth ofthe impeller at inlet.

-----l


t

\ I
rf I

r1
,] CAS TURBINES AND TET PROPULSION
-qos
r

,l
2. Constant volume conbustion gas turbine
,In almost aII the fietds onerySycle gas_.turbine plonts clre used. Closed cycle plants
il

2L introduced at one stage beca.use of tiei, bittty to birn chmp


remained slow because ofavailabilitJ ofcheap oii anil natural gas.fuer.In
again, the attention is being paid toclosed cycle plants.
uere
betweer their progr.ess
Because orririrrg oll p"i'"u.lro*

t;
il
Gas Turbines and. Jet propulsion 21.3. MERITS OF GAS TI,]RBINES
j
21 1'G^mtubinsleneralaspects-21.2.Classifietionofgasturbiaes.21.8.Meritsofgasturbine. (j) Merits over I.C. engines
21.4. Constant pessrue combustion gus tu.nir."_O'p",
; \
turbines_Methods for 1. The.mechanicalefcienryofagasturbine(952r)isquitehighascomparedwithl.C
"y"lu Cu" of operating
i
irnprovement of themal efficiency ofopei cycle-ga,
l turbi"i-f""'tirr""t variables on engrne (85/o) since the LC. engine has a large
I
thermal efficiency-crosed cycle gro tu.ti"-"-"t.
,"Jro-Jt" or"ro."a cycle gas tubine over .ro-"" ofsliding pu.t".
open cycle gas t'rbine. 21.5. constantvolr*u co-bo.iiorlrrre". 2. A gas turbine does not require a flyv*beel as the torquc on the shaft is continuous
zr.o. uses ofgas turbins. and
21.7. Gas turbines tels 21.8. Jet prooulsion_Turbo_J"t_u..rlptioo*Basic
cycle for tubo_Jet
unifom. Whereas a flywheel is a must in case of an I.C. engine.
: engine-Thrust, thnst-power, propulsive efciency
i l a" t"-*"f um"i"ncy_Tbrbo_prop_Rau_
jet -Pulse-jetmgine-Rocketengines-Reqrirem"n*ofariJ"ui.o"t"tp.opulrent-Appriations o. The weight ofgas turbine per H.p. developed
is less tha that ofan I.C. engine.
I

ockets-Thrust work. ronulsiveworkand proprl"ir" 4. The gas tubine can be driven at a very high
i of speeds (40000 r.p.m.) whereas this is
I
" "^"'
Questions-Theoretical Quesiiom-Ur"otr"a b*".pt".. "ffi"i"lr"y_ffigfrlight_Objective Type not possible with LC. engines.
iii 5. The. work developed b a-gas turbine per kg ofair is more as compared to an
I I.C.
2T,1. GAS TURBINES_GENERAL ASPECTS engine. This is due to the fact that gu"es .an"
11 upto atmospheric pressure
in case ofa gas turbine whereas in an I.C. engine "*panded
expansion upto atmospheric pres_
Probably a wind-mill.was.the first turbine to produce sure is not possible.
I
pre-compression and no combustion. The useful work, wherein the there is no
I

name todav include a compressionprocess and


characterist'ic features ofa gas turbine as we
think ofthe 6 The components of the gas turbine can be made righter since the pressures used in it
a i"l ad.dition(".;";b;Ji;;"]ro"""". The gas are very low, say 5 bar compared with I.C. engine, say 60 bar.,
turbrne represents oerhaos
in a self-contained nd compact lh",rno:l
sarisfacroD, wav pi"""i"! *;;;;;;rr*s of power 7' In the gas turbine the ignition and lubication systems are much simpler as corn-
unit..The g"" tu.i;; "i ;"y have a future uie in conjunction with pared with I.C. engines.
the oil engine For smalleigas.turbine uiit",
tr*'in"mciencies in compression and expansion 8.
p_rocesses b-ecome greater and-to improve cheaper fuels such as paraffine type, residue oils or powdered coal can be used
tn tnu.-ai efficiency it is necessary to use a whereas
exchanser. In order that a smalr gas rurbine may heat special grade fuels are employed in petrol engine io check knocking
.".,ne."'u ;;-;;;;;lililimal oil engine or pinking.
or petrol engine it is necessary^ that a compact 9' The exhaust from gas turbine is less polluting comparativery since excess
efiective heat exchanger be used in the gas turbine air is used
cvclc' The themal efficiericv.of the gas tur'bine for combustion.
.till quite moest20foi
that ofa modern steam t".;"e plait s8 i" iiir.'ifi"'p"ssible
"io""'i. to construct combined"ompar"a
-ltt l0 Because of low specific weight the gas turbines are particurarly
effi ciencies are of order of 4s7o r plants whose suitable for use in
-o.". Higrr"i'urnuJ*t". might be attained in future. aircrafts_
The following are the major fields ofapplication
ofgas turbines : Dernerits of gas turbines
1. Aviation
3 o' and gasindustry
'r'he efficiency ofa gas turbine is not the
i. il"JiijiljJlil?1"
1' Thethermalefficiency.ofasimpleturbinecycreislow(r5to20v,)ascomparedwith
I.C. engines (25 to SOVo).
criteia fo-r the choice ofthis plant. A gas turbine 2. With wide operating speeds the fuel control is comparatively diffrcult.
is used in aviation and marine fields because
it; ;" 3' Duetohigheroperatingspeedsoftheturbine,itisimperativetohaveaspeedreduc-
cooting
laater ;; s;;;,;; fit.into the i,,,"',n"p" lr;!':!"2?;!:1iil,:.i:":::i::T^::, tion device.
generation becausc of its simplcity, lach
of cooling water, needs quich installation and quich 4. It is difficult to start
starting' It is used in oil and gas_industry bec.usu a gas turbine as compared to an I.C. engine.
or""l.prr ruppry offuer and, rou inst(rllation cost.
The gas turbines have the following rimitations: (i) 5. The gas turbine blades need a special cooling system.
?hey are not serf starting ; (ii) Iow 6.
"!t:;;::';;:::f:rt toad,s ; (iii) non-reuersotiy ; r;"iieher rotor speerts od iu) o,",ou efficienqr One of the main demeits of a gas turbine is its uery poor thermal efficiency
loads, as the quantity of air emains same irrespective of road, and
at part
ouiput is reducecl
by reducing the quantity offuel supplied.
2I.2. CJ,ASSIFICATION OF GAS TT,JRBINES 7' Owing to the use of nickel-chromium alloy, the manufacture of the blades
and costly.
is difiicult
The gas turhines are mainly divided into two groups
1. Constant pressure,combustion gas tuibin"
:
8. For the same output the gas turbine produces five times exhaust gases than LC.
engine.
(o) Open cycle constant pressure gas
turbine 9 Becauseofprevalenceofhightemperature(1000Kforbradesand2s00Kforcombus-
() Cl,rsed cycle constant pressure gas
turbine. tion chamber) and centrifugal fcrrce tle life ofthe combustion chambe and bladcs is
short/s:nall.
804
I--- ---
807
806 INTERNAL coMgusrroN ENcrNEs CAS TURRINES AND JET PROPULSION

(i;) Merits over steam turbines : . 1-2 represents: deal isentrop'c compression'
The gas turbine entails the following ad,uantages ouer stearn turbnes : a 3-4 represents: ideal isentropc erpottsion'
1. Capital and running cost less.
2. For the same output the space required is far less.
3. Starting is more easy and quick.
4. Weight per H.P. is far less.
5. Can be installed anywhere.
6. Control ofgas turbine is much easier.
7. Boiler along with accessoies not required.

21.4. CONSTANT PRESSURE COMBUSTION GAS TURBINES


21.4.1. Open Cycle Gas T\rbinee
Refer Fig. 21.1. The fundamental gas urbine unit is one operating on the open cycle in
which a rotary compressor and a turbine are mounted on a common shafl. Air is drawn into the
compressor and after cornpression passes to a combustion chamber. Energy is supplied in the
combustion chamber by spraying frel into the air stream, and the resulting hot gases expand
through the turbine to the atmosphee. In order to achieve net work output from the unit, the
turbine must develop more gross work output than is required to drive the compressor and to
overcome mechanical losses in the drive. The products ofcombustion coming out from the turbine
are exhausted to the atmosphere as they cannot be used any more. The working fluids (air and
fuel) must be replaced continuously as they are exhausted into the atmosphere.

Fuel (Heat) Fis.21.2


points in the cycle to be negligibly
Assuming change in kinetic energr between the various part of cvcle'
--,t v ,-"tl Jnthalpv clanges;d then applving the flow equation to each
_---<1 Combustion
F-
3
";;;;;;;itr,
for unit mass. we have
Work input (compressor) = cP(T; - T)
chamber
Heat supplied (combustion chamber) = cp(Ts- Tz)
Compressor Turbine Work Work output (turbine) = co (T"- Ta)
()
Shaft fi) .'. Net work outPut = Work output - Work inPut
- ce(Ts- Ti) - co(Ti - Tr)

and 'l*.-t=Hm
c,(T3-Ta')-c,(T2'-\)
--- -rr)
"rTt
Air in 4'
Exhausl
s# Compressor isentropic efticiency, *,
Work inpu rquieil in isentropic- compression
-
Fig. 21.1. Open cycle gm turbine.
# Achal work required
c^(Tq-T) T" -T, ...(21.1)
",11
..iti - cr(T2'-T) Tz',-Tt
If pressure loss in the combustion chamber is neglected, this cycle may be drawn on a ?-s .{ r-}
diagram as shown in Fig. 2L.2. Turbine isentropic elficiency, n@ie
a 1-2'represents : irreuersible adiabatic cornpression. '," Actual wort outPut
o 2'-3 repesentl . constant pressure heat supply in the combustion chamber. Isentmpic work outPut
o 3-4'represents : irreuersible adiabatic expansion.

I
I
I
J

I
808
INTERNAL COMBUSTION
ENGINES
_Tt') CAS TURBINES AND JET PROPULSION
cb(Ts T^ _T q0e

Nore,withrheva*.,"",.,;;T#:"#
open cvcle, the specifi"
..*1.2) . The ideal cycle for this arrangement is 1-2-3-4-5-6 the
intercooling is shown as 1-z' in the aclual case, and ; eompression process without
r rtr tn" ideal isentropic cse.
compressor becau." fr"I
gseT;Hffi;,].iil:1t-tf:!:t*:l"ltora
heat of the ESes in the combustion
char
real sas varies, and atsoin
re
hr" b.,^"
c"wirhtemperatu*,""ivru"il-1-9^d'-*a".r'"-iJ"r.i#'#"?X,,tT,i:J:1"i:"H:T,l;Tf"H,tljl;
;;:"":,:"*..;,th:i*":i:t"""::lFF"ftX*Ur:*:l;ill*k;i",*:i*,"";T*hT1*":"I
ln an open cvcle
is cleater fha' tha;;;;T-t::n-process fe
ruer rarios used ue r": ft;i:11"-ti
nass ortuet r''rn;;iil'::,f"ffii"1llff l?:*Tr"t"f
of ai.-oan at higf
';;d;"lT,lYcases'
airis bled from thr ffiril,,*;
"pp,o*i_ut"iy tf,;#;H;lij:itTfj,?jj*;i#:XffTlT"Tff,fl:;1,+fl,*1T:,T,1,lti:i3
21.4.2. Methods for fmprovement of Thema
turbine piant I Efificiency of Open Cycle Gas
methods are emploved to
,h":A:rrfill"*ing increase the specific output
"f and thermal efficiency
1. Intercooling
2. Reheating
I' Intercooling. A compress ' a ----'-
e'ur ru
o xegeneration.
3. Regeneration.
sas turbne .t*_"_lrlr-*r the.major
,:d uy tr,"
l""11t.on9a
aiiin
the air .y
t" s,"
sr" iiffi:dffi:l;Htilffyff-.:*_"_l'l'-*'
lirij".'3
uc wur requed bv the compressor percenrage of
percentage or power
1ri" 1"i"r ;;;';;,",;;:,:::;tT:.:j,:li::.T::."1.:1"u"i.[;;;.ompressing
th_e in t,i".-t-"?", #;;; can be ;a.i"" tv.""-;;#:;
I

;1ilT1:"0r"
follows :
i;;;","'f:|iT
correspondins ?-s diagram
o.,r," ."i,
.^y"1: #:::?:;,r:,yif*;;;;;;ffi;"r""'Fig.
i"".i,til?:'ffi:;,H :ffi ;|ffi* k"rl:;il:
2r.8. rhe

t-2' L.P. (Low pressure) compression


2'-3 Intercooliug
Fig. 21.4. ?-s diagram for the unit.
3-4', H.p. (High pressure) compressron Now,
+-a C.C. (Combustion chamber)-heating Work input (with intercooling)
a-b T (Turbinelexpansion co(Tr' - Tr) + c _ lt1 e\
o(To, T")
lntercooler
Work input (w itho ut i n te rcool ing)
=(
B.{. compari ng 0,
" is",i""
;"1', ;,li ;r$i;; ;l;' i:^' ;l
"! ou.u*" a th th:,.(2:,:)
tnput tDrth LntercoolrnE
. Iess. the worh input with no intercooling, when c, (?r, _ ?r) is ^t
ftris is so if^than
it is assumed thai isentropic efficiencies of the two compressors,
1"::-'lr-"l^1?- -zr)' are each.equal to the isentropic efliciency of the single
operatrng separately,
compressor which
would be required if no inter_coolingwere used. Then'({,
diverge on the ?-s diagram from lft to the right. - \) <iTL, -rr,) siice lhn p."s"ur" li.u.
Again, work ratio Net work output
- Gross
work output
_ Work of expansion - Work of cornpression

FIo-
. is this we may concrude that whenY:':,Ir:;::::::,,
ratio .
increased. *,* ,"duced. then the worh

However the heat supplied in the combustion chamber


whcn intercooling is used in the
cycle, is given by,
Heat supplied with intercooling = cp(Ts_ ?4,)
Also the heat supplietl when intercooling is not used, with the
same maximum cycle tem-
perature 7u, is given by
Fig. 21.3. T\rrbine plant with
intercooler Heat supplied without intercooling co(Tu_ T)
=
810 INTERNAT_ coMBUsrIoN ENctNEs
CAS TURBINES AND JET PITOPULSION
Thus' the heat suonlied when intercooling is used. greater
Although the net work oiiput is increased, ty ntiiioiine
s than wittt. no intercooling.
Neglecting mechanical losses the worh output of the H.P. turbine must be exactly equal to
n heat to be supplied causes the thermaiefficln"y"ro it is found. in general that the increase the worh input requred for the compressor .e., c * (Tz' - T L, = c o, (T s - T a)
d.""r"or". When intercooling is used a
supply of cooling water must be readily availble. Tie
additional bulk of the ,roii-.y offset the The wok output (net output) of L.P. turbine is given by,
advantage to be gained by increasing the work
ratio. (Tu - T{)
2' Reheating' The output of a gas turbine can Net work output (with reheating) 1 co,
in two stages with a reheater tetween"the t.'o ;. J;; be ampry improverr by expanding the gases and Net work output (without reheating) = cor(Tn'- Ti)
in Fig. 21.b. The H.p. turbine drives the
compressor and the L.p. tubine provides the Since the pressue lines diverge to the right on T's diagram it can be seen that the tempera-
useful oower output. The corresponding ?-s diagram
is shown in Fig' 21 6' The line 4'i' represents ture difference QE - T;) is always greae r than (Tr' - Tr'), so that reheating ncreases the net worh
irt" in the r,.p. turbine if reheating is zof
employed. "*'p""rlo" output.
Although net work is increased by reheating the heat to be supplied is also increased, and
the net effect can be to red,uce the thermal efficzncy
Heat supplied = crr(Tr- Tr') + corgs- fP.
Note. cro and c* stand for specifrc heats of air md gas respectively at constmt pressure.
3. Regeneration. The exhaust gases lorn a gas turbine carry a large quantity ofheat with
them since their temperature is far above the ambient temperature. They can be used to heat the
air coming from the compressor thereby reducing the mass of fuel supplied in the combustion
chamber. Fig.21.7 shows a gas turbine plant with a regenerator. The corresponding ?-s diagram
is shown in Fig. 21.8. 2',-3 represents the heat flow into the compressed ir during its passage
through the heat exchanger and 3-4 represents the heat taken in from the combustion of fuel.
Point 6 represents the temperature of exhaust gases at discharge fiom the heat exchanger. The
maximum temperature to which the air could be heated in the heat exchanger is ideally that of
exhaust gases, but less than this is obtained in practice because a temperature gradient must exist
for an unassisted transfer ofenergy. T\e effectiuetuss ofthe heat exchanger is given by:

Fig. 21.5. Gas turbine with eheater. I

Fig. 21.7. Gas tubinewith regenerator

Fig. 21.6. ?-s diagram for the unit.


812
INTERNAL COMBUSTION
ENOINES
j
Effectiveness,
CAS TURBTNES AND JET PROPULSION
l 8I3
,il
not the same as the ratio of c.hange heat rejected.
JI

pressure ratio producing maximum As a consequence, there exists an optimum


thermar- efficbncy i*
ai""1iilr"'irirt" tr^p"r"trr".
ufii.i"o.y"
As the pressure ratio increases, the thermal
...(21.5) mum and then it drops off with a furiher irrcreases until it becomes maxi-
i"""";;;l;';;ssure"lroratio (Fig. 2r.10). Further, as the
turbine inlet temperaiure i""-lu"."r, lir" p".
flatten out giving a greater range of
I

'i
ratiosof pressure optimum efliciencv. "ri,?ir*es
ll
T (Temp.)
I

(Entropy)

Fig. 21.8. ?-s diagram


for the unit
A heat etchanger
''-' '-
is usually used
ind,ustrial power. in large gas turbine units for marne propursion
or
21.4.8. Ef,fecof Operating Variables on ls = 1095'C
Thermal Efficiency
thermal efficiency of actual +24
uu.r"o,lli open cycle depends on the following thermodynamic
I
I

(j) pressure ratio '20


I

(ii) Turbine inlet te G


(?t)
(iii) compressor iol"rnpu""tt"" tu
c
I
(iu) Efnciency or rr;
(u) Efficiency of
;i:r,::ll';1" .9
E.^
the compressor (n ) @ l

i
Effect of turbine inl.t t.mp..i,f,l". .od p.""",r.e 6
ratio :
ov
other variabres beins constant) c
I
f# fi{*,{i'ru::';":;::';,2:;*',"ffiJtrT:
rlcai lrmitation to increasinr H#,1"
*.r.1i3j
oran F
i
avairabre for rhe rurbine
o;i.Ll"-t:"bii:.t"'"t ff#::;lr:ti:':{,,.tt;";:;,ii:n
Rufi, ris zr s r""'1"';:":",::;f:::,":! :" 1:,rota e t'inJlf,i' J,"".
iu
, ^ . supptied
neat . ""
,;:irr;,":"i!: "ii"
as uell as lte heat rejected,
ii"r"'"""., 3 4 5 .6 7 I
::;:::;"r:;r;":;"::;r;;i;n:
Pressure ralo ---'
I0. Effect ofpressre atio and turbine inlet tcnrneraturc
f'-.'...-..--

815
814 INTRNAL COMBUSTION ENCINES GAS TURBINES AND JET PROPULSION

Following particulars are worthnoting : +28


I
I
Gas temperatures Efficency (gas turbine) I

'24
550 to 600'C 20 to 22Vo
f
900 to 1000"C 32 to 35Vo

Above 1300'C more than 507o


:0
Effect of turbine and compressor efficiencies : E IA

Refer Fig. 21.11. The thermal.efciency ofthe actual gas turbine cycle is very sensitive to i6
variations in the efficiencies ofthe compressor ancl turbine. Thee is a particular pressure ratio at 812
which maximum efficiencies occur. For tower efficiencies, the peak of the themal effrciency occurs c
n -n -na
at lowe pressure ratios and vice versa.
t3 = 8.15.5"c

48
'1 2 3 4 5 6 7 I 9 10

Pressure ratio ---)

40 Fig.21.12

,t -- 21.4.4. Closed Cycle Gas Turbine (Constant pt"..u'" orjoule cycle)'


I
132 Fig. 21.13 shorils a gas turbine operating on a constant pressure cycle in which the closed
I
system consists ofair behaving as an ideal gas. The various operations are as follows : Refer
Figs
8za 2L.14 and 27.75.
Operation l'2 : The air is compressed isentropically from the lower pressure P1 to thc
.6 z+ upper pressure p2, the temperature rising from ?, to ?r' No heat flow
.o
occurs,
o20
Operation 2'3 : Heat flow into the system increasing the volume frorn V, to Vt and
temperature from Trto T, whilst the pressure remains constant at p2'
816
c Heat received = mcp(Ts- Tz\.
F
l2 Operation 3'4 : The air is expanded isentropically fromp, top' the temperature falling
from T, to ?0. No heat flow occus.
Operation 4-1 : Heat is rejected fom the system as the volume decreases from Vn to Vt
and the temperature from Tn to ?, whilst the pressure remalns con-
stant at pr. Heat rejected = mcp (74 - Tr)
Work done
1 2 \or'snndard =
3 4 5 6 7 I 9 10 Heat received
Pressure ralio -------| Heat received/cycle - Heat rejected/cycle
Fig. 21.11. Effect ofcomponent efficiency. Heat receivecYcle
mc.(T3-72)-mc,(Ta-T) . Tq-7\
Effect of co{rpressor inlet temperature :
*"Jrt-T)='-
Refer Fig. 2I.t2 (on next page). With tILe d.ecrease in the compressor inlet tenxperature \-k
there is
increase in thermal efficiency ofthe plant, Also the peaks ofthermal e{ciency occur at Now, from isentroPic exPansion
high pressure iatios and tle c!rves become flatter giving thermal efliciency over a wider pressure y-1
ratio range. T, _( n2\ r
4 -tp'j

I
I
5
L
tf '
ljii
lill
816
INTERNAL COMBUSTION ENGINES
rc
AND JE-T PROPULSION
lri,i
iii
,iir
itl
rlil
ifi
ii

lrir
llll
Cooling
medium

lil Fig. 21.13. Closed cyclegas


turbine. Fig. 21.15. T-s diagram.

li 'td.stondod - t - ,1
- ( 21.6)
t: t-l
To(rr) r -T1t)
i1 The expression shows that the efficiency of the
1
Qt
li idcat joute cycle increases with the pressure
ratio' The absolute limit of pressure is determined
by the limitng temperature of the material of
the turbine at the point at which this temperatures
reache{by the cornpression process alone,
iir no further heating of the gas in the combistion
chamber would be permissible and the uorh of
tii erpans-ion uould ideally
iust balance the work of compression so that no excess worh utoulcl be
auailable for external use.
ii Now we shall prove that the pressu re ratio
iii temperature ratio. for maainurn worh is a function of the timitin
Work output during the cycle
= Heat receivecycle - Heat rejectecycle
= rnc o (T t - Tr) - mc o (T 4 - T ) = rnco (T, - mc, (72
T n) - - T r)

_ *^ l, - -&) ^ (r, _,\


=mcp'tsl'-",J_",
14-|
In case of a given turbine the minimum temperature ?, ad the maximum tempeature
Fig. 21.14.p-Vdiagram. ?J
.1L-T:.,"Iib"9, .", Pgi".e.
the temperarure of the aimosphe.e ind ?, rhe ;;;i;; remperarure
{-1 turbine would withsrand. Consider ihe specific nrt .i"*ri."l p."r",."
Tr= Tt Gr)-r-, where ro = pressure ratio il""::ilff#iii"":f .. tn

a 3=o,,tT
7'4=la')'i
Similarly 1-l Since,
i
or Te = Tt (r)'T i =1"
(. pr
,/
4 "
'1
7't
i

v-l
constant ,z'=
;

i
Using the
I
v

.-=.,&,-
818 INTERNAL COMBUSTION ENCTNES GAS TURBINI]S AND JEI PROPULSION

Operation 3-4' : Air is compressed in the H.P. compressor ftom p;to p,


we have, work outpuucycle w--K Operation 4'-5 : [Iigh pressure air is heated at constant pressure by exhaust gases from
power turbine in the heat exchanger to ?u.
['.[' *) ",','-,,] Operation High ptessure air further heated at constant prssure to the maximum
Differentiating with respect to ro
hriprature ?u by an air heater (through extenal combustion)'
dwf-z'l Operation 6-7' The air is expanded in the H.P. turbine fromp, top, producing work to
=x
# * -'r',,;'
-1\ :
o for a maximum
* Lrt " f= drive the compressor.
zTs Operation 7'-8 Exhaust air from the H.P. turbine is heated at constant pressure in the
. - 'n -(- \G-r')
:

air heate (through external combustion) to the maximum tempera-


r G +ll - 'l'\' P'
'p ture ?r(= ?u).
,2"
T.
Operation &9' : The air is expanded in the L.P. turbine from p' to pr, producrng energy
'p t1 for a flow ofwork externallY
f Operation 9'-10 : Air from L.P. turbine is passed to the heat exchanger where energy is
rp= (TJTr)u2" i.e., ro= (T3tTt)2(1-r) tansferred to the air delivered from the H'P' compressor' The tem-
Thus the pressure ratio for mmimum worh is a function of the limting temperature ratio. perature of air Ieaving the heat exchanger and entering the cooler is
Fig. 21.J.6 shows an arrangement ofclosed cycle stationary gas turbine plant in which air is T to'
continuously circulated. This ensures that the air is not polluted by the addition of combustion
waste product, since the heating ofair is carried out in the form ofheat exchanger shown in the
diagram as air heater. The air exhausted fom the power turbine is cooled before eadmission to
L.P. compressor. The various operations as indicated on ?-s diagram (Fig. 21.l?) ae as follows :
Operation 1-2' : Air is compressed from plto p, in the L.P. compressor.
Operation 2'-3 : Air is cooled in the intercooler at constant pressurep:.

External
--*

Fig. 21.1?.7-s diagram for the plart'

Operation 10-11 : Air cooled to ?, by the cooler before entering the L'P' compressor'
The energy balance for the whole plant is as follows :

At + Qz- Qs- Qa= W

Fig. 21.16. Closed cycle gas turbine plmt.

ra
I 820
I INTERNAL coMBusrtoN ENctNEs
CAS TURBINES AND JET PROPULSION
in practice, the control
,n"r" n}*i;':'":"tlplant' of power ourput is achieved
by varying rhe
temperature ,"d ;;,;;r"::"::;;::: y:;;:,:,r#:;::::::^ii1,^i1,, ,n" i,L, p"",,u," od A, B = Valves
roads' In this cycle sincl it co::td:t leuel of efficiency
is a*;;l -*"i"r::::::': *ith rauourabre'p;;;;;,, forbe
uarying C = Compressor
r,,*r,""-o"u iii,;;il"'ili:'ff",1,fr::,"J!:'r{::.f: used,
s"ible
for,ineftciency n this qtcle t, ,u" heaters. The naior "", ,""ponl ; D = Combustion chamber

hedters betueen the


i" i)"rr"""";';o.l"tt""
"** tenxperaturc drop which
facto,
P= Fuel pump
furnace ona occurs in the air
- "ir"utoti'rg'i"r."
Note l. In a closed cvcle sas ii,rhi.o" -r.i^:-, . , S = Spark ptug
used, the use orhe,ium'which
:Hiie,:""iffii#li#*'#":'j*j!itt!:"'',lT':H.1:"1."**"^'*.:" T= Turbine
:"::::!:;;ii;;;;'#;;;T.;;tr:";i:,;;:,";:;;Xi:Wify;i!iiytii!#i:j";J;i
tts,orrni*.,i,Jini,#liJln"pto,t
::::t:;"itill!:;'::::he-tiumcanbehepa.siowi,
araa required. ratio and for *ins
for nrlr^,
tsconsidernbty sn",,",*0","r,',!""rl'n'i'nii""l'i'1:Jfi.,Tt:::1T:ii:r"'".:ffrTilj
^u"l,t?!etemperature
2. Some gas turbine
!i;:;;:X*ii
olar on a combination of twocycles
a_combinationis';;ff#'".work the open cycle and the closed
Here a pqrt of the workne cJrcIe. such
irl another part fiowt
rr"^ flrid isi""n*'.iii the ptant and
""iii"!rl,ji::^7;.. Ar in Exhaust
,lll 21.4.5, Merits and lDemerits
of Closed Cycle Gas Turbine (From atmosphere) (To atmosphere)
Turbine Over Open Cycle Gas
I Merits of closed cycle ; Fig. 2f .18. Constant volme combustion gas turbine

l Higher thermal efficiency 21.6, USES OF GAS TI,]RBINES


2. Reduced size
3. No contamination
4. Improved heat transmission Gas turbines find wide applications in the following
5. Improved part load efliciency felds :
6. Lesser fluid friction 1. Supercharging 2. Turbo-jet and turbo-propeller engines
7. No loss of working medium
8. Greater output 3. Marine field 4. Railway
9. Inexpensive fuel.
5. Road transport 6. Electric power generation
Demerits of closed cycle : 7. Industry.
1. Complexity
21.7. GAS TIIRBINE FUELS
' fHffilTl"';L::]::1.:"t"r is re,quired. This limits its
water is available in abundance' use to srarionary installation The various fuels used in gas turbines are enumerated and discussed below ;
3. Dependent 1. Gaseous fuels
t """r"Ir".t" 2. Liquid fuels
l*':"'it*:S#il"iffT,j.:r H'P developecl is high comparativery, thererore nor eco- 3. Solid fuels
5. Requires the use of a very l Gaseous ftels. Natural gas is the ideal fuet for gas turbines, but this is not available
large air heater.
every'where.
21.5. CONSTANT VOLUME
COMBUSTION TURBINES Blast furnace and, prod.ucer gdses may also be used for gr,,s turbine power plants.
Refer Fig. 21.1g. In 2' Liquid fuels. Liquid fuels ofpetroleum origin such as distillate oils or rcsjdual oils arc
compessorci""a-ittuJiffi
r

most commonly used for gas turbine plant, The essential qualities of these fuels include pro;cr
liil!:il:T,:fi
is closed' the fuel is admitted,into lT,,,r*,Jr?ll"";:t:::tg.Hfi:H"T uolatility, uiscosity and calorific ualue. At the same time it slould be free
tl".rrr-r".''rilio, pump p. Then the mixture from any contents of'
;-rT:::"11.:trJ;j;jT"l prug s' rhe ;;;J;ff;es prace"iu fuel is
at constanr vorume w*h increase
mosture and' suspended impurities that woulcl log the small passages of the nozzles
ancl d,atnagt
ualues and plungers of the fuel pumps.
discharged, inro atmosphe.f:?il:1*i?t,f:T:-T: to.the turbine i, they are Minerals ltke sodum, uanadium and calcium prove uery harrnful for the tu.rbine btatli:g
energy. For aootinuor. *nr, ".Ji,..1",
v !u ruL gasesis thereby converted
-,* .^..
into mechanical
o,^.''-'.o'
the turbine these operation" as these buiid deposits or corrode the blades. The sotlium in ash shoukl be less than 3oTo
ol'ttte
u"u ."p""t".
I . Thc main clemeit ."r,o"
uelocties of ltot with this- tvpe of tubine. is that vanadium content as otherwise the ratio tends to be critical" The actual sodium content may
bt:
I sases ot constant ; so the turbine ihe or"""ur" d,ifference and betneen 5 ppm to 10 ppm (part per million). If the vanatlium is over 2 ppm, the magnesiurn
"." ":?"]::i1 speed, fluctuatei.r'----,e in ash
tends to become critir'al. It is necessary that the magnesum t ash s at least three times
l
tlu,

:,.
*tt-**I=.
822 823
.NTERNAL COMB USTION ENCINES C^S TURBINFJ AND JET PROPULSION

qudntit! of uanadum. The content of calcium and lead should not be over 10 ppm and 5 ppm 435.4 - 293
respectivelv.
na_
T2'. - 293
Sodiurn is removed from residual oils by mixing with 570 ofwater and then double centrifug-
4,-
435.4 -293 + 293 = 47lK
ing when sodium leaves with water. Magnesium is added to the washed oil in the form ofepsom
0.8
salts, before the oil is sent into the combustor. This checks the corrosive action of vanailium. -
Residual oils burn with less ease than distillate oils and the latter are often used to start the unit
from cold, after which the residual oils are fed in the combustor. In cold conditions residual oils
-
need to be preheated.
3. Solid fuels. Theuse ofsolid fuels such as coal in pulverised form in gas turbines presents
several difficulties most ofwhich have been only partially overcorne yet. The pulverising plant for
coal in gas turbines applications is much lighter and small than its counterpart in sream genera-
tors. Irtrod'uction of fuel in the combustion chamber of a gas turbine is required. to be d.one
against a high pressure whereas th pressure in the furnace of a steam plant is almospheric.
Furthermore, the degree of completeness of combustion in gas turbne applications has to be uery
high as otheruse soot and dust in gas would, d,eposit on the turbine blading.
Some practical applications ofsolid fuel burning in tubine eombustors have been commer- 1t=2O+273
cially, made available in recent years. In one such desigrr finely crushed coal is used instead of =293K
pulveriseC fuel' This fuel is carried in stream of air tangentially into one end of a cylindrical (a)
furnace while gas comes out at the centre of opposite end. As the fuel particles roll around the
II8' 2L;
circumference ofthe furnace they are bumt and a high temperature ofabout 1650'C is maintained
gases
which causes the mineral matter offuel to be conveted into a liquid slag, The slag covers the walls Heat supplied by fuel = Heat taken by burning
ofthe furnace and runs out through a top hole in the bottom. The result is that fly-ash is reduced mrx C = (mo+ m) co(Tr- Tr)
to a very small content in the gases. ln another desig a regenerator is used to.transfer the heat to (where rno = mass of air, rnl = mass of fuel)
ai, the combustion chamber being located on the outlet of the turbine, and the combustion is
carried out in the turbine exhaust stream. The advantage is that only clean air is handled by the ce (Ts- T2)
turbine.
Example 2t.1. The air enters the compressor of an open cycle constant pressure gas
"=\ft.')
41800 (90 + 1) x 1.0x(?r-471)
=
t.urbine at a pressure of I bar and temperature of 20"C. The prissure of the oir a.fter conpression
s 4 bar' The isentropb efficiencies of compressor and. turbne are 80Vo atd. 85Vo respectiiely. The +471=e30K
air-fuel ratio used. s g0 : 1. If flow rate of air is 8.0 hgt s, find, : "r= #
'Y-1
(i) Power deueloped.
(ii) Thermal efficiency of the cycle. Again, \'t =(u\i
[paj
=flf'on
\4/
=o.u'
Assume cp = 7.0 W I hg K and, f = L4 of air ond. gases
I=930 x0.672=624.9K
Calorfic ualue of fuel = 41800 hJ / hg.
Solution, Pr = 1 bar i Tt= 20 + 273 = 293K
Tt -Tt'
|ltu,ne= Tg-T
Pz = 4 bar i \"o po""o, = 80vo ; 4ruror* - 85vo
Ai-fuel ratio = 90 : 1 ; Air flow rate, rno = 3.0 kg/s 930 - T4',
0.85 = 6fi-e%e
(l) Power developed, P r s
K
Refer Fig. 21.I9 ()
\
.n'
'.)
.. T4' = 930 - 0'85 (930 - 624'S) = 670'6

1-l r.4-r Wturbn"= g, cp * Qg- TP


lq lPol''l t4\7.4 (where n, is the mass of hot gases formed per kg of air)
;=t:l
tt \Pll =l;l
\r/ =1.a86 -,
-'. T2= Q0 + 273) x 1.486 = 435.4K
wturoLne
. =fq1)
90 /
x1.0x(e3o-6?0.6)
[
-
'11 Tz-Tt = 262.28 kJ/kg of air
tcomDresfi, =
' 12 - tl
)

'
*

\ I
j
l

) Q1A
eL-
i1
INTERNAL COMBUSTTON
ENGINES GAS TURBINES AND JET PROPULSION
W"*o"* = o Y cp x (Tr,
- Tr) = l x 1.0 x (47I _ 298)
= 178 kJ&g oi air Tz = 288 x 1.67 481 K
=
w = Wn*-W-^o**
n- Tz-\
I
262.28 - 1?S = 6a.26 kJlkg of air. t2 - tl
j
u1il;."-"j#i::,:,:::",_:.1;i:;",-,:',;Hi$ff
of cycte, tttrmt I
iiiil.,*"., 0.8 =
481 - 288
ll
I
3]3:::r..fficiency
Heat supplied per kg of air passing rf -Tt
I through combustion chamber
jl
T'_ 481 - 288
I ------:- + 288 = 529 K
I
= 56 x 41800 = u.
I 464.44 kJlkg ofair
output
I
Wok g4.Zg similarty ror rhe turbine, 3 =lOl?
tj:,^:::tlr 11=J,^]#
I
?thrmot= - _!*.zu = 0.1814 or o'rau'
i
supp-Gd = AA
Heat DuPpueo 18.14%. tAls'.,
(Ans.) \P) = l.aba
-":'* "so!
Exanrple 21,2, A gas turbit 4ti4.44 \rrl
perature of 610'c. rhe
respectiuelv' catcutate
il.ntriplc'lfi"i"i1,:f:;"::::::,*
1 il io."c.Fi;',:;;,;:;i::f;:;,"!;::::;::::!ii:"r
calculate th,
of 6
u :.l
:
.1 and masimum
maximun cycte
cycre tem.
and turbine are
tem -'o = T" =,*
889
= uu x
the power output n or" o.so l-E
^, ::::::_i
0.Bo an
i
-0""'l:;:'":-":i*';::-i*";:;"';;':i7i:'::!;":'"i!tr:
i",,Y"'.'::t!
when the air. ".r;';;;;;;i;";n[":;::;ftressor
enters the compr"ssor
kiroutntto
at IS"c at rho -t- 2electric
generator geared to
and, 6.ez
0.82
the turbine Ts_T4'_ggs_ir|'
i
ol to hgls. Also, n -_ _Ta
x,, and-Tahe
co = 1.00s hJlkgK.and, 'Itwbire
ii y= t < r, ti'c*' tression T3 ggg _ 564
,il T = 1'333 for the expanston process. . process, and take
cp = r.I1 kJ / kg
n!, 883-?1'
lr Solution. Tr= t5 -'"- - 883 _564
+273 =2ggK;Tr=6t0 +273= ,e
I
88g K T' = 883 0.S2 (883
4"o^p*""- = 0.80 i lru,re 0.82 ot =
^ Hence,
- - 564) = 62t.4 K
= ; Air flow rate
For compression pror = ,u U*
: cP = 1'005 kJlkg [ Compressor work input, W,o^rr"""* = cp (Tz, _ T)
For expansion 7 = t'al
r= r.s3a = r.005 (529 _ 288) = 242.2 kilt<s
'"o"",*u" output,
,,
r4 ' .^,:rl"T,.
rb carcurarer;"""i",i"
,'n"
""rij".ut#j,rfii*
rurDlne work Wtu,bi= cr(7"_ Tn)
we must first or",rut"
i, "nrii:"Jx:ff ji.'*::ji[:ffiffi:jures rr'and
... Nerworkoutput, *^,,=l:,r,,::;::r_:_:2eo.4kl/kg
For an isenrropic process,
*=(*)+=tat#
rl \h) = r.ez
power in k'owatrs :lit,
Example 21.8. A pas terbine unit receiues
rti?ir;i?'*y?:"".,
adiabaticallv to 6'2 bar''tr'."o:ll1tlo!.efficiency air at I bar and 800 K and compresses it
hJ/hs and ttu fuet-air rato is 0.'0rZ is ggvo. The has "' fuel a heating value of 441g6
kJ/;i;;;.""
The turbine internal efficiency is g,vo.
colculate the work of turbine and compressor per hg
of air compressed, and, thermal effiiency.
For products of combustion, cp
= L147 hJlhg K and 1 = 1.gg3- (u.p.s,c. 1997)
Solution. Giuen : p, Q pn) i a", f1 g00
= = K ; p2G p)= 6.2 bar i r^^*-_^_.o. = 88vo l
C = 44186 kJ/kg ; Fuel-air d.orz kJ/ks jil'fi,1,,"" eoz
cp = 1.14? kJ&e K; y "ato = "f = ;
= t.SgA.
ror Lsentropr.c compression process
I-2 :
. -,t-t 1.4_l
r,
4 =l;l
=(a)T _fo.zl?
=lrJ = 1684
I Tz = 3O0 x 1.684 = b0S.2 K
J Now, -'fcompru"so. T"-7,
i = #t2
Fig.2l.20 - tl

I
826 INTERNAL COMBUSTION ENGINES GAS TURBINES AND JET PROPULSION 821

Net work = Wt.bi" - W.o.o*""ot


= 479.1 - 234.4 = 244.7 l<JA<e
Heat supplied per kg of air
= 0.017 x 44786 = 75l.2kJtks,

.'. Thermal efliciency, n- = #iffiu\A


244'7 or
751.2
- = 0.3257 32.574o. (Ans')

Example 21.4. Fnd, the required, air-fuel ratio in a gas turbine whose turbne and com-
pressor efficiencies are 857o and,807o, respeetively, Ma.ximum cycle temperature is 875"C. The
uorking fluid can be taken as air (co= 1.0 hJlhs K,'l = I'4) which enters the compressor at 7 bar
and 27"C. Tlte pressure ratio is 4. The fuel used has calorific value of 42000 hJ / hg. There is loss
of lUilo of calorific ualue in the combustion chamber. (GATE 1998)
Solution.Giuen: r,.,',n"=857o;r*o*""o,=807o;Tt=273 +875=1148K,Tt=27+273
= 300 K i c. = 1.0 kJ/kg K i = 1.4, p 1 = 7 bar, pz= 4 ba (Since pressure ratio is 4) i C = 42000
i kJl
Fig.2t.2L kg K n""
j907, (since loss in the combustion chambe is 109o)
505.2 _ 300 for isentropic compresson 7-2 :
n qe _
T2' _ 300 / rt-t 1.4*1
^
+=lPtl
l1 \Pr,/
'=()7=r.so
= (!s?:-qgq.to, = 5ss.2 K
",' Tz = 300 x 1.486 = 445.8 K
Heat supplied = (mo + m.) x cr(Ts - T"') = m, x C
t2- tl
(- m,) r, I"o.p."""o, =
or Ir+-|xc-(T"-!^"=J, Tr' 4
\ m") -P'-r 'r'' *o "" 445.8 - 300
or (1 + 0.017) x 1.005(?3 -bgg.2) 0.01? x 44186
= o'8 = rr'-3oo
t, = dt##ffiu + 583.2 = 1268 K r;= 445.8 - 300
+300=482.2K
For isentropic erpresson process J-4
8
:

_ y-r ri$-l T(K)

3=l'lr=lll'"'3
T3 [p / \6.2)
=o.r*
Tt = 1268 x 0.634 = 803.9 K ('; 1" = 1333 ......Given)
rs -Ti
Now, Tlt*bio. = T, I,
no
1268 -7,'-
, ""- - 1268_803.9
,. Tn' = 1268 - 0.9(126,8 - 803.9) = 850.3 K
W.u.p."""*. = cp(Tz' - 4) = 1'005(533'2 - 30O) = 234'4 kilkg
Wt*i," = cor(Ts- To) = 1.L47(1268 - 850.4) = 179.1-kltkg
s (kJ/kg K)

Fig.2l.22
INTERNAL COMBUSTION
ENCINES GAS TURBINES AND JET
Now, heat supplied by the PROPULSION
fuel = heat taken by the
burning gases
o.D x mrx c (mn+
= mr) x co, dr_ ;J;-" T(K)

"
= ("#). e{}# = (ro * t)* ",irt:.
r"' t

/\
4zooo -13.,
\,? )I ^
roo(1148-4822)
=rrn.rr(-*r)
0.e
\nr )
'ns
mf =
42OOO
?S9.?8 -l=55.?7say56
A/F ratio = 56 : I.
(Ans.)
3:1T_of:,rl.u. catcurate the th.ermat lf"irr"y
that co ood .,
for the comoustton process is , ,, , ,,,]]|.'otio of the plant is exampre
"o'"';:,u:;*'^:j""::!:ii,;i;;;:":*;:;::'!":'::;i,i1'ff;i;
:i-::c
Solution. Hea supplied
= co(Tr_ 7",)
= 1.11 (888 _ E2s) = 892.9 kJ/kg
Tttw'not=
H"r.*"=!*#=# = 0.1226; r2.26vo. (Ans.)
s (kJ/kg K)
Now, worh rotk o,4put
_ 4g.z 48.2
"olio = J9!
Goss work output -
Fi,g.2L.23
E;; = 29a4 = 0'166' (Ans')
ir
.. Example zt,G. In For isentropic expansion process S-4,
:.:!,!*,,'^!!:.,";;;;;;;;";"if'!;:'#;;"":,';,,f:l;{:i:",:ine air enters at r bar and 20"c m | .t-t . ,14_1
:;:':;: ;,"::"'::^o=';;:;,'i':':"':_,!:*ru::i'il;;;,fi?::f:i:T:_:::es entering the
= BSV'' 4,*,n" rs \,rri'
+=[ltl =l_l_]T=o.esr
\.s/
.ri!"!"#'il"",a'"i;:,;,i*";lnf;:::,:;;iuii;rt"'^p'"""'
supptied per
*"' I = 953 x 0.635 = 605 K
hg of air ,"toin".*"' -Tr'
,(.:t,!!:t
\t-tl).the thermal Now,
Ts
efftciency of the cycle. 'Iturbine - T3 _ T4
Mass of the fuel may be
neglecte--' 953 - T,',
solution. Given: pr= tyinrer,2o00) na-
""- 953-605-
1l"rpz
\=:o + rrt^:3rt r,
= 5 ba", ps = 5 _0.r = 4.9 o*,
r, Jffi
t7"o^p,""ro,
= 85vo, \y,,9iw =
i = oeo + 2zs = ess K,
?' = 953 - 0.8(953
W"*p,"""o, = cp (Tz' - Tr) = 7'O24 (4g4
- 605) = 674.6 K
For air and gases : cp
80Vo, ,l"o^triron = g1vo, - 2gB) = 205.8 kJ/kg
= 1.024 kJlke W tu.bin = c (T" - To)
rower developed by the plant, p _
R,.;,':"i:; o = 1.024 (953 - 67 4.6) = 28b. 1 kJ/kC.

-".. "r,t. ili;# ;J';"-u\::r;";"',t:"1";*".t ; Jrt;t"Lt*:""*" ,"


1065 kW.
() The quantity
of air circulation. ii,nu
For isentropic compression "", m
"'" :'
,_2, P no, Wn"rkW
=
1065=oxT9.B
/ .1-I 1.4-r
^
!2=tU r _[s) rr :. nt"= 1065
rt l p' j -l i] = 1'584 il = 13.43 kc,
Tz = 298 x 1.584 = 464
i.e., Quantity of air circulation = lS.4B kg. (Ans.l
K (ii) Heat supplied per kg of air circulation
:
Now, ^
4"o^p"","o,=
T" -7,
i.e., Actual heai supplied per kg of air circulition
ii 0.85 = ,*#
464 _ 293 _co(Ts _ Tz') _r.024(953 - 4g4)
', = -.EE- + 299 = 494 K \combustion 0.85 = 552.9 kJ/kC.
(iil) Thermal efficiency
"r"fililbe, n,o*^", I

!
\nrnot =
#H:;:ffi =
#r'o =0.14s4 or r4.B4vo. (Ans.)
'-t t

,t
._,__,-5- __
830 INTERNAL COMBUSTION ENCINES
CAS TURBINES AND JET PIIOPULSION
Example 2L.7, In a gas turbine the compressor is driuen by the hgh pressure turbine. The
exhaust frorn the high pressure turbne goes ti a
free low pressu,re turbine-which runs the load,.
Tenperature of the exhaust from tJre unit To :
rhe.ly lot1l rate s 20 h6 ls and the minimum o,r ,rrori^r^ temperatures are respectiuery J00 K
and- 1000 K The compressor pressure ratio is 4. Calcvlate the piessure ratio of t^he low pressure
t.urbine and the tempera,ture of exhaust gases = 1.269
from the uiit. The compressor and. turbine are
Bentropc. crof air and, exh.aust gases = I kJlk7 K ant.l j.4 (GATE 1995)
=
,,=h=T#=6zsI<.
Solution. Giuen : ri.to = 20 kg/s ; ?r = 300 K ; ?, = 1000 K, lkJn<g K . ^( 1.4,
* = 4 ; co = = Example 21.8. In on air-stand,ard, regenerotiue gas turbine cycle the prcssure rato is 5.
Pressure ratio of low pressure turbine, Ia Air enters the cornpressor at 1 bor, 300 K ond leaves at 490 K. The maxinum tentperature in the
:
cycle is 1000 K. Calculate the cycle efficiency, given that the efficiency of the regenerotor and the
Since the compressor is driven fy frigh p.essulJturbine, adiabatic effcency of tlw turbine ore each 80!a. Assune for air, the rato of specific heats is 7.4.
Also, shoto'the ,yil" o, a T-s diagrarn. (GATE 1997)

.'. T^ 04
^-\+ =(4)14
'2 --l/ 2l' *
Solution. Giuen : p, = 1 bar ; ?r = 300 K, T2' = 490 K ; ?, = 1000 K
1.486
Tr Iprl 0.8;1=
Tz = 3OO x 1.486 = 445.8 K t = 5, 1,,,.",.. =8O%, e=8O%= 1.4

T(K)

5-/

s (kJikg K)

w.21.24 "t

Also, rhoco(T"- Tr) = ritoc, (Ts


- Tt) (neglecting.mass of fuel) s (kJ/ks K)
or T"- Tr=Ta-Tn Fig.2r.25
445.8 - 300 = 1000 - To, or To -_ 854.2 K
For process 3-4 : | al-t L{-r
Now, li=llzl r
^
=(5)T=1.bgs8
T4 l.pr/
+=l*l?,
T4 \pt) ", .=f.3]#
p4 lTo) r.e= Ts = looo = 681.4 K
'&s 1.5838 15838
u5 Tt -T
r)
(a=(ffi),'=r.zae Also, \tu,biw= Tr-f^
Now, pe=pt*!r=4^& e.= e" $

P Ps P4 Pl
["
\PsPrl
=4] or u'5
1000 - 14'
= looo:651.4
p5 ?'= 1000 - 0.8 (1000 - K
... p4 =f[ral=f
4\p4 ) 4
x1.?86=0.4s4
631.4) = 705

-i
Effectiveness of heat exchanse r. ,= 3;2-
Ti
ffence pressure ratio oflow pressure turbine = 2.3. (Ans.) -Tz'
Ps 0.434 = 4-490
'= zob-490

t
ti
ii'
tl
li

ii INTERNAL COMBUSTION ENGNES


CAS TTIRBINES AND JE-T PROPUI-SION
? = 0.8 (205 _ 490) + 490
il
-_- -
Work consurned by compressor = 662 X
= c, (T2, * Ty) T(K)

= 1.005 (490 - 800) = 190.9 kJrkg


Work done by turbine (73_ T1)
= ce
= r.005 o0OO * 205)
Heat supplied = 296.5 kilkg
= cn (Tr* T) j

= 1.005 (1000 _ 66D = Es,? kJlke


,i cycle efficiency,
_. _-
,cycrc.
= . -_Ig!Igt
Heat supplied
Turbine work Compressor
- work
Heatsulffi
_ 296.5- 190.9
---$qt-- = 0.31 o 317o. (AnsJ
-q'v"' Plsnt
:T,::;,'::;:,fr::;::T,,lf::::"::,:or
drive compresso, coss'sof two turbines
turbines. one
one compressor
compressor turbne to s (kJlkg K)
""d;;;; -. .
own combustion chambers ueueLop power output (Lnd
qnd. both
boti are
o,, r,our,s
hauing tt,i,.
their
(b)

X.T!:':"'at
:mpressor t at 1 bar
bar and,2BB "::;;::':::::"^"::j":!?:,r"outp::t
zs:8";;2i;',:':::1::^::."i1e1tu
x frotn the.conpressor.
lrol.n th,e Air enters
"","-"i"i.'ntr enters rhe
rhe FiE.2r.26
D.ue to hcat add.ectr in"^ ""-:r'rr^*irriJ'r'fi''
the combustion uith an isentropc efficiency of Z6o/e. Considering isentropic
?;;t ;;y,::i::,,;::#";:::;,,:i,ffr!",::i;",;;,";::::,:,:";2ni:,{::;Tj!.r:;i
?:0,":,r!,
";.;:;:_*,::.o,bartenperature of sas to both turbines
isentropic ,ffr"i"";";;;;;;;,i"i'!i;i"T"l!t is "ompressirrn
I
process 1_2, we have
t.s,23 hq/s. The calorific udtue ratc of air at the compressor l-l
of fuel is 42nn bttL- ::,^:tt flo.ut
t\lmat ':,0:::;i;,I:,:"i1,;:{#::";;''Ji;:':i::;;!:f"':,';,::,7;:;",i;::r::f'*
;;:'^:f erricicncy r and the
7-"
- 1;."i =!I I I
;lr/
l4
=1.811
,.o ^l""nl,"l"iti";:';1;:::i;:::,::,::,';;,:::i:;!r;:;:lant
nrlhg I \ Pl ./
hr K and, , : 1.4
/ hg Kandl=
"r,""1":;:';"i;; :i "i :#rtr:; d,;::;r*::
forairand,c,n= I tEU nil*rl# a _ , c, ^,,
r", rc ce= L00s ".
lnz .. :lEr x 1 811 = 521.6 K
(AMIE summer' 2001) AIso, .- f9 - nl
'V 4,
j
r! n,,,"::':'#;;,I::;{l ,t o:: j = 288 K ; p, = g
",
lj
xi r,
= t 4:cw = 1.126-r"- fl:
v4
;:='; ii:**-fi{i;i.lT':',,.!!':;[a;!!2'irr:'-riil: o.ro =
5-?Ll.-?89
tt the plant and the co*esponding 72'. -.288
4,, ,rt lJ""r,liii,ifjment ?-s diagram are shown in
Fig. 21.26 S2r.6 2Bl
_
' , = --f.te -- + 288 = 595.4 K
Considering scntr,9i ezpansrcn
process ,\_4, we have
.T-l l{4 1
!i p], _( r)'-rd-
r'-l.t/
=f =ls/ =o'Se
.. To = 1173 x 0.59 = 692.1 K
Also, n-. Ts-Ti
- _4
.jIisen.) _
4
_ _l-j7s:4_
,.*__
n o^
ir73_692.1
Tq'= 1173 _ 0.86(1173 _ 692.1)
^
Consider I kg ofair flow through compessor
= 759.4K
C = Compressor
LT = Compressor turbine cr(T/ -Tl) = 1.008(s95.4 _288) a08.9
rr =r,owerturbine rl.r :-
nLj_ \,.,,0**,,=
ls equ.l to u,ork of compressor turhine.
= kJ
(a) '" 308.9 = .nr * ,r, ( Tr- ?'0,).
neglecting l.uel mass
CAS TURBINES AND JE'T PROPULSION
834 INTERNAL COMBUSTION ENGINES
ii
30&9
or -r = iizeffi:t5e. = 0'662 ks
and flow through the power turbine = I - m= I -0.662 = 0.gSg kg
Wn=$ - m)x e*(T"-Ti)
= 0.338 x 1.128 (1129 - 259.4) = \SLT kJ
Power output =2B x !57.7 x I_".t x rlc.n.
= 23 x 157.7 x 0.95 x 0.96 = ggg7.9 (As.) kJ.
Q'^ou= corTg'c-Tr'
= 1.t28 x l1?3 - 1.005 x 595.4 = 724.7 kJkg of art
Thermal efliciency, .\^=ffi x 100= 2t.!gJg. (Ans.
cExample ZL,lo. Air is drawn in a gas turbine unit at r|'c and I.0r bo,r and, pressutz
ratio is ! : l.Jhe compressor--is.driuen bx the H.p. turbine ond L.p, turbip d,rives d separate
P^oy_er sryaf. The isentropic efficicncies of *,mpressor, and the H,p, and L.p. turbines aie 0.g2,
0.85 and 0.85 respectiuely. If tlu maeimum cXclc tcmperature s 610,C, calculate F9.21.27
:
(i) The pressure and. temperature of tlv ga*s enterng the puter turbhle.
(ii) The net power d.eveloped, by the unit per hgt s mass flow.
... r'= 888-
' ry!
1.15
=6s4. K
Qii) The worh ratio. 1.e., Temperature of gases entering the power turbine = 654'4 K. (Ans.)
(iu) Thethermal efficency of the unt. Again, for H.P. turbine :
Neglect the mass of fuel and aesurne tlw following :
For cornpression process cp = 7.005 hlths K and y = L4 4urbirc=ffi i'e', o'85
For combustion and, expansion pr.ocesses ; = 7.15 kJlkg and
cps | = I,BJS, ... T,= aas _
'\0.85) Irs__=.9_
n.o)
= u,n x
Solution. Giuen:Tr= 16+2?B=2B8Kpr= l.0lbar,preseure ,^to= b =7, Now, conaiilering isentropic upansbn prccess 3-4, we have
fl*^pn* = 0'82, 4u'"*,p.) = 0.8, 4r.6*
- 1"., = 0.85,
Maximum cycle temperature, ?, = 610 + 2?3 ggl K
=
q=lo)?
(i) heesure and tenperature of he ga.ees entering the power
turbine, p4, and Tn, :
r,-l'n)
Considering isentropic compression f-2, we have _ -t Ls3

{-t
Da fA)r-t 1883\o.sg
r \- L{-1 pl l.{j
-=lif =r_t =4.32
^ \614/
+=l!zl
rl \Pr.i '=(?)T=r.?s
, T"= 288 x 1.7,15 = 502.6 K
o,= &=H= 1.636 bar
i

,.e., Pressure of gases entering the = 1.636 bar. (Ans.)


power iurbne
AIso D
tpft.*- = +;+
n,_fl
(di) Net power developed per kds mass flow, P :
t1

ii
i

To find the power output it is now necessary to calculate fu'. rii

r, 0.82 = V!_:E . P is civen bv EL*b


lrl
!i
ll
The pressure ratio, - pt ps
502:q:288
;; , - il,l
" T2' = + 288 = 549'6 K !1 =P *Pz =J_ ,I

W,*rn"* =
i.e. ,
ps h Pt 4.AZ
=t.az (... Pz= Psan.d P5= Pr) I

cp.(Tz' - ?r) = 1.005


x (549.6 - 288) = 262.9 kltkg rl
Now, the worh output of H.p. turbinz Work input to conpressor
= ^,./ .1-t
1 o.s3 j I

cor(Tr- To') = 262-9 Then, +=lel


t5 \Ps) =11.52i.or =1.127
L.., 1.15(883 _ To,) = 262.9
i
I

I
,l
iil
I

s- I
l
I

t
I

* ,t
INTERNAL COMBUSTION ENCfNES CA,S TURBINES AND JE'T PROPULSION

- T5=.T:-=95,14 T (K)
' Lr27 L727
Again, for L.p. turiine
r1u,bn,
=
n'-n'
,^, 4
L,., 654'4-4'
0.85 - 654.4 _ 580.6
r;
= 654.4 _ 0.8S (654.4 _ 580.6) = 591.T K
,.
wL.p. turhlu = cpg (f
rrence t pouer output (per kg6 - T;) = 1.15 (654.4 _ 591.?) = 72.LkJ7*E
mase flow)
(ili) Work ratio : = Zg.f kW. 6ns.)

llorh, ratio _ Net nork output 7Zr


,;]ji::l-r,"fficiency tr rtp't =
-ifu = o'215' (Ans')
of rhe unit,,;;;, :
Heat supplied
= cor(73- Tr,) = t.t5 (88g _ 549.6) g8g.4
= krkg
n .
'tthermot
_ Net work output 72.1
- T; t"ppll"d = = 0'188 or 18'87o' (Ans')
ffi s (kJrkg K)
._-v vt q,. vpe,t-cyc.e gas
:-#:"?,\';:r^:::::::::::!.'"-?o?
taken at J7"Cand I bar, Thi "r"* *s
turbine power plant
turrne ptant is 5.6..Ar
5.6. Air isis Fig.21.28
between. The ,";::::::::i,:::::!
rhe maximum tr-o"ro,r""-^lr;;"":.:^:r:: ":.t
vrLLE Ls":,
Ln tuo stdges with perfect intercooling
':ry?to "3:: -,i,n i"i"i,
i,,","ootins in
efficiency of each compre,,y,":::::.:r::;'!!'j
each compress ::!:^i,:o
Lllntted' t:.710"c: e""*ls-i"n"
Tuu"C, Assuming the isbntropic lo - l1 389.23 - 303
*1u:" Also,
ai,"top";";;;;,;:;;';7:;li23,1'"7,::!-,!:,':f .o'soc", i"i,)hi)" ,n" p,.", 'tcomp. - TL'-7. 72',-3O3
::;::::::#:n:;7""1:::;-,ni;j"i^l:''ii
neglected, and.
it may be ass that cp = 1.02 hJlhs K dnd, y
u^med.
ni,!'f;""iT';",::",::;:;,\i
't 'ttc utr J'ow s 1'z hEls'
m, 389.23-303
q^r..:^_ lo =
Solution. Fig.
Refer 21.2g.
= 1.41. (P.u.) U.D
+ 303 = 404.44 K
-_---
Work input to 2-stage compressor,
Pressure ratio of the open_cycle gas W"o^p. = 2 x tn x co(?:2, - Tr)
tubine _ 5.6
Temperature ofintake air, ?,
B0 + 273 = g0A K
= 2 x 1.2 x l.O2(4O4.44 - 303) = 248.32kJ/s
= For turbine, we have
Pessure oi.intake air, p, 1
= 6"
Maximum temperature of tn" . .1-I jl'''^
Isentropic effi ciency of each "y.l!,
?s = 700 + 2ZB 9Zg K
= I v r,r r.65
T,
or .. = t=**
q?e

Isentropic efficiency orru.o,rillot""to"' =


]**
t=1fr)' = 156- = = 58e.2 K
2u^'"
41a6n" = 9ovo
^, Ts-Ti
Rate of ai flow, ,h" = t 2;;" Also, \n"bne=;;=i
cp = r.02 kJilg K and 973 _ T^'
_
power developed T = 1.41.
and efficiency ofthe power plant or 0.9 =
OZ3_ 5B9J or Tu' = 973 - 0.9(9?9 - 589.?) = 628 K
Assuming that the pressure ratio :
in each .tugu i, ,Joq *" fr*u . . Wok output of turbin E, W tu,b* = m x cp(Ts- Ta,)
pz_p lpo . =L.2 x 1.O2(973-628) = 422.28W/s
-
pr= n= {_=J5.6=2.866
since the pressure r":"^:ro^^t!--.entropicefficiencyof
Net work output, w.=w.-w tufDtne
' uorh input
the each compressor is the same then = 422.28 - Z4B.2 = 128.96 kJ/s or kW
reqtLired' fu'""_:"! ,o^pr"r"o,
saIe inlet temperature (perftct interciotiiij i, ;-;;-*" since both the compressors
vv,pr csDL have
Hence power deueloped = 173.96 k\il. (Ans.)
*.,";: : ;, and ir, : i4:'.- ,re
Heat supplied, Q"= m, cox (Tu- Tn')
r3r-r
Now, ! (P'\' =(2366) rar' = 1.2846 or
= t.2 x l.O2 x (973 - 404.44) = 695.92 kJ/s
7\=l;) ?2 = 303 x r.2846= s8e.23 K .'. w-^, 173.96
Power plant efficiency, .. = --ie - O.25 or 2SVo (Ans.)
A. 695.92
838 INTERNAL COMBUSTION ENCINES 839 lr
CAS TURBINES AND JET PROPULSION

Example 2L.12. (a) Why are the bach work ratios relatively high in gas turbine plants
compared to those of steam power plants ? li
?, = ?., x 1r) 2Y i
(b) In a gas turbine plant compression is carrid, out in
two stages with perfect interaoling
and expanson in one stoe turbine, If the mnamum temperdture (i^* n ind minimum tem- W*.r*-. = 2[co (T2' - ?t)l for both compressors

p:rat.ur: (T,,t,K) in the cycle remain constant, show thit


for maxrtm specific output of the
ptant, the optlmunT ouerall pressure ratio is given by =*o',1;', =fr ,^,^l,r?t - rl, u, r, = r-,"
I T Y(t-t) 'f-l
hn=l\r k.-f:l'^mtn J Also,
4 al ' =t.l
T6=f[prl
\
where y = Adiabatc nd.er ; 1r= Isentropic effrciency of the turbine.
r1c = Isentropic efficiency of compressor. (AMIE Summer, lg0g) T-T
Solution. (o) Back work ratio nay be defined the ratio of negatue work to tfu turbine
as
.. r6=
=Giffir, as Iu = ?.*
worh in a pouer plant. In gas turbine plants, air is compresseil from the turbine exhaust pressuro f11 n-n'
l"-* -Giffi"l
to the combustion chamber pressure. This work is
siven Uy - J udp. As the specific volume of air is Wr*ui." = cp(T6-Ts')= .o rl, s rl= i;
very high (even in closed cyclo gas turbine plants), the compressoi work reqired is very
high, and
also bulky compressor is required., In steam powerplants, the turbine exhaust is changed
to liquid
phase in the condense. The pressure of condensate is raised to
boiler pressure by condeniate =cor^uft-.
' J-,"ln,
L (r/' ')
extraction pump and boiler feed pump in series stnce the specific volume f water is very small as
compared to that of air, the pump work (- j udp), is also very small. From the above resons, Wnct = Wtubite - W.-o-*t
the .
1-l
back work ratio f f t" r
-lud'P
\tr*Lt- t*--l- "#'-'(r) 21
- I
- T\rrbine work For maximum *".n
for gas turbine plants is relatiuery high compared to that for steam power plants. ""r;:,
dWoa
() Refe Ft9.2I.29. =O
dr
T (K) /--r\1-l -'=o .

p .. ,.*
,
-cortrr*l-Y-1)(ri( f'-?tr",-
_^,n lIJlta y
I zr J. "
.

\ lJrrr !l i

r, (r)rt- ut
nr nc = on simplification'
ffi
Hence, the optimum pressuro ratio is

,.=L,b I r"'l#
*.r.rl proved.

Example Zlla. In a gds turbiw the compressor takes n or at o temperature of 15"C and.
compressesii to four times lhe intial pressure with an isentropir ef!'ciency of.82%- The ar is ll'
in piiiia'tn*igh a heat erchanger ttzatecl bX the turbine eihaust before reaching tlw cotnbus'
manimum

Das könnte Ihnen auch gefallen